{"id": 402, "year": 2016, "question_id_specific": 183, "full_question": "Juan, residente de segundo a\u00f1o, atiende en urgencias a Sofia, una muchacha de 15 a\u00f1os que, al parecer, se ha desmayado en la escuela sin llegar a perder la conciencia. La paciente cuenta que estaba pendiente de realizar un examen, lo que le causaba mucha ansiedad. Por el interrogatorio, parece entreverse una situaci\u00f3n de cierto acoso por parte de sus compa\u00f1eros y la posibilidad de que sufra un trastorno alimentario. Las constantes vitales y exploraci\u00f3n neurol\u00f3gica son normales. Juan mantiene a So\ufb01a en observaci\u00f3n a la espera de que sus padres acudan al servicio echando, peri\u00f3dicamente, una ojeada a c\u00f3mo se encuentra la paciente. Tras el susto inicial, la paciente parece encontrarse cada vez m\u00e1s animada y es muy simp\u00e1tica. En una ocasi\u00f3n, Juan la encuentra chateando activamente con su m\u00f3vil. Juan le indica que ser\u00eda mejor que dejase el m\u00f3vil y descansase y para tranquilizarla, le cuenta que el tambi\u00e9n utiliza mucho las redes sociales desde la facultad. So\ufb01a pide perd\u00f3n por desconocer que ten\u00eda que tener el m\u00f3vil apagado, y tras apagarlo, le pregunta si podr\u00e1 hacerle una solicitud de amistad en Facebook. \u00bfCu\u00e1l cree que es la mejor respuesta de Juan?", "full_answer": "Es indudable que la relaci\u00f3n m\u00e9dico-paciente hoy d\u00eda trasciende el entorno f\u00edsico de la consulta. Pero no debemos olvidar que debemos mantener en ella los mismos c\u00f3digos \u00e9ticos y profesionales que en el entorno real. El c\u00f3digo deontol\u00f3gIco Art\u00edculo 26-3. El ejercicio cl\u00ednico de la medicina mediante consultas exclusivamente por carta, tel\u00e9fono, radio, prensa o internet, es contrario a las normas deontol\u00f3gicas. La actuaci\u00f3n correcta implica ineludiblemente el contacto personal y directo entre el m\u00e9dico y el paciente.", "type": "ATENCI\u00d3N PRIMARIA Y REDES SOCIALES", "options": {"1": "Decirle que haga la solicitud de amistad y que la aceptar\u00e1, pues est\u00e1 seguro de que en su p\u00e1gina no hay elementos inapropiados para una chica de la edad de So\ufb01a.", "2": "Dado que considera a So\ufb01a una paciente vulnerable y le preocupa que pueda interpretar mal un rechazo, aceptar que haga la solicitud pero solamente permitirle el acceso a determinados contenidos de su p\u00e1gina.", "3": "Contestarle que es importante mantener unos ciertos limites profesionales entre pacientes y facultativos y que, desafortunadamente, si hace la petici\u00f3n no podr\u00e1 aceptarla, por lo que mejor que no lo haga.", "4": "Decirle que haga la petici\u00f3n pero sin intenci\u00f3n de aceptarla.", "5": NaN}, "correct_option": 3, "explanations": {"1": {"exist": false, "char_ranges": [], "word_ranges": [], "text": ""}, "2": {"exist": false, "char_ranges": [], "word_ranges": [], "text": ""}, "3": {"exist": true, "char_ranges": [[413, 520]], "word_ranges": [[62, 78]], "text": "La actuaci\u00f3n correcta implica ineludiblemente el contacto personal y directo entre el m\u00e9dico y el paciente."}, "4": {"exist": false, "char_ranges": [], "word_ranges": [], "text": ""}, "5": {"exist": false, "char_ranges": [], "word_ranges": [], "text": ""}}} {"id": 237, "year": 2014, "question_id_specific": 144, "full_question": "Tras un accidente de tr\u00e1fico un paciente de 38 a\u00f1os ingresa en UCI en coma. Tras varios d\u00edas el paciente no mejora neurol\u00f3gicamente y en la TAC se visualizan lesiones puntiformes hemorr\u00e1gicas en cuerpo calloso y en uni\u00f3n c\u00f3rtico-subcortical. \u00bfCu\u00e1l es su diagn\u00f3stico?", "full_answer": "La lesi\u00f3n axonal difusa produce un deterioro precoz y mantenido del nivel de conciencia (como comenta el enunciado del caso) sin que haya en el TC una lesi\u00f3n que justifique el cuadro. En ocasiones, se evidencia en esta prueba de imagen hemorragias puntiformes a nivel del cuerpo calloso, uni\u00f3n corticosubcortical y porci\u00f3n dorsolateral del tronco encef\u00e1lico.", "type": "NEUROLOG\u00cdA", "options": {"1": "Hematoma subdural agudo.", "2": "P\u00farpura trobocitop\u00e9nica.", "3": "Contusi\u00f3n hemorr\u00e1gica cerebral.", "4": "Lesi\u00f3n axonal difusa grave.", "5": "Encefalopat\u00eda hip\u00f3xico-isqu\u00e9mica."}, "correct_option": 4, "explanations": {"1": {"exist": false, "char_ranges": [], "word_ranges": [], "text": ""}, "2": {"exist": false, "char_ranges": [], "word_ranges": [], "text": ""}, "3": {"exist": false, "char_ranges": [], "word_ranges": [], "text": ""}, "4": {"exist": true, "char_ranges": [[0, 183]], "word_ranges": [[0, 32]], "text": "La lesi\u00f3n axonal difusa produce un deterioro precoz y mantenido del nivel de conciencia (como comenta el enunciado del caso) sin que haya en el TC una lesi\u00f3n que justifique el cuadro."}, "5": {"exist": false, "char_ranges": [], "word_ranges": [], "text": ""}}} {"id": 620, "year": 2022, "question_id_specific": 121, "full_question": "Mujer de 79 a\u00f1os que ingresa por una fractura osteopor\u00f3tica de cadera. Respecto a la prevenci\u00f3n secundaria de las fracturas por fragilidad, se\u00f1ale la respuesta INCORRECTA:", "full_answer": "En la osteoporosis, uno de los principales riesgos asociados al incremento de riesgo de fractua es la baja adherencia al tratamiento, por lo que la respuesta 1 es correcta. La respuesta 2 se encuentra en la gu\u00eda de la SER, en la que se confirma que algunos estudios concluyen que los marcadores de remodelado \u00f3seo pueden ser \u00fatiles para monitorizar de forma precoz el cumplimiento y la respuesta al tratamiento. La respuesta 4 es correcta ya que nuevamente en la gu\u00eda de la SER 2019 citan textualmente: \u201cLa evidencia cient\u00edfica actual permite afirmar que ni aumentar el calcio diet\u00e9tico ni tomar suplementos de calcio de forma aislada protege frente a la aparici\u00f3n de fracturas\u201d. Por ello la respuesta correcta a esta pregunta es la opci\u00f3n 3. Los pacientes en tratamiento farmacol\u00f3gico para la OP se deben utilizar suplementos de calcio y vitamina D porque pr\u00e1cticamente todos los ensayos cl\u00ednicos que han demostrado eficacia de los f\u00e1rmacos antiosteopor\u00f3ticos incluyen de forma rutinaria suplementos de calcio y colecalciferol (vitamina D3), pero no en monoterapia.", "type": "TRAUMATOLOG\u00cdA", "options": {"1": "La baja adherencia al tratamiento se asocia a un incremento del riesgo de fractura.", "2": "Los marcadores de remodelado \u00f3seo pueden ser \u00fatiles para monitorizar de forma precoz la respuesta al tratamiento.", "3": "La vitamina D en monoterapia es eficaz en la reducci\u00f3n de dichas fracturas en personas mayores no institucionalizadas.", "4": "El aumento de calcio diet\u00e9tico o tomar suplementos de calcio de forma aislada no protegen frente a la aparici\u00f3n de fracturas.", "5": NaN}, "correct_option": 3, "explanations": {"1": {"exist": true, "char_ranges": [[0, 172]], "word_ranges": [[0, 29]], "text": "En la osteoporosis, uno de los principales riesgos asociados al incremento de riesgo de fractua es la baja adherencia al tratamiento, por lo que la respuesta 1 es correcta."}, "2": {"exist": true, "char_ranges": [[173, 411]], "word_ranges": [[29, 70]], "text": "La respuesta 2 se encuentra en la gu\u00eda de la SER, en la que se confirma que algunos estudios concluyen que los marcadores de remodelado \u00f3seo pueden ser \u00fatiles para monitorizar de forma precoz el cumplimiento y la respuesta al tratamiento."}, "3": {"exist": true, "char_ranges": [[680, 1066]], "word_ranges": [[114, 172]], "text": "Por ello la respuesta correcta a esta pregunta es la opci\u00f3n 3. Los pacientes en tratamiento farmacol\u00f3gico para la OP se deben utilizar suplementos de calcio y vitamina D porque pr\u00e1cticamente todos los ensayos cl\u00ednicos que han demostrado eficacia de los f\u00e1rmacos antiosteopor\u00f3ticos incluyen de forma rutinaria suplementos de calcio y colecalciferol (vitamina D3), pero no en monoterapia."}, "4": {"exist": true, "char_ranges": [[412, 679]], "word_ranges": [[70, 114]], "text": "La respuesta 4 es correcta ya que nuevamente en la gu\u00eda de la SER 2019 citan textualmente: \u201cLa evidencia cient\u00edfica actual permite afirmar que ni aumentar el calcio diet\u00e9tico ni tomar suplementos de calcio de forma aislada protege frente a la aparici\u00f3n de fracturas\u201d."}, "5": {"exist": false, "char_ranges": [], "word_ranges": [], "text": ""}}} {"id": 396, "year": 2016, "question_id_specific": 136, "full_question": "Un hombre de 41 a\u00f1os acude a Urgencias por un cuadro de tres d\u00edas de evoluci\u00f3n de tumefacci\u00f3n y dolor en la rodilla derecha, con impotencia funcional y febr\u00edcula Dos semanas antes hab\u00eda tenido un cuadro de diarrea autolimitada En la exploraci\u00f3n existe derrame articular, por lo que se procede a realizar una artrocentesis y se obtienen 50 cc de l\u00edquido de color turbio, con viscosidad disminuida y los siguientes par\u00e1metros anal\u00edticos: leucocitos 40.000/microL (85% de neutr\u00f3filos), glucosa 40 mg/dL, ausencia de cristales, tinci\u00f3n de Gram: no se observan microorganismos. \u00bfCu\u00e1l de las siguientes afirmaciones sobre este paciente es \u00ecNCORRECTA?:", "full_answer": "El gram negativo NO descarta la infecci\u00f3n. Marcamos la cuatro.", "type": "TRAUMATOLOG\u00cdA Y ORTOPED\u00cdA", "options": {"1": "Se debe iniciar tratamiento con cloxacilina y ceftriaxona en espera del resultado del cultivo del l\u00edquido.", "2": "Es aconsejable realizar artrocentesis diarias para aliviar los s\u00edntomas y evitar la destrucci\u00f3n articular.", "3": "Si el cultivo es negativo, es probable que se trate de una artritis reactiva.", "4": "La negatividad de la tinci\u00f3n de Gram descarta que se trate de una artritis s\u00e9ptica.", "5": NaN}, "correct_option": 4, "explanations": {"1": {"exist": false, "char_ranges": [], "word_ranges": [], "text": ""}, "2": {"exist": false, "char_ranges": [], "word_ranges": [], "text": ""}, "3": {"exist": false, "char_ranges": [], "word_ranges": [], "text": ""}, "4": {"exist": true, "char_ranges": [[0, 42]], "word_ranges": [[0, 7]], "text": "El gram negativo NO descarta la infecci\u00f3n."}, "5": {"exist": false, "char_ranges": [], "word_ranges": [], "text": ""}}} {"id": 253, "year": 2014, "question_id_specific": 86, "full_question": "Mujer de 37 a\u00f1os afecta de una colitis ulcerosa extensa, presenta un brote grave por el que se inicia tratamiento con prednisona en dosis de 1 mg/kg. Tras una semana de tratamiento, la paciente no presenta mejor\u00eda. \u00bfCu\u00e1l es la siguiente medida terap\u00e9utica a realizar?", "full_answer": "Cuando un brote grave no responde a corticoterapia iv a dosis plenas (1mg/kg de peso) hay que pasar a ciclosporina o infliximab. Etanercept no est\u00e1 aprobado en Europa para la enfermedad inflamatorio intestinal (de momento).", "type": "APARATO DIGESTIVO", "options": {"1": "Colectom\u00eda subtotal de urgencia y en un segundo tiempo prostectom\u00eda y reservorio ileoanal.", "2": "Asociar un inmunosupresor como azatioprina.", "3": "Asociar mesalazina en dosis de 4g al d\u00eda oral y triamcinolona rectal 1 aplicaci\u00f3n cada 12 horas.", "4": "Ciclosporina endovenosa 2mg/kg.", "5": "Debe valorarse el tratamiento con etanercept (un anticuerpo anti-TNFa)."}, "correct_option": 4, "explanations": {"1": {"exist": false, "char_ranges": [], "word_ranges": [], "text": ""}, "2": {"exist": false, "char_ranges": [], "word_ranges": [], "text": ""}, "3": {"exist": false, "char_ranges": [], "word_ranges": [], "text": ""}, "4": {"exist": true, "char_ranges": [[0, 128]], "word_ranges": [[0, 22]], "text": "Cuando un brote grave no responde a corticoterapia iv a dosis plenas (1mg/kg de peso) hay que pasar a ciclosporina o infliximab."}, "5": {"exist": true, "char_ranges": [[129, 223]], "word_ranges": [[22, 35]], "text": "Etanercept no est\u00e1 aprobado en Europa para la enfermedad inflamatorio intestinal (de momento)."}}} {"id": 224, "year": 2014, "question_id_specific": 63, "full_question": "Paciente de 52 a\u00f1os de edad que refiere disnea de medianos esfuerzos desde hace 6 meses. No ha presentado angina ni s\u00edncopes. La exploraci\u00f3n f\u00edsica y las pruebas diagn\u00f3sticas demuestran la existencia de un gradiente transvalvular a\u00f3rtico medio de 55 mmHg y un \u00e1rea calculada de 0,7 cm\u00b2. Fracci\u00f3n de eyecci\u00f3n del 65%. El tratamiento que debe indicarse a este paciente es:", "full_answer": "Algo a\u00f3rtico. Me dan un gradiente, que sin saber los valores parece algo alto (\u00a155 mmHg!) y el \u00e1rea, que a ojo se ve chiquit\u00edn: esto va a ir de estenosis a\u00f3rtica\u2026 Y tiene disnea. Estenosis a\u00f3rtica sintom\u00e1tica, \u00bfquerr\u00e1 el examinador que le mande a operar? Con las gu\u00edas en la mano, as\u00ed es: se trata de una estenosis severa tanto por el \u00e1rea (<1 cm\u00b2) como por el gradiente medio (>40 mmHg), con una fracci\u00f3n de eyecci\u00f3n conservada, y sintom\u00e1tica. Indicaci\u00f3n quir\u00fargica. Si hacemos la 1 le quitamos esperanza de vida, porque permitimos que el ventr\u00edculo claudique. La 2 est\u00e1 desfasada: \u00bfvalvuloplastia con bal\u00f3n? S\u00f3lo si el riesgo quir\u00fargico es inasumible, y en ese caso har\u00edamos la 5, implante de una TAVI (ver [1]). Pero se trata de un hombre de 52 casta\u00f1as, que asumimos que tiene buena carrocer\u00eda porque camina para tener disnea. As\u00ed que a quir\u00f3fano. \u00bfUn homoinjerto? Eso es la t\u00e9cnica de Ross, le quito la pulmonar y se la pongo donde la a\u00f3rtica. Pero es raro, complejo y s\u00f3lo se hace en pedi\u00e1tricos. Lo correcto es la 3, sin duda: sustituir por una pr\u00f3tesis, preferiblemente mec\u00e1nica a no ser que el paciente tenga contraindicaci\u00f3n para la anticoagulaci\u00f3n.", "type": "CARDIOLOG\u00cdA", "options": {"1": "Diur\u00e9ticos y controles m\u00e1s frecuentes por un especialista.", "2": "En caso de aumentar la disnea de esfuerzo, debe practicarse una dilataci\u00f3n percut\u00e1nea de la v\u00e1lvula a\u00f3rtica con cat\u00e9ter de bal\u00f3n.", "3": "Sustituci\u00f3n de la v\u00e1lvula a\u00f3rtica por una pr\u00f3tesis/biopr\u00f3tesis.", "4": "Sustituci\u00f3n de la v\u00e1lvula a\u00f3rtica por un homoinjerto.", "5": "Implante de una v\u00e1lvula percut\u00e1nea."}, "correct_option": 3, "explanations": {"1": {"exist": true, "char_ranges": [[468, 561]], "word_ranges": [[83, 98]], "text": "Si hacemos la 1 le quitamos esperanza de vida, porque permitimos que el ventr\u00edculo claudique."}, "2": {"exist": true, "char_ranges": [[562, 652]], "word_ranges": [[98, 112]], "text": "La 2 est\u00e1 desfasada: \u00bfvalvuloplastia con bal\u00f3n? S\u00f3lo si el riesgo quir\u00fargico es inasumible,"}, "3": {"exist": true, "char_ranges": [[289, 467]], "word_ranges": [[54, 83]], "text": "se trata de una estenosis severa tanto por el \u00e1rea (<1 cm\u00b2) como por el gradiente medio (>40 mmHg), con una fracci\u00f3n de eyecci\u00f3n conservada, y sintom\u00e1tica. Indicaci\u00f3n quir\u00fargica."}, "4": {"exist": true, "char_ranges": [[954, 1002]], "word_ranges": [[169, 178]], "text": "es raro, complejo y s\u00f3lo se hace en pedi\u00e1tricos."}, "5": {"exist": true, "char_ranges": [[720, 830]], "word_ranges": [[126, 145]], "text": "se trata de un hombre de 52 casta\u00f1as, que asumimos que tiene buena carrocer\u00eda porque camina para tener disnea."}}} {"id": 430, "year": 2018, "question_id_specific": 115, "full_question": "Mujer de 65 a\u00f1os, sometida a trasplante de precursores hematopoy\u00e9tlcos hace 30 dias y neutropenia grave. Comienza con tos y expectoraci\u00f3n hemoptoica adem\u00e1s de fiebre y disnea de moderados esfuerzos que no mejora a pesar de cinco d\u00edas de tratamiento con amoxicilina-clavul\u00e1nico (875/125 mg/8 h) y levofloxacino (500 mg/12 h). En la radiograf\u00eda de t\u00f3rax se observan m\u00faltiples n\u00f3dulos pulmonares mal definidos, alguna de ellos cavitado. De entre los siguientes, \u00bfcual es el diagn\u00f3stico de sospecha m\u00e1s probable?", "full_answer": "Paciente en su d\u00eda 30 post-transplante de precursores hematopoy\u00e9ticos, en neutropenia grave que acude por tos con expectoraci\u00f3n hemoptoica. Este cuadro usualmente se aprecia en la infecci\u00f3n por Aspergillus spp. Podemos descartar la infecci\u00f3n por Staphylococcus aureus por el tratamiento previo, aunque no sabemos si plantea la respuesta tambi\u00e9n un posible MRSA.", "type": "ENFERMEDADES INFECCIOSAS Y MICROBIOLOG\u00cdA", "options": {"1": "Embolismos s\u00e9pticos.", "2": "Aspergilosis pulmonar invasiva.", "3": "Neumon\u00eda por Staphylococcus aureus.", "4": "Candidiasis sist\u00e9mica.", "5": NaN}, "correct_option": 2, "explanations": {"1": {"exist": false, "char_ranges": [], "word_ranges": [], "text": ""}, "2": {"exist": true, "char_ranges": [[0, 210]], "word_ranges": [[0, 30]], "text": "Paciente en su d\u00eda 30 post-transplante de precursores hematopoy\u00e9ticos, en neutropenia grave que acude por tos con expectoraci\u00f3n hemoptoica. Este cuadro usualmente se aprecia en la infecci\u00f3n por Aspergillus spp."}, "3": {"exist": true, "char_ranges": [[211, 293]], "word_ranges": [[30, 41]], "text": "Podemos descartar la infecci\u00f3n por Staphylococcus aureus por el tratamiento previo,"}, "4": {"exist": false, "char_ranges": [], "word_ranges": [], "text": ""}, "5": {"exist": false, "char_ranges": [], "word_ranges": [], "text": ""}}} {"id": 588, "year": 2022, "question_id_specific": 63, "full_question": "Var\u00f3n de 36 a\u00f1os que consulta por hiperemia conjuntival y sensaci\u00f3n de cuerpo extra\u00f1o. En la exploraci\u00f3n presenta afectaci\u00f3n del ganglio preauricular. \u00bfCu\u00e1l de las siguientes patolog\u00edas NO se corresponde con dicha exploraci\u00f3n?:", "full_answer": "La afectaci\u00f3n del ganglio preauricular es la linfadenopat\u00eda t\u00edpica de los cuadros infecciosos conjuntivales. Bien es cierto que las conjuntivitis bacterianas t\u00edpicas no suelen cursar con adenopat\u00edas. Son de un curso m\u00e1s r\u00e1pido y autorresolutivo y no despierta la respuesta linfoide como hacen t\u00edpicamente los adenovirus. O ya puestos, algunas bacterias m\u00e1s at\u00edpicas que producen conjuntivitis cr\u00f3nicas, como la Chlamydia, o ya bacterias mucho m\u00e1s infrecuentes como las responsables del s\u00edndrome oculoglandular de Parinaud (Bartonella henselae, Francisella tularensis, Sporothrix schenckii, etc). Siendo la adenopat\u00eda muy t\u00edpica de la conjuntivitis adenov\u00edrica, esta opci\u00f3n es la m\u00e1s f\u00e1cil de descartar. Teniendo en cuenta que las clamidias despiertan la respuesta linfoide de forma parecida a los adenovirus (producen tambi\u00e9n fol\u00edculos conjuntivales, por ejemplo), esta ser\u00eda la segunda opci\u00f3n m\u00e1s f\u00e1cil de descartar. Quiz\u00e1s lo dif\u00edcil es descartar el s\u00edndrome oculoglandular de Parinaud, porque es una enfermedad muy infrecuente. Si no caemos en la cuenta que es una enfermedad infecciosa, podr\u00edamos dudar. En cualquier caso, la conjuntivitis al\u00e9rgica no es un problema infeccioso. As\u00ed que no va a tener adenopat\u00edas. Aunque dudemos con alguna opci\u00f3n anterior (sobre todo con el Parinaud), si nos quedamos con los conceptos b\u00e1sicos (la conjuntivitis al\u00e9rgica no cursa con adenopat\u00edas) podemos acertar. La opci\u00f3n 2 es la correcta.", "type": "OFTALMOLOG\u00cdA", "options": {"1": "Conjuntivitis adenov\u00edrica.", "2": "Conjuntivitis al\u00e9rgica.", "3": "Sindrome oculoglandular de Parinaud.", "4": "Conjuntivitis por Chlamydia.", "5": NaN}, "correct_option": 2, "explanations": {"1": {"exist": true, "char_ranges": [[596, 702]], "word_ranges": [[81, 98]], "text": "Siendo la adenopat\u00eda muy t\u00edpica de la conjuntivitis adenov\u00edrica, esta opci\u00f3n es la m\u00e1s f\u00e1cil de descartar."}, "2": {"exist": true, "char_ranges": [[1218, 1429]], "word_ranges": [[176, 209]], "text": "Aunque dudemos con alguna opci\u00f3n anterior (sobre todo con el Parinaud), si nos quedamos con los conceptos b\u00e1sicos (la conjuntivitis al\u00e9rgica no cursa con adenopat\u00edas) podemos acertar. La opci\u00f3n 2 es la correcta."}, "3": {"exist": true, "char_ranges": [[1108, 1217]], "word_ranges": [[158, 176]], "text": "En cualquier caso, la conjuntivitis al\u00e9rgica no es un problema infeccioso. As\u00ed que no va a tener adenopat\u00edas."}, "4": {"exist": true, "char_ranges": [[703, 917]], "word_ranges": [[98, 129]], "text": "Teniendo en cuenta que las clamidias despiertan la respuesta linfoide de forma parecida a los adenovirus (producen tambi\u00e9n fol\u00edculos conjuntivales, por ejemplo), esta ser\u00eda la segunda opci\u00f3n m\u00e1s f\u00e1cil de descartar."}, "5": {"exist": false, "char_ranges": [], "word_ranges": [], "text": ""}}} {"id": 11, "year": 2011, "question_id_specific": 74, "full_question": "Diez d\u00edas despu\u00e9s de practicarle una hemitiroidectom\u00eda a un paciente con un n\u00f3dulo tiroideo de 1,5 cm recibe el siguiente informe anatomopatol\u00f3gico: \u201cpatr\u00f3n folicular muy celular sin invasi\u00f3n capsular y que infiltra vasos sangu\u00edneos y nervios adyacentes\u201d. Se\u00f1ale la respuesta correcta:", "full_answer": "Creo que la respuesta correcta es la 3 las respuestas 2,4 y 5 son claramente falsas. La duda se produce entre la 1 y la 2. El enunciado de la 1 es correcto en casos de carcinoma folicular sin invasi\u00f3n capsular puede en realizarse una tiroidectom\u00eda total. Pero si existen datos de mal pron\u00f3stico y la posibilidad de met\u00e1stasis- como en este caso- lo mejor es completar la tiroidectom\u00eda y realizar un rastreo con Iodo radioactivo para descartar met\u00e1stasis o tratarlas si est\u00e1n presentes.", "type": "CIRUG\u00cdA", "options": {"1": "Como no existe invasi\u00f3n capsular no es necesaria m\u00e1s cirug\u00eda.", "2": "El patr\u00f3n folicular es indicaci\u00f3n de vaciamiento ganglionar cervical profil\u00e1ctico.", "3": "Se trata de un carcinoma folicular y requiere completar la tiroidectom\u00eda.", "4": "El informe permite descartar con total seguridad un carcinoma papilar en el tiroides remanente.", "5": "La medici\u00f3n de calcitonina plasm\u00e1tica permitir\u00e1 distinguir entre carcinoma papilar y folicular."}, "correct_option": 3, "explanations": {"1": {"exist": true, "char_ranges": [[123, 485]], "word_ranges": [[26, 84]], "text": "El enunciado de la 1 es correcto en casos de carcinoma folicular sin invasi\u00f3n capsular puede en realizarse una tiroidectom\u00eda total. Pero si existen datos de mal pron\u00f3stico y la posibilidad de met\u00e1stasis- como en este caso- lo mejor es completar la tiroidectom\u00eda y realizar un rastreo con Iodo radioactivo para descartar met\u00e1stasis o tratarlas si est\u00e1n presentes."}, "2": {"exist": false, "char_ranges": [], "word_ranges": [], "text": ""}, "3": {"exist": true, "char_ranges": [[260, 485]], "word_ranges": [[48, 84]], "text": "si existen datos de mal pron\u00f3stico y la posibilidad de met\u00e1stasis- como en este caso- lo mejor es completar la tiroidectom\u00eda y realizar un rastreo con Iodo radioactivo para descartar met\u00e1stasis o tratarlas si est\u00e1n presentes."}, "4": {"exist": false, "char_ranges": [], "word_ranges": [], "text": ""}, "5": {"exist": false, "char_ranges": [], "word_ranges": [], "text": ""}}} {"id": 535, "year": 2021, "question_id_specific": 121, "full_question": "Mujer de 27 a\u00f1os de raza negra que consulta por la aparici\u00f3n de edemas en miembros inferiores, disminuci\u00f3n de diuresis, fiebre y un rash malar en alas de mariposa de 20 d\u00edas de evoluci\u00f3n. Anal\u00edticamente destaca creatinina 3 mg/dl, leucopenia, hemoglobina 10,5 g/dl (normoc\u00edtica normocr\u00f3mica), PCR 9 mg/dl y VSG 60 mm. Respecto a la patolog\u00eda que presenta esta paciente, se\u00f1ale la respuesta correcta:", "full_answer": "Otra pregunta sobre los anticuerpos del LES. Caso cl\u00ednico de LES con afectaci\u00f3n renal, en el que los anti-ADN bicatenario positivos aumentan el riesgo de nefritis. En cuanto al resto, los anti-Sm son espec\u00edficos del LES, los ANA se detectan en m\u00e1s del 90% de los casos, y los anti-centr\u00f3mero son t\u00edpicos de la esclerodermia.", "type": "REUMATOLOG\u00cdA", "options": {"1": "Los anti-Sm no son espec\u00edficos de esta patolog\u00eda.", "2": "Los anticuerpos anticentr\u00f3mero se detectan en el 90 % de los casos.", "3": "Los ANA (anticuerpos antinucleares) se detectan en el 50 % de los casos.", "4": "Los anti-ADN bicatenario o nativo se correlacionan con el riesgo de nefritis.", "5": NaN}, "correct_option": 4, "explanations": {"1": {"exist": true, "char_ranges": [[184, 219]], "word_ranges": [[30, 36]], "text": "los anti-Sm son espec\u00edficos del LES,"}, "2": {"exist": true, "char_ranges": [[272, 324]], "word_ranges": [[48, 55]], "text": "los anti-centr\u00f3mero son t\u00edpicos de la esclerodermia."}, "3": {"exist": true, "char_ranges": [[221, 268]], "word_ranges": [[36, 47]], "text": "los ANA se detectan en m\u00e1s del 90% de los casos,"}, "4": {"exist": true, "char_ranges": [[45, 163]], "word_ranges": [[7, 26]], "text": "Caso cl\u00ednico de LES con afectaci\u00f3n renal, en el que los anti-ADN bicatenario positivos aumentan el riesgo de nefritis."}, "5": {"exist": false, "char_ranges": [], "word_ranges": [], "text": ""}}} {"id": 452, "year": 2018, "question_id_specific": 149, "full_question": "En una mujer con una crisis epil\u00e9ptica que presenta las siguientes caracter\u00edsticas cl\u00ednicas: aura epig\u00e1strica, olor desagradable, desconexi\u00f3n del medio, automatismos motores (chupeteo, degluci\u00f3n, apertura y cierre de una mano) y amnesia postcritica, \u00bfcual es su sospecha diagn\u00f3stica?", "full_answer": "Claramente la respuesta es la 4, con una cl\u00ednica muy caracter\u00edstica de crisis temporales.", "type": "NEUROLOG\u00cdA", "options": {"1": "Crisis generalizada no convulsiva o ausencia tipica.", "2": "Epilepsia parcial continua.", "3": "Crisis amiot\u00f3nica.", "4": "Crisis parcial compleja del l\u00f3bulo temporal.", "5": NaN}, "correct_option": 4, "explanations": {"1": {"exist": false, "char_ranges": [], "word_ranges": [], "text": ""}, "2": {"exist": false, "char_ranges": [], "word_ranges": [], "text": ""}, "3": {"exist": false, "char_ranges": [], "word_ranges": [], "text": ""}, "4": {"exist": true, "char_ranges": [[0, 89]], "word_ranges": [[0, 14]], "text": "Claramente la respuesta es la 4, con una cl\u00ednica muy caracter\u00edstica de crisis temporales."}, "5": {"exist": false, "char_ranges": [], "word_ranges": [], "text": ""}}} {"id": 55, "year": 2011, "question_id_specific": 74, "full_question": "Diez d\u00edas despu\u00e9s de practicarle una hemitiroidectom\u00eda a una paciente con un n\u00f3dulo tiroideo de 1,5 cm recibe el siguiente informe definitivo por parte del anatomopat\u00f3logo: \u00abpatr\u00f3n folicular muy celular sin invasi\u00f3n capsular que invade los vasos sangu\u00edneos y nervios adyacentes\u00bb. Se\u00f1ale la respuesta correcta:", "full_answer": "Me parece una pregunta un poco m\u00e1s dif\u00edcil ya que hay que tener conceptos claros de anatom\u00eda patol\u00f3gica. De todas formas, se podr\u00eda sacar por exclusi\u00f3n y por la forma en la que se expresan las respuestas: Respuesta 2: el vaciamiento profil\u00e1ctico est\u00e1 indicado en el medular. Respuesta 4: simplemente por la forma de expresarla ya se deber\u00eda haber descartado. Respuesta 5: ya sab\u00e9is que la calcitonina se utiliza en el carcinoma medular y no en el diferenciado. Las respuestas 1 y 3 son contrarias, una recomienda completar la tiroidectom\u00eda y otra no. En los \u00fanicos que no se recomienda tiroidectom\u00eda completa son los microcarcinomas que midan menos de 1 cm.", "type": "ENDOCRINOLOG\u00cdA", "options": {"1": "Como no existe invasi\u00f3n capsular no es conveniente realizar m\u00e1s cirug\u00eda.", "2": "El patr\u00f3n folicular es indicaci\u00f3n de vaciamiento ganglionar cervical profil\u00e1ctico.", "3": "Se trata de un carcinoma folicular y requiere completar la tiroidectom\u00eda.", "4": "El informe permite descartar un carcinoma papilar con total seguridad en el tiroides remanente.", "5": "La medici\u00f3n de la calcitonina plasm\u00e1tica nos permitir\u00e1 diferenciar entre un carcinoma papilar y folicular."}, "correct_option": 3, "explanations": {"1": {"exist": true, "char_ranges": [[461, 657]], "word_ranges": [[78, 111]], "text": "Las respuestas 1 y 3 son contrarias, una recomienda completar la tiroidectom\u00eda y otra no. En los \u00fanicos que no se recomienda tiroidectom\u00eda completa son los microcarcinomas que midan menos de 1 cm."}, "2": {"exist": true, "char_ranges": [[218, 274]], "word_ranges": [[39, 47]], "text": "el vaciamiento profil\u00e1ctico est\u00e1 indicado en el medular."}, "3": {"exist": true, "char_ranges": [[461, 657]], "word_ranges": [[78, 111]], "text": "Las respuestas 1 y 3 son contrarias, una recomienda completar la tiroidectom\u00eda y otra no. En los \u00fanicos que no se recomienda tiroidectom\u00eda completa son los microcarcinomas que midan menos de 1 cm."}, "4": {"exist": true, "char_ranges": [[288, 358]], "word_ranges": [[49, 60]], "text": "simplemente por la forma de expresarla ya se deber\u00eda haber descartado."}, "5": {"exist": true, "char_ranges": [[386, 460]], "word_ranges": [[65, 78]], "text": "la calcitonina se utiliza en el carcinoma medular y no en el diferenciado."}}} {"id": 281, "year": 2016, "question_id_specific": 33, "full_question": "Mujer de 67 a\u00f1os diagnosticada de un carcinoma ductal infiltrante de mama y sin historia familiar de neoplasia. \u00bfQu\u00e9 estudios adicionales deben realizarse en el tumor por sus implicaciones cl\u00ednico-terap\u00e9uticas?", "full_answer": "La respuesta correcta ser\u00eda la 2, ESTUDIO DE RECEPTORES HORMONALES Y DE HER2 ya que unos receptores positivos para HER2 implican una mayor agresividad del tumor pero tratamiento con trastuzumab y unos receptores hormonales positivos implicar\u00edan la utilizaci\u00f3n de hormonoterapia. La respuesta 1 es descartable de visu, mientras que la 3 y la 4 debieran descartarse ya que al tratarse de la primera mujer en la familia con carcinoma de mama, no ser\u00eda obligado el estudio de los familiares de la paciente. As\u00ed mismo, la e-cadherina es importante para el diagn\u00f3stico diferencial de un carcinoma lobulillar de mama, no de uno ductal.", "type": "ANATOM\u00cdA PATOL\u00d3GICA", "options": {"1": "Estudio fenot\u00edpico completo mediante citometr\u00eda de flujo.", "2": "Estudio de receptores hormonales y de HER2.", "3": "Estudio de receptores hormonales, e-cadherina y estudio de familiares de primer grado.", "4": "Estudio de BRCA 1-2 y estudio de familiares de primer grado.", "5": NaN}, "correct_option": 2, "explanations": {"1": {"exist": true, "char_ranges": [[279, 316]], "word_ranges": [[40, 47]], "text": "La respuesta 1 es descartable de visu,"}, "2": {"exist": true, "char_ranges": [[84, 278]], "word_ranges": [[15, 40]], "text": "unos receptores positivos para HER2 implican una mayor agresividad del tumor pero tratamiento con trastuzumab y unos receptores hormonales positivos implicar\u00edan la utilizaci\u00f3n de hormonoterapia."}, "3": {"exist": true, "char_ranges": [[331, 502]], "word_ranges": [[49, 82]], "text": "la 3 y la 4 debieran descartarse ya que al tratarse de la primera mujer en la familia con carcinoma de mama, no ser\u00eda obligado el estudio de los familiares de la paciente."}, "4": {"exist": true, "char_ranges": [[331, 502]], "word_ranges": [[49, 82]], "text": "la 3 y la 4 debieran descartarse ya que al tratarse de la primera mujer en la familia con carcinoma de mama, no ser\u00eda obligado el estudio de los familiares de la paciente."}, "5": {"exist": false, "char_ranges": [], "word_ranges": [], "text": ""}}} {"id": 310, "year": 2016, "question_id_specific": 216, "full_question": "Unos padres acuden a urgencias con su hijo de dos a\u00f1os porque refieren que se ha dado un golpe cerca del ojo derecho jugando. Efectivamente se observa un hematoma en el p\u00e1rpado derecho aparentemente sin importancia. En el fondo de ojo se observan hemorragias intrarretinianas no s\u00f3lo en el ojo que refieren los padres sino tambi\u00e9n en el otro ojo. Llama la atenci\u00f3n que el ni\u00f1o parece adormilado y con poco tono. \u00bfCu\u00e1l de las siguientes afirmaciones deber\u00eda considerar respecto a este cuadro cl\u00ednico?", "full_answer": "Pregunta en principio f\u00e1cil, pero con un poco de trampa. Si al leerla tienes la intuici\u00f3n correcta y vas directamente a lo que piensas, posiblemente la aciertes. El problema es la opci\u00f3n 4, que nos puede confundir. Veamos: tenemos un golpe en la zona del ojo derecho, con un hematoma en el p\u00e1rpado. Hasta aqu\u00ed todo normal. El problema es que aparecen hemorragias retinianas no s\u00f3lo en ese ojo (que ya de por s\u00ed es raro, porque por un golpe externo del ojo en un ni\u00f1o raramente se produce sangrado en la retina), sino en el otro. Con lo cual, algo que no ha sido el golpe ha causado hemorragias en ambas retinas. Luego nos dicen que el ni\u00f1o parece adormilado y con poco tono. Aqu\u00ed ya tenemos las dos claves; tenemos que sospechar lo que no se menciona en las respuestas, pero que es lo que tiene el ni\u00f1o: el s\u00edndrome del ni\u00f1o sacudido (shaken baby syndrome). Lo caracter\u00edstico de este s\u00edndrome son las hemorragias retinianas bilaterales, edema cerebral y hematoma subdural. Tambi\u00e9n puede haber fractura de huesos largos y otros problemas. El adormilamiento y la hipoton\u00eda nos permiten sospechar las complicaciones intracraneales. La causa del s\u00edndrome del ni\u00f1o sacudido es casi siempre el maltrato, as\u00ed que la respuesta es la 1. Las opciones 2 y 3 son f\u00e1ciles de descartar: nada nos indica traumatismo perforante, y no explica las hemorragias del otro ojo. Y el edema de berlin es un edema de la m\u00e1cula y aparece como una lesi\u00f3n blanquecina, que no est\u00e1 descrita en el enunciado. La trampa es la opci\u00f3n 4. La retinopat\u00eda de Purstcher puede aparecer tambi\u00e9n en el contexto de del ni\u00f1o del ni\u00f1o sacudido, y por tanto ocurrir en el caso de maltrato infantil. Al ver estas dos preguntas en parte relacionadas, podemos dudar. Sin embargo, si leemos bien la respuesta 4, no nos habla de que si hay maltrato infantil se pueda producir un Purstcher. Nos afirma que el cuadro descrito antes (hemorragias intrarretinianas, sin otros hallazgos) es caracter\u00edstico de la angiopat\u00eda de Purstcher. Y no es as\u00ed. En el Purstcher hay hemorragias, pero tambi\u00e9n exudados algodonosos y edema. Si s\u00f3lo hay hemorragias, no es un Purstcher. Aqu\u00ed est\u00e1 lo lioso, pero realmente est\u00e1 bien redactado. Si tuviera un Purstcher tambi\u00e9n podr\u00eda ser por maltrato infantil, pero no lo tiene.", "type": "OFTALMOLOG\u00cdA", "options": {"1": "Es una historia altamente sugestiva de maltrato infantil.", "2": "Es la evoluci\u00f3n normal de un traumatismo intraocular no perforante.", "3": "El diagn\u00f3stico m\u00e1s probable es un edema de Berlin.", "4": "Es el cuadro caracter\u00edstico de la angiopat\u00eda retiniana traum\u00e1tica de Purstcher.", "5": NaN}, "correct_option": 1, "explanations": {"1": {"exist": true, "char_ranges": [[1129, 1227]], "word_ranges": [[194, 213]], "text": "La causa del s\u00edndrome del ni\u00f1o sacudido es casi siempre el maltrato, as\u00ed que la respuesta es la 1."}, "2": {"exist": true, "char_ranges": [[1228, 1355]], "word_ranges": [[213, 235]], "text": "Las opciones 2 y 3 son f\u00e1ciles de descartar: nada nos indica traumatismo perforante, y no explica las hemorragias del otro ojo."}, "3": {"exist": true, "char_ranges": [[1228, 1355]], "word_ranges": [[213, 235]], "text": "Las opciones 2 y 3 son f\u00e1ciles de descartar: nada nos indica traumatismo perforante, y no explica las hemorragias del otro ojo."}, "4": {"exist": true, "char_ranges": [[1995, 2115]], "word_ranges": [[346, 365]], "text": "En el Purstcher hay hemorragias, pero tambi\u00e9n exudados algodonosos y edema. Si s\u00f3lo hay hemorragias, no es un Purstcher."}, "5": {"exist": false, "char_ranges": [], "word_ranges": [], "text": ""}}} {"id": 293, "year": 2016, "question_id_specific": 214, "full_question": "Un paciente acude a urgencias por presentar una eritrodermia con fiebre y malestar general. En la exploraci\u00f3n se aprecia en las u\u00f1as la presencia de piqueteado ungueal y unas zonas amarillentas distales en mancha de aceite. \u00bfCu\u00e1l es la enfermedad primaria que ha originado el cuadro?", "full_answer": "Cualquiera de las 4 opciones puede ser la causa de la eritrodermia, aunque la descripci\u00f3n de lesiones ungueales altamente sugestivas de psoriasis hacen de este diagn\u00f3stico el m\u00e1s probable.", "type": "DERMATOLOG\u00cdA, VENEREOLOG\u00cdA Y CIRUG\u00cdA PL\u00c1STICA", "options": {"1": "Linfoma cut\u00e1neo.", "2": "Dermatitis at\u00f3pica.", "3": "Psoriasis.", "4": "Ictiosis.", "5": NaN}, "correct_option": 3, "explanations": {"1": {"exist": false, "char_ranges": [], "word_ranges": [], "text": ""}, "2": {"exist": false, "char_ranges": [], "word_ranges": [], "text": ""}, "3": {"exist": true, "char_ranges": [[75, 188]], "word_ranges": [[13, 29]], "text": "la descripci\u00f3n de lesiones ungueales altamente sugestivas de psoriasis hacen de este diagn\u00f3stico el m\u00e1s probable."}, "4": {"exist": false, "char_ranges": [], "word_ranges": [], "text": ""}, "5": {"exist": false, "char_ranges": [], "word_ranges": [], "text": ""}}} {"id": 132, "year": 2012, "question_id_specific": 229, "full_question": "Hombre de 54 a\u00f1os, con antecedentes personales de DM, HTA e insuficiencia renal cr\u00f3nica leve, que presenta somnolencia y hemiparesia derecha. En la anal\u00edtica: creatinina de 2.3 mg/dl, Hb 10.3 g/dl y plaquetas 20000 mm3, con coagulaci\u00f3n normal. En TAC craneal: lesiones isqu\u00e9micas con necrosis microhemorr\u00e1gicas. Posteriormente comienza con deterioro progresivo del nivle de conciencia y aumento de Cr, LDH y bilirrubina no conjugada. Se realiza Coombs directo que es negativo. Esquistocitos en frotis. Ante la sospecha diagn\u00f3stica la conducta a seguir es:", "full_answer": "Veamos: un paciente con ciertos antecedentes que tiene: \u2013 Esquistocitos en frotis: indica anemia microangiop\u00e1tica, ruptura de hemat\u00edes de causa mec\u00e1nica, cosa que se puede ver fundamentalmente en PTT, anemia hemol\u00edtica autoinmune y CID. \u2013 Bicitopenia: ah\u00ed las causas pueden ser muuuuuy variadas. \u2013 Elevaci\u00f3n de LDH y Bilirrubina, lo cual indica ruptura de hemat\u00edes a cascoporro. \u2013 Coombs directo negativo: dato esencial, indica que no hay ning\u00fan anticuerpo que se pegue al hemat\u00ede. Con estos mimbres vamos a tejer el cesto: Respuesta 1: no puede ser porque una transformaci\u00f3n hemorr\u00e1gica no da una elevaci\u00f3n de LDH y Cr tan brutal, adem\u00e1s de no explicar la esquistocitosis. Respuesta 2: plasmaf\u00e9resis urgente\u2026en caso de p\u00farpura tromb\u00f3tica trombocitop\u00e9nica s\u00ed; esta respuesta me gusta porque justifica la bicitopenia, ya que se produce anemia hemol\u00edtica microangiop\u00e1tica por ruptura de los hemat\u00edes al ser fragmentados por los mult\u00edmeros de factor de von Willebrand ultralargos en caso de deficiencia de la enzima ADAMST13, con lo que se producen agregados plaquetarios brutales y el cit\u00f3metro de flujo no cuenta 400 plaquetas, sino un molondr\u00f3n de plaquetas que forman una masa, de ah\u00ed la trombocitopenia. Tambi\u00e9n explica el Coombs directo negativo, la elevaci\u00f3n de LDH y Cr. Respuesta 3: una situaci\u00f3n tan cr\u00edtica como la de ese paciente indica que algo gordo est\u00e1 pasando y yo no me sentar\u00eda a esperar a que los glucocorticoides hicieran efecto\u2026y no caigo ahora mismo qu\u00e9 patolog\u00eda da esa sintomatolog\u00eda y tenga ese tratamiento como est\u00e1ndar. Respuesta 4: buscar desencadenante de cuadro de CID\u2026tendr\u00eda sentido de no ser porque en el CID es MUY caracter\u00edstico el consumo de factores de coagulaci\u00f3n, lo cual no se da en este caso. Respuesta 5:un mieloma no da esquistocitosis ni tiene esa sintomatolog\u00eda, tendr\u00edan que dar otros datos que apoyara esa patolog\u00eda, como el dolor \u00f3seo y la hipercalcemia\u2026", "type": "NEUROLOG\u00cdA Y NEUROCIRUG\u00cdA", "options": {"1": "Nuevo TAC craneal ante la sospecha de transformaci\u00f3n hemorr\u00e1gica de ictus isqu\u00e9mico.", "2": "Iniciar plasmaf\u00e9resis urgente.", "3": "Inicio de tratamiento con glucocorticoides a dosis de 1 mg/kg/d\u00eda.", "4": "Buscar causa desencadente de cuadro de CID.", "5": "Sospecha de Mieloma M\u00faltiple: aspirado de m\u00e9dula \u00f3sea."}, "correct_option": 2, "explanations": {"1": {"exist": true, "char_ranges": [[524, 673]], "word_ranges": [[82, 107]], "text": "Respuesta 1: no puede ser porque una transformaci\u00f3n hemorr\u00e1gica no da una elevaci\u00f3n de LDH y Cr tan brutal, adem\u00e1s de no explicar la esquistocitosis."}, "2": {"exist": true, "char_ranges": [[674, 759]], "word_ranges": [[107, 117]], "text": "Respuesta 2: plasmaf\u00e9resis urgente\u2026en caso de p\u00farpura tromb\u00f3tica trombocitop\u00e9nica s\u00ed;"}, "3": {"exist": true, "char_ranges": [[1276, 1446]], "word_ranges": [[199, 229]], "text": "Respuesta 3: una situaci\u00f3n tan cr\u00edtica como la de ese paciente indica que algo gordo est\u00e1 pasando y yo no me sentar\u00eda a esperar a que los glucocorticoides hicieran efecto\u2026y"}, "4": {"exist": true, "char_ranges": [[1545, 1731]], "word_ranges": [[244, 277]], "text": "Respuesta 4: buscar desencadenante de cuadro de CID\u2026tendr\u00eda sentido de no ser porque en el CID es MUY caracter\u00edstico el consumo de factores de coagulaci\u00f3n, lo cual no se da en este caso."}, "5": {"exist": true, "char_ranges": [[1732, 1900]], "word_ranges": [[277, 303]], "text": "Respuesta 5:un mieloma no da esquistocitosis ni tiene esa sintomatolog\u00eda, tendr\u00edan que dar otros datos que apoyara esa patolog\u00eda, como el dolor \u00f3seo y la hipercalcemia\u2026"}}} {"id": 49, "year": 2011, "question_id_specific": 159, "full_question": "Ni\u00f1o de 8 a\u00f1os de edad que refiere cojera de la extremidad inferior derecha, de presentaci\u00f3n insidiosa y un mes de evoluci\u00f3n. No existen antecedentes traum\u00e1ticos ni signos constitucionales. A la exploraci\u00f3n se evidencia restricci\u00f3n a la movilizaci\u00f3n pasiva de dicha extremidad, especialmente en la abducci\u00f3n y la rotaci\u00f3n interna. De las siguientes afirmaciones todas son ciertas EXCEPTO una:", "full_answer": "La respuesta correcta es la 3. La enfermedad de Perthes es m\u00e1s frecuente y tiene mejor pron\u00f3stico en los m\u00e1s peque\u00f1os.", "type": "PEDIATR\u00cdA", "options": {"1": "Se aconseja la realizaci\u00f3n de una radiograf\u00eda de caderas.", "2": "En la sinovitis transitoria el comienzo suele ser agudo.", "3": "En la enfermedad de Legg-Calv\u00e9-Perthes el pron\u00f3stico es peor cuanto menor es la edad de comienzo.", "4": "La causa puede ser un estado de hipercoagulabilidad.", "5": "La opci\u00f3n terap\u00e9utica variar\u00e1 dependiendo del grado de afectaci\u00f3n."}, "correct_option": 3, "explanations": {"1": {"exist": false, "char_ranges": [], "word_ranges": [], "text": ""}, "2": {"exist": false, "char_ranges": [], "word_ranges": [], "text": ""}, "3": {"exist": true, "char_ranges": [[31, 118]], "word_ranges": [[6, 21]], "text": "La enfermedad de Perthes es m\u00e1s frecuente y tiene mejor pron\u00f3stico en los m\u00e1s peque\u00f1os."}, "4": {"exist": false, "char_ranges": [], "word_ranges": [], "text": ""}, "5": {"exist": false, "char_ranges": [], "word_ranges": [], "text": ""}}} {"id": 120, "year": 2012, "question_id_specific": 192, "full_question": "M\u00e9dico traumat\u00f3logo de 40 a\u00f1os que en una serolog\u00eda rutinaria se le detecta niveles de antiHBs de 30 UI/l. Refiere vacunaci\u00f3n completa seg\u00fan pauta est\u00e1ndar frente a hepatitis B hace 4 a\u00f1os. Ser\u00eda recomendable:", "full_answer": "Mientras los niveles serol\u00f3gicos de antiHBs est\u00e9n por encima de 10 UI/l no es necesario revacunar.", "type": "MEDICINA PREVENTIVA Y EPIDEMIOLOG\u00cdA", "options": {"1": "Reiniciar pauta vacunal (0-1-6 meses).", "2": "Reiniciar pauta vacunal (0-1-2-12 meses).", "3": "No revacunar.", "4": "Darle una dosis de recuerdo o booster.", "5": "Darle inmunoglobulina anti-hepatitis B y reiniciar pauta vacunal (0-1-2-12 meses)."}, "correct_option": 3, "explanations": {"1": {"exist": false, "char_ranges": [], "word_ranges": [], "text": ""}, "2": {"exist": false, "char_ranges": [], "word_ranges": [], "text": ""}, "3": {"exist": true, "char_ranges": [[0, 98]], "word_ranges": [[0, 16]], "text": "Mientras los niveles serol\u00f3gicos de antiHBs est\u00e9n por encima de 10 UI/l no es necesario revacunar."}, "4": {"exist": false, "char_ranges": [], "word_ranges": [], "text": ""}, "5": {"exist": false, "char_ranges": [], "word_ranges": [], "text": ""}}} {"id": 304, "year": 2016, "question_id_specific": 174, "full_question": "Hombre de 87 a\u00f1os con antecedentes de hipertensi\u00f3n y gonartrosis. Situaci\u00f3n basal con autonom\u00eda funcional y cognitiva plena que le permite continuar viviendo solo en la comunidad. Realiza tratamiento habitual con perindopril y diur\u00e9tico tiaz\u00eddico para control de la presi\u00f3n arterial y toma de forma rutinaria ibuprofeno 1800 mg/d\u00eda para control de los s\u00edntomas derivados de su gonartrosis. Tras control rutinario se objetiva, de forma persistente, una presi\u00f3n arterial de 190 y TAD 80 mmHg. \u00bfCu\u00e1l ser\u00eda la modificaci\u00f3n terap\u00e9utica m\u00e1s razonable para conseguir el control de las cifras tensionales?", "full_answer": "Antes de intensificar el tratamiento antihipertensivo se deben solucionar las causas potenciales. En este caso se trata de un paciente a\u00f1oso con un tratamiento cr\u00f3nico con AINEs que produce hipertensi\u00f3n arterial secundaria a hipoperfusi\u00f3n renal. La opci\u00f3n m\u00e1s razonable ser\u00eda retirar el ibuprofeno y sustituirlo por un analg\u00e9sico de un grupo diferente a los AINEs (paracetamol, opi\u00e1ceos), vigilando posteriormente la evoluci\u00f3n de las cifras tensionales.", "type": "NEFROLOG\u00cdA", "options": {"1": "A\u00f1adir\u00eda un bloqueante del calcio.", "2": "Aumentar\u00eda la dosis de hidroclorotiazida a 25 mg/d\u00eda.", "3": "Cambiar\u00eda el ibuprofeno por paracetamol para evitar la posible influencia del mismo sobre el efecto de los hipotensores.", "4": "A\u00f1adir\u00eda un bloqueante alfa por la elevada prevalencia de s\u00edndrome prost\u00e1tico en varones de esta edad.", "5": NaN}, "correct_option": 3, "explanations": {"1": {"exist": false, "char_ranges": [], "word_ranges": [], "text": ""}, "2": {"exist": false, "char_ranges": [], "word_ranges": [], "text": ""}, "3": {"exist": true, "char_ranges": [[246, 453]], "word_ranges": [[35, 65]], "text": "La opci\u00f3n m\u00e1s razonable ser\u00eda retirar el ibuprofeno y sustituirlo por un analg\u00e9sico de un grupo diferente a los AINEs (paracetamol, opi\u00e1ceos), vigilando posteriormente la evoluci\u00f3n de las cifras tensionales."}, "4": {"exist": false, "char_ranges": [], "word_ranges": [], "text": ""}, "5": {"exist": false, "char_ranges": [], "word_ranges": [], "text": ""}}} {"id": 214, "year": 2014, "question_id_specific": 228, "full_question": "Hombre de 19 a\u00f1os de edad que consulta por un cuadro de 24 horas de evoluci\u00f3n de dolor, tumefacci\u00f3n e impotencia funcional de la rodilla derecha acompa\u00f1ado de fiebre de 38\u00b0C. La exploraci\u00f3n f\u00edsica pone de manifiesto signos inflamatorios y derrame articular en la rodilla derecha. Los datos anal\u00edticos muestran una leucocitosis con neutrofilia y una elevaci\u00f3n de la proteina C reactiva. Se hace el diagn\u00f3stico sindr\u00f3mico de monoartritis aguda. \u00bfCu\u00e1l es el diagn\u00f3stico etiol\u00f3gico m\u00e1s probable?", "full_answer": "Una monoartritis aguda asociada a fiebre, leucocitosis con neutrofilia y aumento de reactantes de fase aguda no siempre tiene un origen s\u00e9ptico. En ausencia de m\u00e1s informaci\u00f3n (anamnesis m\u00e1s completa sobre la enfermedad actual, factores de riesgo, antecedentes personales y familiares, s\u00edntomas o signos extraarticulares, etc.) cabe decir que tambi\u00e9n la 1 y 2 (y muy excepcionalmente la 5) podr\u00edan debutar con un cuadro cl\u00ednico y biol\u00f3gico similar. Con los datos que nos proporcionan y teniendo en cuenta que se trata de un var\u00f3n joven, la opci\u00f3n m\u00e1s probable ser\u00eda la de una artritis infecciosa bacteriana (la causada por mycobacterias suelen tener un curso cr\u00f3nico). Y sobre todo, por sus implicaciones, la primera que siempre hay que descartar.", "type": "REUMATOLOG\u00cdA", "options": {"1": "Artritis por microcristales.", "2": "Artritis reactiva.", "3": "Artritis infecciosa bacteriana.", "4": "Artritis infecciosa por mycobacterias.", "5": "Artritis reumatoide."}, "correct_option": 3, "explanations": {"1": {"exist": true, "char_ranges": [[669, 747]], "word_ranges": [[105, 118]], "text": "Y sobre todo, por sus implicaciones, la primera que siempre hay que descartar."}, "2": {"exist": false, "char_ranges": [], "word_ranges": [], "text": ""}, "3": {"exist": true, "char_ranges": [[487, 606]], "word_ranges": [[75, 96]], "text": "teniendo en cuenta que se trata de un var\u00f3n joven, la opci\u00f3n m\u00e1s probable ser\u00eda la de una artritis infecciosa bacteriana"}, "4": {"exist": true, "char_ranges": [[608, 666]], "word_ranges": [[96, 105]], "text": "(la causada por mycobacterias suelen tener un curso cr\u00f3nico)."}, "5": {"exist": false, "char_ranges": [], "word_ranges": [], "text": ""}}} {"id": 249, "year": 2014, "question_id_specific": 120, "full_question": "Mujer de 53 a\u00f1os, que consulta por fiebre de 15 d\u00edas de evoluci\u00f3n, sin s\u00edntomas de focalidad infecciosa. En la exploraci\u00f3n se detecta una hepatomegalia dolorosa a 5 cm del reborde costal y se palpa el bazo a 14 cm del reborde costal izquierdo. En el hemograma se aprecia Hb 8,5 g/dl, leucocitos 630/ml (linfocitos 63%, monocitos 20%, neutr\u00f3filos 17%) y plaquetas 35.000/ml. La bioqu\u00edmica muestra una elevaci\u00f3n moderada de la bioqu\u00edmica hep\u00e1tica, LDH es normal y se observa en el proteinograma una hipergammaglobulinemia policlonal (3,5 g/dL). Tiene antecedentes de infecci\u00f3n por VIH conocida desde hace 10 a\u00f1os y adherencia irregular al tratamiento antiretroviral, con determinaciones recientes de linfocitos CD4 350 cel/mL y carga viral de VIH de 154 copias/ml. Durante los \u00faltimos 3 meses ha estado tratada por poliartritis sim\u00e9trica seronegativa con 10-20 mg/d\u00eda de prednisona. \u00bfCu\u00e1l de las siguientes afirmaciones es correcta?", "full_answer": "Pregunta f\u00e1cil y bastante t\u00edpica. Nos hablan de una paciente VIH con seguimiento irregular del tratamiento antirretroviral, que presenta fiebre, hepatoesplenomegalia, pancitopenia y un una hipergammaglobulinemia policlonal. Todos estos datos son sugestivos de Leishmaniasis visceral, por lo que habr\u00eda que realizar una biopsia de MO para buscar amastigotes y confirmar el diagn\u00f3stico.", "type": "ENFERMEDADES INFECCIOSAS", "options": {"1": "Realizar\u00eda una biopsia de m\u00e9dula \u00f3sea, ya que el diagn\u00f3stico mas probables es una leishmaniasis visceral.", "2": "La pancitopenia se justifica por una cirrosis asociada al virus C y no realizar\u00eda mas pruebas.", "3": "Probablemente se trate de una toxicidad medular por prednisona que tratar\u00eda con retirada del f\u00e1rmaco y filgastrim.", "4": "Solicitar\u00eda una determinaci\u00f3n de ANAs para descartar lupus sist\u00e9mico diseminado.", "5": "Intensificar\u00eda el tratamiento antirretroviral, pues es probable que todas las manifestaciones se deban al VIH."}, "correct_option": 1, "explanations": {"1": {"exist": true, "char_ranges": [[224, 384]], "word_ranges": [[27, 52]], "text": "Todos estos datos son sugestivos de Leishmaniasis visceral, por lo que habr\u00eda que realizar una biopsia de MO para buscar amastigotes y confirmar el diagn\u00f3stico."}, "2": {"exist": false, "char_ranges": [], "word_ranges": [], "text": ""}, "3": {"exist": false, "char_ranges": [], "word_ranges": [], "text": ""}, "4": {"exist": false, "char_ranges": [], "word_ranges": [], "text": ""}, "5": {"exist": false, "char_ranges": [], "word_ranges": [], "text": ""}}} {"id": 53, "year": 2011, "question_id_specific": 146, "full_question": "Hombre de 31 a\u00f1os sin antecedentes psiquiatricos acude a urgencias refiriendo\u2026 s\u00edntomas de ansiedad)\u00a8. Hace dos d\u00edas experimento otro cuadro igual\u2026. Hace 1 semana ha roto su relaci\u00f3n de pareja\u2026 Descartada cualquier patolog\u00eda org\u00e1nica\u2026:", "full_answer": "Aqu\u00ed esta bien que te comenten sus antecedentes (todos tenemos antecedentes porque todos tenemos una vida) y que descarten patolog\u00eda org\u00e1nica. Eso es fundamental. No olvidemos que tambi\u00e9n los hipocondr\u00edacos tienen cuerpo. Eliminamos primero lo m\u00e1s llamativo trastorno \u00abagudo\u00bb de personalidad. La personalidad nos acompa\u00f1a toda la vida. No hay caracter\u00edsticas \u00abagudas\u00bb en nuestra personalidad. \u00bfTrastorno depresivo? Existe un criterio temporal que no se cumple, puede que este triste, pero triste no es depresi\u00f3n (1). \u00bfagorafobia? si fuese algo de eso nos habr\u00edan comentado el contexto \u00bfreacci\u00f3n de estres postraumatico? Esa es buena. El t\u00e9rmino correcto ser\u00eda trastorno de estr\u00e9s postraumatico o reacci\u00f3n a estres agudo, y seg\u00fan CIE 10: \u00abEl agente estresante puede ser una experiencia traum\u00e1tica devastadora que implica una amenaza seria a la seguridad o integridad f\u00edsica del enfermo o de persona o personas queridas (por ejemplo, cat\u00e1strofes naturales, accidentes, batallas, atracos, violaciones) o un cambio brusco y amenazador del rango o del entorno social del individuo (por ejemplo, p\u00e9rdidas de varios seres queridos, incendio de la vivienda, etc.)\u00bb o m\u00e1s brevemente: \u00abnaturaleza excepcionalmente amenazante o catastr\u00f3fica\u00bb Romper con la pareja entiendo que es duro, pero no excepcional. As\u00ed que por eliminaci\u00f3n 5. Crisis de angustia. Es una pregunta \u00fatil, habitual en la practica diaria, y as\u00ed vemos que hay que evitar dar un diagn\u00f3stico de larga evoluci\u00f3n por una reaccion natural. Los diagn\u00f3sticos del 1 al 4 ser\u00eda como llamar diab\u00e9tico al que tiene hiperglucemia tras comer tres magdalenas. Pregunta \u00fatil y realista.", "type": "PSIQUIATR\u00cdA", "options": {"1": "Reacci\u00f3n de estres postraumatico.", "2": "Trastorno \u00abagudo\u00bb de personalidad dependiente.", "3": "Trastorno depresivo.", "4": "Agorafobia.", "5": "Crisis de angustia."}, "correct_option": 5, "explanations": {"1": {"exist": true, "char_ranges": [[1233, 1296]], "word_ranges": [[180, 191]], "text": "Romper con la pareja entiendo que es duro, pero no excepcional."}, "2": {"exist": true, "char_ranges": [[222, 392]], "word_ranges": [[32, 55]], "text": "Eliminamos primero lo m\u00e1s llamativo trastorno \u00abagudo\u00bb de personalidad. La personalidad nos acompa\u00f1a toda la vida. No hay caracter\u00edsticas \u00abagudas\u00bb en nuestra personalidad."}, "3": {"exist": true, "char_ranges": [[415, 511]], "word_ranges": [[57, 74]], "text": "Existe un criterio temporal que no se cumple, puede que este triste, pero triste no es depresi\u00f3n"}, "4": {"exist": true, "char_ranges": [[530, 584]], "word_ranges": [[76, 86]], "text": "si fuese algo de eso nos habr\u00edan comentado el contexto"}, "5": {"exist": false, "char_ranges": [], "word_ranges": [], "text": ""}}} {"id": 48, "year": 2011, "question_id_specific": 158, "full_question": "Un ni\u00f1o de 10 a\u00f1os es tra\u00eddo a Urgencias porque desde hace 2 horas se le gira involuntariamente el cuello a la derecha asociado a marcado dolor cervical en cada giro. La abuela, posteriormente, nos dice que est\u00e1 vomitando desde ayer, por lo que le dio un jarabe; \u00bfcu\u00e1l ser\u00eda la actitud terap\u00e9utica m\u00e1s acertada?", "full_answer": "La respuesta correcta es la 1. El anticolin\u00e9rgico se puede administrar por v\u00eda endovenosa y tambi\u00e9n intramuscular si no me equivoco. La aparici\u00f3n de la diston\u00eda como efecto secundario es frecuente en ni\u00f1os y es uno de los principales motivos por los que los pediatras no solemos recetar algunos antiem\u00e9ticos en los ni\u00f1os.", "type": "PEDIATR\u00cdA", "options": {"1": "Inyectar un anticolin\u00e9rgico endovenoso.", "2": "No tratar hasta no realizar en los d\u00edas pr\u00f3ximos un electroencefalograma.", "3": "No tratar hasta no saber en d\u00edas pr\u00f3ximos el resultado del cultivo del l\u00edquido cefalorraqu\u00eddeo.", "4": "Llamar al psiquiatra de guardia.", "5": "Poner un antibi\u00f3tico intravenoso tras obtener muestra del exudado far\u00edngeo."}, "correct_option": 1, "explanations": {"1": {"exist": false, "char_ranges": [], "word_ranges": [], "text": ""}, "2": {"exist": false, "char_ranges": [], "word_ranges": [], "text": ""}, "3": {"exist": false, "char_ranges": [], "word_ranges": [], "text": ""}, "4": {"exist": false, "char_ranges": [], "word_ranges": [], "text": ""}, "5": {"exist": false, "char_ranges": [], "word_ranges": [], "text": ""}}} {"id": 490, "year": 2020, "question_id_specific": 106, "full_question": "Mujer de 65 a\u00f1os derivada a urgencias por fiebre y alteraciones en la anal\u00edtica: Hemoglobina 11,4 g/dL, leucocitos 0,86 x103/\u03bcL,(neutr\u00f3filos 41,9 %, linfocitos 55,8 %),plaquetas 48,0 x103/\u03bcL, fibrin\u00f3geno 118 mg/dL,d\u00edmero D 20,2 \u03bcg/mL. Se realiza examen de m\u00e9dula \u00f3sea con la que se le diagnostica una leucemia aguda con t(15;17) en el 60 % de lasc\u00e9lulas. \u00bfCu\u00e1l de las siguientes respuestas es correcta?:", "full_answer": "Se trata de una leucemia promieloc\u00edtica definitoria con la t(15;17), y como es bien sabido por todos vosotros el tratamiento es con tri\u00f3xido de ars\u00e9nico + ATRA. La opci\u00f3n 1 queda incompleto el tratamiento solo hay ATRA y hay que tratarla siempre que las condiciones del enfermo lo permitan, haya o no s\u00edntomas. La opci\u00f3n 3 hace 3 a\u00f1os ser\u00eda la opci\u00f3n correcta pero hoy en dia ni se trata con quimioterapia ni se pone heparina. Y la opci\u00f3n 4 es falsa porque esa fiebre se debe a la propia leucemia y lo prioritario es iniciar el tratamiento espec\u00edfico de la leucemia.", "type": "HEMATOLOGIA", "options": {"1": "Si est\u00e1 asintom\u00e1tica se iniciar\u00e1 \u00e1cido transretinoico (ATRA) y se recomendar\u00e1n controles en hospital de d\u00eda.", "2": "Se iniciar\u00e1 tratamiento con tri\u00f3xido de ars\u00e9nico, ATRA y terapia de soporte.", "3": "Es una leucemia mielobl\u00e1stica tipo M3, por lo que se iniciara tratamiento quimioter\u00e1pico y heparina para controlar la coagulaci\u00f3n intravascular diseminada.", "4": "Se debe iniciar tratamiento antibi\u00f3tico. Cuando desaparezca la fiebre se iniciar\u00e1 el tratamiento de la leucemia.", "5": NaN}, "correct_option": 2, "explanations": {"1": {"exist": true, "char_ranges": [[161, 310]], "word_ranges": [[27, 53]], "text": "La opci\u00f3n 1 queda incompleto el tratamiento solo hay ATRA y hay que tratarla siempre que las condiciones del enfermo lo permitan, haya o no s\u00edntomas."}, "2": {"exist": true, "char_ranges": [[0, 160]], "word_ranges": [[0, 27]], "text": "Se trata de una leucemia promieloc\u00edtica definitoria con la t(15;17), y como es bien sabido por todos vosotros el tratamiento es con tri\u00f3xido de ars\u00e9nico + ATRA."}, "3": {"exist": true, "char_ranges": [[311, 426]], "word_ranges": [[53, 76]], "text": "La opci\u00f3n 3 hace 3 a\u00f1os ser\u00eda la opci\u00f3n correcta pero hoy en dia ni se trata con quimioterapia ni se pone heparina."}, "4": {"exist": true, "char_ranges": [[429, 566]], "word_ranges": [[77, 102]], "text": "la opci\u00f3n 4 es falsa porque esa fiebre se debe a la propia leucemia y lo prioritario es iniciar el tratamiento espec\u00edfico de la leucemia."}, "5": {"exist": false, "char_ranges": [], "word_ranges": [], "text": ""}}} {"id": 416, "year": 2018, "question_id_specific": 76, "full_question": "Mujer de 78 a\u00f1os con demencia e institucionalizada es tra\u00edda por sus cuidadores por dolor abdominal importante con deterioro del estado general y distensi\u00f3n abdominal. Anal\u00edtica con leucocitosis, hematocrito elevado, insuficiencia renal y acidosis metab\u00f3lica. El ECG muestra fibrilaci\u00f3n auricular. La tomografia abdominal detecta asas de intestino delgado edematizadas, con acumatosis Intestinal y portal. El diagnostico mas probable es:", "full_answer": "Este es un cuadro cl\u00e1sico y el autor de la pregunta se ha preocupado de darnos las suficientes pistas para que no la fallemos. Al leer este enunciado se encienden m\u00faltiples alarmas: \u2013 Fibrilaci\u00f3n auricular: En el mir y en la puerta de urgencias, cualquier anciano con una FA y dolor abdominal es una isquemia mesent\u00e9rica de causa embol\u00edgena hasta que no se demuestre lo contrario. \u2013 Dolor abdominal agudo, SIRS, fracaso renal, acidosis metab\u00f3lica (isquemia!), probablemente un l\u00e1ctico elevado\u2026 \u2013 TAC: El hallazgo de gas portal ambi\u00e9n es altamente indicativo de una isquemia mesent\u00e9rica, tambi\u00e9n nos podr\u00edan haber hablado de neumatosis intestinal. \u00bfPor qu\u00e9 no son los otros cuadros? \u2013 Ulcus perforado: Nos habr\u00edan hablado de historia de consumo de AINES, o de una historia de dolor que mejora con la ingesta, de neumoperitoneo en Rx de t\u00f3rax. \u2013 \u00cdleo Biliar: Existir\u00edan antecedentes de c\u00f3licos biliares o de dolor en hipocondrio derecho, igualmente hablar\u00edan de aerobilia en prueba de imagen y dilataci\u00f3n de asas de delgado con imagen (o no) de litiasis biliar en ileon terminal. \u2013 Neo de sigma: En el mir nos hablar\u00edan de p\u00e9rdida de peso, cambio de h\u00e1bito deposicional, neumoperitoneo en Rx o TAC y exploraci\u00f3n abdominal periton\u00edtica pudiendo incluso palpar la masa.", "type": "CIRUG\u00cdA GENERAL", "options": {"1": "Perforaci\u00f3n de ulcus gastrico o duodenal.", "2": "lleo biliar.", "3": "Neoplasia obstructiva de sigma con perforaci\u00f3n.", "4": "Isquemia mesent\u00e9rica.", "5": NaN}, "correct_option": 4, "explanations": {"1": {"exist": true, "char_ranges": [[701, 841]], "word_ranges": [[113, 139]], "text": "Nos habr\u00edan hablado de historia de consumo de AINES, o de una historia de dolor que mejora con la ingesta, de neumoperitoneo en Rx de t\u00f3rax."}, "2": {"exist": true, "char_ranges": [[857, 1077]], "word_ranges": [[142, 177]], "text": "Existir\u00edan antecedentes de c\u00f3licos biliares o de dolor en hipocondrio derecho, igualmente hablar\u00edan de aerobilia en prueba de imagen y dilataci\u00f3n de asas de delgado con imagen (o no) de litiasis biliar en ileon terminal."}, "3": {"exist": true, "char_ranges": [[1094, 1265]], "word_ranges": [[181, 208]], "text": "En el mir nos hablar\u00edan de p\u00e9rdida de peso, cambio de h\u00e1bito deposicional, neumoperitoneo en Rx o TAC y exploraci\u00f3n abdominal periton\u00edtica pudiendo incluso palpar la masa."}, "4": {"exist": true, "char_ranges": [[219, 380]], "word_ranges": [[39, 66]], "text": "en la puerta de urgencias, cualquier anciano con una FA y dolor abdominal es una isquemia mesent\u00e9rica de causa embol\u00edgena hasta que no se demuestre lo contrario."}, "5": {"exist": false, "char_ranges": [], "word_ranges": [], "text": ""}}} {"id": 45, "year": 2011, "question_id_specific": 154, "full_question": "A un ni\u00f1o de 9 a\u00f1os, asintom\u00e1tico con soplo inocente, le realiza un ECG que documenta un S\u00edndrome de Wolf-Parkinson-White. \u00bfCu\u00e1l de las siguientes afirmaciones es FALSA?", "full_answer": "La respuesta correcta es la 1. Adem\u00e1s queda contestada con el enunciado que empieza diciendo que el ni\u00f1o est\u00e1 asintom\u00e1tico. El error me parece de concepto porque para el estudio de un soplo no se suele solicitar un ECG sin\u00f3 una ecocardiograf\u00eda. La insuficiencia card\u00edaca es una complicaci\u00f3n del s\u00edndrome de Wolf-Parkinson-White y raramente una forma de debut de la enfermedad.", "type": "PEDIATR\u00cdA", "options": {"1": "Necesita tratamiento de la insuficiencia card\u00edaca.", "2": "Puede presentar una taquicardia parox\u00edstica supraventricular.", "3": "La ecograf\u00eda 2D-Doppler descartar\u00e1 su asociaci\u00f3n con la enfermedad de Ebstein.", "4": "Es conveniente se le practique un ECG de 24 horas (Holter) y una ergometr\u00eda.", "5": "En determinados pacientes la ablaci\u00f3n de la v\u00eda accesoria con radiofrecuencia constituye la acci\u00f3n terap\u00e9utica."}, "correct_option": 1, "explanations": {"1": {"exist": true, "char_ranges": [[38, 123]], "word_ranges": [[7, 20]], "text": "queda contestada con el enunciado que empieza diciendo que el ni\u00f1o est\u00e1 asintom\u00e1tico."}, "2": {"exist": false, "char_ranges": [], "word_ranges": [], "text": ""}, "3": {"exist": false, "char_ranges": [], "word_ranges": [], "text": ""}, "4": {"exist": false, "char_ranges": [], "word_ranges": [], "text": ""}, "5": {"exist": false, "char_ranges": [], "word_ranges": [], "text": ""}}} {"id": 336, "year": 2016, "question_id_specific": 31, "full_question": "Mujer de 20 a\u00f1os con tumoraci\u00f3n ov\u00e1rica de 15 cm, s\u00f3lido-qu\u00edstica, detectada por ecograf\u00eda tras presentar s\u00edntomas abdominales inespect\u00edficos. En el estudio histopatol\u00f3gico de la pieza correspondiente se encuentran dientes, pelos, zonas de epitelio intestinal, \u00e1reas de epitelio escamoso (15%) y bronquial, as\u00ed como elementos neuroectod\u00e9rmicos y embrionarios en varias de las preparaciones histol\u00f3gicas. En referencia a este caso, se\u00f1ale el diagn\u00f3stico correcto:", "full_answer": "El teratoma qu\u00edstico maduro es un tumor benigno del ovario frecuente en la mujer adulta. Los tejidos que componen el tumor son bien diferenciados (maduros o de tipo adulto); aparte estructuras cut\u00e1neas se pueden observar muchos otros tejidos, particularmente en un engrosamiento o espol\u00f3n que hace eminencia hacia la cavidad, en el que con frecuencia hay dientes, cart\u00edlago y hueso.", "type": "GINECOLOG\u00cdA Y OBSTETRICIA", "options": {"1": "Teratocarcinoma.", "2": "Teratoma inmaduro.", "3": "Teratoma qu\u00edstico maduro.", "4": "Disgerminoma.", "5": NaN}, "correct_option": 3, "explanations": {"1": {"exist": false, "char_ranges": [], "word_ranges": [], "text": ""}, "2": {"exist": false, "char_ranges": [], "word_ranges": [], "text": ""}, "3": {"exist": true, "char_ranges": [[0, 382]], "word_ranges": [[0, 60]], "text": "El teratoma qu\u00edstico maduro es un tumor benigno del ovario frecuente en la mujer adulta. Los tejidos que componen el tumor son bien diferenciados (maduros o de tipo adulto); aparte estructuras cut\u00e1neas se pueden observar muchos otros tejidos, particularmente en un engrosamiento o espol\u00f3n que hace eminencia hacia la cavidad, en el que con frecuencia hay dientes, cart\u00edlago y hueso."}, "4": {"exist": false, "char_ranges": [], "word_ranges": [], "text": ""}, "5": {"exist": false, "char_ranges": [], "word_ranges": [], "text": ""}}} {"id": 609, "year": 2022, "question_id_specific": 115, "full_question": "Var\u00f3n de 27 a\u00f1os, deportista habitual, que refiere dolor en la pierna derecha tras la pr\u00e1ctica de carrera continua. Ha acudido en varias ocasiones a un fisioterapeuta, siendo diagnosticado de una sobrecarga en gemelos. Han transcurrido varios meses, no ha mejorado y refiere dolor intenso tras la actividad f\u00edsica que cede con el reposo en las siguientes horas del ejercicio. \u00bfQu\u00e9 prueba puede ayudar en el diagn\u00f3stico?:", "full_answer": "El s\u00edndrome compartimental de esfuerzo es una afecci\u00f3n de la pierna inducida por el ejercicio. Se caracteriza por la isquemia reversible de los m\u00fasculos de un compartimento muscular. El diagn\u00f3stico se realiza mediante la medici\u00f3n de las presiones compartimentales en reposo, durante el ejercicio y despu\u00e9s del mismo (respuesta 2 correcta). El tratamiento suele consistir en fasciotom\u00edas de los compartimentos afectados. Aunque la \u200b\u200bresonancia magn\u00e9tica no es muy \u00fatil para establecer el diagn\u00f3stico, puede ayudar en el diagn\u00f3stico diferencial.", "type": "TRAUMATOLOG\u00cdA", "options": {"1": "Tomograf\u00eda de emisi\u00f3n de positrones con 18 FDG.", "2": "Determinaci\u00f3n de la presi\u00f3n del compartimento posterior inmediatamente tras actividad.", "3": "Ecograf\u00eda-Doppler para descartar un trastorno circulatorio de la extremidad inferior.", "4": "Espectrometr\u00eda por resonancia magn\u00e9tica.", "5": NaN}, "correct_option": 2, "explanations": {"1": {"exist": false, "char_ranges": [], "word_ranges": [], "text": ""}, "2": {"exist": true, "char_ranges": [[0, 339]], "word_ranges": [[0, 51]], "text": "El s\u00edndrome compartimental de esfuerzo es una afecci\u00f3n de la pierna inducida por el ejercicio. Se caracteriza por la isquemia reversible de los m\u00fasculos de un compartimento muscular. El diagn\u00f3stico se realiza mediante la medici\u00f3n de las presiones compartimentales en reposo, durante el ejercicio y despu\u00e9s del mismo (respuesta 2 correcta)."}, "3": {"exist": false, "char_ranges": [], "word_ranges": [], "text": ""}, "4": {"exist": true, "char_ranges": [[420, 543]], "word_ranges": [[61, 79]], "text": "Aunque la \u200b\u200bresonancia magn\u00e9tica no es muy \u00fatil para establecer el diagn\u00f3stico, puede ayudar en el diagn\u00f3stico diferencial."}, "5": {"exist": false, "char_ranges": [], "word_ranges": [], "text": ""}}} {"id": 389, "year": 2016, "question_id_specific": 235, "full_question": "Lactante de 3 meses correctamente vacunado para su edad, que tras dos semanas de rinorrea, estornudos y tos, ingresa por intensificaci\u00f3n de los accesos de tos, con cianosis al final de los mismos, que finalizan con inspiraci\u00f3n profunda o gallo inspiratorio, precisando estimulaci\u00f3n, aspiraci\u00f3n de secreciones y ox\u00edgeno para recuperarse de los mismos. Seg\u00fan su sospecha diagn\u00f3stica y con respecto a la vacuna de dicha enfermedad, todas son verdaderas EXCEPTO:", "full_answer": "Nos est\u00e1n preguntando a cerca de la vacuna de la tos ferina. La opci\u00f3n 1 es correcta, es la vacuna difteria-t\u00e9tanos-pertussis. La opci\u00f3n 4 tambi\u00e9n lo es, ya que son microorganismos inactivados. Ahora ya nos quedar\u00eda la duda entre la 2 y la 3\u2026 La opci\u00f3n 3 es la que \u201cno es verdadera entre todas las que nos proponen\u201d: la inmunidad perdura unos 10a\u00f1os despu\u00e9s de la \u00faltima dosis.", "type": "PEDIATR\u00cdA", "options": {"1": "La vacuna en Espa\u00f1a se administra combinada con la antidift\u00e9rica y antitet\u00e1nica.", "2": "Se deben vacunar a los adultos que vayan a estar en contacto con lactantes menores de 6 meses.", "3": "La inmunidad tanto natural como vacunal, permanece de por vida.", "4": "La vacunaci\u00f3n est\u00e1 preparada a partir de microorganismos muertos.", "5": NaN}, "correct_option": 3, "explanations": {"1": {"exist": true, "char_ranges": [[0, 126]], "word_ranges": [[0, 21]], "text": "Nos est\u00e1n preguntando a cerca de la vacuna de la tos ferina. La opci\u00f3n 1 es correcta, es la vacuna difteria-t\u00e9tanos-pertussis."}, "2": {"exist": false, "char_ranges": [], "word_ranges": [], "text": ""}, "3": {"exist": true, "char_ranges": [[243, 377]], "word_ranges": [[44, 69]], "text": "La opci\u00f3n 3 es la que \u201cno es verdadera entre todas las que nos proponen\u201d: la inmunidad perdura unos 10a\u00f1os despu\u00e9s de la \u00faltima dosis."}, "4": {"exist": true, "char_ranges": [[127, 193]], "word_ranges": [[21, 32]], "text": "La opci\u00f3n 4 tambi\u00e9n lo es, ya que son microorganismos inactivados."}, "5": {"exist": false, "char_ranges": [], "word_ranges": [], "text": ""}}} {"id": 201, "year": 2013, "question_id_specific": 76, "full_question": "Ante un paciente que presenta problemas para la comprensi\u00f3n del lenguaje tanto hablado como escrito, incapacidad para denominar objetos y repetir palabras que se le dicen, habla flu\u00edda incomprensible con parafasias sem\u00e1nticas y fon\u00e9micas. Se trata de una:", "full_answer": "Las transcorticales tienen la repetici\u00f3n conservada. La de Broca tiene la comprensi\u00f3n conservada y ausencia de habla flu\u00edda. La afasia de Wernicke se caracteriza por una habla fluida (incluso en exceso) con parafasias y problemas para la comprensi\u00f3n, nominaci\u00f3n y repetici\u00f3n.", "type": "NEUROLOG\u00cdA", "options": {"1": "Afasia global.", "2": "Afasia de Wernicke.", "3": "Afasia de Broca.", "4": "Afasia transcortical sensitiva.", "5": "Afasia transcortical motora."}, "correct_option": 2, "explanations": {"1": {"exist": false, "char_ranges": [], "word_ranges": [], "text": ""}, "2": {"exist": true, "char_ranges": [[125, 275]], "word_ranges": [[18, 41]], "text": "La afasia de Wernicke se caracteriza por una habla fluida (incluso en exceso) con parafasias y problemas para la comprensi\u00f3n, nominaci\u00f3n y repetici\u00f3n."}, "3": {"exist": true, "char_ranges": [[53, 124]], "word_ranges": [[6, 18]], "text": "La de Broca tiene la comprensi\u00f3n conservada y ausencia de habla flu\u00edda."}, "4": {"exist": true, "char_ranges": [[0, 52]], "word_ranges": [[0, 6]], "text": "Las transcorticales tienen la repetici\u00f3n conservada."}, "5": {"exist": true, "char_ranges": [[0, 52]], "word_ranges": [[0, 6]], "text": "Las transcorticales tienen la repetici\u00f3n conservada."}}} {"id": 345, "year": 2016, "question_id_specific": 159, "full_question": "Mujer de 69 a\u00f1os que acude a su consulta refiriendo sangrado genital de varios meses de evoluci\u00f3n. Niega tratamiento hormonal sustitutivo y anticoagulaci\u00f3n. Aporta citolog\u00eda cervicovaginal normal. Exploraci\u00f3n f\u00edsica general y genital sin hallazgos de inter\u00e9s. IMC de 38kg/m2. Indique la actitud m\u00e1s correcta:", "full_answer": "Nos est\u00e1n presentando un caso de metrorragia postmenop\u00e1usica. Ante un caso como este debemos descartar una neoplasia endometrial, por lo que estar\u00eda indicado realizar una biopsia endometrial, bien mediante c\u00e1nula de Cornier si es posible, o bien mediante histeroscopia. La realizaci\u00f3n de biopsia cervical no es de inter\u00e9s en este caso dado que la citolog\u00eda es normal y la valoraci\u00f3n hormonal tampoco va a ser \u00fatil en el diagn\u00f3stico de una neoplasia endometrial. Por \u00faltimo, prescribir progesterona c\u00edclica no est\u00e1 indicado al ser una mujer postmenop\u00e1usica.", "type": "GINECOLOG\u00cdA Y OBSTETRICIA", "options": {"1": "Prescribir progesterona c\u00edclica.", "2": "Biopsia endometrial.", "3": "Biopsias de c\u00e9rvix al azar.", "4": "Valoraci\u00f3n hormonal con FSH, LH y estradiol.", "5": NaN}, "correct_option": 2, "explanations": {"1": {"exist": true, "char_ranges": [[474, 556]], "word_ranges": [[75, 86]], "text": "prescribir progesterona c\u00edclica no est\u00e1 indicado al ser una mujer postmenop\u00e1usica."}, "2": {"exist": true, "char_ranges": [[0, 190]], "word_ranges": [[0, 27]], "text": "Nos est\u00e1n presentando un caso de metrorragia postmenop\u00e1usica. Ante un caso como este debemos descartar una neoplasia endometrial, por lo que estar\u00eda indicado realizar una biopsia endometrial,"}, "3": {"exist": true, "char_ranges": [[270, 366]], "word_ranges": [[39, 57]], "text": "La realizaci\u00f3n de biopsia cervical no es de inter\u00e9s en este caso dado que la citolog\u00eda es normal"}, "4": {"exist": true, "char_ranges": [[369, 461]], "word_ranges": [[58, 73]], "text": "la valoraci\u00f3n hormonal tampoco va a ser \u00fatil en el diagn\u00f3stico de una neoplasia endometrial."}, "5": {"exist": false, "char_ranges": [], "word_ranges": [], "text": ""}}} {"id": 166, "year": 2013, "question_id_specific": 82, "full_question": "Mujer de 45 a\u00f1os que presenta hipertensi\u00f3n arterial (190/120 mmHg) acompa\u00f1ada de K 2,5 mEq/l. Se realiza ecograf\u00eda abdominal que muestra estenosis de ambas arterias renales. Indique que tratamiento est\u00e1 contraindicado:", "full_answer": "Si las arterias renales est\u00e1n estenosadas, la sangre tiene dificultades para llegar a los ri\u00f1ones. Es decir, ambos est\u00e1n poco perfundidos y s\u00f3lo pueden crear el gradiente de presi\u00f3n necesario para filtrar mediante la contracci\u00f3n de la arteriola eferente. Si administramos un IECA, provocaremos la relajaci\u00f3n de esta arteriola y la ca\u00edda de la tasa de filtrado glomerular. Y si ninguno de los dos ri\u00f1ones filtra, porque ambos tienen una estenosis arterial, tenemos un problema\u2026", "type": "CARDIOLOG\u00cdA Y CIRUG\u00cdA CARDIOVASCULAR", "options": {"1": "Enalapril.", "2": "Propanolol.", "3": "Amiloride.", "4": "Prazosin.", "5": "Amlodipino."}, "correct_option": 1, "explanations": {"1": {"exist": true, "char_ranges": [[255, 371]], "word_ranges": [[39, 58]], "text": "Si administramos un IECA, provocaremos la relajaci\u00f3n de esta arteriola y la ca\u00edda de la tasa de filtrado glomerular."}, "2": {"exist": false, "char_ranges": [], "word_ranges": [], "text": ""}, "3": {"exist": false, "char_ranges": [], "word_ranges": [], "text": ""}, "4": {"exist": false, "char_ranges": [], "word_ranges": [], "text": ""}, "5": {"exist": false, "char_ranges": [], "word_ranges": [], "text": ""}}} {"id": 481, "year": 2020, "question_id_specific": 171, "full_question": "Mujer de 67 a\u00f1os con antecedentes de dislipemia, que acude a urgencias por un cuadro de disuria y poliaquiuria seguido de fiebre, escalofr\u00edos y deterioro del estado general. A su llegada impresiona de gravedad y est\u00e1 taquic\u00e1rdica, taquipneica, con tensi\u00f3n arterial 60/40 mmHg y temperatura de 39\u00b0C. \u00bfQu\u00e9 medida de entre las siguientes NO estar\u00eda incluida en el manejo INICIAL?:", "full_answer": "Paciente que presenta un cuadro de fiebre+taquipnea+FC>100 = 3 criterios de SRIS (Sindrome Respuesta Inflamatoria Sist\u00e9mica). Recuerdo: la sepsis es un SIRS por infecci\u00f3n, que conlleva disminuci\u00f3n de las RVS, dando lugar a una disminuci\u00f3n de precarga y del volumen sist\u00f3lico llevando a hipotensi\u00f3n. Las primeras medidas terap\u00e9utica son la administraci\u00f3n de antibi\u00f3tico emp\u00edrico de amplio espectro y la reposici\u00f3n de fluidos (respuesta 4) para intentar compensar esa disminuci\u00f3n de la precarga. De elecci\u00f3n, cristaloides equilibrados y soluciones de alb\u00famina [1] Pero cuando esto es insuficiente, debemos intentar corregir la disminuci\u00f3n de las RVP mediante el uso de drogas vasoactivas [2]. Pero el vasoactivo de elecci\u00f3n en el contexto de shock s\u00e9ptico es la noradrenalina. ([3], [4]) (respuesta 1 FALSA) Es obligatoria la realizacion de un m\u00ednimo de dos Hemocultivos (respuesta 3), para el diagn\u00f3stico de bacteriemia y facilitando un tratamiento antibi\u00f3tico dirigido.[2] Por otro lado, es interesante determinar los niveles de \u00e1cido l\u00e1ctico en sangre, con dos fines: actualmente se considera el mejor marcador de hipoperfusi\u00f3n/hipoxia tisular y predice la respuesta al tratamiento.[2] (respuesta 2)", "type": "CUIDADOS CR\u00cdTICOS", "options": {"1": "Perfusi\u00f3n intravenosa de dobutamina.", "2": "Medici\u00f3n de lactato s\u00e9rico.", "3": "Extracci\u00f3n de hemocultivos.", "4": "Administraci\u00f3n de fluidos.", "5": NaN}, "correct_option": 1, "explanations": {"1": {"exist": true, "char_ranges": [[696, 805]], "word_ranges": [[102, 120]], "text": "el vasoactivo de elecci\u00f3n en el contexto de shock s\u00e9ptico es la noradrenalina. ([3], [4]) (respuesta 1 FALSA)"}, "2": {"exist": true, "char_ranges": [[973, 1200]], "word_ranges": [[143, 176]], "text": "Por otro lado, es interesante determinar los niveles de \u00e1cido l\u00e1ctico en sangre, con dos fines: actualmente se considera el mejor marcador de hipoperfusi\u00f3n/hipoxia tisular y predice la respuesta al tratamiento.[2] (respuesta 2)"}, "3": {"exist": true, "char_ranges": [[806, 969]], "word_ranges": [[120, 143]], "text": "Es obligatoria la realizacion de un m\u00ednimo de dos Hemocultivos (respuesta 3), para el diagn\u00f3stico de bacteriemia y facilitando un tratamiento antibi\u00f3tico dirigido.[2]"}, "4": {"exist": true, "char_ranges": [[136, 493]], "word_ranges": [[17, 72]], "text": "la sepsis es un SIRS por infecci\u00f3n, que conlleva disminuci\u00f3n de las RVS, dando lugar a una disminuci\u00f3n de precarga y del volumen sist\u00f3lico llevando a hipotensi\u00f3n. Las primeras medidas terap\u00e9utica son la administraci\u00f3n de antibi\u00f3tico emp\u00edrico de amplio espectro y la reposici\u00f3n de fluidos (respuesta 4) para intentar compensar esa disminuci\u00f3n de la precarga."}, "5": {"exist": false, "char_ranges": [], "word_ranges": [], "text": ""}}} {"id": 612, "year": 2022, "question_id_specific": 116, "full_question": "Mujer de 95 a\u00f1os que vive en una residencia, independiente para sus actividades b\u00e1sicas de la vida diaria, sale al jard\u00edn a pasear. Tiene antecedentes de HTA, dislipidemia, osteoporosis y deterioro cognitivo leve. Sufre una ca\u00edda al levantarse por la noche al ba\u00f1o. La radiograf\u00eda muestra una fractura subcapital desplazada de cadera derecha. \u00bfCu\u00e1l es el tratamiento recomendado?:", "full_answer": "Respuesta 1 correcta: Dado que se trata de una fractura subcapital desplazada de cadera el tratamiento quir\u00fargico que debemos plantear es la sustituci\u00f3n de la articulaci\u00f3n de la cadera dado que la lesi\u00f3n descrita presenta una alta probabilidad de necrosis de la cabeza femoral. Las opciones ser\u00edan una artroplastia total o parcial: en pacientes de edad avanzada se prefiere la parcial dado que es una cirug\u00eda m\u00e1s corta y menos agresiva que la total de cadera. Respuesta 2 incorrecta: Nos plantear\u00edamos esta t\u00e9cnica en fracturas basicervicales y lesiones del macizo trocant\u00e9rico, no en subcapitales. Respuesta 3 incorrecta: Nos plantear\u00edamos esta t\u00e9cnica en fracturas subcapitales no desplazadas de pacientes j\u00f3venes. Respuesta 4 incorrecta: Solo plantear\u00edamos un tratamiento no quir\u00fargico en un paciente con un estado general basal muy malo: pacientes encamados con escasa esperanza de vida.", "type": "TRAUMATOLOG\u00cdA", "options": {"1": "Hemiartroplastia de cadera.", "2": "Fijaci\u00f3n con clavo trocant\u00e9rico.", "3": "Fijaci\u00f3n con tornillos canulados.", "4": "Conservador: vida cama-sill\u00f3n.", "5": NaN}, "correct_option": 1, "explanations": {"1": {"exist": true, "char_ranges": [[0, 459]], "word_ranges": [[0, 76]], "text": "Respuesta 1 correcta: Dado que se trata de una fractura subcapital desplazada de cadera el tratamiento quir\u00fargico que debemos plantear es la sustituci\u00f3n de la articulaci\u00f3n de la cadera dado que la lesi\u00f3n descrita presenta una alta probabilidad de necrosis de la cabeza femoral. Las opciones ser\u00edan una artroplastia total o parcial: en pacientes de edad avanzada se prefiere la parcial dado que es una cirug\u00eda m\u00e1s corta y menos agresiva que la total de cadera."}, "2": {"exist": true, "char_ranges": [[460, 598]], "word_ranges": [[76, 94]], "text": "Respuesta 2 incorrecta: Nos plantear\u00edamos esta t\u00e9cnica en fracturas basicervicales y lesiones del macizo trocant\u00e9rico, no en subcapitales."}, "3": {"exist": true, "char_ranges": [[599, 716]], "word_ranges": [[94, 109]], "text": "Respuesta 3 incorrecta: Nos plantear\u00edamos esta t\u00e9cnica en fracturas subcapitales no desplazadas de pacientes j\u00f3venes."}, "4": {"exist": true, "char_ranges": [[717, 891]], "word_ranges": [[109, 135]], "text": "Respuesta 4 incorrecta: Solo plantear\u00edamos un tratamiento no quir\u00fargico en un paciente con un estado general basal muy malo: pacientes encamados con escasa esperanza de vida."}, "5": {"exist": false, "char_ranges": [], "word_ranges": [], "text": ""}}} {"id": 585, "year": 2022, "question_id_specific": 74, "full_question": "Primigesta de 34 semanas que presenta cifras tensionales de 165/95 y cefalea de dos d\u00edas de evoluci\u00f3n. Anal\u00edtica: hemoglobina 10,5 g/dL, plaquetas 98.000/mm3, AST 356 UI/L (0-31), ALT 234 UI/L (0-31), LDH 878 UI/L (125-243). Llaman del laboratorio por presencia de esquistocitos en el frotis de sangre perif\u00e9rica. Ha recibido la segunda dosis de corticoides para maduraci\u00f3n fetal hace 24 h. En la ecograf\u00eda obst\u00e9trica el peso fetal estimado est\u00e1 en el percentil 1 para la edad gestacional y el Doppler de arteria umbilical presenta ausencia del flujo telediast\u00f3lico. \u00bfCu\u00e1l de las siguientes es la actitud cl\u00ednica m\u00e1s indicada?:", "full_answer": "Nos est\u00e1n describiendo un Sd de HELLP. Con esas semanas de gestaci\u00f3n y con el feto reci\u00e9n madurado (ya tiene administrada las dos dosis de corticoides), la actitud es la finalizaci\u00f3n inmediata.", "type": "OBSTETRICIA Y GINECOLOG\u00cdA", "options": {"1": "Actitud expectante con antihipertensivos y sulfato de magnesio hasta la mejora de las plaquetas maternas.", "2": "Actitud expectante con tratamiento antihipertensivo a domicilio y controles cada 48 h.", "3": "Finalizaci\u00f3n de la gestaci\u00f3n cuando complete la maduraci\u00f3n pulmonar fetal.", "4": "Finalizaci\u00f3n inmediata de la gestaci\u00f3n.", "5": NaN}, "correct_option": 4, "explanations": {"1": {"exist": false, "char_ranges": [], "word_ranges": [], "text": ""}, "2": {"exist": false, "char_ranges": [], "word_ranges": [], "text": ""}, "3": {"exist": false, "char_ranges": [], "word_ranges": [], "text": ""}, "4": {"exist": true, "char_ranges": [[0, 193]], "word_ranges": [[0, 32]], "text": "Nos est\u00e1n describiendo un Sd de HELLP. Con esas semanas de gestaci\u00f3n y con el feto reci\u00e9n madurado (ya tiene administrada las dos dosis de corticoides), la actitud es la finalizaci\u00f3n inmediata."}, "5": {"exist": false, "char_ranges": [], "word_ranges": [], "text": ""}}} {"id": 3, "year": 2011, "question_id_specific": 37, "full_question": "Una mujer de 76 a\u00f1os sin otros antecedentes que hipertensi\u00f3n arterial, consulta por ictericia indolora y prurito con anorexia. Anal\u00edticamente destaca bilirrubina de 12 mg/dl (9,5 de directa). La ecograf\u00eda muestra dilataci\u00f3n biliar intra y extrahep\u00e1tica junto con un n\u00f3dulo hep\u00e1tico \u00fanico menor de 2 cm localizado perif\u00e9ricamente en la cara anterior de l\u00f3bulo izquierdo. La TC confirma estos hallazgos demostrando adem\u00e1s la presencia de masa pancre\u00e1tica de 3,5 cm. En la cabeza pancre\u00e1tica. La punci\u00f3n aspiraci\u00f3n del n\u00f3dulo hep\u00e1tico resulta concluyente para adenocarcinoma. Se\u00f1ale la mejor opci\u00f3n terap\u00e9utica:", "full_answer": "Caso que no me parece tan f\u00e1cil. Es un adenocarcinoma de p\u00e1ncreas con met\u00e1stasis, por lo cual el tratamiento debe ser paliativo con lo que quitamos las opciones 1 y 5. En principio es una paciente con moderadamente buen estado general con lo que lo ideal ser\u00eda hacer una derivaci\u00f3n biliar permanente y quimioterapia siendo la 4 la opci\u00f3n que considero m\u00e1s correcta.", "type": "DIGESTIVO", "options": {"1": "Quimio/radioterapia neoadyuvante, condicionando la opci\u00f3n de cirug\u00eda radical a la respuesta inicial.", "2": "Drenaje biliar externo percut\u00e1neo con car\u00e1cter paliativo con eventual reconversi\u00f3n a drenaje interno en caso de intolerancia o complicaciones.", "3": "Derivaci\u00f3n biliar quir\u00fargica paliativa con o sin gastroyeyunostom\u00eda profil\u00e1ctica en funci\u00f3n de los hallazgos intraoperatorios.", "4": "Pr\u00f3tesis biliar met\u00e1lica mediante colangiopancreatograf\u00eda retr\u00f3grada endosc\u00f3pica con opci\u00f3n a quimioterapia paliativa.", "5": "Duodenopancreatectom\u00eda cef\u00e1lica, con ablaci\u00f3n percut\u00e1nea mediante radiofrecuencia o alcoholizaci\u00f3n de la lesi\u00f3n hep\u00e1tica."}, "correct_option": 4, "explanations": {"1": {"exist": true, "char_ranges": [[33, 167]], "word_ranges": [[7, 31]], "text": "Es un adenocarcinoma de p\u00e1ncreas con met\u00e1stasis, por lo cual el tratamiento debe ser paliativo con lo que quitamos las opciones 1 y 5."}, "2": {"exist": false, "char_ranges": [], "word_ranges": [], "text": ""}, "3": {"exist": false, "char_ranges": [], "word_ranges": [], "text": ""}, "4": {"exist": true, "char_ranges": [[168, 365]], "word_ranges": [[31, 63]], "text": "En principio es una paciente con moderadamente buen estado general con lo que lo ideal ser\u00eda hacer una derivaci\u00f3n biliar permanente y quimioterapia siendo la 4 la opci\u00f3n que considero m\u00e1s correcta."}, "5": {"exist": true, "char_ranges": [[33, 167]], "word_ranges": [[7, 31]], "text": "Es un adenocarcinoma de p\u00e1ncreas con met\u00e1stasis, por lo cual el tratamiento debe ser paliativo con lo que quitamos las opciones 1 y 5."}}} {"id": 439, "year": 2018, "question_id_specific": 96, "full_question": "Mujer de 64 a\u00f1os. Menopausia a los 54 a\u00f1os. Dos embarazos y partos eut\u00f3cicos. No toma ninguna medicaci\u00f3n. Acude al servicio de urgencias refiriendo sangrado vaginal de dos d\u00edas de duraci\u00f3n con molestias en hipogastrio. La exploraci\u00f3n cl\u00ednica realizada por el ginec\u00f3logo no objetiva lesiones en genitales externos, vagina ni c\u00e9rvix. El tacto vaginal no es concluyente. \u00bfCual de las siguientes conductas le parece mas adecuada como primera aproximaci\u00f3n diagn\u00f3stica?", "full_answer": "Siendo la causa m\u00e1s frecuente de sangrado vaginal en mujer postmenop\u00e1usica la atrofia uro-genital, no se puede descartar la existencia de una patolog\u00eda endometrial. Seg\u00fan la SEGO: \u00abEl sangrado genital an\u00f3malo es el signo principal de sospecha que nos debe llevar a descartar una patolog\u00eda neopl\u00e1sica endometrial, en particular en pacientes postmenop\u00e1usicas, o con factores de riesgo asociados. (\u2026) Ante el signo de sospecha la realizaci\u00f3n de una ecograf\u00eda transvaginal, o en su defecto transrectal, nos permite: descartar patolog\u00eda org\u00e1nica (miomas, p\u00f3lipos); medici\u00f3n del grosor endometrial en un corte longitudinal. Se recomienda utilizar un punto de corte de 3mm para la realizaci\u00f3n de biopsia endometrial en la paciente sintom\u00e1tica\u00bb.", "type": "GINECOLOG\u00cdA Y OBSTETRICIA", "options": {"1": "Ecografia transvaginal y medici\u00f3n del espesor endometrial. En funci\u00f3n de \u00e9ste tomar biopsia endometrial o no.", "2": "Biopsia endometrial por aspiraci\u00f3n en consulta. Con ella puedo obtener el diagn\u00f3stico muy fiablemente y es econ\u00f3mica.", "3": "Histeroscopia con sedaci\u00f3n y biopsia dirigida si se ve alguna lesi\u00f3n.", "4": "Realizar\u00eda un microlegrado, ya que es la prueba que me proporcionar\u00eda el diagn\u00f3stico definitivo.", "5": NaN}, "correct_option": 1, "explanations": {"1": {"exist": true, "char_ranges": [[99, 617]], "word_ranges": [[14, 90]], "text": "no se puede descartar la existencia de una patolog\u00eda endometrial. Seg\u00fan la SEGO: \u00abEl sangrado genital an\u00f3malo es el signo principal de sospecha que nos debe llevar a descartar una patolog\u00eda neopl\u00e1sica endometrial, en particular en pacientes postmenop\u00e1usicas, o con factores de riesgo asociados. (\u2026) Ante el signo de sospecha la realizaci\u00f3n de una ecograf\u00eda transvaginal, o en su defecto transrectal, nos permite: descartar patolog\u00eda org\u00e1nica (miomas, p\u00f3lipos); medici\u00f3n del grosor endometrial en un corte longitudinal."}, "2": {"exist": false, "char_ranges": [], "word_ranges": [], "text": ""}, "3": {"exist": false, "char_ranges": [], "word_ranges": [], "text": ""}, "4": {"exist": false, "char_ranges": [], "word_ranges": [], "text": ""}, "5": {"exist": false, "char_ranges": [], "word_ranges": [], "text": ""}}} {"id": 192, "year": 2013, "question_id_specific": 160, "full_question": "Los padres de un lactante de 5 meses acuden porque su hijo, que asiste a guarder\u00eda, presenta desde hace 3 d\u00edas, coincidiendo con una infecci\u00f3n respiratoria de v\u00edas altas, heces l\u00edquidas en n\u00famero de 4 al d\u00eda, alg\u00fan v\u00f3mito alimentario ocasional y temperatura axilar de 38,3\u00baC. Es alimentado con lactancia mixta. A la exploraci\u00f3n el lactante tiene buen estado general, est\u00e1 bien nutrido e hidratado y su respiraci\u00f3n es eupneica; su peso es de 4.730g y ha descendido 70 g respecto al de la semana anterior. El abdomen est\u00e1n blando y depresible, sin masas ni megalias, y la fontanela, normotensa. Salvo la presencia de rinorrea acuosa, el resto de exploraci\u00f3n por \u00f3rganos y aparatos es normal. De las siguientes afirmaciones, se\u00f1ale la respuesta que considera CORRECTA:", "full_answer": "El beb\u00e9 no est\u00e1 deshidratado y tiene un buen estado general. Se trata de evitar la deshidrataci\u00f3n reponiendo lo que se va perdiendo, evitando en lo posible cambios dr\u00e1sticos en sus h\u00e1bitos alimentarios. El resto de respuestas toca muchos de los \u201cmitos\u201d y costumbres entorno a la gastroenteritis. Tampoco hay datos de intolerancia a lactosa (irritaci\u00f3n perianal, heces espumosas y \u00e1cidas, etc) que aconsejen una leche sin lactosa.", "type": "PEDIATR\u00cdA", "options": {"1": "Se debe realizar estimaci\u00f3n de las p\u00e9rdidas, recomendar un ayuno de 4 horas y rehidratar durante este tiempo con soluci\u00f3n rehidratante oral.", "2": "Es aconsejable la introducci\u00f3n de cereales de arroz por su efecto astringente.", "3": "Se debe realizar coprocultivo tan pronto como sea posible para excluir un origen bacteriano.", "4": "Se debe recomendar el empleo de f\u00f3rmula sin lactosa.", "5": "Se ha de aconsejar reponer las p\u00e9rdidas tras cada deposici\u00f3n, con soluci\u00f3n rehidratante oral, y continuar con su alimentaci\u00f3n habitual."}, "correct_option": 5, "explanations": {"1": {"exist": true, "char_ranges": [[203, 295]], "word_ranges": [[33, 48]], "text": "El resto de respuestas toca muchos de los \u201cmitos\u201d y costumbres entorno a la gastroenteritis."}, "2": {"exist": true, "char_ranges": [[203, 295]], "word_ranges": [[33, 48]], "text": "El resto de respuestas toca muchos de los \u201cmitos\u201d y costumbres entorno a la gastroenteritis."}, "3": {"exist": true, "char_ranges": [[203, 295]], "word_ranges": [[33, 48]], "text": "El resto de respuestas toca muchos de los \u201cmitos\u201d y costumbres entorno a la gastroenteritis."}, "4": {"exist": true, "char_ranges": [[296, 429]], "word_ranges": [[48, 68]], "text": "Tampoco hay datos de intolerancia a lactosa (irritaci\u00f3n perianal, heces espumosas y \u00e1cidas, etc) que aconsejen una leche sin lactosa."}, "5": {"exist": true, "char_ranges": [[0, 202]], "word_ranges": [[0, 33]], "text": "El beb\u00e9 no est\u00e1 deshidratado y tiene un buen estado general. Se trata de evitar la deshidrataci\u00f3n reponiendo lo que se va perdiendo, evitando en lo posible cambios dr\u00e1sticos en sus h\u00e1bitos alimentarios."}}} {"id": 261, "year": 2014, "question_id_specific": 100, "full_question": "Mujer de 63 a\u00f1os que acude al servicio de Urgencias refiriendo cefalea intensa con signos de irritaci\u00f3n men\u00edngea, alteraciones visuales bilaterales y oftalmoplejia. Se realiza una TAC que muestra lesi\u00f3n ocupante de espacio en silla turca de 2 cm compatible con adenoma hipofisario con signos de hemorragia intratumoral, con desviaci\u00f3n del tallo hipofisario y compresi\u00f3n del tejido glandular. Se\u00f1ale cu\u00e1l de la siguientes respuestas es INCORRECTA:", "full_answer": "Pregunta muy dif\u00edcil de apoplej\u00eda hipofisaria. Es un tema marginal en el MIR y adem\u00e1s la respuesta es compleja. Eliminando respuestas nos quedamos con intervenir (respuesta 4) o no intervenir (respuesta 2). En pacientes con inestabilidad hemodin\u00e1mica, disminuci\u00f3n de nivel de conciencia, disminuci\u00f3n de la agudeza visual y defectos extensos en el campo visual se recomienda realizar descompresi\u00f3n quir\u00fargica en la primera semana tras el inicio de los s\u00edntomas. Creo que la respuesta es la 4, aunque est\u00e1 formulada de forma rara. No s\u00e9 si el neurocirujano apoyar\u00e1 mi respuesta.", "type": "ENDOCRINOLOG\u00cdA", "options": {"1": "La sospecha diagn\u00f3stica es una apoplejia hipofisaria.", "2": "Se deber\u00eda iniciar tratamiento con corticoides a dosis altas y observar la evoluci\u00f3n, puesto que este tratamiento podr\u00eda reducir el volumen de la lesi\u00f3n y evitar la intervenci\u00f3n.", "3": "Debe plantearse el tratamiento con glucocorticoides para evitar una insuficiencia adrenal secundaria que comprometer\u00eda el pron\u00f3stico vital de la paciente.", "4": "La presencia de oftalmoplejia y los defectos visuales consituyen indicaciones para intervenir sin demora mediante descompresi\u00f3n quir\u00fargica urgente.", "5": "Tras la resoluci\u00f3n del cuadro agudo, es frecuente el desarrollo de panhipopituitarismo."}, "correct_option": 4, "explanations": {"1": {"exist": false, "char_ranges": [], "word_ranges": [], "text": ""}, "2": {"exist": false, "char_ranges": [], "word_ranges": [], "text": ""}, "3": {"exist": false, "char_ranges": [], "word_ranges": [], "text": ""}, "4": {"exist": true, "char_ranges": [[207, 460]], "word_ranges": [[32, 69]], "text": "En pacientes con inestabilidad hemodin\u00e1mica, disminuci\u00f3n de nivel de conciencia, disminuci\u00f3n de la agudeza visual y defectos extensos en el campo visual se recomienda realizar descompresi\u00f3n quir\u00fargica en la primera semana tras el inicio de los s\u00edntomas."}, "5": {"exist": false, "char_ranges": [], "word_ranges": [], "text": ""}}} {"id": 289, "year": 2016, "question_id_specific": 65, "full_question": "Se\u00f1ale de entre las siguientes la complicaci\u00f3n m\u00e1s probable que presenta un paciente intervenido, hace 20 a\u00f1os, de \u00falcera g\u00e1strica mediante antrecto\u00edn\u00eda y gastroyeyunostom\u00eda (Billroth II) que acude a su consulta refiriendo dolor abdominal postpradial, distensi\u00f3n abdominal, diarrea y datos anal\u00edticos de malabsorci\u00f3n de grasas y de vitamina B12:", "full_answer": "El enunciado ha descrito la cl\u00ednica de este s\u00edndrome. Suele ser una complicacion tard\u00eda (el paciente fue intervenido hace 20 a\u00f1os de un Bilroth II, recordemos que es una gastrectomia parcial, cierre del mu\u00f1\u00f3n gastrico y anastomosis de primera asa yeyunal al mu\u00f1\u00f3n), se acompa\u00f1a de dolor postprandial, distensi\u00f3n abdominal. Lo que ha ocurrido es una estenosis del asa aferente, por lo que las secreciones biliares y pancre\u00e1ticas no se mezclan con el bolo, se acumulan, producen dolor que se alivia con el v\u00f3mito, no es posible la absorci\u00f3n de las grasas (falta la lipasa pancreatica), por lo que hay diarreas, y si se asocia con sobrecrecimiento bacteriano, hay malabsorci\u00f3n de vitamina B12. Si se tratara de Adenocarcinoma g\u00e1strico, habr\u00edan hablado de s\u00edndrome constitucional, manifestaciones extradigestiva (n\u00f3dulo hermana Maria Jose, tumor de Krukenberg, n\u00f3dulo Virchow). No es dumping porque no habla de clinica vegetativa (palpitaciones, rubefacci\u00f3n facial, etc\u2026). La gastropat\u00eda por reflujo biliar es un diagnostico endosc\u00f3pico, ademas no se asocia con diarreas ni con malabsorci\u00f3n.", "type": "CIRUG\u00cdA GENERAL", "options": {"1": "Gastropat\u00eda por reflujo biliar.", "2": "Adenocarcinoma g\u00e1strico.", "3": "S\u00edndrome de evacuaci\u00f3n g\u00e1strica r\u00e1pida (dumping).", "4": "S\u00edndrome de asa aferente con sobrecrecimiento bacteriano.", "5": NaN}, "correct_option": 4, "explanations": {"1": {"exist": true, "char_ranges": [[969, 1087]], "word_ranges": [[148, 166]], "text": "La gastropat\u00eda por reflujo biliar es un diagnostico endosc\u00f3pico, ademas no se asocia con diarreas ni con malabsorci\u00f3n."}, "2": {"exist": true, "char_ranges": [[691, 873]], "word_ranges": [[113, 135]], "text": "Si se tratara de Adenocarcinoma g\u00e1strico, habr\u00edan hablado de s\u00edndrome constitucional, manifestaciones extradigestiva (n\u00f3dulo hermana Maria Jose, tumor de Krukenberg, n\u00f3dulo Virchow)."}, "3": {"exist": true, "char_ranges": [[874, 968]], "word_ranges": [[135, 148]], "text": "No es dumping porque no habla de clinica vegetativa (palpitaciones, rubefacci\u00f3n facial, etc\u2026)."}, "4": {"exist": true, "char_ranges": [[323, 690]], "word_ranges": [[50, 113]], "text": "Lo que ha ocurrido es una estenosis del asa aferente, por lo que las secreciones biliares y pancre\u00e1ticas no se mezclan con el bolo, se acumulan, producen dolor que se alivia con el v\u00f3mito, no es posible la absorci\u00f3n de las grasas (falta la lipasa pancreatica), por lo que hay diarreas, y si se asocia con sobrecrecimiento bacteriano, hay malabsorci\u00f3n de vitamina B12."}, "5": {"exist": false, "char_ranges": [], "word_ranges": [], "text": ""}}} {"id": 366, "year": 2016, "question_id_specific": 106, "full_question": "Un hombre de 32 a\u00f1os natural de Camer\u00fan consulta por fiebre tos y dolor en hemit\u00f3rax izquierdo de 1 mes de evoluci\u00f3n. Tom\u00f3 durante 1semana amoxicilina-clavul\u00e1nico sin mejor\u00eda de los s\u00edntomas. Se le realiza un an\u00e1lisis donde destaca una cifra de leucocitos de 8000/microL y una hemoglobina de 12,8 g/dL. En la radiograf\u00eda de t\u00f3rax se observa un derrame pleural izquierdo loculado que ocupa un tercio del hemit\u00f3rax. Una toracentesis muestra un l\u00edquido amarillento con las siguientes caracter\u00edsticas: hemat\u00edes 2000/\u00b5L, leucocitos 2500/\u00b5L con 90% de linfocitos, prote\u00ednas 4,9 g/dL, lactato deshidrogenasa 550 VIL, glucosa 67 mg/dL y ausencia de c\u00e9lulas malignas en el estudio citol\u00f3gico. \u00bfCu\u00e1l de las siguientes pruebas ser\u00eda m\u00e1s \u00fatil para diagnosticar la causa del derrame pleural?", "full_answer": "La sospecha es muy alta para derrame pleural tuberculoso, en paciente joven de una zona con alta tasa de TBC, un exudado linfocitario que apunta aTBC, linfoma o tumoral. Tumoral poco probable por edad, citolog\u00eda negativa. Linfoma podr\u00eda ser pero no hay evidencia de patolog\u00eda ganglionar asociada. La Adenosin deaminasa alta en LP permitir\u00eda apuntar con mucha seguridad a una TBC pleural aunque convendr\u00eda confirmar el diagnostico con una biopsia pleural dado que se puede presentar tambi\u00e9n en linfomas y empiemas de otro origen.", "type": "NEUMOLOG\u00cdA Y CIRUG\u00cdA TOR\u00c1CICA", "options": {"1": "Una tomograf\u00eda computarizada (TC) tor\u00e1cica.", "2": "Medici\u00f3n del pH del l\u00edquido pleural.", "3": "Medici\u00f3n de la adenos\u00edna desaminasa del l\u00edquido pleural.", "4": "Prueba de la tuberculina.", "5": NaN}, "correct_option": 3, "explanations": {"1": {"exist": false, "char_ranges": [], "word_ranges": [], "text": ""}, "2": {"exist": false, "char_ranges": [], "word_ranges": [], "text": ""}, "3": {"exist": true, "char_ranges": [[297, 528]], "word_ranges": [[47, 84]], "text": "La Adenosin deaminasa alta en LP permitir\u00eda apuntar con mucha seguridad a una TBC pleural aunque convendr\u00eda confirmar el diagnostico con una biopsia pleural dado que se puede presentar tambi\u00e9n en linfomas y empiemas de otro origen."}, "4": {"exist": false, "char_ranges": [], "word_ranges": [], "text": ""}, "5": {"exist": false, "char_ranges": [], "word_ranges": [], "text": ""}}} {"id": 544, "year": 2022, "question_id_specific": 40, "full_question": "Mujer de 16 a\u00f1os diagnosticada de rinoconjuntivitis al\u00e9rgica por sensibilizaci\u00f3n a p\u00f3lenes de gram\u00edneas desde los 10 a\u00f1os, que no controla totalmente la sintomatolog\u00eda en \u00e9poca de polinizaci\u00f3n con antihistam\u00ednicos orales y corticoides t\u00f3picos nasales. \u00bfQu\u00e9 opci\u00f3n terap\u00e9utica etiol\u00f3gica estar\u00eda indicada?:", "full_answer": "Actualmente el \u00fanico tratamiento eficaz y capaz de modificar el curso natural de la alergia respiratoria es la inmunoterapia espec\u00edfica con al\u00e9rgenos durante un m\u00ednimo de 3 a\u00f1os consecutivos. En este paciente el tratamiento convencional no ha funcionado por lo que se puede escalar a la inmunoterapia espec\u00edfica con al\u00e9rgenos. Si que es verdad que no es necesario esperar 10 a\u00f1os para iniciarla dado que est\u00e1 indicada y se permite su uso si existe una relevancia cl\u00ednica en mayores de 5 a\u00f1os. Omalizumab es un tratamiento biol\u00f3gico que solo estar\u00eda indicado en caso de asma bronquial moderada grave secundaria a alergia respiratoria.", "type": "ALERGOLOG\u00cdA", "options": {"1": "Inmunoterapia espec\u00edfica v\u00eda sublingual o v\u00eda subcut\u00e1nea durante un periodo m\u00ednimo de 3 a\u00f1os.", "2": "Inmunoterapia espec\u00edfica v\u00eda sublingual o v\u00eda subcut\u00e1nea durante un periodo m\u00e1ximo de 1 a\u00f1o.", "3": "Omalizumab v\u00eda subcut\u00e1nea cada 4 semanas durante un per\u00edodo m\u00e1ximo de 1 a\u00f1o.", "4": "Omalizumab v\u00eda subcut\u00e1nea cada 4 semanas durante un per\u00edodo m\u00ednimo de 6 meses.", "5": NaN}, "correct_option": 1, "explanations": {"1": {"exist": true, "char_ranges": [[0, 191]], "word_ranges": [[0, 29]], "text": "Actualmente el \u00fanico tratamiento eficaz y capaz de modificar el curso natural de la alergia respiratoria es la inmunoterapia espec\u00edfica con al\u00e9rgenos durante un m\u00ednimo de 3 a\u00f1os consecutivos."}, "2": {"exist": true, "char_ranges": [[0, 191]], "word_ranges": [[0, 29]], "text": "Actualmente el \u00fanico tratamiento eficaz y capaz de modificar el curso natural de la alergia respiratoria es la inmunoterapia espec\u00edfica con al\u00e9rgenos durante un m\u00ednimo de 3 a\u00f1os consecutivos."}, "3": {"exist": true, "char_ranges": [[493, 633]], "word_ranges": [[82, 102]], "text": "Omalizumab es un tratamiento biol\u00f3gico que solo estar\u00eda indicado en caso de asma bronquial moderada grave secundaria a alergia respiratoria."}, "4": {"exist": true, "char_ranges": [[493, 633]], "word_ranges": [[82, 102]], "text": "Omalizumab es un tratamiento biol\u00f3gico que solo estar\u00eda indicado en caso de asma bronquial moderada grave secundaria a alergia respiratoria."}, "5": {"exist": false, "char_ranges": [], "word_ranges": [], "text": ""}}} {"id": 533, "year": 2021, "question_id_specific": 179, "full_question": "Mujer de 32 a\u00f1os con s\u00edndrome de neoplasia endocrina m\u00faltiple de tipo 2A (MEN-2A) y portadora de una mutaci\u00f3n en RET. En una ecograf\u00eda de cuello se identifica un n\u00f3dulo hipoecog\u00e9nico de 6 mm con calcificaciones en su interior. Se decide tiroidectom\u00eda total y vaciamiento ganglionar cervical. En el estudio macrosc\u00f3pico se identifican un total de tres n\u00f3dulos, dos en el l\u00f3bulo derecho de 5 y 6 mm, y uno en el l\u00f3bulo izquierdo de 4 mm. En el estudio microsc\u00f3pico las tres lesiones est\u00e1n constituidas por una proliferaci\u00f3n uniforme de c\u00e9lulas redondeadas que se disponen con un patr\u00f3n s\u00f3lido y se acompa\u00f1an de calcificaciones y de dep\u00f3sitos de amiloide. Los n\u00facleos no son claros, ni muestran hendiduras, ni pseudoinclusiones. La tinci\u00f3n inmunohistoqu\u00edmica para sinaptofisina es positiva. En el vaciamiento ganglionar cervical se identifican met\u00e1stasis. \u00bfCu\u00e1l es el diagn\u00f3stico anatomopatol\u00f3gico de las lesiones identificadas en la tiroidectom\u00eda total?:", "full_answer": "RET t\u00edpico de medular de tiroides (c\u00e9lulas C, calcitonina elevada, heredado MEN-2\u00aa, t\u00edpica pregunta MIR).", "type": "ONCOLOG\u00cdA", "options": {"1": "Carcinoma medular multifocal.", "2": "Carcinoma papilar multifocal.", "3": "Carcinoma folicular.", "4": "Hiperplasia de c\u00e9lulas parafoliculares.", "5": NaN}, "correct_option": 1, "explanations": {"1": {"exist": true, "char_ranges": [[0, 105]], "word_ranges": [[0, 15]], "text": "RET t\u00edpico de medular de tiroides (c\u00e9lulas C, calcitonina elevada, heredado MEN-2\u00aa, t\u00edpica pregunta MIR)."}, "2": {"exist": false, "char_ranges": [], "word_ranges": [], "text": ""}, "3": {"exist": false, "char_ranges": [], "word_ranges": [], "text": ""}, "4": {"exist": false, "char_ranges": [], "word_ranges": [], "text": ""}, "5": {"exist": false, "char_ranges": [], "word_ranges": [], "text": ""}}} {"id": 80, "year": 2012, "question_id_specific": 37, "full_question": "Hombre de 52 a\u00f1os que consulta por una segunda opini\u00f3n sobre la necesidad de realizarse biopsia hep\u00e1tica para estudio de hipertransaminasemia detectada desde hace dos a\u00f1os en an\u00e1lisis rutinarios de empresa. Entre sus antecedentes familiares destaca el fallecimiento de su padre de una cirrosis epatica de etiolog\u00edaa no filiada. Asintom\u00e1tico y realizando vida social y laboral sin l\u00edmitaciones. Niega consumo de alcohol. En la exploraci\u00f3n f\u00edsica destaca pigmentaci\u00f3n met\u00e1lica de piel y m\u00ednima hepatomegalia no dolorosa. Resto de la exploraci\u00f3n f\u00edsica normal, \u00edndice de masa corporal 23. Aporta anal\u00edtica con los siguientes resultados: billirrubina, alb\u00famina, transaminasas AST y ALT, hemograma y tiempo de protrombina normales: glucemia 150 mg/dl; ferritina s\u00e9rica 950; saturaci\u00f3n de trasferrina >45%.Estudio de virus hepatotropos negativo. Ecograf\u00eda abdominal normal. Le han realizado estudio gen\u00e9tico HFE siendo homocigoto para la mutaci\u00f3n C282Y. \u00bfCu\u00e1l ser\u00eda la recomendaci\u00f3n m\u00e1s acertada con a informaci\u00f3n disponible?:", "full_answer": "Es un paciente que ya tiene diagn\u00f3stico de hemocromatosis, por estudio gen\u00e9tico positivo y datos de sobrecarga f\u00e9rrica (IST >45% y ferritina elevada). La biopsia nos servir\u00eda si el diagn\u00f3stico es incierto o si quisieramos valorar afectaci\u00f3n hep\u00e1tica que no se sospecha con esa anal\u00edtica. El tratamiento de elecci\u00f3n son las flebotom\u00edas.", "type": "APARATO DIGESTIVO", "options": {"1": "Realizar biopsia hep\u00e1tica.", "2": "Realizar resonancia magn\u00e9tica hep\u00e1tica.", "3": "Iniciar tratamiento con flebotom\u00edas.", "4": "Iniciar tratamiento con desferroxamina.", "5": "Iniciar tratamiento con Vitamina E."}, "correct_option": 3, "explanations": {"1": {"exist": true, "char_ranges": [[151, 287]], "word_ranges": [[23, 45]], "text": "La biopsia nos servir\u00eda si el diagn\u00f3stico es incierto o si quisieramos valorar afectaci\u00f3n hep\u00e1tica que no se sospecha con esa anal\u00edtica."}, "2": {"exist": false, "char_ranges": [], "word_ranges": [], "text": ""}, "3": {"exist": false, "char_ranges": [], "word_ranges": [], "text": ""}, "4": {"exist": false, "char_ranges": [], "word_ranges": [], "text": ""}, "5": {"exist": false, "char_ranges": [], "word_ranges": [], "text": ""}}} {"id": 394, "year": 2016, "question_id_specific": 225, "full_question": "Mariano es un se\u00f1or de 53 a\u00f1os de edad que acude a su consulta refiriendo sentir un gran malestar desde hace ya unos a\u00f1os. Relaciona el malestar con una conducta que encuentra absurda pero que es incapaz de evitar. Esta conducta que lleva a cabo siempre al llegar a su casa de vuelta del trabajo consiste en accionar el pomo del ba\u00f1o del piso superior de la vivienda antes de hacer cualquier otra cosa, incluso antes de saludar a su familia. Algunas veces ha intentado resistirse a hacerlo pero solo ha conseguido angustiarse y demorar la conducta unos minutos. \u00c9ste comportamiento que Mariano realiza de un modo incoercible y autom\u00e1tico es lo que se denomina:", "full_answer": "En esta pregunta piden el nombre del s\u00edntoma, no de la enfermedad, as\u00ed que por ello la respuesta correcta es compulsi\u00f3n. Seg\u00fan el DSM-5, una compulsi\u00f3n es un comportamiento (p.ej: lavarse las manos, ordenar, comprobar cosas) o acto mental (p.ej: rezar, contar, repetir palabras en silencio) repetitivo que el sujeto realiza como respuesta a una obsesi\u00f3n o de acuerdo con reglas que ha de aplicar de forma r\u00edgida. El objetivo de los comportamientos o actos mentales es prevenir o disminuir la ansiedad o malestar, o evitar alg\u00fan suceso o situaci\u00f3n temida; sin embargo, estos comportamientos o actos mentales no est\u00e1n conectados de una manera realista con los destinados a neutralizar o prevenir o bien resultan claramente excesivos.", "type": "PSIQUIATR\u00cdA", "options": {"1": "Trastorno obsesivo-compulsivo.", "2": "Obsesi\u00f3n.", "3": "Impulsi\u00f3n.", "4": "Compulsi\u00f3n.", "5": NaN}, "correct_option": 4, "explanations": {"1": {"exist": false, "char_ranges": [], "word_ranges": [], "text": ""}, "2": {"exist": false, "char_ranges": [], "word_ranges": [], "text": ""}, "3": {"exist": false, "char_ranges": [], "word_ranges": [], "text": ""}, "4": {"exist": true, "char_ranges": [[121, 412]], "word_ranges": [[21, 68]], "text": "Seg\u00fan el DSM-5, una compulsi\u00f3n es un comportamiento (p.ej: lavarse las manos, ordenar, comprobar cosas) o acto mental (p.ej: rezar, contar, repetir palabras en silencio) repetitivo que el sujeto realiza como respuesta a una obsesi\u00f3n o de acuerdo con reglas que ha de aplicar de forma r\u00edgida."}, "5": {"exist": false, "char_ranges": [], "word_ranges": [], "text": ""}}} {"id": 229, "year": 2014, "question_id_specific": 178, "full_question": "Lactante de 10 meses previamente sano con gastroenteritis aguda de un d\u00eda de evoluci\u00f3n y signos de deshidrataci\u00f3n leve, sin sangre ni moco en las heces y sin tolerancia oral. \u00bfCu\u00e1l es el tratamiento de elecci\u00f3n inicial en nuestro medio?", "full_answer": "El lactante presenta una GEA aguda sin signos de alarma ni de deshidrataci\u00f3n grave. El primer paso ser\u00eda intentar tolerancia oral con suero de rehidrataci\u00f3n oral y continuar con su alimentaci\u00f3n normal si buena tolerancia. La respuesta 1 puede dar lugar a dudas, por el hecho de que en el enunciado comentan que el paciente \u201cno tolera a la v\u00eda oral\u201d Podr\u00eda en ese caso iniciarse rehidrataci\u00f3n intravenosa\u2026 pero no ser\u00eda conveniente dejar en dieta absoluta 8 horas ni tampoco una dieta astringente posterior, por lo que yo me decanto por la 2. El resto de tratamientos, no aportan ning\u00fan beneficio.", "type": "PEDIATR\u00cdA", "options": {"1": "Rehidrataci\u00f3n intravenosa, dieta absoluta 8 horas y comenzar alimentaci\u00f3n con dieta astringente.", "2": "Rehidrataci\u00f3n con soluci\u00f3n de rehidrataci\u00f3n oral de baja osmolalidad (sodio 60-75 mEq/L) y continuar con su alimentaci\u00f3n habitual.", "3": "Rehidrataci\u00f3n con soluci\u00f3n de rehidrataci\u00f3n oral de baja osmolalidad (sodio 60-75 mEq/L) mantener alimentaci\u00f3n habitual y amoxicilina oral 10 d\u00edas.", "4": "Rehidrataci\u00f3n con soluci\u00f3n de rehidrataci\u00f3n oral de baja osmolalidad (sodio 60-75 mEq/L) y comenzar alimentaci\u00f3n con f\u00f3rmulas sin lactosa.", "5": "Rehidrataci\u00f3n con soluci\u00f3n de rehidrataci\u00f3n oral de baja osmolalidad (sodio 60-75 mEq/L) mantener alimentaci\u00f3n habitual y loperamida 7 d\u00edas."}, "correct_option": 2, "explanations": {"1": {"exist": true, "char_ranges": [[84, 541]], "word_ranges": [[14, 93]], "text": "El primer paso ser\u00eda intentar tolerancia oral con suero de rehidrataci\u00f3n oral y continuar con su alimentaci\u00f3n normal si buena tolerancia. La respuesta 1 puede dar lugar a dudas, por el hecho de que en el enunciado comentan que el paciente \u201cno tolera a la v\u00eda oral\u201d Podr\u00eda en ese caso iniciarse rehidrataci\u00f3n intravenosa\u2026 pero no ser\u00eda conveniente dejar en dieta absoluta 8 horas ni tampoco una dieta astringente posterior, por lo que yo me decanto por la 2."}, "2": {"exist": true, "char_ranges": [[84, 541]], "word_ranges": [[14, 93]], "text": "El primer paso ser\u00eda intentar tolerancia oral con suero de rehidrataci\u00f3n oral y continuar con su alimentaci\u00f3n normal si buena tolerancia. La respuesta 1 puede dar lugar a dudas, por el hecho de que en el enunciado comentan que el paciente \u201cno tolera a la v\u00eda oral\u201d Podr\u00eda en ese caso iniciarse rehidrataci\u00f3n intravenosa\u2026 pero no ser\u00eda conveniente dejar en dieta absoluta 8 horas ni tampoco una dieta astringente posterior, por lo que yo me decanto por la 2."}, "3": {"exist": true, "char_ranges": [[542, 596]], "word_ranges": [[93, 101]], "text": "El resto de tratamientos, no aportan ning\u00fan beneficio."}, "4": {"exist": true, "char_ranges": [[542, 596]], "word_ranges": [[93, 101]], "text": "El resto de tratamientos, no aportan ning\u00fan beneficio."}, "5": {"exist": true, "char_ranges": [[542, 596]], "word_ranges": [[93, 101]], "text": "El resto de tratamientos, no aportan ning\u00fan beneficio."}}} {"id": 198, "year": 2013, "question_id_specific": 73, "full_question": "Un hombre de 77 a\u00f1os con antecedentes de Diabetes Mellitus tipo 2, fue diagnosticado hace 2 a\u00f1os de deterioro cognitivo leve. En aquel momento se practic\u00f3 un Minimental (Folstein) 28/30, Test de Yesavage abreviado 14/15, una anal\u00edtica con TSH y factores de maduraci\u00f3n normales junto con serolog\u00edas de LUES y VIH negativas y una resonancia magn\u00e9tica (RM) que mostraba atrofia cortical difusa predominante en zona posterior. El paciente acude acompa\u00f1ado de su esposa que refiere evoluci\u00f3n progresiva y lenta del deterioro de la memoria (p.e. en ocasiones se ha confundido con las dosis de insulina). No existen alucinaciones ni trastorno conductual. El Minimental actual es de 24/30 y el Test de Yesavage 14/15, no existe focalidad neurol\u00f3gica, temblor ni trastornos del tono o de la marcha a la exploraci\u00f3n f\u00edsica. Hace 2 meses acudi\u00f3 a Urgencias tras un traumatismo craneo\u00adencef\u00e1lico leve por caida accidental (la \u00fanica en 2 a\u00f1os) y se realiz\u00f3 una tomograf\u00eda que no aportaba nueva informaci\u00f3n respecto a la RM previa. \u00bfCu\u00e1l es el diagn\u00f3stico m\u00e1s probable en este paciente?", "full_answer": "Este caso describe una Enfermedad de Alzheimer con comienzo insidioso de problemas de memoria a corto plazo. Cl\u00ednicamente se puede distinguir de otras opciones por ausencia de problemas conductuales (t\u00edpicos de la Demencia frontotemporal) y alucinaciones (precoces en la Enfermedad por cuerpos de Lewy). La demencia vascular se diagnostica por criterios que incluyen d\u00e9ficit de memoria y en otras dos \u00e1reas cognitivas con deterioro funcional consiguiente, pero enfermedad vascular cerebral seg\u00fan datos de la historia, de la exploraci\u00f3n y/o de las pruebas de neuroimagen, las cuales no se indican en el caso. La enfermedad de Creutzfeldt-Jacob es sobretodo de r\u00e1pida evoluci\u00f3n y suele presentarse con otra cl\u00ednica, exploraci\u00f3n neurol\u00f3gica anormal y alteraciones en la RM.", "type": "NEUROLOG\u00cdA", "options": {"1": "Demencia frontotemporal.", "2": "Enfermedad de Alzheimer.", "3": "Demencia vascular.", "4": "Enfermedad por cuerpos de Lewy.", "5": "Enfermedad de Creutzfeldt-Jakob."}, "correct_option": 2, "explanations": {"1": {"exist": true, "char_ranges": [[0, 303]], "word_ranges": [[0, 44]], "text": "Este caso describe una Enfermedad de Alzheimer con comienzo insidioso de problemas de memoria a corto plazo. Cl\u00ednicamente se puede distinguir de otras opciones por ausencia de problemas conductuales (t\u00edpicos de la Demencia frontotemporal) y alucinaciones (precoces en la Enfermedad por cuerpos de Lewy)."}, "2": {"exist": true, "char_ranges": [[0, 303]], "word_ranges": [[0, 44]], "text": "Este caso describe una Enfermedad de Alzheimer con comienzo insidioso de problemas de memoria a corto plazo. Cl\u00ednicamente se puede distinguir de otras opciones por ausencia de problemas conductuales (t\u00edpicos de la Demencia frontotemporal) y alucinaciones (precoces en la Enfermedad por cuerpos de Lewy)."}, "3": {"exist": true, "char_ranges": [[304, 454]], "word_ranges": [[44, 66]], "text": "La demencia vascular se diagnostica por criterios que incluyen d\u00e9ficit de memoria y en otras dos \u00e1reas cognitivas con deterioro funcional consiguiente,"}, "4": {"exist": true, "char_ranges": [[0, 303]], "word_ranges": [[0, 44]], "text": "Este caso describe una Enfermedad de Alzheimer con comienzo insidioso de problemas de memoria a corto plazo. Cl\u00ednicamente se puede distinguir de otras opciones por ausencia de problemas conductuales (t\u00edpicos de la Demencia frontotemporal) y alucinaciones (precoces en la Enfermedad por cuerpos de Lewy)."}, "5": {"exist": true, "char_ranges": [[608, 770]], "word_ranges": [[92, 115]], "text": "La enfermedad de Creutzfeldt-Jacob es sobretodo de r\u00e1pida evoluci\u00f3n y suele presentarse con otra cl\u00ednica, exploraci\u00f3n neurol\u00f3gica anormal y alteraciones en la RM."}}} {"id": 351, "year": 2016, "question_id_specific": 161, "full_question": "Mujer de 27 a\u00f1os remitida a consulta de ginecolog\u00eda para su valoraci\u00f3n refiriendo dispareunia desde hace unos 8 meses, junto con disquecia y rectorragia ocasional coincidiendo con la menstruaci\u00f3n desde hace 3-4 meses. Tambi\u00e9n refiere dismenorrea desde hace a\u00f1os que controla bien con ibuprofeno. Lleva intentando quedarse embarazada unos 16 meses sin haberlo conseguido a\u00fan. En la exploraci\u00f3n ginecol\u00f3gica tan s\u00f3lo se aprecia dolor al presionar fondo de saco vaginal posterior. \u00bfQu\u00e9 prueba considera Ud que le permitir\u00eda llegar al diagn\u00f3stico de certeza de su patolog\u00eda?", "full_answer": "El caso cl\u00ednico que presentan es una mujer en edad f\u00e9rtil (27 a\u00f1os) que presenta dispareunia + dismenorrea + esterilidad, lo cual debe hacernos pensar en una endometriosis. La historia cl\u00ednica y la ecograf\u00eda dan un diagn\u00f3stico de sospecha, pero el diagn\u00f3stico de certeza lo da la observaci\u00f3n directa (con biopsia asociada o no) de implantes de tejido endometrial ect\u00f3pico, bien por laparotom\u00eda o por laparoscopia.", "type": "GINECOLOG\u00cdA Y OBSTETRICIA", "options": {"1": "Ecograf\u00eda transvaginal.", "2": "Laparoscopia diagn\u00f3stica.", "3": "Resonancia magn\u00e9tica.", "4": "Colonoscopia.", "5": NaN}, "correct_option": 2, "explanations": {"1": {"exist": false, "char_ranges": [], "word_ranges": [], "text": ""}, "2": {"exist": true, "char_ranges": [[173, 413]], "word_ranges": [[28, 66]], "text": "La historia cl\u00ednica y la ecograf\u00eda dan un diagn\u00f3stico de sospecha, pero el diagn\u00f3stico de certeza lo da la observaci\u00f3n directa (con biopsia asociada o no) de implantes de tejido endometrial ect\u00f3pico, bien por laparotom\u00eda o por laparoscopia."}, "3": {"exist": false, "char_ranges": [], "word_ranges": [], "text": ""}, "4": {"exist": false, "char_ranges": [], "word_ranges": [], "text": ""}, "5": {"exist": false, "char_ranges": [], "word_ranges": [], "text": ""}}} {"id": 69, "year": 2012, "question_id_specific": 124, "full_question": "La hija de una anciana de 82 a\u00f1os nos consulta porque ha encontrado a su madre m\u00e1s confusa de lo normal. La paciente est\u00e1 diagnosticada de enfermedad de Alzheimer en grado moderado. Tiene adem\u00e1s FA, depresi\u00f3n y artrosis. Su tratamiento estable desde hace 3 a\u00f1os, incluye digoxina, acenocumarol, fluoxetina desde hace 4 meses e ibuprofeno desde hace un mes por dolores articulares. A la exploraci\u00f3n, su tensi\u00f3n es de 130/80, frecuencia cardiaca de 48 lpm y respiratoria de 10 rpm. \u00bfCu\u00e1l de los siguientes es la causa m\u00e1s probable del aumento de confusi\u00f3n?", "full_answer": "La causa m\u00e1s probable es una intoxicaci\u00f3n digit\u00e1lica, que producir\u00eda ese enlentecimiento de la conducci\u00f3n, que se ve en la frecuencia card\u00edaca. Probablemente secundario a una insuficiencia renal por toxicidad con ibuprofeno (pero esto ya es rizar el rizo).", "type": "ANESTESIOLOG\u00cdA, CUIDADOS CR\u00cdTICOS Y URGENCIAS", "options": {"1": "Progresi\u00f3n del Alzheimer.", "2": "Empeoramiento de la depresi\u00f3n.", "3": "Intoxicaci\u00f3n digit\u00e1lica.", "4": "Demencia por cuerpos de Lewy.", "5": "Toxicidad por ibuprofeno."}, "correct_option": 3, "explanations": {"1": {"exist": false, "char_ranges": [], "word_ranges": [], "text": ""}, "2": {"exist": false, "char_ranges": [], "word_ranges": [], "text": ""}, "3": {"exist": true, "char_ranges": [[0, 143]], "word_ranges": [[0, 22]], "text": "La causa m\u00e1s probable es una intoxicaci\u00f3n digit\u00e1lica, que producir\u00eda ese enlentecimiento de la conducci\u00f3n, que se ve en la frecuencia card\u00edaca."}, "4": {"exist": false, "char_ranges": [], "word_ranges": [], "text": ""}, "5": {"exist": false, "char_ranges": [], "word_ranges": [], "text": ""}}} {"id": 140, "year": 2012, "question_id_specific": 134, "full_question": "Un reci\u00e9n nacido de 37 semanas de edad gestacional, sin hallazgos patol\u00f3gicos en la ecograf\u00eda prenatal, presenta en la exploraci\u00f3n realizada en el paritorio un \u201cstop\u201d al paso de la sonda nasog\u00e1strica. La radiograf\u00eda de t\u00f3rax y abdomen muestra un bols\u00f3n esof\u00e1gico atr\u00e9sico, con una neumatizaci\u00f3n gastrointestinal normal. Tras evaluaci\u00f3n diagn\u00f3stica que descarta otras anomal\u00edas y encontr\u00e1ndose en situaci\u00f3n respiratoria estable, se decide intervenci\u00f3n quir\u00fargica. \u00bfEn qu\u00e9 se basa la prioridad de intervenir este paciente?", "full_answer": "La respuesta correcta es la 3. El hecho de que haya aireaci\u00f3n intestinal orienta a la existencia de una f\u00edstula del segmento inferior esof\u00e1gico con la tr\u00e1quea o con un bronquio. En este caso las complicaciones pulmonares son frecuentes por el paso del contenido g\u00e1strico a la v\u00eda respiratoria.", "type": "PEDIATR\u00cdA", "options": {"1": "Por la imposibilidad de deglutir saliva.", "2": "Por las malformaciones cardiacas frecuentemente asociadas.", "3": "Por el riesgo de neumonitis por aspiraci\u00f3n.", "4": "Por la imposibilidad de alimentaci\u00f3n enteral.", "5": "Por la traqueomalacia asociada."}, "correct_option": 3, "explanations": {"1": {"exist": false, "char_ranges": [], "word_ranges": [], "text": ""}, "2": {"exist": false, "char_ranges": [], "word_ranges": [], "text": ""}, "3": {"exist": true, "char_ranges": [[31, 293]], "word_ranges": [[6, 49]], "text": "El hecho de que haya aireaci\u00f3n intestinal orienta a la existencia de una f\u00edstula del segmento inferior esof\u00e1gico con la tr\u00e1quea o con un bronquio. En este caso las complicaciones pulmonares son frecuentes por el paso del contenido g\u00e1strico a la v\u00eda respiratoria."}, "4": {"exist": false, "char_ranges": [], "word_ranges": [], "text": ""}, "5": {"exist": false, "char_ranges": [], "word_ranges": [], "text": ""}}} {"id": 475, "year": 2020, "question_id_specific": 89, "full_question": "Un hombre de 46 a\u00f1os con trastorno bipolar es llevado a urgencias tras una sobreingesta de carbonato de litio. En la exploraci\u00f3n destaca temblor intenso, ataxia, disartria, mioclon\u00edas y fasciculaciones. La litemia es de 4,1 mEq/L (toxicidad > 1,6 mEq/L). \u00bfCu\u00e1l de las siguientes opciones terap\u00e9utica estar\u00eda m\u00e1s indicada?:", "full_answer": "Estamos ante un caso de intoxicaci\u00f3n grave por litio (litemia > 3,5 mEq/L amenaza la vida), por tanto, la opci\u00f3n terap\u00e9utica de elecci\u00f3n es el inicio de hemodi\u00e1lisis de forma urgente. Recordad que el carb\u00f3n activado NO est\u00e1 indicado en la intoxicaci\u00f3n por litio.", "type": "CUIDADOS CR\u00cdTICOS", "options": {"1": "Aminofilina asociado a un cat\u00e1rtico.", "2": "Carb\u00f3n activado.", "3": "Hemodi\u00e1lisis.", "4": "Diuresis forzada.", "5": NaN}, "correct_option": 3, "explanations": {"1": {"exist": false, "char_ranges": [], "word_ranges": [], "text": ""}, "2": {"exist": true, "char_ranges": [[184, 262]], "word_ranges": [[31, 44]], "text": "Recordad que el carb\u00f3n activado NO est\u00e1 indicado en la intoxicaci\u00f3n por litio."}, "3": {"exist": true, "char_ranges": [[0, 183]], "word_ranges": [[0, 31]], "text": "Estamos ante un caso de intoxicaci\u00f3n grave por litio (litemia > 3,5 mEq/L amenaza la vida), por tanto, la opci\u00f3n terap\u00e9utica de elecci\u00f3n es el inicio de hemodi\u00e1lisis de forma urgente."}, "4": {"exist": false, "char_ranges": [], "word_ranges": [], "text": ""}, "5": {"exist": false, "char_ranges": [], "word_ranges": [], "text": ""}}} {"id": 85, "year": 2012, "question_id_specific": 50, "full_question": "Un paciente con antecedentes de cuadro febril y dolor tor\u00e1cico acude al hospital con disnea y taquipnea. En la exploraci\u00f3n f\u00edsica las cifras de presi\u00f3n arterial est\u00e1n bajas, la presi\u00f3n yugular venosa est\u00e1 elevada con un seno \u201cX\u201d descendente profundo y tiene pulso parad\u00f3jico. \u00bfQu\u00e9 patolog\u00eda debe sospechar?", "full_answer": "El pulso parad\u00f3jico es un descenso de la presi\u00f3n arterial > 10 mmHg durante la inspiraci\u00f3n; representa una exageraci\u00f3n del fen\u00f3meno fisiol\u00f3gico consistente en el descenso inspiratorio de la TA (normal hasta 10 mmHg). En el taponamiento cardiaco, la inspiraci\u00f3n, que provoca un incremento del aflujo sangu\u00edneo a las cavidades derechas, aumentando el volumen de las mismas, provoca de forma secundaria un desplazamiento del tabique interventricular hacia la izquierda, de tal modo que el coraz\u00f3n izquierdo aloja y expulsa menos sangre durante la s\u00edstole y el pulso, por tanto, disminuye. En un coraz\u00f3n normal no se da este desplazamiento exagerado, provocado por la presi\u00f3n que ejerce el taponamiento sobre la pared libre del VD. El seno X representa el colapso sist\u00f3lico del pulso venoso, es decir, la ca\u00edda de presi\u00f3n por la relajaci\u00f3n auricular (tambi\u00e9n en parte por un desplazamiento hacia abajo de la base del VD durante el per\u00edodo de s\u00edstole). El seno Y representa el colapso diast\u00f3lico del pulso venoso, es decir, la ca\u00edda de presi\u00f3n que tiene lugar desde el momento en que la sangre penetra por la v\u00e1lvula tric\u00faspide hacia el ventr\u00edculo. En el taponamiento cardiaco lo caracter\u00edstico es el seno X profundo. En la pericarditis constrictiva, el seno Y profundo. Por todas estas razones, la respuesta correcta es la 5.", "type": "CARDIOLOG\u00cdA Y CIRUG\u00cdA VASCULAR", "options": {"1": "Cardiopat\u00eda isqu\u00e9mica.", "2": "Miocardiopat\u00eda dilatada.", "3": "Estenosis valvular a\u00f3rtica severa.", "4": "Pericarditis constrictiva.", "5": "Derrame peric\u00e1rdico con taponamiento cardiaco."}, "correct_option": 5, "explanations": {"1": {"exist": false, "char_ranges": [], "word_ranges": [], "text": ""}, "2": {"exist": false, "char_ranges": [], "word_ranges": [], "text": ""}, "3": {"exist": false, "char_ranges": [], "word_ranges": [], "text": ""}, "4": {"exist": false, "char_ranges": [], "word_ranges": [], "text": ""}, "5": {"exist": true, "char_ranges": [[728, 1265]], "word_ranges": [[114, 205]], "text": "El seno X representa el colapso sist\u00f3lico del pulso venoso, es decir, la ca\u00edda de presi\u00f3n por la relajaci\u00f3n auricular (tambi\u00e9n en parte por un desplazamiento hacia abajo de la base del VD durante el per\u00edodo de s\u00edstole). El seno Y representa el colapso diast\u00f3lico del pulso venoso, es decir, la ca\u00edda de presi\u00f3n que tiene lugar desde el momento en que la sangre penetra por la v\u00e1lvula tric\u00faspide hacia el ventr\u00edculo. En el taponamiento cardiaco lo caracter\u00edstico es el seno X profundo. En la pericarditis constrictiva, el seno Y profundo."}}} {"id": 527, "year": 2021, "question_id_specific": 31, "full_question": "El resultado de la gasometr\u00eda arterial de un paciente es: pH: 7,40, PaCO2 60 mmHg; bicarbonato 36 mM/L. \u00bfCu\u00e1l es la alteraci\u00f3n que presenta?:", "full_answer": "Las gasometr\u00edas pueden ser enga\u00f1osas ya que un pH normal puede sugerir que no hay ning\u00fan trastorno del equilibrio \u00e1cido-base, en cambio est\u00e1 enmascarando un doble trastorno en direcciones opuestas como en este caso. El paciente presenta acidosis respiratoria (PaCO2 60 mmHg; normal hasta 50) y alcalosis metab\u00f3lica compensatoria (bicarbonato 36 mmol/l; normal hasta 28). Es la t\u00edpica gasometr\u00eda del paciente con EPOC e insuficiencia respiratoria cr\u00f3nica compensada.", "type": "NEFROLOG\u00cdA", "options": {"1": "Ninguna, el pH es normal.", "2": "Acidosis respiratoria.", "3": "Acidosis respiratoria y alcalosis metab\u00f3lica.", "4": "Alcalosis metab\u00f3lica.", "5": NaN}, "correct_option": 3, "explanations": {"1": {"exist": false, "char_ranges": [], "word_ranges": [], "text": ""}, "2": {"exist": false, "char_ranges": [], "word_ranges": [], "text": ""}, "3": {"exist": true, "char_ranges": [[216, 465]], "word_ranges": [[34, 68]], "text": "El paciente presenta acidosis respiratoria (PaCO2 60 mmHg; normal hasta 50) y alcalosis metab\u00f3lica compensatoria (bicarbonato 36 mmol/l; normal hasta 28). Es la t\u00edpica gasometr\u00eda del paciente con EPOC e insuficiencia respiratoria cr\u00f3nica compensada."}, "4": {"exist": false, "char_ranges": [], "word_ranges": [], "text": ""}, "5": {"exist": false, "char_ranges": [], "word_ranges": [], "text": ""}}} {"id": 407, "year": 2016, "question_id_specific": 81, "full_question": "Acude a la consulta una mujer de 30 a\u00f1os de edad refiriendo en los tres \u00faltimos meses ansiedad, p\u00e9rdida de unos 6 kgs de peso y sensaci\u00f3n de \u201cnerviosismo\u201d. En la exploraci\u00f3n f\u00edsica destaca taquicardia, hiperreflexia y ausencia de bocio. En la anal\u00edtica realizada los valores de la TSH son < 0.01 microU/mL, la T4 est\u00e1 elevada y los niveles de tiroglobulina descendidos. Al realizarle una gammagraf\u00eda se detecta una ausencia de captaci\u00f3n en la regi\u00f3n tiroidea. \u00bfCu\u00e1l le parece el diagn\u00f3stico m\u00e1s probable?", "full_answer": "Los pacientes que padecen este trastorno ingieren voluntaria o accidentalmente cantidades excesivas de hormona tiroidea, lo que provoca hipertiroidismo en ausencia de bocio. En contraste con todas las dem\u00e1s causas de hipertiroidismo, la tiroglobulina s\u00e9rica no est\u00e1 elevada y est\u00e1 casi siempre baja o en el l\u00edmite inferior de la normalidad. La duda fundamental la tuvimos con el teratoma del ovario (struma ov\u00e1rico) que es una variante poco frecuente de teratoma maduro monodermal compuesto por tejido tiroideo descrito a principios del siglo XX. Recibe este nombre cuando el teratoma est\u00e1 compuesto predominantemente por tejido tiroideo maduro (al menos un 50%, para poder clasificarlo como struma ovarii). Representa entre el 2,5 y 5% de los teratomas ov\u00e1ricos. En s\u00ed, es una tumoraci\u00f3n benigna, pero hasta en un 5% de casos puede desarrollar una transformaci\u00f3n maligna de sus elementos, denomin\u00e1ndose cl\u00e1sicamente en estos casos struma ovarii maligno. Principalmente, el tejido neopl\u00e1sico desarrollado es un carcinoma papilar, menos frecuentemente corresponde a carcinoma folicular. Esta transformaci\u00f3n maligna surge mayoritariamente de las formas cl\u00e1sicas de struma, pero es extremadamente infrecuente su aparici\u00f3n en casos de teratomas qu\u00edsticos maduros. Su comportamiento es similar al de otros tumores ov\u00e1ricos primarios, tendiendo a metastatizar en cavidad peritoneal y v\u00eda hemat\u00f3gena a hueso, h\u00edgado y cerebro. La edad media de presentaci\u00f3n del carcinoma tiroideo sobre struma ovarii se sit\u00faa entre 42,9-44 a\u00f1os y entre 21 a 63 para la variante de carcinoma papilar. Cl\u00ednicamente, la forma de presentaci\u00f3n predominante es como masa p\u00e9lvica o dolor abdominal (45%), menos frecuentemente hipertiroidismo (5-8%) o ascitis (17%). Se postula que el tejido tiroideo ov\u00e1rico en el struma es aut\u00f3nomo en la producci\u00f3n de hormonas tiroideas, por lo cual es fundamental la medici\u00f3n de niveles s\u00e9ricos de tiroglobulina en pacientes afectadas de hipertiroidismo, ya que orientan hacia la producci\u00f3n extratiroidea de hormona tiroidea.", "type": "ONCOLOG\u00cdA (ECT\u00d3PICO)", "options": {"1": "Tirotoxicosis facticia.", "2": "Hipertiroidismo por Enfermedad de Graves.", "3": "Teratoma de ovario (estruma ov\u00e1rico).", "4": "Tiroiditis subaguda.", "5": NaN}, "correct_option": 1, "explanations": {"1": {"exist": false, "char_ranges": [], "word_ranges": [], "text": ""}, "2": {"exist": false, "char_ranges": [], "word_ranges": [], "text": ""}, "3": {"exist": false, "char_ranges": [], "word_ranges": [], "text": ""}, "4": {"exist": false, "char_ranges": [], "word_ranges": [], "text": ""}, "5": {"exist": false, "char_ranges": [], "word_ranges": [], "text": ""}}} {"id": 15, "year": 2011, "question_id_specific": 126, "full_question": "Por tercera vez en los \u00faltimos 6 meses un hombre de 84 a\u00f1os de edad con demencia avanzada ingresa en un hospital por neumon\u00eda aspirativa. Ha perdido 9.5 kg en los \u00faltimos 10 meses. Tiene una \u00falcera por presi\u00f3n en el sacro. No se comunica verbalmente, no deambula y es dependiente para todas las actividades de la vida diaria. Su mujer cuida de \u00e9l en casa y no quiere ingresar en una residencia. Un estudio de la degluci\u00f3n indica que se atraganta con todo tipo de consistencias de los alimentos. El m\u00e9dico del hospital sugiere alimentaci\u00f3n por sonda. La \"Voluntad Vital Anticipada\" del paciente establece que su mujer es la representante que debe tomar las decisiones respecto a su atenci\u00f3n m\u00e9dica y que \u00e9l no quiere medidas extraordinarias para prolongar su vida incluyendo la nutrici\u00f3n artificial. \u00bfCu\u00e1l de las siguientes es la recomendaci\u00f3n m\u00e1s apropiada para este paciente?:", "full_answer": "Quiz\u00e1s sea esta la menos cient\u00edfica de las preguntas MIR de este a\u00f1o, por cuanto su respuesta se basa en la interpretaci\u00f3n de una ley, que adem\u00e1s tiene car\u00e1cter auton\u00f3mico. Probablemente en el esp\u00edritu del examinador se quiera reflejar esta ley, en un caso repito extremo. Veamos, partiendo del art\u00edculo 11 de la Ley 41 /2002, de 14 de noviembre (1), b\u00e1sica reguladora de la autonom\u00eda del paciente y de derechos y obligaciones en materia de informaci\u00f3n y documentaci\u00f3n cl\u00ednica. (B.O.E. N\u00ba 274, de 15 de noviembre) de car\u00e1cter nacional, en Castilla-La Mancha se legisla la Ley 6/2005, de 7 de julio (2), de Castilla-La Mancha, sobre la Declaraci\u00f3n de Voluntades Anticipadas en materia de la propia salud. (D.O.C.M. N\u00ba 141, de 15 de julio) y el Decreto 15/2006, de 21 de febrero (3), del Registro de Voluntades Anticipadas de Castilla-La Mancha. (D.O.C.M. N\u00ba 42, de 24 de febrero). En el resumen que publica la JCCM (4) dice textualmente: \"La declaraci\u00f3n de voluntades anticipadas (tambi\u00e9n conocida como documento de instrucciones previas o testamento vital) es la manifestaci\u00f3n escrita de una persona capaz que, actuando libremente, expresa las instrucciones que deben tenerse en cuenta acerca de la asistencia sanitaria que desea recibir en situacio\u00adnes que le impidan comunicar personalmente su voluntad, o sobre el destino de su cuerpo o sus \u00f3rganos una vez producido el fallecimiento. En nuestra declaraci\u00f3n de voluntades anticipadas podemos hacer referencia a: 1. La expresi\u00f3n de los valores personales, a fin de ayudar en su d\u00eda a la interpretaci\u00f3n del propio documento y orientar a los m\u00e9dicos en el momento de tomar decisiones cl\u00ednicas. 2. Las instrucciones sobre los cuidados y tratamientos relacionados con nuestra salud que deseemos o no recibir. 3. Podemos nombrar un representante que actuar\u00e1 como interlocutor ante el m\u00e9dico responsable o el equipo sanitario para que se cumplan las instrucciones contenidas en la declaraci\u00f3n. 4. Tambi\u00e9n podemos incluir nuestra decisi\u00f3n respecto a la donaci\u00f3n de \u00f3rganos. En estos casos no se requerir\u00e1 la autorizaci\u00f3n de la familia para la extracci\u00f3n o utilizaci\u00f3n de los \u00f3rganos donados\". Parece pues claro que la respuesta que se pretende es la 5.", "type": "CUIDADOS PALIATIVOS", "options": {"1": "Colocar una SNG permanente y dar el alta a una residencia con cuidados especializados.", "2": "Colocar una SNG a medio plazo y dar el alta a una residencia con cuidados especializados hasta que se curen las escaras.", "3": "Dar el alta a una residencia con cuidados especializados hasta que se curen las \u00falceras por presi\u00f3n.", "4": "Poner una sonda por gastrostom\u00eda y dar el alta a domicilio.", "5": "Dar el alta a domicilio con cuidados paliativos del \u00e1rea sanitaria."}, "correct_option": 5, "explanations": {"1": {"exist": false, "char_ranges": [], "word_ranges": [], "text": ""}, "2": {"exist": false, "char_ranges": [], "word_ranges": [], "text": ""}, "3": {"exist": false, "char_ranges": [], "word_ranges": [], "text": ""}, "4": {"exist": false, "char_ranges": [], "word_ranges": [], "text": ""}, "5": {"exist": false, "char_ranges": [], "word_ranges": [], "text": ""}}} {"id": 322, "year": 2016, "question_id_specific": 85, "full_question": "Un chico de 22 a\u00f1os de edad con hiposmia presenta falta de desarrollo de caracteres sexuales secundarios e infertilidad. Volumen testicular de 4 mL bilateral. Anal\u00edticamente, EST-I 1,2 U/L (vn 5-15); LH 0,6 U/L (vn 3-15); testosterona 100 ng/dL (vn 300-1200), prolactina normal. Se\u00f1ale el tratamiento que le propondr\u00e1 para conseguir fertilidad:", "full_answer": "Se trata de un s\u00edndrome de Kallman (hipogonadismo hipogonadotropo + alteraci\u00f3n olfactiva). La triptorelina es un agonista de la GnRH, que mediante acci\u00f3n continua reduce la FSH y LH, y se usa en c\u00e1nceres dependientes de hormonas sexuales, as\u00ed como pubertad precoz. La bromocriptina es un agonista dopamin\u00e9rgico que se usa en hiperPRL (este tiene la PRL normal), tumores hipofisarios\u2026 Nos quedamos entre la 1 y la 3. Para inducir la maduraci\u00f3n testicular, se pude usar GnRH puls\u00e1til con bomba, hCG o FSH+hCG. No se usa la LH como tratamiento, se estimular\u00eda a trav\u00e9s de la GnRH intermitente.", "type": "UROLOG\u00cdA", "options": {"1": "Bomba de infusi\u00f3n de GnRH.", "2": "Administraci\u00f3n intramuscular mensual de triptorelina.", "3": "Administraci\u00f3n intramuscular de FSH y LH una vez por semana.", "4": "Tratamiento con bromocriptina.", "5": NaN}, "correct_option": 1, "explanations": {"1": {"exist": true, "char_ranges": [[416, 507]], "word_ranges": [[68, 83]], "text": "Para inducir la maduraci\u00f3n testicular, se pude usar GnRH puls\u00e1til con bomba, hCG o FSH+hCG."}, "2": {"exist": true, "char_ranges": [[91, 264]], "word_ranges": [[12, 42]], "text": "La triptorelina es un agonista de la GnRH, que mediante acci\u00f3n continua reduce la FSH y LH, y se usa en c\u00e1nceres dependientes de hormonas sexuales, as\u00ed como pubertad precoz."}, "3": {"exist": true, "char_ranges": [[508, 590]], "word_ranges": [[83, 98]], "text": "No se usa la LH como tratamiento, se estimular\u00eda a trav\u00e9s de la GnRH intermitente."}, "4": {"exist": true, "char_ranges": [[265, 383]], "word_ranges": [[42, 60]], "text": "La bromocriptina es un agonista dopamin\u00e9rgico que se usa en hiperPRL (este tiene la PRL normal), tumores hipofisarios\u2026"}, "5": {"exist": false, "char_ranges": [], "word_ranges": [], "text": ""}}} {"id": 610, "year": 2022, "question_id_specific": 116, "full_question": "Mujer de 95 a\u00f1os que vive en una residencia, independiente para sus actividades b\u00e1sicas de la vida diaria, sale al jard\u00edn a pasear. Tiene antecedentes de HTA, dislipidemia, osteoporosis y deterioro cognitivo leve. Sufre una ca\u00edda al levantarse por la noche al ba\u00f1o. La radiograf\u00eda muestra una fractura subcapital desplazada de cadera derecha. \u00bfCu\u00e1l es el tratamiento recomendado?:", "full_answer": "Fractura de cadera desplazada, Garden III-IV. Tratamiento quir\u00fargico. Hemiartroplastia de cadera. Opci\u00f3n 2 tratamiento para fracturas pertrocantereas. Opci\u00f3n 3. Fracturas no desplazadas. Opci\u00f3n 4. Pacientes no deambulantes con m\u00ednimo dolor y alto riesgo quir\u00fargico.", "type": "TRAUMATOLOG\u00cdA", "options": {"1": "Hemiartroplastia de cadera.", "2": "Fijaci\u00f3n con clavo trocant\u00e9rico.", "3": "Fijaci\u00f3n con tornillos canulados.", "4": "Conservador: vida cama-sill\u00f3n.", "5": NaN}, "correct_option": 1, "explanations": {"1": {"exist": true, "char_ranges": [[0, 97]], "word_ranges": [[0, 11]], "text": "Fractura de cadera desplazada, Garden III-IV. Tratamiento quir\u00fargico. Hemiartroplastia de cadera."}, "2": {"exist": true, "char_ranges": [[98, 150]], "word_ranges": [[11, 17]], "text": "Opci\u00f3n 2 tratamiento para fracturas pertrocantereas."}, "3": {"exist": true, "char_ranges": [[151, 185]], "word_ranges": [[17, 22]], "text": "Opci\u00f3n 3. Fracturas no desplazadas."}, "4": {"exist": true, "char_ranges": [[187, 265]], "word_ranges": [[22, 34]], "text": "Opci\u00f3n 4. Pacientes no deambulantes con m\u00ednimo dolor y alto riesgo quir\u00fargico."}, "5": {"exist": false, "char_ranges": [], "word_ranges": [], "text": ""}}} {"id": 391, "year": 2016, "question_id_specific": 222, "full_question": "Hombre de 26 an\u0303os, soltero, que es tra\u00eddo a consulta por su familia por llevar 3 meses nega\u0301ndose a salir se su casa. La razo\u0301n que aduce el paciente es el convencimiento de que tiene la mandi\u0301bula asime\u0301trica y la cara torcida. Segu\u0301n refiere esta situacio\u0301n es progresiva y cada vez se ve mas deforme cuando se observa en el espejo. Se avergu\u0308enza de su aspecto, por lo que no quiere salir, se angustia mucho cuando ve su imagen y no puede dejar de pensar todo el di\u0301a en su deformidad. Ha consultado con varios cirujanos maxilofaciales pero e\u0301stos le dicen que no presenta asimetri\u0301a facial y lo remiten al psiquiatra. El diagno\u0301stico del paciente es:", "full_answer": "Otra pregunta sencilla de respuesta inmediata, que no ofrece duda. Nos describe a un paciente preocupado por un defecto f\u00edsico inexistente, cuya preocupaci\u00f3n le angustia e impide salir de casa. Como residente de psiquiatr\u00eda me gustar\u00eda que las preguntas del MIR de mi especialidad invitaran algo m\u00e1s a la reflexi\u00f3n y a la profundidad, aunque s\u00e9 que los segundos que os habr\u00e9is ahorrado al marcar la cuarta directamente son muy valiosos.", "type": "PSIQUIATR\u00cdA", "options": {"1": "Trastorno depresivo mayor con ideas delirantes incongruentes con el estado de a\u0301nimo.", "2": "Trastorno obsesivo compulsivo.", "3": "Esquizofrenia paranoide.", "4": "Trastorno dismo\u0301rfico corporal.", "5": NaN}, "correct_option": 4, "explanations": {"1": {"exist": false, "char_ranges": [], "word_ranges": [], "text": ""}, "2": {"exist": false, "char_ranges": [], "word_ranges": [], "text": ""}, "3": {"exist": false, "char_ranges": [], "word_ranges": [], "text": ""}, "4": {"exist": true, "char_ranges": [[67, 193]], "word_ranges": [[10, 30]], "text": "Nos describe a un paciente preocupado por un defecto f\u00edsico inexistente, cuya preocupaci\u00f3n le angustia e impide salir de casa."}, "5": {"exist": false, "char_ranges": [], "word_ranges": [], "text": ""}}} {"id": 324, "year": 2016, "question_id_specific": 55, "full_question": "Una paciente de 70 a\u00f1os ingresa en UCl tras sufrir IAM anterior tratado mediante angioplastia coronaria y colocaci\u00f3n de stent en la arteria descendente anterior. 4 d\u00edas despu\u00e9s presenta bruscamente hipotensi\u00f3n que obliga a aporte vigoroso de volumen, inicio de drogas vasoactivas, intubaci\u00f3n orotraqueal y conexi\u00f3n a la ventilaci\u00f3n mec\u00e1nica. A la exploraci\u00f3n f\u00edsica destaca un soplo no presente previamente. Ante la sospecha de complicaci\u00f3n mec\u00e1nica del infarto, se realiza ecocardiografia transtor\u00e1cica que muestra derrame peric\u00e1rdico. Se\u00f1ale la Respuesta CORRECTA:", "full_answer": "La mortalidad no es tan baja, el salto oxim\u00e9trico se da en el caso de rotura del septum, al igual que el fr\u00e9mito (de ah\u00ed que la respuesta correcta sea la 3) y las complicaciones se pueden presentar hasta en la primera semana.", "type": "CUIDADOS CR\u00cdTICOS, PALIATIVOS Y URGENCIAS", "options": {"1": "La mortalidad con tratamiento m\u00e9dico es del 20%.", "2": "En caso de rotura de pared libre hay saltooxim\u00e9trico en el ventr\u00edculo derecho en el cateterismo de Swan-Ganz.", "3": "En caso de rotura de pared libre no hay fr\u00e9mito palpable.", "4": "Las complicaciones mec\u00e1nicas suelen aparecer en el primer d\u00eda postinfarto.", "5": NaN}, "correct_option": 3, "explanations": {"1": {"exist": false, "char_ranges": [], "word_ranges": [], "text": ""}, "2": {"exist": false, "char_ranges": [], "word_ranges": [], "text": ""}, "3": {"exist": true, "char_ranges": [[0, 156]], "word_ranges": [[0, 32]], "text": "La mortalidad no es tan baja, el salto oxim\u00e9trico se da en el caso de rotura del septum, al igual que el fr\u00e9mito (de ah\u00ed que la respuesta correcta sea la 3)"}, "4": {"exist": false, "char_ranges": [], "word_ranges": [], "text": ""}, "5": {"exist": false, "char_ranges": [], "word_ranges": [], "text": ""}}} {"id": 240, "year": 2014, "question_id_specific": 150, "full_question": "Hombre de 76 a\u00f1os de edad que consulta por deterioro cognitivo, lentitud y torpeza de movimientos, de ocho meses de evoluci\u00f3n. La familia refiere que el paciente presentaba alucinaciones visuales, por lo que su m\u00e9dico de Atenci\u00f3n Primaria paut\u00f3 dosis bajas de risperidona, con un importante empeoramiento del estado motor. A la vista de estos datos, \u00bfcu\u00e1l es el diagn\u00f3stico m\u00e1s probable?", "full_answer": "La respuesta correcta es la 2 ya que presenta el caso con los s\u00edntomas t\u00edpicos de la demencia por cuerpos de Lewy, siendo el s\u00edntoma gu\u00eda las alucinaciones visuales. Tambi\u00e9n se mencionan deterioro cognitivo progresivo, signos motores parkinsonianos (se deduce de la torpeza de movimientos) y la hipersensibilidad a los neurol\u00e9pticos incluso en peque\u00f1as dosis.", "type": "NEUROLOG\u00cdA", "options": {"1": "Enfermedad de Alzheimer.", "2": "Demencia por cuerpos de Lewy.", "3": "Demencia frontotemporal.", "4": "Demencia vascular.", "5": "Enfermedad de Creutzfeldt-Jakob espor\u00e1dica."}, "correct_option": 2, "explanations": {"1": {"exist": false, "char_ranges": [], "word_ranges": [], "text": ""}, "2": {"exist": true, "char_ranges": [[0, 165]], "word_ranges": [[0, 29]], "text": "La respuesta correcta es la 2 ya que presenta el caso con los s\u00edntomas t\u00edpicos de la demencia por cuerpos de Lewy, siendo el s\u00edntoma gu\u00eda las alucinaciones visuales."}, "3": {"exist": false, "char_ranges": [], "word_ranges": [], "text": ""}, "4": {"exist": false, "char_ranges": [], "word_ranges": [], "text": ""}, "5": {"exist": false, "char_ranges": [], "word_ranges": [], "text": ""}}} {"id": 68, "year": 2012, "question_id_specific": 123, "full_question": "Un hombre de 64 a\u00f1os de edad con antecedentes de HTA y cardiopat\u00eda isqu\u00e9mica acude a Urgencias por aparici\u00f3n de dolor tor\u00e1cico opresivo mientras ve\u00eda la televisi\u00f3n. En la consulta de clasificaci\u00f3n (triage) de Urgencias, se detectan cifras de T. art de 155/95 mmHg y una saturaci\u00f3n capilar de O2 de 95% \u00bfCu\u00e1l es la conducta m\u00e1s correcta de las que a continuaci\u00f3n se citan?", "full_answer": "Si el ECG tiene cambios isqu\u00e9micos, evidentemente, le atenderemos en el box de emergencia y avisaremos al cardi\u00f3logo. Si no, se le tratar\u00e1 como si fuera una angina y se observar\u00e1, seriando enzimas y con tratamiento en el \u00e1rea de observaci\u00f3n.", "type": "ANESTESIOLOG\u00cdA, CUIDADOS CR\u00cdTICOS Y URGENCIAS", "options": {"1": "Indicarle que vaya a la sala de espera. Le avisar\u00e1n para realizaci\u00f3n de pruebas.", "2": "Este paciente debe ser atendido en el box de emergencia vital (cuarto de shock).", "3": "Avisaremos al cardi\u00f3logo de guardia para la valoraci\u00f3n del paciente.", "4": "Desde la consulta de Triage se le remitir\u00e1 a la Unidad coronaria.", "5": "Le haremos un ECG en menos de 10 minutos."}, "correct_option": 5, "explanations": {"1": {"exist": false, "char_ranges": [], "word_ranges": [], "text": ""}, "2": {"exist": false, "char_ranges": [], "word_ranges": [], "text": ""}, "3": {"exist": false, "char_ranges": [], "word_ranges": [], "text": ""}, "4": {"exist": false, "char_ranges": [], "word_ranges": [], "text": ""}, "5": {"exist": false, "char_ranges": [], "word_ranges": [], "text": ""}}} {"id": 206, "year": 2014, "question_id_specific": 125, "full_question": "Ante un hombre de 49 a\u00f1os, asintom\u00e1tico, con el antecedente familiar de padre fallecido por c\u00e1ncer de pr\u00f3stata, que en un control rutinario de empresa se identifica un PSA (Ant\u00edgeno Prost\u00e1tico Espec\u00edfico) de 5,9 ng/ml, con un cociente de PSA libre/PSA total del 11% y que en un tacto rectal se aprecia aumento de consistencia en el l\u00f3bulo derecho prost\u00e1tico, \u00bfcu\u00e1l es la siguiente indicaci\u00f3n cl\u00ednica?", "full_answer": "El paciente tiene PSA >4 y tacto con aumento de consistencia, lo que implica la necesidad de una biopsia guiada por ecograf\u00eda transrectal, para diagnosticar si se trata de una Hiperplasia Benigna de Pr\u00f3stata o C\u00e1ncer de pr\u00f3stata.", "type": "UROLOG\u00cdA", "options": {"1": "Plantear al paciente la realizaci\u00f3n de una ecograf\u00eda transrectal y biopsia prost\u00e1tica.", "2": "Realizar un TAC abdominop\u00e9lvico.", "3": "Iniciar tratamiento con Inhibidores de la 5 alfa Reductasa para reducir a la mitad los niveles del PSA.", "4": "Iniciar tratamiento combinado de an\u00e1logos de la LHRH y antiandr\u00f3genos.", "5": "Realizar una gammagraf\u00eda \u00f3sea."}, "correct_option": 1, "explanations": {"1": {"exist": true, "char_ranges": [[0, 229]], "word_ranges": [[0, 38]], "text": "El paciente tiene PSA >4 y tacto con aumento de consistencia, lo que implica la necesidad de una biopsia guiada por ecograf\u00eda transrectal, para diagnosticar si se trata de una Hiperplasia Benigna de Pr\u00f3stata o C\u00e1ncer de pr\u00f3stata."}, "2": {"exist": false, "char_ranges": [], "word_ranges": [], "text": ""}, "3": {"exist": false, "char_ranges": [], "word_ranges": [], "text": ""}, "4": {"exist": false, "char_ranges": [], "word_ranges": [], "text": ""}, "5": {"exist": false, "char_ranges": [], "word_ranges": [], "text": ""}}} {"id": 570, "year": 2022, "question_id_specific": 153, "full_question": "Var\u00f3n de 19 a\u00f1os sin antecedentes de inter\u00e9s que acude a urgencias porque despu\u00e9s de una infecci\u00f3n respiratoria presenta astenia, malestar general, oliguria y cefalea. Exploraci\u00f3n f\u00edsica: TA 210/120 mmHg, fondo de ojo con retinopat\u00eda hipertensiva grado III. Anal\u00edtica: Hb 7,4 g/dl, plaquetas 85.000/mm3, 2-3 esquistocitos en la extensi\u00f3n de sangre perif\u00e9rica, LDH 950 UI/ml, creatinina s\u00e9rica 8,75 mg/dl. Sistem\u00e1tico de orina: proteinuria 300 mg/dl, sedimento 15 hemat\u00edes por campo. Niveles de ADAMTS-13 normales. Ante estos hallazgos \u00bfcu\u00e1l ser\u00eda su principal sospecha diagn\u00f3stica?:", "full_answer": "Pregunta dif\u00edcil. El paciente presenta una microangiopat\u00eda tromb\u00f3tica (MAT) con fracaso renal agudo. Esto orienta hacia SHU, PTT, MAT inducida por f\u00e1rmacos, o MAT mediada por complemento, coagulaci\u00f3n o metabolismo. Los niveles normales de ADAMTS-13 descartan la PTT (opci\u00f3n 1 incorrecta). La ausencia de cl\u00ednica gastrointestinal descarta el SHU t\u00edpico por toxina Shiga (opci\u00f3n 4 incorrecta). Las infecciones por neumococo tambi\u00e9n pueden causar SHU, aunque es raro y m\u00e1s en adultos. El antecedente de infecci\u00f3n respiratoria (no especificada, podr\u00eda ser una neumon\u00eda neumoc\u00f3cica) junto con la ausencia de datos de CID (no mencionan hemorragias ni pruebas de coagulaci\u00f3n alteradas) me hace pensar en un SHU at\u00edpico (opci\u00f3n 2 correcta).", "type": "NEFROLOG\u00cdA", "options": {"1": "P\u00farpura tromb\u00f3tica trombocitop\u00e9nica.", "2": "S\u00edndrome hemol\u00edtico ur\u00e9mico at\u00edpico.", "3": "Coagulaci\u00f3n intravascular diseminada.", "4": "S\u00edndrome hemol\u00edtico ur\u00e9mico t\u00edpico.", "5": NaN}, "correct_option": 2, "explanations": {"1": {"exist": true, "char_ranges": [[215, 288]], "word_ranges": [[30, 41]], "text": "Los niveles normales de ADAMTS-13 descartan la PTT (opci\u00f3n 1 incorrecta)."}, "2": {"exist": true, "char_ranges": [[482, 732]], "word_ranges": [[71, 109]], "text": "El antecedente de infecci\u00f3n respiratoria (no especificada, podr\u00eda ser una neumon\u00eda neumoc\u00f3cica) junto con la ausencia de datos de CID (no mencionan hemorragias ni pruebas de coagulaci\u00f3n alteradas) me hace pensar en un SHU at\u00edpico (opci\u00f3n 2 correcta)."}, "3": {"exist": false, "char_ranges": [], "word_ranges": [], "text": ""}, "4": {"exist": true, "char_ranges": [[289, 391]], "word_ranges": [[41, 56]], "text": "La ausencia de cl\u00ednica gastrointestinal descarta el SHU t\u00edpico por toxina Shiga (opci\u00f3n 4 incorrecta)."}, "5": {"exist": false, "char_ranges": [], "word_ranges": [], "text": ""}}} {"id": 506, "year": 2020, "question_id_specific": 87, "full_question": "Ante un trastorno obsesivo-compulsivo (TOC) de inicio s\u00fabito en un ni\u00f1o de 9 a\u00f1os que adem\u00e1s presenta tics y corea como manifestaciones neurol\u00f3gicas se deber\u00eda descartar una infecci\u00f3n por:", "full_answer": "Hay numerosa bibliograf\u00eda al respecto. No dejan de asombrarnos los extra\u00f1os caminos de la biolog\u00eda y que el axioma \u201cla m\u00e1s larga es la correcta\u201d se cumpla en este caso.", "type": "PSIQUIATR\u00cdA", "options": {"1": "Estreptococo viridans.", "2": "Estreptococo alfa-hemol\u00edtico.", "3": "Estreptococo beta-hemol\u00edtico, grupo A.", "4": "Enterococo.", "5": NaN}, "correct_option": 3, "explanations": {"1": {"exist": false, "char_ranges": [], "word_ranges": [], "text": ""}, "2": {"exist": false, "char_ranges": [], "word_ranges": [], "text": ""}, "3": {"exist": false, "char_ranges": [], "word_ranges": [], "text": ""}, "4": {"exist": false, "char_ranges": [], "word_ranges": [], "text": ""}, "5": {"exist": false, "char_ranges": [], "word_ranges": [], "text": ""}}} {"id": 619, "year": 2022, "question_id_specific": 121, "full_question": "Mujer de 79 a\u00f1os que ingresa por una fractura osteopor\u00f3tica de cadera. Respecto a la prevenci\u00f3n secundaria de las fracturas por fragilidad, se\u00f1ale la respuesta INCORRECTA:", "full_answer": "Existen dudas respecto a la eficacia del calcio o la vitamina D administradas en monoterapia para el tratamiento de la osteoporosis; se\u00f1alan que \u201cadministrados de forma conjunta parecen poseer un cierto grado de eficacia en la prevenci\u00f3n de fracturas no vertebrales, que es m\u00e1s claro en personas deficitarias en ellos, como puede ser el caso de ancianos que viven en residencias. No hay pr\u00e1cticamente evidencia de su eficacia en la prevenci\u00f3n de fracturas vertebrales.", "type": "TRAUMATOLOG\u00cdA", "options": {"1": "La baja adherencia al tratamiento se asocia a un incremento del riesgo de fractura.", "2": "Los marcadores de remodelado \u00f3seo pueden ser \u00fatiles para monitorizar de forma precoz la respuesta al tratamiento.", "3": "La vitamina D en monoterapia es eficaz en la reducci\u00f3n de dichas fracturas en personas mayores no institucionalizadas.", "4": "El aumento de calcio diet\u00e9tico o tomar suplementos de calcio de forma aislada no protegen frente a la aparici\u00f3n de fracturas.", "5": NaN}, "correct_option": 3, "explanations": {"1": {"exist": false, "char_ranges": [], "word_ranges": [], "text": ""}, "2": {"exist": false, "char_ranges": [], "word_ranges": [], "text": ""}, "3": {"exist": true, "char_ranges": [[0, 132]], "word_ranges": [[0, 21]], "text": "Existen dudas respecto a la eficacia del calcio o la vitamina D administradas en monoterapia para el tratamiento de la osteoporosis;"}, "4": {"exist": false, "char_ranges": [], "word_ranges": [], "text": ""}, "5": {"exist": false, "char_ranges": [], "word_ranges": [], "text": ""}}} {"id": 161, "year": 2013, "question_id_specific": 118, "full_question": "Una enferma de 60 a\u00f1os diagnosticada de broncopat\u00eda cr\u00f3nica, diabetes mellitus, hipertensi\u00f3n arterial y tratada desde hace 4 meses con Omeprazol, Metformina, Salbutamol, Bromuro de Ipratropio y Enalapril 20 mg + Hidroclorotiazida 25 mg acude a su consulta por cansancio, disminuci\u00f3n del apetito, con ligera disnea y tos ocasional, deposiciones variables, a veces blandas y sin s\u00edntomas urinarios. Unos an\u00e1lisis muestran leucocitos 10.000/mm\u00b3, Hto 35%, VCM 80, Glucosa 150 mg/dl, Urea 80 mg/dl, Creatinina 1,6 mg/dl, Sodio 133 mEq/l y Potasio 2,9 mEq/l. \u00bfCu\u00e1l es la causa m\u00e1s probable de la hipopotasemia?", "full_answer": "La respuesta correcta es: 4. Antihipertensivo. En particular la hidroclorotiazida, como tiazida, es el principal responsable de la hipopotasemia de este paciente.", "type": "NEFROLOG\u00cdA", "options": {"1": "Insuficiencia renal.", "2": "Hiponatremia.", "3": "D\u00e9ficit de aporte de potasio.", "4": "Antihipertensivo.", "5": "Metformina."}, "correct_option": 4, "explanations": {"1": {"exist": false, "char_ranges": [], "word_ranges": [], "text": ""}, "2": {"exist": false, "char_ranges": [], "word_ranges": [], "text": ""}, "3": {"exist": false, "char_ranges": [], "word_ranges": [], "text": ""}, "4": {"exist": true, "char_ranges": [[29, 162]], "word_ranges": [[5, 22]], "text": "Antihipertensivo. En particular la hidroclorotiazida, como tiazida, es el principal responsable de la hipopotasemia de este paciente."}, "5": {"exist": false, "char_ranges": [], "word_ranges": [], "text": ""}}} {"id": 269, "year": 2014, "question_id_specific": 141, "full_question": "Un paciente asm\u00e1tico de 55 a\u00f1os acude a Urgencias con una agudizaci\u00f3n. Una hora despu\u00e9s de la administraci\u00f3n de ox\u00edgeno suplementario y dos nebulizaciones de salbutamol el paciente no mejora. A la exploraci\u00f3n respira a 42 rpm con tiraje supraclavicular v presenta sibilancias inspiratorias y espiratorias diseminadas. El flujo pico ha bajado de 310 a 220 L/min. Una gasometr\u00eda extra\u00edda con ox\u00edgeno al 28% muestra una p02 de 54 mmHg y una pC02 de 35 mm Hg. \u00bfCu\u00e1l de las siguientes actitudes le parece MENOS indicada?", "full_answer": "En una crisis grave de asma todas las opciones estan indicadas o pueden estarl antes de opciones mas agresivas. En el caso de mala evolucion con acidosis etc, el paciente seria tributario de medicacion invasiva.", "type": "NEUMOLOG\u00cdA", "options": {"1": "Aumentar el flujo de ox\u00edgeno.", "2": "Administrar 80 mg de metilprednisolona iv.", "3": "Nebulizar bromuro de ipratropio junto con salbutamol cada 20 minutos.", "4": "Administrar magnesio intravenoso.", "5": " Iniciar ventilaci\u00f3n mec\u00e1nica no invasiva."}, "correct_option": 5, "explanations": {"1": {"exist": false, "char_ranges": [], "word_ranges": [], "text": ""}, "2": {"exist": false, "char_ranges": [], "word_ranges": [], "text": ""}, "3": {"exist": false, "char_ranges": [], "word_ranges": [], "text": ""}, "4": {"exist": false, "char_ranges": [], "word_ranges": [], "text": ""}, "5": {"exist": true, "char_ranges": [[112, 211]], "word_ranges": [[19, 35]], "text": "En el caso de mala evolucion con acidosis etc, el paciente seria tributario de medicacion invasiva."}}} {"id": 18, "year": 2011, "question_id_specific": 138, "full_question": "Si un paciente refiere tener desde hace meses aumento del tama\u00f1o de la lengua, sin otras alteraciones de la misma provocando que se muerda constantemente, en qu\u00e9 enfermedad pensar\u00eda de las siguientes:", "full_answer": "Pregunta compartida con ORL. La Amiloidosis es un dep\u00f3sito localizado o sist\u00e9mico de prote\u00ednas fibrilares. Puede ser primaria o secundaria. Los \u00f3rganos m\u00e1s frecuentemente afectados, son los ri\u00f1ones y el coraz\u00f3n. En ocasiones aparecen dep\u00f3sitos de amiloide en la lengua que pueden producir macroglosia.", "type": "DERMATOLOG\u00cdA", "options": {"1": "Par\u00e1lisis del hipogloso.", "2": "Hipertiroidismo.", "3": "Mastocitosis.", "4": "Amiloidosis.", "5": "Granuloma facial."}, "correct_option": 4, "explanations": {"1": {"exist": false, "char_ranges": [], "word_ranges": [], "text": ""}, "2": {"exist": false, "char_ranges": [], "word_ranges": [], "text": ""}, "3": {"exist": false, "char_ranges": [], "word_ranges": [], "text": ""}, "4": {"exist": true, "char_ranges": [[29, 301]], "word_ranges": [[4, 44]], "text": "La Amiloidosis es un dep\u00f3sito localizado o sist\u00e9mico de prote\u00ednas fibrilares. Puede ser primaria o secundaria. Los \u00f3rganos m\u00e1s frecuentemente afectados, son los ri\u00f1ones y el coraz\u00f3n. En ocasiones aparecen dep\u00f3sitos de amiloide en la lengua que pueden producir macroglosia."}, "5": {"exist": false, "char_ranges": [], "word_ranges": [], "text": ""}}} {"id": 250, "year": 2014, "question_id_specific": 78, "full_question": "A un hombre de 55 a\u00f1os, padre de un hijo celiaco, con anemia ferrop\u00e9nica y aumento reciente del ritmo deposicional, se le ha realizado una determinaci\u00f3n de los alelos HLA-DQ2 y HLA-DQ8 que ha resultado negativa. \u00bfQu\u00e9 estudio diagn\u00f3stico es el m\u00e1s apropiado en este caso?", "full_answer": "Var\u00f3n de edad media con anemia ferrop\u00e9nica y aumento del n\u00famero de deposiciones, \u00a1\u00a1hay que hacer una colonoscopia para descartar un c\u00e1ncer colorrectal!! Si el HLA es negativo es muy poco probable que se trate de una celiaqu\u00eda (elevado valor predictivo negativo).", "type": "APARATO DIGESTIVO", "options": {"1": "Determinaci\u00f3n de anticuerpos antitransglutaminasa IgA.", "2": "Endoscopia digestiva alta con toma de biopsias duodenales.", "3": "Test de D-xilosa.", "4": "Evaluar la respuesta a dieta sin gluten.", "5": "Colonoscopia."}, "correct_option": 5, "explanations": {"1": {"exist": false, "char_ranges": [], "word_ranges": [], "text": ""}, "2": {"exist": false, "char_ranges": [], "word_ranges": [], "text": ""}, "3": {"exist": false, "char_ranges": [], "word_ranges": [], "text": ""}, "4": {"exist": true, "char_ranges": [[153, 262]], "word_ranges": [[23, 42]], "text": "Si el HLA es negativo es muy poco probable que se trate de una celiaqu\u00eda (elevado valor predictivo negativo)."}, "5": {"exist": true, "char_ranges": [[0, 152]], "word_ranges": [[0, 23]], "text": "Var\u00f3n de edad media con anemia ferrop\u00e9nica y aumento del n\u00famero de deposiciones, \u00a1\u00a1hay que hacer una colonoscopia para descartar un c\u00e1ncer colorrectal!!"}}} {"id": 5, "year": 2011, "question_id_specific": 44, "full_question": "Un hombre de 66 a\u00f1os de edad, refiere pirosis diaria desde la juventud que trata con alcalinos. Se le practica una endoscopia digestiva alta que muestra esofagitis erosiva leve y la uni\u00f3n escamosa columnar desplazada aproximadamente 7 cm. Respecto a la porci\u00f3n m\u00e1s proximal de los pliegues g\u00e1stricos. Las biopsias del es\u00f3fago distal revelan que el epitelio escamoso normal ha sido reemplazado por epitelio columnar de tipo intestinal con displasia de bajo grado \u00bfCu\u00e1l es la opci\u00f3n m\u00e1s apropiada para el manejo de este paciente?", "full_answer": "Este paciente presenta un es\u00f3fago de Barrett con displasia de bajo grado. No me gusta la pregunta, ya que al no estar con inhibici\u00f3n de producci\u00f3n \u00e1cida la displasia puede estar condicionada por la inflamaci\u00f3n y no ser real. De todas formas, el es\u00f3fago de Barrett debe seguir vigilancia endosc\u00f3pica y tratamiento con IBP de por vida o cirug\u00eda antirreflujo para evitar evoluci\u00f3n. La respuesta correcta es la 4.", "type": "DIGESTIVO", "options": {"1": "Dado que la esofagitis es leve y la displasia de bajo grado, se aconseja continuar tratamiento con alcalinos.", "2": "Tratamiento indefinido con dosis altas de IBP ya que se ha demostrado que as\u00ed se evita la progresi\u00f3n del es\u00f3fago de Barrett a adenocarcinoma haciendo innecesaria la vigilancia endosc\u00f3pica.", "3": "Endoscopias de vigilancia peri\u00f3dicas y tratamiento con IBP durante menos de 12 semanas ya que tratamientos prolongados se asocian a un alto riesgo de desarrollo de gastrinoma.", "4": "Endoscopias de vigilancia peri\u00f3dicas y tratamiento indefinido con IBP.", "5": "Esofaguectom\u00eda."}, "correct_option": 4, "explanations": {"1": {"exist": false, "char_ranges": [], "word_ranges": [], "text": ""}, "2": {"exist": false, "char_ranges": [], "word_ranges": [], "text": ""}, "3": {"exist": false, "char_ranges": [], "word_ranges": [], "text": ""}, "4": {"exist": true, "char_ranges": [[225, 409]], "word_ranges": [[39, 69]], "text": "De todas formas, el es\u00f3fago de Barrett debe seguir vigilancia endosc\u00f3pica y tratamiento con IBP de por vida o cirug\u00eda antirreflujo para evitar evoluci\u00f3n. La respuesta correcta es la 4."}, "5": {"exist": false, "char_ranges": [], "word_ranges": [], "text": ""}}} {"id": 359, "year": 2016, "question_id_specific": 91, "full_question": "La determinaci\u00f3n en una mujer de 70 a\u00f1os de hematocrito 32%, Hb 9,6 g/dL, VCM 85 fL (70-99), Fe 25 ug/dL (37-145) y ferritina 350 ng/mL (15-150), es m\u00e1s sugestivo de:", "full_answer": "Polimialgia reum\u00e1tica. Clasico ejemplo supongo, y se ve en cl\u00ednica a mansalva. Mujer mayor con anemia normocitica y patr\u00f3n ferrocinetico claro de trastorno cr\u00f3nico (hiposideremia pero hiperferritinemia). La \u00fanica que cuadra, y de lujo, es la 3; solo les faltaba darnos una VSG alta. La 1: si es p\u00e9rdida reciente, el VCM podr\u00eda mantenerse normal, pero la ferritina deber\u00eda afectarse, y desde luego no estar elevada. La 2: lo mismo. Si es por malabsorci\u00f3n de Fe, la ferritina deber\u00eda ser baja. Y si fuera por malabsorci\u00f3n de B12, el VCM nos lo pondr\u00edan claramente mas alto. La 4: 70 tacos y VCM normal. Esta sobra.", "type": "HEMATOLOG\u00cdA", "options": {"1": "Perdida de sangre reciente por aparato digestivo.", "2": "Malabsorcion intestinal.", "3": "Polimialgia reum\u00e1tica.", "4": "Talasemia minor.", "5": NaN}, "correct_option": 3, "explanations": {"1": {"exist": true, "char_ranges": [[283, 414]], "word_ranges": [[44, 66]], "text": "La 1: si es p\u00e9rdida reciente, el VCM podr\u00eda mantenerse normal, pero la ferritina deber\u00eda afectarse, y desde luego no estar elevada."}, "2": {"exist": true, "char_ranges": [[415, 571]], "word_ranges": [[66, 96]], "text": "La 2: lo mismo. Si es por malabsorci\u00f3n de Fe, la ferritina deber\u00eda ser baja. Y si fuera por malabsorci\u00f3n de B12, el VCM nos lo pondr\u00edan claramente mas alto."}, "3": {"exist": true, "char_ranges": [[79, 244]], "word_ranges": [[12, 37]], "text": "Mujer mayor con anemia normocitica y patr\u00f3n ferrocinetico claro de trastorno cr\u00f3nico (hiposideremia pero hiperferritinemia). La \u00fanica que cuadra, y de lujo, es la 3;"}, "4": {"exist": true, "char_ranges": [[572, 612]], "word_ranges": [[96, 105]], "text": "La 4: 70 tacos y VCM normal. Esta sobra."}, "5": {"exist": false, "char_ranges": [], "word_ranges": [], "text": ""}}} {"id": 108, "year": 2012, "question_id_specific": 219, "full_question": "Un hombre presenta una deleci\u00f3n parcial en el gen de la distrofina (cromosoma Xp21) que le ocasiona la semiolog\u00eda propia de la distrofia muscular de Becker. Acude a la consulta de gen\u00e9tica con su esposa, para valorar los riesgos de transmisi\u00f3n de la enfermedad. \u00bfQu\u00e9 informaci\u00f3n correcta ser\u00e1 proporcionada en el transcurso del consejo gen\u00e9tico?", "full_answer": "Es el hombre quien presenta la enfermedad, que nos dicen que est\u00e1 ligada al cromosoma X, por lo que s\u00f3lo quedan dos opciones: 2 y 3. La 2 no es porque los varones recibir\u00e1n el cromosoma Y del padre y el X de la madre, que est\u00e1 sana, por lo que las hijas podr\u00e1n portar la enfermedad (Opci\u00f3n correcta la 3).", "type": "GEN\u00c9TICA E INMUNOLOG\u00cdA", "options": {"1": "Seg\u00fan la herencia autos\u00f3mica dominante, al mitad de sus hijos heredar\u00e1n la enfermedad y sin distinci\u00f3n de sexos.", "2": "Sus hijas no heredar\u00e1n la enfermedad, pero todos sus futuros hijos varones ser\u00e1n portadores y pueden transmitir la mutaci\u00f3n al 50%.", "3": "Sus hijos varones no heredar\u00e1n la enfermedad, pero todas sus futuras hijas ser\u00e1n portadoras y pueden transmitir la mutaci\u00f3n al 50%.", "4": "No hay riesgo: la herencia de la enfermedad es del tipo mitocondrial, nunca transmitida por los varones.", "5": "Seg\u00fan la herencia autos\u00f3mica recesiva, un 25% de sus hijos manifestar\u00e1n la enfermedad en la infancia, sin distinci\u00f3n de sexos."}, "correct_option": 3, "explanations": {"1": {"exist": false, "char_ranges": [], "word_ranges": [], "text": ""}, "2": {"exist": true, "char_ranges": [[0, 305]], "word_ranges": [[0, 61]], "text": "Es el hombre quien presenta la enfermedad, que nos dicen que est\u00e1 ligada al cromosoma X, por lo que s\u00f3lo quedan dos opciones: 2 y 3. La 2 no es porque los varones recibir\u00e1n el cromosoma Y del padre y el X de la madre, que est\u00e1 sana, por lo que las hijas podr\u00e1n portar la enfermedad (Opci\u00f3n correcta la 3)."}, "3": {"exist": true, "char_ranges": [[0, 305]], "word_ranges": [[0, 61]], "text": "Es el hombre quien presenta la enfermedad, que nos dicen que est\u00e1 ligada al cromosoma X, por lo que s\u00f3lo quedan dos opciones: 2 y 3. La 2 no es porque los varones recibir\u00e1n el cromosoma Y del padre y el X de la madre, que est\u00e1 sana, por lo que las hijas podr\u00e1n portar la enfermedad (Opci\u00f3n correcta la 3)."}, "4": {"exist": false, "char_ranges": [], "word_ranges": [], "text": ""}, "5": {"exist": false, "char_ranges": [], "word_ranges": [], "text": ""}}} {"id": 255, "year": 2014, "question_id_specific": 94, "full_question": "Mujer de 65 a\u00f1os de edad con antecedentes de dolores articulares en tratamiento con antiinflamatorios que es remitida a estudio por presentar anemia. En los estudios complementarios presenta hemat\u00edes 3.164.000, Hto. 32%, Hb 11g/dl, VCM 69 fl, Leucocitos 7800, Plaquetas 370.000, PCR 0,29 mg/dl, Fe 20ng/ml, Ferritina 18 ng/ml, Glucosa 105 mg/dl, GOT, GPT, GGT, F.Alcalina, Bilirrubina total, Colesterol, Creatinina, Calcio y F\u00f3sforo normales. Ac.antitransglutaminasa y Ac antigliadina negativos. Gastroscopia: hernia de hiato de 3 cm, resto sin alteraciones. Colonoscopia: hasta ciego, aislados divert\u00edculos en sigma. Tr\u00e1nsito intestinal sin alteraciones. Ecograf\u00eda de abdomen sin alteraciones. \u00bfCu\u00e1l de las exploraciones que se enumeran a continuaci\u00f3n le parece m\u00e1s adecuada para completar el estudio?", "full_answer": "Se trata de un estudio de anemia ferrop\u00e9nica con gastroscopia y colonoscopia normales, habi\u00e9ndose descartado posible celiaqu\u00eda. Nos faltan por ver yeyuno e \u00edleon, por lo que habr\u00e1 que realizar una c\u00e1psula endosc\u00f3pica. En una anemia ferrop\u00e9nica siempre hay que descartar causa digestiva de la misma.", "type": "APARATO DIGESTIVO", "options": {"1": "Biopsia de yeyuno.", "2": "Arteriograf\u00eda.", "3": "C\u00e1psula endosc\u00f3pica.", "4": "Radiois\u00f3topos.", "5": "RM p\u00e9lvica."}, "correct_option": 3, "explanations": {"1": {"exist": false, "char_ranges": [], "word_ranges": [], "text": ""}, "2": {"exist": false, "char_ranges": [], "word_ranges": [], "text": ""}, "3": {"exist": true, "char_ranges": [[0, 217]], "word_ranges": [[0, 33]], "text": "Se trata de un estudio de anemia ferrop\u00e9nica con gastroscopia y colonoscopia normales, habi\u00e9ndose descartado posible celiaqu\u00eda. Nos faltan por ver yeyuno e \u00edleon, por lo que habr\u00e1 que realizar una c\u00e1psula endosc\u00f3pica."}, "4": {"exist": false, "char_ranges": [], "word_ranges": [], "text": ""}, "5": {"exist": false, "char_ranges": [], "word_ranges": [], "text": ""}}} {"id": 303, "year": 2016, "question_id_specific": 111, "full_question": "En una paciente con artritis reumatoide en tratamiento con metotrexate, prednisona e indometacina que presenta de forma aguda edemas y aumento de la creatinina plasm\u00e1tica con un sedimento de orina poco expresivo y proteinuria inferior a 100 mg/24 h la causa m\u00e1s probable es:", "full_answer": "La amiloidosis cursa con proteinuria habitualmente de rango nefr\u00f3tico, y tiene un curso m\u00e1s cr\u00f3nico. Las GMN asociadas a la artritis reumatoide alteran de forma caracter\u00edstica el sedimento, y la insuficiencia renal por AINEs es de curso m\u00e1s cr\u00f3nico, con necrosis papilar. As\u00ed, lo m\u00e1s probable dado el curso y las alteraciones descritas es la nefritis intersticial por metotrexate, que es m\u00e1s t\u00f3xica directamente que al\u00e9rgica, y de ah\u00ed la ausencia de piuria, eosinofilia, fiebre u otros datos de hipersensibilidad.", "type": "NEFROLOG\u00cdA", "options": {"1": "Amiloidosis renal.", "2": "Glomerulonefritis secundaria a la artritis reumatoide.", "3": "Insuficiencia renal por antiinflamatorios no esteroideos.", "4": "Nefritis intersticial por metotrexate.", "5": NaN}, "correct_option": 4, "explanations": {"1": {"exist": true, "char_ranges": [[0, 100]], "word_ranges": [[0, 15]], "text": "La amiloidosis cursa con proteinuria habitualmente de rango nefr\u00f3tico, y tiene un curso m\u00e1s cr\u00f3nico."}, "2": {"exist": true, "char_ranges": [[101, 188]], "word_ranges": [[15, 28]], "text": "Las GMN asociadas a la artritis reumatoide alteran de forma caracter\u00edstica el sedimento,"}, "3": {"exist": true, "char_ranges": [[192, 271]], "word_ranges": [[29, 42]], "text": "la insuficiencia renal por AINEs es de curso m\u00e1s cr\u00f3nico, con necrosis papilar."}, "4": {"exist": true, "char_ranges": [[277, 513]], "word_ranges": [[43, 80]], "text": "lo m\u00e1s probable dado el curso y las alteraciones descritas es la nefritis intersticial por metotrexate, que es m\u00e1s t\u00f3xica directamente que al\u00e9rgica, y de ah\u00ed la ausencia de piuria, eosinofilia, fiebre u otros datos de hipersensibilidad."}, "5": {"exist": false, "char_ranges": [], "word_ranges": [], "text": ""}}} {"id": 539, "year": 2021, "question_id_specific": 3, "full_question": "Paciente de 45 a\u00f1os que presenta un primer episodio de lumbalgia de 3 semanas de evoluci\u00f3n que le impide realizar su vida normal. No refiere traumatismo ni otra patolog\u00eda subyacente. En la exploraci\u00f3n f\u00edsica no hay d\u00e9ficit neurol\u00f3gico. \u00bfQu\u00e9 prueba de imagen estar\u00eda indicada?:", "full_answer": "Un primer episodio de lumbalgia aguda (menos de 6 semanas de duraci\u00f3n), no requiere la realizaci\u00f3n de pruebas de imagen en ausencia de \u00abred flags\u00bb que puedan indicar un proceso subyacente, como una fractura, una infecci\u00f3n o un tumor maligno.", "type": "TRAUMATOLOG\u00cdA", "options": {"1": "Al ser el primer episodio de lumbalgia, \u00fanicamente una radiograf\u00eda simple de columna lumbar.", "2": "Resonancia magn\u00e9tica, que aporta m\u00e1s informaci\u00f3n sobre partes blandas y posibles herniaciones.", "3": "TC, para valorar mejor la estructura \u00f3sea y las posibles fracturas.", "4": "No hay indicaci\u00f3n de prueba de imagen.", "5": NaN}, "correct_option": 4, "explanations": {"1": {"exist": false, "char_ranges": [], "word_ranges": [], "text": ""}, "2": {"exist": false, "char_ranges": [], "word_ranges": [], "text": ""}, "3": {"exist": false, "char_ranges": [], "word_ranges": [], "text": ""}, "4": {"exist": true, "char_ranges": [[0, 241]], "word_ranges": [[0, 40]], "text": "Un primer episodio de lumbalgia aguda (menos de 6 semanas de duraci\u00f3n), no requiere la realizaci\u00f3n de pruebas de imagen en ausencia de \u00abred flags\u00bb que puedan indicar un proceso subyacente, como una fractura, una infecci\u00f3n o un tumor maligno."}, "5": {"exist": false, "char_ranges": [], "word_ranges": [], "text": ""}}} {"id": 408, "year": 2018, "question_id_specific": 190, "full_question": "Mujer de 45 diagnosticada de c\u00e1ncer de mama estadio IV con met\u00e1stasis \u00f3seas m\u00faltiples en tratamiento hormonal y bifosfonatos iv. Acude a su consulta refiriendo dolor \u00f3seo intenso (EVA=7), en varias localizaciones sin sintomatolog\u00eda neurol\u00f3gica asociada y exarcerbaciones del dolor (EVA=9) varias veces al dia que le obligan a permanecer en reposo. En el tratamiento con ibuprofeno (600 mg cada 8 h ) y tramadol (150 mg al d\u00eda). \u00bfQu\u00e9 tratamiento considerar\u00eda m\u00e1s adecuado en este paciente para mejorar el dolor?", "full_answer": "La respuesta correcta es la 4 porque si la paciente tiene met\u00e1stasis \u00f3seas y puede sufrir fracturas podemos controlar el dolor de tipo inflamatorio con el ibuprofeno y adem\u00e1s nos ir\u00e1 bien para utilizar la sinergia anti\u00e1lgica. Debemos cambiar el tratamiento a un opioide mayor y hacer tratamiento del dolor disrrupativo de forma en\u00e9rgica ya que est\u00e1 en casi el dolor m\u00e1ximo que puede soportar.", "type": "ATENCI\u00d3N PRIMARIA", "options": {"1": "Suspender ibuprofeno y tramadol e iniciar un opioide mayor como morfina u oxicodona.", "2": "Suspender tramadol, mantener ibuprofeno e iniciar un opioide mayor como morfina y oxicodona.", "3": "Suspender ibuprofeno y tramadol e iniciar un opioide mayor como morfina u oxicodona y a la vez iniciar tratamiento del dolor irruptivo con fentanilo sublingual", "4": "Suspender tramadol, mantener ibuprofeno e iniciar un opioide mayor como morfina y oxicodona y a la vez tratamiento para el dolor irruptivo con fentanilo sublingual.", "5": NaN}, "correct_option": 4, "explanations": {"1": {"exist": false, "char_ranges": [], "word_ranges": [], "text": ""}, "2": {"exist": false, "char_ranges": [], "word_ranges": [], "text": ""}, "3": {"exist": false, "char_ranges": [], "word_ranges": [], "text": ""}, "4": {"exist": true, "char_ranges": [[0, 392]], "word_ranges": [[0, 65]], "text": "La respuesta correcta es la 4 porque si la paciente tiene met\u00e1stasis \u00f3seas y puede sufrir fracturas podemos controlar el dolor de tipo inflamatorio con el ibuprofeno y adem\u00e1s nos ir\u00e1 bien para utilizar la sinergia anti\u00e1lgica. Debemos cambiar el tratamiento a un opioide mayor y hacer tratamiento del dolor disrrupativo de forma en\u00e9rgica ya que est\u00e1 en casi el dolor m\u00e1ximo que puede soportar."}, "5": {"exist": false, "char_ranges": [], "word_ranges": [], "text": ""}}} {"id": 29, "year": 2011, "question_id_specific": 60, "full_question": "Paciente de 71 a\u00f1os, fumador de 2 paquetes de cigarrillos/d\u00eda desde hace 50 a\u00f1os, con buena calidad de vida previa y sin otros antecedentes patol\u00f3gicos de inter\u00e9s que consulta por tos y expectoraci\u00f3n hemopt\u00f3ica. A la exploraci\u00f3n f\u00edsica no se aprecian datos patol\u00f3gicos significativos. En la radiograf\u00eda simple de t\u00f3rax se evidencia una masa de 4x3,4 cm inferior derecha, hallazgo que se confirma en la TC toracoabdominal que localiza la lesi\u00f3n en el l\u00f3bulo inferior y no halla adenopat\u00edas ni im\u00e1genes de met\u00e1stasis. La broncoscopia objetiva una lesi\u00f3n imfiltratnte en la pir\u00e1mide basal derecha cuya biopsia es de carcinoma epidermoide bien diferenciado. En cuanto a las pruebas de funci\u00f3n respiratoria la FVC es de 3.300 cc (84%) y la FEV 1 de 2.240 cc (80%) \u00bfCu\u00e1l deber\u00eda ser el siguiente paso a seguir en este paciente?", "full_answer": "1 y 5: Son pruebas diagn\u00f3sticas, no tienen sentido puesto que ya tienes el diagn\u00f3stico hecho. 2: La edad no debe ser un factor limitante, y menos teniendo en cuenta que son solo 71 a\u00f1os, pero sobre todo porque la \u201ccalidad de vida\u201d del paciente es buena (y entrecomillo \u201ccalidad de vida\u201d porque es un t\u00e9rmino utilizado err\u00f3neamente, se supone que quieren decir que mantiene un buen grado funcional). 3: Presenta una limitaci\u00f3n respiratoria muy leve, no requiere de dicha prueba. 4: Es la correcta.", "type": "NEUROLOG\u00cdA Y CIRUG\u00cdA TOR\u00c1CICA", "options": {"1": "Realizar una mediastinoscopia.", "2": "Debe contraindicarse la intervenci\u00f3n quir\u00fargica con intenci\u00f3n curativa por la edad avanzada del paciente.", "3": "Se debe llevar a cabo un test de consumo de ox\u00edgeno, dada la limitaci\u00f3n respiratoria que presenta.", "4": "Debe indicarse la resecci\u00f3n quir\u00fargica (lobectom\u00eda del l\u00f3bulo inferior del pulm\u00f3n derecho) de forma directa.", "5": "Debe realizarse una punci\u00f3n trantor\u00e1cica."}, "correct_option": 4, "explanations": {"1": {"exist": true, "char_ranges": [[7, 93]], "word_ranges": [[3, 16]], "text": "Son pruebas diagn\u00f3sticas, no tienen sentido puesto que ya tienes el diagn\u00f3stico hecho."}, "2": {"exist": true, "char_ranges": [[97, 398]], "word_ranges": [[17, 69]], "text": "La edad no debe ser un factor limitante, y menos teniendo en cuenta que son solo 71 a\u00f1os, pero sobre todo porque la \u201ccalidad de vida\u201d del paciente es buena (y entrecomillo \u201ccalidad de vida\u201d porque es un t\u00e9rmino utilizado err\u00f3neamente, se supone que quieren decir que mantiene un buen grado funcional)."}, "3": {"exist": true, "char_ranges": [[402, 477]], "word_ranges": [[70, 81]], "text": "Presenta una limitaci\u00f3n respiratoria muy leve, no requiere de dicha prueba."}, "4": {"exist": false, "char_ranges": [], "word_ranges": [], "text": ""}, "5": {"exist": true, "char_ranges": [[7, 93]], "word_ranges": [[3, 16]], "text": "Son pruebas diagn\u00f3sticas, no tienen sentido puesto que ya tienes el diagn\u00f3stico hecho."}}} {"id": 10, "year": 2011, "question_id_specific": 37, "full_question": "Una mujer de 76 a\u00f1os sin otros antecedentes que hipertensi\u00f3n arterial consulta por ictericia indolora y prurito con anorexia. Anal\u00edticamente destaca una bilirrubina de 12 mg/dl (con 9,5 de bilirrubina directa). La ecograf\u00eda abdominal muestra dilataci\u00f3n de la v\u00eda biliar intra y extrahep\u00e1tica, junto con un n\u00f3dulo \u00fanico de 2 cm localizado perif\u00e9ricamente en la cara anterior del l\u00f3bulo izquierdo. La Tc confirma lo hallazgos, demostrando adem\u00e1s la presencia de una masa en la cabeza del p\u00e1ncreas de 3,5 cm. La punci\u00f3n aspiraci\u00f3n con aguja fina confirma el diagn\u00f3stico de adenocarcinoma. Se\u00f1ale la mejor opci\u00f3n terap\u00e9utica:", "full_answer": "Creo que la respuesta correcta es la 4, aunque las dos lesiones son resecables el adenocarcinoma de p\u00e1ncreas con met\u00e1stasis hep\u00e1tica sincr\u00f3nicas no tiene indicaci\u00f3n quir\u00fargica debido a que no prolonga la supervivencia y tiene una morbilidad elevada. La derivaci\u00f3n quir\u00fargica tienen m\u00e1s complicaciones que colocar una pr\u00f3tesis met\u00e1lica autoexpandible que tiene una permeabilidad media de 6 meses que es similar a la supervivencia mediana de un paciente con met\u00e1stasis hep\u00e1ticas de adenocarcinoma de p\u00e1ncreas.", "type": "CIRUG\u00cdA", "options": {"1": "Quimio/radioterapia adyuvante, condicionando la cirug\u00eda radical a la respuesta inicial.", "2": "Drenaje externo percut\u00e1neo don car\u00e1cter paliativo, con eventual conversi\u00f3n a drenaje interno en caso de intolerancia o complicaciones.", "3": "Derivaci\u00f3n biliar quir\u00fargica, con o sin gastroyeyunostom\u00eda profil\u00e1ctica en funci\u00f3n de los hallazgos intraoperatorios.", "4": "Pr\u00f3tesis biliar met\u00e1lica mediante colangiograf\u00eda endosc\u00f3pica retr\u00f3grada, con opci\u00f3n a quimioterapia paliativa.", "5": "Duodenopancreatectom\u00eda cef\u00e1lica con ablaci\u00f3n por radiofrecuencia o alcoholizaci\u00f3n de la lesi\u00f3n hep\u00e1tica."}, "correct_option": 4, "explanations": {"1": {"exist": false, "char_ranges": [], "word_ranges": [], "text": ""}, "2": {"exist": false, "char_ranges": [], "word_ranges": [], "text": ""}, "3": {"exist": false, "char_ranges": [], "word_ranges": [], "text": ""}, "4": {"exist": true, "char_ranges": [[0, 249]], "word_ranges": [[0, 38]], "text": "Creo que la respuesta correcta es la 4, aunque las dos lesiones son resecables el adenocarcinoma de p\u00e1ncreas con met\u00e1stasis hep\u00e1tica sincr\u00f3nicas no tiene indicaci\u00f3n quir\u00fargica debido a que no prolonga la supervivencia y tiene una morbilidad elevada."}, "5": {"exist": false, "char_ranges": [], "word_ranges": [], "text": ""}}} {"id": 284, "year": 2016, "question_id_specific": 57, "full_question": "Mujer de 72 a\u00f1os que acude al hospital por dolor tor\u00e1cico opresivo de 2 horas de evoluci\u00f3n. La presi\u00f3n arterial es de 68/32 mm Hg, la frecuencia cardiaca es de 124 latidos/min, la frecuencia respiratoria es de 32 respiraciones/min, la saturaci\u00f3n de ox\u00edgeno del 91% con ox\u00edgeno al 50%.A la auscultaci\u00f3n pulmonar se aprecian crepitantes bilaterales. Se realiza un ECG que muestra lesi\u00f3n subepic\u00e1rdica en cara anterior, con imagen especular en cara inferior. En la radiograf\u00eda de t\u00f3rax se aprecia patr\u00f3n alveolar bilateral. \u00bfCu\u00e1l es su decisi\u00f3n terape\u00fatica?", "full_answer": "Realizar coronariografia urgente.", "type": "CARDIOLOG\u00cdA Y CIRUG\u00cdA VASCULAR", "options": {"1": "Iniciar infusi\u00f3n de nitratos i.v.", "2": "Realizar fibrinolisis con tenecteplase (TNK).", "3": "Realizar angioplastia coronaria urgente.", "4": "Administrar furosemida 40 mg i.v.", "5": NaN}, "correct_option": 3, "explanations": {"1": {"exist": false, "char_ranges": [], "word_ranges": [], "text": ""}, "2": {"exist": false, "char_ranges": [], "word_ranges": [], "text": ""}, "3": {"exist": true, "char_ranges": [[0, 33]], "word_ranges": [[0, 3]], "text": "Realizar coronariografia urgente."}, "4": {"exist": false, "char_ranges": [], "word_ranges": [], "text": ""}, "5": {"exist": false, "char_ranges": [], "word_ranges": [], "text": ""}}} {"id": 188, "year": 2013, "question_id_specific": 230, "full_question": "Un paciente de 57 a\u00f1os tuvo un infarto agudo de miocardio y presenta en la anal\u00edtica colesterol de 312 mg/dl, LDL 241 mg/dl, HDL 29 y TG normales. Fumador 30 cig/d\u00eda, no obesidad, presi\u00f3n arterial de 145/90 mm Hg y no ten\u00eda diabetes. Interrogado sobre antecedentes familiares refiri\u00f3 que un t\u00edo por l\u00ednea materna falleci\u00f3 de forma brusca a los 53 a\u00f1os. \u00bfCon qu\u00e9 respuesta est\u00e1 m\u00e1s de acuerdo?", "full_answer": "Pregunta de un tema de moda en los \u00faltimos a\u00f1os: las dislipemia; yo la contestar\u00eda por descarte. El tratamiento de la hipercolesterolemia son las estatinas por lo que descartamos la respuesta 2 y 4. La estatina en un paciente con enfermedad cardiovascular (como es un IAM) es un tratamiento de por vida (descartamos la 1) ya que no se toma hasta que se normalice los niveles de LDL. Entre la 3 y la 5, la 5 es m\u00e1s completa ya que incluye el tratamiento y el control de otros factores de riesgo. Descartar una dislipemia secundaria es f\u00e1cil con la historia cl\u00ednica del paciente y una anal\u00edtica con funci\u00f3n tiroidea.", "type": "ENDOCRINOLOG\u00cdA", "options": {"1": "Tratar con estatinas hasta que se normalice el colesterol y recomendar h\u00e1bito de vida saludable (dieta, ejercicio y no fumar).", "2": "Descartar hipotiroidismo u otra enfermedad sist\u00e9mica, recomendar h\u00e1bito de vida saludable y tratar con fibratos para normalizar el colesterol.", "3": "No precisa descartar dislipemia secundaria por ser infrecuente. Se debe realizar perfil lip\u00eddico familiar.", "4": "Descartar dislipemia secundaria, tratar con resinas (resincolestiramina) y fibratos y re-comendar h\u00e1bito de vida saludable. Prohibir alcohol.", "5": "Descartar dislipemia secundaria, solicitar estudio familiar y tratar con estatinas para mantener un buen perfil lip\u00eddico. Control de otros factores de riesgo."}, "correct_option": 5, "explanations": {"1": {"exist": true, "char_ranges": [[199, 382]], "word_ranges": [[34, 68]], "text": "La estatina en un paciente con enfermedad cardiovascular (como es un IAM) es un tratamiento de por vida (descartamos la 1) ya que no se toma hasta que se normalice los niveles de LDL."}, "2": {"exist": true, "char_ranges": [[97, 198]], "word_ranges": [[17, 34]], "text": "El tratamiento de la hipercolesterolemia son las estatinas por lo que descartamos la respuesta 2 y 4."}, "3": {"exist": true, "char_ranges": [[383, 494]], "word_ranges": [[68, 92]], "text": "Entre la 3 y la 5, la 5 es m\u00e1s completa ya que incluye el tratamiento y el control de otros factores de riesgo."}, "4": {"exist": true, "char_ranges": [[97, 198]], "word_ranges": [[17, 34]], "text": "El tratamiento de la hipercolesterolemia son las estatinas por lo que descartamos la respuesta 2 y 4."}, "5": {"exist": true, "char_ranges": [[383, 494]], "word_ranges": [[68, 92]], "text": "Entre la 3 y la 5, la 5 es m\u00e1s completa ya que incluye el tratamiento y el control de otros factores de riesgo."}}} {"id": 349, "year": 2016, "question_id_specific": 160, "full_question": "Mujer de 45 a\u00f1os, madre de 3 hijos, acude a consulta de diagn\u00f3stico precoz. La citolog\u00eda cervicovaginal es compatible con lesi\u00f3n escamosa intraepitelial de alto grado. \u00bfCu\u00e1l de las siguientes opciones elegir\u00eda?", "full_answer": "La respuesta correcta es la 2. Seg\u00fan la Oncogu\u00eda de la SEGO prevenci\u00f3n de c\u00e1ncer de cuello de \u00fatero de 2014, ante una citolog\u00eda con una lesi\u00f3n escamosa intraepitelial de alto grado, la actuaci\u00f3n a seguir es una colposcopia, ya con toma de biopsia \u00f3 no en funci\u00f3n de los hallazgos (independientemente de la edad de la paciente y la paridad).", "type": "GINECOLOG\u00cdA Y OBSTETRICIA", "options": {"1": "Repetir la citolog\u00eda en 1 mes.", "2": "Colposcopia con eventual biopsia.", "3": "Histerectom\u00eda con salpinguectom\u00eda bilateral y conservaci\u00f3n de ovarios.", "4": "Legrado uterino fraccionado.", "5": NaN}, "correct_option": 2, "explanations": {"1": {"exist": false, "char_ranges": [], "word_ranges": [], "text": ""}, "2": {"exist": true, "char_ranges": [[31, 340]], "word_ranges": [[6, 61]], "text": "Seg\u00fan la Oncogu\u00eda de la SEGO prevenci\u00f3n de c\u00e1ncer de cuello de \u00fatero de 2014, ante una citolog\u00eda con una lesi\u00f3n escamosa intraepitelial de alto grado, la actuaci\u00f3n a seguir es una colposcopia, ya con toma de biopsia \u00f3 no en funci\u00f3n de los hallazgos (independientemente de la edad de la paciente y la paridad)."}, "3": {"exist": false, "char_ranges": [], "word_ranges": [], "text": ""}, "4": {"exist": false, "char_ranges": [], "word_ranges": [], "text": ""}, "5": {"exist": false, "char_ranges": [], "word_ranges": [], "text": ""}}} {"id": 290, "year": 2016, "question_id_specific": 50, "full_question": "Un hombre de 58 a\u00f1os refer\u00eda una historia de 3 semanas de evoluci\u00f3n con lesiones cut\u00e1neas progresivas, medianamente dolorosas, en su brazo izquierdo. Hab\u00eda empezado como una lesi\u00f3n eritematosa en su pulgar izquierdo. Ten\u00eda unas estr\u00edas rojizas visibles como l\u00edneas de conexi\u00f3n entre las lesiones. El paciente no presentaba fiebre ni otros s\u00edntomas generales. Hab\u00eda estado trabajando en su jard\u00edn pero no recordaba haberse hecho ninguna herida. El diagn\u00f3stico etiol\u00f3gico se realiz\u00f3 por cultivo de una biopsia cut\u00e1nea. \u00bfCu\u00e1l es el agente causal m\u00e1s probable de este proceso?", "full_answer": "Las lesiones con patr\u00f3n esporotricoide o linfocut\u00e1neo que se describen pueden corresponder a una esporotricosis o a un granuloma de los acuarios, de modo que las dos primeras quedar\u00edan descartadas. El antecedente de haber estado trabajando en el jard\u00edn orienta m\u00e1s a una esporotricosis, aunque despista un poco el hecho de que la pregunta pide por el \u201cagente causal\u201d y no el nombre de la enfermedad, de modo que si consideramos la esporotricosis en realidad la respuesta deber\u00eda ser Sporothrix schenckii Pese a ello, a sabiendas que esta pregunta ser\u00e1 correcta para muchos examinados aunque est\u00e9 mal redactada -lo cual la anula directamente- no se promover\u00e1 la impugnaci\u00f3n.", "type": "DERMATOLOG\u00cdA, VENEREOLOG\u00cdA Y CIRUG\u00cdA PL\u00c1STICA", "options": {"1": "Dermatofitosis por Microsporum gypseum.", "2": "Infecci\u00f3n cut\u00e1nea por Staphylococcus aureus.", "3": "Esporotricosis.", "4": "Infecci\u00f3n cut\u00e1nea por Mycobacterium marinum.", "5": NaN}, "correct_option": 3, "explanations": {"1": {"exist": true, "char_ranges": [[0, 197]], "word_ranges": [[0, 30]], "text": "Las lesiones con patr\u00f3n esporotricoide o linfocut\u00e1neo que se describen pueden corresponder a una esporotricosis o a un granuloma de los acuarios, de modo que las dos primeras quedar\u00edan descartadas."}, "2": {"exist": true, "char_ranges": [[0, 197]], "word_ranges": [[0, 30]], "text": "Las lesiones con patr\u00f3n esporotricoide o linfocut\u00e1neo que se describen pueden corresponder a una esporotricosis o a un granuloma de los acuarios, de modo que las dos primeras quedar\u00edan descartadas."}, "3": {"exist": true, "char_ranges": [[198, 285]], "word_ranges": [[30, 44]], "text": "El antecedente de haber estado trabajando en el jard\u00edn orienta m\u00e1s a una esporotricosis,"}, "4": {"exist": true, "char_ranges": [[198, 285]], "word_ranges": [[30, 44]], "text": "El antecedente de haber estado trabajando en el jard\u00edn orienta m\u00e1s a una esporotricosis,"}, "5": {"exist": false, "char_ranges": [], "word_ranges": [], "text": ""}}} {"id": 305, "year": 2016, "question_id_specific": 174, "full_question": "Hombre de 87 a\u00f1os con antecedentes de hipertensi\u00f3n y gonartrosis. Situaci\u00f3n basal con autonom\u00eda funcional y cognitiva plena que le permite continuar viviendo solo en la comunidad. Realiza tratamiento habitual con perindopril y diur\u00e9tico tiaz\u00eddico para control de la presi\u00f3n arterial y toma de forma rutinaria ibuprofeno 1800 mg/d\u00eda para control de los s\u00edntomas derivados de su gonartrosis. Tras control rutinario se objetiva, de forma persistente, una presi\u00f3n arterial de 190 y TAD 80 mmHg. \u00bfCu\u00e1l ser\u00eda la modificaci\u00f3n terap\u00e9utica m\u00e1s razonable para conseguir el control de las cifras tensionales?", "full_answer": "Uno de los mecanismos que producen el efecto hipotensor de los IECAs como el perindopril es el aumento de la acci\u00f3n de las prostaglan-dinas vasodilatadores sobre el ri\u00f1\u00f3n, lo que disminuye la retenci\u00f3n hidrosalina y por tanto aumenta la diuresis y disminuye la tensi\u00f3n arterial. Los AINEs inhiben la s\u00edntesis de pros-taglandinas y disminuyen la perfusi\u00f3n renal, por lo que antagonizan el efecto hipotensor de muchos antihipertensivos, incluyendo los IECAs. Por eso, lo preferible en este paciente, antes de aumentar la medicaci\u00f3n, es cambiar el ibuprofeno por paracetamol, que no tiene este efecto, y volver a evaluarlo posteriormente.", "type": "NEFROLOG\u00cdA", "options": {"1": "A\u00f1adir\u00eda un bloqueante del calcio.", "2": "Aumentar\u00eda la dosis de hidroclorotiazida a 25 mg/d\u00eda.", "3": "Cambiar\u00eda el ibuprofeno por paracetamol para evitar la posible influencia del mismo sobre el efecto de los hipotensores.", "4": "A\u00f1adir\u00eda un bloqueante alfa por la elevada prevalencia de s\u00edndrome prost\u00e1tico en varones de esta edad.", "5": NaN}, "correct_option": 3, "explanations": {"1": {"exist": false, "char_ranges": [], "word_ranges": [], "text": ""}, "2": {"exist": false, "char_ranges": [], "word_ranges": [], "text": ""}, "3": {"exist": true, "char_ranges": [[279, 635]], "word_ranges": [[45, 98]], "text": "Los AINEs inhiben la s\u00edntesis de pros-taglandinas y disminuyen la perfusi\u00f3n renal, por lo que antagonizan el efecto hipotensor de muchos antihipertensivos, incluyendo los IECAs. Por eso, lo preferible en este paciente, antes de aumentar la medicaci\u00f3n, es cambiar el ibuprofeno por paracetamol, que no tiene este efecto, y volver a evaluarlo posteriormente."}, "4": {"exist": false, "char_ranges": [], "word_ranges": [], "text": ""}, "5": {"exist": false, "char_ranges": [], "word_ranges": [], "text": ""}}} {"id": 104, "year": 2012, "question_id_specific": 91, "full_question": "Hombre de 34 a\u00f1os remitido a la consulta por presentar cifas de colesterol LDL superiores a 250 mg/dl, de forma persistente. El paciente se encuentra asintom\u00e1tico pero en la exploraci\u00f3n de aprecia la presencia de manchas induradas blanquecinas en codos, rodillas y p\u00e1rpados. Su TA es de 135/85 mmHg. Su padre muri\u00f3 de infarto de miocardio a la edad de 48 a\u00f1os. En la anal\u00edtica, realizada en ayunas, destaca un colesterol total de 346 mg/dl, un LDL colesterol de 278 mg/dl, un HDL colesterol de 42 mg/dl, unos triglic\u00e9ridos de 130 mg/dl y una glucosa de 93 mg/dl. Respecto a la enfermedad que padece este paciente, una de las siguientes afirmaciones es FALSA. Se\u00f1ale cu\u00e1l:", "full_answer": "Se trata de una hipercolesterolemia familiar heterocig\u00f3tica. Hay que saber reconocer la enfermedad y conocer los criterios. Me parece dificultad alta.", "type": "ENDOCRINOLOG\u00cdA", "options": {"1": "Se trata de una enfermedad hereditaria autos\u00f3mica codominante, con una alta penetrancia, por lo que es frecuente que alguno de los progenitores y hermanos del individuo la padezcan tambi\u00e9n.", "2": "El trastorno gen\u00e9tico afecta al gen que codifica el receptor del LDL colesterol y se han descrito m\u00e1s de 900 mutaciones.", "3": "El trastorno gen\u00e9tico afecta tambi\u00e9n a la s\u00edntesis de los triglic\u00e9ridos, provocando en las fases avanzadas de la enfermedad, un incremento de los mismos, que suele ser superior a los 500 mg/dl.", "4": "Los varones no tratados tienen una probabilidad cercana al 50% de sufrir un evento coronario antes de los 60 a\u00f1os de edad, y las mujeres con la enfermedad presentan una prevalencia de cardiopat\u00eda isqu\u00e9mica superior a la de la poblaci\u00f3n general femenina.", "5": "Las estatinas son eficaces en el tratamiento de la forma heterocigota de la enfermedad, pero a menudo necesitan combinarse con inhibidores de la absorcio\u0301n de colesterol para alcanzar los objetivos terape\u0301uticos y el control de la enfermedad."}, "correct_option": 3, "explanations": {"1": {"exist": false, "char_ranges": [], "word_ranges": [], "text": ""}, "2": {"exist": false, "char_ranges": [], "word_ranges": [], "text": ""}, "3": {"exist": true, "char_ranges": [[0, 123]], "word_ranges": [[0, 17]], "text": "Se trata de una hipercolesterolemia familiar heterocig\u00f3tica. Hay que saber reconocer la enfermedad y conocer los criterios."}, "4": {"exist": false, "char_ranges": [], "word_ranges": [], "text": ""}, "5": {"exist": false, "char_ranges": [], "word_ranges": [], "text": ""}}} {"id": 38, "year": 2011, "question_id_specific": 232, "full_question": "Mujer de 52 a\u00f1os que consulta por haber notado durante la semana previa coloraci\u00f3n amarillenta de conjuntivas. No refiere conductas sexuales de riesgo ni antecedentes epidemiol\u00f3gicos de riesgo de hepatitis v\u00edrica. No consume alcohol ni f\u00e1rmacos hepatot\u00f3xicos. Relata historia de un a\u00f1o de evoluci\u00f3n de prurito generalizado astenia sequedad de boca y ausencia de lagrimeo de causa no filiada. Resto de la anamnesis sin datos patol\u00f3gicos. En la exploraci\u00f3n f\u00edsica se observan lesiones de rascado ictericia conjuntival y hepatomegalia no dolorosa. Aporta anal\u00edtica realizada en su empresa con los siguientes resultados patol\u00f3gicos: bilirrubina total 3 mg/dl fosfatasa alcalina 400 UI velocidad de sedimentaci\u00f3n 40 mm a la primera hora. Indique cual ser\u00eda la recomendaci\u00f3n m\u00e1s acertada para establecer el diagn\u00f3stico etiol\u00f3gico del cuadro que presenta la paciente:", "full_answer": "Una mujer de mediana edad con s\u00edntomas similares al Sj\u00f6gren (ausencia de lagrimeo, sequedad de boca), con hiperbilirrubinemia, ictericia y hepatomegalia, nos hace pensar en una cirrosis biliar primaria. Para diagnosticarlo utilizaremos anticuerpos antimitocondriales (AMA IgG). Por lo tanto, ser\u00eda la respuesta 1.", "type": "OFTALMOLOG\u00cdA", "options": {"1": "Anticuerpos antimitocondriales.", "2": "Estudio del metabolismo del hierro.", "3": "Estudio del metabolismo del cobre.", "4": "Resonancia magn\u00e9tica hep\u00e1tica.", "5": "Serolog\u00eda de virus B y virus C."}, "correct_option": 1, "explanations": {"1": {"exist": true, "char_ranges": [[0, 277]], "word_ranges": [[0, 36]], "text": "Una mujer de mediana edad con s\u00edntomas similares al Sj\u00f6gren (ausencia de lagrimeo, sequedad de boca), con hiperbilirrubinemia, ictericia y hepatomegalia, nos hace pensar en una cirrosis biliar primaria. Para diagnosticarlo utilizaremos anticuerpos antimitocondriales (AMA IgG)."}, "2": {"exist": false, "char_ranges": [], "word_ranges": [], "text": ""}, "3": {"exist": false, "char_ranges": [], "word_ranges": [], "text": ""}, "4": {"exist": false, "char_ranges": [], "word_ranges": [], "text": ""}, "5": {"exist": false, "char_ranges": [], "word_ranges": [], "text": ""}}} {"id": 376, "year": 2016, "question_id_specific": 130, "full_question": "Hombre de 39 a\u00f1os de edad que en los 3 \u00faltimos meses ha tenido dos episodios de movimientos saltatorios amplios e involuntarios de miembros izquierdos, que en unos dos minutos llevan a p\u00e9rdida de conciencia. En el estudio neuroradiol\u00f3gico se ha encontrado una lesi\u00f3n expansiva frontal derecha sugestiva de glioma cerebral.\u00bfC\u00f3mo clasificar\u00eda la epilepsia de este paciente?", "full_answer": "Hacen referencia a una crisis parcial que se sigue de p\u00e9rdida de nivel de consciencia, por lo que senso estricto ser\u00eda una crisis parcial compleja sintom\u00e1tica, no una crisis parcial secundariamente generalizada (que esto se da cuando los movimientos t\u00f3nico-cl\u00f3nicos se extienden al resto de extremidades). Por este motivo la opci\u00f3n m\u00e1s correcta ser\u00eda la dos porque no especifica que es una crisis parcial simple sintom\u00e1tica, sino que deja abierta la posibilidad de que se trate de una crisis parcial simple o compleja sintom\u00e1tica.", "type": "NEUROLOG\u00cdA", "options": {"1": "Generalizada sintom\u00e1tica.", "2": "Parcial sintom\u00e1tica.", "3": "Parcial criptog\u00e9nica.", "4": "Parcial secundariamente generalizada sintom\u00e1tica.", "5": NaN}, "correct_option": 2, "explanations": {"1": {"exist": false, "char_ranges": [], "word_ranges": [], "text": ""}, "2": {"exist": true, "char_ranges": [[322, 530]], "word_ranges": [[49, 84]], "text": "la opci\u00f3n m\u00e1s correcta ser\u00eda la dos porque no especifica que es una crisis parcial simple sintom\u00e1tica, sino que deja abierta la posibilidad de que se trate de una crisis parcial simple o compleja sintom\u00e1tica."}, "3": {"exist": false, "char_ranges": [], "word_ranges": [], "text": ""}, "4": {"exist": true, "char_ranges": [[0, 305]], "word_ranges": [[0, 46]], "text": "Hacen referencia a una crisis parcial que se sigue de p\u00e9rdida de nivel de consciencia, por lo que senso estricto ser\u00eda una crisis parcial compleja sintom\u00e1tica, no una crisis parcial secundariamente generalizada (que esto se da cuando los movimientos t\u00f3nico-cl\u00f3nicos se extienden al resto de extremidades)."}, "5": {"exist": false, "char_ranges": [], "word_ranges": [], "text": ""}}} {"id": 163, "year": 2013, "question_id_specific": 122, "full_question": "Hombre de 66 a\u00f1os con antecedente de espondilitis anquilosante de larga evoluci\u00f3n. Presenta proteinuria de 6 gramos al d\u00eda con hipoalbuminemia y edemas. En el sedimento urinario no se detecta hematuria. Su creatinina plasm\u00e1tica es de 1,6 mg/dL y su filtrado glomerular de 45 mL/min. Glucemia 110 mg/dl. Su severa deformidad de columna vertebral dificulta realizar biopsia renal percut\u00e1nea. \u00bfQu\u00e9 actitud inicial es la correcta?", "full_answer": "La respuesta correcta es: 3. Biopsia de grasa subcut\u00e1nea. El lupus eritematoso, algunas vasculitis, la enfermedad de Goodpasture y otras enfermedades sist\u00e9micas (la espondilitis anquilosante es una de ellas) afectan al ri\u00f1\u00f3n y condicionan su pron\u00f3stico. La aparici\u00f3n de anomal\u00edas urinarias o deterioro renal de causa parenquimatosa son indicaci\u00f3n de biopsia renal, incluso con rangos de proteinuria inferiores a los aceptados en las nefropat\u00edas primarias. La determinaci\u00f3n de anticuerpos anti-MBG y ANCAS ayudan para el diagn\u00f3stico, pero no sustituyen a la biopsia renal, ya que carecen de valor pron\u00f3stico y no ayudan a planificar el tratamiento. En este caso, en el que nos dirigen a pensar que la biopsia renal no es posible, debemos relacionar EA con la insuficiencia renal. En la mayor\u00eda de los casos, la relaci\u00f3n se establece por una Nefropat\u00eda IgA, que cursa con hematuria, s\u00edntoma que dice no existir en este caso. Una nada despreciable cantidad de pacientes con EA evolucionada es la amiloidosis, que cuadra tambi\u00e9n con la sintomatolog\u00eda del paciente. Por tanto, y dado que la amiloidosis se puede diagnosticar con una biopsia de grasa subcut\u00e1nea, es la respuesta 3 la que debemos dar por v\u00e1lida.", "type": "NEFROLOG\u00cdA", "options": {"1": "Iniciar corticoides por sospecha de enfermedad glomerular a cambios m\u00ednimos.", "2": "Tratarle con ciclofosfamida por sospechar glomerulopat\u00eda membranosa.", "3": "Biopsia de grasa subcut\u00e1nea.", "4": "Iniciar di\u00e1lisis.", "5": "Realizar una prueba de sobrecarga con glucosa para descartar nefropat\u00eda diab\u00e9tica."}, "correct_option": 3, "explanations": {"1": {"exist": false, "char_ranges": [], "word_ranges": [], "text": ""}, "2": {"exist": false, "char_ranges": [], "word_ranges": [], "text": ""}, "3": {"exist": true, "char_ranges": [[1074, 1205]], "word_ranges": [[169, 192]], "text": "dado que la amiloidosis se puede diagnosticar con una biopsia de grasa subcut\u00e1nea, es la respuesta 3 la que debemos dar por v\u00e1lida."}, "4": {"exist": false, "char_ranges": [], "word_ranges": [], "text": ""}, "5": {"exist": false, "char_ranges": [], "word_ranges": [], "text": ""}}} {"id": 148, "year": 2012, "question_id_specific": 158, "full_question": "Una mujer de 23 a\u00f1os acude a urgencias por encontrarse muy nerviosa tras una discusi\u00f3n con su parea. En su historia cl\u00ednica se reflejan varias demandas semejantes en el a\u00f1o anterior, en dos de ellas tras un gesto autol\u00edtico. Se constatan tambi\u00e9n conflictos frecuentes en las relaciones de pareja, cambios laborales y discusiones familiares. Dice sentirse incomprendida por todos incluidos los psiquiatras que la atienden. El diagn\u00f3stico es:", "full_answer": "La mujer tiene 23 a\u00f1os y est\u00e1 cabreada. Le hace falta apoyo y bastante educaci\u00f3n afectiva para frenar su impulsividad\u2026 se siente incomprendida\u2026 a lo mejor tiene raz\u00f3n\u2026 El tema esta en que no soporta el rechazo y tiene gestos autol\u00edticos. El diagn\u00f3stico es el 1. El 3,4 y 5 ni por asomo, no hay cl\u00ednica descrita en la pregunta. Luego en la vida real explorando bien casi seguro que habr\u00eda s\u00edntomas disociativos y de despersonalizaci\u00f3n, as\u00ed como s\u00edntomas depresivos\u2026 pero s\u00edntoma no es trastorno. \u00bfPor que no el 2? Podr\u00eda ser tambi\u00e9n puntualizando mucho e hilando muy fino\u2026 aqu\u00ed la discusi\u00f3n est\u00e1 servida. Os pongo los criterios de cada uno y lo hablamos,\u2026 pero la intenci\u00f3n del examinador era la 1 y la veo bien defendida. No se puede impugnar. \u2022 El l\u00edmite: Un patr\u00f3n general de inestabilidad en las relaciones interpersonales, la autoimagen y la efectividad, y una notable impulsividad, que comienzan al principio de la edad adulta y se dan en diversos contextos, como lo indican cinco (o m\u00e1s) de los siguientes \u00edtems: 1. Esfuerzos fren\u00e9ticos para evitar un abandono real o imaginado. Nota: No incluir los comportamientos suicidas o de automutilaci\u00f3n que se recogen en el Criterio 5. 2. Un patr\u00f3n de relaciones interpersonales inestables e intensas caracterizado por la alternancia entre los extremos de idealizaci\u00f3n y devaluaci\u00f3n. 3. Alteraci\u00f3n de la identidad: autoimagen o sentido de s\u00ed mismo acusada y persistentemente inestable. 4. Impulsividad en al menos dos \u00e1reas, que es potencialmente da\u00f1ina para s\u00ed mismo (p. ej., gastos, sexo, abuso de sustancias, conducci\u00f3n temeraria, atracones de comida). Nota: No incluir los comportamientos suicidas o de automutilaci\u00f3n que se recogen en el Criterio 5. 5. Comportamientos, intentos o amenazas suicidas recurrentes, o comportamiento de automutilaci\u00f3n. 6. Inestabilidad afectiva debida a una notable reactividad del estado de \u00e1nimo (p. ej., episodios de intensa disforia, irritabilidad o ansiedad, que suelen durar unas horas y rara vez unos d\u00edas). 7. Sentimientos cr\u00f3nicos de vac\u00edo. 8. Ira inapropiada e intensa o dificultades para controlar la ira (p. ej., muestras frecuentes de mal genio, enfado constante, peleas f\u00edsicas recurrentes). 9. Ideaci\u00f3n paranoide transitoria relacionada con el estr\u00e9s o s\u00edntomas disociativos graves. \u2022 El histri\u00f3nico: Un patr\u00f3n general de excesiva emotividad y una b\u00fasqueda de atenci\u00f3n, que empiezan al principio de la edad adulta y que se dan en diversos contextos, como lo indican cinco (o m\u00e1s) de los siguientes \u00edtems: 1. No se siente c\u00f3modo en las situaciones en las que no es el centro de la atenci\u00f3n. 2. La interacci\u00f3n con los dem\u00e1s suele estar caracterizada por un comportamiento sexualmente seductor o provocador. 3. Muestra una expresi\u00f3n emocional superficial y r\u00e1pidamente cambiante. 4. Utiliza permanentemente el aspecto f\u00edsico para llamar la atenci\u00f3n sobre s\u00ed mismo. 5. Tiene una forma de hablar excesivamente subjetiva y carente de matices. 6. Muestra autodramatizaci\u00f3n, teatralidad y exagerada expresi\u00f3n emocional. 7. Es sugestionable, por ejemplo, f\u00e1cilmente influenciable por los dem\u00e1s o por las circunstancias. 8. considera sus relaciones m\u00e1s \u00edntimas de lo que son en realidad.", "type": "PSIQUIATR\u00cdA", "options": {"1": "Trastorno de la personalidad l\u00edmite.", "2": "Trastorno de la personalidad histri\u00f3nico.", "3": "Distimia.", "4": "Trastorno disociativo.", "5": "Trastorno de despersonalizaci\u00f3n."}, "correct_option": 1, "explanations": {"1": {"exist": true, "char_ranges": [[0, 261]], "word_ranges": [[0, 46]], "text": "La mujer tiene 23 a\u00f1os y est\u00e1 cabreada. Le hace falta apoyo y bastante educaci\u00f3n afectiva para frenar su impulsividad\u2026 se siente incomprendida\u2026 a lo mejor tiene raz\u00f3n\u2026 El tema esta en que no soporta el rechazo y tiene gestos autol\u00edticos. El diagn\u00f3stico es el 1."}, "2": {"exist": false, "char_ranges": [], "word_ranges": [], "text": ""}, "3": {"exist": true, "char_ranges": [[262, 326]], "word_ranges": [[46, 60]], "text": "El 3,4 y 5 ni por asomo, no hay cl\u00ednica descrita en la pregunta."}, "4": {"exist": true, "char_ranges": [[262, 326]], "word_ranges": [[46, 60]], "text": "El 3,4 y 5 ni por asomo, no hay cl\u00ednica descrita en la pregunta."}, "5": {"exist": true, "char_ranges": [[262, 326]], "word_ranges": [[46, 60]], "text": "El 3,4 y 5 ni por asomo, no hay cl\u00ednica descrita en la pregunta."}}} {"id": 178, "year": 2013, "question_id_specific": 94, "full_question": "Ni\u00f1o de 18 meses de edad, con calendario vacu\u00adnal completo hasta la fecha, que consulta en el Servicio de urgencias por tumefacci\u00f3n de la rodilla derecha tras jugar en el parque, sin traumatismo evidente. En la anamnesis dirigida, la madre refiere que un t\u00edo de ella ten\u00eda proble\u00ad mas similares. La exploraci\u00f3n ecogr\u00e1fica es compatible con hemartros y en la anal\u00edtica que se realiza s\u00f3lo destaca un alargamiento del APTT de 52\u2033 (normal 25-35\u2033). \u00bfCu\u00e1l es la hip\u00f3tesis diagn\u00f3stica m\u00e1s probable?", "full_answer": "En esta pregunta tampoco paran de soltar pistas para lanzarnos de cabeza al diagn\u00f3stico correcto. Repasemos: ni\u00f1o peque\u00f1o que sin traumatismo claro sufre un hemartros. Un t\u00edo suyo ten\u00eda problemas similares, por lo que sospechamos enfermedad hereditaria. El TTPA, que eval\u00faa la v\u00eda intr\u00ednseca de la coagulaci\u00f3n (en la que participan los factores V, VIII, IX, X, XI y XII) est\u00e1 alargado. Problema hereditario de coagulaci\u00f3n. Descartamos as\u00ed la respuestas 1, 2 y 5. Nos quedan la enfermedad de von Willebrand y la hemofilia A. Considero que la correcta es la 4, hemofilia A, ya que el patr\u00f3n de herencia es ligado a X y su manifestaci\u00f3n cl\u00ednica principal son los hematomas de tejidos blandos y los hemartros espont\u00e1neos, sin traumatismo aparente, mientras que la enfermedad de von Willebrand se caracteriza por hemorragia tras cirug\u00eda o traumatismo y es caracter\u00edstico un tiempo de sangr\u00eda prolongado con una cifra de plaquetas normales (datos que no nombra el enunciado).", "type": "HEMATOLOG\u00cdA", "options": {"1": "S\u00edndrome de Marfan.", "2": "Enfermedad de Von Willebrand.", "3": "Enfermedad de Ehlers-Danlos.", "4": "Hemofilia A.", "5": "Enfermedad de Bemard-Soulier."}, "correct_option": 4, "explanations": {"1": {"exist": true, "char_ranges": [[254, 462]], "word_ranges": [[37, 74]], "text": "El TTPA, que eval\u00faa la v\u00eda intr\u00ednseca de la coagulaci\u00f3n (en la que participan los factores V, VIII, IX, X, XI y XII) est\u00e1 alargado. Problema hereditario de coagulaci\u00f3n. Descartamos as\u00ed la respuestas 1, 2 y 5."}, "2": {"exist": true, "char_ranges": [[254, 462]], "word_ranges": [[37, 74]], "text": "El TTPA, que eval\u00faa la v\u00eda intr\u00ednseca de la coagulaci\u00f3n (en la que participan los factores V, VIII, IX, X, XI y XII) est\u00e1 alargado. Problema hereditario de coagulaci\u00f3n. Descartamos as\u00ed la respuestas 1, 2 y 5."}, "3": {"exist": true, "char_ranges": [[744, 933]], "word_ranges": [[122, 151]], "text": "mientras que la enfermedad de von Willebrand se caracteriza por hemorragia tras cirug\u00eda o traumatismo y es caracter\u00edstico un tiempo de sangr\u00eda prolongado con una cifra de plaquetas normales"}, "4": {"exist": true, "char_ranges": [[524, 742]], "word_ranges": [[85, 122]], "text": "Considero que la correcta es la 4, hemofilia A, ya que el patr\u00f3n de herencia es ligado a X y su manifestaci\u00f3n cl\u00ednica principal son los hematomas de tejidos blandos y los hemartros espont\u00e1neos, sin traumatismo aparente,"}, "5": {"exist": true, "char_ranges": [[254, 462]], "word_ranges": [[37, 74]], "text": "El TTPA, que eval\u00faa la v\u00eda intr\u00ednseca de la coagulaci\u00f3n (en la que participan los factores V, VIII, IX, X, XI y XII) est\u00e1 alargado. Problema hereditario de coagulaci\u00f3n. Descartamos as\u00ed la respuestas 1, 2 y 5."}}} {"id": 100, "year": 2012, "question_id_specific": 85, "full_question": "Hombre de 24 a\u00f1os que refiere astenia progresiva desde al menos 6 meses, voz ronca, lentitud del habla, somnolencia e hinchaz\u00f3n de manos, pies y cara. Exploraci\u00f3n: pulso a 52 latidos por minutos, cara abotargada y piel seca y p\u00e1lida. Anal\u00edtica: Ligera anemia, colesterol 385 mg/dL (normal <220), creatinina 1,3 mg/dL (normal: 0,5-1,1), proteinuria negativa, TSH 187\u03bcUI/mL (normal 0,35-5,5) y T4 Libre 0,2 ng/dL (normal 0,85-1,86). \u00bfQu\u00e9 estrategia le parece m\u00e1s adecuada?", "full_answer": "Se trata de un hipotiroidismo primario severo. El tratamiento se realiza con tiroxina. Si no hay n\u00f3dulo no est\u00e1 indicado ni eco ni PAAF.", "type": "ENDOCRINOLOG\u00cdA", "options": {"1": "Realizar una ecograf\u00eda tiroidea antes de iniciar el tratamiento.", "2": "Determinar la T3 libre y realizar una resonancia magn\u00e9tica hipofisaria.", "3": "Tratar con L-Triyodotironina y una estatina.", "4": "Tratar con L-Tiroxina y determinar anticuerpos antitiroideos.", "5": "Realizar una citolog\u00eda tiroidea, previa al tratamiento."}, "correct_option": 4, "explanations": {"1": {"exist": true, "char_ranges": [[87, 136]], "word_ranges": [[13, 24]], "text": "Si no hay n\u00f3dulo no est\u00e1 indicado ni eco ni PAAF."}, "2": {"exist": false, "char_ranges": [], "word_ranges": [], "text": ""}, "3": {"exist": false, "char_ranges": [], "word_ranges": [], "text": ""}, "4": {"exist": true, "char_ranges": [[0, 86]], "word_ranges": [[0, 13]], "text": "Se trata de un hipotiroidismo primario severo. El tratamiento se realiza con tiroxina."}, "5": {"exist": true, "char_ranges": [[87, 136]], "word_ranges": [[13, 24]], "text": "Si no hay n\u00f3dulo no est\u00e1 indicado ni eco ni PAAF."}}} {"id": 398, "year": 2016, "question_id_specific": 140, "full_question": "Usted valora a un paciente de 66 a\u00f1os con dolor inguinal acentuado con la bipedestaci\u00f3n prolongada algunos d\u00edas al mes. Una radiograf\u00eda simple de caderas muestra estrechamiento del espacio articular f\u00e9moro-acetabular, esclerosis y ostefitos. \u00bfCu\u00e1l es su actitud?", "full_answer": "Hablamos de un paciente de 66 a\u00f1os con dolor algunos d\u00edas, un dolor de naturaleza ocasional. Aunque el diagn\u00f3stico sea claro de artrosis ello no conlleva directamente a una indicaci\u00f3n quir\u00fargica. El primer paso es probar con tratamiento conservador (descartamos la primera opci\u00f3n). La artrosis es un proceso degenerativo que desde luego no detienen los opioides por lo que descartamos la opci\u00f3n dos. La opci\u00f3n cuatro tambi\u00e9n la podemos descartar r\u00e1pidamente: el enunciado nos deja claro el diagn\u00f3stico y el tratamiento a seguir en este caso la RMN no nos va a aportar ninguna informaci\u00f3n de inter\u00e9s.", "type": "TRAUMATOLOG\u00cdA Y ORTOPED\u00cdA", "options": {"1": "Hago el diagn\u00f3stico de coxartrosis y env\u00edo al traumat\u00f3logo para colocar una pr\u00f3tesis de cadera.", "2": "Inicio tratamiento con opioides d\u00e9biles que han demostrado evidencia en detener la progresi\u00f3n de la enfermedad.", "3": "Instauro tratamiento con paracetamol, explico que la evoluci\u00f3n es muy variable y la indicaci\u00f3n quir\u00fargica depende de la funcionalidad y control del dolor.", "4": "Por las caracter\u00edsticas radiol\u00f3gicas descritas, necesito una RMN de cadera antes de tomar una decisi\u00f3n terap\u00e9utica.", "5": NaN}, "correct_option": 3, "explanations": {"1": {"exist": true, "char_ranges": [[196, 281]], "word_ranges": [[31, 43]], "text": "El primer paso es probar con tratamiento conservador (descartamos la primera opci\u00f3n)."}, "2": {"exist": true, "char_ranges": [[282, 399]], "word_ranges": [[43, 63]], "text": "La artrosis es un proceso degenerativo que desde luego no detienen los opioides por lo que descartamos la opci\u00f3n dos."}, "3": {"exist": true, "char_ranges": [[196, 281]], "word_ranges": [[31, 43]], "text": "El primer paso es probar con tratamiento conservador (descartamos la primera opci\u00f3n)."}, "4": {"exist": true, "char_ranges": [[400, 599]], "word_ranges": [[63, 97]], "text": "La opci\u00f3n cuatro tambi\u00e9n la podemos descartar r\u00e1pidamente: el enunciado nos deja claro el diagn\u00f3stico y el tratamiento a seguir en este caso la RMN no nos va a aportar ninguna informaci\u00f3n de inter\u00e9s."}, "5": {"exist": false, "char_ranges": [], "word_ranges": [], "text": ""}}} {"id": 28, "year": 2011, "question_id_specific": 58, "full_question": "Un hombre de 30 a\u00f1os asno se realiza una radiograf\u00eda de t\u00f3rax para formalizar un contrato laboral en una empresa. En la radiograf\u00eda se observa un patr\u00f3n intersticial bilateral de tipo reticular, adenopat\u00edas hiliares bilaterales y mediast\u00ednicas. Se indica una broncofibroscopia con lavado broncoalveolar que en el recuento celular muestra los siguientes resultados: linfocitos 50 %, histiocitos 40%, eosin\u00f3filos 2%, polimorfonucleares 8% y el cociente CD4/Cd8 5 \u00bfCu\u00e1l es el diagn\u00f3stico m\u00e1s probable?", "full_answer": "Est\u00e1 describiendo una sarcoidosis.", "type": "NEUROLOG\u00cdA Y CIRUG\u00cdA TOR\u00c1CICA", "options": {"1": "Alveolitis al\u00e9rgica extr\u00ednseca.", "2": "Neumoconiosis.", "3": "Sarcoidosis.", "4": "Linfangitis carcinomatosa.", "5": "Hemosiderosis pulmonar idiop\u00e1tica."}, "correct_option": 3, "explanations": {"1": {"exist": false, "char_ranges": [], "word_ranges": [], "text": ""}, "2": {"exist": false, "char_ranges": [], "word_ranges": [], "text": ""}, "3": {"exist": true, "char_ranges": [[0, 34]], "word_ranges": [[0, 4]], "text": "Est\u00e1 describiendo una sarcoidosis."}, "4": {"exist": false, "char_ranges": [], "word_ranges": [], "text": ""}, "5": {"exist": false, "char_ranges": [], "word_ranges": [], "text": ""}}} {"id": 587, "year": 2022, "question_id_specific": 76, "full_question": "Mujer de 40 a\u00f1os que consulta porque desde hace un mes se nota un bulto en el cuadrante superoexterno de la mama derecha. Aporta un informe de una mamograf\u00eda que describe una lesi\u00f3n BIRADS 3. \u00bfCu\u00e1l es la actitud a seguir?:", "full_answer": "El resto de respuestas son incorrectas. Con un BIRADS 3 esa es la actitud correcta a seguir.", "type": "OBSTETRICIA Y GINECOLOG\u00cdA", "options": {"1": "Tranquilizarle, puesto que ya se ha hecho una prueba de imagen y se ha descartado malignidad.", "2": "Esa clasificaci\u00f3n implica probablemente cirug\u00eda dado que la probabilidad de c\u00e1ncer es mayor del 10 %. Se lo explica y le deriva de forma preferente a la Unidad de Mama.", "3": "Se trata de un hallazgo probablemente benigno, dado que hay menos de un 2 % de probabilidad de c\u00e1ncer. Le explica que requiere de un seguimiento cada 6-12 meses hasta los 24 meses o una biopsia.", "4": "Los hallazgos son de baja sospecha de c\u00e1ncer (entre un 2 y un 10 %) pero es necesario hacer una biopsia.", "5": NaN}, "correct_option": 3, "explanations": {"1": {"exist": false, "char_ranges": [], "word_ranges": [], "text": ""}, "2": {"exist": false, "char_ranges": [], "word_ranges": [], "text": ""}, "3": {"exist": true, "char_ranges": [[40, 92]], "word_ranges": [[6, 17]], "text": "Con un BIRADS 3 esa es la actitud correcta a seguir."}, "4": {"exist": false, "char_ranges": [], "word_ranges": [], "text": ""}, "5": {"exist": false, "char_ranges": [], "word_ranges": [], "text": ""}}} {"id": 554, "year": 2022, "question_id_specific": 50, "full_question": "Var\u00f3n de 58 a\u00f1os que acude a control programado de una diabetes mellitus tipo 2 diagnosticada hace 6 a\u00f1os. Tiene antecedentes personales de HTA, dislipidemia, cardiopat\u00eda isqu\u00e9mica y pancreatitis de origen biliar. Su medicaci\u00f3n incluye lisinopril, metoprolol, metformina, AAS y atorvastatina. En la exploraci\u00f3n f\u00edsica presenta tensi\u00f3n arterial 151/93 mmHg, IMC 27,1 kg/m2. La hemoglobina glicada es de 8,3 %.\u00bfCu\u00e1l de los siguientes es el tratamiento m\u00e1s adecuado para este paciente?:", "full_answer": "Paciente con obesidad, HTA y aumento de HbA1C. En estudios recientes, Empaglifozina, inhibidor altamente selectivo del SGLT210, reduce de forma significativa la TA, la HbA1C y el IMC. La glipizida es del grupo de las sulfonilureas. Se debe evitar su uso en pacientes con cardiopat\u00eda. La acarbosa puede interaccionar con betabloqueantes, como los que toma nuestro paciente. La sitagliptina ha demostrado disminuir HbA1C pero no HTA ni IMC.", "type": "ENDOCRINOLOG\u00cdA", "options": {"1": "Empaglifozina.", "2": "Glipizida.", "3": "Acarbosa.", "4": "Sitagliptina.", "5": NaN}, "correct_option": 1, "explanations": {"1": {"exist": true, "char_ranges": [[0, 183]], "word_ranges": [[0, 28]], "text": "Paciente con obesidad, HTA y aumento de HbA1C. En estudios recientes, Empaglifozina, inhibidor altamente selectivo del SGLT210, reduce de forma significativa la TA, la HbA1C y el IMC."}, "2": {"exist": true, "char_ranges": [[184, 283]], "word_ranges": [[28, 45]], "text": "La glipizida es del grupo de las sulfonilureas. Se debe evitar su uso en pacientes con cardiopat\u00eda."}, "3": {"exist": true, "char_ranges": [[284, 372]], "word_ranges": [[45, 57]], "text": "La acarbosa puede interaccionar con betabloqueantes, como los que toma nuestro paciente."}, "4": {"exist": true, "char_ranges": [[373, 438]], "word_ranges": [[57, 68]], "text": "La sitagliptina ha demostrado disminuir HbA1C pero no HTA ni IMC."}, "5": {"exist": false, "char_ranges": [], "word_ranges": [], "text": ""}}} {"id": 251, "year": 2014, "question_id_specific": 81, "full_question": "Mujer de 50 a\u00f1os que acude al servicio de urgencias por astenia, anorexia, p\u00e9rdida de peso, ictericia, fiebre y dolor abdominal. No refiere antecedentes recientes de transfusi\u00f3n sangu\u00ednea, contactos sexuales de riesgo ni adicci\u00f3n a drogas de uso parenteral. Consumo habitual de 60 gramos diarios de alcohol durante los \u00faltimos cinco a\u00f1os, consumo que ha incrementado durante el mes previo por problemas familiares. Temperatura 38,5\u00baC, tensi\u00f3n arterial 100/60 mmHg. En la exploraci\u00f3n f\u00edsica se observa desorientaci\u00f3n temporo-espacial, malnutrici\u00f3n, asterixis, ictericia y hepatomegalia dolorosa. No ascitis ni datos de irritaci\u00f3n peritoneal. En los ex\u00e1menes de laboratorio destacan leucocitos 15.000/microlitro con 90% de polimorfonucleares, hemat\u00edes 3 millones/mm3, hemoglobina 10g/dl, volumen corpuscular medio 115 fl, bilirrubina 15 mg/dl de predominio directo, AST (GOT) 300 UI/L, ALT (GPT) 120 UI/L, GGT 635 UI/L, prolongaci\u00f3n del tiempo de protrombina mayor del 50%. \u00bfCu\u00e1l es el diagn\u00f3stico m\u00e1s probable del cuadro que presenta la paciente?", "full_answer": "Paciente con consumo perjudicial de alcohol (60g y mujer), fiebre alta, encefalopat\u00eda hep\u00e1tica (desorientaci\u00f3n, asterixis), bilirrubina directa tan alta, ictericia, hepatomegalia dolorosa, coagulopat\u00eda (tiempo de protrombina elevado) y una elevaci\u00f3n de transaminasas leve (inferior a 5 veces el l\u00edmite superior de la normalidad) hacen pensar en una hepatitis aguda por alcohol.", "type": "APARATO DIGESTIVO", "options": {"1": "Absceso hep\u00e1tico.", "2": "Colecistitis aguda.", "3": "Colangitis aguda.", "4": "Hepatitis alcoh\u00f3lica.", "5": "Pancreatitis aguda."}, "correct_option": 4, "explanations": {"1": {"exist": false, "char_ranges": [], "word_ranges": [], "text": ""}, "2": {"exist": false, "char_ranges": [], "word_ranges": [], "text": ""}, "3": {"exist": false, "char_ranges": [], "word_ranges": [], "text": ""}, "4": {"exist": true, "char_ranges": [[0, 377]], "word_ranges": [[0, 51]], "text": "Paciente con consumo perjudicial de alcohol (60g y mujer), fiebre alta, encefalopat\u00eda hep\u00e1tica (desorientaci\u00f3n, asterixis), bilirrubina directa tan alta, ictericia, hepatomegalia dolorosa, coagulopat\u00eda (tiempo de protrombina elevado) y una elevaci\u00f3n de transaminasas leve (inferior a 5 veces el l\u00edmite superior de la normalidad) hacen pensar en una hepatitis aguda por alcohol."}, "5": {"exist": false, "char_ranges": [], "word_ranges": [], "text": ""}}} {"id": 128, "year": 2012, "question_id_specific": 79, "full_question": "Una mujer de 42 a\u00f1os, casada y con 2 ni\u00f1os en edad escolar, consulta a su m\u00e9dico de familia por un fuerte dolor de cabeza de unos 7 d\u00edas de evoluci\u00f3n. Aunque ya hace m\u00e1s de un a\u00f1o viene teniendo episodios similares, en los 2 \u00faltimos meses se han agravado notablemente. El dolor es como un peso que comienza en la zona occipital, se extiende a ambas regiones temporales y apenas se le alivia tomarndo pastillas de 650 mg de paracetamol, por lo que solicita una TC (est\u00e1 convencida de que \u201calgo tiene que tener en la cabeza\u201d). \u00bfCu\u00e1l de las siguientes es la causa m\u00e1s plausible de su cefalea?", "full_answer": "As\u00ed como el a\u00f1o pasado nos quisieron pintar de mala a una enfermera por despertarnos por un paciente con miastenia este a\u00f1o se entretienen present\u00e1ndonos a una paciente que pide un TC. Interpreto que muestran a una paciente estresada por sus ni\u00f1os con cefaleas recurrentes de localizaci\u00f3n occipital que antes aliviaba con una dosis baja de paracetamol y ahora no. Tanto la localizaci\u00f3n inicialmente occipital, el tiempo de evoluci\u00f3n de 2 a\u00f1os, como la sensaci\u00f3n de peso orientan a la respuesta 4, una cefalea tensional. R\u00e1pidamente se descartan la migra\u00f1a y los trastornos vasculares, acompa\u00f1ados de otros s\u00edntomas neurol\u00f3gicos o na\u00faseas, fotofobia, etc; as\u00ed como la arteritis de la temporal rara antes de los 50 a\u00f1os. La cefalea tumoral suele ser \u201cin crescendo\u201d, no recurrente como hace pensar el caso y acompa\u00f1a otros s\u00edntomas neurol\u00f3gicos o de hipertensi\u00f3n intracraneal.", "type": "NEUROLOG\u00cdA Y NEUROCIRUG\u00cdA", "options": {"1": "La migra\u00f1a.", "2": "Los trastornos vasculares.", "3": "La arteritis de la arteria temporal.", "4": "La tensional.", "5": "La oncol\u00f3gica."}, "correct_option": 4, "explanations": {"1": {"exist": true, "char_ranges": [[520, 718]], "word_ranges": [[85, 116]], "text": "R\u00e1pidamente se descartan la migra\u00f1a y los trastornos vasculares, acompa\u00f1ados de otros s\u00edntomas neurol\u00f3gicos o na\u00faseas, fotofobia, etc; as\u00ed como la arteritis de la temporal rara antes de los 50 a\u00f1os."}, "2": {"exist": true, "char_ranges": [[520, 718]], "word_ranges": [[85, 116]], "text": "R\u00e1pidamente se descartan la migra\u00f1a y los trastornos vasculares, acompa\u00f1ados de otros s\u00edntomas neurol\u00f3gicos o na\u00faseas, fotofobia, etc; as\u00ed como la arteritis de la temporal rara antes de los 50 a\u00f1os."}, "3": {"exist": true, "char_ranges": [[520, 718]], "word_ranges": [[85, 116]], "text": "R\u00e1pidamente se descartan la migra\u00f1a y los trastornos vasculares, acompa\u00f1ados de otros s\u00edntomas neurol\u00f3gicos o na\u00faseas, fotofobia, etc; as\u00ed como la arteritis de la temporal rara antes de los 50 a\u00f1os."}, "4": {"exist": true, "char_ranges": [[364, 519]], "word_ranges": [[60, 85]], "text": "Tanto la localizaci\u00f3n inicialmente occipital, el tiempo de evoluci\u00f3n de 2 a\u00f1os, como la sensaci\u00f3n de peso orientan a la respuesta 4, una cefalea tensional."}, "5": {"exist": true, "char_ranges": [[719, 873]], "word_ranges": [[116, 139]], "text": "La cefalea tumoral suele ser \u201cin crescendo\u201d, no recurrente como hace pensar el caso y acompa\u00f1a otros s\u00edntomas neurol\u00f3gicos o de hipertensi\u00f3n intracraneal."}}} {"id": 0, "year": 2011, "question_id_specific": 34, "full_question": "Mujer de 27 a\u00f1os en estudio en el Servicio de dermatolog\u00eda por presentar episodios de aftas bucales recidivantes. Es remitida a las consultas de Aparato Digestivo por objetivar en una anal\u00edtica Hb 11,5 gr/dl Fe 38 AST 52 ALT 64 Ac antitrasglutaminasa IgA 177. La paciente s\u00f3lo refiere presentar molestias abdominales difusas de forma ocasional. No refiere signos de hemorragia ni ictericia.. La exploraci\u00f3n f\u00edsica es normal. \u00bfQu\u00e9 prueba de las que se mencionan a continuaci\u00f3n considera que es la m\u00e1s adecuada para realizar el diagn\u00f3stico?", "full_answer": "En esta pregunta, dan un dato importante a la hora de sospechar un diagn\u00f3stico que son los Ac. Antitrasglutaminasa que tienen que llevarte a sospechar enf. Celiaca. El resto de s\u00edntomas entran perfectamente dentro del cuadro y aunque probablemente se hagan varias de las pruebas que pone, la respuesta que nos dar\u00e1 el diagn\u00f3stico es la 4 (biopsia intestinal).", "type": "DIGESTIVO", "options": {"1": "C\u00e1psula endosc\u00f3pica.", "2": "Colonoscopia.", "3": "Serolog\u00eda para virus de la hepatitis b y c.", "4": "Biopsia intestinal.", "5": "Ecograf\u00eda abdominal."}, "correct_option": 4, "explanations": {"1": {"exist": false, "char_ranges": [], "word_ranges": [], "text": ""}, "2": {"exist": false, "char_ranges": [], "word_ranges": [], "text": ""}, "3": {"exist": false, "char_ranges": [], "word_ranges": [], "text": ""}, "4": {"exist": true, "char_ranges": [[0, 359]], "word_ranges": [[0, 59]], "text": "En esta pregunta, dan un dato importante a la hora de sospechar un diagn\u00f3stico que son los Ac. Antitrasglutaminasa que tienen que llevarte a sospechar enf. Celiaca. El resto de s\u00edntomas entran perfectamente dentro del cuadro y aunque probablemente se hagan varias de las pruebas que pone, la respuesta que nos dar\u00e1 el diagn\u00f3stico es la 4 (biopsia intestinal)."}, "5": {"exist": false, "char_ranges": [], "word_ranges": [], "text": ""}}} {"id": 256, "year": 2014, "question_id_specific": 162, "full_question": "Mujer de 80 a\u00f1os que consulta por presentar, desde hace 1 mes, aumento progresivo del per\u00edmetro abdominal. Durante los 4-5 meses previos nota astenia, anorexia y adelgazamiento no precisado. En la exploraci\u00f3n f\u00edsica se aprecia ascitis a tensi\u00f3n y edemas maleolares, sin otros signos relevantes. Una ecograf\u00eda y TAC de abdomen demuestran la presencia de ascitis, de densidad homog\u00e9nea, sin apreciarse implantes peritoneales ni masas abdominales o p\u00e9lvicas. H\u00edgado, p\u00e1ncreas, suprarrenales, bazo y ri\u00f1ones sin hallazgos significativos. Se realiza paracentesis a trav\u00e9s de aguja gruesa, obteni\u00e9ndose con dificultad un l\u00edquido algo amarillento, denso y gelatinoso. \u00bfCu\u00e1l es la etiolog\u00eda m\u00e1s probable?", "full_answer": "Las caracter\u00edsticas del l\u00edquido que nos describen son lechosas, es decir, l\u00edquido asc\u00edtico quiloso. Teniendo en cuenta la edad de la paciente y el s\u00edndrome constitucional que nos describen, lo m\u00e1s probable es que sea un linfoma no Hodgkin y no una tuberculosis, que tambi\u00e9n cursa con l\u00edquido asc\u00edtico quiloso, dado que no mencionan cl\u00ednica infecciosa. El resto de las entidades que nos ofrecen no cursan con l\u00edquido asc\u00edtico quiloso.", "type": "APARATO DIGESTIVO", "options": {"1": "Descompensaci\u00f3n hidr\u00f3pica secundaria a cirrosis hep\u00e1tica.", "2": "Ascitis quilosa por linfoma no Hodgkin.", "3": "Tuberculosis peritoneal.", "4": "Met\u00e1stasis peritoneal de adenocarcinoma.", "5": "Ascitis secundaria a pericarditis constrictiva."}, "correct_option": 2, "explanations": {"1": {"exist": true, "char_ranges": [[352, 433]], "word_ranges": [[56, 70]], "text": "El resto de las entidades que nos ofrecen no cursan con l\u00edquido asc\u00edtico quiloso."}, "2": {"exist": true, "char_ranges": [[100, 351]], "word_ranges": [[14, 56]], "text": "Teniendo en cuenta la edad de la paciente y el s\u00edndrome constitucional que nos describen, lo m\u00e1s probable es que sea un linfoma no Hodgkin y no una tuberculosis, que tambi\u00e9n cursa con l\u00edquido asc\u00edtico quiloso, dado que no mencionan cl\u00ednica infecciosa."}, "3": {"exist": true, "char_ranges": [[100, 351]], "word_ranges": [[14, 56]], "text": "Teniendo en cuenta la edad de la paciente y el s\u00edndrome constitucional que nos describen, lo m\u00e1s probable es que sea un linfoma no Hodgkin y no una tuberculosis, que tambi\u00e9n cursa con l\u00edquido asc\u00edtico quiloso, dado que no mencionan cl\u00ednica infecciosa."}, "4": {"exist": true, "char_ranges": [[352, 433]], "word_ranges": [[56, 70]], "text": "El resto de las entidades que nos ofrecen no cursan con l\u00edquido asc\u00edtico quiloso."}, "5": {"exist": true, "char_ranges": [[352, 433]], "word_ranges": [[56, 70]], "text": "El resto de las entidades que nos ofrecen no cursan con l\u00edquido asc\u00edtico quiloso."}}} {"id": 604, "year": 2022, "question_id_specific": 113, "full_question": "Var\u00f3n de 35 a\u00f1os, cartero, sin antecedentes de inter\u00e9s, que acude a urgencias por un dolor agudo cervical de 24 horas de evoluci\u00f3n, sin traumatismo previo, que irradia a brazo izquierdo hasta la mano y se acompa\u00f1a de parestesias en el borde radial del antebrazo. No presenta p\u00e9rdida de fuerza objetivable, conserva la movilidad del cuello aunque es dolorosa y se aprecia contractura de la musculatura paravertebral. La primera actitud ser\u00e1:", "full_answer": "Cervicalgia sin signos de alarma. Tratamiento conservador con un \u00e9xito entre el 75%-90%.", "type": "TRAUMATOLOG\u00cdA", "options": {"1": "Tratamiento conservador con antiinflamatorios no esteroideos, calor local y reposo relativo.", "2": "Llamada urgente al neurocirujano para valoraci\u00f3n quir\u00fargica.", "3": "Solicitud preferente de resonancia magn\u00e9tica y electromiograma.", "4": "Derivaci\u00f3n preferente a consultas externas de traumatolog\u00eda.", "5": NaN}, "correct_option": 1, "explanations": {"1": {"exist": true, "char_ranges": [[0, 88]], "word_ranges": [[0, 13]], "text": "Cervicalgia sin signos de alarma. Tratamiento conservador con un \u00e9xito entre el 75%-90%."}, "2": {"exist": false, "char_ranges": [], "word_ranges": [], "text": ""}, "3": {"exist": false, "char_ranges": [], "word_ranges": [], "text": ""}, "4": {"exist": false, "char_ranges": [], "word_ranges": [], "text": ""}, "5": {"exist": false, "char_ranges": [], "word_ranges": [], "text": ""}}} {"id": 362, "year": 2016, "question_id_specific": 96, "full_question": "Una mujer de 25 a\u00f1os se presenta en el servicio de Urgencias por presentar en los \u00faltimos d\u00edas equimosis y petequias en extremidades inferiores, as\u00ed como gingivorragia. En el hemograma presenta los siguientes datos: Hb 13 g/dL, leucocitos 8.500/microL con f\u00f3rmula leucocitaria normal y plaquetas 9.000/mm3. La determinaci\u00f3n de los tiempos de coagulaci\u00f3n es normal. \u00bf Cu\u00e1l de las siguientes pruebas diagn\u00f3sticas NO es preciso realizar?", "full_answer": "Lo primero es saber el diagn\u00f3stico del caso cl\u00ednico que nos plantean, se trata de una trombocitopenia inmune primaria (PTI), s\u00f3lo esta alterada la serie plaquetar con coagulaci\u00f3n normal. La primera prueba que hay que realizar es un frotis de sangre perif\u00e9rica para comprobar si esas plaquetas son reales y no hay pseudotrombopenia por agregados plaquetarios. En el estudio de ampliaci\u00f3n se solicitan los anticuerpos antinucleares (ANA) para descartar patolog\u00edas autoinmunes que sean las causantes de la trombopenia y otra prueba que estar\u00eda indicada aunque en este caso la paciente es joven y no presenta signos de gravedad (leucopenia, anemia, no repsuesta al tratmiento de 1\u00ba l\u00ednea\u2026) es la realizaci\u00f3n de un aspirado de m\u00e9dula \u00f3sea, para ver que si existen megacariocitos tromboformadores y as\u00ed confirmar que la destrucci\u00f3n plaquetar es de origen perif\u00e9rico. Pero lo que s\u00ed que no esta indicado es la determinaci\u00f3n de la mutaci\u00f3n JAK-2, que se usa en el diagn\u00f3stico de los s\u00edndromes mieloproliferativos cr\u00f3nicos, que no es el caso que se nos presenta.", "type": "HEMATOLOG\u00cdA", "options": {"1": "Mutaci\u00f3n JAK-2.", "2": "Aspirado de m\u00e9dula \u00f3sea.", "3": "Anticuerpos antinucleares.", "4": "Frotis de sangre perif\u00e9rica.", "5": NaN}, "correct_option": 1, "explanations": {"1": {"exist": true, "char_ranges": [[866, 1053]], "word_ranges": [[135, 169]], "text": "lo que s\u00ed que no esta indicado es la determinaci\u00f3n de la mutaci\u00f3n JAK-2, que se usa en el diagn\u00f3stico de los s\u00edndromes mieloproliferativos cr\u00f3nicos, que no es el caso que se nos presenta."}, "2": {"exist": true, "char_ranges": [[517, 860]], "word_ranges": [[79, 134]], "text": "otra prueba que estar\u00eda indicada aunque en este caso la paciente es joven y no presenta signos de gravedad (leucopenia, anemia, no repsuesta al tratmiento de 1\u00ba l\u00ednea\u2026) es la realizaci\u00f3n de un aspirado de m\u00e9dula \u00f3sea, para ver que si existen megacariocitos tromboformadores y as\u00ed confirmar que la destrucci\u00f3n plaquetar es de origen perif\u00e9rico."}, "3": {"exist": true, "char_ranges": [[359, 514]], "word_ranges": [[56, 78]], "text": "En el estudio de ampliaci\u00f3n se solicitan los anticuerpos antinucleares (ANA) para descartar patolog\u00edas autoinmunes que sean las causantes de la trombopenia"}, "4": {"exist": true, "char_ranges": [[187, 358]], "word_ranges": [[29, 56]], "text": "La primera prueba que hay que realizar es un frotis de sangre perif\u00e9rica para comprobar si esas plaquetas son reales y no hay pseudotrombopenia por agregados plaquetarios."}, "5": {"exist": false, "char_ranges": [], "word_ranges": [], "text": ""}}} {"id": 410, "year": 2018, "question_id_specific": 195, "full_question": "Un hombre de 75 a ingresa en el hospital porque se niega a comer y ha perdido un 30% de su peso. Tiene historia de esclerosis m\u00faltiple que ha empeorado en los \u00faltimos meses. Su \u00fanico familiar que era su hermana muri\u00f3 hace 3 meses y desde entonces es cuando \u00e9l se ha deteriorado. Se ha vuelto incontinente, ha dejado de comer y de participar en eventos sociales. Impresiona de depresi\u00f3n psic\u00f3tica. F\u00edsicamente es capaz de comer. Se inicia medicaci\u00f3n psic\u00f3tropa que se suspende por eventos adversos. Si su estado nutricional mejora es posible retomar sus medicaciones. Rechaza l\u00edquidos intravenosos y luego los acepta pero se arranca la v\u00eda IV unas horas m\u00e1s tarde. Una alternativa es realizar una gastrostom\u00eda endosc\u00f3pica percut\u00e1nea (PEG) y darle alta a su residencia. \u00bfCu\u00e1l de las siguientes actuaciones es la m\u00e1s apropiada para este paciente?", "full_answer": "La respuesta correcta es la 1 ya que el paciente est\u00e1 en una situaci\u00f3n de depresi\u00f3n psic\u00f3tica y no es capaz de decidir por su vida y no parece que el paciente haya rellenado el documento de \u00faltimas voluntades lo m\u00e1s conveniente es presentar el caso a la comisi\u00f3n de \u00e9tica del centro para que tome una decisi\u00f3n al respecto ajustada a derecho.", "type": "ATENCI\u00d3N PRIMARIA", "options": {"1": "Solicitar a la comisi\u00f3n de \u00e9tica del hospital que ayude a determinar la decisi\u00f3n que sea de mayor inter\u00e9s para el paciente", "2": "Valorar se la residencia acoge al paciente si no se realiza la PEG", "3": "Pedir una orden judicial para realizar la PEG", "4": "Enrolar al paciente en cuidados paliativos terminales", "5": NaN}, "correct_option": 1, "explanations": {"1": {"exist": true, "char_ranges": [[37, 341]], "word_ranges": [[8, 63]], "text": "el paciente est\u00e1 en una situaci\u00f3n de depresi\u00f3n psic\u00f3tica y no es capaz de decidir por su vida y no parece que el paciente haya rellenado el documento de \u00faltimas voluntades lo m\u00e1s conveniente es presentar el caso a la comisi\u00f3n de \u00e9tica del centro para que tome una decisi\u00f3n al respecto ajustada a derecho."}, "2": {"exist": false, "char_ranges": [], "word_ranges": [], "text": ""}, "3": {"exist": false, "char_ranges": [], "word_ranges": [], "text": ""}, "4": {"exist": false, "char_ranges": [], "word_ranges": [], "text": ""}, "5": {"exist": false, "char_ranges": [], "word_ranges": [], "text": ""}}} {"id": 361, "year": 2016, "question_id_specific": 94, "full_question": "Hombre de 51 a\u00f1os que es ingresado desde Urgencias por anemia macroc\u00edtica (Hb 6,3 g/dL, VCm 120 fL). En los estudios realizados se descarta un origen carencial. Los reticulocitos son de 24000/microL. El estudio de m\u00e9dula \u00f3sea es compatible con un s\u00edndrome mielodispl\u00e1sico (SMD). La citogen\u00e9tica muestra una delecci\u00f3n en 5q \u00bf Cu\u00e1l es la afirmaci\u00f3n correcta en relaci\u00f3n a este paciente?", "full_answer": "Estamos ante un paciente joven y sin comorbilidades por lo que es candidato a recibir tratamiento con intenci\u00f3n curativa por lo que la opci\u00f3n 4 se descarta. La opci\u00f3n 3 tampoco es la correcta ya que no podemos saber el IPSS que tiene el paciente ya que nos falta saber el n\u00famero de blastos y el n\u00famero de leucocitos y plaquetas para calcular el riesgo. La opci\u00f3n 2 tambi\u00e9n es incorrecta ya que la indicaci\u00f3n de un trasplante de m\u00e9dula \u00f3sea en SMD es aquellos pacientes con IPSS alto o intermedio-2, y en este caso desconocemos el riesgo, lo \u00fanico que conocemos del riesgo es la citogen\u00e9tica que es de buen pron\u00f3stico que nos dar\u00eda 0 puntos. Por lo tanto la correcta es la 1 ya que es cierto que tiene una citog\u00e9netica favorable (5q-) y tiene un tratamiento espec\u00edfico la lenalidomida, tambi\u00e9n usada en otras patolog\u00edas como el mieloma multiple.", "type": "HEMATOLOG\u00cdA", "options": {"1": "Esta delecci\u00f3n (5q-) es una alteraci\u00f3n de buen pron\u00f3stico y tiene un tratamiento espec\u00edfico (lenalidomida).", "2": "Lo recomendable en este paciente ser\u00eda realizar tipajes HLA para organizar un trasplante alog\u00e9nico.", "3": "Se trata de un paciente con un \u00cdndice Internacional de pron\u00f3stico (IPSS) alto.", "4": "El tratamiento en este caso ser\u00edan las transfusi\u00f3n \u00fanicamente.", "5": NaN}, "correct_option": 1, "explanations": {"1": {"exist": true, "char_ranges": [[641, 844]], "word_ranges": [[117, 151]], "text": "Por lo tanto la correcta es la 1 ya que es cierto que tiene una citog\u00e9netica favorable (5q-) y tiene un tratamiento espec\u00edfico la lenalidomida, tambi\u00e9n usada en otras patolog\u00edas como el mieloma multiple."}, "2": {"exist": true, "char_ranges": [[353, 640]], "word_ranges": [[65, 117]], "text": "La opci\u00f3n 2 tambi\u00e9n es incorrecta ya que la indicaci\u00f3n de un trasplante de m\u00e9dula \u00f3sea en SMD es aquellos pacientes con IPSS alto o intermedio-2, y en este caso desconocemos el riesgo, lo \u00fanico que conocemos del riesgo es la citogen\u00e9tica que es de buen pron\u00f3stico que nos dar\u00eda 0 puntos."}, "3": {"exist": true, "char_ranges": [[157, 352]], "word_ranges": [[27, 65]], "text": "La opci\u00f3n 3 tampoco es la correcta ya que no podemos saber el IPSS que tiene el paciente ya que nos falta saber el n\u00famero de blastos y el n\u00famero de leucocitos y plaquetas para calcular el riesgo."}, "4": {"exist": true, "char_ranges": [[0, 156]], "word_ranges": [[0, 27]], "text": "Estamos ante un paciente joven y sin comorbilidades por lo que es candidato a recibir tratamiento con intenci\u00f3n curativa por lo que la opci\u00f3n 4 se descarta."}, "5": {"exist": false, "char_ranges": [], "word_ranges": [], "text": ""}}} {"id": 172, "year": 2013, "question_id_specific": 50, "full_question": "Un paciente con EPOC grave acude agudizado a Urgencias y presenta una gasometr\u00eda arterial extra\u00edda con Fi02 del 31% a nivel del mar con una Pa02 de 86 mm Hg, PaC02 65 mm Hg, pH 7.13 y Bicarbonato 27 mmol/litro. \u00bfCu\u00e1l de las siguientes afirmaciones es FALSA?", "full_answer": "La gasometr\u00eda demuestra una acidosis respiratoria aguda, por incremento en la PCO2 de poco tiempo de evoluci\u00f3n dado que el ri\u00f1\u00f3n aun no ha tenido tiempo de retener bicarbonatos (cifra de bicarbonato en limite alto de la normalidad). El paciente no esta hiperventilando sino hipoventilando dado que la PCO2 esta elevada. El gradiente alv\u00e9olo arterial de oxigeno esta alterado dado que el paciente presenta una PaO2 normal, pero porque esta con una FiO2 alta. Si estuviera sin oxigeno, estar\u00eda hipox\u00e9mico. La acidosis respiratoria por debajo de 7.20 es un motivo para considerar la ventilaci\u00f3n mec\u00e1nica no invasiva de forma aguda y en principio temporal en un paciente con EPOC grave.", "type": "NEUMOLOG\u00cdA", "options": {"1": "El paciente est\u00e1 hiperventilando.", "2": "El gradiente alveolo arterial de ox\u00edgeno est\u00e1 elevado.", "3": "El paciente est\u00e1 en acidosis respiratoria.", "4": "El nivel de bicarbonato es normal.", "5": "Se deber\u00eda considerar iniciar ventilaci\u00f3n mec\u00e1nica."}, "correct_option": 1, "explanations": {"1": {"exist": true, "char_ranges": [[233, 319]], "word_ranges": [[38, 51]], "text": "El paciente no esta hiperventilando sino hipoventilando dado que la PCO2 esta elevada."}, "2": {"exist": true, "char_ranges": [[320, 457]], "word_ranges": [[51, 74]], "text": "El gradiente alv\u00e9olo arterial de oxigeno esta alterado dado que el paciente presenta una PaO2 normal, pero porque esta con una FiO2 alta."}, "3": {"exist": true, "char_ranges": [[0, 232]], "word_ranges": [[0, 38]], "text": "La gasometr\u00eda demuestra una acidosis respiratoria aguda, por incremento en la PCO2 de poco tiempo de evoluci\u00f3n dado que el ri\u00f1\u00f3n aun no ha tenido tiempo de retener bicarbonatos (cifra de bicarbonato en limite alto de la normalidad)."}, "4": {"exist": true, "char_ranges": [[120, 232]], "word_ranges": [[19, 38]], "text": "el ri\u00f1\u00f3n aun no ha tenido tiempo de retener bicarbonatos (cifra de bicarbonato en limite alto de la normalidad)."}, "5": {"exist": true, "char_ranges": [[504, 682]], "word_ranges": [[80, 110]], "text": "La acidosis respiratoria por debajo de 7.20 es un motivo para considerar la ventilaci\u00f3n mec\u00e1nica no invasiva de forma aguda y en principio temporal en un paciente con EPOC grave."}}} {"id": 83, "year": 2012, "question_id_specific": 48, "full_question": "\u00bfQu\u00e9 intervenci\u00f3n terap\u00e9utica considerar\u00eda en un paciente de 67 a\u00f1os con miocardiopat\u00eda dilatada idiop\u00e1tica, bloqueo de rama izquierda, fracci\u00f3n de eyecci\u00f3n del ventr\u00edculo izquierdo de 26%, insuficiencia mitral grado II-III/IV, sometido a tratamiento con furosemida, espironolactona, enalapril y bisoprolol y que se mantiene en clase funcional III de la NYHA?", "full_answer": "Nos est\u00e1n hablando de un paciente con una insuficiencia cardiaca con disfunci\u00f3n VI severa (FE < 35%), BRIHH (QRS ancho), que sigue un tto. m\u00e9dico \u00f3ptimo y que se mantiene en clase funcional avanzada a pesar de ello. La revascularizaci\u00f3n coronaria no parece que vaya a aportar mucho en este paciente en este momento. El reemplazo valvular mitral tampoco, primero porque lo m\u00e1s probable es que la insuficiencia mitral sea a consecuencia de la dilataci\u00f3n del anillo mitral, y segundo porque necesitar\u00edamos de una valoraci\u00f3n ecocardiogr\u00e1fica de dicha v\u00e1lvula para saber si presenta alguna alteraci\u00f3n. El implante del bal\u00f3n de contrapulsaci\u00f3n intraa\u00f3rtico no estar\u00eda indicado en este paciente (se utiliza en situaciones de shock cardiog\u00e9nico, en complicaciones mec\u00e1nicas post-IAM, como puente al Tx cardiaco\u2026). El dispositivo de asistencia ventricular tampoco estar\u00eda indicado en este paciente. Lo que verdaderamente le beneficiar\u00eda de todas las respuestas es la 5, el dispositivo de resincronizaci\u00f3n. Al conseguir una estimulaci\u00f3n simult\u00e1nea de ambos ventr\u00edculos, se consigue una sinergia mec\u00e1nica que mejora la funci\u00f3n cardiaca. Las indicaciones de aplicaci\u00f3n de una TRC (terapia de resincronizaci\u00f3n cardiaca) son precisamente las comentadas al principio: Disfunci\u00f3n ventricular severa, asincron\u00eda demostrada por la presencia de QRS ancho en el ECG (habitualmente por un BRIHH) y clase funcional avanzada (III-IV) a pesar de un tto. m\u00e9dico \u00f3ptimo.", "type": "CARDIOLOG\u00cdA Y CIRUG\u00cdA VASCULAR", "options": {"1": "Revascularizaci\u00f3n coronaria quir\u00fargica.", "2": "Reemplazo valvular mitral.", "3": "Implante de un bal\u00f3n a\u00f3rtico de contrapulsaci\u00f3n.", "4": "Implante de un dispositivo de asistencia ventricular.", "5": "Implante de un sistema de resincronizaci\u00f3n cardiaca."}, "correct_option": 5, "explanations": {"1": {"exist": true, "char_ranges": [[216, 315]], "word_ranges": [[38, 54]], "text": "La revascularizaci\u00f3n coronaria no parece que vaya a aportar mucho en este paciente en este momento."}, "2": {"exist": true, "char_ranges": [[316, 596]], "word_ranges": [[54, 95]], "text": "El reemplazo valvular mitral tampoco, primero porque lo m\u00e1s probable es que la insuficiencia mitral sea a consecuencia de la dilataci\u00f3n del anillo mitral, y segundo porque necesitar\u00edamos de una valoraci\u00f3n ecocardiogr\u00e1fica de dicha v\u00e1lvula para saber si presenta alguna alteraci\u00f3n."}, "3": {"exist": true, "char_ranges": [[597, 805]], "word_ranges": [[95, 124]], "text": "El implante del bal\u00f3n de contrapulsaci\u00f3n intraa\u00f3rtico no estar\u00eda indicado en este paciente (se utiliza en situaciones de shock cardiog\u00e9nico, en complicaciones mec\u00e1nicas post-IAM, como puente al Tx cardiaco\u2026)."}, "4": {"exist": true, "char_ranges": [[806, 889]], "word_ranges": [[124, 135]], "text": "El dispositivo de asistencia ventricular tampoco estar\u00eda indicado en este paciente."}, "5": {"exist": true, "char_ranges": [[997, 1125]], "word_ranges": [[151, 169]], "text": "Al conseguir una estimulaci\u00f3n simult\u00e1nea de ambos ventr\u00edculos, se consigue una sinergia mec\u00e1nica que mejora la funci\u00f3n cardiaca."}}} {"id": 141, "year": 2012, "question_id_specific": 135, "full_question": "Ni\u00f1o de 2 a\u00f1os, en sus antecedentes personales destacan: 3 episodios de otitis media aguda, 1 meningitis meningoc\u00f3cica y 2 neumon\u00edas (una de l\u00f3bulo medio y otra de l\u00f3bulo superior izquierdo). Ha ingresado en 3 ocasiones por p\u00farpura trombop\u00e9nica (en tres ocasiones los anticuerpos antiplaquetas fueron negativos y en la m\u00e9dula \u00f3sea se observaban megacariocitos normales). Varios varones de la familia materna hab\u00edan fallecido en la infancia por presentar procesos infecciosos. En la exploraci\u00f3n presenta lesiones t\u00edpicas de dermatitis at\u00f3pica. En el estudio inmunol\u00f3gico destaca una leve disminuci\u00f3n de subpoblaciones de linfocitos T; elevaci\u00f3n de IgA e IgE; disminuci\u00f3n de IgM e IgG en el l\u00edmite inferior de la normalidad. \u00bfCu\u00e1l es el diagn\u00f3stico m\u00e1s probable?", "full_answer": "La respuesta correcta es la 1. El s\u00edndrome de Wiskott-Aldrich asocia inmunodeficiencia, trombopenia y dermatitis at\u00f3pica con el estudio inmunol\u00f3gico descrito en el enunciado.", "type": "PEDIATR\u00cdA", "options": {"1": "S\u00edndrome de Wiskott-Aldrich.", "2": "S\u00edndrome de hiperIgE.", "3": "Hipogammaglobulinemia transitoria de la infancia.", "4": "Inmunodeficiencia combinada severa asociada al X.", "5": "Inmunodeficiencia variable com\u00fan."}, "correct_option": 1, "explanations": {"1": {"exist": true, "char_ranges": [[31, 174]], "word_ranges": [[6, 24]], "text": "El s\u00edndrome de Wiskott-Aldrich asocia inmunodeficiencia, trombopenia y dermatitis at\u00f3pica con el estudio inmunol\u00f3gico descrito en el enunciado."}, "2": {"exist": false, "char_ranges": [], "word_ranges": [], "text": ""}, "3": {"exist": false, "char_ranges": [], "word_ranges": [], "text": ""}, "4": {"exist": false, "char_ranges": [], "word_ranges": [], "text": ""}, "5": {"exist": false, "char_ranges": [], "word_ranges": [], "text": ""}}} {"id": 317, "year": 2016, "question_id_specific": 139, "full_question": "Mujer de 70 a\u00f1os con antecedente de anorexia, p\u00e9rdida de peso, molestias en la musculatura y articulaciones proximales m\u00e1s dolorimiento en la regi\u00f3n temporo-mandibular que acude al servicio de urgencias por p\u00e9rdida de visi\u00f3n unilaterai (movimiento de manos), de aparici\u00f3n brusca e indolora (defecto pupilar aferente).\u00bfQu\u00e9 prueba solicitar\u00eda en primer lugar con fines diagn\u00f3sticos?", "full_answer": "Es posiblemente una arteritis de la temporal, basta con el cuadro cl\u00ednico y una anal\u00edtica con aumento de reactantes para confirmar la sospecha. Despu\u00e9s habr\u00eda que hacer una valoraci\u00f3n ocular para descartar NOIA, instaurar tratamiento urgente (si hay NOIA 3 bolos de metilprednisolona 1 gr, si no la hay prednisona mg/kg) y valorar biopsiar.", "type": "REUMATOLOG\u00cdA", "options": {"1": "Punci\u00f3n lumbar.", "2": "Prote\u00edna C Reactiva.", "3": "Angio Resonancia Magn\u00e9tica.", "4": "Ecograf\u00eda carot\u00eddea.", "5": NaN}, "correct_option": 2, "explanations": {"1": {"exist": false, "char_ranges": [], "word_ranges": [], "text": ""}, "2": {"exist": true, "char_ranges": [[0, 143]], "word_ranges": [[0, 23]], "text": "Es posiblemente una arteritis de la temporal, basta con el cuadro cl\u00ednico y una anal\u00edtica con aumento de reactantes para confirmar la sospecha."}, "3": {"exist": false, "char_ranges": [], "word_ranges": [], "text": ""}, "4": {"exist": false, "char_ranges": [], "word_ranges": [], "text": ""}, "5": {"exist": false, "char_ranges": [], "word_ranges": [], "text": ""}}} {"id": 550, "year": 2022, "question_id_specific": 129, "full_question": "Var\u00f3n de 65 a\u00f1os que acude a urgencias por aparici\u00f3n de lesiones moteadas cian\u00f3ticas en los dedos de ambos pies. Antecedentes personales: tabaquismo, hipertensi\u00f3n arterial y dislipemia. Rx t\u00f3rax: ensanchamiento mediast\u00ednico. Angio-TC t\u00f3raco-abd\u00f3mino-p\u00e9lvica: aneurisma de aorta tor\u00e1cica descendente de 7 cm de di\u00e1metro, distal a arteria subclavia izquierda y con trombo mural. De las siguientes, se\u00f1ale la opci\u00f3n correcta:", "full_answer": "Est\u00e1 indicado el implante de una endopr\u00f3tesis tor\u00e1cica.", "type": "CARDIOLOG\u00cdA", "options": {"1": "Est\u00e1 indicado el tratamiento con estatinas para estabilizar el trombo.", "2": "Es necesaria la realizaci\u00f3n de una aortograf\u00eda diagn\u00f3stica urgente.", "3": "En la cirug\u00eda se sustituye el segmento aneurism\u00e1tico por una pr\u00f3tesis tubular con reimplante de los troncos supraa\u00f3rticos.", "4": "Est\u00e1 indicado el implante de una endopr\u00f3tesis tor\u00e1cica.", "5": NaN}, "correct_option": 4, "explanations": {"1": {"exist": false, "char_ranges": [], "word_ranges": [], "text": ""}, "2": {"exist": false, "char_ranges": [], "word_ranges": [], "text": ""}, "3": {"exist": false, "char_ranges": [], "word_ranges": [], "text": ""}, "4": {"exist": true, "char_ranges": [[0, 55]], "word_ranges": [[0, 8]], "text": "Est\u00e1 indicado el implante de una endopr\u00f3tesis tor\u00e1cica."}, "5": {"exist": false, "char_ranges": [], "word_ranges": [], "text": ""}}} {"id": 110, "year": 2012, "question_id_specific": 152, "full_question": "Paciente de 76 a\u00f1os, paridad 3-0-3-1, con menopausia a los 52 a\u00f1os. Relata que desde hace almenos 4-5 a\u00f1os presenta prurito vulvar de intensidad variable que ha sido tratado algunes veces con automedicaci\u00f3n y otras por indicaci\u00f3n de su medico generalista con Preparados t\u00f3picos (cremas y lavados). El prurito ha evolucionado as\u00ed con intermitencias pero desde hace 3-4 meses nota adem\u00e1s una peque\u00f1a tumoraci\u00f3n en el labio mayor iquierdo de la vulva y de la que se producen p\u00e9rdidas serohem\u00e1ticas al roce desde hace algunos d\u00edas; por ello consulta al ginec\u00f3logo. Al interrogatorio refiere dis\u00faria ocasional y el estado general es bueno. \u00bfCu\u00e1l es el diagn\u00f3stico m\u00e1s probable en esta paciente?", "full_answer": "La respuesta correcta es la 5. El carcinoma vulvar se caracteriza por prurito vulvar cr\u00f3nico resistente a m\u00faltiples tratamientos, aparici\u00f3n de tumoraci\u00f3n o ulceraci\u00f3n, disuria y tenesmo vesical, y tard\u00edamente dolor y hemorragia.", "type": "GINECOLOG\u00cdA Y OBSTETRICIA", "options": {"1": "Herpes genital.", "2": "Enfermedad de Paget de la vulva.", "3": "Car\u00fancula uretral.", "4": "Granuloma cr\u00f3nico de la vulva.", "5": "Carcinoma vulvar escamoso."}, "correct_option": 5, "explanations": {"1": {"exist": false, "char_ranges": [], "word_ranges": [], "text": ""}, "2": {"exist": false, "char_ranges": [], "word_ranges": [], "text": ""}, "3": {"exist": false, "char_ranges": [], "word_ranges": [], "text": ""}, "4": {"exist": false, "char_ranges": [], "word_ranges": [], "text": ""}, "5": {"exist": true, "char_ranges": [[31, 228]], "word_ranges": [[6, 33]], "text": "El carcinoma vulvar se caracteriza por prurito vulvar cr\u00f3nico resistente a m\u00faltiples tratamientos, aparici\u00f3n de tumoraci\u00f3n o ulceraci\u00f3n, disuria y tenesmo vesical, y tard\u00edamente dolor y hemorragia."}}} {"id": 419, "year": 2018, "question_id_specific": 77, "full_question": "Mujer de 80 a\u00f1os que acude urgencias con dolor abdominal de inicio en epigastrio e irradiado posteriormente a fosa iliaca izquierda. Se acompa\u00f1a de fiebre de 37,5\u00baC, Se le realiza una TC abdominal, objetivando inflamaci\u00f3n en las paredes del sigma y absceso mesenterico de 2 cm. El tratamiento de elecci\u00f3n es:", "full_answer": "En esta pregunta nos presentan un cuadro cl\u00ednico cuyas manifestaciones son compatibles con una diverticulitis aguda, ya preguntada en numerosas ocasiones anteriormente. Se expone un caso de diverticulitis aguda complicada con absceso mesent\u00e9rico de peque\u00f1o tama\u00f1o (< 2-3 cm). Los abscesos deben ser drenados mediente punci\u00f3n percut\u00e1nea guiada por TC, ya que el abordaje intraabdominal puede propagar el contenido hacia la cavidad abdominal. Sin embargo, en abscesos peque\u00f1os con divertciulitis Hinchey grado I, en pacientes sin gran deterioro del estado general como en el caso propuesto, se pueden manejar mediante tratamiento conservador con antiobioticoterapia v\u00eda intravenosa y dieta absoluta, por lo tanto la opci\u00f3n correcta es la 1. El abordaje quir\u00fargico urgente se debe reservar para casos que presentan peritonitis (grado III, IV) mediante resecci\u00f3n del segmento afecto y anastomosis primaria en pacientes estables o llevar a cabo intervenci\u00f3n de Hartamn que incluye resecci\u00f3n con colostom\u00eda terminal y cierre mu\u00f1on rectal distal realizando anastomosis en un segundo tiempo quir\u00fargico.", "type": "CIRUG\u00cdA GENERAL", "options": {"1": "lngreso en planta con dieta absoluta y tratamiento antibi\u00f3tico de amplio espectro.", "2": "Colostomia de descarga.", "3": "Drenaje mediante cirug\u00eda laparosc\u00f3pica.", "4": "Cirug\u00eda urgente con sigmoidectom\u00eda y anastomosis colorrectal.", "5": NaN}, "correct_option": 1, "explanations": {"1": {"exist": true, "char_ranges": [[454, 738]], "word_ranges": [[65, 108]], "text": "en abscesos peque\u00f1os con divertciulitis Hinchey grado I, en pacientes sin gran deterioro del estado general como en el caso propuesto, se pueden manejar mediante tratamiento conservador con antiobioticoterapia v\u00eda intravenosa y dieta absoluta, por lo tanto la opci\u00f3n correcta es la 1."}, "2": {"exist": false, "char_ranges": [], "word_ranges": [], "text": ""}, "3": {"exist": false, "char_ranges": [], "word_ranges": [], "text": ""}, "4": {"exist": true, "char_ranges": [[739, 839]], "word_ranges": [[108, 123]], "text": "El abordaje quir\u00fargico urgente se debe reservar para casos que presentan peritonitis (grado III, IV)"}, "5": {"exist": false, "char_ranges": [], "word_ranges": [], "text": ""}}} {"id": 37, "year": 2011, "question_id_specific": 227, "full_question": "Una mujer de 78 a\u00f1os consulta por p\u00e9rdida de visi\u00f3n aguda del ojo izquierdo. En las \u00faltimas 3 semanas ha presentado febr\u00edcula dolor en hombros cuello y caderas as\u00ed como cefalea moderada. La funduscopia evidencia un disco \u00f3ptico p\u00e1lido y edematoso. La movilizaci\u00f3n de hombros y caderas produce dolor. No se observan alteraciones en la palpaci\u00f3n de las arterias temporales. Datos anal\u00edticos: hemoglobina 9,7 g/dL ferritina 450 ng/mL velocidad de sedimentaci\u00f3n globular 115 mm/h. \u00bfCu\u00e1l es la acci\u00f3n inmediata m\u00e1s apropiada en esta paciente?", "full_answer": "La cl\u00ednica que nos cuentan es fuertemente sugerente de arteritis de c\u00e9lulas gigantes (ACG), con febr\u00edcula, dolor en cintura escapular y p\u00e9lvica, y cefalea. La edad y el sexo tambi\u00e9n apuntan a esa posibilidad. La p\u00e9rdida visual unilateral brusca, con edema p\u00e1lido de papila es muy indicativo de neuropat\u00eda \u00f3ptica isqu\u00e9mica arter\u00edtica por ACG. La VSG est\u00e1 elevada, que nos viene a aclarar m\u00e1s todav\u00eda el diagn\u00f3stico. Tenemos que iniciar r\u00e1pidamente el tratamiento corticoideo. Ya podemos descartar la 1 y la 5 porque no aportan nada. La 4 podr\u00eda hacernos dudar, y es cierto que tendremos que pedir una biopsia de arteria temporal, pero eso no es lo inmediato: en el enunciado nos lo dejan muy claro que tenemos que actuar en seguida. La tendencia actual es utilizar megadosis de corticoides intravenosos, y a pesar de que tradicionalmente se ha propuesto la aspirina para reducir los eventos isqu\u00e9micos, la evidencia se\u00f1ala en este momento que los antiagregantes no aportan mucho. Una dosis oral de corticoide en torno de 1mg/Kg tambi\u00e9n ser\u00eda aceptable. Por lo tanto, si descartamos la 3 porque la dosis es demasiado baja, tendr\u00edamos que quedarnos con la 2. Con reservas, ya que la aspirina tiene un papel m\u00e1s bien dudoso, y la dosis de corticoide queda ambigua (no sabemos cu\u00e1nto pesa la paciente). De todas las preguntas de oftalmolog\u00eda, quiz\u00e1s esta es la que tiene m\u00e1s posibilidades de impugnarse.", "type": "OFTALMOLOG\u00cdA", "options": {"1": "Solicitar una ecobraf\u00eda doppler de arterias temporales.", "2": "Iniciar tratamiento con prednisona 60mg al d\u00eda y aspirina 100 mg al d\u00eda.", "3": "Iniciar tratamiento con prednisona 10 mg al d\u00eda.", "4": "Solicitar una biopsia de arteria temporal.", "5": "Solicitar una resonancia nuclear magn\u00e9tica cerebral."}, "correct_option": 2, "explanations": {"1": {"exist": false, "char_ranges": [], "word_ranges": [], "text": ""}, "2": {"exist": true, "char_ranges": [[0, 341]], "word_ranges": [[0, 54]], "text": "La cl\u00ednica que nos cuentan es fuertemente sugerente de arteritis de c\u00e9lulas gigantes (ACG), con febr\u00edcula, dolor en cintura escapular y p\u00e9lvica, y cefalea. La edad y el sexo tambi\u00e9n apuntan a esa posibilidad. La p\u00e9rdida visual unilateral brusca, con edema p\u00e1lido de papila es muy indicativo de neuropat\u00eda \u00f3ptica isqu\u00e9mica arter\u00edtica por ACG."}, "3": {"exist": true, "char_ranges": [[1069, 1119]], "word_ranges": [[176, 185]], "text": "descartamos la 3 porque la dosis es demasiado baja,"}, "4": {"exist": true, "char_ranges": [[532, 978]], "word_ranges": [[86, 160]], "text": "La 4 podr\u00eda hacernos dudar, y es cierto que tendremos que pedir una biopsia de arteria temporal, pero eso no es lo inmediato: en el enunciado nos lo dejan muy claro que tenemos que actuar en seguida. La tendencia actual es utilizar megadosis de corticoides intravenosos, y a pesar de que tradicionalmente se ha propuesto la aspirina para reducir los eventos isqu\u00e9micos, la evidencia se\u00f1ala en este momento que los antiagregantes no aportan mucho."}, "5": {"exist": false, "char_ranges": [], "word_ranges": [], "text": ""}}} {"id": 532, "year": 2021, "question_id_specific": 67, "full_question": "Una mujer de 26 a\u00f1os, con antecedente de un embarazo a los 24 a\u00f1os finalizado en aborto espont\u00e1neo en la semana 12\u00aa que no requiri\u00f3 legrado, acude a la consulta presentando sangrado vaginal diario de un mes de evoluci\u00f3n, sin fiebre ni dolor. En la ecograf\u00eda se observa un aumento de la l\u00ednea endometrial, con una imagen intracavitaria de aspecto polipoideo. La biopsia endometrial muestra proliferaci\u00f3n del trofoblasto intermedio. \u00bfQu\u00e9 diagn\u00f3stico es el correcto?:", "full_answer": "Eleva gonadotropina cori\u00f3nica. Tiene tratamiento curativo.", "type": "ONCOLOG\u00cdA", "options": {"1": "Tumor del lecho placentario.", "2": "Hiperplasia endometrial at\u00edpica.", "3": "P\u00f3lipo endometrial.", "4": "Coriocarcinoma.", "5": NaN}, "correct_option": 4, "explanations": {"1": {"exist": false, "char_ranges": [], "word_ranges": [], "text": ""}, "2": {"exist": false, "char_ranges": [], "word_ranges": [], "text": ""}, "3": {"exist": false, "char_ranges": [], "word_ranges": [], "text": ""}, "4": {"exist": true, "char_ranges": [[0, 30]], "word_ranges": [[0, 3]], "text": "Eleva gonadotropina cori\u00f3nica."}, "5": {"exist": false, "char_ranges": [], "word_ranges": [], "text": ""}}} {"id": 272, "year": 2016, "question_id_specific": 69, "full_question": "Un hombre de 60 a\u00f1os de edad con colitis ulcerosa extensa de 15 a\u00f1os de evoluci\u00f3n y en remisi\u00f3n cl\u00ednica los \u00faltimos 3 a\u00f1os, acude a nuestra consulta para informarse sobre el riesgo de c\u00e1ncer colorrectal y sobre la posibilidad de participar en programas de prevenci\u00f3n. Es correcto informarle de que:", "full_answer": "Los pacientes con enfermedad inflamatoria intestinal (EII) tienen un riesgo aumentado de c\u00e1ncer colorrectal, sean o no fumadores. Se recomienda empezar el cribado a los 8-10 a\u00f1os del diagn\u00f3stico de la EII (o antes si aparece en ese intervalo una colangitis esclerosante primaria o concurren otros hechos). Las t\u00e9cnicas recomendadas son la colonoscopia con biopsias al azar (respuesta 3) y la cromoendoscopia pancol\u00f3nica con tinci\u00f3n, que es actualmente el gold standard porque detecta mejor la displasia, aunque requiere mayor experiencia t\u00e9cnica.", "type": "APARATO DIGESTIVO", "options": {"1": "La colitis ulcerosa solo se asocia a un incremento en el riesgo de c\u00e1ncer colorrectal en fumadores.", "2": "En su caso, dado que est\u00e1 en remisi\u00f3n de larga duraci\u00f3n, se considera adecuado el cribado aconsejado a la poblaci\u00f3n general.", "3": "En casos como el suyo se considera adecuado someterse a colonoscopias peri\u00f3dicas con toma de m\u00faltiples biopsias escalonadas a lo largo de todo el colon.", "4": "En casos como el suyo se considera adecuado el cribado mediante estudios peri\u00f3dicos de sangre oculta en heces pero con una frecuencia superior a la utilizada en el cribado de la poblaci\u00f3n general.", "5": NaN}, "correct_option": 3, "explanations": {"1": {"exist": false, "char_ranges": [], "word_ranges": [], "text": ""}, "2": {"exist": false, "char_ranges": [], "word_ranges": [], "text": ""}, "3": {"exist": true, "char_ranges": [[130, 431]], "word_ranges": [[18, 65]], "text": "Se recomienda empezar el cribado a los 8-10 a\u00f1os del diagn\u00f3stico de la EII (o antes si aparece en ese intervalo una colangitis esclerosante primaria o concurren otros hechos). Las t\u00e9cnicas recomendadas son la colonoscopia con biopsias al azar (respuesta 3) y la cromoendoscopia pancol\u00f3nica con tinci\u00f3n,"}, "4": {"exist": false, "char_ranges": [], "word_ranges": [], "text": ""}, "5": {"exist": false, "char_ranges": [], "word_ranges": [], "text": ""}}} {"id": 428, "year": 2018, "question_id_specific": 61, "full_question": "Hombre de 62 a\u00f1os que acude a urgencias del hospital por presentar fiebre de 38,3\u2033C y escalofr\u00edos. Est\u00e1 taquic\u00e1rdico y taquipn\u00e9ico. Anal\u00edtica de sangre: leucocitosis con desviaci\u00f3n izquierda. La presi\u00f3n arterial es de 90/60 mmHg. No se escuchan soplos en la auscultaci\u00f3n card\u00edaca y la RX de t\u00f3rax descarta una neumon\u00eda. Como historial de inter\u00e9s refiere infecciones del tracto urinario de repetici\u00f3n. Hace una semana el m\u00e9dico de cabecera le recet\u00f3 cefuroxima, f\u00e1rmaco que est\u00e1 tomando actualmente. Se le extraen dos hemocultivos y se remite orina para examen del sedimento y cultivo microbiol\u00f3gico. El sedimento es patol\u00f3gico, con intensa piuria, pera nitritos negativos. A las 24 horas se informa desde el laboratorio de microbiolog\u00eda que tanta la orina como los hemocultivos remitidos son positivos y en elias se observa cocos grampositivos agrupados en cadenas, pendientes de identificaci\u00f3n y antibiograma. A la espera del resultada definitiva, \u00bfqu\u00e9 recomendar\u00eda usted?", "full_answer": "Con la prueba de nitritos en orina detectamos la presencia en la orina de nitritos. Estos aparecen por la presencia en la orina de gran concentraci\u00f3n de bacterias que posean la enzima nitrato reductasa por la cual son capaces de reducir a nitritos los nitratos presentes en la orina. Esta enzima es activa en la mayor\u00eda de bacterias que producen infecci\u00f3n urinaria con m\u00e1s frecuencia, como muchos bacilos Gram negativos. Otros microorganismos, causa relativamente frecuentes de infecci\u00f3n urinaria, no poseen esta enzima y no reducen los nitratos como enterococos, estafilococos y levaduras. Como la tinci\u00f3n de Gram nos indica que las bacterias se agrupan en cadenas, al ser esto t\u00edpico de enterococo, sugerimos la necesidad de cubrir con el tratamiento antibi\u00f3tico a este grupo de bacterias. Los estafilococos t\u00edpicamente se observan en la tinci\u00f3n de Gram como cocos Gram positivos formando racimos y las levaduras, aunque con igual coloraci\u00f3n, tienen una morfolog\u00eda t\u00edpica diferente a los cocos Gram positivos.", "type": "ENFERMEDADES INFECCIOSAS Y MICROBIOLOG\u00cdA", "options": {"1": "Continuar con cefuroxima a la espera del resultada definitivo, pues el paciente ha llevado m\u00faltiples tratamientos y no deber\u00edamos equivocamos en su tratamiento actual.", "2": "Cambiar a ertapenem, pensando en la posibilidad de microorganismos multirresistentes.", "3": "Cambiar a un tratamiento antimicrobiano de amplio espectro que cubra Enterococcus spp.", "4": "Cambiar a un tratamiento antimicrobiana de amplio espectro que contenga imipenem, pensando en la posibilidad de Staphylococcus aureus, ya que nuestro hospital tiene una elevada incidencia de S. aureus resistente a meticilina (SARM).", "5": NaN}, "correct_option": 3, "explanations": {"1": {"exist": false, "char_ranges": [], "word_ranges": [], "text": ""}, "2": {"exist": false, "char_ranges": [], "word_ranges": [], "text": ""}, "3": {"exist": true, "char_ranges": [[591, 791]], "word_ranges": [[92, 126]], "text": "Como la tinci\u00f3n de Gram nos indica que las bacterias se agrupan en cadenas, al ser esto t\u00edpico de enterococo, sugerimos la necesidad de cubrir con el tratamiento antibi\u00f3tico a este grupo de bacterias."}, "4": {"exist": true, "char_ranges": [[792, 1011]], "word_ranges": [[126, 159]], "text": "Los estafilococos t\u00edpicamente se observan en la tinci\u00f3n de Gram como cocos Gram positivos formando racimos y las levaduras, aunque con igual coloraci\u00f3n, tienen una morfolog\u00eda t\u00edpica diferente a los cocos Gram positivos."}, "5": {"exist": false, "char_ranges": [], "word_ranges": [], "text": ""}}} {"id": 393, "year": 2016, "question_id_specific": 224, "full_question": "Una paciente presenta una larga historia de m\u00faltiples s\u00edntomas f\u00edsicos: p\u00e9rdida de memoria, dolor de cabeza, mareos, v\u00f3mitos, dolor genital, dolor en extremidades, distensi\u00f3n abdominal e irregularidades menstruales. Los diversos ex\u00e1menes m\u00e9dicos han descartado enfermedad m\u00e9dica alguna. \u00bfCu\u00e1l de los siguientes cuadros es m\u00e1s probable que padezca?", "full_answer": "Seg\u00fan la CIE-10 los criterios para diagnosticar el Trastorno de somatizaci\u00f3n (F45.0) son: A. Historia de m\u00faltiples s\u00edntomas f\u00edsicos, que empieza antes de los 30 a\u00f1os, persiste durante varios a\u00f1os y obliga a la b\u00fasqueda de atenci\u00f3n m\u00e9dica o provoca un deterioro significativo social, laboral, o de otras \u00e1reas importantes de la actividad del individuo. B. Deben cumplirse todos los criterios que se exponen a continuaci\u00f3n, y cada s\u00edntoma puede aparecer en cualquier momento de la alteraci\u00f3n: 1. Cuatro s\u00edntomas dolorosos: historia de dolor relacionada con al menos cuatro zonas del cuerpo o cuatro funciones (p. ej., cabeza, abdomen, dorso, articulaciones, extremidades, t\u00f3rax, recto; durante la menstruaci\u00f3n, el acto sexual, o la micci\u00f3n). 2. Dos s\u00edntomas gastrointestinales: historia de al menos dos s\u00edntomas gastrointestinales distintos al dolor (p. ej., n\u00e1useas, distensi\u00f3n abdominal, v\u00f3mitos [no durante el embarazo], diarrea o intolerancia a diferentes alimentos). 3. Un s\u00edntoma sexual: historia de al menos un s\u00edntoma sexual o reproductor al margen del dolor (p. ej., indiferencia sexual, disfunci\u00f3n er\u00e9ctil o eyaculatoria, menstruaciones irregulares, p\u00e9rdidas menstruales excesivas, v\u00f3mitos durante el embarazo). 4. Un s\u00edntoma pseudoneurol\u00f3gico: historia de al menos un s\u00edntoma o d\u00e9ficit que sugiera un trastorno neurol\u00f3gico no limitado a dolor (s\u00edntomas de conversi\u00f3n del tipo de la alteraci\u00f3n de la coordinaci\u00f3n psicomotora o del equilibrio, par\u00e1lisis o debilidad muscular localizada, dificultad para deglutir, sensaci\u00f3n de nudo en la garganta, afon\u00eda, retenci\u00f3n urinaria, alucinaciones, p\u00e9rdida de la sensibilidad t\u00e1ctil y dolorosa, diplop\u00eda, ceguera, sordera, convulsiones; s\u00edntomas disociativos como amnesia; o p\u00e9rdida de conciencia distinta del desmayo). C. Cualquiera de las dos caracter\u00edsticas siguientes: 1. Tras un examen adecuado, ninguno de los s\u00edntomas del Criterio B puede explicarse por la presencia de una enfermedad m\u00e9dica conocida o por los efectos directos de una sustancia (p. ej., drogas o f\u00e1rmacos). 2. Si hay una enfermedad m\u00e9dica, los s\u00edntomas f\u00edsicos o el deterioro social o laboral son excesivos en comparaci\u00f3n con lo que cabr\u00eda esperar por la historia cl\u00ednica, la exploraci\u00f3n f\u00edsica o los hallazgos de laboratorio. D. Los s\u00edntomas no se producen intencionadamente y no son simulados (a diferencia de lo que ocurre en el trastorno facticio y en la simulaci\u00f3n). Todo cuadra con lo descrito en el enunciado. Descartamos el trastorno conversivo o disociativo porque \u00e9stos suelen estar relacionados con factores psicol\u00f3gicos, asociados al s\u00edntoma o al d\u00e9ficit (no deliberado). El inicio o la exacerbaci\u00f3n del cuadro vienen precedidos por conflictos u otros desencadenantes, algo que no menciona el enunciado. Tampoco ser\u00eda un Trastorno hipocondr\u00edaco porque esto supone una preocupaci\u00f3n y miedo a tener, o la convicci\u00f3n de padecer, una enfermedad grave a partir de la interpretaci\u00f3n personal de s\u00edntomas som\u00e1ticos. Pero no conlleva la existencia de signos o s\u00edntomas f\u00edsicos.", "type": "PSIQUIATR\u00cdA", "options": {"1": "Trastorno conversivo.", "2": "Trastorno hipocondr\u00edaco.", "3": "Trastorno de somatizaci\u00f3n.", "4": "Trastomo disociativo.", "5": NaN}, "correct_option": 3, "explanations": {"1": {"exist": true, "char_ranges": [[2439, 2737]], "word_ranges": [[364, 406]], "text": "Descartamos el trastorno conversivo o disociativo porque \u00e9stos suelen estar relacionados con factores psicol\u00f3gicos, asociados al s\u00edntoma o al d\u00e9ficit (no deliberado). El inicio o la exacerbaci\u00f3n del cuadro vienen precedidos por conflictos u otros desencadenantes, algo que no menciona el enunciado."}, "2": {"exist": true, "char_ranges": [[2738, 3003]], "word_ranges": [[406, 447]], "text": "Tampoco ser\u00eda un Trastorno hipocondr\u00edaco porque esto supone una preocupaci\u00f3n y miedo a tener, o la convicci\u00f3n de padecer, una enfermedad grave a partir de la interpretaci\u00f3n personal de s\u00edntomas som\u00e1ticos. Pero no conlleva la existencia de signos o s\u00edntomas f\u00edsicos."}, "3": {"exist": false, "char_ranges": [], "word_ranges": [], "text": ""}, "4": {"exist": true, "char_ranges": [[2439, 2737]], "word_ranges": [[364, 406]], "text": "Descartamos el trastorno conversivo o disociativo porque \u00e9stos suelen estar relacionados con factores psicol\u00f3gicos, asociados al s\u00edntoma o al d\u00e9ficit (no deliberado). El inicio o la exacerbaci\u00f3n del cuadro vienen precedidos por conflictos u otros desencadenantes, algo que no menciona el enunciado."}, "5": {"exist": false, "char_ranges": [], "word_ranges": [], "text": ""}}} {"id": 124, "year": 2012, "question_id_specific": 46, "full_question": "Una mujer de 58 a\u00f1os acude para una visita de seguimiento por diabetes mellitus e hipertensi\u00f3n. Se siente bien pero afirma que ha dejado de tomar el verapamilo por estre\u00f1imiento. Presenta intolerancia a los IECAs por tos. En la exploraci\u00f3n, la tensi\u00f3n arterial es de 156/92 mm Hg. En la anal\u00edtica incluyen una creatinina de 1.6 mg/dL, excreci\u00f3n de prote\u00ednas en orina de 24 horas de 1.5 g/d\u00eda y un aclaramiento de la creatinina de 45 ml/min. Sobre esta base, \u00bfcu\u00e1l es el tratamiento m\u00e1s eficaz para enlentecer la progresi\u00f3n de la nefropat\u00eda diab\u00e9tica tipo 2 de la paciente?", "full_answer": "Esta pregunta es t\u00edpica y bastante f\u00e1cil. Los f\u00e1rmacos antihipertensivos que han demostrado enlentecer la progresi\u00f3n de la nefropat\u00eda diab\u00e9tica son los IECAS y los ARAII. En concreto, los bloqueadores del receptor de la angiotensina II han demostrado su eficacia en este campo para la DM tipo 2, lo que unido a los efectos secundarios que le producen los IECAS a la paciente hace que un ARAII sea el f\u00e1rmaco de elecci\u00f3n. No me parece una pregunta impugnable. (Esto sin entrar en asuntos farmacoecon\u00f3micos, claro\u2026).", "type": "NEFROLOG\u00cdA", "options": {"1": "Inhibidor de la enzima convertidora de la angiotensina.", "2": "Bloqueador del receptor de la angiotensina.", "3": "Antagonista del calcio.", "4": "Alfa-bloqueante.", "5": "Beta-bloqueante."}, "correct_option": 2, "explanations": {"1": {"exist": true, "char_ranges": [[184, 420]], "word_ranges": [[28, 72]], "text": "los bloqueadores del receptor de la angiotensina II han demostrado su eficacia en este campo para la DM tipo 2, lo que unido a los efectos secundarios que le producen los IECAS a la paciente hace que un ARAII sea el f\u00e1rmaco de elecci\u00f3n."}, "2": {"exist": true, "char_ranges": [[185, 420]], "word_ranges": [[28, 72]], "text": "los bloqueadores del receptor de la angiotensina II han demostrado su eficacia en este campo para la DM tipo 2, lo que unido a los efectos secundarios que le producen los IECAS a la paciente hace que un ARAII sea el f\u00e1rmaco de elecci\u00f3n."}, "3": {"exist": false, "char_ranges": [], "word_ranges": [], "text": ""}, "4": {"exist": false, "char_ranges": [], "word_ranges": [], "text": ""}, "5": {"exist": false, "char_ranges": [], "word_ranges": [], "text": ""}}} {"id": 405, "year": 2016, "question_id_specific": 31, "full_question": "Mujer de 20 a\u00f1os con tumoraci\u00f3n ov\u00e1rica de 15 cm, s\u00f3lido-qu\u00edstica, detectada por ecograf\u00eda tras presentar s\u00edntomas abdominales inespec\u00edficos. En el estudio histopatol\u00f3gico de la pieza correspondiente se encuentran dientes, pelos, zonas de epitelio intestinal, \u00e1reas de epitelio escamoso (15%) y bronquial, as\u00ed como elementos neuroectod\u00e9rmicos y embrionarios en varias de las preparaciones histol\u00f3gicas. En referencia a este caso, se\u00f1ale el diagn\u00f3stico correcto:", "full_answer": "El teratoma es el tumor germinal m\u00e1s frecuente. Presenta diferenciaci\u00f3n en elementos de las tres capas embrionarias: endoderma, mesoderma y ectoderma. Macrosc\u00f3picamente pueden ser qu\u00edsticos o s\u00f3lidos, e histol\u00f3gicamente los tejidos que componen el tumor pueden ser maduros (bien diferenciados, como los tejidos adultos) o inmaduros (como tejidos embrionarios).El teratoma qu\u00edstico maduro es el m\u00e1s frecuente: representan en promedio el 10% de los tumores ov\u00e1ricos (5-25% seg\u00fan las casu\u00edsticas). Ocurren a cualquier edad. Predominan los tejidos del ectoderma, como piel, que revisten una cavidad de contenido querat\u00ednico. En la cavidad se reconoce un espol\u00f3n del que nacen frecuentemente estructuras pilosas o dentarias. Entre los tejidos frecuentes est\u00e1n: tejido nervioso, generalmente glial y epitelio ependimario, epitelios de tipo respiratorio y digestivo y diversas estructuras mesod\u00e9rmicas. Son bilaterales en cerca del 10% de los casos. El teratoma qu\u00edstico maduro es benigno, pero en un 2% de ellos puede desarrollarse un tumor maligno a partir de alguno de los componentes tisulares (carcinoma espinocelular, carcinoide, adenocarcinoma, carcinoma de tejido tiroideo, sarcoma). En este caso en un principio pensamos en que la opci\u00f3n era el teratoma qu\u00edstico maduro pero gracias a Ram\u00f3n, al leerlo con con m\u00e1s atenci\u00f3n, comprobamos que ten\u00eda elementos neuroectod\u00e9rmicos y embrionarios en varias de las preparaciones histol\u00f3gicas, por lo que consideramos que la respuesta correcta es la 2. Teratoma inmaduro.", "type": "ONCOLOG\u00cdA (ECT\u00d3PICO)", "options": {"1": "Teratocarcinoma.", "2": "Teratoma inmaduro.", "3": "Teratoma qu\u00edstico maduro.", "4": "Disgerminoma.", "5": NaN}, "correct_option": 2, "explanations": {"1": {"exist": false, "char_ranges": [], "word_ranges": [], "text": ""}, "2": {"exist": true, "char_ranges": [[1326, 1513]], "word_ranges": [[193, 219]], "text": "comprobamos que ten\u00eda elementos neuroectod\u00e9rmicos y embrionarios en varias de las preparaciones histol\u00f3gicas, por lo que consideramos que la respuesta correcta es la 2. Teratoma inmaduro."}, "3": {"exist": true, "char_ranges": [[1326, 1513]], "word_ranges": [[193, 219]], "text": "comprobamos que ten\u00eda elementos neuroectod\u00e9rmicos y embrionarios en varias de las preparaciones histol\u00f3gicas, por lo que consideramos que la respuesta correcta es la 2. Teratoma inmaduro."}, "4": {"exist": false, "char_ranges": [], "word_ranges": [], "text": ""}, "5": {"exist": false, "char_ranges": [], "word_ranges": [], "text": ""}}} {"id": 497, "year": 2020, "question_id_specific": 127, "full_question": "Hombre de 60 a\u00f1os, fumador de 40 paquetes/a\u00f1o, que consulta por dolor en borde cubital del brazo izquierdo y ptosis palpebral izquierda desde hace un mes. La radiograf\u00eda de t\u00f3rax muestra una masa en el l\u00f3bulo superior izquierdo y la TC confirma la lesi\u00f3n con invasi\u00f3n de la segunda costilla. En relaci\u00f3n con el diagn\u00f3stico de sospecha, el tratamiento m\u00e1s adecuado es:", "full_answer": "Pancoast t\u00edpico de fumador, probablemente un epidermoide, con sdr de Horner. Lo mejor ser\u00eda quimiorradio seguido de rescate quir\u00fargico. Antiguamente era solo RT y cirug\u00eda. Quiz\u00e1 no muy lejos entre la inmunoterapia en primera l\u00ednea. Cada vez hay m\u00e1s cosas nuevas.", "type": "ONCOLOG\u00cdA M\u00c9DICA", "options": {"1": "Quimioterapia neoadyuvante con quimiorradioterapia combinada, seguida de cirug\u00eda.", "2": "Quimioterapia neoadyuvante sin cirug\u00eda, seguida de radioterapia.", "3": "Cirug\u00eda seguida de quimiorradioterapia combinada.", "4": "Radioterapia sin quimioterapia, seguida de cirug\u00eda.", "5": NaN}, "correct_option": 1, "explanations": {"1": {"exist": true, "char_ranges": [[0, 135]], "word_ranges": [[0, 19]], "text": "Pancoast t\u00edpico de fumador, probablemente un epidermoide, con sdr de Horner. Lo mejor ser\u00eda quimiorradio seguido de rescate quir\u00fargico."}, "2": {"exist": false, "char_ranges": [], "word_ranges": [], "text": ""}, "3": {"exist": false, "char_ranges": [], "word_ranges": [], "text": ""}, "4": {"exist": true, "char_ranges": [[136, 171]], "word_ranges": [[19, 25]], "text": "Antiguamente era solo RT y cirug\u00eda."}, "5": {"exist": false, "char_ranges": [], "word_ranges": [], "text": ""}}} {"id": 307, "year": 2016, "question_id_specific": 207, "full_question": "Paciente de 34 a\u00f1os que jugando a tenis recibe el impacto de la bola a nivel de la \u00f3rbita izquierda. A la exploraci\u00f3n presenta importante hematoma palpebral, hiposfagma, diplopia a la visi\u00f3n superior con limitaci\u00f3n de la versi\u00f3n superior del globo ocular. \u00bfQu\u00e9 sospechar\u00eda?", "full_answer": "Esta pregunta tiene una peque\u00f1a errata (se dir\u00eda \u00abal tenis\u00bb), pero personalmente me molesta un poco m\u00e1s el error en la descripci\u00f3n de la exploraci\u00f3n. Concretamente cuando dicen \u00abdiplopia a la visi\u00f3n superior con limitaci\u00f3n de la versi\u00f3n superior del globo ocular\u00bb. Las versiones son movimientos bilaterales conjugados de ambos ojos. Es decir, una versi\u00f3n se compone de la suma de dos ducciones, ya que la ducci\u00f3n es el movimiento de un solo ojo. Cuando exploras las versiones, concretamente la versi\u00f3n superior o supraversi\u00f3n, ambos ojos giran hacia arriba. En ese momento, puedes encontrar una limitaci\u00f3n del movimiento de un ojo. En este caso, al explorar la supraversi\u00f3n, sospechas o detectas una limitaci\u00f3n en el movimiento del ojo izquierdo. Y a esto lo llamar\u00edamos una limitaci\u00f3n a la supraducci\u00f3n de ese ojo. No tiene sentido hablar de limitaci\u00f3n a la versi\u00f3n del globo ocular, porque la versi\u00f3n siempre es de ambos globos oculares. Vale que se entiende, pero es un error importante de concepto que no deber\u00eda cometer en que redacta la pregunta (que se supone que es un experto en la materia).Pero bueno, como no creo que sea motivo para impugnar la pregunta, sigamos. Se trata de un paciente que ha recibido un golpe. Las cuatro respuestas son fracturas, as\u00ed que tenemos bastante definido el cuadro. El hematoma palpebral y el hiposfagma (hemorragia subconjuntival) no nos aporta el dato clave. Lo importante es que el ojo que ha recibido el golpe no sube, y por tanto hay visi\u00f3n doble en la mirada superior. Podr\u00eda ser tentador contestar la 3, pensando que al atraparse el recto superior, este no funciona y por eso el ojo no puede subir. Sin embargo, los m\u00fasculos atrapados en una fractura no pierden autom\u00e1ticamente su acci\u00f3n muscular. Lo que pasa es que precisamente se atrapan, quedan \u00abenganchados\u00bb, y no pueden estirarse. El problema es que se ha roto la pared inferior, se ha atrapado el recto inferior, y cuando el ojo debe mirar hacia arriba, este recto inferior no se puede estirar como es debido. No se trata de par\u00e1lisis, sino un problema restrictivo. Tambi\u00e9n podr\u00eda contestarse esta pregunta por descarte. Las opciones 2 y 4 podemos ignorarlas porque est\u00e1n demasiado lejos de los m\u00fasculos extraoculares para producir diplopia. Y en cuanto a las paredes de la \u00f3rbita, las que se rompen con m\u00e1s frecuencia es la pared inferior y medial. La pared superior se rompe muy raramente.", "type": "OFTALMOLOG\u00cdA", "options": {"1": "Fractura de la pared inferior del suelo de la \u00f3rbita con atrapamiento del m\u00fasculo recto inferior.", "2": "Fractura del arco cigom\u00e1tico.", "3": "Fractura de la pared superior de la \u00f3rbita con atrapamiento del m\u00fasculo recto superior.", "4": "Fractura dentoalveolar.", "5": NaN}, "correct_option": 1, "explanations": {"1": {"exist": true, "char_ranges": [[2248, 2355]], "word_ranges": [[374, 396]], "text": "Y en cuanto a las paredes de la \u00f3rbita, las que se rompen con m\u00e1s frecuencia es la pared inferior y medial."}, "2": {"exist": true, "char_ranges": [[2127, 2247]], "word_ranges": [[356, 374]], "text": "Las opciones 2 y 4 podemos ignorarlas porque est\u00e1n demasiado lejos de los m\u00fasculos extraoculares para producir diplopia."}, "3": {"exist": true, "char_ranges": [[2356, 2397]], "word_ranges": [[396, 403]], "text": "La pared superior se rompe muy raramente."}, "4": {"exist": true, "char_ranges": [[2127, 2247]], "word_ranges": [[356, 374]], "text": "Las opciones 2 y 4 podemos ignorarlas porque est\u00e1n demasiado lejos de los m\u00fasculos extraoculares para producir diplopia."}, "5": {"exist": false, "char_ranges": [], "word_ranges": [], "text": ""}}} {"id": 589, "year": 2022, "question_id_specific": 63, "full_question": "Var\u00f3n de 36 a\u00f1os que consulta por hiperemia conjuntival y sensaci\u00f3n de cuerpo extra\u00f1o. En la exploraci\u00f3n presenta afectaci\u00f3n del ganglio preauricular. \u00bfCu\u00e1l de las siguientes patolog\u00edas NO se corresponde con dicha exploraci\u00f3n?:", "full_answer": "Nos est\u00e1n presentando un cuadro de inflamaci\u00f3n conjuntival o conjuntivitis y preguntando cu\u00e1l de ellos no se asocia a linfadenopat\u00eda pre-auricular. La conjuntivitis por adenovirus se puede asociar a inflamaci\u00f3n del ganglio linf\u00e1tico pre-auricular o sub-mandibular ipsilateral (opci\u00f3n 1 descartada). La conjuntivitis al\u00e9rgica es un cuadro de hipersensibilidad que puede producirse de forma inmediata tras el contacto con el est\u00edmulo desencadenante o transcurridas 24-72 horas, y se asocia a signos acompa\u00f1antes como edema y eritema palpebral, cambios en la pigmentaci\u00f3n cut\u00e1nea periocular, y a un s\u00edntoma clave: el picor. A lo que no se asocia es a adenopat\u00edas (opci\u00f3n 2 correcta). El s\u00edndrome oculoglandular de Parinaud es un cuadro que por definici\u00f3n se compone de conjuntivitis de causa infecciosa asociada a linfadenopat\u00eda (una o varias) ipsilateral pre-auricular o latero-cervical, producida generalmente por bacterias como Bartonella henselae (enfermedad por ara\u00f1azo de gato) o Francisella tularensis (tularemia o fiebre de los conejos): opci\u00f3n 3 descartada. La conjuntivitis por Chlamydia puede presentarse como 3 cuadros cl\u00ednicos muy definidos: el tracoma, la oftalm\u00eda neonatal o la conjuntivitis por cuerpos de inclusi\u00f3n del adulto. Esta \u00faltima se considera una enfermedad de transmisi\u00f3n sexual y se asocia a ganglios pre-auriculares ipsilaterales (opci\u00f3n 4 descartada).", "type": "OFTALMOLOG\u00cdA", "options": {"1": "Conjuntivitis adenov\u00edrica.", "2": "Conjuntivitis al\u00e9rgica.", "3": "Sindrome oculoglandular de Parinaud.", "4": "Conjuntivitis por Chlamydia.", "5": NaN}, "correct_option": 2, "explanations": {"1": {"exist": true, "char_ranges": [[148, 298]], "word_ranges": [[21, 40]], "text": "La conjuntivitis por adenovirus se puede asociar a inflamaci\u00f3n del ganglio linf\u00e1tico pre-auricular o sub-mandibular ipsilateral (opci\u00f3n 1 descartada)."}, "2": {"exist": true, "char_ranges": [[299, 680]], "word_ranges": [[40, 101]], "text": "La conjuntivitis al\u00e9rgica es un cuadro de hipersensibilidad que puede producirse de forma inmediata tras el contacto con el est\u00edmulo desencadenante o transcurridas 24-72 horas, y se asocia a signos acompa\u00f1antes como edema y eritema palpebral, cambios en la pigmentaci\u00f3n cut\u00e1nea periocular, y a un s\u00edntoma clave: el picor. A lo que no se asocia es a adenopat\u00edas (opci\u00f3n 2 correcta)."}, "3": {"exist": true, "char_ranges": [[681, 1063]], "word_ranges": [[101, 153]], "text": "El s\u00edndrome oculoglandular de Parinaud es un cuadro que por definici\u00f3n se compone de conjuntivitis de causa infecciosa asociada a linfadenopat\u00eda (una o varias) ipsilateral pre-auricular o latero-cervical, producida generalmente por bacterias como Bartonella henselae (enfermedad por ara\u00f1azo de gato) o Francisella tularensis (tularemia o fiebre de los conejos): opci\u00f3n 3 descartada."}, "4": {"exist": true, "char_ranges": [[1064, 1378]], "word_ranges": [[153, 198]], "text": "La conjuntivitis por Chlamydia puede presentarse como 3 cuadros cl\u00ednicos muy definidos: el tracoma, la oftalm\u00eda neonatal o la conjuntivitis por cuerpos de inclusi\u00f3n del adulto. Esta \u00faltima se considera una enfermedad de transmisi\u00f3n sexual y se asocia a ganglios pre-auriculares ipsilaterales (opci\u00f3n 4 descartada)."}, "5": {"exist": false, "char_ranges": [], "word_ranges": [], "text": ""}}} {"id": 350, "year": 2016, "question_id_specific": 160, "full_question": "Mujer de 45 a\u00f1os, madre de 3 hijos, acude a consulta de diagn\u00f3stico precoz. La citolog\u00eda cervicovaginal es compatible con lesi\u00f3n escamosa intraepitelial de alto grado. \u00bfCu\u00e1l de las siguientes opciones elegir\u00eda?", "full_answer": "Colposcopia con eventual biopsia. H-SIL (Lesi\u00f3n escamosa intraepitelial de alto grado)*: incluye cambios sugestivos de CIN2 y CIN3/CIS. El diagn\u00f3stico citol\u00f3gico debe ser confirmado mediante biopsia, para ello es necesario realizar una biopsia dirigida mediante colposcopia.", "type": "GINECOLOG\u00cdA Y OBSTETRICIA", "options": {"1": "Repetir la citolog\u00eda en 1 mes.", "2": "Colposcopia con eventual biopsia.", "3": "Histerectom\u00eda con salpinguectom\u00eda bilateral y conservaci\u00f3n de ovarios.", "4": "Legrado uterino fraccionado.", "5": NaN}, "correct_option": 2, "explanations": {"1": {"exist": false, "char_ranges": [], "word_ranges": [], "text": ""}, "2": {"exist": true, "char_ranges": [[136, 274]], "word_ranges": [[18, 36]], "text": "El diagn\u00f3stico citol\u00f3gico debe ser confirmado mediante biopsia, para ello es necesario realizar una biopsia dirigida mediante colposcopia."}, "3": {"exist": false, "char_ranges": [], "word_ranges": [], "text": ""}, "4": {"exist": false, "char_ranges": [], "word_ranges": [], "text": ""}, "5": {"exist": false, "char_ranges": [], "word_ranges": [], "text": ""}}} {"id": 25, "year": 2011, "question_id_specific": 116, "full_question": "Una mujer de 40 a\u00f1os ingresa en el Servicio de Urgencias con fiebre de 38\u00baC y dolor lumbar derecho. En la anal\u00edtica destacan 25000 leucocitos/mm3 con desviaci\u00f3n izquierda y en la eco abd se evidencia una litiasis coraliforme. \u00bfQu\u00e9 microorganismo esperamos encontrar?", "full_answer": "Pregunta de epidemiolog\u00eda compartida con URO. Las litiasis coraliformes se asocian a g\u00e9rmenes desdobladores de la urea: Proteus en 1\u00ba lugar, Pseudomonas, Klebsiella y algunos estafilococos.", "type": "INFECCIOSAS", "options": {"1": "Escherichia coli.", "2": "Enterococcus faecalis.", "3": "Salmonella typhi.", "4": "Candida albicans.", "5": "Proteus mirabilis."}, "correct_option": 5, "explanations": {"1": {"exist": false, "char_ranges": [], "word_ranges": [], "text": ""}, "2": {"exist": false, "char_ranges": [], "word_ranges": [], "text": ""}, "3": {"exist": false, "char_ranges": [], "word_ranges": [], "text": ""}, "4": {"exist": false, "char_ranges": [], "word_ranges": [], "text": ""}, "5": {"exist": true, "char_ranges": [[46, 190]], "word_ranges": [[6, 26]], "text": "Las litiasis coraliformes se asocian a g\u00e9rmenes desdobladores de la urea: Proteus en 1\u00ba lugar, Pseudomonas, Klebsiella y algunos estafilococos."}}} {"id": 103, "year": 2012, "question_id_specific": 89, "full_question": "Paciente de 55 a\u00f1os de edad diagn\u00f3sticado de HTA hace 2 meses en revisi\u00f3nn de empresa. Anal\u00edtica: glucosa 129 mg/dl, colesterol 202 mg/dl, LDLc 160 mg/dl, HDLc 38 mg/dl, triglic\u00e9ridos 171 mg/dl, creatinina 1.1 mg/dl. Consulta por mal control de la tensi\u00f3n arterial, edemas maleolares y disnea que ocasionalmente es nocturna. Exploraci\u00f3n f\u00edsica: IMC 38 kg/m2, per\u00edmetro abdominal 110 cm, TA 157/93, FC 70 lpm, FR 14 rpm. Soplo sist\u00f3lico con refuerzo del segundo ruido. Pulsos perif\u00e9ricos con discreta asimetr\u00eda en pedio y tibial posterior derechos que son m\u00e1s d\u00e9biles respecto a extremidad izquierda. Rx de t\u00f3rax y ECG sin alteraciones \u00bfC\u00faal de las siguientes pruebas complementarias es la menos necesaria para la detecci\u00f3n de lesiones en \u00f3rganos diana?", "full_answer": "La HbA1c nos servir\u00eda en este paciente para el diagn\u00f3stico de diabetes pero no para la afectaci\u00f3n sobre \u00f3rgano diana de la HTA.", "type": "ENDOCRINOLOG\u00cdA", "options": {"1": "Ecocardiograma transtor\u00e1cico.", "2": "Realizaci\u00f3n de fondo de ojo.", "3": "Determinaci\u00f3n de microalbuminuria.", "4": "\u00cdndice tobillo-brazo.", "5": "Determinaci\u00f3n de hemoglobina glicosilada."}, "correct_option": 5, "explanations": {"1": {"exist": false, "char_ranges": [], "word_ranges": [], "text": ""}, "2": {"exist": false, "char_ranges": [], "word_ranges": [], "text": ""}, "3": {"exist": false, "char_ranges": [], "word_ranges": [], "text": ""}, "4": {"exist": false, "char_ranges": [], "word_ranges": [], "text": ""}, "5": {"exist": true, "char_ranges": [[0, 127]], "word_ranges": [[0, 23]], "text": "La HbA1c nos servir\u00eda en este paciente para el diagn\u00f3stico de diabetes pero no para la afectaci\u00f3n sobre \u00f3rgano diana de la HTA."}}} {"id": 22, "year": 2011, "question_id_specific": 112, "full_question": "Un paciente acude al Servicio de Urgencias con Fiebre sin foco de 24 h de evoluci\u00f3n, hipoTA e impresi\u00f3n de gravedad. Sufri\u00f3 una esplenectom\u00eda hace 2 a\u00f1os para estadificaci\u00f3n de enfermedad de Hosgkin. \u00bfQu\u00e9 microorganismo se implica con mayor frecuencia?", "full_answer": "Otra de epidemiolog\u00eda. En pacientes esplenectomizados, el riesgo de adquisi\u00f3n de infecciones graves por g\u00e9rmenes encapsulados, se encuentra aumentado, siendo nuevamente Neumococo el m\u00e1s frecuente.", "type": "INFECCIOSAS", "options": {"1": "Streptococcus pneumoniae.", "2": "Pseudomonas aeruginosa.", "3": "Salmonella no typhi.", "4": "Pneumocystis jirovecii.", "5": "Streptococcus viridans."}, "correct_option": 1, "explanations": {"1": {"exist": true, "char_ranges": [[23, 196]], "word_ranges": [[3, 25]], "text": "En pacientes esplenectomizados, el riesgo de adquisi\u00f3n de infecciones graves por g\u00e9rmenes encapsulados, se encuentra aumentado, siendo nuevamente Neumococo el m\u00e1s frecuente."}, "2": {"exist": false, "char_ranges": [], "word_ranges": [], "text": ""}, "3": {"exist": false, "char_ranges": [], "word_ranges": [], "text": ""}, "4": {"exist": false, "char_ranges": [], "word_ranges": [], "text": ""}, "5": {"exist": false, "char_ranges": [], "word_ranges": [], "text": ""}}} {"id": 488, "year": 2020, "question_id_specific": 105, "full_question": "Paciente de 67 a\u00f1os que en los \u00faltimos 6 meses, en dos anal\u00edticas de rutina, presenta linfocitosis progresiva. En la \u00faltima, hemoglobina 15,4 g/dl; leucocitos 18,5 x103/\u03bcL con 82 % de linfocitos maduros que por citometr\u00eda de flujo expresan los ant\u00edgenos CD5/CD19/CD23 y plaquetas 240 x103/\u03bcL. \u00bfQu\u00e9 actitud le parece correcta?:", "full_answer": "Pregunta que creo que puede ser complicada para un opositor MIR. Nos hablan de una leucocitosis a expensas de linfocitos con fenotipo de LLC-B. Parece que no hay otros datos de alarma. No nos dan datos de cl\u00ednica (as\u00ed que suponemos que el paciente est\u00e1 asintom\u00e1tico). No hay citopenias\u2026 Conclusi\u00f3n: SEGUIMIENTO. La principal duda con la respuesta deber\u00eda ser con la opci\u00f3n 1. \u00bfPor qu\u00e9 considero que es incorrecta? Porque la mutaci\u00f3n TP53, aunque establece pron\u00f3stico, no est\u00e1 indicada hacerla al diagn\u00f3stico, sino cuando se va a iniciar tratamiento (en primera l\u00ednea y previo a sucesivas l\u00edneas en caso de necesitarlas). Se podr\u00eda rizar mucho el rizo en esta pregunta en la que el porcentaje que nos da, no se refiere directamente a linfocitos clonales, con lo que incluso podr\u00eda tratarse de una linfocitosis B monoclonal. Pero esto es excedernos.", "type": "HEMATOLOGIA", "options": {"1": "Estudio de mutaciones de TP53 para establecer el pron\u00f3stico.", "2": "Aspirado/biopsia \u00f3sea para confirmar el diagn\u00f3stico.", "3": "PET/CT para establecer la actitud terap\u00e9utica.", "4": "Nuevo control cl\u00ednico y anal\u00edtico en 6 meses.", "5": NaN}, "correct_option": 4, "explanations": {"1": {"exist": true, "char_ranges": [[421, 620]], "word_ranges": [[70, 101]], "text": "la mutaci\u00f3n TP53, aunque establece pron\u00f3stico, no est\u00e1 indicada hacerla al diagn\u00f3stico, sino cuando se va a iniciar tratamiento (en primera l\u00ednea y previo a sucesivas l\u00edneas en caso de necesitarlas)."}, "2": {"exist": false, "char_ranges": [], "word_ranges": [], "text": ""}, "3": {"exist": false, "char_ranges": [], "word_ranges": [], "text": ""}, "4": {"exist": true, "char_ranges": [[65, 311]], "word_ranges": [[11, 51]], "text": "Nos hablan de una leucocitosis a expensas de linfocitos con fenotipo de LLC-B. Parece que no hay otros datos de alarma. No nos dan datos de cl\u00ednica (as\u00ed que suponemos que el paciente est\u00e1 asintom\u00e1tico). No hay citopenias\u2026 Conclusi\u00f3n: SEGUIMIENTO."}, "5": {"exist": false, "char_ranges": [], "word_ranges": [], "text": ""}}} {"id": 111, "year": 2012, "question_id_specific": 153, "full_question": "Ante una mujer de 59 a\u00f1os que consulta por secreci\u00f3n hem\u00e1tica por el pez\u00f3n derecho (unilateral), espont\u00e1nea y uniorificial sin nodularidad palpable. \u00bfCu\u00e1l es el diagnostico m\u00e1s probable?", "full_answer": "La respuesta correcta es la 2. Ante una telorrea hem\u00e1tica unilateral y uniorificial, la causa m\u00e1s frecuente es el papiloma intraductal (50%), seguida de la ectasia ductal y del carcinoma. Cuando se asocia a masa palpable, el 60% corresponden a un carcinoma. La enfermedad de Paget es de tipo eccematoso. En hiperprolactinemia la telorrea es bilateral y pluriorificial. La mamograf\u00eda a menudo no muestra los papilomas, siendo m\u00e1s \u00fatiles la ecografia y la galactograf\u00eda.", "type": "GINECOLOG\u00cdA Y OBSTETRICIA", "options": {"1": "Dado el car\u00e1cter hem\u00e1tico de la secreci\u00f3n el diagn\u00f3stico m\u00e1s probable es el de carcinoma ductal infiltrante.", "2": "Papiloma intraductal.", "3": "Enfermedad de Paget del pez\u00f3n.", "4": "Hiperprolactinemia tumoral.", "5": "La mamograf\u00eda nos indicar\u00e1 el diagn\u00f3stico."}, "correct_option": 2, "explanations": {"1": {"exist": true, "char_ranges": [[188, 257]], "word_ranges": [[30, 42]], "text": "Cuando se asocia a masa palpable, el 60% corresponden a un carcinoma."}, "2": {"exist": true, "char_ranges": [[31, 140]], "word_ranges": [[6, 22]], "text": "Ante una telorrea hem\u00e1tica unilateral y uniorificial, la causa m\u00e1s frecuente es el papiloma intraductal (50%),"}, "3": {"exist": true, "char_ranges": [[258, 303]], "word_ranges": [[42, 50]], "text": "La enfermedad de Paget es de tipo eccematoso."}, "4": {"exist": true, "char_ranges": [[304, 368]], "word_ranges": [[50, 58]], "text": "En hiperprolactinemia la telorrea es bilateral y pluriorificial."}, "5": {"exist": true, "char_ranges": [[369, 468]], "word_ranges": [[58, 74]], "text": "La mamograf\u00eda a menudo no muestra los papilomas, siendo m\u00e1s \u00fatiles la ecografia y la galactograf\u00eda."}}} {"id": 119, "year": 2012, "question_id_specific": 119, "full_question": "Mujer de 17 a\u00f1os que acude a urgencias por un cuadro de fiebre elevada, dolor far\u00edngeo y adenopat\u00edas cervicales. Previamente hab\u00eda sido diagnosticada de faringitis aguda y recibi\u00f3 tratamiento con amoxicilina, presentado posteriormente un exantema cut\u00e1neo maculoso generalizado. Se realiza anal\u00edtica que presenta ligera leucocitosis con linfocitosis y presencia de linfocitos activados, ligera trombopenia y transaminasas levemente aumentadas. \u00bfCu\u00e1l ser\u00eda el diagn\u00f3stico m\u00e1s probable de este cuadro cl\u00ednico?", "full_answer": "Desde luego s\u00ed que es un cuadro t\u00edpico (VEB, CMV, VH6, Toxoplasma), de los que luego no ver\u00e9is tan claramente en la puerta de urgencias cuando est\u00e9is de guardia. Ojo con la mala leche de la respuesta 3, pues una toxoplasmosis aguda puede comportarse as\u00ed y adem\u00e1s da un exantema macular, aunque el de esta chica es en relaci\u00f3n con la toma de amoxicilina. Por cierto, a\u00f1ado un apunte, si en vez de una chica de 17 a\u00f1os tan clara fuera un/una joven con relaciones sexuales de riesgo, no olvid\u00e9is la primoinfecci\u00f3n por VIH como causa de s\u00edndrome mononucle\u00f3sico.", "type": "INFECTOLOG\u00cdA", "options": {"1": "Es un cuadro t\u00edpico de mononucleosis infecciosa.", "2": "Infecci\u00f3n por virus varicela z\u00f3ster.", "3": "Toxoplasmosis aguda.", "4": "Enfermedad de Lyme.", "5": "Infecci\u00f3n por virus herpes 8."}, "correct_option": 1, "explanations": {"1": {"exist": false, "char_ranges": [], "word_ranges": [], "text": ""}, "2": {"exist": false, "char_ranges": [], "word_ranges": [], "text": ""}, "3": {"exist": true, "char_ranges": [[208, 353]], "word_ranges": [[39, 64]], "text": "una toxoplasmosis aguda puede comportarse as\u00ed y adem\u00e1s da un exantema macular, aunque el de esta chica es en relaci\u00f3n con la toma de amoxicilina."}, "4": {"exist": false, "char_ranges": [], "word_ranges": [], "text": ""}, "5": {"exist": false, "char_ranges": [], "word_ranges": [], "text": ""}}} {"id": 562, "year": 2022, "question_id_specific": 183, "full_question": "Mujer de 45 a\u00f1os sin antecedentes de inter\u00e9s que consulta por presentar sensaci\u00f3n disneica de unos 4 d\u00edas de evoluci\u00f3n. La exploraci\u00f3n muestra taquicardia r\u00edtmica, sin soplos y la auscultaci\u00f3n pulmonar es normal. La gasometr\u00eda arterial muestra una pO2 de 70 mmHg y una pCO2 32 mmHg. El hemograma, la funci\u00f3n renal y hep\u00e1tica son normales. Tiempo de protrombina 90 %, tiempo de tromboplastina parcial activada (TTPA) con ratio de 2 respecto al control (N <1,2). \u00bfDe los siguientes diagn\u00f3sticos cu\u00e1l es el m\u00e1s probable?:", "full_answer": "En esta pregunta lo que asumen es que se trata de una mujer de 45 a\u00f1os a la que se le diagnostica un tromboembolismo pulmonar (TEP). Con este diagn\u00f3stico, se excluyen directamente las opciones 3 y 4 (adem\u00e1s, es mujer, as\u00ed que no tendr\u00edamos que pensar en la hemofilia) Como dato anal\u00edtico, dice que hay un alargamiento del TTPA. Por lo tanto, quieren que sepas, que tiene ac l\u00fapico positivo y que lo relaciones directamente con el s\u00edndrome antifosfol\u00edpido (SAF). Un Leiden no altera la coagulaci\u00f3n, as\u00ed que se descarta tamb\u00eden De todos modos, aunque no entra dentro de las posibles respuestas, la positividad a l\u00fapicos se pueden relacionar con un sinf\u00edn de situaciones cl\u00ednicas, s\u00edndromes y patolog\u00edas. Asumir que un TEP + l\u00fapico es un SAF, me parece un tanto exagerado, pero es lo m\u00e1s probable con los datos que se aportan.", "type": "HEMATOLOGIA", "options": {"1": "S\u00edndrome antifosfol\u00edpido.", "2": "Factor V de Leiden.", "3": "Hemofilia.", "4": "Pericarditis aguda.", "5": NaN}, "correct_option": 1, "explanations": {"1": {"exist": true, "char_ranges": [[268, 461]], "word_ranges": [[49, 80]], "text": "Como dato anal\u00edtico, dice que hay un alargamiento del TTPA. Por lo tanto, quieren que sepas, que tiene ac l\u00fapico positivo y que lo relaciones directamente con el s\u00edndrome antifosfol\u00edpido (SAF)."}, "2": {"exist": true, "char_ranges": [[462, 525]], "word_ranges": [[80, 91]], "text": "Un Leiden no altera la coagulaci\u00f3n, as\u00ed que se descarta tamb\u00eden"}, "3": {"exist": true, "char_ranges": [[0, 266]], "word_ranges": [[0, 49]], "text": "En esta pregunta lo que asumen es que se trata de una mujer de 45 a\u00f1os a la que se le diagnostica un tromboembolismo pulmonar (TEP). Con este diagn\u00f3stico, se excluyen directamente las opciones 3 y 4 (adem\u00e1s, es mujer, as\u00ed que no tendr\u00edamos que pensar en la hemofilia)"}, "4": {"exist": true, "char_ranges": [[0, 267]], "word_ranges": [[0, 49]], "text": "En esta pregunta lo que asumen es que se trata de una mujer de 45 a\u00f1os a la que se le diagnostica un tromboembolismo pulmonar (TEP). Con este diagn\u00f3stico, se excluyen directamente las opciones 3 y 4 (adem\u00e1s, es mujer, as\u00ed que no tendr\u00edamos que pensar en la hemofilia)"}, "5": {"exist": false, "char_ranges": [], "word_ranges": [], "text": ""}}} {"id": 514, "year": 2021, "question_id_specific": 97, "full_question": "Tras un accidente de tr\u00e1fico usted atiende a un herido. El paciente abre los ojos a la llamada, pero \u00fanicamente emite palabras que son inapropiadas. En la exploraci\u00f3n motora localiza el dolor en extremidades derechas, pero extiende al estimular las extremidades izquierdas. \u00bfC\u00f3mo describir\u00eda su situaci\u00f3n seg\u00fan la escala de coma de Glasgow?:", "full_answer": "Lo primero que debemos saber al aplicar la escala de coma de Glasgow en un paciente es que siempre debemos quedarnos con el mejor valor en cada esfera (respuesta ocular, respuesta motora y respuesta verbal). En este caso, tenemos un paciente que tiene apertura ocular a la llamada (respuesta ocular 3 sobre 4), emite palabras inapropiadas (respuesta verbal 3 sobre 5) y localiza al dolor con extremidades derechas (respuesta motora 5 sobre 6). La puntuaci\u00f3n en la escala de coma de Glasgow ser\u00eda de 11 (sobre 15). Se toma el mejor valor de la exploraci\u00f3n porque en muchos casos (valoraci\u00f3n de paciente politraumatizado, ictus isqu\u00e9mico o sangrado cerebral agudo espont\u00e1neo), la exploraci\u00f3n motora puede estar artefactada por otros elementos. Por ejemplo: en un paciente politraumatizado (como el que nos ata\u00f1e), la respuesta que presenta con extremidades izquierdas puede ser debida a traumatismo localizado en dicha zona, y no propiamente a una lesi\u00f3n cerebral traum\u00e1tica subyacente. Igualmente, en ocasiones se puede infraestimar la respuesta verbal en el paciente politraumatizado por obstrucci\u00f3n de la v\u00eda a\u00e9rea (por ejemplo, por retroceso de la lengua hacia la orofaringe). Al valorar siempre la mejor respuesta, tendremos una visi\u00f3n m\u00e1s real del estado del paciente.", "type": "CUIDADOS CR\u00cdTICOS Y URGENCIAS", "options": {"1": "E3V2M4.", "2": "E3V3M5.", "3": "E2V3M4.", "4": "E2V4M5.", "5": NaN}, "correct_option": 2, "explanations": {"1": {"exist": false, "char_ranges": [], "word_ranges": [], "text": ""}, "2": {"exist": false, "char_ranges": [], "word_ranges": [], "text": ""}, "3": {"exist": false, "char_ranges": [], "word_ranges": [], "text": ""}, "4": {"exist": false, "char_ranges": [], "word_ranges": [], "text": ""}, "5": {"exist": false, "char_ranges": [], "word_ranges": [], "text": ""}}} {"id": 101, "year": 2012, "question_id_specific": 86, "full_question": "Una mujer de 55 a\u00f1os consulta por fatiga y poliuria de dos a\u00f1os de evoluci\u00f3n. Los an\u00e1lisis revelan hipercalcemia, aumento de la PTH e incremento de los marcadores de recambio \u00f3seo. El estudio radiogr\u00e1fico muestra resorci\u00f3n subperi\u00f3stica y osteoporosis. La gammagraf\u00eda con sestamibi pone de manifiesto un adenoma paratiroideo. Se efect\u00faa paratiroidectom\u00eda con cirug\u00eda m\u00ednimamente invasiva. En el postoperatorio la enferma desarrolla hipocalcemia intensa y tetania, siendo la PTH inferior a 5 ng/l. La enferma responde bien al tratamiento inicial con calcio intravenoso y posteriormente con calcio y vitamina D orales \u00bfCu\u00e1l es el diagn\u00f3stico m\u00e1s probable?", "full_answer": "Pregunta de dificultad muy alta, yo no me preocupar\u00eda por haberla fallado. Es f\u00e1cil confundirse con la respuesta de hipoparatiroidismo. Es raro un hipoparatiroidismo postquir\u00fargico permanente tras una paratiroidectomia de un adenoma con cirug\u00eda m\u00ednimamente invasiva. Recordad adem\u00e1s que para que sea permanente debe durar permanencer 6 meses tras la intervenci\u00f3n.Os escribo un breve resumen del s\u00edndrome de hueso ambriento que he encontrado en un caso cl\u00ednico de los anales de medicina interna [1]: \"La fisiopatolog\u00eda del s\u00edndrome del hueso hambriento (SHH) est\u00e1 relacionada con un disbalance entre la formaci\u00f3n y resorci\u00f3n \u00f3sea, que se asocia a hipocalcemia, hipofosfatemia e hipomagnesemia (1). Este s\u00edndrome se puede observar tras la cirug\u00eda de pacientes con hiperparatiroidismo (HPT) primario, as\u00ed como en el HPT terciario de la insuficiencia renal cr\u00f3nica, y en menor medida tras tratamiento de procesos que cursan con exceso de hormonas tiroideas circulantes (2). En el HPT, existe un exceso de hormona paratiroidea (PTH) que estimula la actividad osteocl\u00e1stica causando desmineralizaci\u00f3n de la matriz \u00f3sea y liberando calcio al torrente sangu\u00edneo. Tras paratiroidectomia, los niveles s\u00e9ricos de PTH caen dram\u00e1ticamente, por ello, la resorci\u00f3n \u00f3sea inducida por la PTH cesa, mientras que la actividad osteobl\u00e1stica contin\u00faa dando como resultado un incremento en la captaci\u00f3n \u00f3sea de calcio, fosfato y magnesio, apareciendo as\u00ed el SHH (3).\"", "type": "ENDOCRINOLOG\u00cdA", "options": {"1": "Hipoparatiroidismo quir\u00fargico permanente.", "2": "Transfusi\u00f3n de sangre citratada.", "3": "Insuficiencia de vitamina D.", "4": "S\u00edndrome del hueso hambriento.", "5": "Osteomalacia."}, "correct_option": 4, "explanations": {"1": {"exist": true, "char_ranges": [[136, 266]], "word_ranges": [[20, 36]], "text": "Es raro un hipoparatiroidismo postquir\u00fargico permanente tras una paratiroidectomia de un adenoma con cirug\u00eda m\u00ednimamente invasiva."}, "2": {"exist": false, "char_ranges": [], "word_ranges": [], "text": ""}, "3": {"exist": false, "char_ranges": [], "word_ranges": [], "text": ""}, "4": {"exist": true, "char_ranges": [[500, 691]], "word_ranges": [[74, 101]], "text": "\"La fisiopatolog\u00eda del s\u00edndrome del hueso hambriento (SHH) est\u00e1 relacionada con un disbalance entre la formaci\u00f3n y resorci\u00f3n \u00f3sea, que se asocia a hipocalcemia, hipofosfatemia e hipomagnesemia"}, "5": {"exist": false, "char_ranges": [], "word_ranges": [], "text": ""}}} {"id": 230, "year": 2014, "question_id_specific": 179, "full_question": "Acude al centro de salud un ni\u00f1os de 4 a\u00f1os que, 5 minutos antes, comienza con cuadro de angioedema en cara, conjuntivitis, congesti\u00f3n nasal y ronquera, coincidiendo con la ingesta de una cucharada de yogur que le dieron por error en el colegio. Entre los antecedentes est\u00e1 diagnosticado de alergia a prote\u00ednas de leche de vaca. En la exploraci\u00f3n se constata hipotensi\u00f3n leve, frecuencia cardiaca 110 lat/min, Sat O2 93%, est\u00e1 p\u00e1lido y algo sudoroso, con sibilancias diseminadas. \u00bfCu\u00e1l es el primer tratamiento de elecci\u00f3n?", "full_answer": "Estamos ante un caso de Anafilaxia por lo que la primera medida es administrar Adrenalina 1/1000 intramuscular (0,01mg/kg) Posteriormente, se dar\u00e1n corticoides sist\u00e9micos como la metilprednisolona, que tardar\u00e1 unas horas en hacer efecto. As\u00ed que primero adrenalina, por ser de acci\u00f3n m\u00e1s r\u00e1pida, despu\u00e9s el resto. (Pregunta muy similar a una del a\u00f1o pasado, en lugar de yogur, fue tortilla).", "type": "PEDIATR\u00cdA", "options": {"1": "Provocar el v\u00f3mito.", "2": "Adrenalina 1/1000 subcut\u00e1nea.", "3": "Adrenalina 1/1000 intramuscular.", "4": "Metilprednisolona intramuscular.", "5": "Salbutamol nebulizado."}, "correct_option": 3, "explanations": {"1": {"exist": false, "char_ranges": [], "word_ranges": [], "text": ""}, "2": {"exist": false, "char_ranges": [], "word_ranges": [], "text": ""}, "3": {"exist": true, "char_ranges": [[246, 294]], "word_ranges": [[35, 43]], "text": "primero adrenalina, por ser de acci\u00f3n m\u00e1s r\u00e1pida,"}, "4": {"exist": true, "char_ranges": [[123, 237]], "word_ranges": [[18, 33]], "text": "Posteriormente, se dar\u00e1n corticoides sist\u00e9micos como la metilprednisolona, que tardar\u00e1 unas horas en hacer efecto."}, "5": {"exist": false, "char_ranges": [], "word_ranges": [], "text": ""}}} {"id": 479, "year": 2020, "question_id_specific": 164, "full_question": "Hombre de 72 a\u00f1os que presenta desde hace 48 horas fiebre de 38,7\u00b0C, tos, expectoraci\u00f3n purulenta y disnea. Constantes: TA 85/60 mmHg, frecuencia cardiaca 100 lpm, frecuencia respiratoria 35 rpm, SatO2 80%. Destaca la presencia de confusi\u00f3n y de crepitantes en campo pulmonar inferior izquierdo. La radiograf\u00eda de t\u00f3rax confirma la existencia de una neumon\u00eda de la l\u00edngula y del l\u00f3bulo inferior izquierdo. \u00bfCu\u00e1l ser\u00eda el lugar de atenci\u00f3n adecuado?:", "full_answer": "Aplicando la escala de gravedad CURB-65 (una de las m\u00e1s utilizadas en urgencias para la estratificaci\u00f3n de la gravedad de la neumon\u00eda, aunque en este caso simplemente aplicando el sentido com\u00fan, el paciente debe ingresar en UCI), tendr\u00edamos una puntuaci\u00f3n de al menos 4 puntos (nos faltar\u00eda el dato de la urea s\u00e9rica), lo cual nos da criterios de gravedad. Al no darnos ning\u00fan criterio que contraindique su admisi\u00f3n en UCI estar\u00eda indicado el ingreso en la unidad.", "type": "CUIDADOS CR\u00cdTICOS", "options": {"1": "Tratamiento ambulatorio en su domicilio.", "2": "Ingreso en una unidad de observaci\u00f3n de urgencias.", "3": "Ingreso hospitalario en planta.", "4": "Ingreso en la Unidad de Cuidados Intensivos.", "5": NaN}, "correct_option": 4, "explanations": {"1": {"exist": false, "char_ranges": [], "word_ranges": [], "text": ""}, "2": {"exist": false, "char_ranges": [], "word_ranges": [], "text": ""}, "3": {"exist": false, "char_ranges": [], "word_ranges": [], "text": ""}, "4": {"exist": true, "char_ranges": [[0, 228]], "word_ranges": [[0, 37]], "text": "Aplicando la escala de gravedad CURB-65 (una de las m\u00e1s utilizadas en urgencias para la estratificaci\u00f3n de la gravedad de la neumon\u00eda, aunque en este caso simplemente aplicando el sentido com\u00fan, el paciente debe ingresar en UCI),"}, "5": {"exist": false, "char_ranges": [], "word_ranges": [], "text": ""}}} {"id": 515, "year": 2021, "question_id_specific": 101, "full_question": "Var\u00f3n de 15 a\u00f1os de edad, ingresado tras haber sido atropellado en la v\u00eda p\u00fablica, resultando despedido. Ingresa consciente y orientado, con intenso dolor en hemit\u00f3rax izquierdo y disnea. En la exploraci\u00f3n f\u00edsica se halla tensi\u00f3n arterial 90/60 mmHg, frecuencia cardiaca 130 lpm, saturaci\u00f3n basal de ox\u00edgeno 90 %, frecuencia respiratoria 35 rpm. Hay hipofonesis completa en el hemit\u00f3rax izquierdo y matidez a la percusi\u00f3n. En la radiograf\u00eda de t\u00f3rax se ven m\u00faltiples fracturas costales izquierdas y derrame pleural ipsilateral masivo. Se indica la colocaci\u00f3n de un drenaje pleural, con salida de 1700 cc de l\u00edquido hem\u00e1tico. \u00bfCu\u00e1l es la decisi\u00f3n que se debe tomar?:", "full_answer": "Hay dos respuestas que podemos descartar inmediatamente si nos atenemos al manejo inicial del traumatismo tor\u00e1cico grave: la 1 (la ventilaci\u00f3n no invasiva no est\u00e1 indicada porque no asegura la v\u00eda a\u00e9rea) y la 3 (la observaci\u00f3n en estos casos, esperando el empeoramiento, s\u00f3lo conlleva un aumento de la morbimortalidad). Entre la opci\u00f3n 2 y la 4 podr\u00eda haber controversia entre la literatura y la pr\u00e1ctica cl\u00ednica habitual, en la que normalmente tenemos que enfrentar en un delicado equilibrio la estabilidad cl\u00ednica del paciente (y lo que podemos llegar a \u201cmaquillar\u201d dicha estabilidad con nuestras terapias de soporte vital) y las actuaciones que deriven en una mejor atenci\u00f3n al paciente para el tratamiento definitivo de las lesiones. Nos encontramos ante un paciente politraumatizado grave. Con los datos que nos aporta la pregunta, un paciente que impresiona de encontrarse en shock (taquicardia, hipoxemia, taquipnea\u2026 aunque no nos dan datos, por ejemplo, de lactacidemia), y en el que tenemos hasta donde sabemos un traumatismo tor\u00e1cico izquierdo, que no s\u00f3lo implica lesi\u00f3n \u00f3sea, sino que puede enmascarar otras lesiones de elevada gravedad y que no se diagnostican con una radiograf\u00eda de t\u00f3rax: contusi\u00f3n mioc\u00e1rdica, derrame peric\u00e1rdico, lesi\u00f3n espl\u00e9nica, lesi\u00f3n diafragm\u00e1tica\u2026 Al paciente le colocamos un drenaje pleural izquierdo y obtenemos un d\u00e9bito de 1700cc de l\u00edquido hem\u00e1tico. Si seguimos las gu\u00edas de atenci\u00f3n al politraumatizado grave de la ATLS (ATLS-Advanced Trauma Life Support 10th edition), nos dicen que un sangrado inmediato de m\u00e1s de 1500cc de sangre, es indicaci\u00f3n de toracotom\u00eda urgente, y que incluso en pacientes con un sangrado menor a 1500cc, si el d\u00e9bito persiste a raz\u00f3n de 200mL/h durante 2 a 4h, igualmente existir\u00eda esta indicaci\u00f3n. Por tanto, el que ha hecho la pregunta del MIR quiere que respondamos la opci\u00f3n 2. Ahora bien, en la vida real, la pauta de actuaci\u00f3n va a depender de la estabilidad del paciente cuando hayamos aplicado los tratamientos que se indican en la opci\u00f3n 2: adecuada analgesia, oxigenoterapia e inicio de trasfusi\u00f3n sangu\u00ednea. Si optimizando al paciente conseguimos estabilizarlo y valoramos que es seguro trasladarlo, la realizaci\u00f3n de un bodyTC va a definir con much\u00edsimo m\u00e1s detalle las lesiones que tiene, y va a ayudar a que la cirug\u00eda a la que probablemente se acabe sometiendo, sea una cirug\u00eda definitiva y no \u00fanicamente cirug\u00eda de control de da\u00f1os. En este caso, surgir\u00eda la duda con la opci\u00f3n 4, aunque, como digo, todo depender\u00e1 de la estabilidad cl\u00ednica del paciente y el tiempo que tengamos antes de conseguir el control definitivo del foco hemorr\u00e1gico (que es, al fin y al cabo, lo que perseguimos en esta situaci\u00f3n).", "type": "CUIDADOS CR\u00cdTICOS Y URGENCIAS", "options": {"1": "Instaurar ventilaci\u00f3n mec\u00e1nica no invasiva y solicitar transfusi\u00f3n sangu\u00ednea.", "2": "Analgesia, oxigenoterapia, solicitar transfusi\u00f3n sangu\u00ednea e indicar una intervenci\u00f3n quir\u00fargica urgente.", "3": "Intubaci\u00f3n orotraqueal, solicitar transfusi\u00f3n sangu\u00ednea y observaci\u00f3n para, en caso de empeoramiento, indicar la intervenci\u00f3n quir\u00fargica.", "4": "Intubaci\u00f3n orotraqueal y realizaci\u00f3n de una TC urgente para valoraci\u00f3n exacta de las lesiones.", "5": NaN}, "correct_option": 2, "explanations": {"1": {"exist": true, "char_ranges": [[128, 202]], "word_ranges": [[19, 32]], "text": "(la ventilaci\u00f3n no invasiva no est\u00e1 indicada porque no asegura la v\u00eda a\u00e9rea)"}, "2": {"exist": true, "char_ranges": [[1854, 2090]], "word_ranges": [[292, 330]], "text": "Ahora bien, en la vida real, la pauta de actuaci\u00f3n va a depender de la estabilidad del paciente cuando hayamos aplicado los tratamientos que se indican en la opci\u00f3n 2: adecuada analgesia, oxigenoterapia e inicio de trasfusi\u00f3n sangu\u00ednea."}, "3": {"exist": true, "char_ranges": [[213, 317]], "word_ranges": [[35, 50]], "text": "(la observaci\u00f3n en estos casos, esperando el empeoramiento, s\u00f3lo conlleva un aumento de la morbimortalidad)."}, "4": {"exist": true, "char_ranges": [[2091, 2420]], "word_ranges": [[330, 385]], "text": "Si optimizando al paciente conseguimos estabilizarlo y valoramos que es seguro trasladarlo, la realizaci\u00f3n de un bodyTC va a definir con much\u00edsimo m\u00e1s detalle las lesiones que tiene, y va a ayudar a que la cirug\u00eda a la que probablemente se acabe sometiendo, sea una cirug\u00eda definitiva y no \u00fanicamente cirug\u00eda de control de da\u00f1os."}, "5": {"exist": false, "char_ranges": [], "word_ranges": [], "text": ""}}} {"id": 234, "year": 2014, "question_id_specific": 105, "full_question": "Acude al Servicio de Urgencias un hombre de 72 a\u00f1os con fractura patol\u00f3gica en f\u00e9mur izquierdo. Tras la intervenci\u00f3n quir\u00fargica, se realiza un estudio diagn\u00f3stico para averiguar la patolog\u00eda subyacente con los siguientes hallazgos: hemoglobina 9.5 g/dl, protenias totales 11 g/dl, (VN: 6-8 g/dl), alb\u00famina s\u00e9rica 2 g/dl, (VN 3.5-5,0 g/dl), beta 2 microglobulina 6 mg/l (VN 1,1-2,4 mg/l), creatinina s\u00e9rica 1,8 mg/dl (VN: 0,1-1,4 mg/dl). Indique cu\u00e1les ser\u00edan las pruebas diagn\u00f3sticas necesarias para confirmar el diagn\u00f3stico m\u00e1s probable:", "full_answer": "Va a tener un mieloma. Para diagn\u00f3stico, una muestra (m\u00e9dula \u00f3sea) y electroforesis para determinar el tipo de pico monoclonal. Luego, para extensi\u00f3n, valor pron\u00f3stico y decidir tratamientos, lo dem\u00e1s, pero una vez diagnosticado.", "type": "HEMATOLOG\u00cdA", "options": {"1": "Serie \u00f3sea radiol\u00f3gica y aspirado de m\u00e9dula \u00f3sea.", "2": "Electroforesis s\u00e9rica y urinaria y pruebas de funci\u00f3n renal.", "3": "Aspirado de m\u00e9dula \u00f3sea y concentracion de calcio s\u00e9rico.", "4": "Aspirado de m\u00e9dula \u00f3sea y electroforesis s\u00e9rica y urinaria.", "5": "Biopsia de la fractura patol\u00f3gica y serie \u00f3sea radiol\u00f3gica."}, "correct_option": 4, "explanations": {"1": {"exist": false, "char_ranges": [], "word_ranges": [], "text": ""}, "2": {"exist": false, "char_ranges": [], "word_ranges": [], "text": ""}, "3": {"exist": false, "char_ranges": [], "word_ranges": [], "text": ""}, "4": {"exist": true, "char_ranges": [[0, 127]], "word_ranges": [[0, 20]], "text": "Va a tener un mieloma. Para diagn\u00f3stico, una muestra (m\u00e9dula \u00f3sea) y electroforesis para determinar el tipo de pico monoclonal."}, "5": {"exist": false, "char_ranges": [], "word_ranges": [], "text": ""}}} {"id": 174, "year": 2013, "question_id_specific": 52, "full_question": "Un hombre de 67 a\u00f1os consulta por disnea de moderados esfuerzos de aparici\u00f3n progresiva en los \u00faltimos a\u00f1os. Tiene expectoraci\u00f3n blanquecina diaria y a veces ruidos respiratorios, especialmente en invierno con las infecciones respiratorias. Ha sido fumador de unos 20 cigarrillos diarios durante los \u00faltimos 45 a\u00f1os. En la exploraci\u00f3n se encuentra eupneico, normocoloreado, con una saturaci\u00f3n de ox\u00edgeno del 94%, y presenta disminuci\u00f3n generalizada del murmullo vesicular en la auscultaci\u00f3n tor\u00e1cica como \u00fanicos hallazgos de inter\u00e9s. La radiograf\u00eda de t\u00f3rax muestra una silueta cardiaca alargada, con signos de hiperinsuflaci\u00f3n o atrapamiento a\u00e9reo pulmonar, sin otras alteraciones. Se realiza una espirometr\u00eda el siguiente resultado: FVC 84%, FEV1 58%, FEV1/FVC 61%, sin cambios tras broncodilatador. Entre los se\u00f1alados, indique el tratamiento m\u00e1s adecuado para este paciente:", "full_answer": "En un paciente con una EPOC poco sintom\u00e1tico que se podr\u00eda clasificar probablemente como GOLD 2, el tratamiento de inicio puede ser tanto con tiotropio inhalado como con un beta dos de larga duraci\u00f3n. Por lo tanto no hay duda en la respuesta correcta.", "type": "NEUMOLOG\u00cdA", "options": {"1": "Corticoide inhalado diario.", "2": "Oxigenoterapia domiciliaria con fuente port\u00e1til de oxigeno para deambulaci\u00f3n.", "3": "Antagonista de leucotrienos por v\u00eda oral.", "4": "Corticoide oral durante tres meses.", "5": "Tiotropio inhalado."}, "correct_option": 5, "explanations": {"1": {"exist": false, "char_ranges": [], "word_ranges": [], "text": ""}, "2": {"exist": false, "char_ranges": [], "word_ranges": [], "text": ""}, "3": {"exist": false, "char_ranges": [], "word_ranges": [], "text": ""}, "4": {"exist": false, "char_ranges": [], "word_ranges": [], "text": ""}, "5": {"exist": true, "char_ranges": [[0, 200]], "word_ranges": [[0, 34]], "text": "En un paciente con una EPOC poco sintom\u00e1tico que se podr\u00eda clasificar probablemente como GOLD 2, el tratamiento de inicio puede ser tanto con tiotropio inhalado como con un beta dos de larga duraci\u00f3n."}}} {"id": 143, "year": 2012, "question_id_specific": 141, "full_question": "Ante un ni\u00f1o de 7 meses con fiebre e irritabilidad, fontanela abombada y un estudio de l\u00edquido cefalorraqu\u00eddeo con 110 c\u00e9lulas/mm3 (75% linfocitos), prote\u00ednas 120 mg/dl y glucosa 28 mg/dl (glucemia s\u00e9rica 89 mg/dl), \u00bfcu\u00e1l es la sospecha diagn\u00f3stica m\u00e1s razonable?", "full_answer": "La respuesta correcta es la 3. Define un l\u00edquido cefalorraqu\u00eddeo t\u00edpico de tuberculosis. En la pr\u00e1ctica cl\u00ednica las cosas quiz\u00e1 no ser\u00edan tan sencillas\u2026..", "type": "PEDIATR\u00cdA", "options": {"1": "Meningitis v\u00edrica.", "2": "Meningitis bacteriana.", "3": "Meningitis tuberculosa.", "4": "S\u00edndrome mononucle\u00f3sico.", "5": "S\u00edndrome de Guillain-Barr\u00e9."}, "correct_option": 3, "explanations": {"1": {"exist": false, "char_ranges": [], "word_ranges": [], "text": ""}, "2": {"exist": false, "char_ranges": [], "word_ranges": [], "text": ""}, "3": {"exist": true, "char_ranges": [[31, 88]], "word_ranges": [[6, 13]], "text": "Define un l\u00edquido cefalorraqu\u00eddeo t\u00edpico de tuberculosis."}, "4": {"exist": false, "char_ranges": [], "word_ranges": [], "text": ""}, "5": {"exist": false, "char_ranges": [], "word_ranges": [], "text": ""}}} {"id": 9, "year": 2011, "question_id_specific": 36, "full_question": "Un paciente de 87 a\u00f1os con antecedentes de bronquitis cr\u00f3nica e insuficiencia cardiaca, ha sido diagnosticado de colecistitis aguda liti\u00e1sica . Tras cuatro d\u00edas de tratamiento con dieta absoluta, sueroterapia y piperaciliana /tazobactam, el paciente continua con fiebre, dolor abdominal persistente y leucocitosis. La actitud m\u00e1s adecuada en este momento ser\u00eda:", "full_answer": "Creo que la respuesta correcta es la 2 aunque el tratamiento de la colecistitis aguda es la colecistectom\u00eda, para esto es necesario que el paciente sea un candidato quir\u00fargico, en este caso se trata de una paciente anciano, con enfermedades previas que aumentan el riesgo quir\u00fargico y ser\u00eda un ASA IV para una intervenci\u00f3n urgente. En estos casos la colecistostom\u00eda puede curar la colecistitis; y tras la recuperaci\u00f3n, valorar la cirug\u00eda programada seg\u00fan la situaci\u00f3n del paciente.", "type": "CIRUG\u00cdA", "options": {"1": "Tratamiento quir\u00fargico (colecistectom\u00eda urgente).", "2": "Drenaje biliar mediante colecistostom\u00eda percut\u00e1nea.", "3": "Sustituir la piperaciliana /tazobactam por metronidazol+cefotaxima.", "4": "Sustituir la piperaciliana /tazobactam por amikacina+clindamicina.", "5": "A\u00f1adir gentamicina."}, "correct_option": 2, "explanations": {"1": {"exist": false, "char_ranges": [], "word_ranges": [], "text": ""}, "2": {"exist": true, "char_ranges": [[0, 331]], "word_ranges": [[0, 55]], "text": "Creo que la respuesta correcta es la 2 aunque el tratamiento de la colecistitis aguda es la colecistectom\u00eda, para esto es necesario que el paciente sea un candidato quir\u00fargico, en este caso se trata de una paciente anciano, con enfermedades previas que aumentan el riesgo quir\u00fargico y ser\u00eda un ASA IV para una intervenci\u00f3n urgente."}, "3": {"exist": false, "char_ranges": [], "word_ranges": [], "text": ""}, "4": {"exist": false, "char_ranges": [], "word_ranges": [], "text": ""}, "5": {"exist": false, "char_ranges": [], "word_ranges": [], "text": ""}}} {"id": 601, "year": 2022, "question_id_specific": 112, "full_question": "Mujer de 61 a\u00f1os, administrativa, con antecedentes de sobrepeso, hipertensi\u00f3n, dislipemia y s\u00edndrome metab\u00f3lico, que consulta por dolor en ambas nalgas, regi\u00f3n trocant\u00e9rea izquierda, cara lateral del muslo izquierdo hasta la rodilla y pierna izquierda hasta el tercio medio. El dolor aparece cuando levantamos el miembro inferior con la rodilla extendida, pero se alivia al flexionar la rodilla. \u00bfCu\u00e1l es la primera sospecha cl\u00ednica?:", "full_answer": "Signo de Lasegue positivo, reaparici\u00f3n de la cl\u00ednica al ejecutar la maniobra de extensi\u00f3n del miembro afectado, compatible con afectaci\u00f3n de ra\u00edces nerviosas a nivel de columna lumbosacra.", "type": "TRAUMATOLOG\u00cdA", "options": {"1": "Artritis gotosa de cadera izquierda.", "2": "Artrosis coxofemoral izquierda.", "3": "Lumbalgia irradiada / lumbociatalgia.", "4": "Claudicaci\u00f3n por estenosis de canal.", "5": NaN}, "correct_option": 3, "explanations": {"1": {"exist": false, "char_ranges": [], "word_ranges": [], "text": ""}, "2": {"exist": false, "char_ranges": [], "word_ranges": [], "text": ""}, "3": {"exist": true, "char_ranges": [[0, 188]], "word_ranges": [[0, 28]], "text": "Signo de Lasegue positivo, reaparici\u00f3n de la cl\u00ednica al ejecutar la maniobra de extensi\u00f3n del miembro afectado, compatible con afectaci\u00f3n de ra\u00edces nerviosas a nivel de columna lumbosacra."}, "4": {"exist": false, "char_ranges": [], "word_ranges": [], "text": ""}, "5": {"exist": false, "char_ranges": [], "word_ranges": [], "text": ""}}} {"id": 202, "year": 2013, "question_id_specific": 44, "full_question": "Paciente con pancreatitis aguda. En Tomograf\u00eda Axial Computerizada (TAC) realizada a las 72 horas de ingreso se aprecia una necrosis del 50% del p\u00e1ncreas. A la 3\u00aa semana de ingreso el paciente comienza con fiebre elevada y leucocitosis. Se solicita radiograf\u00eda de t\u00f3rax urgente y sedimento urinario, siendo ambos normales. Se\u00f1ale el siguiente paso a seguir:", "full_answer": "aunque las \u00faltimas recomendaciones sobre el manejo de la pancreatitis aguda grave desaconsejan la PAAF por el riesgo de infecci\u00f3n del necrosis est\u00e9ril. El manejo actual se rige por el tratamiento escalonado. En la situaci\u00f3n descrita en la pregunta el tratamiento actual es comenzar con antibi\u00f3ticos y valorar respuesta.", "type": "CIRUG\u00cdA GENERAL", "options": {"1": "Punci\u00f3n aspiraci\u00f3n con aguja fina de necrosis pancre\u00e1tica guiada por ecograf\u00eda o TAC.", "2": "Aspirado broncoalveolar, cultivo de orina y hemocultivos.", "3": "Resonancia Magn\u00e9tica Nuclear abdominal.", "4": "Colecistectom\u00eda urgente.", "5": "Ecoendoscopia con punci\u00f3n de la necrosis."}, "correct_option": 1, "explanations": {"1": {"exist": false, "char_ranges": [], "word_ranges": [], "text": ""}, "2": {"exist": false, "char_ranges": [], "word_ranges": [], "text": ""}, "3": {"exist": false, "char_ranges": [], "word_ranges": [], "text": ""}, "4": {"exist": false, "char_ranges": [], "word_ranges": [], "text": ""}, "5": {"exist": false, "char_ranges": [], "word_ranges": [], "text": ""}}} {"id": 259, "year": 2014, "question_id_specific": 97, "full_question": "Hombre e 54 a\u00f1os que acude a revisi\u00f3n en su empresa. Se detecta un \u00edndice de masa corporal de 32,8 kg/m2 y glucemia en ayunas 138 mg/l. Un mes despu\u00e9s, glucemia 13 mg/dl. \u00bfQu\u00e9 recomendaci\u00f3n terap\u00e9utica efectuar\u00eda en primer lugar?", "full_answer": "Cambios conductuales: dieta y ejercicio f\u00edsico. Es lo primero que hacemos ante el diagn\u00f3stico de una diabetes tipo 2. Pregunta bonita para recordar que antes de cualquier f\u00e1rmaco, hay que insistir en los cambios de h\u00e1bitos. Seguidamente pautar\u00edamos Metformina.", "type": "ENDOCRINOLOG\u00cdA", "options": {"1": "Administrar metformina.", "2": "Prescribir una sulfonilurea.", "3": "Cambios conductuales. Dieta y ejercicio f\u00edsico.", "4": "Insulina antes de cada comida.", "5": "Tomar acarbosa por la noche, antes de acostarse."}, "correct_option": 3, "explanations": {"1": {"exist": false, "char_ranges": [], "word_ranges": [], "text": ""}, "2": {"exist": false, "char_ranges": [], "word_ranges": [], "text": ""}, "3": {"exist": true, "char_ranges": [[48, 117]], "word_ranges": [[6, 19]], "text": "Es lo primero que hacemos ante el diagn\u00f3stico de una diabetes tipo 2."}, "4": {"exist": false, "char_ranges": [], "word_ranges": [], "text": ""}, "5": {"exist": false, "char_ranges": [], "word_ranges": [], "text": ""}}} {"id": 204, "year": 2014, "question_id_specific": 220, "full_question": "Un hombre de 43 a\u00f1os consulta por s\u00edndrome diarreico, y refiere entre sus antecedentes 3 neumon\u00edas en la edad adulta. \u00bfCu\u00e1l de los siguientes estudios inmunol\u00f3gicos debemos solicitar?:", "full_answer": "M\u00e1s de 2 neumon\u00edas lobares hacen necesario descartar un d\u00e9ficit inmunol\u00f3gico.", "type": "GEN\u00c9TICA E INMUNOLOG\u00cdA", "options": {"1": "Recuento de inmunoglobulinas s\u00e9ricas y test de capacidad de producci\u00f3n de anticuerpos.", "2": "Test de fagocitosis y metabolismo oxidativo de los neutr\u00f3filos.", "3": "Test de apoptosis (muerte celular programada) en los linfocitos circulantes del paciente.", "4": "Estudio del repertorio y clonalidad de los linfocitos T (alfa/beta).", "5": "En este paciente no proceder\u00eda solicitar estudio inmunol\u00f3gico alguno."}, "correct_option": 1, "explanations": {"1": {"exist": true, "char_ranges": [[0, 77]], "word_ranges": [[0, 11]], "text": "M\u00e1s de 2 neumon\u00edas lobares hacen necesario descartar un d\u00e9ficit inmunol\u00f3gico."}, "2": {"exist": false, "char_ranges": [], "word_ranges": [], "text": ""}, "3": {"exist": false, "char_ranges": [], "word_ranges": [], "text": ""}, "4": {"exist": false, "char_ranges": [], "word_ranges": [], "text": ""}, "5": {"exist": false, "char_ranges": [], "word_ranges": [], "text": ""}}} {"id": 4, "year": 2011, "question_id_specific": 40, "full_question": "Un hombre presenta se forma brusca astenia e ictericia detect\u00e1ndose unas transaminasas superiores a 2000 UI/L Los marcadores serol\u00f3gicos muestran el siguiente patr\u00f3n: Anti VHA IgM negativo, HBsAg negativo AntiHBc IgM positivo, anti VHC negativo. \u00bfCu\u00e1l es el diagn\u00f3stico?", "full_answer": "Una interesante de algo que en general hay que estudiarse. Por la forma de presentaci\u00f3n, es claramente una hepatitis aguda con lo que la 1 se descarta. La IgM de VHA negativa descarta la 5 y el HBs Ag negativo la 4 (para tener virus delta necesitas ant\u00edgeno de superficie B). Al tener IgM anti HBc hablamos de una hepatitis aguda B (3), ya que hay un periodo ventana de negativizaci\u00f3n del ant\u00edgeno.", "type": "DIGESTIVO", "options": {"1": "Hepatitis cr\u00f3nica B.", "2": "Hepatitis aguda no viral.", "3": "Hepatitis aguda B.", "4": "Sobreinfecci\u00f3n por virus D (delta).", "5": "Hepatitis aguda A y B."}, "correct_option": 3, "explanations": {"1": {"exist": true, "char_ranges": [[59, 151]], "word_ranges": [[10, 27]], "text": "Por la forma de presentaci\u00f3n, es claramente una hepatitis aguda con lo que la 1 se descarta."}, "2": {"exist": false, "char_ranges": [], "word_ranges": [], "text": ""}, "3": {"exist": true, "char_ranges": [[276, 398]], "word_ranges": [[51, 73]], "text": "Al tener IgM anti HBc hablamos de una hepatitis aguda B (3), ya que hay un periodo ventana de negativizaci\u00f3n del ant\u00edgeno."}, "4": {"exist": true, "char_ranges": [[191, 275]], "word_ranges": [[36, 51]], "text": "el HBs Ag negativo la 4 (para tener virus delta necesitas ant\u00edgeno de superficie B)."}, "5": {"exist": true, "char_ranges": [[152, 188]], "word_ranges": [[27, 35]], "text": "La IgM de VHA negativa descarta la 5"}}} {"id": 372, "year": 2016, "question_id_specific": 127, "full_question": "Hombre de 55 a\u00f1os que consulta por disfon\u00eda. En la anamnesis refiere llevar un mes con astenia y p\u00e9rdida de peso no cuantificada. La radiograf\u00eda de t\u00f3rax presenta un aumento de densidad en l\u00f3bulo superior izquierdo y ocupaci\u00f3n de la ventana aortopulmonar. La broncoscopia constata una par\u00e1lisis de la cuerda vocal izquierda, sin imagen endosc\u00f3pica sugestiva de neoplasia. \u00bfCu\u00e1l es el diagn\u00f3stico m\u00e1s probable?:", "full_answer": "La causa mas frecuente de par\u00e1lisis del nervio recurrente es el carcinoma broncopulmonar. Que no sea accesible a la visi\u00f3n endosc\u00f3pica no quiere decir que no este.", "type": "NEUMOLOG\u00cdA Y CIRUG\u00cdA TOR\u00c1CICA", "options": {"1": "Neoplasia pulmonar.", "2": "Sarcoidosis.", "3": "Silicosis.", "4": "Tuberculosis.", "5": NaN}, "correct_option": 1, "explanations": {"1": {"exist": true, "char_ranges": [[0, 89]], "word_ranges": [[0, 13]], "text": "La causa mas frecuente de par\u00e1lisis del nervio recurrente es el carcinoma broncopulmonar."}, "2": {"exist": false, "char_ranges": [], "word_ranges": [], "text": ""}, "3": {"exist": false, "char_ranges": [], "word_ranges": [], "text": ""}, "4": {"exist": false, "char_ranges": [], "word_ranges": [], "text": ""}, "5": {"exist": false, "char_ranges": [], "word_ranges": [], "text": ""}}} {"id": 377, "year": 2016, "question_id_specific": 131, "full_question": "Una paciente de 65 a\u00f1os debut\u00f3 dos a\u00f1os antes con una apraxia del habla y evolutivamente ha desarrollado un parkinsonismo rigido-acin\u00e9tico de predominio en hemicuerpo derecho con mioclonias sobreimpuestas y fen\u00f3meno de mano alien\u00edgena o miembro extra\u00f1o. \u00bfQu\u00e9 diagn\u00f3stico considera m\u00e1s probable?", "full_answer": "Por como nos lo describen como un fen\u00f3meno apr\u00e1xico que se sigue de un parkinsonismo r\u00edgido-acin\u00e9tico asim\u00e9trico y con mioclon\u00edas sobrea\u00f1adidas es t\u00edpico de la degeneraci\u00f3n c\u00f3rticobasal. La enfermedad de parkinson no puede ser porque no presentan apraxia como rasgo inicial, el Alzheimer no presenta un parkinsonismo asim\u00e9trico y el Hungtinton t\u00edpico no es de personas tan mayores y cuando debuta con edad avanzada se comporta como una corea.", "type": "NEUROLOG\u00cdA", "options": {"1": "Enfermedad de Parkinson.", "2": "Degeneraci\u00f3n corticobasal.", "3": "Enfermedad de Alzheimer.", "4": "Enfermedad de Huntington.", "5": NaN}, "correct_option": 2, "explanations": {"1": {"exist": true, "char_ranges": [[187, 273]], "word_ranges": [[27, 41]], "text": "La enfermedad de parkinson no puede ser porque no presentan apraxia como rasgo inicial,"}, "2": {"exist": true, "char_ranges": [[0, 186]], "word_ranges": [[0, 27]], "text": "Por como nos lo describen como un fen\u00f3meno apr\u00e1xico que se sigue de un parkinsonismo r\u00edgido-acin\u00e9tico asim\u00e9trico y con mioclon\u00edas sobrea\u00f1adidas es t\u00edpico de la degeneraci\u00f3n c\u00f3rticobasal."}, "3": {"exist": true, "char_ranges": [[275, 327]], "word_ranges": [[41, 48]], "text": "el Alzheimer no presenta un parkinsonismo asim\u00e9trico"}, "4": {"exist": true, "char_ranges": [[331, 442]], "word_ranges": [[49, 69]], "text": "el Hungtinton t\u00edpico no es de personas tan mayores y cuando debuta con edad avanzada se comporta como una corea."}, "5": {"exist": false, "char_ranges": [], "word_ranges": [], "text": ""}}} {"id": 145, "year": 2012, "question_id_specific": 143, "full_question": "Ni\u00f1o de 1 mes, alimentado con lactancia materna exclusiva, consulta porque realiza una deposici\u00f3n cada 5 \u00f3 6 d\u00edas con esfuerzo pero de consistencia blanda, \u00bfcu\u00e1l ser\u00eda nuestra actitud?", "full_answer": "La respuesta correcta es la 4. La normalidad en la emisi\u00f3n de deposiciones en los lactantes peque\u00f1os es muy amplia. Para sospechar un megacolon agangli\u00f3nico suele haber un retraso en la evacuaci\u00f3n del meconio y con frecuencia otros s\u00edntomas asociados como distensi\u00f3n abdominal, dificultad para ganar peso, etc.", "type": "PEDIATR\u00cdA", "options": {"1": "Iniciar tratamiento laxante.", "2": "Suplementar la lactancia materna con f\u00f3rmula antiestre\u00f1imiento.", "3": "Iniciar estimulaci\u00f3n rectal diaria.", "4": "Se considera ritmo intestinal normal.", "5": "Se deriva a Digestivo Infantil para descartar megacolon agangli\u00f3nico."}, "correct_option": 4, "explanations": {"1": {"exist": false, "char_ranges": [], "word_ranges": [], "text": ""}, "2": {"exist": false, "char_ranges": [], "word_ranges": [], "text": ""}, "3": {"exist": false, "char_ranges": [], "word_ranges": [], "text": ""}, "4": {"exist": true, "char_ranges": [[31, 115]], "word_ranges": [[6, 20]], "text": "La normalidad en la emisi\u00f3n de deposiciones en los lactantes peque\u00f1os es muy amplia."}, "5": {"exist": true, "char_ranges": [[116, 310]], "word_ranges": [[20, 48]], "text": "Para sospechar un megacolon agangli\u00f3nico suele haber un retraso en la evacuaci\u00f3n del meconio y con frecuencia otros s\u00edntomas asociados como distensi\u00f3n abdominal, dificultad para ganar peso, etc."}}} {"id": 567, "year": 2022, "question_id_specific": 150, "full_question": "Mujer de 47 a\u00f1os que consulta por s\u00edndrome miccional. Se le diagnostica de infecci\u00f3n urinaria no complicada y se le trata con ciprofloxacino durante 5 d\u00edas. A la semana vuelve a consultar por mal estado general, artralgias y aparici\u00f3n de un rash cut\u00e1neo. En la anal\u00edtica destaca un filtrado glomerular de 45 ml/min (CKD-EPI), siendo el de hace 6 meses de 100 ml/min. En el sedimento se identifican leucocitos, siendo negativo para nitritos. \u00bfDe las siguientes, cu\u00e1l es la causa m\u00e1s probable de su insuficiencia renal?:", "full_answer": "El cuadro de malestar general, artralgias, rash cut\u00e1neo, fracaso renal agudo y sedimento activo con leucocituria est\u00e9ril siguiendo a la toma de un f\u00e1rmaco es altamente sugestivo de NIA (opci\u00f3n 4 correcta). Dentro de los f\u00e1rmacos, los antibi\u00f3ticos son la causa m\u00e1s frecuente, y el ciprofloxacino uno de los habituales. La nefrotoxicidad directa es muy rara y se caracteriza por la cristalizaci\u00f3n en los t\u00fabulos renales (opci\u00f3n 2 incorrecta). La glomerulonefritis postinfecciosa es m\u00e1s propia de ni\u00f1os y adultos mayores de 60 a\u00f1os, y suele manifestarse con hematuria tras infecciones estreptoc\u00f3cicas (opci\u00f3n 1 incorrecta). La pielonefritis aguda nos la habr\u00edan presentado con fiebre alta y dolor en fosa renal (opci\u00f3n 3 incorrecta).", "type": "NEFROLOG\u00cdA", "options": {"1": "Glomerulonefritis postinfecciosa.", "2": "Nefrotoxicidad por ciprofloxacino.", "3": "Pielonefritis aguda.", "4": "Nefritis intersticial aguda.", "5": NaN}, "correct_option": 4, "explanations": {"1": {"exist": true, "char_ranges": [[441, 620]], "word_ranges": [[69, 94]], "text": "La glomerulonefritis postinfecciosa es m\u00e1s propia de ni\u00f1os y adultos mayores de 60 a\u00f1os, y suele manifestarse con hematuria tras infecciones estreptoc\u00f3cicas (opci\u00f3n 1 incorrecta)."}, "2": {"exist": true, "char_ranges": [[318, 440]], "word_ranges": [[50, 69]], "text": "La nefrotoxicidad directa es muy rara y se caracteriza por la cristalizaci\u00f3n en los t\u00fabulos renales (opci\u00f3n 2 incorrecta)."}, "3": {"exist": true, "char_ranges": [[621, 730]], "word_ranges": [[94, 112]], "text": "La pielonefritis aguda nos la habr\u00edan presentado con fiebre alta y dolor en fosa renal (opci\u00f3n 3 incorrecta)."}, "4": {"exist": true, "char_ranges": [[0, 205]], "word_ranges": [[0, 32]], "text": "El cuadro de malestar general, artralgias, rash cut\u00e1neo, fracaso renal agudo y sedimento activo con leucocituria est\u00e9ril siguiendo a la toma de un f\u00e1rmaco es altamente sugestivo de NIA (opci\u00f3n 4 correcta)."}, "5": {"exist": false, "char_ranges": [], "word_ranges": [], "text": ""}}} {"id": 177, "year": 2013, "question_id_specific": 93, "full_question": "A una mujer de 30 a\u00f1os, asintom\u00e1tica, en un examen rutinario se le detecta anemia. En la exploraci\u00f3n f\u00edsica se aprecia ictericia conjuntival y esplenomegalia. La paciente refiere historia familiar de litiasis biliar en edades tempranas. Todo ello sugiere el diagn\u00f3stico m\u00e1s probable de:", "full_answer": "Seguramente se os haya ido la vista a la respuesta 5, esferocitosis hereditaria, asociada a litiasis biliar a temprana edad. Exacto, \u00e9sa es la correcta. En el enunciado no han parado de dar pistas: ictericia conjuntival, esplenomegalia e historia familiar de litiasis biliar a temperana edad. Ha llegado a los 30 a\u00f1os sin sufrir una crisis hemol\u00edtica, sin sufrir ning\u00fan s\u00edntoma. No han hablado de microcitosis o macrocitosis para orientar a una talasemia minor o un d\u00e9ficit de B12 y/o f\u00f3lico. Ni de crisis hemol\u00edtica asociada al consumo de f\u00e1rmacos, infecciones o ingesta de habas o guisantes. No han dado pistas para pararse a pensar en otras posibilidades.", "type": "HEMATOLOG\u00cdA", "options": {"1": "D\u00e9ficit de glucosa 6-fosfato deshidrogenasa.", "2": "Talasemia minor.", "3": "Deficiencia de vitamina Bl2 y/o \u00e1cido f\u00f3lico.", "4": "D\u00e9ficit familiar de piruvatokinasa.", "5": "Esferocitosis hereditaria."}, "correct_option": 5, "explanations": {"1": {"exist": false, "char_ranges": [], "word_ranges": [], "text": ""}, "2": {"exist": false, "char_ranges": [], "word_ranges": [], "text": ""}, "3": {"exist": false, "char_ranges": [], "word_ranges": [], "text": ""}, "4": {"exist": false, "char_ranges": [], "word_ranges": [], "text": ""}, "5": {"exist": true, "char_ranges": [[198, 593]], "word_ranges": [[34, 97]], "text": "ictericia conjuntival, esplenomegalia e historia familiar de litiasis biliar a temperana edad. Ha llegado a los 30 a\u00f1os sin sufrir una crisis hemol\u00edtica, sin sufrir ning\u00fan s\u00edntoma. No han hablado de microcitosis o macrocitosis para orientar a una talasemia minor o un d\u00e9ficit de B12 y/o f\u00f3lico. Ni de crisis hemol\u00edtica asociada al consumo de f\u00e1rmacos, infecciones o ingesta de habas o guisantes."}}} {"id": 125, "year": 2012, "question_id_specific": 105, "full_question": "Mujer de 19 a\u00f1os, peso 60 kg, con deshidrataci\u00f3n aguda por larga exposici\u00f3n al sol. Presi\u00f3n arterial tumbada 100/60 mmHg. De pie, 70/50 mmHg con sensaci\u00f3n de mareo. Niveles de sodio s\u00e9rico 155 mmol/L. \u00bfCu\u00e1l es el tratamiento m\u00e1s correcto, en las primeras 24 horas, teniendo en cuenta los datos de los que disponemos?", "full_answer": "Esta pregunta no es f\u00e1cil. La literatura disponible es algo controvertida (unas gu\u00edas aconsejan usar suero glucosado, otras lo proscriben absolutamente\u2026) por lo que he decidido basarme en el tratamiento recomendado por el Harrison, que es el manual en el que suelen basarse los examinadores del MIR. Esta chica presenta una hipernatremia por p\u00e9rdidas de agua libre extrarrenales. El objetivo es reponer el agua perdida y reducir la natremia a una velocidad no superior a 12 mmol/d\u00eda. Para calcular la fluidoterapia, hay que calcular primero el d\u00e9ficit de agua. D\u00e9f. Agua = (Natremia-140/140)xAgua corporal total. ACT = Peso en kg x 0,4 en mujeres/0,5 en varones. En este caso, el d\u00e9ficit de agua es: ([[tel:155-140/140|155-140/140]])x60x0,4 = 2,57 litros. S\u00f3lo hay dos respuestas que se aproximen a esta cifra, pero la 2 parece m\u00e1s correcta, pues es la \u00fanica que cubre completamente el d\u00e9ficit de agua y utiliza una soluci\u00f3n hipos\u00f3dica.", "type": "NEFROLOG\u00cdA", "options": {"1": "Suero salino hipert\u00f3nico (3%), 500 ml + 500 ml de glucosado de 5%.", "2": "Suero hiposalino (0.45%), 3000 ml.", "3": "Suero glucosado 5% 1000 ml.", "4": "Hidrataci\u00f3n oral con 1 litro de agua.", "5": "Suero salino isot\u00f3nico (0.9%), 2000 ml."}, "correct_option": 2, "explanations": {"1": {"exist": false, "char_ranges": [], "word_ranges": [], "text": ""}, "2": {"exist": true, "char_ranges": [[663, 936]], "word_ranges": [[106, 150]], "text": "En este caso, el d\u00e9ficit de agua es: ([[tel:155-140/140|155-140/140]])x60x0,4 = 2,57 litros. S\u00f3lo hay dos respuestas que se aproximen a esta cifra, pero la 2 parece m\u00e1s correcta, pues es la \u00fanica que cubre completamente el d\u00e9ficit de agua y utiliza una soluci\u00f3n hipos\u00f3dica."}, "3": {"exist": false, "char_ranges": [], "word_ranges": [], "text": ""}, "4": {"exist": false, "char_ranges": [], "word_ranges": [], "text": ""}, "5": {"exist": true, "char_ranges": [[663, 936]], "word_ranges": [[106, 150]], "text": "En este caso, el d\u00e9ficit de agua es: ([[tel:155-140/140|155-140/140]])x60x0,4 = 2,57 litros. S\u00f3lo hay dos respuestas que se aproximen a esta cifra, pero la 2 parece m\u00e1s correcta, pues es la \u00fanica que cubre completamente el d\u00e9ficit de agua y utiliza una soluci\u00f3n hipos\u00f3dica."}}} {"id": 510, "year": 2021, "question_id_specific": 151, "full_question": "Var\u00f3n de 43 a\u00f1os sin antecedentes de inter\u00e9s que consulta por un cuadro de diez d\u00edas de evoluci\u00f3n de ictericia de piel y mucosas, coluria y acolia. No refiere dolor abdominal, p\u00e9rdida de peso ni otra cl\u00ednica, salvo prurito generalizado. Niega consumo de alcohol. No toma medicaci\u00f3n salvo ibuprofeno ocasional por dolores musculares tras actividad deportiva. La anal\u00edtica muestra una bilirrubina aumentada a expensas de directa. Una ecograf\u00eda urgente muestra un h\u00edgado normal, colesterolosis vesicular y ausencia de dilataci\u00f3n de v\u00eda biliar, sin otras alteraciones de inter\u00e9s. \u00bfCu\u00e1l es el diagn\u00f3stico m\u00e1s probable?:", "full_answer": "Creemos que esta pregunta es impugnable. La hepatitis por ibuprofeno es un efecto adverso raro con una prevalencia de alrededor 3.7/100.000 usuarios y de 1.1/100.000 prescripciones. Parece que puede estar relacionado con la dosis y con la duraci\u00f3n del tratamiento, pero existe poca evidencia sobre ello. Dentro de los AINEs que mayor riesgo presentan est\u00e1n el diclofenaco y el sulindac. Atendiendo a t\u00e9rminos de frecuencia, la prevalencia de colangiocarcinoma es de 2/100.000 pacientes, siendo una frecuencia muy equiparable. La ausencia de dilataci\u00f3n de la v\u00eda biliar en colangiocarcinomas puede verse en casos de colangiocarcinomas intrahep\u00e1ticos o en aquellos con histolog\u00eda mixta con hepatocarcinoma. Es cierto que el curso agudo y la ausencia de s\u00edndrome constitucional puede hacernos pensar en contra de esta opci\u00f3n e inclinar la balanza hacia la hepatitis por ibuprofeno. Respecto a las otras opciones: una colangitis aguda se caracteriza por la tr\u00edada ictericia, dolor abdominal y fiebre; el s\u00edndrome de Gilbert es un aumento de bilirrubina a expensas de indirecta.", "type": "DIGESTIVO", "options": {"1": "Colangitis aguda por barro biliar.", "2": "Hepatitis t\u00f3xica por ibuprofeno.", "3": "Colangiocarcinoma.", "4": "S\u00edndrome de Gilbert.", "5": NaN}, "correct_option": 2, "explanations": {"1": {"exist": true, "char_ranges": [[910, 996]], "word_ranges": [[137, 150]], "text": "una colangitis aguda se caracteriza por la tr\u00edada ictericia, dolor abdominal y fiebre;"}, "2": {"exist": true, "char_ranges": [[705, 878]], "word_ranges": [[103, 132]], "text": "Es cierto que el curso agudo y la ausencia de s\u00edndrome constitucional puede hacernos pensar en contra de esta opci\u00f3n e inclinar la balanza hacia la hepatitis por ibuprofeno."}, "3": {"exist": true, "char_ranges": [[705, 878]], "word_ranges": [[103, 132]], "text": "Es cierto que el curso agudo y la ausencia de s\u00edndrome constitucional puede hacernos pensar en contra de esta opci\u00f3n e inclinar la balanza hacia la hepatitis por ibuprofeno."}, "4": {"exist": true, "char_ranges": [[997, 1073]], "word_ranges": [[150, 163]], "text": "el s\u00edndrome de Gilbert es un aumento de bilirrubina a expensas de indirecta."}, "5": {"exist": false, "char_ranges": [], "word_ranges": [], "text": ""}}} {"id": 268, "year": 2014, "question_id_specific": 140, "full_question": "Paciente de 50 a\u00f1os que presenta un derrame pleural con las siguientes caracter\u00edsticas: aspecto pajizo, pH 7.3, cociente de prote\u00ednas pleura/suero 0.8, cociente de LDH pleura/suero 0.9, Gram y Ziehl negativos, l\u00edpidos totales, colesterol y triglic\u00e9ridos normales, c\u00e9lulas mesoteliales <5%, intensa linfocitosis sin atipias, ADA 64 U/l. \u00bfQu\u00e9 diagn\u00f3stico le sugiere?", "full_answer": "Es un exudado linfocitario cuyos principales diagnosticos son el tumoral y la pleuritis tuberculosa. No dan el valor de la glucosa que deberia de estar descendida, pero si la citologia que muestra ausencia de celulas malignas por lo que nos quedamos con la pleuritis tuberculosa. Otros diagnosticos posible como linfoma, no estan rese\u00f1ados en las respuestas.", "type": "NEUMOLOG\u00cdA", "options": {"1": "Empicma pleural.", "2": "Derrame pleural por insuficiencia cardiaca (trasudado).", "3": "Mesotelioma pleural.", "4": "Derrame pleural tuberculoso.", "5": "Derrame secundario a infarto pulmonar."}, "correct_option": 4, "explanations": {"1": {"exist": false, "char_ranges": [], "word_ranges": [], "text": ""}, "2": {"exist": false, "char_ranges": [], "word_ranges": [], "text": ""}, "3": {"exist": false, "char_ranges": [], "word_ranges": [], "text": ""}, "4": {"exist": true, "char_ranges": [[0, 279]], "word_ranges": [[0, 45]], "text": "Es un exudado linfocitario cuyos principales diagnosticos son el tumoral y la pleuritis tuberculosa. No dan el valor de la glucosa que deberia de estar descendida, pero si la citologia que muestra ausencia de celulas malignas por lo que nos quedamos con la pleuritis tuberculosa."}, "5": {"exist": false, "char_ranges": [], "word_ranges": [], "text": ""}}} {"id": 465, "year": 2020, "question_id_specific": 116, "full_question": "En un sujeto mayor de 65 a\u00f1os una prueba de tuberculina ha mostrado una induraci\u00f3n de 3 mm. La induraci\u00f3n en una segunda prueba, realizada 10 d\u00edas despu\u00e9s, es de 13 mm. Se\u00f1ale la respuesta correcta:", "full_answer": "Veamos, si ese sujeto no estaba dentro de ning\u00fan grupo de riesgo, el positivo est\u00e1 en los 15mm, por lo tanto las dos pruebas dan negativo, y como solo una respuesta refiere a negativos, ya est\u00e1. Pero podr\u00edan ser las dos falsos negativos.", "type": "BIOESTAD\u00cdSTICA", "options": {"1": "La primera reacci\u00f3n es un falso positivo.", "2": "La segunda reacci\u00f3n es un verdadero positivo.", "3": "La primera reacci\u00f3n es un verdadero negativo.", "4": "La segunda reacci\u00f3n es un falso positivo.", "5": NaN}, "correct_option": 3, "explanations": {"1": {"exist": false, "char_ranges": [], "word_ranges": [], "text": ""}, "2": {"exist": false, "char_ranges": [], "word_ranges": [], "text": ""}, "3": {"exist": true, "char_ranges": [[0, 194]], "word_ranges": [[0, 36]], "text": "Veamos, si ese sujeto no estaba dentro de ning\u00fan grupo de riesgo, el positivo est\u00e1 en los 15mm, por lo tanto las dos pruebas dan negativo, y como solo una respuesta refiere a negativos, ya est\u00e1."}, "4": {"exist": false, "char_ranges": [], "word_ranges": [], "text": ""}, "5": {"exist": false, "char_ranges": [], "word_ranges": [], "text": ""}}} {"id": 340, "year": 2016, "question_id_specific": 33, "full_question": "Mujer de 67 a\u00f1os diagnosticada de un carcinoma ductal infiltrante de mama y sin historia familiar de neoplasia. \u00bfQu\u00e9 estudios adicionales deben realizarse en el tumor por sus implicaciones cl\u00ednico-terap\u00e9uticas?", "full_answer": "La respuesta es la 2, estudio de receptores hormonales y de HER2. Ello es debido a que no procede ning\u00fan estudio de familiares de primer grado, ya que no parece que sea un c\u00e1ncer de mama hereditario puesto que no hay historia familiar de neoplasia. Por otro lado, no hay indicaci\u00f3n de estudio de BRCA 1-2.", "type": "GINECOLOG\u00cdA Y OBSTETRICIA", "options": {"1": "Estudio fenot\u00edpico completo mediante citometr\u00eda de flujo.", "2": "Estudio de receptores hormonales y de HER2.", "3": "Estudio de receptores hormonales, e-cadherina y estudio de familiares de primer grado.", "4": "Estudio de BRCA 1-2 y estudio de familiares de primer grado.", "5": NaN}, "correct_option": 2, "explanations": {"1": {"exist": false, "char_ranges": [], "word_ranges": [], "text": ""}, "2": {"exist": false, "char_ranges": [], "word_ranges": [], "text": ""}, "3": {"exist": true, "char_ranges": [[87, 248]], "word_ranges": [[17, 45]], "text": "no procede ning\u00fan estudio de familiares de primer grado, ya que no parece que sea un c\u00e1ncer de mama hereditario puesto que no hay historia familiar de neoplasia."}, "4": {"exist": true, "char_ranges": [[87, 248]], "word_ranges": [[17, 45]], "text": "no procede ning\u00fan estudio de familiares de primer grado, ya que no parece que sea un c\u00e1ncer de mama hereditario puesto que no hay historia familiar de neoplasia."}, "5": {"exist": false, "char_ranges": [], "word_ranges": [], "text": ""}}} {"id": 561, "year": 2022, "question_id_specific": 180, "full_question": "Mujer de 26 a\u00f1os diagnosticada de lupus eritematoso sist\u00e9mico, en tratamiento con hidroxicloroquina, que consulta por sensaci\u00f3n de debilidad generalizada que se ha instaurado progresivamente en los \u00faltimos 15 d\u00edas. En la exploraci\u00f3n f\u00edsica se aprecia palidez cut\u00e1nea y en la anal\u00edtica destaca Hb 7,4 gr/dL, Hcto 31 %,VCM 108. \u00bfCu\u00e1l de las siguientes determinaciones ser\u00e1 m\u00e1s \u00fatil para decidir la actuaci\u00f3n?:", "full_answer": "Nos habla de una paciente con diagn\u00f3stico de lupus y con una anemia macroc\u00edtica. Al final, todos los datos los suelen dar por algo. La instauraci\u00f3n del cuadro parece relativamente r\u00e1pida y adem\u00e1s asocia una enfermedad autoinmune, por lo que parece que quieren que lo asocies con una anemia hemol\u00edtica autoinmune. La haptoglobina baja nos dir\u00eda que es \u00abhemol\u00edtica\u00bb, el test de coombs nos dir\u00eda que es \u00abautoinmune\u00bb. La vitamina B12, probablemente el VCM ser\u00eda m\u00e1s elevado que 108 y la instauraci\u00f3n del cuadro cl\u00ednico ser\u00eda m\u00e1s lento. Los ANA no nos aportar\u00edan nada en relaci\u00f3n a la anemia, puesto que ya tiene un diagn\u00f3stico. Por lo tanto, respuesta correcta 2. Como anotaci\u00f3n, las anemias hemol\u00edticas autoinmunes suelen ser ligeramente macroc\u00edticas debido a la reticulocitosis. Recordad pedir reticulocitos.", "type": "HEMATOLOGIA", "options": {"1": "Haptoglobina.", "2": "Test de Coombs.", "3": "Vitamina B12.", "4": "Anticuerpos antinucleares.", "5": NaN}, "correct_option": 2, "explanations": {"1": {"exist": false, "char_ranges": [], "word_ranges": [], "text": ""}, "2": {"exist": true, "char_ranges": [[132, 413]], "word_ranges": [[24, 68]], "text": "La instauraci\u00f3n del cuadro parece relativamente r\u00e1pida y adem\u00e1s asocia una enfermedad autoinmune, por lo que parece que quieren que lo asocies con una anemia hemol\u00edtica autoinmune. La haptoglobina baja nos dir\u00eda que es \u00abhemol\u00edtica\u00bb, el test de coombs nos dir\u00eda que es \u00abautoinmune\u00bb."}, "3": {"exist": true, "char_ranges": [[414, 531]], "word_ranges": [[68, 88]], "text": "La vitamina B12, probablemente el VCM ser\u00eda m\u00e1s elevado que 108 y la instauraci\u00f3n del cuadro cl\u00ednico ser\u00eda m\u00e1s lento."}, "4": {"exist": true, "char_ranges": [[532, 624]], "word_ranges": [[88, 105]], "text": "Los ANA no nos aportar\u00edan nada en relaci\u00f3n a la anemia, puesto que ya tiene un diagn\u00f3stico."}, "5": {"exist": false, "char_ranges": [], "word_ranges": [], "text": ""}}} {"id": 451, "year": 2018, "question_id_specific": 175, "full_question": "Hombre de 36 a\u00f1os sin AP de inter\u00e9s que acude al servicio de urgencias por un cuadro de aparici\u00f3n brusca de intenso dolor en la cara posterior del miembro inferior derecho que llega hasta el pie. En la exploraci\u00f3n presenta: Las\u00e8ge derecho positivo a 10\u00ba, disminuci\u00f3n de la fuerza en la flexi\u00f3n plantar del pie derecho, hipoestesia en el borde externo del pie derecho y ausencia de reflejo aqu\u00edleo derecho. La Rx simple de columna lumbar no muestra alteraciones significativas. \u00bfCu\u00e1l de los siguientes es el diagn\u00f3stico m\u00e1s probable?:", "full_answer": "Cuadro de c\u00edatica aguda probablemente por extrusi\u00f3n discal. Tanto la distribuci\u00f3n sensitiva, como la paresia para la flexi\u00f3n plantar (puntillas) como la ausencia de reflejo aqu\u00edleo son t\u00edpicas de compromiso de la ra\u00edz S1, que se afecta t\u00edpicamente con la hernia discal L5/S1 (posterolateral). Una hernia (posterolateral) L1/L2 afectar\u00eda a la ra\u00edz L2, una hernia L4/L5 a la L5 y la cola de caballo implicar\u00eda m\u00e1s sintomatolog\u00eda que una simple afectaci\u00f3n S1.", "type": "NEUROCIRUG\u00cdA", "options": {"1": "Hernia discal L1/L2.", "2": "S\u00edndrome de cola de caballo.", "3": "Hernia discal L4/L5.", "4": "Hernia discal L5/S1.", "5": NaN}, "correct_option": 4, "explanations": {"1": {"exist": true, "char_ranges": [[293, 349]], "word_ranges": [[44, 53]], "text": "Una hernia (posterolateral) L1/L2 afectar\u00eda a la ra\u00edz L2,"}, "2": {"exist": true, "char_ranges": [[378, 456]], "word_ranges": [[60, 72]], "text": "la cola de caballo implicar\u00eda m\u00e1s sintomatolog\u00eda que una simple afectaci\u00f3n S1."}, "3": {"exist": true, "char_ranges": [[351, 375]], "word_ranges": [[53, 59]], "text": "una hernia L4/L5 a la L5"}, "4": {"exist": true, "char_ranges": [[0, 292]], "word_ranges": [[0, 44]], "text": "Cuadro de c\u00edatica aguda probablemente por extrusi\u00f3n discal. Tanto la distribuci\u00f3n sensitiva, como la paresia para la flexi\u00f3n plantar (puntillas) como la ausencia de reflejo aqu\u00edleo son t\u00edpicas de compromiso de la ra\u00edz S1, que se afecta t\u00edpicamente con la hernia discal L5/S1 (posterolateral)."}, "5": {"exist": false, "char_ranges": [], "word_ranges": [], "text": ""}}} {"id": 399, "year": 2016, "question_id_specific": 141, "full_question": "Chico de 20 a\u00f1os, que consulta por dolor lumbosacro de ritmo inflamatorio de 4 meses de evoluci\u00f3n. Tambi\u00e9n talalgia bilateral y rigidez matutina de 1 hora. En los \u00faltimos 2 meses aparici\u00f3n de cuadros diarreicos con p\u00e9rdida de 4 kg de peso. \u00bfCu\u00e1l es la aproximaci\u00f3n diagn\u00f3stica m\u00e1s correcta?", "full_answer": "El paciente presenta las caracter\u00edsticas cl\u00ednicas de una espondilartropat\u00eda, que a menudo se asocia a enfermedad inflamatoria intestinal, como sugiere este caso.", "type": "TRAUMATOLOG\u00cdA Y ORTOPED\u00cdA", "options": {"1": "Dada la edad del paciente, lo m\u00e1s probable es que padezca lumbalgia inespec\u00edfica y una tendinitis en los pies. Si persisten las diarreas realizar\u00eda estudio digestivo.", "2": "Realizar\u00eda estudio digestivo para descartar patolog\u00eda tumoral. El dolor lumbar puede ser debido a patolog\u00eda visceral.", "3": "El cuadro cl\u00ednico es muy sugestivo de espondiloartritis. Habr\u00eda que descartar enfermedad inflamatoria intestinal.", "4": "Solicitar\u00eda RMN lumbar para descartar hernia discal y si persiste diarrea, estudio digestivo.", "5": NaN}, "correct_option": 3, "explanations": {"1": {"exist": false, "char_ranges": [], "word_ranges": [], "text": ""}, "2": {"exist": false, "char_ranges": [], "word_ranges": [], "text": ""}, "3": {"exist": true, "char_ranges": [[0, 161]], "word_ranges": [[0, 22]], "text": "El paciente presenta las caracter\u00edsticas cl\u00ednicas de una espondilartropat\u00eda, que a menudo se asocia a enfermedad inflamatoria intestinal, como sugiere este caso."}, "4": {"exist": false, "char_ranges": [], "word_ranges": [], "text": ""}, "5": {"exist": false, "char_ranges": [], "word_ranges": [], "text": ""}}} {"id": 12, "year": 2011, "question_id_specific": 92, "full_question": "Un hombre de 30 a\u00f1os sufre una ca\u00edda de una altura de 2 metros de pie. A su llegada a urgencias en camilla consciente y orientado, con sondaje vesical con orina clara. Muestra tumefacci\u00f3n de tal\u00f3n izquierdo y hormigueos leves por cara anterior del muslo izquierdo y escroto as\u00ed como disestesias en ambos pies. El paciente se despide del camillero con un apret\u00f3n de manos con una palmada simult\u00e1nea en el hombro. La rotaci\u00f3n de los miembros inferiores en la camilla cogidos por los tobillos es indolora y la compresi\u00f3n de la pelvis es asintom\u00e1tica. A continuaci\u00f3n:", "full_answer": "Creo que la respuesta correcta es la 3 el paciente por el mecanismo lesional-ca\u00edda de pie- , por la fractura de calc\u00e1neo y por la sintomatolog\u00eda neurol\u00f3gica sugiere una lesi\u00f3n medular baja por fractura de la columna lumbar. La lesi\u00f3n no es alta porque mueve los brazos. Tampoco parece una lesi\u00f3n de la pelvis por la ausencia de dolor a la movilizaci\u00f3n o palpaci\u00f3n. Si hay una sospecha de lesi\u00f3n de la pelvis en la evaluaci\u00f3n primaria de todo politraumatizado (ATLS) se debe realizar una exploraci\u00f3n de la columna rot\u00e1ndolo en bloque para evitar producir m\u00e1s lesi\u00f3n. La palpaci\u00f3n abdominal tambi\u00e9n forma parte de la evaluaci\u00f3n inicial. La ecograf\u00eda abdominal puede estar indicada en pacientes inestables hemodin\u00e1micamente si se sospechan lesiones abdominales pero en todo caso se hace despu\u00e9s de la evaluaci\u00f3n inicial.", "type": "CIRUG\u00cdA", "options": {"1": "Lo sentaremos en la camilla para poder explorar la columna cervical.", "2": "Remitirlo a la radiolog\u00eda para realizar estudio de las extremidades inferiores (pies, f\u00e9mures, pelvis).", "3": "Palparemos el abdomen y rodaremos en bloque para palpar la columna toracolumbar.", "4": "Le colocaremos un collarin cervical y remitiremos a radiolog\u00eda para estudio de la imagen de extremidades inferiores y columna cervical.", "5": "Solicitaremos una ecograf\u00eda abdominal."}, "correct_option": 3, "explanations": {"1": {"exist": false, "char_ranges": [], "word_ranges": [], "text": ""}, "2": {"exist": false, "char_ranges": [], "word_ranges": [], "text": ""}, "3": {"exist": true, "char_ranges": [[0, 223]], "word_ranges": [[0, 38]], "text": "Creo que la respuesta correcta es la 3 el paciente por el mecanismo lesional-ca\u00edda de pie- , por la fractura de calc\u00e1neo y por la sintomatolog\u00eda neurol\u00f3gica sugiere una lesi\u00f3n medular baja por fractura de la columna lumbar."}, "4": {"exist": false, "char_ranges": [], "word_ranges": [], "text": ""}, "5": {"exist": false, "char_ranges": [], "word_ranges": [], "text": ""}}} {"id": 605, "year": 2022, "question_id_specific": 113, "full_question": "Var\u00f3n de 35 a\u00f1os, cartero, sin antecedentes de inter\u00e9s, que acude a urgencias por un dolor agudo cervical de 24 horas de evoluci\u00f3n, sin traumatismo previo, que irradia a brazo izquierdo hasta la mano y se acompa\u00f1a de parestesias en el borde radial del antebrazo. No presenta p\u00e9rdida de fuerza objetivable, conserva la movilidad del cuello aunque es dolorosa y se aprecia contractura de la musculatura paravertebral. La primera actitud ser\u00e1:", "full_answer": "En este caso nos describen un caso de cervicobraquialgia sin signos de alarma en el momento actual, por lo que el tratamiento m\u00e1s indicado en este momento ser\u00eda el conservador, basado en AINES, calor local y reposo. No nos encontramos ante un cuadro urgente por lo que la respuesta 2 no ser\u00eda v\u00e1lida. La realizaci\u00f3n de una RMN y un electromiograma son opciones v\u00e1lidas para el estudio etiol\u00f3gico del cuadro, sin embargo, es necesario destacar que primero deber\u00edamos realizar un correcto manejo del dolor, por lo que debido a que nos preguntan por la primera actitud, ser\u00eda m\u00e1s valida la opci\u00f3n 1.", "type": "TRAUMATOLOG\u00cdA", "options": {"1": "Tratamiento conservador con antiinflamatorios no esteroideos, calor local y reposo relativo.", "2": "Llamada urgente al neurocirujano para valoraci\u00f3n quir\u00fargica.", "3": "Solicitud preferente de resonancia magn\u00e9tica y electromiograma.", "4": "Derivaci\u00f3n preferente a consultas externas de traumatolog\u00eda.", "5": NaN}, "correct_option": 1, "explanations": {"1": {"exist": true, "char_ranges": [[0, 215]], "word_ranges": [[0, 37]], "text": "En este caso nos describen un caso de cervicobraquialgia sin signos de alarma en el momento actual, por lo que el tratamiento m\u00e1s indicado en este momento ser\u00eda el conservador, basado en AINES, calor local y reposo."}, "2": {"exist": true, "char_ranges": [[216, 300]], "word_ranges": [[37, 53]], "text": "No nos encontramos ante un cuadro urgente por lo que la respuesta 2 no ser\u00eda v\u00e1lida."}, "3": {"exist": true, "char_ranges": [[301, 596]], "word_ranges": [[53, 102]], "text": "La realizaci\u00f3n de una RMN y un electromiograma son opciones v\u00e1lidas para el estudio etiol\u00f3gico del cuadro, sin embargo, es necesario destacar que primero deber\u00edamos realizar un correcto manejo del dolor, por lo que debido a que nos preguntan por la primera actitud, ser\u00eda m\u00e1s valida la opci\u00f3n 1."}, "4": {"exist": false, "char_ranges": [], "word_ranges": [], "text": ""}, "5": {"exist": false, "char_ranges": [], "word_ranges": [], "text": ""}}} {"id": 443, "year": 2018, "question_id_specific": 102, "full_question": "Mujer de 64 a\u00f1os, sin comorbilidades relevantes, intervenida de un c\u00e1ncer de mama derecha mediante una tumorectomia ampliada y biopsia selectiva del ganglio centinela, con el siguiente resultado anatomopatologico: carcinoma ductal Infiltrante pobremente dlferenciado (grado 3), tama\u00f1o de 2,2 cm (pT2), receptores de estr\u00f3genos y progesterona positivo., Ki67 25%, HER2 negativo. Presenta afectaci\u00f3n macrosc\u00f3pica del ganglio centinela, aunque no hay afectaci\u00f3n del resto de ganglios axilares aislados (pN1). \u00bfQu\u00e9 tratamiento sist\u00e9mico adyuvante considera mas adecuado para esta paciente?", "full_answer": "Se trata de un Luminal B. Seg\u00fan la SEGO, esta paciente s\u00ed es candidata a tratamiento con quimioterapia por ser de alto grado histol\u00f3gico y elevada proliferaci\u00f3n tumoral.", "type": "GINECOLOG\u00cdA Y OBSTETRICIA", "options": {"1": "Hormonoterapia adyuvante que incluya inhibidores de aromatasa.", "2": "Quimioterapia adyuvante, preferentemente con antraciclinas y taxanos, seguida de hormonoterapia.", "3": "Quimioterapia adyuvante en asociaci\u00f3n con el anticuerpo monoclonal trastuzumab, seguida de hormonoterapia.", "4": "Quimioterapia adyuvante, preferiblemente con antraciclinas y taxanos.", "5": NaN}, "correct_option": 2, "explanations": {"1": {"exist": false, "char_ranges": [], "word_ranges": [], "text": ""}, "2": {"exist": true, "char_ranges": [[0, 169]], "word_ranges": [[0, 28]], "text": "Se trata de un Luminal B. Seg\u00fan la SEGO, esta paciente s\u00ed es candidata a tratamiento con quimioterapia por ser de alto grado histol\u00f3gico y elevada proliferaci\u00f3n tumoral."}, "3": {"exist": false, "char_ranges": [], "word_ranges": [], "text": ""}, "4": {"exist": false, "char_ranges": [], "word_ranges": [], "text": ""}, "5": {"exist": false, "char_ranges": [], "word_ranges": [], "text": ""}}} {"id": 197, "year": 2013, "question_id_specific": 71, "full_question": "Ni\u00f1o de 10 a\u00f1os con episodios breves de distracciones (< 1 minuto) en los que no responde a llamadas y parpadea. Un EEG muestra descargas de punta-onda a 3 ciclos por segundo. El tratamiento electivo de primera l\u00ednea lo har\u00eda con:", "full_answer": "Las ausencias se comportan como breves episodios de p\u00e9rdida brusca del nivel de conciencia sin alteraci\u00f3n del control postural; caracter\u00edsticamente duran segundos y pueden repetirse muchas veces al d\u00eda, suelen acompa\u00f1arse de peque\u00f1os signos motores bilaterales (parpadeo, masticaci\u00f3n) y se recupera la conciencia de forma igualmente brusca, sin confusi\u00f3n posterior ni memoria del episodio (MIR 03-04, 248; MIR 97-98, 51). La edad de comienzo suele estar entre los 4 a\u00f1os y el inicio de la adolescencia, siendo la causa m\u00e1s frecuente de crisis en este rango de edad. Los hallazgos en el EEG son t\u00edpicamente descargas generalizadas y sim\u00e9tricas de punta-onda a 3 Hz (MIR 99-00F, 67) coincidiendo con las crisis, aunque en el EEG interictal existen m\u00e1s per\u00edodos de actividad anormal que los visibles cl\u00ednicamente. Los f\u00e1rmacos m\u00e1s adecuados para el tratamiento de las ausencias son el Valproato y la Etosuximida (Manual CTO).", "type": "NEUROLOG\u00cdA", "options": {"1": "Valproato.", "2": "Carbamacepina.", "3": "Fenito\u00edna.", "4": "Gabapentina.", "5": "Clonazepam."}, "correct_option": 1, "explanations": {"1": {"exist": true, "char_ranges": [[811, 922]], "word_ranges": [[126, 144]], "text": "Los f\u00e1rmacos m\u00e1s adecuados para el tratamiento de las ausencias son el Valproato y la Etosuximida (Manual CTO)."}, "2": {"exist": false, "char_ranges": [], "word_ranges": [], "text": ""}, "3": {"exist": false, "char_ranges": [], "word_ranges": [], "text": ""}, "4": {"exist": false, "char_ranges": [], "word_ranges": [], "text": ""}, "5": {"exist": false, "char_ranges": [], "word_ranges": [], "text": ""}}} {"id": 478, "year": 2020, "question_id_specific": 134, "full_question": "Var\u00f3n de 35 a\u00f1os, trasladado a urgencias tras un accidente de tr\u00e1fico. A su ingreso se observa una puntuaci\u00f3n en la escala de Glasgow de 15, tensi\u00f3n arterial de 140/90 mmHg, frecuencia respiratoria de 35 rpm y frecuencia cardiaca de 110 lpm, con una saturaci\u00f3n de ox\u00edgeno basal del 91 %. En la exploraci\u00f3n hay hipofonesis tor\u00e1cica derecha y timpanismo a la percusi\u00f3n. \u00bfCu\u00e1l es el diagn\u00f3stico de presunci\u00f3n? :", "full_answer": "Pregunta f\u00e1cil y cl\u00e1sica que adem\u00e1s pod\u00e9is aplicar a la pr\u00e1ctica cl\u00ednica diaria. Hipofonesis en un hemit\u00f3rax + timpanismo = neumot\u00f3rax. Si hubiera matidez a la percusi\u00f3n: hemot\u00f3rax. En cualquiera de los dos casos, ante hipofonesis de un hemit\u00f3rax en un politraumatizado grave e inestable: drenaje tor\u00e1cico (a ser posible, de gran calibre si se coloca en medio hospitalario, puesto que aunque la lesi\u00f3n predominante sea el neumot\u00f3rax, puede encontrarse un componente de hemot\u00f3rax que en caso de colocar un drenaje fino, podr\u00eda obstruir el mismo).", "type": "CUIDADOS CR\u00cdTICOS", "options": {"1": "Taponamiento card\u00edaco traum\u00e1tico.", "2": "Neumot\u00f3rax a tensi\u00f3n derecho.", "3": "Hemot\u00f3rax masivo derecho.", "4": "Contusi\u00f3n pulmonar.", "5": NaN}, "correct_option": 2, "explanations": {"1": {"exist": false, "char_ranges": [], "word_ranges": [], "text": ""}, "2": {"exist": true, "char_ranges": [[81, 135]], "word_ranges": [[13, 21]], "text": "Hipofonesis en un hemit\u00f3rax + timpanismo = neumot\u00f3rax."}, "3": {"exist": true, "char_ranges": [[137, 181]], "word_ranges": [[21, 28]], "text": "Si hubiera matidez a la percusi\u00f3n: hemot\u00f3rax."}, "4": {"exist": false, "char_ranges": [], "word_ranges": [], "text": ""}, "5": {"exist": false, "char_ranges": [], "word_ranges": [], "text": ""}}} {"id": 97, "year": 2012, "question_id_specific": 171, "full_question": "Paciente que presenta desde hace 2 semanas una erupci\u00f3n pruriginosa constituida por m\u00faltiples p\u00e1pulas eritemato-viol\u00e1ceas poligonales, de superficie aplanada, localizadas preferentemente en la cara anterior de mu\u00f1ecas, zona pretibial y \u00e1rea lumbar. Adem\u00e1s presenta lesiones reticuladas blanquecinas en mucosa yugal, \u00bfCu\u00e1l es el diagn\u00f3stico m\u00e1s probable?", "full_answer": "Nos est\u00e1n describiendo sin lugar a dudas las t\u00edpicas lesiones (tanto en piel como en mucosa oral) de un liquen plano (5).", "type": "DERMATOLOG\u00cdA, VENEREOLOG\u00cdA Y CIRUG\u00cdA PL\u00c1STICA", "options": {"1": "Pitiriasis versicolor.", "2": "Micosis fungoides.", "3": "Psoriasis en gotas.", "4": "Pitiriasis rosada de Gibert.", "5": "Liquen plano."}, "correct_option": 5, "explanations": {"1": {"exist": false, "char_ranges": [], "word_ranges": [], "text": ""}, "2": {"exist": false, "char_ranges": [], "word_ranges": [], "text": ""}, "3": {"exist": false, "char_ranges": [], "word_ranges": [], "text": ""}, "4": {"exist": false, "char_ranges": [], "word_ranges": [], "text": ""}, "5": {"exist": true, "char_ranges": [[0, 116]], "word_ranges": [[0, 21]], "text": "Nos est\u00e1n describiendo sin lugar a dudas las t\u00edpicas lesiones (tanto en piel como en mucosa oral) de un liquen plano"}}} {"id": 426, "year": 2018, "question_id_specific": 94, "full_question": "Un paciente de 73 a\u00f1os\u2026 degluci\u00f3n una disfagia neurol\u00f3gica completa, secundaria a un ictus cardioembolico en el territorio de la arteria cerebral media izquierda. va a precisar apoyo nutricional a largo plazo. De las siguientes modalidades de tratamiento, \u00bfcual considera mas indicada para este caso?", "full_answer": "La nutrici\u00f3n enteral es una t\u00e9cnica de soporte nutricional que consiste en administrar los nutrientes directamente en el tracto gastrointestinal mediante sonda. La lecci\u00f3n de la v\u00eda de administraci\u00f3n de la f\u00f3rmula depende del estado nutricional, la edad del paciente y la enfermedad de base. Si se considera que la nutrici\u00f3n enteral es de corta duraci\u00f3n, la v\u00eda de elecci\u00f3n es la sonda nasog\u00e1strica o nasoduodenal-nasoyeyunal\u2026 Si se prev\u00e9 una nutrici\u00f3n prolongada la v\u00eda de elecci\u00f3n es la gastrostom\u00eda (opci\u00f3n 3), quedando la yeyunostom\u00eda reservada para casos en los que no es posible usar la v\u00eda g\u00e1strica.", "type": "ENDOCRINOLOG\u00cdA", "options": {"1": "Nutrici\u00f3n enteral por sonda nasogastrica.", "2": "Nutrici\u00f3n enteral por sonda nasoduodenal.", "3": "Nutrici\u00f3n enteral por gastrostomia.", "4": "Nutrici\u00f3n enteral por yeyunostomia.", "5": NaN}, "correct_option": 3, "explanations": {"1": {"exist": true, "char_ranges": [[292, 426]], "word_ranges": [[45, 66]], "text": "Si se considera que la nutrici\u00f3n enteral es de corta duraci\u00f3n, la v\u00eda de elecci\u00f3n es la sonda nasog\u00e1strica o nasoduodenal-nasoyeyunal\u2026"}, "2": {"exist": true, "char_ranges": [[292, 426]], "word_ranges": [[45, 66]], "text": "Si se considera que la nutrici\u00f3n enteral es de corta duraci\u00f3n, la v\u00eda de elecci\u00f3n es la sonda nasog\u00e1strica o nasoduodenal-nasoyeyunal\u2026"}, "3": {"exist": true, "char_ranges": [[427, 512]], "word_ranges": [[66, 81]], "text": "Si se prev\u00e9 una nutrici\u00f3n prolongada la v\u00eda de elecci\u00f3n es la gastrostom\u00eda (opci\u00f3n 3),"}, "4": {"exist": true, "char_ranges": [[514, 606]], "word_ranges": [[81, 97]], "text": "quedando la yeyunostom\u00eda reservada para casos en los que no es posible usar la v\u00eda g\u00e1strica."}, "5": {"exist": false, "char_ranges": [], "word_ranges": [], "text": ""}}} {"id": 262, "year": 2014, "question_id_specific": 58, "full_question": "Mujer de 70 a\u00f1os con antecedentes de hipertensi\u00f3n e insuficiencia cardiaca moderada que acude a la consulta por presentar tos persistente y seca que se inicia como una sensaci\u00f3n de picor en la garganta. En la anal\u00edtica se observa hiperpotasemia. \u00bfCu\u00e1l es el f\u00e1rmaco sospechoso de causar la cl\u00ednica y la alteraci\u00f3n anal\u00edtica de la paciente?", "full_answer": "Son efectos adversos t\u00edpicos de los IECAs, en el caso de la tos suele ser seca e irritativa y aparece hasta en el 3% de pacientes. Tambi\u00e9n puede aparecer hiperpotasemia en el 1%.", "type": "FARMACOLOG\u00cdA", "options": {"1": "Hidroclorotiazida.", "2": "Bisoprolol.", "3": "Furosemida.", "4": "Enalapril.", "5": "Hidralazina."}, "correct_option": 4, "explanations": {"1": {"exist": false, "char_ranges": [], "word_ranges": [], "text": ""}, "2": {"exist": false, "char_ranges": [], "word_ranges": [], "text": ""}, "3": {"exist": false, "char_ranges": [], "word_ranges": [], "text": ""}, "4": {"exist": true, "char_ranges": [[0, 178]], "word_ranges": [[0, 33]], "text": "Son efectos adversos t\u00edpicos de los IECAs, en el caso de la tos suele ser seca e irritativa y aparece hasta en el 3% de pacientes. Tambi\u00e9n puede aparecer hiperpotasemia en el 1%."}, "5": {"exist": false, "char_ranges": [], "word_ranges": [], "text": ""}}} {"id": 346, "year": 2016, "question_id_specific": 159, "full_question": "Mujer de 69 a\u00f1os que acude a su consulta refiriendo sangrado genital de varios meses de evoluci\u00f3n. Niega tratamiento hormonal sustitutivo y anticoagulaci\u00f3n. Aporta citolog\u00eda cervicovaginal normal. Exploraci\u00f3n f\u00edsica general y genital sin hallazgos de inter\u00e9s. IMC de 38kg/m2. Indique la actitud m\u00e1s correcta:", "full_answer": "La respuesta correcta es la 2. Estamos ante una caso de una metrorragia postmenop\u00e1usica, cuya causa m\u00e1s frecuente en primer lugar es la atrofia genital y en segunda lugar el c\u00e1ncer de endometrio. El dato que nos dan de obesidad, nos debe hacer sospechar un c\u00e1ncer de endometrio puesta que la grasa perif\u00e9rica pasa a estrona, y estos estr\u00f3genos hacen proliferar al endometrio. Por otro lado, aporta una citolog\u00eda normal, por lo que descarta la opci\u00f3n 3. La valoraci\u00f3n hormonal no es oportuna, ya que entendemos que la paciente est\u00e1 en menopausia. No debemos prescribir tratamiento sin antes descartar causa org\u00e1nica.", "type": "GINECOLOG\u00cdA Y OBSTETRICIA", "options": {"1": "Prescribir progesterona c\u00edclica.", "2": "Biopsia endometrial.", "3": "Biopsias de c\u00e9rvix al azar.", "4": "Valoraci\u00f3n hormonal con FSH, LH y estradiol.", "5": NaN}, "correct_option": 2, "explanations": {"1": {"exist": true, "char_ranges": [[546, 615]], "word_ranges": [[92, 101]], "text": "No debemos prescribir tratamiento sin antes descartar causa org\u00e1nica."}, "2": {"exist": true, "char_ranges": [[31, 195]], "word_ranges": [[6, 33]], "text": "Estamos ante una caso de una metrorragia postmenop\u00e1usica, cuya causa m\u00e1s frecuente en primer lugar es la atrofia genital y en segunda lugar el c\u00e1ncer de endometrio."}, "3": {"exist": true, "char_ranges": [[391, 452]], "word_ranges": [[66, 77]], "text": "aporta una citolog\u00eda normal, por lo que descarta la opci\u00f3n 3."}, "4": {"exist": true, "char_ranges": [[453, 545]], "word_ranges": [[77, 92]], "text": "La valoraci\u00f3n hormonal no es oportuna, ya que entendemos que la paciente est\u00e1 en menopausia."}, "5": {"exist": false, "char_ranges": [], "word_ranges": [], "text": ""}}} {"id": 170, "year": 2013, "question_id_specific": 89, "full_question": "Un paciente de 62 a\u00f1os de edad, portador de una biopr\u00f3tesis de pericardio en posici\u00f3n mitral desde hace dos a\u00f1os. Ingresa de urgencia por presentar un s\u00edndrome coronario agudo. Dos d\u00edas despu\u00e9s y tras una evoluci\u00f3n favorable, presenta de forma s\u00fabita un cuadro de insuficiencia cardiaca con edema agudo de pulm\u00f3n. Se\u00f1ale cu\u00e1l es la opci\u00f3n diagn\u00f3stica en este caso entre las siguientes posibilidades:", "full_answer": "La v\u00e1lvula es biol\u00f3gica y, por lo general, no se cosen los velos a los m\u00fasculos papilares, as\u00ed que nos da igual lo que les pase. Un aneurisma ventricular deprime la FEVI, pero no te mete de repente en EAP. Por el contrario, una CIV aumenta bruscamente el flujo al lecho pulmonar, provocando edema de pulm\u00f3n, y adem\u00e1s es una de las complicaciones m\u00e1s frecuentes del IAM. La CIA isqu\u00e9mica\u2026 no niego que exista, pero ni siquiera aparece en las gu\u00edas. Y para una endocarditis no har\u00edan falta tantas alforjas.", "type": "CARDIOLOG\u00cdA Y CIRUG\u00cdA CARDIOVASCULAR", "options": {"1": "Insuficiencia mitral aguda posinfarto.", "2": "Aneurisma ventricular izquierdo.", "3": "Comunicaci\u00f3n interventricular aguda.", "4": "Comunicaci\u00f3n interauricular posinfarto.", "5": "Disfunci\u00f3n de la biopr\u00f3tesis por probable endocarditis aguda."}, "correct_option": 3, "explanations": {"1": {"exist": true, "char_ranges": [[370, 447]], "word_ranges": [[67, 81]], "text": "La CIA isqu\u00e9mica\u2026 no niego que exista, pero ni siquiera aparece en las gu\u00edas."}, "2": {"exist": true, "char_ranges": [[129, 205]], "word_ranges": [[26, 40]], "text": "Un aneurisma ventricular deprime la FEVI, pero no te mete de repente en EAP."}, "3": {"exist": true, "char_ranges": [[224, 369]], "word_ranges": [[43, 67]], "text": "una CIV aumenta bruscamente el flujo al lecho pulmonar, provocando edema de pulm\u00f3n, y adem\u00e1s es una de las complicaciones m\u00e1s frecuentes del IAM."}, "4": {"exist": true, "char_ranges": [[370, 447]], "word_ranges": [[67, 81]], "text": "La CIA isqu\u00e9mica\u2026 no niego que exista, pero ni siquiera aparece en las gu\u00edas."}, "5": {"exist": true, "char_ranges": [[450, 504]], "word_ranges": [[82, 90]], "text": "para una endocarditis no har\u00edan falta tantas alforjas."}}} {"id": 40, "year": 2011, "question_id_specific": 136, "full_question": "Paciente de 64 a\u00f1os de edad, fumador y bebedor importante que refiere adenopat\u00eda yugulodig\u00e1strica derecha no dolorosa de crecimiento progresivo en los \u00faltimos dos meses cuya punci\u00f3n-aspiraci\u00f3n con aguja fina fue informada como carcinoma epidermoide. Cu\u00e1l de las siguientes localizaciones es la m\u00e1s probable del tumor primario:", "full_answer": "Est\u00e1s en medio del bloque de ORL. Ya has pasado el bloque de Neumolog\u00eda y el de Digestivo. Falta por salir el de Dermatolog\u00eda, pero un dermat\u00f3logo (respuesta 1) no preguntar\u00eda esto. As\u00ed que nos queda par\u00f3tida y laringe; pero el carcinoma epidermoide de par\u00f3tida es raro; yo marcar\u00eda la 5 y me quedar\u00eda tranquilo. Quiz\u00e1s porque todas las adenopat\u00edas a este nivel que yo he visto han acabado correspondiendo a carcinomas de la v\u00eda a\u00e9rodigestiva superior.", "type": "OTORRINOLARINGOLOG\u00cdA Y CIRUG\u00cdA MAXILOFACIAL", "options": {"1": "Cuero cabelludo.", "2": "Par\u00f3tida.", "3": "Pulm\u00f3n.", "4": "Es\u00f3fago.", "5": "Laringe."}, "correct_option": 5, "explanations": {"1": {"exist": false, "char_ranges": [], "word_ranges": [], "text": ""}, "2": {"exist": false, "char_ranges": [], "word_ranges": [], "text": ""}, "3": {"exist": false, "char_ranges": [], "word_ranges": [], "text": ""}, "4": {"exist": false, "char_ranges": [], "word_ranges": [], "text": ""}, "5": {"exist": true, "char_ranges": [[327, 452]], "word_ranges": [[57, 77]], "text": "todas las adenopat\u00edas a este nivel que yo he visto han acabado correspondiendo a carcinomas de la v\u00eda a\u00e9rodigestiva superior."}}} {"id": 126, "year": 2012, "question_id_specific": 59, "full_question": "Un hombre de 65 a\u00f1os, oficinista jubilado y fu\u00admador de 1 paquete diario de cigarrillos, acude a consulta por un cuadro de tos persistente, gene\u00adralmente seca y disnea progresiva de 2 a\u00f1os de evoluci\u00f3n que en la actualidad es de grado 2. El paciente niega otros s\u00edntomas. La exploraci\u00f3n f\u00edsica no muestra datos relevantes. La actitud a seguir ser\u00eda:", "full_answer": "La respuesta correcta es la 2 (Realizar RX de Torax y espirometr\u00eda con prueba broncodilatadora). Probablemente sea un EPOC pero hay que confirmarlo y valorar la gravedad de la obstruccion con la espirometr\u00eda antes de iniciar tratamiento. Es un paciente de riesgo para cancer de pulmon por lo que es obligado realizar Rx de torax para descartar esta u otras patologias que justificarian los s\u00edntomas, aunque lo mas probable es que sean por el debut de su EPOC.", "type": "NEUMOLOG\u00cdA Y CIRUG\u00cdA TOR\u00c1CICA", "options": {"1": "Iniciar tratamiento con broncodilatadores inhalados.", "2": "Realizar RX de Torax y espirometr\u00eda con prueba broncodilatadora.", "3": "Pautar corticoides orales.", "4": "Realizar TC tor\u00e1cica.", "5": "Realizar gasometr\u00eda arterial basal."}, "correct_option": 2, "explanations": {"1": {"exist": false, "char_ranges": [], "word_ranges": [], "text": ""}, "2": {"exist": true, "char_ranges": [[238, 459]], "word_ranges": [[37, 78]], "text": "Es un paciente de riesgo para cancer de pulmon por lo que es obligado realizar Rx de torax para descartar esta u otras patologias que justificarian los s\u00edntomas, aunque lo mas probable es que sean por el debut de su EPOC."}, "3": {"exist": false, "char_ranges": [], "word_ranges": [], "text": ""}, "4": {"exist": false, "char_ranges": [], "word_ranges": [], "text": ""}, "5": {"exist": false, "char_ranges": [], "word_ranges": [], "text": ""}}} {"id": 568, "year": 2022, "question_id_specific": 151, "full_question": "Var\u00f3n de 22 a\u00f1os que acude a urgencias por presentar orina \u201ccasi negra\u201d desde hace 12 horas. No tiene antecedentes de inter\u00e9s ni toma medicamentos. Refiere un cuadro catarral de v\u00edas altas desde hace 4-5 d\u00edas para el que ha tomado paracetamol. Exploraci\u00f3n: temperatura 37,3 \u00baC, PA 150/95, FC 85 lpm, ligero eritema far\u00edngeo, resto normal. En anal\u00edtica destaca: Hb 12,8 g/dl, Hcto 39 %, leucocitos 10.500/mm3 con f\u00f3rmula normal, plaquetas 250.000/mm3, coagulaci\u00f3n normal, urea 25 mg/dl, creatinina 0,8 mg/dl, iones, perfil hep\u00e1tico, lip\u00eddico, alb\u00famina y prote\u00ednas totales normales. Estudio de autoinmunidad normal y serolog\u00eda v\u00edrica negativa. Orina de 24 h con proteinuria de 0,75 g/24 h, sedimento con 10 eritrocitos por campo (90% dism\u00f3rficos), no leucocituria. Ecograf\u00eda renal normal. \u00bfQu\u00e9 diagn\u00f3stico le parece m\u00e1s probable?:", "full_answer": "Un cuadro de hematuria macrosc\u00f3pica y proteinuria leve con niveles normales de creatinina, acompa\u00f1ando a un cuadro catarral de v\u00edas altas, es muy sugestivo de nefropat\u00eda IgA (opci\u00f3n 3 correcta).", "type": "NEFROLOG\u00cdA", "options": {"1": "Nefropat\u00eda con cambios m\u00ednimos.", "2": "Glomerulonefritis postinfecciosa.", "3": "Nefropat\u00eda IgA.", "4": "Nefritis intersticial aguda.", "5": NaN}, "correct_option": 3, "explanations": {"1": {"exist": false, "char_ranges": [], "word_ranges": [], "text": ""}, "2": {"exist": false, "char_ranges": [], "word_ranges": [], "text": ""}, "3": {"exist": true, "char_ranges": [[0, 194]], "word_ranges": [[0, 30]], "text": "Un cuadro de hematuria macrosc\u00f3pica y proteinuria leve con niveles normales de creatinina, acompa\u00f1ando a un cuadro catarral de v\u00edas altas, es muy sugestivo de nefropat\u00eda IgA (opci\u00f3n 3 correcta)."}, "4": {"exist": false, "char_ranges": [], "word_ranges": [], "text": ""}, "5": {"exist": false, "char_ranges": [], "word_ranges": [], "text": ""}}} {"id": 286, "year": 2016, "question_id_specific": 60, "full_question": "Hombre de 60 a\u00f1os de edad con antecedentes de hipertensi\u00f3n arterial y dislipemia ha sido recientemente diagnosticado de insuficiencia cardiaca. Tras realizar las pruebas pertinentes se objetiva que conserva la fracci\u00f3n de eyecci\u00f3n del ventr\u00edculo izquierdo (>50%). En relaci\u00f3n al tratamiento se\u00f1ale la respuesta lNCORRECTA:", "full_answer": "El tratamiento con diur\u00e9ticos debe iniciarse a dosis altas.", "type": "CARDIOLOG\u00cdA Y CIRUG\u00cdA VASCULAR", "options": {"1": "El tratamiento inicial deber\u00eda dirigirse alproceso patol\u00f3gico subyacente.", "2": "El tratamiento con diur\u00e9ticos debe iniciarse a dosis altas.", "3": "En caso de precisar tratamiento con nitratos debe iniciarse a dosis bajas.", "4": "La disnea puede tratarse reduciendo la activaci\u00f3n neurohormonal con inhibidores de la enzima convertidora de la angiotensina o antagonistas de los receptores de la angiotensina.", "5": NaN}, "correct_option": 2, "explanations": {"1": {"exist": false, "char_ranges": [], "word_ranges": [], "text": ""}, "2": {"exist": true, "char_ranges": [[0, 59]], "word_ranges": [[0, 9]], "text": "El tratamiento con diur\u00e9ticos debe iniciarse a dosis altas."}, "3": {"exist": false, "char_ranges": [], "word_ranges": [], "text": ""}, "4": {"exist": false, "char_ranges": [], "word_ranges": [], "text": ""}, "5": {"exist": false, "char_ranges": [], "word_ranges": [], "text": ""}}} {"id": 228, "year": 2014, "question_id_specific": 75, "full_question": "En un paciente con cl\u00ednica de insuficiencia cardiaca que presenta una ascitis desproporcionadamente elevada en relaci\u00f3n al edema perif\u00e9rico, la etiolog\u00eda m\u00e1s probable ser\u00eda:", "full_answer": "Nos cuentan cl\u00ednica de fallo derecho, con edemas y ascitis. Por lo tanto, descartamos los fallos del coraz\u00f3n izquierdo: EAo, dilatada con DSVI y una MHO. De los dos que nos quedan, en la pericarditis constrictiva tenemos presiones de llenado aumentadas en cavidades derechas, que provocan aumento de presi\u00f3n retr\u00f3grado con congesti\u00f3n hep\u00e1tica, ascitis, edemas, etc. Por el contrario, en la hipertensi\u00f3n pulmonar, la v\u00e1lvula tric\u00faspide se ocupa (a priori) de \u201ccontener\u201d esa sobrepresi\u00f3n.", "type": "CARDIOLOG\u00cdA", "options": {"1": "Una estenosis a\u00f3rtica grave.", "2": "Una miocardiopat\u00eda dilatada con disfunci\u00f3n ventricular izquierda significativa.", "3": "Una hipertensi\u00f3n pulmonar primaria.", "4": "Una miocardiopat\u00eda hipertr\u00f3fica obstructiva.", "5": "Una pericarditis constrictiva."}, "correct_option": 5, "explanations": {"1": {"exist": true, "char_ranges": [[0, 153]], "word_ranges": [[0, 26]], "text": "Nos cuentan cl\u00ednica de fallo derecho, con edemas y ascitis. Por lo tanto, descartamos los fallos del coraz\u00f3n izquierdo: EAo, dilatada con DSVI y una MHO."}, "2": {"exist": true, "char_ranges": [[0, 153]], "word_ranges": [[0, 26]], "text": "Nos cuentan cl\u00ednica de fallo derecho, con edemas y ascitis. Por lo tanto, descartamos los fallos del coraz\u00f3n izquierdo: EAo, dilatada con DSVI y una MHO."}, "3": {"exist": true, "char_ranges": [[384, 486]], "word_ranges": [[59, 74]], "text": "en la hipertensi\u00f3n pulmonar, la v\u00e1lvula tric\u00faspide se ocupa (a priori) de \u201ccontener\u201d esa sobrepresi\u00f3n."}, "4": {"exist": true, "char_ranges": [[0, 153]], "word_ranges": [[0, 26]], "text": "Nos cuentan cl\u00ednica de fallo derecho, con edemas y ascitis. Por lo tanto, descartamos los fallos del coraz\u00f3n izquierdo: EAo, dilatada con DSVI y una MHO."}, "5": {"exist": true, "char_ranges": [[181, 365]], "word_ranges": [[32, 56]], "text": "en la pericarditis constrictiva tenemos presiones de llenado aumentadas en cavidades derechas, que provocan aumento de presi\u00f3n retr\u00f3grado con congesti\u00f3n hep\u00e1tica, ascitis, edemas, etc."}}} {"id": 217, "year": 2014, "question_id_specific": 122, "full_question": "Un hombre de 54 a\u00f1os de edad es diagnosticado de tumor renal izquierdo sugestivo de carcinoma de c\u00e9lulas renales. En su estudio anal\u00edtico preoperatorio se detectan niveles elevados de GPT, fosfatasa alcalina y alfa-2-globulina y tiempo de protrombina alargado. El h\u00edgado est\u00e1 aumentado de tama\u00f1o de forma difusa, pero sin defectos de infiltraci\u00f3n hep\u00e1tica. La justificaci\u00f3n m\u00e1s probable de estos hallazgos se debe a:", "full_answer": "Pregunta muy f\u00e1cil en la que nos est\u00e1n presentando a un paciente con s\u00edndrome de Stauffer, un s\u00edndrome paraneopl\u00e1sico que consiste en una disfunci\u00f3n hep\u00e1tica secundaria a productos t\u00f3xicos segregados por una serie de tumores (el m\u00e1s frecuente, el carcinoma de c\u00e9lulas renales, como el que nos cuenta el caso). Se produce, como en este caso, una colestasis (elevaci\u00f3n de fosfatasa alcalina), movilizaci\u00f3n de enzimas hep\u00e1ticsa y alargamiento del tiempo de protrombina que indican disfunci\u00f3n hep\u00e1tica. Se descartan las met\u00e1stasis (no hay defectos de infiltraci\u00f3n hep\u00e1tica), la trombosis tumoral intrahep\u00e1tica (dar\u00eda otra sintomatolog\u00eda y esta ser\u00eda m\u00e1s aguda), y la hepatitis aguda y hemocromatosis (no est\u00e1n en contexto ni es lo que el autor de la pregunta quiere que pensemos).", "type": "NEFROLOG\u00cdA", "options": {"1": "Met\u00e1stasis hep\u00e1ticas.", "2": "Trombosis tumoral intrahep\u00e1tica.", "3": "Hepatitis aguda.", "4": "Presencia de sustancias hepatot\u00f3xicas producidas por el tumor.", "5": "Hemocromatosis."}, "correct_option": 4, "explanations": {"1": {"exist": true, "char_ranges": [[500, 570]], "word_ranges": [[76, 86]], "text": "Se descartan las met\u00e1stasis (no hay defectos de infiltraci\u00f3n hep\u00e1tica),"}, "2": {"exist": true, "char_ranges": [[573, 657]], "word_ranges": [[86, 98]], "text": "la trombosis tumoral intrahep\u00e1tica (dar\u00eda otra sintomatolog\u00eda y esta ser\u00eda m\u00e1s aguda),"}, "3": {"exist": true, "char_ranges": [[662, 777]], "word_ranges": [[99, 120]], "text": "la hepatitis aguda y hemocromatosis (no est\u00e1n en contexto ni es lo que el autor de la pregunta quiere que pensemos)."}, "4": {"exist": true, "char_ranges": [[311, 499]], "word_ranges": [[50, 76]], "text": "Se produce, como en este caso, una colestasis (elevaci\u00f3n de fosfatasa alcalina), movilizaci\u00f3n de enzimas hep\u00e1ticsa y alargamiento del tiempo de protrombina que indican disfunci\u00f3n hep\u00e1tica."}, "5": {"exist": true, "char_ranges": [[662, 777]], "word_ranges": [[99, 120]], "text": "la hepatitis aguda y hemocromatosis (no est\u00e1n en contexto ni es lo que el autor de la pregunta quiere que pensemos)."}}} {"id": 142, "year": 2012, "question_id_specific": 138, "full_question": "Ni\u00f1a de 12 a\u00f1os, hija \u00fanica de padres sanos no consangu\u00edneos. No refieren antecedentes personales ni familiares de inter\u00e9s. En el examen cl\u00ednico para iniciar una actividad deportiva, usted encuentra una discreta hepatomegalia de consistencia normal, por lo que solicita una anal\u00edtica, en la que destaca una ASAT de 80U/l, ALAT 105 U/l, proteinograma electrofor\u00e9tico con todas las fracciones proteicas en rango normal y negatividad de la serolog\u00eda de virus hepatotropos. \u00bfQu\u00e9 deber\u00eda descartar y qu\u00e9 prueba indicar\u00eda para ello?", "full_answer": "En realidad yo a esta paciente no le har\u00eda ninguna prueba en este momento, y comprobar\u00eda si la cifra de transaminasas se ha normalizado en un tiempo posterior, ya que la causa m\u00e1s frecuente, por lo menos en los ni\u00f1os, es una elevaci\u00f3n transitoria en contexto de enfermedades v\u00edricas (las m\u00e1s frecuentes citomegalovirus y virus de Epstein-Barr). La elevaci\u00f3n m\u00ednima de las enzimas hep\u00e1ticas y el hecho de que tenga un perfil proteico normal va en contra de cronicidad\u2026 Si tuviera que elegir me quedar\u00eda con la respuesta 5, que me parece la menos agresiva, y puestos a hace anal\u00edtica le a\u00f1adir\u00eda la determinaci\u00f3n de alfa-1-antitripsina (sin TC) y marcadores de celiaqu\u00eda, pues la celiaqu\u00eda tambi\u00e9n puede asociar cierta hipertransaminemia.", "type": "PEDIATR\u00cdA", "options": {"1": "D\u00e9ficit de alfa 1 antitripsina. TAC tor\u00e1cico para confirmar enfisema.", "2": "Mucoviscidosis. Determinaci\u00f3n de cloro en sudor.", "3": "Hepatitis autoinmune. Biopsia hep\u00e1tica.", "4": "Glucogenosis tipo VI (d\u00e9ficit de fosforilasa). Biopsia muscular.", "5": "Enfermedad de Wilson. Determinar ceruloplasmina y cobre en sangre y orina."}, "correct_option": 5, "explanations": {"1": {"exist": false, "char_ranges": [], "word_ranges": [], "text": ""}, "2": {"exist": false, "char_ranges": [], "word_ranges": [], "text": ""}, "3": {"exist": false, "char_ranges": [], "word_ranges": [], "text": ""}, "4": {"exist": false, "char_ranges": [], "word_ranges": [], "text": ""}, "5": {"exist": false, "char_ranges": [], "word_ranges": [], "text": ""}}} {"id": 267, "year": 2014, "question_id_specific": 139, "full_question": "En un paciente con cirrosis hep\u00e1tica y ascitis, que desarrolla un derrame pleural con los siguientes datos en el l\u00edquido pleural: LDH 45 U/L (s\u00e9rica 220 U/L), cociente prote\u00ednas pleura/suero 0,3 y cociente LDH pleura/suero 0,2. \u00bfCu\u00e1l ser\u00eda la actitud adecuada?", "full_answer": "Describen parametros de liquido pleural de trasudado, por lo que si el paciente no esta en insuficiencia cardiaca se trata de una descompensacion hidropica. No necesita otros estudios.", "type": "NEUMOLOG\u00cdA", "options": {"1": "Debe ampliarse el estudio del l\u00edquido con recuento celular, glucosa, pH. ADA, colesterol y cultivo.", "2": "Biopsia pleural ciega.", "3": "Vidcotoracoscopia diagn\u00f3stica.", "4": "Antibioterapia emp\u00edrica por sospecha de derrame paraneum\u00f3nico.", "5": "Continuar el tratamiento de su hepatopat\u00eda."}, "correct_option": 5, "explanations": {"1": {"exist": false, "char_ranges": [], "word_ranges": [], "text": ""}, "2": {"exist": false, "char_ranges": [], "word_ranges": [], "text": ""}, "3": {"exist": false, "char_ranges": [], "word_ranges": [], "text": ""}, "4": {"exist": false, "char_ranges": [], "word_ranges": [], "text": ""}, "5": {"exist": true, "char_ranges": [[0, 184]], "word_ranges": [[0, 28]], "text": "Describen parametros de liquido pleural de trasudado, por lo que si el paciente no esta en insuficiencia cardiaca se trata de una descompensacion hidropica. No necesita otros estudios."}}} {"id": 99, "year": 2012, "question_id_specific": 84, "full_question": "Un paciente diab\u00e9tico de 60 a\u00f1os consulta por primera vez respecto al tratamiento de su enfermedad. \u00bfQu\u00e9 objetivo a alcanzar de los siguientes le recomendar\u00eda en primer lugar?", "full_answer": "El resto no son indicaciones. El objetivo de TA es < 130/85 mmHg.", "type": "ENDOCRINOLOG\u00cdA", "options": {"1": "Mantener la tensi\u00f3n arterial por debajo de 110/70 mmHg.", "2": "Abandono del h\u00e1bito tab\u00e1quico.", "3": "Mantener un \u00edndice de masa corporal (IMC) menor de 21.", "4": "Realizarse glucemia capilar basal a diario.", "5": "Evitar grasas animales en la dieta."}, "correct_option": 2, "explanations": {"1": {"exist": true, "char_ranges": [[0, 65]], "word_ranges": [[0, 13]], "text": "El resto no son indicaciones. El objetivo de TA es < 130/85 mmHg."}, "2": {"exist": true, "char_ranges": [[0, 29]], "word_ranges": [[0, 5]], "text": "El resto no son indicaciones."}, "3": {"exist": true, "char_ranges": [[0, 29]], "word_ranges": [[0, 5]], "text": "El resto no son indicaciones."}, "4": {"exist": true, "char_ranges": [[0, 29]], "word_ranges": [[0, 5]], "text": "El resto no son indicaciones."}, "5": {"exist": true, "char_ranges": [[0, 29]], "word_ranges": [[0, 5]], "text": "El resto no son indicaciones."}}} {"id": 380, "year": 2016, "question_id_specific": 135, "full_question": "Un paciente de 25 a\u00f1os, durante un partido de tenis, tiene dolor intenso en el cuello y en el ojo izquierdo. La ma\u00f1ana siguiente se despierta con sensaci\u00f3n de inestabilidad de la marcha y tiene ptosis palpebral del ojo izquierdo y anisocoria, siendo la pupila izquierda m\u00e1s peque\u00f1a que la derecha. El paciente mantiene buena agudeza visual. \u00bfD\u00f3nde localizar\u00eda con m\u00e1s probabilidad la lesi\u00f3n?", "full_answer": "El cuadro que nos presentan es el de un s\u00edndrome de Horner (miosis con ptosis) por lo que las posibilidades quedan reducidas a las opciones 3 o 4, pero especifican que el paciente presenta inestabilidad de la marcha, que no se justifica por un Horner por disecci\u00f3n carot\u00eddea que afecte el ganglio cervical superior, que es lo que nos quieren hacer creer cuando hablan del dolor en el cuello y en el ojo. Si el paciente verdaderamente tiene una ataxia, la \u00fanica localizaci\u00f3n posible ser\u00eda el bulbo raqu\u00eddeo.", "type": "NEUROLOG\u00cdA", "options": {"1": "III par craneal.", "2": "Quiasma \u00f3ptico.", "3": "Ganglio cervical superior.", "4": "Bulbo raqu\u00eddeo.", "5": NaN}, "correct_option": 4, "explanations": {"1": {"exist": false, "char_ranges": [], "word_ranges": [], "text": ""}, "2": {"exist": false, "char_ranges": [], "word_ranges": [], "text": ""}, "3": {"exist": true, "char_ranges": [[152, 314]], "word_ranges": [[29, 54]], "text": "especifican que el paciente presenta inestabilidad de la marcha, que no se justifica por un Horner por disecci\u00f3n carot\u00eddea que afecte el ganglio cervical superior,"}, "4": {"exist": true, "char_ranges": [[404, 506]], "word_ranges": [[73, 88]], "text": "Si el paciente verdaderamente tiene una ataxia, la \u00fanica localizaci\u00f3n posible ser\u00eda el bulbo raqu\u00eddeo."}, "5": {"exist": false, "char_ranges": [], "word_ranges": [], "text": ""}}} {"id": 341, "year": 2016, "question_id_specific": 33, "full_question": "Mujer de 67 a\u00f1os diagnosticada de un carcinoma ductal infiltrante de mama y sin historia familiar de neoplasia. \u00bfQu\u00e9 estudios adicionales deben realizarse en el tumor por sus implicaciones cl\u00ednico-terap\u00e9uticas?", "full_answer": "Estudio de receptores hormonales y de HER2. Tanto para el tipo de tratamiento como para estudiar el factor pron\u00f3stico y el tratamiento adyuvante, es necesario conocer los diferentes receptores hormonales (estrogenos y progesterona) as\u00ed como el HER2-neu.", "type": "GINECOLOG\u00cdA Y OBSTETRICIA", "options": {"1": "Estudio fenot\u00edpico completo mediante citometr\u00eda de flujo.", "2": "Estudio de receptores hormonales y de HER2.", "3": "Estudio de receptores hormonales, e-cadherina y estudio de familiares de primer grado.", "4": "Estudio de BRCA 1-2 y estudio de familiares de primer grado.", "5": NaN}, "correct_option": 2, "explanations": {"1": {"exist": false, "char_ranges": [], "word_ranges": [], "text": ""}, "2": {"exist": true, "char_ranges": [[44, 253]], "word_ranges": [[7, 37]], "text": "Tanto para el tipo de tratamiento como para estudiar el factor pron\u00f3stico y el tratamiento adyuvante, es necesario conocer los diferentes receptores hormonales (estrogenos y progesterona) as\u00ed como el HER2-neu."}, "3": {"exist": false, "char_ranges": [], "word_ranges": [], "text": ""}, "4": {"exist": false, "char_ranges": [], "word_ranges": [], "text": ""}, "5": {"exist": false, "char_ranges": [], "word_ranges": [], "text": ""}}} {"id": 308, "year": 2016, "question_id_specific": 215, "full_question": "Un hombre con miop\u00eda magna, de 47 a\u00f1os, intervenido de cataratas hace 2 a\u00f1os acude a urgencias refiriendo una p\u00e9rdida profunda e indolora de la visi\u00f3n en su ojo derecho. \u00bfCu\u00e1l de los siguientes diagn\u00f3sticos puede provocar esta sintomatolog\u00eda?", "full_answer": "Se refiere a un paciente post operado hace 2 a\u00f1os de catarata con perdida profunda e indolora en OD. La endoftalmitis postquirurgica se asocia mas a pacientes en post operatorio inmediato, ademas de ser muy dolorosa con disminucion de la agudeza visual rapidamente progresiva porlo que no es correcta. El Desprendimiento de retina es coherente con el caso clinico por tratarse ademas de un paciente miope quienes suelen tener especial predisposicion al desprendimiento. La DMRE suele ocurrir en pacientes mayores y la disminucion de la vision es progresiva. En el DVP puede o no haber leve disminucion de la agudeza visual y el paciente suele referir cierto disconfort o dolor al inicio del cuadro clinico.", "type": "OFTALMOLOG\u00cdA", "options": {"1": "Endoftalmitis postquir\u00fargica.", "2": "Desprendimiento de retina.", "3": "Degeneraci\u00f3n macular asociada a la edad, forma h\u00fameda.", "4": "Desprendimiento posterior de v\u00edtreo.", "5": NaN}, "correct_option": 2, "explanations": {"1": {"exist": true, "char_ranges": [[101, 301]], "word_ranges": [[19, 49]], "text": "La endoftalmitis postquirurgica se asocia mas a pacientes en post operatorio inmediato, ademas de ser muy dolorosa con disminucion de la agudeza visual rapidamente progresiva porlo que no es correcta."}, "2": {"exist": true, "char_ranges": [[302, 469]], "word_ranges": [[49, 73]], "text": "El Desprendimiento de retina es coherente con el caso clinico por tratarse ademas de un paciente miope quienes suelen tener especial predisposicion al desprendimiento."}, "3": {"exist": true, "char_ranges": [[470, 557]], "word_ranges": [[73, 88]], "text": "La DMRE suele ocurrir en pacientes mayores y la disminucion de la vision es progresiva."}, "4": {"exist": true, "char_ranges": [[558, 706]], "word_ranges": [[88, 115]], "text": "En el DVP puede o no haber leve disminucion de la agudeza visual y el paciente suele referir cierto disconfort o dolor al inicio del cuadro clinico."}, "5": {"exist": false, "char_ranges": [], "word_ranges": [], "text": ""}}} {"id": 519, "year": 2021, "question_id_specific": 176, "full_question": "Una mujer de 25 a\u00f1os acude a urgencias acompa\u00f1ada de una amiga relatando que ha sido v\u00edctima de una agresi\u00f3n sexual hace 4 horas. La paciente se encuentra muy afectada, contestando a sus preguntas de forma incongruente, con lentitud y sin ser capaz de recordar c\u00f3mo se ha producido la agresi\u00f3n. Sus constantes vitales (tensi\u00f3n arterial, frecuencia cardiaca y temperatura) son normales. \u00bfCu\u00e1l de las siguientes actuaciones ser\u00eda INCORRECTA?:", "full_answer": "Todas las actuaciones que se detallan son correctas salvo la detallada en la opci\u00f3n 2. La toma de muestras de inter\u00e9s legal s\u00f3lo se llevar\u00e1 a cabo en presencia del forense, y, sin la presencia del mismo, la mujer debe dar su consentimiento por escrito. Adem\u00e1s, la toma de muestras ginecol\u00f3gicas debe adecuarse al estado psicol\u00f3gico de la v\u00edctima, por lo que en la mayor\u00eda de casos, es contraproducente realizarla en este momento (m\u00e1s a\u00fan cuando describen el estado de la paciente como \u201cmuy afectada, contestando a sus preguntas de forma incongruente, con lentitud y sin ser capaz de recordar c\u00f3mo se ha producido la agresi\u00f3n\u201d.", "type": "CUIDADOS CR\u00cdTICOS Y URGENCIAS", "options": {"1": "Contactar con el juzgado de guardia.", "2": "Realizar de inmediato una exploraci\u00f3n ginecol\u00f3gica con toma de citolog\u00eda cervical.", "3": "Administrar 500 mg de ceftriaxona intramuscular, 1 g de azitromicina v\u00eda oral y 2 g de tinidazol v\u00eda oral en dosis \u00fanicas.", "4": "Administrar 1500 mg de levonorgestrel v\u00eda oral en dosis \u00fanica.", "5": NaN}, "correct_option": 2, "explanations": {"1": {"exist": false, "char_ranges": [], "word_ranges": [], "text": ""}, "2": {"exist": true, "char_ranges": [[87, 251]], "word_ranges": [[15, 45]], "text": "La toma de muestras de inter\u00e9s legal s\u00f3lo se llevar\u00e1 a cabo en presencia del forense, y, sin la presencia del mismo, la mujer debe dar su consentimiento por escrito."}, "3": {"exist": false, "char_ranges": [], "word_ranges": [], "text": ""}, "4": {"exist": false, "char_ranges": [], "word_ranges": [], "text": ""}, "5": {"exist": false, "char_ranges": [], "word_ranges": [], "text": ""}}} {"id": 63, "year": 2011, "question_id_specific": 200, "full_question": "Un alba\u00f1il de 31 a\u00f1os acude al servicio de urgencias con una tenosinovitis producida por una herida penetrante. En la exploraci\u00f3n se objetiva que la bolsa cubital est\u00e1 infectada con la consiguiente necrosis tendinosa. \u00bfQu\u00e9 tend\u00f3n se ver\u00e1 afectado?", "full_answer": "Tanto el flexor radial del carpo como el abductor largo del pulgar est\u00e1n anat\u00f3micamente lejos del cubital. El palmar largo es medial, as\u00ed como el flexor profundo de los dedos. El tend\u00f3n que va junto al nervio cubital, insert\u00e1ndose en el pisiforme, es el flexor cubital del carpo, por tanto respuesta 1.", "type": "ANATOM\u00cdA", "options": {"1": "Tend\u00f3n del flexor cubital del carpo.", "2": "Tend\u00f3n del abductor largo del pulgar.", "3": "Tend\u00f3n del flexor profundo de los dedos.", "4": "Tend\u00f3n del flexor radial del carpo.", "5": "Tend\u00f3n del palmar largo."}, "correct_option": 1, "explanations": {"1": {"exist": true, "char_ranges": [[176, 302]], "word_ranges": [[30, 52]], "text": "El tend\u00f3n que va junto al nervio cubital, insert\u00e1ndose en el pisiforme, es el flexor cubital del carpo, por tanto respuesta 1."}, "2": {"exist": true, "char_ranges": [[0, 106]], "word_ranges": [[0, 17]], "text": "Tanto el flexor radial del carpo como el abductor largo del pulgar est\u00e1n anat\u00f3micamente lejos del cubital."}, "3": {"exist": true, "char_ranges": [[107, 175]], "word_ranges": [[17, 30]], "text": "El palmar largo es medial, as\u00ed como el flexor profundo de los dedos."}, "4": {"exist": true, "char_ranges": [[0, 106]], "word_ranges": [[0, 17]], "text": "Tanto el flexor radial del carpo como el abductor largo del pulgar est\u00e1n anat\u00f3micamente lejos del cubital."}, "5": {"exist": true, "char_ranges": [[107, 175]], "word_ranges": [[17, 30]], "text": "El palmar largo es medial, as\u00ed como el flexor profundo de los dedos."}}} {"id": 30, "year": 2011, "question_id_specific": 61, "full_question": "Un paciente diagnosticado previamente de EPOC acude a urgencia spor un cuadro de 3 d\u00edas de duraci\u00f3n con aumento de su disnea habitual hasata hacerse de reposo, tos con expectoraci\u00f3n blanquecina y somnolencia acentuada. Tras recibir tratamiento con ox\u00edgeno a bajas concentraciones, broncodilatadores y coticoides se realiza una segunda gasometr\u00eda arterial con fracci\u00f3n inspiratoria de ox\u00edgeno al 28% que muestra un pH de 7.32, pO2 61 mmHg, pCO2 58 mmHg, HCO3- 29 mmol/l \u00bfCu\u00e1l ser\u00eda la interpretaci\u00f3n que har\u00eda usted de la gasometr\u00eda arterial y qu\u00e9 tratamietno utilizar\u00eda?", "full_answer": "Esta es una pregunta con muy mala leche, apostar\u00e9 a una respuesta, pero dependiendo de datos que no nos dan podr\u00edan valer tres respuestas como v\u00e1lidas. La trampa consiste en que nos dan la cl\u00ednica de inicio SIN GASOMETR\u00cdA y luego nos dan la gasometr\u00eda de despu\u00e9s SIN CL\u00cdNICA, as\u00ed no hay forma de saber cu\u00e1l ha sido la evoluci\u00f3n del paciente con nuestro tratamiento por lo que es imposible acertar qu\u00e9 hacer. Si el paciente inicialmente ven\u00eda con una gasometr\u00eda con pH de 7,05 y ahora tiene la que nos dan, lo estamos haciendo bien (aunque tampoco nos dicen el intervalo de tiempo entre una cosa y otra) y, en este caso, podr\u00eda valer seguir con el mismo tratamiento. Si no ha evolucionado nada, la cl\u00ednica es igual y la gasometr\u00eda al principio era parecida habr\u00eda que hacer algo (tambi\u00e9n dependiendo del estado basal del paciente, que tampoco nos cuentan). En caso de decidirnos por hacer algo, en la actualidad puede ser que nos decidi\u00e9ramos por la ventilaci\u00f3n m\u00e1s que por la teofilina aunque con \"tan poca\" alteraci\u00f3n del pH (y m\u00e1s si pensamos que es un cr\u00f3nico que quiz\u00e1 tiene una gasometr\u00eda basal parecida a la que nos presentan) puede parecer \"exagerado\". \"Contra\" la teofilina no tenemos nada (no nos cuentan ninguna contraindicaci\u00f3n), pero en la actualidad es raro recurrir a ella. 1: Err\u00f3nea, es un paciente \u201cretenedor\u201d y aumentando el flujo de O2 es altamente posible que aumentase tambi\u00e9n la hipercapnia y, por tanto, la acidosis. 2: Nos quedaremos con esta como correcta. 3: \u00a1No es una acidosis metab\u00f3lica!, el bicarbonato no est\u00e1 disminu\u00eddo, al contrario, est\u00e1 ligeramente aumentado intentando compensar la acidosis respiratoria. 4: No llega a insuficiencia respiratoria por definici\u00f3n (pO2 60), pero tiene hipercapnia y ligera acidosis. Si supi\u00e9ramos la primera gasometr\u00eda y esta estuviese mucho peor (es decir, estuviese evolucionando muy bien) seguir con la misma medicaci\u00f3n ser\u00eda una opci\u00f3n, \u00a1\u00a1falta ese dato!!, en todo caso parece una respuesta trampa. 5: En la actualidad y -ya comentado- teniendo en cuenta que no hay contraindicaci\u00f3n formal, no se suele utilizar.", "type": "NEUROLOG\u00cdA Y CIRUG\u00cdA TOR\u00c1CICA", "options": {"1": "Aumentar\u00eda el flujo de ox\u00edgeno porque tiene acidosis respiratoria aguda e hipoxemia.", "2": "Iniciar\u00eda ventilaci\u00f3n mec\u00e1nica no invasiva porque tiene una insuficiencia respiratoria hiperc\u00e1pnica aguda con acidosis respiratoria moderada.", "3": "A\u00f1adir\u00eda bicarbonato s\u00f3dico para corregir la acidosis metab\u00f3lica aguda.", "4": "El paciente no tiene insuficiencia respiratoria por lo que continuar\u00eda con la misma pauta farmacol\u00f3gica.", "5": "A\u00f1adir\u00eda aminofilina intravenosa como estimulante respiratorio, ya que aprecio hipoxemia e hipercapnia."}, "correct_option": 4, "explanations": {"1": {"exist": true, "char_ranges": [[1291, 1439]], "word_ranges": [[225, 249]], "text": "Err\u00f3nea, es un paciente \u201cretenedor\u201d y aumentando el flujo de O2 es altamente posible que aumentase tambi\u00e9n la hipercapnia y, por tanto, la acidosis."}, "2": {"exist": true, "char_ranges": [[856, 1159]], "word_ranges": [[151, 204]], "text": "En caso de decidirnos por hacer algo, en la actualidad puede ser que nos decidi\u00e9ramos por la ventilaci\u00f3n m\u00e1s que por la teofilina aunque con \"tan poca\" alteraci\u00f3n del pH (y m\u00e1s si pensamos que es un cr\u00f3nico que quiz\u00e1 tiene una gasometr\u00eda basal parecida a la que nos presentan) puede parecer \"exagerado\"."}, "3": {"exist": true, "char_ranges": [[1485, 1640]], "word_ranges": [[257, 277]], "text": "\u00a1No es una acidosis metab\u00f3lica!, el bicarbonato no est\u00e1 disminu\u00eddo, al contrario, est\u00e1 ligeramente aumentado intentando compensar la acidosis respiratoria."}, "4": {"exist": true, "char_ranges": [[1644, 1748]], "word_ranges": [[278, 293]], "text": "No llega a insuficiencia respiratoria por definici\u00f3n (pO2 60), pero tiene hipercapnia y ligera acidosis."}, "5": {"exist": true, "char_ranges": [[1972, 2082]], "word_ranges": [[328, 346]], "text": "En la actualidad y -ya comentado- teniendo en cuenta que no hay contraindicaci\u00f3n formal, no se suele utilizar."}}} {"id": 226, "year": 2014, "question_id_specific": 71, "full_question": "Mujer de 82 a\u00f1os hipertensa en tratamiento con atenolol, hidroclorotiazida y digoxina. Acude a urgencias por fibrilaci\u00f3n auricular y se le administra verapamil IV. Se evidencia en ECG un bloqueo auriculoventriuclar completo. \u00bfCu\u00e1l es la causa m\u00e1s probable de esta situaci\u00f3n cl\u00ednica?", "full_answer": "Ya toma dos frenadores del nodo AV y le a\u00f1ades un tercero, \u00bfy te extra\u00f1as de que haga un bloqueo AV completo despu\u00e9s de guarrearle con f\u00e1rmacos de distintas familias pero mismo efecto (farmacodin\u00e1mica)? La tres. Y recuerda que los f\u00e1rmacos son como las copas cuando vas de bares: \u00a1no mezcles!", "type": "CARDIOLOG\u00cdA", "options": {"1": "Intoxicaci\u00f3n digit\u00e1lica por interacci\u00f3n farmacocin\u00e9tica por verapamilo.", "2": "Hipopotasemia por la administraci\u00f3n de tiazida y digoxina.", "3": "Interacci\u00f3n farmacodin\u00e1mica del betabloqueante, digoxina y verapamil.", "4": "Efecto hipotensor del diur\u00e9tico tiaz\u00eddico.", "5": "Arritmia cardiaca por verapamil."}, "correct_option": 3, "explanations": {"1": {"exist": false, "char_ranges": [], "word_ranges": [], "text": ""}, "2": {"exist": false, "char_ranges": [], "word_ranges": [], "text": ""}, "3": {"exist": true, "char_ranges": [[0, 202]], "word_ranges": [[0, 34]], "text": "Ya toma dos frenadores del nodo AV y le a\u00f1ades un tercero, \u00bfy te extra\u00f1as de que haga un bloqueo AV completo despu\u00e9s de guarrearle con f\u00e1rmacos de distintas familias pero mismo efecto (farmacodin\u00e1mica)?"}, "4": {"exist": false, "char_ranges": [], "word_ranges": [], "text": ""}, "5": {"exist": false, "char_ranges": [], "word_ranges": [], "text": ""}}} {"id": 565, "year": 2022, "question_id_specific": 148, "full_question": "Paciente de 48 a\u00f1os remitido a consulta de nefrolog\u00eda por un filtrado glomerular estimado (FGe) de 32 ml/min/1,72 m\u00b2. \u00bfCu\u00e1l de los siguientes datos NO orientar\u00eda a pensar en una enfermedad renal cr\u00f3nica?:", "full_answer": "La adecuada diferenciaci\u00f3n corticomedular es sugestiva de que no hay una alteraci\u00f3n parenquimatosa cr\u00f3nica, y es un hallazgo frecuente en el fracaso renal agudo (opci\u00f3n 4 correcta). La historia familiar de nefropat\u00eda podr\u00eda indicar la existencia de un trastorno hereditario (opci\u00f3n 2 incorrecta). Los ri\u00f1ones peque\u00f1os y la hiperfosfatemia son caracter\u00edsticas propias de la enfermedad renal cr\u00f3nica (opci\u00f3n 1 y 3 incorrectas).", "type": "NEFROLOG\u00cdA", "options": {"1": "Ri\u00f1ones de peque\u00f1o tama\u00f1o.", "2": "Antecedentes familiares de nefropat\u00eda.", "3": "Niveles elevados de f\u00f3sforo en sangre.", "4": "Buena diferenciaci\u00f3n ecogr\u00e1fica de corteza y m\u00e9dula renales.", "5": NaN}, "correct_option": 4, "explanations": {"1": {"exist": true, "char_ranges": [[297, 425]], "word_ranges": [[43, 62]], "text": "Los ri\u00f1ones peque\u00f1os y la hiperfosfatemia son caracter\u00edsticas propias de la enfermedad renal cr\u00f3nica (opci\u00f3n 1 y 3 incorrectas)."}, "2": {"exist": true, "char_ranges": [[182, 296]], "word_ranges": [[27, 43]], "text": "La historia familiar de nefropat\u00eda podr\u00eda indicar la existencia de un trastorno hereditario (opci\u00f3n 2 incorrecta)."}, "3": {"exist": true, "char_ranges": [[297, 425]], "word_ranges": [[43, 62]], "text": "Los ri\u00f1ones peque\u00f1os y la hiperfosfatemia son caracter\u00edsticas propias de la enfermedad renal cr\u00f3nica (opci\u00f3n 1 y 3 incorrectas)."}, "4": {"exist": true, "char_ranges": [[0, 181]], "word_ranges": [[0, 27]], "text": "La adecuada diferenciaci\u00f3n corticomedular es sugestiva de que no hay una alteraci\u00f3n parenquimatosa cr\u00f3nica, y es un hallazgo frecuente en el fracaso renal agudo (opci\u00f3n 4 correcta)."}, "5": {"exist": false, "char_ranges": [], "word_ranges": [], "text": ""}}} {"id": 135, "year": 2012, "question_id_specific": 38, "full_question": "Hombre de 64 a\u00f1os con cirrosis hep\u00e1tica Child A sin antecedentes de descompensaci\u00f3n. Tras detecci\u00f3n de una lesi\u00f3n focal hep\u00e1tica en ecograf\u00eda de cribado se completa el estudio con una TC toracoabdominal. En esta exploraci\u00f3n se objetiva la presencia de 4 lesiones hep\u00e1ticas (una de ellas de hasta 6 cm) con patr\u00f3n de captaci\u00f3n t\u00edpico de hepatocarcinoma, invasi\u00f3n vascular tumoral y una met\u00e1stasis en la gl\u00e1ndula suprarrenal derecha. No se observa ascitis. El paciente refiere \u00fanicamente astenia, pero no presenta s\u00edndrome general. El tratamiento de este paciente ser\u00eda:", "full_answer": "Comentario:El paciente presenta un buen estado general con un hepatocarcinoma reci\u00e9n diagnosticado, no darle ning\u00fan tipo de tratamiento m\u00e1s que el sintom\u00e1tico y de soporte a estas alturas no ser\u00eda lo correcto. La TACE no estar\u00eda indicada por tener afectaci\u00f3n extrahep\u00e1tica. El trasplante teniendo enfermedad extrahep\u00e1tica tampoco se lo har\u00edan. Nos quedan las respuestas 3 y 4, las dos con tratamiento sist\u00e9mico (indicado por ser metast\u00e1sico) y la correcta es la 4, por todas las razones que da en la misma.", "type": "ONCOLOG\u00cdA", "options": {"1": "Tratamiento sintom\u00e1tico y de soporte, ya que presenta un hepatocarcinoma avanzado con invasi\u00f3n vascular y met\u00e1stasis.", "2": "Quimioembolizaci\u00f3n transarterial (TACE), ya que el tratamiento aumenta la supervivencia de los pacientes con hepatocarcinoma avanzado.", "3": "Quimioterapia sist\u00e9mica con doxorrubicina iv ya que presenta enfermedad diseminada pero presenta buen estado general.", "4": "Tratamiento con sorafenib oral, ya que se trata de un enfermo con buen estado general, Child A y hepatocarcinoma en estadio avanzado BCLC-C.", "5": "El tratamiento adecuado es el trasplante hep\u00e1tico ya que es el \u00fanico procedimiento que elimina completamente el tumor primario y evita las complicaiones futuras de la cirrosis."}, "correct_option": 4, "explanations": {"1": {"exist": true, "char_ranges": [[11, 209]], "word_ranges": [[0, 32]], "text": "Comentario:El paciente presenta un buen estado general con un hepatocarcinoma reci\u00e9n diagnosticado, no darle ning\u00fan tipo de tratamiento m\u00e1s que el sintom\u00e1tico y de soporte a estas alturas no ser\u00eda lo correcto."}, "2": {"exist": true, "char_ranges": [[210, 273]], "word_ranges": [[32, 41]], "text": "La TACE no estar\u00eda indicada por tener afectaci\u00f3n extrahep\u00e1tica."}, "3": {"exist": true, "char_ranges": [[377, 506]], "word_ranges": [[57, 81]], "text": "las dos con tratamiento sist\u00e9mico (indicado por ser metast\u00e1sico) y la correcta es la 4, por todas las razones que da en la misma."}, "4": {"exist": true, "char_ranges": [[377, 506]], "word_ranges": [[57, 81]], "text": "las dos con tratamiento sist\u00e9mico (indicado por ser metast\u00e1sico) y la correcta es la 4, por todas las razones que da en la misma."}, "5": {"exist": true, "char_ranges": [[274, 343]], "word_ranges": [[41, 50]], "text": "El trasplante teniendo enfermedad extrahep\u00e1tica tampoco se lo har\u00edan."}}} {"id": 556, "year": 2022, "question_id_specific": 55, "full_question": "Paciente diab\u00e9tico de 63 a\u00f1os a quien su m\u00e9dico de familia ha solicitado una anal\u00edtica para determinar los niveles de vitamina B12. \u00bfCu\u00e1l es el f\u00e1rmaco antidiab\u00e9tico que toma que justifica dicha solicitud?:", "full_answer": "La metformina es un antidiab\u00e9tico oral de uso extendido en el tratamiento de la diabetes mellitus tipo 2. Los efectos secundarios m\u00e1s conocidos son los gastrointestinales y la acidosis l\u00e1ctica; sin embargo, la malabsorci\u00f3n de vitamina B 12, es menos conocida. Se observa una disminuci\u00f3n de los niveles de vitamina B 12, en pacientes tratados con metformina. El mecanismo por el cual sucede este d\u00e9ficit no est\u00e1 claro y se sabe que es reversible al interrumpir el tratamiento.", "type": "ENDOCRINOLOG\u00cdA", "options": {"1": "Gliclazida.", "2": "Metformina.", "3": "Repaglinida.", "4": "Pioglitazona.", "5": NaN}, "correct_option": 2, "explanations": {"1": {"exist": false, "char_ranges": [], "word_ranges": [], "text": ""}, "2": {"exist": true, "char_ranges": [[0, 259]], "word_ranges": [[0, 41]], "text": "La metformina es un antidiab\u00e9tico oral de uso extendido en el tratamiento de la diabetes mellitus tipo 2. Los efectos secundarios m\u00e1s conocidos son los gastrointestinales y la acidosis l\u00e1ctica; sin embargo, la malabsorci\u00f3n de vitamina B 12, es menos conocida."}, "3": {"exist": false, "char_ranges": [], "word_ranges": [], "text": ""}, "4": {"exist": false, "char_ranges": [], "word_ranges": [], "text": ""}, "5": {"exist": false, "char_ranges": [], "word_ranges": [], "text": ""}}} {"id": 448, "year": 2018, "question_id_specific": 148, "full_question": "Mujer de 59 a\u00f1os con crisis epil\u00e9pticas de reciente instauraci\u00f3n que acude a urgencias. En la exploraci\u00f3n presenta signos piramidales izquierdos y edema de papila. Se le realiza RM cerebral que muestra una masa hemisf\u00e9rica derecha con edema, desviaci\u00f3n de la l\u00ednea media y signos de herniaci\u00f3n tentorial. Refiere que desde hace una semana sufre dolor de cabeza que ha ido progresando en intensidad. \u00bfCu\u00e1l de las siguientes caracter\u00edsticas asociadas a la cefalea le parece m\u00e1s probable en esta paciente?:", "full_answer": "Nos describen un reciente diagn\u00f3stico de LOE cerebral con signos de hipertensi\u00f3n intracraneal (edema, desviaci\u00f3n de l\u00ednea media, papiledema). La cefalea asociada es t\u00edpicamente matutina, al aumentar por la noche los valores de PIC.", "type": "NEUROCIRUG\u00cdA", "options": {"1": "Predominio matutino.", "2": "No cambia con esfuerzos.", "3": "Fotofobia.", "4": "Sonofobia.", "5": NaN}, "correct_option": 1, "explanations": {"1": {"exist": true, "char_ranges": [[0, 231]], "word_ranges": [[0, 34]], "text": "Nos describen un reciente diagn\u00f3stico de LOE cerebral con signos de hipertensi\u00f3n intracraneal (edema, desviaci\u00f3n de l\u00ednea media, papiledema). La cefalea asociada es t\u00edpicamente matutina, al aumentar por la noche los valores de PIC."}, "2": {"exist": false, "char_ranges": [], "word_ranges": [], "text": ""}, "3": {"exist": false, "char_ranges": [], "word_ranges": [], "text": ""}, "4": {"exist": false, "char_ranges": [], "word_ranges": [], "text": ""}, "5": {"exist": false, "char_ranges": [], "word_ranges": [], "text": ""}}} {"id": 591, "year": 2022, "question_id_specific": 64, "full_question": "Mujer de 68 a\u00f1os que utiliza para visi\u00f3n lejana gafas de +4,00 dioptr\u00edas en ambos ojos. Consulta porque desde hace unas semanas tiene dolor ocular con cefalea y visi\u00f3n borrosa por la noche cuando est\u00e1 viendo la televisi\u00f3n en penumbra. \u00bfCu\u00e1l de las siguientes patolog\u00edas es m\u00e1s probable que presente?", "full_answer": "Nos presentan a una mujer de 68 a\u00f1os con hipermetrop\u00eda moderada y no refieren antecedentes de cirug\u00eda de cataratas. Este mero dato no es casualidad: es importante el hecho de que la paciente sea hiperm\u00e9trope, porque los globos oculares de los hiperm\u00e9tropes son m\u00e1s cortos, con cierto conflicto de espacio en el segmento anterior cuando el cristalino sufre un proceso de intumescencia secundario a la edad (aquello que los pacientes llaman catarata incipiente). En el caso cl\u00ednico, adem\u00e1s, el dolor ocular y de cabeza se produce en situaciones escot\u00f3picas, o de iluminaci\u00f3n ambiental tenue, lo cual produce una midriasis media para optimizar el influjo lum\u00ednico al globo ocular. Es evidente, por tanto, que la paciente sufre procesos de cierre angular primario que, posiblemente, se auto-resuelvan cuando cesa la situaci\u00f3n desencadenante, y no un verdadero ataque agudo de glaucoma. Pero estos cierres angulares intermitentes se contemplan dentro del especto del glaucoma por cierre angular primario (opci\u00f3n 1 correcta). La migra\u00f1a, no siendo esta una pregunta de Neurolog\u00eda, a veces puede ser un confusor en la cl\u00ednica habitual; sin embargo, las pacientes suelen ser m\u00e1s j\u00f3venes, o al menos contar antecedentes claros de migra\u00f1a, y, sobre todo, presentan empeoramiento sintomatol\u00f3gico con est\u00edmulos sensoriales intensos (fotofobia y fonofobia), pero los s\u00edntomas no se empeoran precisamente en casos de iluminaci\u00f3n tenue (opci\u00f3n 2 incorrecta). La queratitis act\u00ednica se produce precisamente en pacientes expuestos a radiaci\u00f3n ultravioleta no filtrada correctamente antes de llegar a la c\u00f3rnea; es t\u00edpica de soldadores (se llama queratitis del soldador), esquiadores y dem\u00e1s personas con exposiciones lum\u00ednicas muy intensas y sin protecci\u00f3n ocular adecuada (opci\u00f3n 3 incorrecta). El s\u00edndrome de ojo seco no suele producir dolor de cabeza, y ocularmente no pasa de unas pertinaces molestias que podr\u00edan definir los pacientes como dolor por su componente neurop\u00e1tico recientemente m\u00e1s estudiado. Algunas formas de ojo seco pueden empeorar con car\u00e1cter vespertino pero otras lo hacen por la ma\u00f1ana, y depende fundamentalmente de la frecuencia de parpadeo. Adem\u00e1s s\u00edndrome de ojo seco es un t\u00e9rmino desactualizado; hoy en d\u00eda se prefiere hablar de enfermedad de ojo seco (opci\u00f3n 4 incorrecta).", "type": "OFTALMOLOG\u00cdA", "options": {"1": "Glaucoma por cierre angular primario.", "2": "Migra\u00f1a.", "3": "Queratitis act\u00ednica.", "4": "S\u00edndrome de ojo seco.", "5": NaN}, "correct_option": 1, "explanations": {"1": {"exist": true, "char_ranges": [[678, 1019]], "word_ranges": [[108, 157]], "text": "Es evidente, por tanto, que la paciente sufre procesos de cierre angular primario que, posiblemente, se auto-resuelvan cuando cesa la situaci\u00f3n desencadenante, y no un verdadero ataque agudo de glaucoma. Pero estos cierres angulares intermitentes se contemplan dentro del especto del glaucoma por cierre angular primario (opci\u00f3n 1 correcta)."}, "2": {"exist": true, "char_ranges": [[1020, 1443]], "word_ranges": [[157, 220]], "text": "La migra\u00f1a, no siendo esta una pregunta de Neurolog\u00eda, a veces puede ser un confusor en la cl\u00ednica habitual; sin embargo, las pacientes suelen ser m\u00e1s j\u00f3venes, o al menos contar antecedentes claros de migra\u00f1a, y, sobre todo, presentan empeoramiento sintomatol\u00f3gico con est\u00edmulos sensoriales intensos (fotofobia y fonofobia), pero los s\u00edntomas no se empeoran precisamente en casos de iluminaci\u00f3n tenue (opci\u00f3n 2 incorrecta)."}, "3": {"exist": true, "char_ranges": [[1444, 1778]], "word_ranges": [[220, 267]], "text": "La queratitis act\u00ednica se produce precisamente en pacientes expuestos a radiaci\u00f3n ultravioleta no filtrada correctamente antes de llegar a la c\u00f3rnea; es t\u00edpica de soldadores (se llama queratitis del soldador), esquiadores y dem\u00e1s personas con exposiciones lum\u00ednicas muy intensas y sin protecci\u00f3n ocular adecuada (opci\u00f3n 3 incorrecta)."}, "4": {"exist": true, "char_ranges": [[1779, 2288]], "word_ranges": [[267, 348]], "text": "El s\u00edndrome de ojo seco no suele producir dolor de cabeza, y ocularmente no pasa de unas pertinaces molestias que podr\u00edan definir los pacientes como dolor por su componente neurop\u00e1tico recientemente m\u00e1s estudiado. Algunas formas de ojo seco pueden empeorar con car\u00e1cter vespertino pero otras lo hacen por la ma\u00f1ana, y depende fundamentalmente de la frecuencia de parpadeo. Adem\u00e1s s\u00edndrome de ojo seco es un t\u00e9rmino desactualizado; hoy en d\u00eda se prefiere hablar de enfermedad de ojo seco (opci\u00f3n 4 incorrecta)."}, "5": {"exist": false, "char_ranges": [], "word_ranges": [], "text": ""}}} {"id": 291, "year": 2016, "question_id_specific": 157, "full_question": "Ante un paciente de 8 a\u00f1os de edad que acude por presentar desde hace 3 d\u00edas lesiones pustulosas superficiales, erosiones y costras amarillentas alrededor de la boca pensaremos en:", "full_answer": "Es un cuadro cl\u00ednico t\u00edpico y frecuente en ni\u00f1os. Est\u00e1 describiendo un imp\u00e9tigo contagioso. No hay mucha discusi\u00f3n. El agente causal m\u00e1s frecuente en la forma ampollosa es S. aureus fago II.", "type": "DERMATOLOG\u00cdA, VENEREOLOG\u00cdA Y CIRUG\u00cdA PL\u00c1STICA", "options": {"1": "Eritema exudativo multiforme.", "2": "Imp\u00e9tigo contagioso.", "3": "Acn\u00e9 infantil.", "4": "Psoriasis pustuloso.", "5": NaN}, "correct_option": 2, "explanations": {"1": {"exist": false, "char_ranges": [], "word_ranges": [], "text": ""}, "2": {"exist": true, "char_ranges": [[0, 91]], "word_ranges": [[0, 14]], "text": "Es un cuadro cl\u00ednico t\u00edpico y frecuente en ni\u00f1os. Est\u00e1 describiendo un imp\u00e9tigo contagioso."}, "3": {"exist": false, "char_ranges": [], "word_ranges": [], "text": ""}, "4": {"exist": false, "char_ranges": [], "word_ranges": [], "text": ""}, "5": {"exist": false, "char_ranges": [], "word_ranges": [], "text": ""}}} {"id": 213, "year": 2014, "question_id_specific": 160, "full_question": "Mujer de 75 a\u00f1os, con menopausia a los 52 a\u00f1os, sin antecedentes familiares ni personales de fractura, diagnosticada de arteritis de la temporal, que va a iniciar tratamiento con prednisona a dosis altas y con expectativa de tratamiento durante al menos un a\u00f1o. La demora de densitometr\u00eda (DXA) en su centro es de 4-5 meses. Se plantea tratamiento preventivo de osteoporosis. Entre las siguientes \u00bfcu\u00e1l es la actitud m\u00e1s adecuada?:", "full_answer": "En los datos que nos proporcionan no hay factores de riesgo cl\u00ednico de osteoporosis previos a la enfermedad actual. Como se instaurar\u00e1 una pauta prolongada y a dosis altas con prednisona es recomendable, adem\u00e1s de las medidas preventivas no farmacol\u00f3gicas, iniciar en cualquier caso un tratamiento para minimizar la p\u00e9rdida de masa \u00f3sea secundaria que se produce con la administraci\u00f3n de glucocorticoides (y, sobre todo, en los primeros meses). Por tanto, y s\u00f3lo por este motivo, quedar\u00edan descartadas las opciones 1, 2 y 5 (las tres anteponen otro condicionante). En este contexto, y al tratarse de una persona de m\u00e1s de 65 a\u00f1os, una actitud adecuada ser\u00eda administrar suplementos de calcio (los corticoides son osteopenizantes por producir, entre otros mecanismos, un balance negativo de este elemento) y vitamina D. La opci\u00f3n de asociar un bifosfonato tambi\u00e9n se podr\u00eda contemplar, pero en la propuesta 3 falta el suplemento de calcio.", "type": "REUMATOLOG\u00cdA", "options": {"1": "Solicitar DXA y esperar al resultado.", "2": "Valorar riesgo absoluto de fractura mediante el cuestionario FRAX sin DMO y tratar s\u00f3lo si es alto.", "3": "Iniciar tratamiento con Bisfosfonatos y vitamina D (800 UI/d\u00eda).", "4": "Administrar suplemento de calcio (1g) y vitamina D (800 UI/d\u00eda).", "5": "Valorar si tiene osteopenia en las radiograf\u00edas y tratar si est\u00e1 presente."}, "correct_option": 4, "explanations": {"1": {"exist": true, "char_ranges": [[116, 564]], "word_ranges": [[19, 89]], "text": "Como se instaurar\u00e1 una pauta prolongada y a dosis altas con prednisona es recomendable, adem\u00e1s de las medidas preventivas no farmacol\u00f3gicas, iniciar en cualquier caso un tratamiento para minimizar la p\u00e9rdida de masa \u00f3sea secundaria que se produce con la administraci\u00f3n de glucocorticoides (y, sobre todo, en los primeros meses). Por tanto, y s\u00f3lo por este motivo, quedar\u00edan descartadas las opciones 1, 2 y 5 (las tres anteponen otro condicionante)."}, "2": {"exist": true, "char_ranges": [[116, 564]], "word_ranges": [[19, 89]], "text": "Como se instaurar\u00e1 una pauta prolongada y a dosis altas con prednisona es recomendable, adem\u00e1s de las medidas preventivas no farmacol\u00f3gicas, iniciar en cualquier caso un tratamiento para minimizar la p\u00e9rdida de masa \u00f3sea secundaria que se produce con la administraci\u00f3n de glucocorticoides (y, sobre todo, en los primeros meses). Por tanto, y s\u00f3lo por este motivo, quedar\u00edan descartadas las opciones 1, 2 y 5 (las tres anteponen otro condicionante)."}, "3": {"exist": true, "char_ranges": [[819, 938]], "word_ranges": [[129, 149]], "text": "La opci\u00f3n de asociar un bifosfonato tambi\u00e9n se podr\u00eda contemplar, pero en la propuesta 3 falta el suplemento de calcio."}, "4": {"exist": true, "char_ranges": [[565, 818]], "word_ranges": [[89, 129]], "text": "En este contexto, y al tratarse de una persona de m\u00e1s de 65 a\u00f1os, una actitud adecuada ser\u00eda administrar suplementos de calcio (los corticoides son osteopenizantes por producir, entre otros mecanismos, un balance negativo de este elemento) y vitamina D."}, "5": {"exist": true, "char_ranges": [[116, 564]], "word_ranges": [[19, 89]], "text": "Como se instaurar\u00e1 una pauta prolongada y a dosis altas con prednisona es recomendable, adem\u00e1s de las medidas preventivas no farmacol\u00f3gicas, iniciar en cualquier caso un tratamiento para minimizar la p\u00e9rdida de masa \u00f3sea secundaria que se produce con la administraci\u00f3n de glucocorticoides (y, sobre todo, en los primeros meses). Por tanto, y s\u00f3lo por este motivo, quedar\u00edan descartadas las opciones 1, 2 y 5 (las tres anteponen otro condicionante)."}}} {"id": 442, "year": 2018, "question_id_specific": 100, "full_question": "Mujer de 34 a\u00f1os de edad que consulta por coitorragias de repetici\u00f3n. Aporta una citologia con una lesi\u00f3n Intraepitelial de alto grado (U-SIL). Posteriormente se realiza colposcopia y biopsia sobre una zona de mosaico y la histolog\u00eda nos habla de un foco de carcinoma escamoso invasor de 2 mm de longitud. \u00bfQu\u00e9 opci\u00f3n terap\u00e9utica es la m\u00e1s adecuada para esta paciente?", "full_answer": "Seg\u00fan la SEGO: \u00abExtirpar la totalidad de la lesi\u00f3n con el objetivo de que pueda ser evaluada histol\u00f3gicamente. Implica la escisi\u00f3n de toda la zona de transformaci\u00f3n. En general, la escisi\u00f3n debe adaptarse al tama\u00f1o y caracter\u00edsticas lesionales. Se distinguen 3 tipos de escisi\u00f3n seg\u00fan la presencia de componente endocervical de la lesi\u00f3n. Escisi\u00f3n tipo 1 (aplicable en casos con zona de transformaci\u00f3n tipo 1, en los que el asa diat\u00e9rmica no debe incluir canal endocervical ni superar los 8 mm de profundidad), escisi\u00f3n tipo 2 (indicada en zonas de transformaci\u00f3n tipo 2, implica resecar una peque\u00f1a parte de canal endocervical visible mediante colposcopia) y escisi\u00f3n tipo 3 (indicada en zonas de transformaci\u00f3n tipo 3, incluye parte de epitelio endocervical).\u00bb. Cuando hablan de escisi\u00f3n, es sin\u00f3nimo de conizaci\u00f3n; como la lesi\u00f3n es menor de 8mm de profundidad, \u00e9ste es el tratamiento indicado.", "type": "GINECOLOG\u00cdA Y OBSTETRICIA", "options": {"1": "Conizaci\u00f3n.", "2": "Histerectomia total sin anexectomia.", "3": "Radioterapia p\u00e9lvica con intenci\u00f3n curativa.", "4": "Repetir una biopsia mas amplia.", "5": NaN}, "correct_option": 1, "explanations": {"1": {"exist": true, "char_ranges": [[818, 897]], "word_ranges": [[128, 142]], "text": "como la lesi\u00f3n es menor de 8mm de profundidad, \u00e9ste es el tratamiento indicado."}, "2": {"exist": false, "char_ranges": [], "word_ranges": [], "text": ""}, "3": {"exist": false, "char_ranges": [], "word_ranges": [], "text": ""}, "4": {"exist": false, "char_ranges": [], "word_ranges": [], "text": ""}, "5": {"exist": false, "char_ranges": [], "word_ranges": [], "text": ""}}} {"id": 491, "year": 2020, "question_id_specific": 106, "full_question": "Mujer de 65 a\u00f1os derivada a urgencias por fiebre y alteraciones en la anal\u00edtica: Hemoglobina 11,4 g/dL, leucocitos 0,86 x103/\u03bcL,(neutr\u00f3filos 41,9 %, linfocitos 55,8 %),plaquetas 48,0 x103/\u03bcL, fibrin\u00f3geno 118 mg/dL,d\u00edmero D 20,2 \u03bcg/mL. Se realiza examen de m\u00e9dula \u00f3sea con la que se le diagnostica una leucemia aguda con t(15;17) en el 60 % de lasc\u00e9lulas. \u00bfCu\u00e1l de las siguientes respuestas es correcta?:", "full_answer": "Esta pregunta me parece muy bonita. No hay duda del diagn\u00f3stico. Porque si no te sabes la traslocaci\u00f3n, no pasa nada\u2026 las respuestas te dan el diagn\u00f3stico. Ahora toca saber c\u00f3mo se trata actualmente una leucemia promieloc\u00edtica aguda (ATRA aparte\u2026 que eso seguro que lo sab\u00e9is). A partir del a\u00f1o 2017, el protocolo PETHEMA para LPA en pacientes de riesgo bajo o intermedio (y/o mayores de 70 a\u00f1os) es ATO+ATRA. En pacientes de alto riesgo, se trata con ATRA+quimioterapia. No creo que quieran que sep\u00e1is c\u00f3mo se establece el riesgo en las promielos, pero se basa en las cifras de leucocitos y plaquetas. En este caso, n o punt\u00faa nada para el score ya que tiene <10.000 leucos y >40.000 plaquetas. Es decir, es un riesgo bajo. Respuesta correcta 2. Quiz\u00e1s haya gente que se haya confundido con la opci\u00f3n 3. Una de las principales causas de mortalidad en la inducci\u00f3n de una promieloc\u00edtica, es la coagulopat\u00eda.\u00a1\u00a1 Pero la 3 es totalmente incorrecta. Solo podr\u00edamos estropear a\u00fan m\u00e1s las cosas poniendo heparina!! No s\u00e9 qu\u00e9 est\u00e1n explicando ahora mismo en las academias, pero est\u00e1 claro que quer\u00edan que supierais que muchas promielos, desde hace 3 a\u00f1itos, no se tratan con quimioterapia.", "type": "HEMATOLOGIA", "options": {"1": "Si est\u00e1 asintom\u00e1tica se iniciar\u00e1 \u00e1cido transretinoico (ATRA) y se recomendar\u00e1n controles en hospital de d\u00eda.", "2": "Se iniciar\u00e1 tratamiento con tri\u00f3xido de ars\u00e9nico, ATRA y terapia de soporte.", "3": "Es una leucemia mielobl\u00e1stica tipo M3, por lo que se iniciara tratamiento quimioter\u00e1pico y heparina para controlar la coagulaci\u00f3n intravascular diseminada.", "4": "Se debe iniciar tratamiento antibi\u00f3tico. Cuando desaparezca la fiebre se iniciar\u00e1 el tratamiento de la leucemia.", "5": NaN}, "correct_option": 2, "explanations": {"1": {"exist": false, "char_ranges": [], "word_ranges": [], "text": ""}, "2": {"exist": true, "char_ranges": [[278, 409]], "word_ranges": [[46, 70]], "text": "A partir del a\u00f1o 2017, el protocolo PETHEMA para LPA en pacientes de riesgo bajo o intermedio (y/o mayores de 70 a\u00f1os) es ATO+ATRA."}, "3": {"exist": true, "char_ranges": [[805, 1008]], "word_ranges": [[141, 172]], "text": "Una de las principales causas de mortalidad en la inducci\u00f3n de una promieloc\u00edtica, es la coagulopat\u00eda.\u00a1\u00a1 Pero la 3 es totalmente incorrecta. Solo podr\u00edamos estropear a\u00fan m\u00e1s las cosas poniendo heparina!!"}, "4": {"exist": false, "char_ranges": [], "word_ranges": [], "text": ""}, "5": {"exist": false, "char_ranges": [], "word_ranges": [], "text": ""}}} {"id": 277, "year": 2016, "question_id_specific": 226, "full_question": "En un programa de cribado de c\u00e1ncer de colon, a un paciente de 52 a\u00f1os se le realiza una colonoscopia. Todo el colon es de aspecto normal, salvo el hallazgo de un p\u00f3lipo pediculado de 2 cm en sigma que se extirpa con asa de diatermia. El resultado anatomopatol\u00f3gico nos indica que existe un carcinoma in situ limitado a la cabeza del p\u00f3lipo. Tiene una TC toracoabdominal informado como normal. Indique cu\u00e1l ser\u00eda la conducta correcta a seguir.", "full_answer": "El carcinoma in situ est\u00e1 delimitado por la membrana basal, o sea que no es invasivo a\u00fan, y por tanto, el tratamiento es la polipectom\u00eda, que ya est\u00e1 hecha. Posteriormente conviene hacer un seguimiento endosc\u00f3pico (con biopsias) con un intervalo de entre 3 y 6 meses en funci\u00f3n de sus antecedentes, edad y patolog\u00edas concomitantes.", "type": "APARATO DIGESTIVO", "options": {"1": "Resecci\u00f3n segmentaria del colon afecto.", "2": "Vigilancia peri\u00f3dica endosc\u00f3pica.", "3": "Resecci\u00f3n local de la base del p\u00f3lipo.", "4": "Sigmoidectom\u00eda m\u00e1s linfadenectom\u00eda.", "5": NaN}, "correct_option": 2, "explanations": {"1": {"exist": false, "char_ranges": [], "word_ranges": [], "text": ""}, "2": {"exist": true, "char_ranges": [[172, 331]], "word_ranges": [[30, 55]], "text": "conviene hacer un seguimiento endosc\u00f3pico (con biopsias) con un intervalo de entre 3 y 6 meses en funci\u00f3n de sus antecedentes, edad y patolog\u00edas concomitantes."}, "3": {"exist": false, "char_ranges": [], "word_ranges": [], "text": ""}, "4": {"exist": false, "char_ranges": [], "word_ranges": [], "text": ""}, "5": {"exist": false, "char_ranges": [], "word_ranges": [], "text": ""}}} {"id": 155, "year": 2012, "question_id_specific": 83, "full_question": "Hombre de 62 a\u00f1os con antecedentes de hipertensi\u00f3n arterial en tratamiento con captoprilo, \u00falcera duodenal y urolitiasis por \u00e1cido \u00farico. Consulta por crisis de podagra t\u00edpica similar a otras presentadas en los \u00faltimos dos a\u00f1os. En la exploraci\u00f3n se observan tofos en ambos pabellones auriculares. Los an\u00e1lisis muestran \u00e1cido \u00farico 10,1 mg/dl, creatina 1,5 mg/dl. Indique c\u00faal de las siguinetes respuestas es FALSA en relaci\u00f3n con su posible tratamiento con alopurinol.", "full_answer": "El alopurinol es siempre el tratamiento de elecci\u00f3n y, sobre todo, si se trata de una gota tof\u00e1cea. Adem\u00e1s, en este caso, los uricos\u00faricos podr\u00edan empeorar la urolitiasis al aumentar la uricosuria (perd\u00f3n por la redundancia\u2026).", "type": "REUMATOLOG\u00cdA", "options": {"1": "Su empleo es de segunda elecci\u00f3n, cuando han fracasado los uricos\u00faricos.", "2": "Su empleo por tiempo prolongado es pr\u00e1cticamente obligado porque existen tofos.", "3": "Su introducci\u00f3n debe demorarse hasta que se haya resulto el ataque actual con antiinflamatorios o con colchina.", "4": "Su introducci\u00f3n debe ser gradual hasta una dosis que consiga una uricemia inferior a 6 mg/dL.", "5": "En los primeros meses de tratamiento se aconsejar asociar dosis bajas de colchicina para prevenir nuevos ataques."}, "correct_option": 1, "explanations": {"1": {"exist": true, "char_ranges": [[0, 99]], "word_ranges": [[0, 18]], "text": "El alopurinol es siempre el tratamiento de elecci\u00f3n y, sobre todo, si se trata de una gota tof\u00e1cea."}, "2": {"exist": false, "char_ranges": [], "word_ranges": [], "text": ""}, "3": {"exist": false, "char_ranges": [], "word_ranges": [], "text": ""}, "4": {"exist": false, "char_ranges": [], "word_ranges": [], "text": ""}, "5": {"exist": false, "char_ranges": [], "word_ranges": [], "text": ""}}} {"id": 373, "year": 2016, "question_id_specific": 86, "full_question": "Nos remiten desde oftalmolog\u00eda a un hombre de 31 a\u00f1os que consult\u00f3 por p\u00e9rdida de visi\u00f3n y cefalea progresiva. En la exploraci\u00f3n fisica se constata una hemianopsia temporal y en la resonancia magn\u00e9tica hipofisaria una turnoraci\u00f3n de 35 x 30 x 20 mm que comprime quiasma y seno cavernoso. Los resultados del estudio funcional son cortisol 14 microg/dL, TSH 1,4 microU/ml, T4L 1,2 ng/dL, prolactina 480 microg/L (vn < 15), testosterona 160 ng/dL (vn 300-1200), FSH 1,2 U/I., (vn 5-15) y LH 2 U/L (vn 3-15). \u00bfCu\u00e1l es el abordaje terap\u00e9utico inicial?:", "full_answer": "Por lo que nos describen se trata de un macroprolactinoma al tratarse de una tumoraci\u00f3n hipofisaria de m\u00e1s de un cent\u00edmetro, que segrega prolactina y con una insuficiencia del resto de hormonas hipofisarias. El tratamiento de elecci\u00f3n de primera l\u00ednea son los agonistas dopamin\u00e9rgicos, aunque cuando afecta al campo visual se puede optar por la opci\u00f3n quir\u00fargica. En este caso al comentarnos que el seno cavernoso se encuentra presionado por la masa por lo que el riesgo quir\u00fargico aumenta. Por ello, en este caso particular, la opci\u00f3n quir\u00fargica parece m\u00e1s arriesgada. Teniendo en cuenta esto y que es un tumor productor de altos niveles de prolactina , lo que nos indica que probablemente responda al bloqueo farmacol\u00f3gico, la opci\u00f3n m\u00e1s adecuada ser\u00eda la 1, el tratamiento con agonistas dopamin\u00e9rgicos.", "type": "NEUROLOG\u00cdA", "options": {"1": "Agonistas dopamin\u00e9rgicos.", "2": "Cirugia transesfenoidal.", "3": "Radioterapia externa o radiocirug\u00eda.", "4": "Tratamiento del hipogonadismo hipogonadotropo con testosterona.", "5": NaN}, "correct_option": 1, "explanations": {"1": {"exist": true, "char_ranges": [[0, 284]], "word_ranges": [[0, 44]], "text": "Por lo que nos describen se trata de un macroprolactinoma al tratarse de una tumoraci\u00f3n hipofisaria de m\u00e1s de un cent\u00edmetro, que segrega prolactina y con una insuficiencia del resto de hormonas hipofisarias. El tratamiento de elecci\u00f3n de primera l\u00ednea son los agonistas dopamin\u00e9rgicos,"}, "2": {"exist": true, "char_ranges": [[364, 569]], "word_ranges": [[57, 91]], "text": "En este caso al comentarnos que el seno cavernoso se encuentra presionado por la masa por lo que el riesgo quir\u00fargico aumenta. Por ello, en este caso particular, la opci\u00f3n quir\u00fargica parece m\u00e1s arriesgada."}, "3": {"exist": false, "char_ranges": [], "word_ranges": [], "text": ""}, "4": {"exist": false, "char_ranges": [], "word_ranges": [], "text": ""}, "5": {"exist": false, "char_ranges": [], "word_ranges": [], "text": ""}}} {"id": 523, "year": 2021, "question_id_specific": 124, "full_question": "Un paciente de 45 a\u00f1os fue diagnosticado de infecci\u00f3n VIH hace tres meses, presentando en ese momento CD4 45 linfocitos/\u00b5l y una carga viral de VIH-1 de 500.000 copias/ml. En ese momento presentaba un Mantoux negativo. Comenz\u00f3 tratamiento con un inhibidor de integrasa y dos inhibidores de transcriptasa inversa, y al mes presentaba 25.000 copias y los CD4 hab\u00edan subido a 80/\u00b5l. Consulta por presentar un cuadro de adenopat\u00edas cervicales y fiebre de dos semanas de evoluci\u00f3n. La punci\u00f3n de una de las adenopat\u00edas muestra bacilos \u00e1cido-alcohol resistentes aislados y granulomas epitelioides:", "full_answer": "Estamos ante un paciente con infecci\u00f3n por VIH en situaci\u00f3n de inmunosupresi\u00f3n grave (<50 CD4+) y una carga viral muy elevada, que ha seguido la evoluci\u00f3n esperable desde el punto de vista inmunovirol\u00f3gico tras el inicio de tratamiento (despu\u00e9s de 12 semanas, la carga viral ha disminuido ostensiblemente \u2013 la respuesta 2 no ser\u00eda correcta, recordemos que lo deseable es que negativice a las 24 semanas tras inicio de tratamiento, aunque en pacientes con CV mas elevadas como el nuestro, podr\u00eda demorarse mas- y ha iniciado el ascenso del recuento de CD4+). El Mantoux negativo no descarta la tuberculosis (respuesta 4 incorrecta), ya que en pacientes con inmunodepresi\u00f3n grave puede dar falsos negativos. El diagn\u00f3stico de tuberculosis ganglionar es inmediato al detectar \u201cbacilos \u00e1cido-alcohol resistentes aislados y granulomas epitelioides\u201d en la punci\u00f3n de una de las adenopat\u00edas, ahora bien: considerando el perfil de enfermo, dicho proceso habr\u00e1 sido desenmascarado cl\u00ednicamente durante la reconstituci\u00f3n inmunol\u00f3gica no siendo la infecci\u00f3n consecuencia de una incompleta recuperaci\u00f3n inmunol\u00f3gica (respuesta correcta la n\u00famero 3).", "type": "ENFERMEDADES INFECCIOSAS", "options": {"1": "Se trata de una tuberculosis ganglionar relacionada con una incompleta recuperaci\u00f3n inmunol\u00f3gica.", "2": "Presenta fracaso al tratamiento y se trata de una infecci\u00f3n oportunista.", "3": "Se trata de una infecci\u00f3n oportunista desenmascarada en el seno de un cuadro de reconstituci\u00f3n inmune.", "4": "El hecho de presentar una prueba de Mantoux negativa descarta la tuberculosis.", "5": NaN}, "correct_option": 3, "explanations": {"1": {"exist": true, "char_ranges": [[706, 1137]], "word_ranges": [[113, 169]], "text": "El diagn\u00f3stico de tuberculosis ganglionar es inmediato al detectar \u201cbacilos \u00e1cido-alcohol resistentes aislados y granulomas epitelioides\u201d en la punci\u00f3n de una de las adenopat\u00edas, ahora bien: considerando el perfil de enfermo, dicho proceso habr\u00e1 sido desenmascarado cl\u00ednicamente durante la reconstituci\u00f3n inmunol\u00f3gica no siendo la infecci\u00f3n consecuencia de una incompleta recuperaci\u00f3n inmunol\u00f3gica (respuesta correcta la n\u00famero 3)."}, "2": {"exist": true, "char_ranges": [[307, 557]], "word_ranges": [[49, 92]], "text": "la respuesta 2 no ser\u00eda correcta, recordemos que lo deseable es que negativice a las 24 semanas tras inicio de tratamiento, aunque en pacientes con CV mas elevadas como el nuestro, podr\u00eda demorarse mas- y ha iniciado el ascenso del recuento de CD4+)."}, "3": {"exist": true, "char_ranges": [[706, 1137]], "word_ranges": [[113, 169]], "text": "El diagn\u00f3stico de tuberculosis ganglionar es inmediato al detectar \u201cbacilos \u00e1cido-alcohol resistentes aislados y granulomas epitelioides\u201d en la punci\u00f3n de una de las adenopat\u00edas, ahora bien: considerando el perfil de enfermo, dicho proceso habr\u00e1 sido desenmascarado cl\u00ednicamente durante la reconstituci\u00f3n inmunol\u00f3gica no siendo la infecci\u00f3n consecuencia de una incompleta recuperaci\u00f3n inmunol\u00f3gica (respuesta correcta la n\u00famero 3)."}, "4": {"exist": true, "char_ranges": [[558, 705]], "word_ranges": [[92, 113]], "text": "El Mantoux negativo no descarta la tuberculosis (respuesta 4 incorrecta), ya que en pacientes con inmunodepresi\u00f3n grave puede dar falsos negativos."}, "5": {"exist": false, "char_ranges": [], "word_ranges": [], "text": ""}}} {"id": 417, "year": 2018, "question_id_specific": 76, "full_question": "Mujer de 78 a\u00f1os con demencia e institucionalizada es tra\u00edda por sus cuidadores por dolor abdominal importante con deterioro del estado general y distensi\u00f3n abdominal. Anal\u00edtica con leucocitosis, hematocrito elevado, insuficiencia renal y acidosis metab\u00f3lica. El ECG muestra fibrilaci\u00f3n auricular. La tomografia abdominal detecta asas de intestino delgado edematizadas, con acumatosis Intestinal y portal. El diagnostico mas probable es:", "full_answer": "Caso t\u00edpico de isquemia mesent\u00e9rica aguda, el cual debemos sospechar ante pacientes ancianos con dolor abdominal y distensi\u00f3n de instauraci\u00f3n brusca, diarrea o v\u00f3mitos y que presentan factores de riesgo cardiovascular, sobretodo cardiopat\u00edas embol\u00edgenas. En este caso nos otorgan datos caracter\u00edsticos tanto bioqu\u00edmicos (leucocitosis, acidosis metab\u00f3lica a expensas de \u00e1cido l\u00e1ctico, hemoconcentraci\u00f3n por p\u00e9rdida de l\u00edquido al tercer espacio\u2026) como en las pruebas de imagen (dilataci\u00f3n de asas, neumatosis, gas a nivel portal..) sugestivos de isquemia mesent\u00e9rica.", "type": "CIRUG\u00cdA GENERAL", "options": {"1": "Perforaci\u00f3n de ulcus gastrico o duodenal.", "2": "lleo biliar.", "3": "Neoplasia obstructiva de sigma con perforaci\u00f3n.", "4": "Isquemia mesent\u00e9rica.", "5": NaN}, "correct_option": 4, "explanations": {"1": {"exist": false, "char_ranges": [], "word_ranges": [], "text": ""}, "2": {"exist": false, "char_ranges": [], "word_ranges": [], "text": ""}, "3": {"exist": false, "char_ranges": [], "word_ranges": [], "text": ""}, "4": {"exist": true, "char_ranges": [[0, 254]], "word_ranges": [[0, 34]], "text": "Caso t\u00edpico de isquemia mesent\u00e9rica aguda, el cual debemos sospechar ante pacientes ancianos con dolor abdominal y distensi\u00f3n de instauraci\u00f3n brusca, diarrea o v\u00f3mitos y que presentan factores de riesgo cardiovascular, sobretodo cardiopat\u00edas embol\u00edgenas."}, "5": {"exist": false, "char_ranges": [], "word_ranges": [], "text": ""}}} {"id": 403, "year": 2016, "question_id_specific": 135, "full_question": "Un paciente de 25 a\u00f1os, durante un partido de tenis, tiene dolor intenso en el cuello y en el ojo izquierdo. La ma\u00f1ana siguiente se despierta con sensaci\u00f3n de inestabilidad de la marcha y tene ptosis palpebral del ojo izquierdo y anisocoria, siendo la pupila izquierda m\u00e1s peque\u00f1a que la derecha. El paciente mantiene buena agudeza visual. \u00bfD\u00f3nde localizar\u00eda con m\u00e1s probabilidad la lesi\u00f3n?", "full_answer": "El s\u00edndrome de Claude-Bernard-Horner es un viejo conocido del examen MIR. No es que lo veamos a diario en la pr\u00e1ctica cl\u00ednica, pero sale en muchos ex\u00e1menes. Lo caracter\u00edstico es la ptosis del p\u00e1rpado superior y la miosis de ese ojo. Esto ocurre por un d\u00e9ficit de la inervaci\u00f3n simp\u00e1tica del terrritorio orbitario. El sistema nervioso simp\u00e1tico inerva el m\u00fasculo de M\u00fcller, que es un m\u00fasculo accesorio del elevador del p\u00e1rpado superior. Si el M\u00fcller deja de funcionar, el p\u00e1rpado desciende un poco. Tambi\u00e9n inerva el m\u00fasculo dilatador del iris. Por lo tanto, el m\u00fasculo esf\u00ednter, su antagonista, no encuentra oposici\u00f3n y el equilibrio de la pupila se desplaza hacia la miosis. Nos explica ptosis y miosis del ojo izquierdo, por lo que sospechamos un Horner en ese ojo. Adem\u00e1s nos describen lo que puede haber pasado: tras un posible movimiento brusco jugando al tenis, tiene dolor en el cuello y un Horner. Deber\u00edamos sospechar un aneurisma disecante de la car\u00f3tida, que es una de las causas m\u00e1s conocidas (y m\u00e1s graves) de un s\u00edndrome de Horner adquirido. El ganglio cervical superior es un ganglio del sistema nervioso simp\u00e1tico que se encuentra en la vaina de la arteria car\u00f3tida, y a este nivel se ha afectado. Tambi\u00e9n podemos intentar acertar esta respuesta por descarte. Una alteraci\u00f3n del III par craneal puede producir ptosis, pero la pupila, de afectarse, deber\u00eda estar en midriasis. Y en un da\u00f1o en el III par tambi\u00e9n deber\u00eda producirse alg\u00fan tipo de oftalmoplejia, con diplopia. Una afectaci\u00f3n del quiasma \u00f3ptico producir\u00eda una p\u00e9rdida de visi\u00f3n bilateral (probablemente una hemianopsia bitemporal). Nos dicen en el enunciado que mantiene buena agudeza visual. Por otra parte, tampoco habr\u00eda ptosis ni anisocoria. Y por \u00faltimo, una alteraci\u00f3n del bulbo raqu\u00eddeo producir\u00eda s\u00edntomas neurol\u00f3gicos diferentes no descritos, y no deber\u00eda producirse ptosis ni alteraciones pupilares, porque el III par se encuentra en el mesenc\u00e9falo y los n\u00facleos simp\u00e1ticos en la m\u00e9dula espinal.", "type": "OFTALMOLOG\u00cdA (ECT\u00d3PICO)", "options": {"1": "III par craneal.", "2": "Quiasma \u00f3ptico.", "3": "Ganglio cervical superior.", "4": "Bulbo raqu\u00eddeo.", "5": NaN}, "correct_option": 3, "explanations": {"1": {"exist": true, "char_ranges": [[1276, 1391]], "word_ranges": [[214, 232]], "text": "Una alteraci\u00f3n del III par craneal puede producir ptosis, pero la pupila, de afectarse, deber\u00eda estar en midriasis."}, "2": {"exist": true, "char_ranges": [[1489, 1670]], "word_ranges": [[249, 274]], "text": "Una afectaci\u00f3n del quiasma \u00f3ptico producir\u00eda una p\u00e9rdida de visi\u00f3n bilateral (probablemente una hemianopsia bitemporal). Nos dicen en el enunciado que mantiene buena agudeza visual."}, "3": {"exist": true, "char_ranges": [[906, 1213]], "word_ranges": [[153, 206]], "text": "Deber\u00edamos sospechar un aneurisma disecante de la car\u00f3tida, que es una de las causas m\u00e1s conocidas (y m\u00e1s graves) de un s\u00edndrome de Horner adquirido. El ganglio cervical superior es un ganglio del sistema nervioso simp\u00e1tico que se encuentra en la vaina de la arteria car\u00f3tida, y a este nivel se ha afectado."}, "4": {"exist": true, "char_ranges": [[1738, 1983]], "word_ranges": [[285, 321]], "text": "una alteraci\u00f3n del bulbo raqu\u00eddeo producir\u00eda s\u00edntomas neurol\u00f3gicos diferentes no descritos, y no deber\u00eda producirse ptosis ni alteraciones pupilares, porque el III par se encuentra en el mesenc\u00e9falo y los n\u00facleos simp\u00e1ticos en la m\u00e9dula espinal."}, "5": {"exist": false, "char_ranges": [], "word_ranges": [], "text": ""}}} {"id": 42, "year": 2011, "question_id_specific": 150, "full_question": "Ni\u00f1o de a\u00f1o y medio que acude a la Urgencia por presentar un cuadro de dolor abdominal e ictericia. En la exploraci\u00f3n se palpa una masa abdominal. Al realizar una ecograf\u00eda se observa la presencia de un quiste de col\u00e9doco. \u00bfQu\u00e9 actitud terap\u00e9utica adoptaremos?", "full_answer": "La respuesta correcta es la 4. El quiste de col\u00e9doco es una patolog\u00eda cong\u00e9nita que suele dar cl\u00ednica m\u00e1s all\u00e1 del periodo neonatal. Lo adecuado es tener una imagen de calidad que delimite la lesi\u00f3n para decidir el abordaje quir\u00fargico.", "type": "PEDIATR\u00cdA", "options": {"1": "Se realizar\u00e1 una punci\u00f3n percut\u00e1nea para lavado peritoneal con objeto de comprobar que el quiste contiene bilis para dejar un drenaje que mejore el cuadro de dolor abdominal.", "2": "Se indicar\u00e1 una laparotom\u00eda exploradora y se proceder\u00e1 a la resecci\u00f3n radical de toda la v\u00eda biliar para sustituirla por un asa de intestino.", "3": "Se indicar\u00e1 una laparotom\u00eda exloradora para drenar el quiste y cuando remita la dilataci\u00f3n de retirar\u00e1 el drenaje.", "4": "Se realizar\u00e1 una colangio-resonancia para delimitar el quiste y se indicar\u00e1 una laparotom\u00eda para la resecci\u00f3n de quiste y anastomosis de la v\u00eda biliar.", "5": "Es necesario un estudio gammagr\u00e1fico HIDA, para delimitar el quiste y poder realizar el drenaje percut\u00e1neo con seguridad."}, "correct_option": 4, "explanations": {"1": {"exist": false, "char_ranges": [], "word_ranges": [], "text": ""}, "2": {"exist": false, "char_ranges": [], "word_ranges": [], "text": ""}, "3": {"exist": false, "char_ranges": [], "word_ranges": [], "text": ""}, "4": {"exist": true, "char_ranges": [[31, 235]], "word_ranges": [[6, 40]], "text": "El quiste de col\u00e9doco es una patolog\u00eda cong\u00e9nita que suele dar cl\u00ednica m\u00e1s all\u00e1 del periodo neonatal. Lo adecuado es tener una imagen de calidad que delimite la lesi\u00f3n para decidir el abordaje quir\u00fargico."}, "5": {"exist": false, "char_ranges": [], "word_ranges": [], "text": ""}}} {"id": 445, "year": 2018, "question_id_specific": 233, "full_question": "Mujer de 45 a\u00f1os. Dos gestaciones previas con partos normales (G2PN2). Sangrado menstrual abundante desde hace aproximadamente 1 a\u00f1o. En ecografia se observa un mioma subseroso de 2 cm que ha permanecido estable desde hace varios a\u00f1os. Se realiza biopsia endometrial que resulta normal. La paciente presenta hemoglobina 10 g/dL a pesar de ferroterapia oral y la cantidad de flujo menstrual no ha disminuido a pesar de tratamiento con \u00e1cido tranexamico y acido mefenamico. \u00bfCual de los siguientes es el tratamiento de primera elecci\u00f3n en esta paciente?", "full_answer": "Seg\u00fan la SEGO, en mujeres perimenop\u00e1usicas con sangrado menstrual abundante y/o prolongado que precisen anticoncepci\u00f3n, el DIU de levonorgestrel (tambi\u00e9n llamado DIU MIRENA) es la primera opci\u00f3n.", "type": "GINECOLOG\u00cdA Y OBSTETRICIA", "options": {"1": "Histerectomia total conservando anejos.", "2": "Anticonceptivos combinados orales.", "3": "Ablaci\u00f3n endometr\u00edal.", "4": "DIU de levonorgestrel.", "5": NaN}, "correct_option": 4, "explanations": {"1": {"exist": false, "char_ranges": [], "word_ranges": [], "text": ""}, "2": {"exist": false, "char_ranges": [], "word_ranges": [], "text": ""}, "3": {"exist": false, "char_ranges": [], "word_ranges": [], "text": ""}, "4": {"exist": true, "char_ranges": [[0, 195]], "word_ranges": [[0, 27]], "text": "Seg\u00fan la SEGO, en mujeres perimenop\u00e1usicas con sangrado menstrual abundante y/o prolongado que precisen anticoncepci\u00f3n, el DIU de levonorgestrel (tambi\u00e9n llamado DIU MIRENA) es la primera opci\u00f3n."}, "5": {"exist": false, "char_ranges": [], "word_ranges": [], "text": ""}}} {"id": 511, "year": 2021, "question_id_specific": 152, "full_question": "Mujer de 79 a\u00f1os hipertensa en tratamiento con olmesart\u00e1n que acude a consulta por diarrea acuosa de 4 a 6 deposiciones al d\u00eda desde hace dos meses. Hace tres meses recibi\u00f3 tratamiento con antiinflamatorios no esteroideos durante 3 semanas por lumboci\u00e1tica. Se realiz\u00f3 gastroscopia y colonoscopia, ambas sin alteraciones macrosc\u00f3picas. Las biopsias duodenales fueron normales, mientras que en las biopsias de colon se observa un infiltrado inflamatorio cr\u00f3nico de la l\u00e1mina propia con una banda irregular de col\u00e1geno inmediatamente por debajo del epitelio de superficie de la mucosa con un grosor mayor a 10 mm y un n\u00famero de linfocitos intraepiteliales >20 por cada 100 c\u00e9lulas epiteliales. \u00bfCu\u00e1l es el diagn\u00f3stico m\u00e1s probable?:", "full_answer": "Los criterios diagn\u00f3sticos de la colitis microsc\u00f3pica son: a)Diarrea acuosa sin sangre, cr\u00f3nica o intermitenteb)Mucosa de colon valorada por colonoscopia macrosc\u00f3picamente normal o casi normal c)Hallazgos histopatol\u00f3gicos caracter\u00edsticos. En nuestro caso, la paciente presenta hallazgos compatibles con colitis col\u00e1gena (banda de col\u00e1geno subyacente al epitelio mayor de 10 micras) y linfoc\u00edtica (m\u00e1s de 20 linfocitos intraepiteliales). El tratamiento de esta patolog\u00eda son corticoides t\u00f3picos con baja biodisponibilidad oral (budesonida principalmente). La enteropat\u00eda por olmesart\u00e1n produce un cuadro similar a la celiaqu\u00eda y se caracteriza por biopsia duodenal con atrofia vellositaria, inflamaci\u00f3n mucosa con incremento de linfocitos intraepiteliales y criptitis. En este caso nos describen una biopsia duodenal normal, pero ser\u00eda interesante quedarnos con el concepto de cara a futuros ex\u00e1menes.", "type": "DIGESTIVO", "options": {"1": "Colitis microsc\u00f3pica.", "2": "Enteropat\u00eda por AINEs.", "3": "Enteropat\u00eda asociada a olmesart\u00e1n.", "4": "S\u00edndrome de intestino irritable.", "5": NaN}, "correct_option": 1, "explanations": {"1": {"exist": true, "char_ranges": [[256, 381]], "word_ranges": [[31, 49]], "text": "la paciente presenta hallazgos compatibles con colitis col\u00e1gena (banda de col\u00e1geno subyacente al epitelio mayor de 10 micras)"}, "2": {"exist": false, "char_ranges": [], "word_ranges": [], "text": ""}, "3": {"exist": true, "char_ranges": [[555, 767]], "word_ranges": [[70, 99]], "text": "La enteropat\u00eda por olmesart\u00e1n produce un cuadro similar a la celiaqu\u00eda y se caracteriza por biopsia duodenal con atrofia vellositaria, inflamaci\u00f3n mucosa con incremento de linfocitos intraepiteliales y criptitis."}, "4": {"exist": false, "char_ranges": [], "word_ranges": [], "text": ""}, "5": {"exist": false, "char_ranges": [], "word_ranges": [], "text": ""}}} {"id": 33, "year": 2011, "question_id_specific": 65, "full_question": "Un hombre de 64 a\u00f1os, diagnosticado de miastenia gravis hace 1 a\u00f1o, en tratamiento con esteroides a dosis bajas (3 mg/d\u00eda de deflazacort) y anticolinester\u00e1sicos, asintom\u00e1tico desde hace 6 meses, consulta por leve dificultad para tragar y diplopia vespertina desde hace unos d\u00edas. Ingresa en el hospital por sospecha de crisis miast\u00e9nica y se inicia tratamiento de la misma. Al d\u00eda siguiente la paciente avisa a las 03 horas de la madrugada porque el paciente hace un ruido extra\u00f1o al inspirar, como un ronquido suave, el paciente est\u00e1 profundamente dormido y muy sudoroso, pero no impresiona de estar fatigado. \u00bfQu\u00e9 actitud es m\u00e1s correcta?", "full_answer": "Se trata de un paciente con miastenia descompensada con s\u00edntomas bulbares (disfagia) y por lo tanto riesgo de afectaci\u00f3n de musculatura respiratoria. La cl\u00ednica respiratoria se corresponde a un agotamiento de los m\u00fasculos, primero inspiratorios y luego accesorios (el paciente no parece fatigado ni taquipneico porque los m\u00fasculos no dan m\u00e1s), progresando a un coma carb\u00f3nico.", "type": "NEUROLOG\u00cdA Y NEUROCIRUG\u00cdA", "options": {"1": "Tranquilizar a la enfermera y a la familia, pues el paciente es roncador habitual y est\u00e1 tranquilamente dormido. Se debe colocar en dec\u00fabito lateral.", "2": "Avisar a cuidados intensivos por sospecha de insuficiencia respiratoria aguda, para valorar posible intubaci\u00f3n orotraqueal y ventilaci\u00f3n asistida.", "3": "Hacer estudio polisomnogr\u00e1fico para descartar apnea del sue\u00f1o.", "4": "Disminuir la dosis de esteroides; si tiene una miopat\u00eda esteroidea mejorar\u00e1.", "5": "Pedir una TC tor\u00e1cica para descartar que timoma comprensivo sobre la tr\u00e1quea asociado a la miastenia."}, "correct_option": 2, "explanations": {"1": {"exist": false, "char_ranges": [], "word_ranges": [], "text": ""}, "2": {"exist": true, "char_ranges": [[0, 149]], "word_ranges": [[0, 22]], "text": "Se trata de un paciente con miastenia descompensada con s\u00edntomas bulbares (disfagia) y por lo tanto riesgo de afectaci\u00f3n de musculatura respiratoria."}, "3": {"exist": false, "char_ranges": [], "word_ranges": [], "text": ""}, "4": {"exist": false, "char_ranges": [], "word_ranges": [], "text": ""}, "5": {"exist": false, "char_ranges": [], "word_ranges": [], "text": ""}}} {"id": 129, "year": 2012, "question_id_specific": 81, "full_question": "Un var\u00f3n de 78 a\u00f1os consulta por un cuadro de deterioro cognitivo progresivo de un a\u00f1o de evoluci\u00f3n con fallos de memoria y orientaci\u00f3n. Su familia refiere alucinaciones visuales recurrentes, ca\u00eddas ocasionales y un enlentecimiento motor llamativo. \u00bfCu\u00e1l es el diagn\u00f3stico m\u00e1s probable?", "full_answer": "Una demencia que presenta al inicio de la evoluci\u00f3n alucinaciones visuales (en los 2 primeros a\u00f1os) es muy sugerente de cuerpos de Lewy, y es el s\u00edntoma gu\u00eda en estas preguntas. Si a ello sumamos las ca\u00eddas y el enlentecimiento motor sugerente de parkinsonismo, hace m\u00e1s f\u00e1cil la pregunta. Por lo tanto, correcta la n\u00famero 3.", "type": "NEUROLOG\u00cdA Y NEUROCIRUG\u00cdA", "options": {"1": "Demencia vascular multi-infarto.", "2": "Demencia tipo Alzheimer.", "3": "Demencia con cuerpos de Lewy difusos.", "4": "Demencia fronto-temporal.", "5": "Hidrocefalia normotensiva."}, "correct_option": 3, "explanations": {"1": {"exist": false, "char_ranges": [], "word_ranges": [], "text": ""}, "2": {"exist": false, "char_ranges": [], "word_ranges": [], "text": ""}, "3": {"exist": true, "char_ranges": [[0, 325]], "word_ranges": [[0, 56]], "text": "Una demencia que presenta al inicio de la evoluci\u00f3n alucinaciones visuales (en los 2 primeros a\u00f1os) es muy sugerente de cuerpos de Lewy, y es el s\u00edntoma gu\u00eda en estas preguntas. Si a ello sumamos las ca\u00eddas y el enlentecimiento motor sugerente de parkinsonismo, hace m\u00e1s f\u00e1cil la pregunta. Por lo tanto, correcta la n\u00famero 3."}, "4": {"exist": false, "char_ranges": [], "word_ranges": [], "text": ""}, "5": {"exist": false, "char_ranges": [], "word_ranges": [], "text": ""}}} {"id": 503, "year": 2020, "question_id_specific": 80, "full_question": "Ni\u00f1o de 8 a\u00f1os que consulta por la aparici\u00f3n de vello p\u00fabico en la base del pene desde hace 12 meses. El test\u00edculo no ha aumentado de volumen y el pene no ha aumentado de tama\u00f1o. No se aprecia un aumento de la velocidad de crecimiento en el \u00faltimo a\u00f1o. La edad \u00f3sea es un a\u00f1o mayor que la cronol\u00f3gica. \u00bfCu\u00e1l ser\u00eda el diagn\u00f3stico m\u00e1s probable?:", "full_answer": "En el var\u00f3n, se considera normal la aparici\u00f3n de pubarquia a partir de los 9 a\u00f1os de edad, por tanto en este caso es una pubarquia prematura. El comienzo de pubertad se define como aumento de volumen testicular (mayor o igual a 4cc); al especificarnos que el tama\u00f1o testicular no ha aumentado, podemos descartar la opci\u00f3n 1 y 2. La secreci\u00f3n excesiva de andr\u00f3genos que produce un tumor suprarrenal originar\u00eda mayor cantidad de vello y aumento de tama\u00f1o peneano, que nuestro paciente no presenta. Por tanto, la opci\u00f3n correcta es adrenarquia precoz.", "type": "PEDIATR\u00cdA", "options": {"1": "Pubertad precoz central.", "2": "Pubertad precoz perif\u00e9rica.", "3": "Adrenarquia precoz.", "4": "Tumor suprarrenal.", "5": NaN}, "correct_option": 3, "explanations": {"1": {"exist": true, "char_ranges": [[329, 548]], "word_ranges": [[59, 92]], "text": "La secreci\u00f3n excesiva de andr\u00f3genos que produce un tumor suprarrenal originar\u00eda mayor cantidad de vello y aumento de tama\u00f1o peneano, que nuestro paciente no presenta. Por tanto, la opci\u00f3n correcta es adrenarquia precoz."}, "2": {"exist": true, "char_ranges": [[329, 548]], "word_ranges": [[59, 92]], "text": "La secreci\u00f3n excesiva de andr\u00f3genos que produce un tumor suprarrenal originar\u00eda mayor cantidad de vello y aumento de tama\u00f1o peneano, que nuestro paciente no presenta. Por tanto, la opci\u00f3n correcta es adrenarquia precoz."}, "3": {"exist": true, "char_ranges": [[329, 548]], "word_ranges": [[59, 92]], "text": "La secreci\u00f3n excesiva de andr\u00f3genos que produce un tumor suprarrenal originar\u00eda mayor cantidad de vello y aumento de tama\u00f1o peneano, que nuestro paciente no presenta. Por tanto, la opci\u00f3n correcta es adrenarquia precoz."}, "4": {"exist": false, "char_ranges": [], "word_ranges": [], "text": ""}, "5": {"exist": false, "char_ranges": [], "word_ranges": [], "text": ""}}} {"id": 300, "year": 2016, "question_id_specific": 106, "full_question": "Un hombre de 32 a\u00f1os natural de Camer\u00fan consulta por fiebre tos y dolor en hemit\u00f3rax izquierdo de 1 mes de evoluci\u00f3n. Tom\u00f3 durante 1 semana amoxicilina-clavul\u00e1nico sin mejor\u00eda de los s\u00edntomas. Se le realiza un an\u00e1lisis donde destaca una cifra de leucocitos de 8000/microL y una hemoglobina de 12,8 g/dL. En la radiograf\u00eda de t\u00f3rax se observa un derrame pleural izquierdo loculado que ocupa un tercio del hemit\u00f3rax. Una toracentesis muestra un l\u00edquido amarillento con las siguientes caracter\u00edsticas: hemat\u00edes 2000/uL, leucocitos 2500/ uL, con 90% de linfocitos, prote\u00ednas 4,9 g/dL, lactato deshidrogenasa 550 U/L, glucosa 67 mg/dL y ausencia de c\u00e9lulas malignas en el estudio citol\u00f3gico. \u00bfCu\u00e1l de las siguientes pruebas ser\u00eda m\u00e1s \u00fatil para diagnosticar la causa del derrame pleural?", "full_answer": "La determinaci\u00f3n de ADA, ante un cuadro de neumon\u00eda de larga evoluci\u00f3n que no mejora con tratamiento antibi\u00f3tico, con un derrame con abundantes leucocitos y predominio linfoc\u00edtico, en que se descarta la presencia de neoplasia, con valores mayor de 70 UI/L nos tiene que hacer tener un alto grado de sospecha sobre la etiolog\u00eda tuberculosa y enviar el l\u00edquido pleural a microbiolog\u00eda para estudio de tuberculosis.", "type": "ENFERMEDADES INFECCIOSAS", "options": {"1": "Una tomograf\u00eda computarizada (TC) tor\u00e1cica.", "2": "Medici\u00f3n del pH del l\u00edquido pleural.", "3": "Medici\u00f3n de la adenosina desaminasa del l\u00edquido pleural.", "4": "Prueba de la tuberculina.", "5": NaN}, "correct_option": 3, "explanations": {"1": {"exist": false, "char_ranges": [], "word_ranges": [], "text": ""}, "2": {"exist": false, "char_ranges": [], "word_ranges": [], "text": ""}, "3": {"exist": true, "char_ranges": [[0, 412]], "word_ranges": [[0, 66]], "text": "La determinaci\u00f3n de ADA, ante un cuadro de neumon\u00eda de larga evoluci\u00f3n que no mejora con tratamiento antibi\u00f3tico, con un derrame con abundantes leucocitos y predominio linfoc\u00edtico, en que se descarta la presencia de neoplasia, con valores mayor de 70 UI/L nos tiene que hacer tener un alto grado de sospecha sobre la etiolog\u00eda tuberculosa y enviar el l\u00edquido pleural a microbiolog\u00eda para estudio de tuberculosis."}, "4": {"exist": false, "char_ranges": [], "word_ranges": [], "text": ""}, "5": {"exist": false, "char_ranges": [], "word_ranges": [], "text": ""}}} {"id": 434, "year": 2018, "question_id_specific": 119, "full_question": "Mujer de 38 a\u00f1os de edad de profesi\u00f3n veterinaria, encargada de la vigilancia de animales salvajes y de ayudar a partos de ganado dom\u00e9stico. Comienza con un cuadro de fiebre alta con escalofr\u00edos, cefalea, mialgias y tos no productiva que interpreta como un proceso gripal. Acude por presentar dolor tor\u00e1cico. En la radiograf\u00eda de t\u00f3rax se objetivan infiltrados pulmonares bilaterales en campos inferiores. Se realiza una prueba serol\u00f3gica con elevaci\u00f3n de t\u00edtulos de anticuerpos frente a ant\u00edgenos en fase Il. \u00bfCu\u00e1I de las siguientes afirmaciones es CIERTA?", "full_answer": "Infecci\u00f3n aguda por Coxiella burnetii que se manifiesta en este caso por Neumon\u00eda. La infecci\u00f3n por Coxiella no se transmite por garrapata. El contacto puede ser con alta probabilidad en el momento del parto de alg\u00fan animal de granja infectado, en el que se disemina la bacteria en aerosoles. El tratamiento de elecci\u00f3n es con doxiciclina en la fase aguda (si hay meningitis se prefieren quinolonas por la penetraci\u00f3n de la barrera hematoencef\u00e1lica). Se puede usar la combinaci\u00f3n de doxiciclina m\u00e1s hicroxicloroquina, pero para la fiebre Q cr\u00f3nica (que es donde se observa elevaci\u00f3n de anticuerpos contra los ant\u00edgenos en fase I y conlleva mayor riesgo de mortalidad).", "type": "ENFERMEDADES INFECCIOSAS Y MICROBIOLOG\u00cdA", "options": {"1": "La forma de trasmisi\u00f3n de esta entidad es por garrapatas.", "2": "Tanto la doxiciclina como la hidroxicloroquina son eficaces para tratar las formas agudas de esta enfermedad.", "3": "En su forma aguda tambi\u00e9n presenta, generalmente, elevaci\u00f3n de anticuerpos frente a ant\u00edgenos en fase l.", "4": "La mortalidad en las formas agudas es casi inexistente.", "5": NaN}, "correct_option": 4, "explanations": {"1": {"exist": true, "char_ranges": [[83, 139]], "word_ranges": [[13, 22]], "text": "La infecci\u00f3n por Coxiella no se transmite por garrapata."}, "2": {"exist": true, "char_ranges": [[451, 668]], "word_ranges": [[73, 108]], "text": "Se puede usar la combinaci\u00f3n de doxiciclina m\u00e1s hicroxicloroquina, pero para la fiebre Q cr\u00f3nica (que es donde se observa elevaci\u00f3n de anticuerpos contra los ant\u00edgenos en fase I y conlleva mayor riesgo de mortalidad)."}, "3": {"exist": false, "char_ranges": [], "word_ranges": [], "text": ""}, "4": {"exist": false, "char_ranges": [], "word_ranges": [], "text": ""}, "5": {"exist": false, "char_ranges": [], "word_ranges": [], "text": ""}}} {"id": 563, "year": 2022, "question_id_specific": 175, "full_question": "Paciente de 62 a\u00f1os que consulta por haber estado en contacto cercano en las \u00faltimas semanas con una persona con una tuberculosis activa. El Mantoux es positivo (12 mm). Entre sus antecedentes destaca estar en tratamiento con anticoagulantes orales (acenocumarol) por una trombosis venosa profunda hace unos meses. No refiere fiebre ni tos y se encuentra asintom\u00e1tico. La Rx de t\u00f3rax es normal. \u00bfCu\u00e1l de los siguientes tratamientos es el m\u00e1s recomendable?:", "full_answer": "A partir del 2018, hay cuatro esquemas posol\u00f3gicos para el tratamiento de la infecci\u00f3n de tuberculosis latente recomendados por los CDC que usan isoniacida (INH), rifapentina (RPT) o rifampina (RIF). Todos los tratamientos son eficaces. Los proveedores de atenci\u00f3n m\u00e9dica deber\u00edan recetar los tratamientos m\u00e1s cortos, que son m\u00e1s convenientes, siempre que sea posible. La combinaci\u00f3n m\u00e1s corta ser\u00eda la de Isoniacida + Rifapentina, durante tres meses o la Rifampicina durante cuatro meses. Sin embargo, el hecho de tomar acenocumarol hace que la indicaci\u00f3n para este paciente sea la Isoniacida durante 9 meses, ya que hay interacci\u00f3n entre los derivados de la rifamicina (rifapentina y rifampicina).", "type": "MEDICINA PREVENTIVA", "options": {"1": "Isoniacida, durante nueve meses.", "2": "Rifampicina, durante cuatro meses.", "3": "Isoniacida y rifampicina, durante tres meses.", "4": "Isoniacida y rifapentina, durante tres meses.", "5": NaN}, "correct_option": 4, "explanations": {"1": {"exist": false, "char_ranges": [], "word_ranges": [], "text": ""}, "2": {"exist": false, "char_ranges": [], "word_ranges": [], "text": ""}, "3": {"exist": false, "char_ranges": [], "word_ranges": [], "text": ""}, "4": {"exist": false, "char_ranges": [], "word_ranges": [], "text": ""}, "5": {"exist": false, "char_ranges": [], "word_ranges": [], "text": ""}}} {"id": 89, "year": 2012, "question_id_specific": 57, "full_question": "Hombre de 64 a\u00f1os que consulta por dolor en la pantorrilla derecha cuando sube cuestas y escaleras y cuando anda en llano durante un tiempo prolongado. Refiere que el dolor se hace tan intenso con el esfuerzo que le obliga a pararse, lo que hace que el dolor mejore en cuesti\u00f3n de minutos. Como antecedentes personales, destaca que el paciente es fumador de 20 cigarrillos al d\u00eda, diab\u00e9tico en tratamiento con metformina e hipertenso. A la exploraci\u00f3n, se objetiva una buena perfusi\u00f3n perif\u00e9rica del pie derecho, pero presenta un pulso pedio disminuido. \u00bfQu\u00e9 prueba complementaria, entre las siguientes, debe solicitar inicialmente para el diagn\u00f3stico y valoraci\u00f3n de la gravedad de la enfermedad de este paciente?", "full_answer": "Aunque el diagn\u00f3stico de las arteriopat\u00edas perif\u00e9ricas es fundamentalmente cl\u00ednico (en este paciente el cuadro es muy t\u00edpico), podemos valorar de forma objetiva la gravedad de la enfermedad con el uso de t\u00e9cnicas incruentas. La m\u00e1s importante es el \u00edndice tobillo-brazo. Por tanto, respuesta 2 correcta. Las t\u00e9cnicas de imagen (arteriograf\u00eda, angioTC o angioRM) se utilizan cuando se considera la reparaci\u00f3n quir\u00fargica de la oclusi\u00f3n arterial.", "type": "CARDIOLOG\u00cdA Y CIRUG\u00cdA VASCULAR", "options": {"1": "Arteriograf\u00eda de extremidades inferiores.", "2": "\u00cdndice tobillo-brazo.", "3": "Tomograf\u00eda axial computerizada con reconstrucci\u00f3n vascular de extremidades inferiores.", "4": "Prueba de esfuerzo en cinta.", "5": "Resonancia magn\u00e9tica."}, "correct_option": 2, "explanations": {"1": {"exist": true, "char_ranges": [[304, 443]], "word_ranges": [[46, 66]], "text": "Las t\u00e9cnicas de imagen (arteriograf\u00eda, angioTC o angioRM) se utilizan cuando se considera la reparaci\u00f3n quir\u00fargica de la oclusi\u00f3n arterial."}, "2": {"exist": true, "char_ranges": [[127, 270]], "word_ranges": [[18, 41]], "text": "podemos valorar de forma objetiva la gravedad de la enfermedad con el uso de t\u00e9cnicas incruentas. La m\u00e1s importante es el \u00edndice tobillo-brazo."}, "3": {"exist": false, "char_ranges": [], "word_ranges": [], "text": ""}, "4": {"exist": false, "char_ranges": [], "word_ranges": [], "text": ""}, "5": {"exist": false, "char_ranges": [], "word_ranges": [], "text": ""}}} {"id": 586, "year": 2022, "question_id_specific": 75, "full_question": "Mujer de 30 a\u00f1os que solicita un m\u00e9todo anticonceptivo. Como antecedentes tiene una ces\u00e1rea urgente hace 7 meses, naciendo una ni\u00f1a de 3.550 g. Est\u00e1 dando lactancia materna exclusiva. Se\u00f1ale la afirmaci\u00f3n correcta:", "full_answer": "Dadas sus circunstancias y valorando ante todo que est\u00e1 en lactancia materna exclusiva, lo mejor ser\u00eda pautarle s\u00f3lo gest\u00e1genos.", "type": "OBSTETRICIA Y GINECOLOG\u00cdA", "options": {"1": "Le explica que, si est\u00e1 en amenorrea, no requiere de otro m\u00e9todo anticonceptivo puesto que la lactancia materna es suficiente como m\u00e9todo anticonceptivo.", "2": "Le informa que el DIU de levonogestrel est\u00e1 contraindicado por haber tenido una ces\u00e1rea hace menos de un a\u00f1o.", "3": "Le explica que podr\u00eda usar un m\u00e9todo hormonal de s\u00f3lo gest\u00e1genos.", "4": "Le ofrece anticoncepci\u00f3n hormonal combinada.", "5": NaN}, "correct_option": 3, "explanations": {"1": {"exist": false, "char_ranges": [], "word_ranges": [], "text": ""}, "2": {"exist": false, "char_ranges": [], "word_ranges": [], "text": ""}, "3": {"exist": true, "char_ranges": [[0, 128]], "word_ranges": [[0, 19]], "text": "Dadas sus circunstancias y valorando ante todo que est\u00e1 en lactancia materna exclusiva, lo mejor ser\u00eda pautarle s\u00f3lo gest\u00e1genos."}, "4": {"exist": false, "char_ranges": [], "word_ranges": [], "text": ""}, "5": {"exist": false, "char_ranges": [], "word_ranges": [], "text": ""}}} {"id": 461, "year": 2018, "question_id_specific": 148, "full_question": "Mujer de 59 a\u00f1os con crisis epil\u00e9pticas de reciente instauraci\u00f3n que acude a urgencias. En la exploraci\u00f3n presenta signos piramidales izquierdos y edema de papila. Se le realiza una RM cerebral que muestra una masa hemisf\u00e9rica derecha con edema, desviaci\u00f3n de la l\u00ednea media y signos de herniaci\u00f3n tentorial. refiere que desde hace una semana subre dolor de cabeza que ha ido progresando en intensidad. \u00bfCu\u00e1l de las siguientes caracter\u00edsticas asociadas a la cefalea le parece m\u00e1s probable a esta paciente?", "full_answer": "Se trata de una cefalea en el contexto de un papiledema. En este caso el papiledema es debido a un aumento de presi\u00f3n intracraneal por un tumor. La cefalea por incremento de presi\u00f3n intracraneal es de tipo posicional (aumenta con las maniobras de Valsalva y al agacharse), as\u00ed que la opci\u00f3n 2 no es correcta. Adem\u00e1s es de predominio matutino (opci\u00f3n 1 correcta) porque por las ma\u00f1anas de forma natural la presi\u00f3n intracraneal sube. Las cefaleas de tipo migra\u00f1a pueden asociar fotofobia o sonofibia (respuestas 3 y 4). Pero esta cefalea no es de tipo migra\u00f1oso.", "type": "OFTALMOLOG\u00cdA (ECT\u00d3PICO)", "options": {"1": "Predominio matutino.", "2": "No cambia con esfuerzos.", "3": "Fotofobia.", "4": "Sonofobia.", "5": NaN}, "correct_option": 1, "explanations": {"1": {"exist": true, "char_ranges": [[309, 431]], "word_ranges": [[55, 74]], "text": "Adem\u00e1s es de predominio matutino (opci\u00f3n 1 correcta) porque por las ma\u00f1anas de forma natural la presi\u00f3n intracraneal sube."}, "2": {"exist": true, "char_ranges": [[145, 308]], "word_ranges": [[27, 55]], "text": "La cefalea por incremento de presi\u00f3n intracraneal es de tipo posicional (aumenta con las maniobras de Valsalva y al agacharse), as\u00ed que la opci\u00f3n 2 no es correcta."}, "3": {"exist": true, "char_ranges": [[432, 560]], "word_ranges": [[74, 96]], "text": "Las cefaleas de tipo migra\u00f1a pueden asociar fotofobia o sonofibia (respuestas 3 y 4). Pero esta cefalea no es de tipo migra\u00f1oso."}, "4": {"exist": true, "char_ranges": [[432, 560]], "word_ranges": [[74, 96]], "text": "Las cefaleas de tipo migra\u00f1a pueden asociar fotofobia o sonofibia (respuestas 3 y 4). Pero esta cefalea no es de tipo migra\u00f1oso."}, "5": {"exist": false, "char_ranges": [], "word_ranges": [], "text": ""}}} {"id": 547, "year": 2022, "question_id_specific": 125, "full_question": "Var\u00f3n de 23 a\u00f1os diagnosticado de s\u00edndrome de Wolff-Parkinson-White que acude a urgencias por palpitaciones. El ECG muestra una taquicardia irregular de QRS ancho, con una frecuencia cardiaca de 205 lpm, compatible con fibrilaci\u00f3n auricular preexcitada. \u00bfCu\u00e1l de los siguientes f\u00e1rmacos considera adecuado administrar para detener la taquicardia?:", "full_answer": "Procainamida. El tratamiento de elecci\u00f3n para el s\u00edndrome de Wolff-Parkinson-White es la cardioversi\u00f3n con corriente continua. Los f\u00e1rmacos que reducen la frecuencia card\u00edaca, empleados en forma habitual en la fibrilaci\u00f3n auricular, no resultan eficaces, y la digoxina y los bloqueantes de los canales de calcio no dihidropiridinas (p. ej., verapamilo, diltiazem) est\u00e1n contraindicados porque podr\u00edan aumentar la frecuencia ventricular y causar fibrilaci\u00f3n ventricular. Si no es posible la cardioversi\u00f3n, se deben usar f\u00e1rmacos que prolongan el periodo refractario de la conexi\u00f3n de los accesorios. Se prefiere procainamida IV o amiodarona, pero cualquier f\u00e1rmaco antiarr\u00edtmico de clase Ia, Ic o III se puede utilizar.", "type": "CARDIOLOG\u00cdA", "options": {"1": "Verapamilo.", "2": "Digoxina.", "3": "Adenosina.", "4": "Procainamida.", "5": NaN}, "correct_option": 4, "explanations": {"1": {"exist": true, "char_ranges": [[257, 469]], "word_ranges": [[35, 63]], "text": "la digoxina y los bloqueantes de los canales de calcio no dihidropiridinas (p. ej., verapamilo, diltiazem) est\u00e1n contraindicados porque podr\u00edan aumentar la frecuencia ventricular y causar fibrilaci\u00f3n ventricular."}, "2": {"exist": true, "char_ranges": [[257, 469]], "word_ranges": [[35, 63]], "text": "la digoxina y los bloqueantes de los canales de calcio no dihidropiridinas (p. ej., verapamilo, diltiazem) est\u00e1n contraindicados porque podr\u00edan aumentar la frecuencia ventricular y causar fibrilaci\u00f3n ventricular."}, "3": {"exist": true, "char_ranges": [[127, 253]], "word_ranges": [[16, 34]], "text": "Los f\u00e1rmacos que reducen la frecuencia card\u00edaca, empleados en forma habitual en la fibrilaci\u00f3n auricular, no resultan eficaces,"}, "4": {"exist": true, "char_ranges": [[470, 639]], "word_ranges": [[63, 90]], "text": "Si no es posible la cardioversi\u00f3n, se deben usar f\u00e1rmacos que prolongan el periodo refractario de la conexi\u00f3n de los accesorios. Se prefiere procainamida IV o amiodarona,"}, "5": {"exist": false, "char_ranges": [], "word_ranges": [], "text": ""}}} {"id": 193, "year": 2013, "question_id_specific": 161, "full_question": "En un lactante de 3 meses todas las manifestaciones cl\u00ednicas rese\u00f1adas a continuaci\u00f3n obligar\u00edan a descartar una fibrosis qu\u00edstica, EXCEPTO:", "full_answer": "La deshidrataci\u00f3n puede deberse a diarreas de otras etiolog\u00edas. Me ha generado dudas la respuesta 3 ya que no especifica de qu\u00e9 secreciones de trata. Tenemos que sobreentender que el crecimiento por Pseudomona aeruginosa es en el cultivo de secreciones bronquiales. Porque claramente la indicaci\u00f3n para realizar un cloro en sudor (para descartar FQP) son las deshidrataciones HIPOnatr\u00e9micas y no las hipernatr\u00e9micas como dice la respuesta.", "type": "PEDIATR\u00cdA", "options": {"1": "Retraso en la evacuaci\u00f3n de meconio.", "2": "Sabor salado de la piel.", "3": "Aislamiento de secreciones de Pseudomonas aeruginosa.", "4": "Deterioro/detenci\u00f3n de la curva ponderal.", "5": "Deshidrataci\u00f3n hipernatr\u00e9mica."}, "correct_option": 5, "explanations": {"1": {"exist": false, "char_ranges": [], "word_ranges": [], "text": ""}, "2": {"exist": false, "char_ranges": [], "word_ranges": [], "text": ""}, "3": {"exist": false, "char_ranges": [], "word_ranges": [], "text": ""}, "4": {"exist": false, "char_ranges": [], "word_ranges": [], "text": ""}, "5": {"exist": true, "char_ranges": [[273, 439]], "word_ranges": [[42, 66]], "text": "claramente la indicaci\u00f3n para realizar un cloro en sudor (para descartar FQP) son las deshidrataciones HIPOnatr\u00e9micas y no las hipernatr\u00e9micas como dice la respuesta."}}} {"id": 431, "year": 2018, "question_id_specific": 116, "full_question": "Hombre de 38 a\u00f1os que acude a urgencias por cuadro de fiebre por encima de 39\u00baC y deterioro del nivel de conciencia tras haber realizado un viaje a Guinea Ecuatorial sin profilaxis antimal\u00e1rica. En la analitica destaca: creatinina de 3,4 mg/dL, AST 764 UlL, ALT 678 UlL. El laboratorio informa de la visualizaci\u00f3n en sangre de un Plasmodium falciparum con un nivel de parasitemia del 6%. \u00bfQu\u00e9 tratamiento instaurar\u00eda en este momento?", "full_answer": "En cuadro severo de infecci\u00f3n por Plasmodium el mayor riesgo de mortalidad se produce en las primeras 24 horas de la presentaci\u00f3n cl\u00ednica por lo que se debe instaurar tratamiento inmediatamente. El tratamiento con artesunato parenteral o intramuscular es m\u00e1s indicado en infecci\u00f3n severa por Plasmodium, tanto en adulto como en ni\u00f1o y en mujer embarazada.", "type": "ENFERMEDADES INFECCIOSAS Y MICROBIOLOG\u00cdA", "options": {"1": "Mefloquina por v\u00eda oral.", "2": "Artesunato \u00edntravenoso.", "3": "Doxicicl\u00edna por v\u00eda oral.", "4": "Sulfato de quinina por v\u00eda oral.", "5": NaN}, "correct_option": 2, "explanations": {"1": {"exist": false, "char_ranges": [], "word_ranges": [], "text": ""}, "2": {"exist": true, "char_ranges": [[0, 355]], "word_ranges": [[0, 56]], "text": "En cuadro severo de infecci\u00f3n por Plasmodium el mayor riesgo de mortalidad se produce en las primeras 24 horas de la presentaci\u00f3n cl\u00ednica por lo que se debe instaurar tratamiento inmediatamente. El tratamiento con artesunato parenteral o intramuscular es m\u00e1s indicado en infecci\u00f3n severa por Plasmodium, tanto en adulto como en ni\u00f1o y en mujer embarazada."}, "3": {"exist": false, "char_ranges": [], "word_ranges": [], "text": ""}, "4": {"exist": false, "char_ranges": [], "word_ranges": [], "text": ""}, "5": {"exist": false, "char_ranges": [], "word_ranges": [], "text": ""}}} {"id": 569, "year": 2022, "question_id_specific": 152, "full_question": "Var\u00f3n de 24 a\u00f1os que presenta microhematuria dism\u00f3rfica, proteinuria de 3 g/24 h, FGe (CKD-EPI) 85 ml/min e hipoacusia neurosensorial. Refiere que su abuela materna precis\u00f3 di\u00e1lisis a los 70 a\u00f1os y tanto su madre como su hermana peque\u00f1a presentan microhematuria aislada. \u00bfCu\u00e1l de las siguientes enfermedades es m\u00e1s probable?:", "full_answer": "Caso de libro, poco que a\u00f1adir. Microhematuria, proteinuria e hipoacusia neurosensorial en var\u00f3n joven, con antecedentes familiares de mujeres con hematuria = s\u00edndrome de Alport.", "type": "NEFROLOG\u00cdA", "options": {"1": "S\u00edndrome de Alport.", "2": "Nefropat\u00eda IgA.", "3": "Enfermedad de Fabry.", "4": "Poliquistosis renal autos\u00f3mica dominante.", "5": NaN}, "correct_option": 1, "explanations": {"1": {"exist": true, "char_ranges": [[32, 178]], "word_ranges": [[6, 25]], "text": "Microhematuria, proteinuria e hipoacusia neurosensorial en var\u00f3n joven, con antecedentes familiares de mujeres con hematuria = s\u00edndrome de Alport."}, "2": {"exist": false, "char_ranges": [], "word_ranges": [], "text": ""}, "3": {"exist": false, "char_ranges": [], "word_ranges": [], "text": ""}, "4": {"exist": false, "char_ranges": [], "word_ranges": [], "text": ""}, "5": {"exist": false, "char_ranges": [], "word_ranges": [], "text": ""}}} {"id": 501, "year": 2020, "question_id_specific": 76, "full_question": "Ni\u00f1o de 5 a\u00f1os diagnosticado de insuficiencia renal cr\u00f3nica. Acude a urgencias por v\u00f3mitos, malestar general y palpitaciones. Se realiza un electrocardiograma donde destaca elevaci\u00f3n de ondas T y anal\u00edtica sangu\u00ednea con niveles de potasio de 5,9 mEq/L. Se decide la administraci\u00f3n inmediata de gluconato c\u00e1lcico. \u00bfCu\u00e1l es el objetivo de este tratamiento?:", "full_answer": "El gluconato c\u00e1lcico no tiene efecto sobre los niveles de potasio en sangre (por tanto se descartan las opciones 1, 2 y 3). Su funci\u00f3n es disminuir la excitabilidad de los miocitos cardiacos para disminuir la probabilidad de desarrollar trastornos del ritmo card\u00edaco.", "type": "PEDIATR\u00cdA", "options": {"1": "Favorecer el desplazamiento de potasio desde el espacio plasm\u00e1tico al espacio intracelular.", "2": "Quelar el potasio circulante para favorecer su eliminaci\u00f3n hep\u00e1tica.", "3": "Quelar el potasio circulante para favorecer su eliminaci\u00f3n renal.", "4": "Antagonizar la acci\u00f3n del potasio sobre la membrana de la c\u00e9lula mioc\u00e1rdica.", "5": NaN}, "correct_option": 4, "explanations": {"1": {"exist": true, "char_ranges": [[0, 123]], "word_ranges": [[0, 23]], "text": "El gluconato c\u00e1lcico no tiene efecto sobre los niveles de potasio en sangre (por tanto se descartan las opciones 1, 2 y 3)."}, "2": {"exist": true, "char_ranges": [[0, 123]], "word_ranges": [[0, 23]], "text": "El gluconato c\u00e1lcico no tiene efecto sobre los niveles de potasio en sangre (por tanto se descartan las opciones 1, 2 y 3)."}, "3": {"exist": true, "char_ranges": [[0, 123]], "word_ranges": [[0, 23]], "text": "El gluconato c\u00e1lcico no tiene efecto sobre los niveles de potasio en sangre (por tanto se descartan las opciones 1, 2 y 3)."}, "4": {"exist": true, "char_ranges": [[124, 267]], "word_ranges": [[23, 43]], "text": "Su funci\u00f3n es disminuir la excitabilidad de los miocitos cardiacos para disminuir la probabilidad de desarrollar trastornos del ritmo card\u00edaco."}, "5": {"exist": false, "char_ranges": [], "word_ranges": [], "text": ""}}} {"id": 242, "year": 2014, "question_id_specific": 111, "full_question": "Hombre de 50 a\u00f1os, bronqu\u00edtico cr\u00f3nico que ingresa por cuadro neum\u00f3nico con hemocultivo positivo a Streptococcus pneumoniae, con una CMI a penicilina de 0,0125 mg/l. Se inicia tratamiento con penicilina 2 millones cada 4h. Al quinto d\u00eda, sigue con fiebre de 38\u00baC. \u00bfCu\u00e1l de las siguientes decisiones le parece correcta?", "full_answer": "Se trata de un hombre con una neumon\u00eda con germen aislado en hemocultivo y antibiograma. Seg\u00fan los datos que nos aportan, se trata de un neumococo sensible a penicilina (CMI< 0,125) por lo que se inicia tratamiento con penicilina a dosis altas cada 4 h, a pesar de lo cual, persiste la fiebre. Si analizamos el caso detenidamente, el tratamiento deber\u00eda estar siendo efectivo ya que el germen es totalmente sensible al f\u00e1rmaco y la posolog\u00eda es la adecuada (de forma que se deber\u00edan estar manteniendo concentraciones elevadas del f\u00e1rmaco en todo momento, recordemos que la eficacia de los betalact\u00e1micos es tiempo-dependiente y las concentraciones del antibi\u00f3tico en sangre deben estar por encima de la CMI de forma permanente para conseguir la efectividad y evitar la producci\u00f3n de resistencias). En tal caso, \u00bfqu\u00e9 puede estar pasando?, probablemente, a pesar de que el antibi\u00f3tico sea el adecuado, no est\u00e1 llegando adecuadamente al foco infeccioso porque probablemente se haya producido un empiema. Deberiamos descartar la existencia del mismo y drenarlo a la vez que proseguimos con el tratamiento antibi\u00f3tico. Si la posolog\u00eda no hubiese sido la adecuada y fuese posible la presencia de resistencias al f\u00e1rmaco, una opci\u00f3n podr\u00eda haber sido tratar con ceftriaxona a dosis altas ya que recordemos que el estreptococo no produce betalactamasas y la resistencia a betalactamicos se produce por otro mecanismo (poner amoxicilina-clavul\u00e1ncio no nos aportar\u00eda nada). Con la quinolona no aumentamos el espectro y es evidente que la mala evoluci\u00f3n del paciente no es solamente un problema de tiempo.", "type": "ENFERMEDADES INFECCIOSAS", "options": {"1": "Cambiar\u00eda el tratamiento a ceftriaxona por su mayor eficacia.", "2": "A\u00f1adir\u00eda al tratamiento una quinolona.", "3": "Cambiar\u00eda a amoxicilina/clavul\u00e1nico.", "4": "Descartar\u00eda la presencia de un empiema pleural.", "5": "Seguir\u00eda con el mismo tratamiento, suponiendo que simplemente es un problema de tiempo."}, "correct_option": 4, "explanations": {"1": {"exist": true, "char_ranges": [[1114, 1344]], "word_ranges": [[177, 214]], "text": "Si la posolog\u00eda no hubiese sido la adecuada y fuese posible la presencia de resistencias al f\u00e1rmaco, una opci\u00f3n podr\u00eda haber sido tratar con ceftriaxona a dosis altas ya que recordemos que el estreptococo no produce betalactamasas"}, "2": {"exist": true, "char_ranges": [[1464, 1594]], "word_ranges": [[230, 253]], "text": "Con la quinolona no aumentamos el espectro y es evidente que la mala evoluci\u00f3n del paciente no es solamente un problema de tiempo."}, "3": {"exist": true, "char_ranges": [[1347, 1463]], "word_ranges": [[215, 230]], "text": "la resistencia a betalactamicos se produce por otro mecanismo (poner amoxicilina-clavul\u00e1ncio no nos aportar\u00eda nada)."}, "4": {"exist": true, "char_ranges": [[853, 1113]], "word_ranges": [[137, 177]], "text": "a pesar de que el antibi\u00f3tico sea el adecuado, no est\u00e1 llegando adecuadamente al foco infeccioso porque probablemente se haya producido un empiema. Deberiamos descartar la existencia del mismo y drenarlo a la vez que proseguimos con el tratamiento antibi\u00f3tico."}, "5": {"exist": false, "char_ranges": [], "word_ranges": [], "text": ""}}} {"id": 487, "year": 2020, "question_id_specific": 68, "full_question": "\u00bfQu\u00e9 consejo gen\u00e9tico y reproductivo indicar\u00eda a una mujer de 30 a\u00f1os que consulta por ser portadora de una premutaci\u00f3n en el gen FMR1, responsable del s\u00edndrome del cromosoma X fr\u00e1gil, y que desea tener descendencia?. Se\u00f1ale la respuesta correcta:", "full_answer": "Otro cl\u00e1sico del MIR, preguntado en 2019 y en 2017, el s\u00edndrrome de X-fr\u00e1gil. Adem\u00e1s, sobre el mismo concepto de la pregunta 44, el fen\u00f3meno de anticipaci\u00f3n gen\u00e9tica. Las enfermedades por expansi\u00f3n de tripletes, como \u00e9sta, puede presentar el fen\u00f3meno de anticipaci\u00f3n por el que mujeres con premutaciones (entre 55-200 repeticiones) pueden tener hijos con mutaciones completas (m\u00e1s de 200 repeticiones del triplete) y cl\u00ednica m\u00e1s evidente, m\u00e1s grave y/o m\u00e1s precoz, tanto en hombres como en mujeres (s\u00edndrome de X-fr\u00e1gil, fallo ov\u00e1rico precoz, temblor/ataxia asociado al X-fr\u00e1gil).", "type": "GEN\u00c9TICA", "options": {"1": "Todos sus hijos varones ser\u00e1n portadores y, por tanto, manifestar\u00e1n la enfermedad.", "2": "Existe un fen\u00f3meno de anticipaci\u00f3n gen\u00e9tica, por lo que su descendencia presentar\u00e1 s\u00edntomas m\u00e1s graves y m\u00e1s precoces.", "3": "El diagn\u00f3stico gen\u00e9tico preimplantacional para la selecci\u00f3n de sexo de los embriones mediante hibridaci\u00f3n in situ fluorescente (FISH) es la mejor opci\u00f3n para tener descendencia sana.", "4": "El 50% de sus hijas ser\u00e1n portadoras, pero no presentar\u00e1n manifestaciones cl\u00ednicas de la enfermedad.", "5": NaN}, "correct_option": 2, "explanations": {"1": {"exist": false, "char_ranges": [], "word_ranges": [], "text": ""}, "2": {"exist": true, "char_ranges": [[167, 580]], "word_ranges": [[28, 88]], "text": "Las enfermedades por expansi\u00f3n de tripletes, como \u00e9sta, puede presentar el fen\u00f3meno de anticipaci\u00f3n por el que mujeres con premutaciones (entre 55-200 repeticiones) pueden tener hijos con mutaciones completas (m\u00e1s de 200 repeticiones del triplete) y cl\u00ednica m\u00e1s evidente, m\u00e1s grave y/o m\u00e1s precoz, tanto en hombres como en mujeres (s\u00edndrome de X-fr\u00e1gil, fallo ov\u00e1rico precoz, temblor/ataxia asociado al X-fr\u00e1gil)."}, "3": {"exist": false, "char_ranges": [], "word_ranges": [], "text": ""}, "4": {"exist": false, "char_ranges": [], "word_ranges": [], "text": ""}, "5": {"exist": false, "char_ranges": [], "word_ranges": [], "text": ""}}} {"id": 364, "year": 2016, "question_id_specific": 176, "full_question": "Mujer de 78 a\u00f1os con antecedentes de diabetes tipo 2 en tratamiento con antidiab\u00e9ticos orales, HTA en tratamiento con beta-bloqueantes e inhibidores de la ECA, e insuficiencia cardiaca congestiva grado 1 de la NYHA con FEVI del 48%, y actualmente asintom\u00e1tica. Estado funcional: ECOG 0. Historia de 2 a\u00f1os de evoluci\u00f3n de peque\u00f1os bultos en el cuello. Biopsia de adenopat\u00eda cervical: linfoma folicular grado 2. Estudio de extensi\u00f3n: Hb 12 gr/dL, Leucocitos 6.900/microL (Neutr\u00f3filos 60%, Linfocitos 27%, Monocitos 6%, Eosin\u00f3filos 4%, Bas\u00f3filos 4%), Plaquetas 220.000/microL. MO: infiltrado por Linfoma folicular. Creatinina 1,5 mg/dL, LDH 235 U/L, Beta2 microglobulina 2,1 microg/mL. TAC: adenopat\u00edas menores de 3 cm en territorios cervical, axilar, retroperitoneo, il\u00edacos e inguinales; h\u00edgado y bazo normales. \u00bfCu\u00e1l de los siguientes tratamientos es el m\u00e1s apropiado?:", "full_answer": "Puede haber debate sobre si comenzar Rituximab con alg\u00fan otro medicamento citost\u00e1tico o en monoterapia, pero dada la ausencia de beneficio en la supervivencia con el inicio precoz del tratamiento, en las gu\u00edas tipo NCCN recomiendan esperar y ver, salvo que se re\u00fanan criterios GELF para iniciar tratamiento: Criterios GELF (Grupo de Estudio de Linfomas Foliculares): \u2013 Compromiso de \u22653 \u00e1reas ganglionares, cada una con un di\u00e1metro \u2265 3 cm. \u2013 Cualquier masa ganglionar o extraganglionar con un di\u00e1metro \u2265 7 cm. \u2013 S\u00edntomas B. \u2013 Esplenomegalia. \u2013 Derrame pleural o ascitis. \u2013 Citopenias (leucocitos < 1.0 x 109 / L y/o plaquetas < 100 x 109 /L). \u2013 Leucemia (> 5.0 x 109 /L de c\u00e9lulas malignas).", "type": "HEMATOLOG\u00cdA", "options": {"1": "Rituximab-CHOP (Ciclofosfamida, Adriamicina, Vincristina, Prednisona).", "2": "Rituximab-CVP (Ciclofosfamida, Vincristina, Prednisona).", "3": "No tratar y vigilar (esperar y ver).", "4": "Rituxirnab-Bendamustina.", "5": NaN}, "correct_option": 3, "explanations": {"1": {"exist": true, "char_ranges": [[0, 245]], "word_ranges": [[0, 39]], "text": "Puede haber debate sobre si comenzar Rituximab con alg\u00fan otro medicamento citost\u00e1tico o en monoterapia, pero dada la ausencia de beneficio en la supervivencia con el inicio precoz del tratamiento, en las gu\u00edas tipo NCCN recomiendan esperar y ver,"}, "2": {"exist": true, "char_ranges": [[0, 245]], "word_ranges": [[0, 39]], "text": "Puede haber debate sobre si comenzar Rituximab con alg\u00fan otro medicamento citost\u00e1tico o en monoterapia, pero dada la ausencia de beneficio en la supervivencia con el inicio precoz del tratamiento, en las gu\u00edas tipo NCCN recomiendan esperar y ver,"}, "3": {"exist": true, "char_ranges": [[0, 245]], "word_ranges": [[0, 39]], "text": "Puede haber debate sobre si comenzar Rituximab con alg\u00fan otro medicamento citost\u00e1tico o en monoterapia, pero dada la ausencia de beneficio en la supervivencia con el inicio precoz del tratamiento, en las gu\u00edas tipo NCCN recomiendan esperar y ver,"}, "4": {"exist": true, "char_ranges": [[0, 245]], "word_ranges": [[0, 39]], "text": "Puede haber debate sobre si comenzar Rituximab con alg\u00fan otro medicamento citost\u00e1tico o en monoterapia, pero dada la ausencia de beneficio en la supervivencia con el inicio precoz del tratamiento, en las gu\u00edas tipo NCCN recomiendan esperar y ver,"}, "5": {"exist": false, "char_ranges": [], "word_ranges": [], "text": ""}}} {"id": 90, "year": 2012, "question_id_specific": 32, "full_question": "Una mujer de 35 a\u00f1os es diagnosticada de c\u00e1ncer de colon localizado en \u00e1ngulo hep\u00e1tico. Presenta antecedentes familiares de c\u00e1ncer de colon en madre, una t\u00eda a la edad de 45 a\u00f1os y un abuelo. La intervenci\u00f3n quir\u00fargica m\u00e1s aceptada es:", "full_answer": "El enunciado con los antecedentes familiares indica que se trata de una caso de c\u00e1ncer colorrectal hereditario no polip\u00f3sico (S\u00edndrome de Lynch). Los datos de edad y localizaci\u00f3n derecha nos enfocan a la respuesta 4 colectom\u00eda total con anastomosis ileorrectal. Si se realiza una cirug\u00eda convencional tipo hemicolectom\u00eda derecha las posibilidades de recidiva son superiores al 30%. La extirpaci\u00f3n del recto no es necesaria profil\u00e1cticamente.", "type": "CIRUG\u00cdA GENERAL", "options": {"1": "Hemicolectom\u00eda derecha.", "2": "Hemicolectom\u00eda derecha ampliada.", "3": "Colectom\u00eda subtotal.", "4": "Colectom\u00eda total con anatomosis ileorrectal.", "5": "Colectom\u00eda total con anatomosis ileoanal."}, "correct_option": 4, "explanations": {"1": {"exist": false, "char_ranges": [], "word_ranges": [], "text": ""}, "2": {"exist": false, "char_ranges": [], "word_ranges": [], "text": ""}, "3": {"exist": false, "char_ranges": [], "word_ranges": [], "text": ""}, "4": {"exist": true, "char_ranges": [[146, 261]], "word_ranges": [[22, 40]], "text": "Los datos de edad y localizaci\u00f3n derecha nos enfocan a la respuesta 4 colectom\u00eda total con anastomosis ileorrectal."}, "5": {"exist": false, "char_ranges": [], "word_ranges": [], "text": ""}}} {"id": 528, "year": 2021, "question_id_specific": 134, "full_question": "Mujer de 53 a\u00f1os de edad con antecedentes personales de obesidad y migra\u00f1a. En repetidas ocasiones se le ha tomado la presi\u00f3n arterial en la consulta m\u00e9dica y de enfermer\u00eda presentando cifras inferiores a 140/90 mmHg. Sin embargo, se ha comprado un aparato homologado para la toma de la presi\u00f3n arterial y se le ha ense\u00f1ado a utilizarlo correctamente. Acude mostrando registros de presi\u00f3n arterial tomados en su domicilio a lo largo de varias semanas con valores superiores a 140/90 mmHg. Se\u00f1ale la respuesta correcta:", "full_answer": "La paciente descrita es la definici\u00f3n de HTA enmascarada, cifras normales en consulta y elevadas al hacer un AMPA o MAPA (opci\u00f3n 2 correcta). No es una HTA secundaria porque no se ha hecho estudio para confirmarlo (opci\u00f3n 1 incorrecta). No es una HTA aislada porque las cifras elevadas se mantienen durante varias semanas (opci\u00f3n 3 incorrecta). No es una HTA refractaria porque a\u00fan no ha recibido tratamiento (opci\u00f3n 4 incorrecta).", "type": "NEFROLOG\u00cdA", "options": {"1": "Presenta una hipertensi\u00f3n arterial secundaria.", "2": "Presenta una hipertensi\u00f3n arterial enmascarada.", "3": "Presenta una hipertensi\u00f3n cl\u00ednica aislada.", "4": "Presenta una hipertensi\u00f3n arterial refractaria.", "5": NaN}, "correct_option": 2, "explanations": {"1": {"exist": true, "char_ranges": [[142, 236]], "word_ranges": [[24, 40]], "text": "No es una HTA secundaria porque no se ha hecho estudio para confirmarlo (opci\u00f3n 1 incorrecta)."}, "2": {"exist": true, "char_ranges": [[0, 141]], "word_ranges": [[0, 24]], "text": "La paciente descrita es la definici\u00f3n de HTA enmascarada, cifras normales en consulta y elevadas al hacer un AMPA o MAPA (opci\u00f3n 2 correcta)."}, "3": {"exist": true, "char_ranges": [[237, 344]], "word_ranges": [[40, 57]], "text": "No es una HTA aislada porque las cifras elevadas se mantienen durante varias semanas (opci\u00f3n 3 incorrecta)."}, "4": {"exist": true, "char_ranges": [[345, 431]], "word_ranges": [[57, 71]], "text": "No es una HTA refractaria porque a\u00fan no ha recibido tratamiento (opci\u00f3n 4 incorrecta)."}, "5": {"exist": false, "char_ranges": [], "word_ranges": [], "text": ""}}} {"id": 583, "year": 2022, "question_id_specific": 70, "full_question": "Mujer de 43 a\u00f1os portadora de dispositivo intrauterino de levonorgestrel que consulta por falta de descanso nocturno con aparici\u00f3n de calor y palpitaciones. En la anal\u00edtica destaca T4 libre 10,5 pmol/L (9-19), tirotropina 2,1 mUI/L (0,30-5,00), FSH 95,6 UI/L (1,38-16,7), LH 21 UI/L (2,4-9,3), estradiol < 0,07 nmol/L (0,07 \u2013 1,14). \u00bfCu\u00e1l es el tratamiento m\u00e1s adecuado?:", "full_answer": "El perfil hormonal que nos cuentan est\u00e1 en rango menop\u00e1usico (FSH >20; estradiol <5) y la paciente tiene s\u00edntomas vasomotores sist\u00e9micos. Por todo ello, ser\u00eda m\u00e1s adecuado administrar estr\u00f3genos sist\u00e9micos. Para compensar, ya tiene a nivel local uterino un progest\u00e1geno (DIU levonorgestrel).", "type": "OBSTETRICIA Y GINECOLOG\u00cdA", "options": {"1": "Benzodiacepinas.", "2": "Estr\u00f3genos vaginales.", "3": "Estr\u00f3genos y progest\u00e1genos orales en pauta continua.", "4": "Estr\u00f3genos transd\u00e9rmicos en pauta continua.", "5": NaN}, "correct_option": 4, "explanations": {"1": {"exist": false, "char_ranges": [], "word_ranges": [], "text": ""}, "2": {"exist": false, "char_ranges": [], "word_ranges": [], "text": ""}, "3": {"exist": false, "char_ranges": [], "word_ranges": [], "text": ""}, "4": {"exist": true, "char_ranges": [[0, 206]], "word_ranges": [[0, 30]], "text": "El perfil hormonal que nos cuentan est\u00e1 en rango menop\u00e1usico (FSH >20; estradiol <5) y la paciente tiene s\u00edntomas vasomotores sist\u00e9micos. Por todo ello, ser\u00eda m\u00e1s adecuado administrar estr\u00f3genos sist\u00e9micos."}, "5": {"exist": false, "char_ranges": [], "word_ranges": [], "text": ""}}} {"id": 195, "year": 2013, "question_id_specific": 163, "full_question": "Acude de urgencia al centro de salud, un ni\u00f1o de 15 meses de edad, que durante la cena, tras ingerir un bocado de tortilla, presenta de forma s\u00fabita: enrojecimiento facial de predominio perioral, lesiones habonosas en tronco y extremidades y tos. A su llegada al centro se encuentra consciente y se objetiva, adem\u00e1s de lo descrito: tiraje supraesternal, rinorrea acuosa abundante, hipoventilaci\u00f3n bilateral sin sibilancias y relleno capilar inferior a 2 segundos. De las siguientes afirmaciones, se\u00f1ale la respuesta CORRECTA:", "full_answer": "Est\u00e1 describiendo una anafilaxia por huevo, cuadro potencialmente mortal. El tratamiento de elecci\u00f3n es la adrenalina intramuscular, y efectivamente, sin perder demasiado tiempo. Lo primero es la adrenalina INTRAMUSCULAR \u00a1YA! La v\u00eda venosa, corticoides, antihistam\u00ednicos, etc vendr\u00e1n despu\u00e9s.", "type": "PEDIATR\u00cdA", "options": {"1": "Lo prioritario es canalizar una v\u00eda venosa.", "2": "La metilprednisolona por v\u00eda intramuscular es el tratamiento de elecci\u00f3n.", "3": "Se trata de un cuadro de urticaria asociado a asma, y debe ser tratado con antihistam\u00ednicos y broncodilatadores inhalados.", "4": "Se debe recomendar a los padres su traslado a un Servicio de Urgencias hospitalario.", "5": "Se debe administrar sin m\u00e1s dilaci\u00f3n, adrenalina por v\u00eda intramuscular."}, "correct_option": 5, "explanations": {"1": {"exist": true, "char_ranges": [[179, 292]], "word_ranges": [[23, 38]], "text": "Lo primero es la adrenalina INTRAMUSCULAR \u00a1YA! La v\u00eda venosa, corticoides, antihistam\u00ednicos, etc vendr\u00e1n despu\u00e9s."}, "2": {"exist": true, "char_ranges": [[179, 292]], "word_ranges": [[23, 38]], "text": "Lo primero es la adrenalina INTRAMUSCULAR \u00a1YA! La v\u00eda venosa, corticoides, antihistam\u00ednicos, etc vendr\u00e1n despu\u00e9s."}, "3": {"exist": true, "char_ranges": [[179, 292]], "word_ranges": [[23, 38]], "text": "Lo primero es la adrenalina INTRAMUSCULAR \u00a1YA! La v\u00eda venosa, corticoides, antihistam\u00ednicos, etc vendr\u00e1n despu\u00e9s."}, "4": {"exist": true, "char_ranges": [[179, 292]], "word_ranges": [[23, 38]], "text": "Lo primero es la adrenalina INTRAMUSCULAR \u00a1YA! La v\u00eda venosa, corticoides, antihistam\u00ednicos, etc vendr\u00e1n despu\u00e9s."}, "5": {"exist": true, "char_ranges": [[0, 178]], "word_ranges": [[0, 23]], "text": "Est\u00e1 describiendo una anafilaxia por huevo, cuadro potencialmente mortal. El tratamiento de elecci\u00f3n es la adrenalina intramuscular, y efectivamente, sin perder demasiado tiempo."}}} {"id": 84, "year": 2012, "question_id_specific": 49, "full_question": "Hombre de 75 a\u00f1os, hipertenso y dislip\u00e9mico con antecedentes de insuficiencia cardiaca por disfunci\u00f3n sist\u00f3lica del ventr\u00edculo izquierdo (FE < 30%). Acude a nuestra consulta por empeoramiento de su disnea habitual en las \u00faltimas semanas. Su tensi\u00f3n arterial sist\u00f3lica es de 160/95 mmHg con frecuencia cardiaca de 65 lpm. En la exploraci\u00f3n f\u00edsica no se auscultan crepitantes y su presi\u00f3n venosa yugular es normal. Aporta una determinaci\u00f3n de creatinina que es de 3.7 mg/dL con iones dentro de la normalidad. \u00bfCu\u00e1l de los siguientes f\u00e1rmacos es el m\u00e1s adecuado para mejorar el pron\u00f3stico de su insuficiencia cardiaca?", "full_answer": "Ante un paciente con m\u00faltiples FRCV con insuficiencia cardiaca y renal concomitante (presumiblemente de etiolog\u00eda m\u00faltiple: hipoperfusi\u00f3n renal, arteriosclerosis\u2026), debemos ser muy cuidadosos a la hora de emplear diur\u00e9ticos e IECA/ARA-II, pues podemos empeorar la funci\u00f3n renal y/o elevar las cifras de potasio (dentro de los diur\u00e9ticos, la espironolactona y la eplerenona son \u201cahorradores de potasio\u201d). Por tanto, en este caso utilizaremos el bisoprolol.", "type": "CARDIOLOG\u00cdA Y CIRUG\u00cdA VASCULAR", "options": {"1": "Enalapril.", "2": "Losart\u00e1n.", "3": "Espironolactona.", "4": "Bisoprolol.", "5": "Eplerenona."}, "correct_option": 4, "explanations": {"1": {"exist": true, "char_ranges": [[0, 403]], "word_ranges": [[0, 56]], "text": "Ante un paciente con m\u00faltiples FRCV con insuficiencia cardiaca y renal concomitante (presumiblemente de etiolog\u00eda m\u00faltiple: hipoperfusi\u00f3n renal, arteriosclerosis\u2026), debemos ser muy cuidadosos a la hora de emplear diur\u00e9ticos e IECA/ARA-II, pues podemos empeorar la funci\u00f3n renal y/o elevar las cifras de potasio (dentro de los diur\u00e9ticos, la espironolactona y la eplerenona son \u201cahorradores de potasio\u201d)."}, "2": {"exist": true, "char_ranges": [[0, 403]], "word_ranges": [[0, 56]], "text": "Ante un paciente con m\u00faltiples FRCV con insuficiencia cardiaca y renal concomitante (presumiblemente de etiolog\u00eda m\u00faltiple: hipoperfusi\u00f3n renal, arteriosclerosis\u2026), debemos ser muy cuidadosos a la hora de emplear diur\u00e9ticos e IECA/ARA-II, pues podemos empeorar la funci\u00f3n renal y/o elevar las cifras de potasio (dentro de los diur\u00e9ticos, la espironolactona y la eplerenona son \u201cahorradores de potasio\u201d)."}, "3": {"exist": true, "char_ranges": [[0, 403]], "word_ranges": [[0, 56]], "text": "Ante un paciente con m\u00faltiples FRCV con insuficiencia cardiaca y renal concomitante (presumiblemente de etiolog\u00eda m\u00faltiple: hipoperfusi\u00f3n renal, arteriosclerosis\u2026), debemos ser muy cuidadosos a la hora de emplear diur\u00e9ticos e IECA/ARA-II, pues podemos empeorar la funci\u00f3n renal y/o elevar las cifras de potasio (dentro de los diur\u00e9ticos, la espironolactona y la eplerenona son \u201cahorradores de potasio\u201d)."}, "4": {"exist": false, "char_ranges": [], "word_ranges": [], "text": ""}, "5": {"exist": true, "char_ranges": [[0, 403]], "word_ranges": [[0, 56]], "text": "Ante un paciente con m\u00faltiples FRCV con insuficiencia cardiaca y renal concomitante (presumiblemente de etiolog\u00eda m\u00faltiple: hipoperfusi\u00f3n renal, arteriosclerosis\u2026), debemos ser muy cuidadosos a la hora de emplear diur\u00e9ticos e IECA/ARA-II, pues podemos empeorar la funci\u00f3n renal y/o elevar las cifras de potasio (dentro de los diur\u00e9ticos, la espironolactona y la eplerenona son \u201cahorradores de potasio\u201d)."}}} {"id": 427, "year": 2018, "question_id_specific": 95, "full_question": "Mujer de 45 a\u00f1os de edad que acude al servicio de urgencias por cuadro confusional que se acompa\u00f1a de disfonia. A la exploraci\u00f3n presenta masa palpable en el cuello y en el an\u00e1lisis de sangre cifras de calcio plasm\u00e1tico de 15 mg/dL (normal hasta 10,2 mg/dL). Ante estos hallazgos, \u00bfcual de los siguiente diagn\u00f3sticos debe sospecharse?", "full_answer": "Masa palpable en el cuello: sospechar patolog\u00eda de tiroides/paratiroides. Nos aportan el dato de que asocia hipercalcemia, por tanto, la opci\u00f3n de ca. Medular de tiroides queda descartada (no afecta el nivel de calcemia). Para pensar en un MEN tipo I deber\u00edamos asociar otras patolog\u00edas (tumores hipofisarios, pancre\u00e1ticos o duodenales). La presencia de una calcemia superior a 13mgr/dl, junto con la disfon\u00eda, nos orienta a tumor maligno (mayor nivel de calcemia e infiltraci\u00f3n), decant\u00e1ndonos por tanto por la opci\u00f3n 1.", "type": "ENDOCRINOLOG\u00cdA", "options": {"1": "Carcinoma paratiroideo.", "2": "Carcinoma medular de tiroides.", "3": "MEN tipo I.", "4": "Adenoma de paratiroides.", "5": NaN}, "correct_option": 1, "explanations": {"1": {"exist": true, "char_ranges": [[338, 521]], "word_ranges": [[50, 80]], "text": "La presencia de una calcemia superior a 13mgr/dl, junto con la disfon\u00eda, nos orienta a tumor maligno (mayor nivel de calcemia e infiltraci\u00f3n), decant\u00e1ndonos por tanto por la opci\u00f3n 1."}, "2": {"exist": true, "char_ranges": [[74, 221]], "word_ranges": [[9, 34]], "text": "Nos aportan el dato de que asocia hipercalcemia, por tanto, la opci\u00f3n de ca. Medular de tiroides queda descartada (no afecta el nivel de calcemia)."}, "3": {"exist": true, "char_ranges": [[222, 337]], "word_ranges": [[34, 50]], "text": "Para pensar en un MEN tipo I deber\u00edamos asociar otras patolog\u00edas (tumores hipofisarios, pancre\u00e1ticos o duodenales)."}, "4": {"exist": false, "char_ranges": [], "word_ranges": [], "text": ""}, "5": {"exist": false, "char_ranges": [], "word_ranges": [], "text": ""}}} {"id": 462, "year": 2018, "question_id_specific": 151, "full_question": "Un ni\u00f1o de 13 a\u00f1os presenta un cuadro subagudo de cefalea y diplopia. La exploraci\u00f3n neurol\u00f3gica muestra una par\u00e1lisis de la mirada vertical y la resonancia magn\u00e9tica una lesi\u00f3n captante de contraste en la regi\u00f3n pineal que obstruye el acueducto de silvio. El diagn\u00f3stico m\u00e1s probable es:", "full_answer": "La diplopia habitualmente est\u00e1 causada por un estrabismo agudo o secundario. La par\u00e1lisis de la mirada vertical se debe a una lesi\u00f3n en el mesenc\u00e9falo. En este caso la resonancia magn\u00e9tica ya nos sit\u00faa el origen de los s\u00edntomas en la gl\u00e1ndula pineal. El tumor m\u00e1s frecuente de esta regi\u00f3n es el de c\u00e9lulas germinales (respuesta 3).", "type": "OFTALMOLOG\u00cdA (ECT\u00d3PICO)", "options": {"1": "Glioblastoma.", "2": "Meduloblastoma.", "3": "Tumor de c\u00e9lulas germinales.", "4": "Meningioma.", "5": NaN}, "correct_option": 3, "explanations": {"1": {"exist": false, "char_ranges": [], "word_ranges": [], "text": ""}, "2": {"exist": false, "char_ranges": [], "word_ranges": [], "text": ""}, "3": {"exist": true, "char_ranges": [[152, 331]], "word_ranges": [[25, 57]], "text": "En este caso la resonancia magn\u00e9tica ya nos sit\u00faa el origen de los s\u00edntomas en la gl\u00e1ndula pineal. El tumor m\u00e1s frecuente de esta regi\u00f3n es el de c\u00e9lulas germinales (respuesta 3)."}, "4": {"exist": false, "char_ranges": [], "word_ranges": [], "text": ""}, "5": {"exist": false, "char_ranges": [], "word_ranges": [], "text": ""}}} {"id": 557, "year": 2022, "question_id_specific": 168, "full_question": "Var\u00f3n de 58 a\u00f1os, obeso, asintom\u00e1tico, bebedor moderado, que presenta una primera glucemia basal de 153 mg/d con glucosuria negativa. En las semanas siguientes ha tenido otras dos cifras de glucemias basales de 118 y 136 mg/dl. \u00bfCu\u00e1l de las siguientes es la actitud m\u00e1s apropiada para confirmar el diagn\u00f3stico de diabetes mellitus?:", "full_answer": "Criterios de Diagn\u00f3stico de DM seg\u00fan ISPAD 2018 *S\u00edntomas cl\u00e1sicos de diabetes con glucemia > 200mgr/dl o: * Glucemia en ayunas \u2265 126mgr/dl (ayunas al menos 8h) o * Glucemia a las 2h de SOG \u2265 200mgr/dl (en 2 ocasiones) o * HbA1C \u2265 6,5%.", "type": "ENDOCRINOLOG\u00cdA", "options": {"1": "Practicar una curva de glucemia con 75 g de glucosa.", "2": "Re\u00fane ya criterios diagn\u00f3sticos de diabetes mellitus.", "3": "Solicitar una determinaci\u00f3n de insulinemia basal o p\u00e9ptido C.", "4": "Solicitar una determinaci\u00f3n de hemoglobina glicosilada.", "5": NaN}, "correct_option": 2, "explanations": {"1": {"exist": false, "char_ranges": [], "word_ranges": [], "text": ""}, "2": {"exist": true, "char_ranges": [[0, 236]], "word_ranges": [[0, 45]], "text": "Criterios de Diagn\u00f3stico de DM seg\u00fan ISPAD 2018 *S\u00edntomas cl\u00e1sicos de diabetes con glucemia > 200mgr/dl o: * Glucemia en ayunas \u2265 126mgr/dl (ayunas al menos 8h) o * Glucemia a las 2h de SOG \u2265 200mgr/dl (en 2 ocasiones) o * HbA1C \u2265 6,5%."}, "3": {"exist": false, "char_ranges": [], "word_ranges": [], "text": ""}, "4": {"exist": false, "char_ranges": [], "word_ranges": [], "text": ""}, "5": {"exist": false, "char_ranges": [], "word_ranges": [], "text": ""}}} {"id": 35, "year": 2011, "question_id_specific": 68, "full_question": "Un paciente de 56 a\u00f1os sin antecedentes personales y familiares de inter\u00e9s es estudiado por presentar desde hace seis meses dificultad para andar con la pierna derecha. No refiere otros s\u00edntomas. La exploraci\u00f3n muestra una debilidad 4/5 para la flexi\u00f3n dorsal y eversi\u00f3n del pie y 4/5 para flexi\u00f3n e inversi\u00f3n del pie, estando los reflejos musculares aumentados y con Babinski presente, siendo el resto normal. El diagn\u00f3stico sindr\u00f3mico ser\u00eda:", "full_answer": "Respuesta 1 incorrecta: No explica el piramidalismo. Respuesta 2 correcta: Una respuesta inespec\u00edfica, pero probablemente la respuesta correcta. Respuesta 3 incorrecta: Nos faltar\u00edan datos para pensar en un s\u00edndrome hemimedular de Brown Sequard. Explicar\u00eda el piramidalismo, pero faltar\u00edan datos sugestivos de este cuadro como una hipoestesia contralateral. Respuesta 4 incorrecta: Podr\u00eda explicar la dificultad para la flexi\u00f3n dorsal y plantar del pie, pero no el piramidalismo. Respuesta 5 incorrecta: De una forma similar a la anterior podr\u00eda explicar parte de la cl\u00ednica, pero no el piramidalismo.", "type": "NEUROLOG\u00cdA Y NEUROCIRUG\u00cdA", "options": {"1": "Mononeuropat\u00eda del N. peroneo com\u00fan derecho.", "2": "Afectaci\u00f3n focal de primera motoneurona y probable de segunda.", "3": "Afectaci\u00f3n hemimedular derecho (Brown-Sequard).", "4": "Mononeuropat\u00eda m\u00faltiple con afectaci\u00f3n de N. peroneo com\u00fan y tibial posterior derecho.", "5": "Plexopat\u00eda lumbar derecha."}, "correct_option": 2, "explanations": {"1": {"exist": true, "char_ranges": [[24, 52]], "word_ranges": [[3, 7]], "text": "No explica el piramidalismo."}, "2": {"exist": false, "char_ranges": [], "word_ranges": [], "text": ""}, "3": {"exist": true, "char_ranges": [[169, 357]], "word_ranges": [[21, 47]], "text": "Nos faltar\u00edan datos para pensar en un s\u00edndrome hemimedular de Brown Sequard. Explicar\u00eda el piramidalismo, pero faltar\u00edan datos sugestivos de este cuadro como una hipoestesia contralateral."}, "4": {"exist": true, "char_ranges": [[382, 479]], "word_ranges": [[50, 66]], "text": "Podr\u00eda explicar la dificultad para la flexi\u00f3n dorsal y plantar del pie, pero no el piramidalismo."}, "5": {"exist": true, "char_ranges": [[504, 601]], "word_ranges": [[69, 86]], "text": "De una forma similar a la anterior podr\u00eda explicar parte de la cl\u00ednica, pero no el piramidalismo."}}} {"id": 271, "year": 2016, "question_id_specific": 170, "full_question": "Acude a su consulta un hombre de 80 a\u00f1os de edad para valoraci\u00f3n de una colecistectom\u00eda programada por laparoscopica. Presenta antecedentes de hipertesi\u00f3n arterial en tratamiento desde hace 10 a\u00f1os. Niega enfermedad cardiaca ni pulmonar. No presenta dolor tor\u00e1cico. Tiene una vida activa y diariamente acude al gimnasio en donde alterna natacion y caminar por la cinta durante al menos una hora. Tratamiento habitual: nebivolol 5 mg cada 24 horas e hidroclorotiazida 12,5 mg al dia. Exploraci\u00f3n f\u00edsica: peso 73 kgs; talla 179 cm; presi\u00f3n arterial 138/80 mmHg; frecuencia cardiaca 60 latidos/minuto. No se auscultan soplos \u00bfCu\u00e1l de los siguientes es el planteamiento preoperatorio m\u00e1s adecuado?", "full_answer": "Pregunta totalmente regalada, de un se\u00f1or m\u00e1s sano que yo que se va a operar de una cirug\u00eda de bajo riesgo de manera programada. La respuesta es evidentemente la 4. De haber existido una quinta opci\u00f3n habr\u00eda sido que no hace falta hacer nada, lo cual podr\u00eda haber generado alguna duda en el estudiante, pero al no existir la pregunta es un caramelito. Aunque existen muchos protocolos sobre los preoperatorios, casi tantos como hospitales, la pregunta lo pone f\u00e1cil al poner a un paciente muy mayor, de 80 a\u00f1os. El rango de edad a partir del cual se considera obligatorio solicitar un ECG var\u00eda entre los 45, 50, 60 o incluso hay quien dice 70 a\u00f1os. Por ej: > 50 a\u00f1os: siempre, excepto si se dispone de ECG previo < 50 a\u00f1os: s\u00f3lo si hay cardiopat\u00eda conocida o sospechada (FRCV), DM severa, hipertiroidismo o EPOC severo.", "type": "ANESTESIOLOG\u00cdA Y CUIDADOS CR\u00cdTICOS", "options": {"1": "Realizar una prueba de esfuerzo.", "2": "Realizar una ecocardiograf\u00eda", "3": "Realizar una gammagraf\u00eda con talio y dipiridamol", "4": "Realizar electrocardiograma.", "5": NaN}, "correct_option": 4, "explanations": {"1": {"exist": false, "char_ranges": [], "word_ranges": [], "text": ""}, "2": {"exist": false, "char_ranges": [], "word_ranges": [], "text": ""}, "3": {"exist": false, "char_ranges": [], "word_ranges": [], "text": ""}, "4": {"exist": true, "char_ranges": [[352, 649]], "word_ranges": [[63, 116]], "text": "Aunque existen muchos protocolos sobre los preoperatorios, casi tantos como hospitales, la pregunta lo pone f\u00e1cil al poner a un paciente muy mayor, de 80 a\u00f1os. El rango de edad a partir del cual se considera obligatorio solicitar un ECG var\u00eda entre los 45, 50, 60 o incluso hay quien dice 70 a\u00f1os."}, "5": {"exist": false, "char_ranges": [], "word_ranges": [], "text": ""}}} {"id": 306, "year": 2016, "question_id_specific": 207, "full_question": "Paciente de 34 a\u00f1os que jugando a tenis recibe el impacto de la bola a nivel de la \u00f3rbita izquierda. A la exploraci\u00f3n presenta importante hematoma palpebral, hiposfagma, diplopia a la visi\u00f3n superior con limitaci\u00f3n de la versi\u00f3n superior del globo ocular. \u00bfQu\u00e9 sospechar\u00eda?", "full_answer": "En el enunciado describen diplopia a la vision superior con limitacion para la ersion superior del globo ocular. Al estar comprometida la funcion de forma mecanica del recto inferior izquierdo, no permitiria completar el movimiento de supraduccion generando diplopia vertical, esto mas el antecedente traumatico permite sospechar de una Fractura de piso orbitario con atrapamiento muscular del recto inferior. La Fractura de techo es mucho menos frecuente y el atrapamiento del recto superior provocaria limitacion para la infraduccion.", "type": "OFTALMOLOG\u00cdA", "options": {"1": "Fractura de la pared inferior del suelo de la \u00f3rbita con atrapamiento del m\u00fasculo recto inferior.", "2": "Fractura del arco cigom\u00e1tico.", "3": "Fractura de la pared superior de la \u00f3rbita con atrapamiento del m\u00fasculo recto superior.", "4": "Fractura dentoalveolar.", "5": NaN}, "correct_option": 1, "explanations": {"1": {"exist": true, "char_ranges": [[113, 409]], "word_ranges": [[18, 59]], "text": "Al estar comprometida la funcion de forma mecanica del recto inferior izquierdo, no permitiria completar el movimiento de supraduccion generando diplopia vertical, esto mas el antecedente traumatico permite sospechar de una Fractura de piso orbitario con atrapamiento muscular del recto inferior."}, "2": {"exist": false, "char_ranges": [], "word_ranges": [], "text": ""}, "3": {"exist": true, "char_ranges": [[410, 536]], "word_ranges": [[59, 78]], "text": "La Fractura de techo es mucho menos frecuente y el atrapamiento del recto superior provocaria limitacion para la infraduccion."}, "4": {"exist": false, "char_ranges": [], "word_ranges": [], "text": ""}, "5": {"exist": false, "char_ranges": [], "word_ranges": [], "text": ""}}} {"id": 328, "year": 2016, "question_id_specific": 83, "full_question": "Hombre de 49 a\u00f1os que consulta por poliuria y polidipsia intensas y p\u00e9rdida involuntaria de 10 kg de peso y es diagnosticado de diabetes mellitus por una glucemia plasm\u00e1tica de 322 mg/dL y una hemoglobina glicosilada de 9,8%. Su m\u00e9dico le da recomendaciones diet\u00e9ticas, la conveniencia de realizar ejercicio f\u00edsico, e inicia tratamiento con metformina 850 mg/12 horas y glimepirida 6 mg/dia. En las semanas siguientes los controles gluc\u00e9micos se van reduciendo progresivamente. A los 4 meses la glucemia es de 94 mg/dL y la HbA1c de 5,9%. El paciente se queja de episodios frecuentes de mareo, dolor epig\u00e1strico, visi\u00f3n borrosa, sudoraci\u00f3n y temblor, que mejoran comiendo algo y que ocurren sobre todo al final de la ma\u00f1ana y al final de la tarde. \u00bfQu\u00e9 modificaci\u00f3n propondria en su tratamiento?", "full_answer": "Suspender la sulfonilurea por Riesgo de hipoglucemia.", "type": "ENDOCRINOLOG\u00cdA", "options": {"1": "Revisar la distribuci\u00f3n de hidratos de carbono de su dieta.", "2": "Suspender la metformina.", "3": "Suspender la sulfonilurea.", "4": "Sustituir la metformina por un inhibidor de la DPP4.", "5": NaN}, "correct_option": 3, "explanations": {"1": {"exist": false, "char_ranges": [], "word_ranges": [], "text": ""}, "2": {"exist": false, "char_ranges": [], "word_ranges": [], "text": ""}, "3": {"exist": true, "char_ranges": [[0, 53]], "word_ranges": [[0, 7]], "text": "Suspender la sulfonilurea por Riesgo de hipoglucemia."}, "4": {"exist": false, "char_ranges": [], "word_ranges": [], "text": ""}, "5": {"exist": false, "char_ranges": [], "word_ranges": [], "text": ""}}} {"id": 223, "year": 2014, "question_id_specific": 62, "full_question": "Ante un paciente de 60 a\u00f1os de edad, diab\u00e9tico, con angina estable secundaria a cardiopat\u00eda isqu\u00e9mica por obstrucci\u00f3n subtotal en el tercio medio de la arteria coronaria descendente anterior, \u00bfqu\u00e9 tratamiento propondr\u00eda?", "full_answer": "Angina estable en diab\u00e9tico: esta pregunta es un gui\u00f1o al estudio BARI-2D (NEJM 2009), que demostr\u00f3 que el manejo m\u00e9dico no era inferior a la revascularizaci\u00f3n (de hecho, las nuevas gu\u00edas recogen esa tendencia de que no hay que abrir sistem\u00e1ticamente toda estenosis coronaria). La respuesta 1 va en ese sentido, aunque echo de menos las estatinas, IECA y antiagregantes, y me sobran los vasodilatadores (los nitratos son clase IIb, si no me falla la memoria). La 2 es absurda: \u00bfque est\u00e9 sentado para que no tenga angina? Es como talar el bosque para evitar incendios. Las 3 a 5 tratan acerca de revascularizar: dado que se trata de un \u00fanico vaso y no es una DA proximal, la revascularizaci\u00f3n habr\u00eda de ser percut\u00e1nea, por lo que descartamos la 3. Entre la 4 y la 5, la quinta es m\u00e1s completa: no s\u00f3lo habr\u00eda que dilatar, sino tambi\u00e9n implantar un stent. As\u00ed que la duda ser\u00eda entre la 1 y la 5. Aplicando las reglas generales de respuesta en el MIR, si dos opciones son parecidas salvo en un detalle, la correcta es una de esas, por lo que marcar\u00edamos la 5 (y es lo que se har\u00eda en cualquier hospital actualmente: una arteria subocluida se dilata directamente). Por otra parte, y aunque la 1 es incompleta, supongo que el autor ha querido ir de listillo y demostrar que se ha le\u00eddo los \u00faltimos art\u00edculos. Conclusi\u00f3n: marcar\u00eda la 5\u2026 pero no me sorprender\u00eda que fuese la 1.", "type": "CARDIOLOG\u00cdA", "options": {"1": "M\u00e9dico con vasodilatadores y betabloqueantes para evitar la angina.", "2": "Expectante con reposo riguroso ya que al disminuir la demanda mioc\u00e1rdica de ox\u00edgeno deber\u00e1 disminuir la angina.", "3": "Quir\u00fargico para revascularizar el miocardio isqu\u00e9mico mediante un puente (bypass) de arteria mamaria izquierda distal a la lesi\u00f3n en la arteria coronaria enferma.", "4": "Dilataci\u00f3n de la lesi\u00f3n en la arteria coronaria mediante un cateterismo terap\u00e9utico.", "5": "Cateterismo terap\u00e9utico para dilatar la lesi\u00f3n de la arteria enferma e implantaci\u00f3n de un \u201cstent\u201d en la zona dilatada."}, "correct_option": 5, "explanations": {"1": {"exist": true, "char_ranges": [[895, 1161]], "word_ranges": [[162, 208]], "text": "Aplicando las reglas generales de respuesta en el MIR, si dos opciones son parecidas salvo en un detalle, la correcta es una de esas, por lo que marcar\u00edamos la 5 (y es lo que se har\u00eda en cualquier hospital actualmente: una arteria subocluida se dilata directamente)."}, "2": {"exist": true, "char_ranges": [[460, 520]], "word_ranges": [[76, 88]], "text": "La 2 es absurda: \u00bfque est\u00e9 sentado para que no tenga angina?"}, "3": {"exist": true, "char_ranges": [[611, 746]], "word_ranges": [[104, 130]], "text": "dado que se trata de un \u00fanico vaso y no es una DA proximal, la revascularizaci\u00f3n habr\u00eda de ser percut\u00e1nea, por lo que descartamos la 3."}, "4": {"exist": true, "char_ranges": [[747, 853]], "word_ranges": [[130, 151]], "text": "Entre la 4 y la 5, la quinta es m\u00e1s completa: no s\u00f3lo habr\u00eda que dilatar, sino tambi\u00e9n implantar un stent."}, "5": {"exist": true, "char_ranges": [[895, 1161]], "word_ranges": [[162, 208]], "text": "Aplicando las reglas generales de respuesta en el MIR, si dos opciones son parecidas salvo en un detalle, la correcta es una de esas, por lo que marcar\u00edamos la 5 (y es lo que se har\u00eda en cualquier hospital actualmente: una arteria subocluida se dilata directamente)."}}} {"id": 468, "year": 2020, "question_id_specific": 122, "full_question": "Un hombre de 70 a\u00f1os consulta por dolor lumbar no irradiado de 1 semana de duraci\u00f3n, de intensidad progresiva, acompa\u00f1ado de marcada limitaci\u00f3n de la movilidad del raquis y febr\u00edcula. La radiograf\u00eda de columna lumbar no muestra alteraciones. \u00bfQu\u00e9 diagn\u00f3stico deber\u00edamos descartar en primer lugar y con qu\u00e9 t\u00e9cnica?:", "full_answer": "Sospechamos una espondilodiscitis \u2013 la febr\u00edcula y la limtaci\u00f3n de movilidad del raquis con el curso agudo dan la clave-, y la prueba indicada es la RM. \u201cLa prueba m\u00e1s demostrativa es la resonancia magn\u00e9tica (RM), que es positiva incluso en las primeras 2 semanas del cuadro, lo que facilita el diagn\u00f3stico precoz, con una sensibilidad del 90%\u201d. EL aplastamiento vertebral se ver\u00eda en la Rx simple, como tambi\u00e9n la hipertostosis anquilosante vertebral y muchas met\u00e1stasis. Adem\u00e1s, no hay datos en la historia que apunten hacia met\u00e1stasis \u00f3seas (antecedentes oncol\u00f3gicos, p\u00e9rdida ponderal etc).", "type": "CIRUG\u00cdA ORTOP\u00c9DICA Y TRAUMATOLOG\u00cdA", "options": {"1": "Aplastamiento vertebral con tomograf\u00eda computarizada (TC).", "2": "Met\u00e1stasis \u00f3seas con gammagraf\u00eda \u00f3sea con tecnecio.", "3": "Hiperostosis anquilosante vertebral con radiograf\u00eda de columna dorsolumbar.", "4": "Espondilodiscitis pi\u00f3gena con resonancia mang\u00e9tica (RM).", "5": NaN}, "correct_option": 4, "explanations": {"1": {"exist": true, "char_ranges": [[346, 472]], "word_ranges": [[58, 76]], "text": "EL aplastamiento vertebral se ver\u00eda en la Rx simple, como tambi\u00e9n la hipertostosis anquilosante vertebral y muchas met\u00e1stasis."}, "2": {"exist": true, "char_ranges": [[481, 593]], "word_ranges": [[77, 93]], "text": "no hay datos en la historia que apunten hacia met\u00e1stasis \u00f3seas (antecedentes oncol\u00f3gicos, p\u00e9rdida ponderal etc)."}, "3": {"exist": true, "char_ranges": [[346, 472]], "word_ranges": [[58, 76]], "text": "EL aplastamiento vertebral se ver\u00eda en la Rx simple, como tambi\u00e9n la hipertostosis anquilosante vertebral y muchas met\u00e1stasis."}, "4": {"exist": true, "char_ranges": [[0, 152]], "word_ranges": [[0, 27]], "text": "Sospechamos una espondilodiscitis \u2013 la febr\u00edcula y la limtaci\u00f3n de movilidad del raquis con el curso agudo dan la clave-, y la prueba indicada es la RM."}, "5": {"exist": false, "char_ranges": [], "word_ranges": [], "text": ""}}} {"id": 96, "year": 2012, "question_id_specific": 170, "full_question": "Enfermo de 60 a\u00f1os que refiere hace 10 d\u00edas la aparici\u00f3n de lesiones ampollosas en dorso de manos despu\u00e9s de exposici\u00f3n solar. El cuadro se acompa\u00f1a de fragilidad cut\u00e1nea. Histopatol\u00f3gicamente existe una ampolla subepid\u00e9rmica con dep\u00f3sitos PAS+ en y alrededor de los vasos d\u00e9rmicos superficiales. El diagn\u00f3stico m\u00e1s veros\u00edmil es:", "full_answer": "La Porfiria cut\u00e1nea tarda (PCT) es la porfiria m\u00e1s frecuente y se puede presentar con tres formas cl\u00ednicas mayores: familiar, espor\u00e1dica y t\u00f3xica. Cl\u00ednicamente, se caracteriza por una marcada fragilidad cut\u00e1nea con aparici\u00f3n de erosiones, ves\u00edculas, y ampollas ante m\u00ednimos traumatismos. Las localizaciones m\u00e1s frecuentes son el dorso de las manos y la cara, es decir las zonas de mayor fotoexposici\u00f3n. Cl\u00ednicamente, el diagn\u00f3stico diferencial m\u00e1s importante se debe establecer con la pseudoporfiria o dermatitis ampollosa fotot\u00f3xica, en la que las ampollas y erosiones cut\u00e1neas se producen por fotosensibilidad, inducidas por medicamentos como las tetraciclinas, sulfonamidas, etc. Sin embargo, y a diferencia de la porfiria, los niveles de porfirinas en suero, heces y orina son normales. El diagn\u00f3stico diferencial histopatol\u00f3gico debe establecerse con aquellas enfermedades que muestran dep\u00f3sitos hialinos d\u00e9rmicos, como son: la hialinosis cutis, en la que a diferencia de la porfiria, los dep\u00f3sitos hialinos PAS positivos son extensos y ocupan de forma masiva la dermis profunda y el tejido celular subcut\u00e1neo; el Milium coloide en el que el material hialino, generalmente PAS negativo expande la dermis papilar, formando hendiduras y fisuras caracter\u00edsticas; y la Proteinosis lipoidea, en la que el material se deposita en toda la dermis, tiene una distribuci\u00f3n no s\u00f3lo perivascular sino m\u00e1s extensa que en la porfiria, y suele afectar a gl\u00e1ndulas sudor\u00edparas.", "type": "DERMATOLOG\u00cdA, VENEREOLOG\u00cdA Y CIRUG\u00cdA PL\u00c1STICA", "options": {"1": "P\u00e9nfigo benigno familiar.", "2": "Penfigoide.", "3": "Porfiria cut\u00e1nea tarda.", "4": "P\u00e9nfigo vulgar.", "5": "S\u00edndrome de la piel escaldada."}, "correct_option": 3, "explanations": {"1": {"exist": false, "char_ranges": [], "word_ranges": [], "text": ""}, "2": {"exist": false, "char_ranges": [], "word_ranges": [], "text": ""}, "3": {"exist": true, "char_ranges": [[0, 402]], "word_ranges": [[0, 61]], "text": "La Porfiria cut\u00e1nea tarda (PCT) es la porfiria m\u00e1s frecuente y se puede presentar con tres formas cl\u00ednicas mayores: familiar, espor\u00e1dica y t\u00f3xica. Cl\u00ednicamente, se caracteriza por una marcada fragilidad cut\u00e1nea con aparici\u00f3n de erosiones, ves\u00edculas, y ampollas ante m\u00ednimos traumatismos. Las localizaciones m\u00e1s frecuentes son el dorso de las manos y la cara, es decir las zonas de mayor fotoexposici\u00f3n."}, "4": {"exist": false, "char_ranges": [], "word_ranges": [], "text": ""}, "5": {"exist": false, "char_ranges": [], "word_ranges": [], "text": ""}}} {"id": 603, "year": 2022, "question_id_specific": 112, "full_question": "Mujer de 61 a\u00f1os, administrativa, con antecedentes de sobrepeso, hipertensi\u00f3n, dislipemia y s\u00edndrome metab\u00f3lico, que consulta por dolor en ambas nalgas, regi\u00f3n trocant\u00e9rea izquierda, cara lateral del muslo izquierdo hasta la rodilla y pierna izquierda hasta el tercio medio. El dolor aparece cuando levantamos el miembro inferior con la rodilla extendida, pero se alivia al flexionar la rodilla. \u00bfCu\u00e1l es la primera sospecha cl\u00ednica?:", "full_answer": "La sintomatolog\u00eda m\u00e1s com\u00fan de un paciente con coxartrosis incluye: Dolor de cadera que limita la deambulaci\u00f3n, molestias nocturnas e incluso en reposo, rigidez de cadera y bloqueo. La artrosis coxo-femoral cursa con un test de Lasegue negativo. Respuesta 2 incorrecta. \u200b\u200bEl dolor lumbar de car\u00e1cter neurop\u00e1tico suele cursar con radiculopat\u00eda unilateral y distribuida seg\u00fan dermatomas. Los factores de riesgo reconocidos son con mayor frecuencia: mujer, obesidad, fumador y sedentarismo o situaciones de sedestaci\u00f3n prolongada. La lumbociat\u00e1lgia se suele referir en la nalga, cara posterior de los muslos e incluso inguinal. El caso cl\u00ednico describe la maniobra o test de Lasegue; una prueba de provocaci\u00f3n que evidencia irritaci\u00f3n radicular en la regi\u00f3n lumbosacra (Respuesta 3 correcta). La claudicaci\u00f3n por estenosis de canal cursa caracter\u00edsticamente con dolor que empeora con la extensi\u00f3n del tronco (caminar, estar de pie) y se alivia con la flexi\u00f3n (sentarse, dormir en posici\u00f3n fetal\u2026) Respuesta 4 incorrecta.", "type": "TRAUMATOLOG\u00cdA", "options": {"1": "Artritis gotosa de cadera izquierda.", "2": "Artrosis coxofemoral izquierda.", "3": "Lumbalgia irradiada / lumbociatalgia.", "4": "Claudicaci\u00f3n por estenosis de canal.", "5": NaN}, "correct_option": 3, "explanations": {"1": {"exist": false, "char_ranges": [], "word_ranges": [], "text": ""}, "2": {"exist": true, "char_ranges": [[182, 269]], "word_ranges": [[28, 41]], "text": "La artrosis coxo-femoral cursa con un test de Lasegue negativo. Respuesta 2 incorrecta."}, "3": {"exist": true, "char_ranges": [[625, 789]], "word_ranges": [[91, 116]], "text": "El caso cl\u00ednico describe la maniobra o test de Lasegue; una prueba de provocaci\u00f3n que evidencia irritaci\u00f3n radicular en la regi\u00f3n lumbosacra (Respuesta 3 correcta)."}, "4": {"exist": true, "char_ranges": [[790, 1017]], "word_ranges": [[116, 151]], "text": "La claudicaci\u00f3n por estenosis de canal cursa caracter\u00edsticamente con dolor que empeora con la extensi\u00f3n del tronco (caminar, estar de pie) y se alivia con la flexi\u00f3n (sentarse, dormir en posici\u00f3n fetal\u2026) Respuesta 4 incorrecta."}, "5": {"exist": false, "char_ranges": [], "word_ranges": [], "text": ""}}} {"id": 574, "year": 2022, "question_id_specific": 100, "full_question": "Mujer de 80 a\u00f1os con antecedentes de hipertensi\u00f3n arterial y diabetes mellitus que presenta de forma brusca una alteraci\u00f3n del lenguaje. A su llegada al hospital se objetiva una fibrilaci\u00f3n auricular no conocida y en la exploraci\u00f3n presenta habla fluente con tendencia a la jergafasia, mala compresi\u00f3n del lenguaje, incapacidad para la repetici\u00f3n y disnomia. Semiol\u00f3gicamente presenta un cuadro compatible con:", "full_answer": "Es una pregunta de semiolog\u00eda que nos ayuda a recordar los tipos de afasia. Hay que tener en cuenta los siguientes aspectos: emisi\u00f3n del lenguaje (fluente, no fluente, no emisi\u00f3n), comprensi\u00f3n (s\u00ed o no), repetici\u00f3n (s\u00ed o no) y nominaci\u00f3n. La afasia de Wernicke o sensitiva tiene un lenguaje fluente, no comprende, no repite y no nomina, tal y como la paciente de la pregunta. La afasia transcortical sensitiva se diferencia de la sensitiva en que los pacientes son capaces de repetir. La afasia de conducci\u00f3n se diferencia de la sensitiva en que la comprensi\u00f3n est\u00e1 conservada, el resto es igual. Por \u00faltimo, en la afasia de broca el lenguaje es no fluente, la comprensi\u00f3n esta preservada, no repite y no nomina.", "type": "NEUROLOG\u00cdA", "options": {"1": "Afasia de Broca.", "2": "Afasia de conducci\u00f3n.", "3": "Afasia transcortical sensitiva.", "4": "Afasia de Wernicke.", "5": NaN}, "correct_option": 4, "explanations": {"1": {"exist": true, "char_ranges": [[597, 712]], "word_ranges": [[101, 122]], "text": "Por \u00faltimo, en la afasia de broca el lenguaje es no fluente, la comprensi\u00f3n esta preservada, no repite y no nomina."}, "2": {"exist": true, "char_ranges": [[485, 596]], "word_ranges": [[82, 101]], "text": "La afasia de conducci\u00f3n se diferencia de la sensitiva en que la comprensi\u00f3n est\u00e1 conservada, el resto es igual."}, "3": {"exist": true, "char_ranges": [[376, 484]], "word_ranges": [[65, 82]], "text": "La afasia transcortical sensitiva se diferencia de la sensitiva en que los pacientes son capaces de repetir."}, "4": {"exist": true, "char_ranges": [[239, 375]], "word_ranges": [[40, 65]], "text": "La afasia de Wernicke o sensitiva tiene un lenguaje fluente, no comprende, no repite y no nomina, tal y como la paciente de la pregunta."}, "5": {"exist": false, "char_ranges": [], "word_ranges": [], "text": ""}}} {"id": 67, "year": 2012, "question_id_specific": 111, "full_question": "Un paciente var\u00f3n de 45 a\u00f1os de edad con antecedentes de litiasis \u00farica con c\u00f3licos nefr\u00edticos expulsivos de repetici\u00f3n desde hace 25 a\u00f1os, acude a Urgencias refiriendo palpitaciones y dolor lumbar derecho intenso desde hace 2 horas antes. El an\u00e1lisis de orina muestra un PH de 5.5 y leucocituria sin proteinuria. El electrocardiograma confirma una FA no conocida previamente. En la bioqu\u00edmica sangu\u00ednea destaca una Creat de 0.9, un Ca de 11 y una LDH de 950. \u00bfCu\u00e1l es la maniobra diagn\u00f3stica m\u00e1s \u00fatil a realizar?", "full_answer": "En principio, lo que nos cuenta no cuaja con un c\u00f3lico nefr\u00edtico. La orina es normal (sin hematuria, ocasionada por la litiasis al progresar), la Creat es normal (puede elevarse hasta 1.9 en c\u00f3licos nefr\u00edticos). El aumento de la LDH es inespec\u00edfico y lo podemos encontrar tanto en lesiones parenquimatosas hep\u00e1ticas, como en IAM o en embolias pulmonares, adem\u00e1s de en otros muchos casos menos frecuentes (tumores, miopat\u00edas, etc). En este caso, tenemos un factor de riesgo importante de embolismo, que es la fibrilaci\u00f3n auricular, que, adem\u00e1s, al ser un var\u00f3n de 45 a\u00f1os, sin antecedentes, parece que no responde a una cardiopat\u00eda estructural de base unido a un dolor lumbar derecho, que podr\u00eda corresponder a una irritaci\u00f3n de la pleura a nivel . El TC espiral con contraste ha adquirido gran valor en el diagn\u00f3stico del TEP, debido a su rapidez y buena sensibilidad, superior al 83 \u2013 92% en la mayor parte de los trabajos, y que seg\u00fan algunos puede llegar al 100% para trombos centrales, situados en arterias pulmonares principales, lobares o segmentarias. Su especificidad es mayor del 90%. Los ganglios pueden producir falsos positivos. Adem\u00e1s puede proporcionar un diagn\u00f3stico alternativo en caso de no confirmarse el TEP, por lo que ser\u00eda una prueba muy \u00fatil. El eco-doppler ser\u00eda la prueba m\u00e1s \u00fatil para diagnosticar la fuente de los \u00e9mbolos, as\u00ed que tambi\u00e9n podr\u00eda ser una respuesta v\u00e1lida dependiendo de lo que quieran decir con \u201c\u00fatil\u201d. Es una t\u00e9cnica no invasiva que ha demostrado su buena sensibilidad y especificidad en el diagn\u00f3stico de la TVP encomparaci\u00f3n con la flebograf\u00eda. Adem\u00e1s permite diagnosticar otras patolog\u00edas quepodr\u00edan justificar la cl\u00ednica del paciente. Sus limitaciones son la valoraci\u00f3n de trombosinfrapopl\u00edteos y, en muchos casos, en pacientes que no son delgados, de las venas il\u00edacas y cava.", "type": "ANESTESIOLOG\u00cdA, CUIDADOS CR\u00cdTICOS Y URGENCIAS", "options": {"1": "Radiograf\u00eda simple de abdomen.", "2": "Urograf\u00eda intravenosa.", "3": "TC espiral con contraste.", "4": "Ecograf\u00eda abdominal.", "5": "Ecograf\u00eda doppler."}, "correct_option": 3, "explanations": {"1": {"exist": false, "char_ranges": [], "word_ranges": [], "text": ""}, "2": {"exist": false, "char_ranges": [], "word_ranges": [], "text": ""}, "3": {"exist": true, "char_ranges": [[748, 1093]], "word_ranges": [[124, 182]], "text": "El TC espiral con contraste ha adquirido gran valor en el diagn\u00f3stico del TEP, debido a su rapidez y buena sensibilidad, superior al 83 \u2013 92% en la mayor parte de los trabajos, y que seg\u00fan algunos puede llegar al 100% para trombos centrales, situados en arterias pulmonares principales, lobares o segmentarias. Su especificidad es mayor del 90%."}, "4": {"exist": false, "char_ranges": [], "word_ranges": [], "text": ""}, "5": {"exist": false, "char_ranges": [], "word_ranges": [], "text": ""}}} {"id": 77, "year": 2012, "question_id_specific": 31, "full_question": "Un muchacho de 17 a\u00f1os de edad con enfermedad de Crohn con afectaci\u00f3n col\u00f3nica de 2 a\u00f1os de evoluci\u00f3n, en tratamiento de mantenimiento con azatioprina, consulta por la aparici\u00f3n desde hace 5 d\u00edas de n\u00f3dulos subcut\u00e1neos rojo viol\u00e1ceos, calientes, dolorosos, de localizaci\u00f3n pretibial, bilaterales asociada a un incremento del n\u00famero de deposiciones y dolor abdominal. La actitud m\u00e1s adecuada en este caso es:", "full_answer": "Parece que el paciente tiene lesiones de eritema nodoso asociado a un brote de su enfermedad, cosa que es habitual. El tratamiento de la enfermedad y su control suele mejorar las lesiones cutaneas. No se debe biopsiar las lesiones.", "type": "APARATO DIGESTIVO", "options": {"1": "Recomendar reposo relativo y pa\u00f1os calientes en ambas piernas y a\u00f1adir tratamiento antidepresivo.", "2": "Biopsiar zonas de piel alejados de las \u00e1reas lesionadas y pautar analg\u00e9sicos opioides de entrada.", "3": "Biopsiar zonas de piel alejados de las \u00e1reas lesionadas y pautar analg\u00e9sicos opioides de entrada.", "4": "Sospechar isquemia bilateral de extremidades inferiores de origen medicamentoso.", "5": "Ajuste del tratamiento de la enfermedad."}, "correct_option": 5, "explanations": {"1": {"exist": false, "char_ranges": [], "word_ranges": [], "text": ""}, "2": {"exist": true, "char_ranges": [[0, 231]], "word_ranges": [[0, 39]], "text": "Parece que el paciente tiene lesiones de eritema nodoso asociado a un brote de su enfermedad, cosa que es habitual. El tratamiento de la enfermedad y su control suele mejorar las lesiones cutaneas. No se debe biopsiar las lesiones."}, "3": {"exist": true, "char_ranges": [[0, 231]], "word_ranges": [[0, 39]], "text": "Parece que el paciente tiene lesiones de eritema nodoso asociado a un brote de su enfermedad, cosa que es habitual. El tratamiento de la enfermedad y su control suele mejorar las lesiones cutaneas. No se debe biopsiar las lesiones."}, "4": {"exist": false, "char_ranges": [], "word_ranges": [], "text": ""}, "5": {"exist": true, "char_ranges": [[0, 231]], "word_ranges": [[0, 39]], "text": "Parece que el paciente tiene lesiones de eritema nodoso asociado a un brote de su enfermedad, cosa que es habitual. El tratamiento de la enfermedad y su control suele mejorar las lesiones cutaneas. No se debe biopsiar las lesiones."}}} {"id": 440, "year": 2018, "question_id_specific": 97, "full_question": "Mujer de 24 a\u00f1os consulta por dolor abdominal bajo de dos semanas de evoluci\u00f3n que ha empeorado con la \u00faltima relaci\u00f3n sexual. Refiere tener pareja reciente con la que s\u00f3lo ocasionalmente utiliza preservativo. A la exploraci\u00f3n presenta regular estado general. T\u00aa de 38,6\u00baC, leucocitos 16.000/uL (85% neutr\u00f3filos); PCR 30 mg/L La exploraci\u00f3n con especulo evidencia flujo vaginal anormal abundante y movilizaci\u00f3n cervical dolorosa. Indique la respuesta FALSA:", "full_answer": "Se sospecha una enfermedad p\u00e9lvica inflamatoria, dados los s\u00edntomas de la paciente. Seg\u00fan el protocolo de la SEGO de 2006 de Enfermedad P\u00e9lvica Inflamatoria, la respuesta 2 es la falsa y, por tanto, la que habr\u00eda que marcar. Esto es as\u00ed porque la histerosalpingograf\u00eda puede propagar la infecci\u00f3n a cavidad p\u00e9lvica y abdominal. El resto de las respuestas son correctas.", "type": "GINECOLOG\u00cdA Y OBSTETRICIA", "options": {"1": "Iniciar\u00edamos tratamiento antibi\u00f3tico tan pronto como sospechemos el diagnostico.", "2": "Indicar\u00edamos como prueba complementaria preferente una histerosalpingografia para valoraci\u00f3n tubarica.", "3": "El retraso en el diagn\u00f3stico y tratamiento aumenta la aparici\u00f3n de secuelas.", "4": "La presencia de absceso seria criterio de hospitalizaci\u00f3n.", "5": NaN}, "correct_option": 2, "explanations": {"1": {"exist": false, "char_ranges": [], "word_ranges": [], "text": ""}, "2": {"exist": true, "char_ranges": [[158, 327]], "word_ranges": [[24, 53]], "text": "la respuesta 2 es la falsa y, por tanto, la que habr\u00eda que marcar. Esto es as\u00ed porque la histerosalpingograf\u00eda puede propagar la infecci\u00f3n a cavidad p\u00e9lvica y abdominal."}, "3": {"exist": false, "char_ranges": [], "word_ranges": [], "text": ""}, "4": {"exist": false, "char_ranges": [], "word_ranges": [], "text": ""}, "5": {"exist": false, "char_ranges": [], "word_ranges": [], "text": ""}}} {"id": 203, "year": 2013, "question_id_specific": 47, "full_question": "Mujer de 55 a\u00f1os, intervenida de apendicitis hace 24 a\u00f1os, que acude porque estando previamente bien, ha comenzado con v\u00f3mitos de repetici\u00f3n tras desayunar, hace unas 12 horas, asociado a distensi\u00f3n abdominal. Ha presentado una deposici\u00f3n diarreica a las pocas horas de comenzar el cuadro. A la exploraci\u00f3n se evidencia distensi\u00f3n abdominal, timpanismo y aumento de ruidos intestinales, pero no irritaci\u00f3n peritoneal. La anal\u00edtica no presenta alteraciones. En la radiograf\u00eda de abdomen aparece dilataci\u00f3n de asas de intestino delgado, sin poder objetivarse gas en la ampolla rectal. Respecto a este caso es cierto que:", "full_answer": "Es la descripci\u00f3n t\u00edpica de un cuadro de obstrucci\u00f3n intestinal por bridas o adherencias debidas a cirug\u00eda previa. La respuesta correcta es la 5 y no la 1 porque la mayor\u00eda de estas situaciones se resuelve espont\u00e1neamente con tratamiento conservador.", "type": "CIRUG\u00cdA GENERAL", "options": {"1": "Se trata de un cuadro de obstrucci\u00f3n intestinal por bridas que requiere laparotom\u00eda de urgencia.", "2": "Lo m\u00e1s probable es que nos encontremos ante un caso de obstrucci\u00f3n a nivel de intestino grueso.", "3": "El hecho de que la paciente haya presentado una deposici\u00f3n diarreica nos permite excluir el diagn\u00f3stico de obstrucci\u00f3n intestinal. Son necesarios m\u00e1s estudios de imagen para asegurar un diagn\u00f3stico.", "4": "Deber\u00edamos indicar la pr\u00e1ctica de una colonoscopia descompresiva.", "5": "El manejo inicial debe ser conservador, con sueroterapia, aspiraci\u00f3n nasog\u00e1strica y vigilancia cl\u00ednica, anal\u00edtica y radiogr\u00e1fica peri\u00f3dica."}, "correct_option": 5, "explanations": {"1": {"exist": true, "char_ranges": [[0, 250]], "word_ranges": [[0, 40]], "text": "Es la descripci\u00f3n t\u00edpica de un cuadro de obstrucci\u00f3n intestinal por bridas o adherencias debidas a cirug\u00eda previa. La respuesta correcta es la 5 y no la 1 porque la mayor\u00eda de estas situaciones se resuelve espont\u00e1neamente con tratamiento conservador."}, "2": {"exist": false, "char_ranges": [], "word_ranges": [], "text": ""}, "3": {"exist": false, "char_ranges": [], "word_ranges": [], "text": ""}, "4": {"exist": false, "char_ranges": [], "word_ranges": [], "text": ""}, "5": {"exist": true, "char_ranges": [[0, 250]], "word_ranges": [[0, 40]], "text": "Es la descripci\u00f3n t\u00edpica de un cuadro de obstrucci\u00f3n intestinal por bridas o adherencias debidas a cirug\u00eda previa. La respuesta correcta es la 5 y no la 1 porque la mayor\u00eda de estas situaciones se resuelve espont\u00e1neamente con tratamiento conservador."}}} {"id": 158, "year": 2012, "question_id_specific": 133, "full_question": "Chico de 12 a\u00f1os obeso (87 kg). Refiere desde hace 4 meses dolor mec\u00e1nico en muslo y rodilla derecha. A la exploraci\u00f3n cl\u00ednica se evidencia una discrepancia de la longitud de las extremidades inferiores de 1 cm y actitud de la extremidad inferior derecha en rotaci\u00f3n externa con p\u00e9rdida de la rotaci\u00f3n interna. Debemos sospechar que presenta:", "full_answer": "Nos presentan un paciente adolescente con sobrepeso y dolor en muslo y rodilla, discrepancia de longitud de 1 cm y el miembro en rotaci\u00f3n externa. Este cuadro nos debe hacer marcar autom\u00e1ticamente la epifisiolisis capitis femoris (epifisiolisis femoral proximal, el hecho de leerlo en n\u00f3mina latina es un poco a mala leche pero no deber\u00eda despistarnos). Debemos pensar en esta porque es la m\u00e1s frecuente y posible (respuesta 2 correcta). Las otras opciones no son imposibles pero si menos frecuentes y no nos lo plantear\u00edan as\u00ed si quisieran que nos decant\u00e1ramos por otra opci\u00f3n. Si fueran secuelas de displasia de cadera no tendr\u00eda este inicio de 4 meses de evoluci\u00f3n y la edad de aparici\u00f3n ser\u00eda m\u00e1s temprana. Las complicaciones de la displasia son luxaciones y subluxaciones que pueden requerir cirug\u00eda y la necrosis avascular de cabeza (respuesta 5 falsa). La enfermedad de Perthes se da en ni\u00f1os m\u00e1s peque\u00f1os (4-9 a\u00f1os) con episodios de cojera y dolor en cadera y rodilla (respuesta 4 falsa). En un ni\u00f1o lo m\u00e1s t\u00edpico ser\u00eda una fractura a nivel de la fisis no a nivel del cuello femoral por lo que descartamos la 3. Para que nos plante\u00e1ramos esta opci\u00f3n tendr\u00edan que habernos planteado un quiste \u00f3seo, algo que justificara una debilidad en la zona del cuello femoral. En cuanto a la artritis cr\u00f3nica juvenil es una opci\u00f3n de reuma rodeada de opciones de trauma. No nos hablan de factores reumatol\u00f3gicos en una anal\u00edtica, ni de un cuadro poliarticular. Quiz\u00e1s podr\u00edamos pensar en esta porque se habla de dolor en rodilla derecha, pero este cuadro que nos presentan se limita al miembro inferior derecho no nos hablan ni de otras articulaciones ni de fiebre ni de otros s\u00edntomas acompa\u00f1antes sist\u00e9micos por lo que no resulta una opci\u00f3n a valorar m\u00e1xime con las otras opciones que nos presentan que ser\u00edan m\u00e1s probables ante el cuadro que nos presentan (opci\u00f3n 1 falsa). Adem\u00e1s, el dolor de rodilla de este caso se refiere a un dolor referido de la cadera, sin afectaci\u00f3n de la rodilla.", "type": "TRAUMATOLOG\u00cdA Y ORTOPEDIA", "options": {"1": "Artritis cr\u00f3nica juvenil.", "2": "Epifisiolisis capitis femoris.", "3": "Fractura del cuello femoral por sobrecarga.", "4": "Enfermedad de Perthes.", "5": "Secuelas de displasia del desarrollo de la cadera."}, "correct_option": 2, "explanations": {"1": {"exist": true, "char_ranges": [[1456, 1871]], "word_ranges": [[246, 317]], "text": "Quiz\u00e1s podr\u00edamos pensar en esta porque se habla de dolor en rodilla derecha, pero este cuadro que nos presentan se limita al miembro inferior derecho no nos hablan ni de otras articulaciones ni de fiebre ni de otros s\u00edntomas acompa\u00f1antes sist\u00e9micos por lo que no resulta una opci\u00f3n a valorar m\u00e1xime con las otras opciones que nos presentan que ser\u00edan m\u00e1s probables ante el cuadro que nos presentan (opci\u00f3n 1 falsa)."}, "2": {"exist": true, "char_ranges": [[354, 437]], "word_ranges": [[56, 70]], "text": "Debemos pensar en esta porque es la m\u00e1s frecuente y posible (respuesta 2 correcta)."}, "3": {"exist": true, "char_ranges": [[997, 1119]], "word_ranges": [[165, 191]], "text": "En un ni\u00f1o lo m\u00e1s t\u00edpico ser\u00eda una fractura a nivel de la fisis no a nivel del cuello femoral por lo que descartamos la 3."}, "4": {"exist": true, "char_ranges": [[860, 996]], "word_ranges": [[140, 165]], "text": "La enfermedad de Perthes se da en ni\u00f1os m\u00e1s peque\u00f1os (4-9 a\u00f1os) con episodios de cojera y dolor en cadera y rodilla (respuesta 4 falsa)."}, "5": {"exist": true, "char_ranges": [[711, 859]], "word_ranges": [[118, 140]], "text": "Las complicaciones de la displasia son luxaciones y subluxaciones que pueden requerir cirug\u00eda y la necrosis avascular de cabeza (respuesta 5 falsa)."}}} {"id": 46, "year": 2011, "question_id_specific": 155, "full_question": "Ni\u00f1o de un a\u00f1o de edad cuya madre acaba de ser diagnosticada de tuberculosis pulmonar bacil\u00edfera, habiendo iniciado tratamiento correcto. El ni\u00f1o est\u00e1 asintom\u00e1tico con una exploraci\u00f3n normal, prueba de tuberculina (PT) negativa y Rx t\u00f3rax normal. \u00bfCu\u00e1l ser\u00eda la actitud a adoptar en el ni\u00f1o?", "full_answer": "La respuesta correcta es la 5. La madre deber\u00eda estar aislada durante las 2-4 primeras semanas de tratamiento, tras las cuales dejar\u00eda de ser contagiosa. Si al repetir la PT el resultado sigue siendo negativo, al ni\u00f1o no hay que hacerle nada m\u00e1s. Si es positiva, hay que repetir la radiograf\u00eda (o TC pulmonar en los ni\u00f1os m\u00e1s peque\u00f1os).", "type": "PEDIATR\u00cdA", "options": {"1": "Solo separarlo de la madre hasta que \u00e9sta concluya su tratamiento (al menos 6 meses).", "2": "Realizar una TC de alta resoluci\u00f3n pulmonar, y si es normal, repetir la PT a las 8-12 semanas.", "3": "Quimiprofilaxis con isoniacida (INH) hasta que la baciloscopia de la madre sea negativa.", "4": "Quimioprofilaxis con INH 6-9 meses. Nueva PT y Rx t\u00f3rax al finalizarlo, para decidir si concluir o no, el tratamiento.", "5": "Quimioprofilaxis con INH, 8-12 semanas. Repetir PT. Si negativa cesar el tratamiento. Si positiva, realizar Rx t\u00f3rax para decidir si hay o no enfermedad."}, "correct_option": 5, "explanations": {"1": {"exist": false, "char_ranges": [], "word_ranges": [], "text": ""}, "2": {"exist": false, "char_ranges": [], "word_ranges": [], "text": ""}, "3": {"exist": false, "char_ranges": [], "word_ranges": [], "text": ""}, "4": {"exist": false, "char_ranges": [], "word_ranges": [], "text": ""}, "5": {"exist": true, "char_ranges": [[31, 336]], "word_ranges": [[6, 59]], "text": "La madre deber\u00eda estar aislada durante las 2-4 primeras semanas de tratamiento, tras las cuales dejar\u00eda de ser contagiosa. Si al repetir la PT el resultado sigue siendo negativo, al ni\u00f1o no hay que hacerle nada m\u00e1s. Si es positiva, hay que repetir la radiograf\u00eda (o TC pulmonar en los ni\u00f1os m\u00e1s peque\u00f1os)."}}} {"id": 593, "year": 2022, "question_id_specific": 146, "full_question": "Mujer de 40 a\u00f1os, sin antecedentes personales ni familiares de inter\u00e9s, que presenta diarrea de varios d\u00edas de evoluci\u00f3n de car\u00e1cter acuoso, muy abundante, acompa\u00f1ada de p\u00e9rdida de 10 kg de peso, rubor y enrojecimiento facial, junto a lipotimias y dolor abdominal c\u00f3lico y que en los datos de laboratorio presenta hipopotasemia. En ecograf\u00eda abdominal se encuentra masa de 1,5 cm de di\u00e1metro en cola pancre\u00e1tica. De entre las siguientes, su sospecha diagn\u00f3stica primaria es:", "full_answer": "P\u00e9ptido intestinal vasoactivo (VIP). VIPoma-diarrea. La 3.", "type": "ONCOLOG\u00cdA", "options": {"1": "Insulinoma.", "2": "PPoma.", "3": "VIPoma.", "4": "Glucagonoma.", "5": NaN}, "correct_option": 3, "explanations": {"1": {"exist": false, "char_ranges": [], "word_ranges": [], "text": ""}, "2": {"exist": false, "char_ranges": [], "word_ranges": [], "text": ""}, "3": {"exist": true, "char_ranges": [[0, 58]], "word_ranges": [[0, 7]], "text": "P\u00e9ptido intestinal vasoactivo (VIP). VIPoma-diarrea. La 3."}, "4": {"exist": false, "char_ranges": [], "word_ranges": [], "text": ""}, "5": {"exist": false, "char_ranges": [], "word_ranges": [], "text": ""}}} {"id": 395, "year": 2016, "question_id_specific": 136, "full_question": "Un hombre de 41 a\u00f1os acude a Urgencias por un cuadro de tres d\u00edas de evoluci\u00f3n de tumefacci\u00f3n y dolor en la rodilla derecha, con impotencia funcional y febr\u00edcula Dos semanas antes hab\u00eda tenido un cuadro de diarrea autolimitada En la exploraci\u00f3n existe derrame articular, por lo que se procede a realizar una artrocentesis y se obtienen 50 cc de l\u00edquido de color turbio, con viscosidad disminuida y los siguientes par\u00e1metros anal\u00edticos: leucocitos 40.000/microL (85% de neutr\u00f3filos), glucosa 40 mg/dL, ausencia de cristales, tinci\u00f3n de Gram: no se observan microorganismos. \u00bfCu\u00e1l de las siguientes afirmaciones sobre este paciente es \u00ecNCORRECTA?:", "full_answer": "Una tinci\u00f3n de Gram negativa nunca descarta la presencia de una infecci\u00f3n.", "type": "TRAUMATOLOG\u00cdA Y ORTOPED\u00cdA", "options": {"1": "Se debe iniciar tratamiento con cloxacilina y ceftriaxona en espera del resultado del cultivo del l\u00edquido.", "2": "Es aconsejable realizar artrocentesis diarias para aliviar los s\u00edntomas y evitar la destrucci\u00f3n articular.", "3": "Si el cultivo es negativo, es probable que se trate de una artritis reactiva.", "4": "La negatividad de la tinci\u00f3n de Gram descarta que se trate de una artritis s\u00e9ptica.", "5": NaN}, "correct_option": 4, "explanations": {"1": {"exist": false, "char_ranges": [], "word_ranges": [], "text": ""}, "2": {"exist": false, "char_ranges": [], "word_ranges": [], "text": ""}, "3": {"exist": false, "char_ranges": [], "word_ranges": [], "text": ""}, "4": {"exist": true, "char_ranges": [[0, 74]], "word_ranges": [[0, 12]], "text": "Una tinci\u00f3n de Gram negativa nunca descarta la presencia de una infecci\u00f3n."}, "5": {"exist": false, "char_ranges": [], "word_ranges": [], "text": ""}}} {"id": 424, "year": 2018, "question_id_specific": 91, "full_question": "Una mujer de 27 a\u00f1os de edad consulta por presentar amenorrea secundaria de un a\u00f1o de evoluci\u00f3n. No refiere antecedentes yatrog\u00e9nicos. Anal\u00edticamente, la concentraci\u00f3n de estradiol es inferior a la normalidad y el valor de prolactina es de 12 ng/mL. Indique cual de las exploraciones que se citan a continuaci\u00f3n solicitar\u00eda como primera medida para tratar de filiar el origen del trastorno:", "full_answer": "Mujer joven con amenorrea secundaria, sin antecedentes de inter\u00e9s. Los valores de prolactina son normales (se considera hiperprolectinemia valores por encima de 20ngr/ml), descart\u00e1ndose por tanto hiperprolactinemia como la causa de la amenorrea. Nos dicen que el nivel estrog\u00e9nico es bajo. En este caso debemos sospechar un fallo ov\u00e1rico precoz. Para su diagn\u00f3stico solicitaremos una FSH basal (en los casos de fallo ov\u00e1rico prematuro se encontrar\u00eda elevada).", "type": "ENDOCRINOLOG\u00cdA", "options": {"1": "Resonancia magn\u00e9tica hipotalamohipofisaria.", "2": "Cariotipo.", "3": "FSH basal.", "4": "Estimulaci\u00f3n con gonadotropina cori\u00f3nica.", "5": NaN}, "correct_option": 3, "explanations": {"1": {"exist": false, "char_ranges": [], "word_ranges": [], "text": ""}, "2": {"exist": false, "char_ranges": [], "word_ranges": [], "text": ""}, "3": {"exist": true, "char_ranges": [[246, 459]], "word_ranges": [[33, 67]], "text": "Nos dicen que el nivel estrog\u00e9nico es bajo. En este caso debemos sospechar un fallo ov\u00e1rico precoz. Para su diagn\u00f3stico solicitaremos una FSH basal (en los casos de fallo ov\u00e1rico prematuro se encontrar\u00eda elevada)."}, "4": {"exist": false, "char_ranges": [], "word_ranges": [], "text": ""}, "5": {"exist": false, "char_ranges": [], "word_ranges": [], "text": ""}}} {"id": 169, "year": 2013, "question_id_specific": 87, "full_question": "Mujer de 70 a\u00f1os, hipertensa y diab\u00e9tica que ha trabajado de dependienta toda su vida. Consulta por una lesi\u00f3n maleolar interna en la pierna derecha con intenso dolor durante todo el d\u00eda que empeora durante la noche. A la exploraci\u00f3n destacan cordones varicosos no complicados en ambas extremidades y pulsos femorales y popl\u00edteos normales con ausencia de distales. \u00bfQu\u00e9 exploraci\u00f3n deber\u00edamos realizar en primer lugar?:", "full_answer": "Al menos yo como medico de familia que soy, suelo empezar por las pruebas menos cruentas y con mayor coste-beneficio, y en el caso que nos describen, un Problema Isquemico en Miembros Inferiores es el Indice tobillo-brazo.", "type": "CARDIOLOG\u00cdA Y CIRUG\u00cdA CARDIOVASCULAR", "options": {"1": "Eco-doppler venoso de miembros inferiores.", "2": "Angiorresonancia de miembros inferiores.", "3": "Arteriograf\u00eda.", "4": "Indice tobillo-brazo.", "5": "AngioTC."}, "correct_option": 4, "explanations": {"1": {"exist": false, "char_ranges": [], "word_ranges": [], "text": ""}, "2": {"exist": false, "char_ranges": [], "word_ranges": [], "text": ""}, "3": {"exist": false, "char_ranges": [], "word_ranges": [], "text": ""}, "4": {"exist": true, "char_ranges": [[44, 222]], "word_ranges": [[9, 37]], "text": "suelo empezar por las pruebas menos cruentas y con mayor coste-beneficio, y en el caso que nos describen, un Problema Isquemico en Miembros Inferiores es el Indice tobillo-brazo."}, "5": {"exist": false, "char_ranges": [], "word_ranges": [], "text": ""}}} {"id": 95, "year": 2012, "question_id_specific": 170, "full_question": "Enfermo de 60 a\u00f1os que refiere hace 10 d\u00edas la aparici\u00f3n de lesiones ampollosas en dorso de manos despu\u00e9s de exposici\u00f3n solar. El cuadro se acompa\u00f1a de fragilidad cut\u00e1nea. Histopatol\u00f3gicamente existe una ampolla subepid\u00e9rmica con dep\u00f3sitos PAS+ en y alrededor de los vasos d\u00e9rmicos superficiales. El diagn\u00f3stico m\u00e1s veros\u00edmil es:", "full_answer": "El hecho de ser un cuadro fotoinducido, la histolog\u00eda y por la edad hacen m\u00e1s probable el diagn\u00f3stico de PCT (3). El resto no cuadran con el cuadro cl\u00ednico ni con la histolog\u00eda.", "type": "DERMATOLOG\u00cdA, VENEREOLOG\u00cdA Y CIRUG\u00cdA PL\u00c1STICA", "options": {"1": "P\u00e9nfigo benigno familiar.", "2": "Penfigoide.", "3": "Porfiria cut\u00e1nea tarda.", "4": "P\u00e9nfigo vulgar.", "5": "S\u00edndrome de la piel escaldada."}, "correct_option": 3, "explanations": {"1": {"exist": true, "char_ranges": [[114, 177]], "word_ranges": [[21, 33]], "text": "El resto no cuadran con el cuadro cl\u00ednico ni con la histolog\u00eda."}, "2": {"exist": true, "char_ranges": [[114, 177]], "word_ranges": [[21, 33]], "text": "El resto no cuadran con el cuadro cl\u00ednico ni con la histolog\u00eda."}, "3": {"exist": true, "char_ranges": [[0, 113]], "word_ranges": [[0, 21]], "text": "El hecho de ser un cuadro fotoinducido, la histolog\u00eda y por la edad hacen m\u00e1s probable el diagn\u00f3stico de PCT (3)."}, "4": {"exist": true, "char_ranges": [[114, 177]], "word_ranges": [[21, 33]], "text": "El resto no cuadran con el cuadro cl\u00ednico ni con la histolog\u00eda."}, "5": {"exist": true, "char_ranges": [[114, 177]], "word_ranges": [[21, 33]], "text": "El resto no cuadran con el cuadro cl\u00ednico ni con la histolog\u00eda."}}} {"id": 537, "year": 2021, "question_id_specific": 1, "full_question": "Paciente de 44 a\u00f1os con lesi\u00f3n (16x8x12 cm) de crecimiento r\u00e1pido que interesa las partes blandas del tercio proximal del muslo derecho. Refiere que ha empezado a notar la masa, que tiene una consistencia dura y est\u00e1 adherida a planos profundos, en los \u00faltimos 6 meses. Previamente hac\u00eda deporte habitualmente, llegando a haber terminado alguna marat\u00f3n. Se observa a la exploraci\u00f3n circulaci\u00f3n colateral, pero no se palpan adenopat\u00edas inguinales derechas. Tras la biopsia se confirma que se trata de un sarcoma pleom\u00f3rfico indiferenciado de alto grado, que en los estudios de imagen se encuentra proximal al troc\u00e1nter menor. La TC de extensi\u00f3n es negativa. \u00bfCu\u00e1l ser\u00eda su actitud terap\u00e9utica?:", "full_answer": "En los m\u00e1rgenes de resecci\u00f3n, tenemos: intralesional \u2192 marginal (opci\u00f3n 3) \u2192 Ampliada \u2192 Radical (opciones 1 y 4). Se trata de un tumor de alto grado, en el que est\u00e1 indicada la resecci\u00f3n ampliada que incluye m\u00e1rgenes libres del tumor. La cirug\u00eda radical de amputaci\u00f3n no ha demostrado mejorar la supervivencia reserv\u00e1ndose para casos seleccionados (invasi\u00f3n de estructuras neurovasculares, fractura patol\u00f3gica\u2026). Por lo tanto, descartamos tanto la opci\u00f3n 1 como 4; la opci\u00f3n 3 tambi\u00e9n queda descartada, porque es un margen menor que la ampliada; por eliminaci\u00f3n nos quedamos con la 2.", "type": "TRAUMATOLOG\u00cdA", "options": {"1": "Desarticulaci\u00f3n de la cadera.", "2": "Tratamiento con ifosfamida y tamoxifeno durante 6 ciclos previos a la cirug\u00eda de resecci\u00f3n para reducir el tama\u00f1o de la lesi\u00f3n.", "3": "Cirug\u00eda marginal extirpando la pseudoc\u00e1psula que suelen formar estos tumores como respuesta a su r\u00e1pido crecimiento.", "4": "Hemipelvectom\u00eda derecha modificada.", "5": NaN}, "correct_option": 2, "explanations": {"1": {"exist": true, "char_ranges": [[235, 464]], "word_ranges": [[41, 72]], "text": "La cirug\u00eda radical de amputaci\u00f3n no ha demostrado mejorar la supervivencia reserv\u00e1ndose para casos seleccionados (invasi\u00f3n de estructuras neurovasculares, fractura patol\u00f3gica\u2026). Por lo tanto, descartamos tanto la opci\u00f3n 1 como 4;"}, "2": {"exist": false, "char_ranges": [], "word_ranges": [], "text": ""}, "3": {"exist": true, "char_ranges": [[465, 545]], "word_ranges": [[72, 86]], "text": "la opci\u00f3n 3 tambi\u00e9n queda descartada, porque es un margen menor que la ampliada;"}, "4": {"exist": true, "char_ranges": [[235, 464]], "word_ranges": [[41, 72]], "text": "La cirug\u00eda radical de amputaci\u00f3n no ha demostrado mejorar la supervivencia reserv\u00e1ndose para casos seleccionados (invasi\u00f3n de estructuras neurovasculares, fractura patol\u00f3gica\u2026). Por lo tanto, descartamos tanto la opci\u00f3n 1 como 4;"}, "5": {"exist": false, "char_ranges": [], "word_ranges": [], "text": ""}}} {"id": 363, "year": 2016, "question_id_specific": 97, "full_question": "Paciente de 35 a\u00f1os que, debido a hematemesis por ulcus, recibe una transfusi\u00f3n de 2 concentrados de hemat\u00edes. A los 5-10 minutos de iniciarse la transfusi\u00f3n comienza con fiebre, escalofr\u00edos, hipotensi\u00f3n y dolor en regi\u00f3n lumbar. \u00bfCu\u00e1l ser\u00eda el diagn\u00f3stico m\u00e1s probable?", "full_answer": "Las reacciones transfusionales hemol\u00edticas agudas pueden ocurrir en cuesti\u00f3n de minutos. Cl\u00ednicamente se caracterizan por escalofr\u00edos, fiebre, urticaria, taquicardia, n\u00e1useas y v\u00f3mitos, dolor lumbar, hemorragia por coagulaci\u00f3n intravascular diseminada (CID)\u2026. Nos cuentan de un paciente que en pocos minutos tras iniciar una transfusi\u00f3n presenta un cuadro agudo y muy grave (con shock). Esa situaci\u00f3n tan aguda y grave solo cuadra con la opci\u00f3n 4.", "type": "HEMATOLOG\u00cdA", "options": {"1": "Contaminaci\u00f3n bacteriana de la sangre.", "2": "Reacci\u00f3n febril secundaria a la transfusi\u00f3n.", "3": "Reacci\u00f3n febril por el plasma que contamina los hemat\u00edes.", "4": "Reacci\u00f3n transfusional hemol\u00edtica.", "5": NaN}, "correct_option": 4, "explanations": {"1": {"exist": false, "char_ranges": [], "word_ranges": [], "text": ""}, "2": {"exist": false, "char_ranges": [], "word_ranges": [], "text": ""}, "3": {"exist": false, "char_ranges": [], "word_ranges": [], "text": ""}, "4": {"exist": true, "char_ranges": [[260, 447]], "word_ranges": [[30, 64]], "text": "Nos cuentan de un paciente que en pocos minutos tras iniciar una transfusi\u00f3n presenta un cuadro agudo y muy grave (con shock). Esa situaci\u00f3n tan aguda y grave solo cuadra con la opci\u00f3n 4."}, "5": {"exist": false, "char_ranges": [], "word_ranges": [], "text": ""}}} {"id": 339, "year": 2016, "question_id_specific": 33, "full_question": "Mujer de 67 a\u00f1os diagnosticada de un carcinoma ductal infiltrante de mama y sin historia familiar de neoplasia. \u00bfQu\u00e9 estudios adicionales deben realizarse en el tumor por sus implicaciones cl\u00ednico-terap\u00e9uticas?", "full_answer": "Todos los carcinomas de la mama deben ir acompa\u00f1ados del estudio de receptores hormonales y de HER-2 por las implicaciones pron\u00f3sticas del tumor, as\u00ed como la adecuaci\u00f3n del tratamiento del mismo, ya que dependiendo del resultado se aplicar\u00e1n unos f\u00e1rmacos u otros. El estudio de familiares y del gen BCRA no est\u00e1 indicado ya que no presenta historia familiar de neoplasia.", "type": "GINECOLOG\u00cdA Y OBSTETRICIA", "options": {"1": "Estudio fenot\u00edpico completo mediante citometr\u00eda de flujo.", "2": "Estudio de receptores hormonales y de HER2.", "3": "Estudio de receptores hormonales, e-cadherina y estudio de familiares de primer grado.", "4": "Estudio de BRCA 1-2 y estudio de familiares de primer grado.", "5": NaN}, "correct_option": 2, "explanations": {"1": {"exist": false, "char_ranges": [], "word_ranges": [], "text": ""}, "2": {"exist": true, "char_ranges": [[0, 194]], "word_ranges": [[0, 31]], "text": "Todos los carcinomas de la mama deben ir acompa\u00f1ados del estudio de receptores hormonales y de HER-2 por las implicaciones pron\u00f3sticas del tumor, as\u00ed como la adecuaci\u00f3n del tratamiento del mismo,"}, "3": {"exist": true, "char_ranges": [[265, 372]], "word_ranges": [[42, 61]], "text": "El estudio de familiares y del gen BCRA no est\u00e1 indicado ya que no presenta historia familiar de neoplasia."}, "4": {"exist": true, "char_ranges": [[265, 372]], "word_ranges": [[42, 61]], "text": "El estudio de familiares y del gen BCRA no est\u00e1 indicado ya que no presenta historia familiar de neoplasia."}, "5": {"exist": false, "char_ranges": [], "word_ranges": [], "text": ""}}} {"id": 144, "year": 2012, "question_id_specific": 142, "full_question": "Una ni\u00f1a de 13 a\u00f1os de edad, nos consulta por presentar p\u00e9rdida de peso, astenia, amenorrea secundaria y estre\u00f1imiento. En la exploraci\u00f3n destaca una temperatura de 35\u00baC, un peso inferior al percentil 3, bradicardia e hipotensi\u00f3n ortost\u00e1tica. En la anal\u00edtica se comprueba anemia con leucopenia leve, colesterol de 230 mg/dl y en el electrocardiograma una onda T aplanada con descenso del ST y QTc prolongado. \u00bfQu\u00e9 proceso sospechar\u00eda y qu\u00e9 har\u00eda a continuaci\u00f3n?", "full_answer": "La respuesta correcta es la 1. Los s\u00edntomas y signos descritos son los hallazgos biol\u00f3gicos asociados a la anorexia nerviosa. Los trastornos en el ECG est\u00e1n derivados de las alteraciones electrol\u00edticas.", "type": "PEDIATR\u00cdA", "options": {"1": "Sospechar\u00eda una anorexia nerviosa e indagar\u00eda acerca de su ingesta, imagen corporal, etc.", "2": "Sospechar\u00eda un hipotiroidismo y solicitar\u00eda TSH y hormonas tiroideas.", "3": "Sospechar\u00eda una miocardiopat\u00eda y solicitar\u00eda una ecocardiograf\u00eda.", "4": "Sospechar\u00eda una enfermedad cel\u00edaca y solicitar\u00eda anticuerpos anti transglutaminasa IgA.", "5": "Sospechar\u00eda un panhipopituitarismo y solicitar\u00eda una TC craneal."}, "correct_option": 1, "explanations": {"1": {"exist": true, "char_ranges": [[31, 125]], "word_ranges": [[6, 20]], "text": "Los s\u00edntomas y signos descritos son los hallazgos biol\u00f3gicos asociados a la anorexia nerviosa."}, "2": {"exist": false, "char_ranges": [], "word_ranges": [], "text": ""}, "3": {"exist": false, "char_ranges": [], "word_ranges": [], "text": ""}, "4": {"exist": false, "char_ranges": [], "word_ranges": [], "text": ""}, "5": {"exist": false, "char_ranges": [], "word_ranges": [], "text": ""}}} {"id": 356, "year": 2016, "question_id_specific": 166, "full_question": "Una embarazada de 10 semanas de gestaci\u00f3n, acude urgencias hospitalarias por presentar una presi\u00f3n arterial de 160/105 mmHg. Buen estado general, s\u00f3lo leve cefalea, motivo por el que se tom\u00f3 la presi\u00f3n arterial. Tras 4 horas de reposo presenta cifras de 150/95mmHg. Se le realiza un hemograma que es normal y una proteinuria que es negativa. \u00bfQu\u00e9 tipo de hipertensi\u00f3n presenta?", "full_answer": "Hipertensi\u00f3n cr\u00f3nica. Es una hipertensi\u00f3n que se detecta antes del embarazo o antes de la semana 20 del mismo y/o persiste a la 12 semana tras el parto.", "type": "GINECOLOG\u00cdA Y OBSTETRICIA", "options": {"1": "Preeclampsia moderada.", "2": "Hipertensi\u00f3n inducida por el embarazo.", "3": "Hipertensi\u00f3n cr\u00f3nica.", "4": "Eclampsia.", "5": NaN}, "correct_option": 3, "explanations": {"1": {"exist": false, "char_ranges": [], "word_ranges": [], "text": ""}, "2": {"exist": false, "char_ranges": [], "word_ranges": [], "text": ""}, "3": {"exist": true, "char_ranges": [[22, 152]], "word_ranges": [[2, 28]], "text": "Es una hipertensi\u00f3n que se detecta antes del embarazo o antes de la semana 20 del mismo y/o persiste a la 12 semana tras el parto."}, "4": {"exist": false, "char_ranges": [], "word_ranges": [], "text": ""}, "5": {"exist": false, "char_ranges": [], "word_ranges": [], "text": ""}}} {"id": 495, "year": 2020, "question_id_specific": 64, "full_question": "Mujer de 65 a\u00f1os sin comorbilidades, operada de cirug\u00eda conservadora de mama izquierda con biopsia selectiva del ganglio centinela. La anatom\u00eda patol\u00f3gica de la pieza muestra un carcinoma ductal infiltrante grado III de 11 mm, dos ganglios centinelas negativos, receptores de estr\u00f3genos: 0%, receptores de progesterona: 0%, Ki 67: 70%, HER2: negativo. El estudio de extensi\u00f3n es negativo. El tratamiento adyuvante ser\u00eda:", "full_answer": "Pregunta t\u00edpica, cirug\u00eda conservadora implica radiar, seguro. Alto grado, y no tiene receptores hormonales ni her2 (la temida triple negativa). Yo estuve cuando no hab\u00eda ni taxanos ni herceptin y estuve en los ensayos que demostraron que se curan cada vez m\u00e1s, por desgracia a\u00fan no todas. Por eso hoy en d\u00eda, antraciclinas (no sufre del coraz\u00f3n) y taxanos antes de la RT. Mi especialidad es bonita, y lo ser\u00e1 m\u00e1s en los pr\u00f3ximos a\u00f1os.", "type": "ONCOLOG\u00cdA M\u00c9DICA", "options": {"1": "Quimioterapia basada en antraciclinas y taxanos, radioterapia y trastuzumab.", "2": "Quimioterapia basada en antraciclinas y taxanos, radioterapia y tamoxifeno.", "3": "Quimioterapia basada en antraciclinas y taxanos, y radioterapia.", "4": "Quimioterapia basada en antraciclinas y taxanos, radioterapia e inhibidores de la aromatasa, dado que es postmenop\u00e1usica.", "5": NaN}, "correct_option": 3, "explanations": {"1": {"exist": false, "char_ranges": [], "word_ranges": [], "text": ""}, "2": {"exist": false, "char_ranges": [], "word_ranges": [], "text": ""}, "3": {"exist": true, "char_ranges": [[297, 371]], "word_ranges": [[49, 63]], "text": "hoy en d\u00eda, antraciclinas (no sufre del coraz\u00f3n) y taxanos antes de la RT."}, "4": {"exist": false, "char_ranges": [], "word_ranges": [], "text": ""}, "5": {"exist": false, "char_ranges": [], "word_ranges": [], "text": ""}}} {"id": 355, "year": 2016, "question_id_specific": 166, "full_question": "Una embarazada de 10 semanas de gestaci\u00f3n, acude urgencias hospitalarias por presentar una presi\u00f3n arterial de 160/105 mmHg. Buen estado general, s\u00f3lo leve cefalea, motivo por el que se tom\u00f3 la presi\u00f3n arterial. Tras 4 horas de reposo presenta cifras de 150/95mmHg. Se le realiza un hemograma que es normal y una proteinuria que es negativa. \u00bfQu\u00e9 tipo de hipertensi\u00f3n presenta?", "full_answer": "La respuesta correcta es la 3. La SEGO en su protocolo de 2006 \u00abTrastornos hipertensivos del embarazo\u00bb refiere que se trata de una hipertensi\u00f3n cr\u00f3nica, aunque diagnosticada en el embarazo. Esto es as\u00ed porque ha sido diagnosticada a las 10 semanas de gestaci\u00f3n. Por encima de las 20 semanas de gestaci\u00f3n, ya hablar\u00edamos de preeclampsia (si HTA + proteinuria \u00f3 indice de pulsatilidad de arterias uterinas por encima del percentil 95), hipertensi\u00f3n inducida por el embarazo (s\u00f3lo HTA) \u00f3 eclampsia (es una forma de preeclampsia grave).", "type": "GINECOLOG\u00cdA Y OBSTETRICIA", "options": {"1": "Preeclampsia moderada.", "2": "Hipertensi\u00f3n inducida por el embarazo.", "3": "Hipertensi\u00f3n cr\u00f3nica.", "4": "Eclampsia.", "5": NaN}, "correct_option": 3, "explanations": {"1": {"exist": true, "char_ranges": [[262, 532]], "word_ranges": [[43, 86]], "text": "Por encima de las 20 semanas de gestaci\u00f3n, ya hablar\u00edamos de preeclampsia (si HTA + proteinuria \u00f3 indice de pulsatilidad de arterias uterinas por encima del percentil 95), hipertensi\u00f3n inducida por el embarazo (s\u00f3lo HTA) \u00f3 eclampsia (es una forma de preeclampsia grave)."}, "2": {"exist": true, "char_ranges": [[262, 532]], "word_ranges": [[43, 86]], "text": "Por encima de las 20 semanas de gestaci\u00f3n, ya hablar\u00edamos de preeclampsia (si HTA + proteinuria \u00f3 indice de pulsatilidad de arterias uterinas por encima del percentil 95), hipertensi\u00f3n inducida por el embarazo (s\u00f3lo HTA) \u00f3 eclampsia (es una forma de preeclampsia grave)."}, "3": {"exist": true, "char_ranges": [[31, 261]], "word_ranges": [[6, 43]], "text": "La SEGO en su protocolo de 2006 \u00abTrastornos hipertensivos del embarazo\u00bb refiere que se trata de una hipertensi\u00f3n cr\u00f3nica, aunque diagnosticada en el embarazo. Esto es as\u00ed porque ha sido diagnosticada a las 10 semanas de gestaci\u00f3n."}, "4": {"exist": true, "char_ranges": [[262, 532]], "word_ranges": [[43, 86]], "text": "Por encima de las 20 semanas de gestaci\u00f3n, ya hablar\u00edamos de preeclampsia (si HTA + proteinuria \u00f3 indice de pulsatilidad de arterias uterinas por encima del percentil 95), hipertensi\u00f3n inducida por el embarazo (s\u00f3lo HTA) \u00f3 eclampsia (es una forma de preeclampsia grave)."}, "5": {"exist": false, "char_ranges": [], "word_ranges": [], "text": ""}}} {"id": 94, "year": 2012, "question_id_specific": 31, "full_question": "Un muchacho de 17 a\u00f1os de edad con enfermedad de Crohn con afectaci\u00f3n col\u00f3nica de 2 a\u00f1os de evoluci\u00f3n, en tratamiento de mantenimiento con azatioprina, consulta por la aparici\u00f3n desde hace 5 d\u00edas de n\u00f3dulos subcut\u00e1neos rojo viol\u00e1ceos, calientes, dolorosos, de localizaci\u00f3n pretibial, bilaterales, asociada a un inrcemento del n\u00ba de deposiciones y dolor abdominal. La actitud m\u00e1s adecuada en este caso es:", "full_answer": "El cuadro que presenta el paciente es, presumiblemente, un eritema nodoso. Alrededor del 10% de casos de eritema nodoso se asocian con enfermedad inflamatoria intestinal, tanto colitis ulcerosa como enfermedad de Crohn. Como se ha mencionado, en la mayor\u00eda de casos, el eritema nodoso tiene un curso autolimitado. Cuando se asocia con enfermedad inflamatoria intestinal, el eritema nodoso habitualmente se resuelve con el tratamiento del brote intestinal, y recurre con las recidivas de la enfermedad. Las medidas locales incluyen la elevaci\u00f3n de las piernas y el reposo en cama. En casos particularmente intensos, la administraci\u00f3n por v\u00eda oral de yoduro pot\u00e1sico se ha demostrado \u00fatil. Aunque el uso de aspirina y otros AINE es eficaz para el eritema nodoso, su uso en pacientes con enfermedad inflamatoria intestinal debe limitarse, ya que pueden empeorar el cuadro intestinal, no se ha descrito la utilidad de antidepresivos en este cuadro, por lo que la respuesta n\u00ba1 no ser\u00eda correcta.", "type": "DERMATOLOG\u00cdA, VENEREOLOG\u00cdA Y CIRUG\u00cdA PL\u00c1STICA", "options": {"1": "Recomendar reposo relativo y pa\u00f1os calientes en ambas piernas y a\u00f1adir tratamiento antidepresivo.", "2": "Biopsiar zonas de piel alejados de las \u00e1reas lesionadas y pautar analg\u00e9sicos opioides de entrada.", "3": "Sospechar la existencia de una lesi\u00f3n tumoral maligna intestinal como desencadenante del proceso cut\u00e1neo.", "4": "Sospechar isquemia bilateral de extremidades inferiores de origen medicamentoso.", "5": "Ajuste del tratamiento de la enfermedad intestinal."}, "correct_option": 5, "explanations": {"1": {"exist": true, "char_ranges": [[688, 991]], "word_ranges": [[105, 156]], "text": "Aunque el uso de aspirina y otros AINE es eficaz para el eritema nodoso, su uso en pacientes con enfermedad inflamatoria intestinal debe limitarse, ya que pueden empeorar el cuadro intestinal, no se ha descrito la utilidad de antidepresivos en este cuadro, por lo que la respuesta n\u00ba1 no ser\u00eda correcta."}, "2": {"exist": false, "char_ranges": [], "word_ranges": [], "text": ""}, "3": {"exist": false, "char_ranges": [], "word_ranges": [], "text": ""}, "4": {"exist": false, "char_ranges": [], "word_ranges": [], "text": ""}, "5": {"exist": true, "char_ranges": [[314, 501]], "word_ranges": [[48, 75]], "text": "Cuando se asocia con enfermedad inflamatoria intestinal, el eritema nodoso habitualmente se resuelve con el tratamiento del brote intestinal, y recurre con las recidivas de la enfermedad."}}} {"id": 39, "year": 2011, "question_id_specific": 134, "full_question": "Mujer de 60 a\u00f1os, con diagn\u00f3stico de c\u00e1ncer de mama tratado con cirug\u00eda y radioterapia hace 10 a\u00f1os. Actualmente con met\u00e1stasis a nivel columna dorsal y articulaci\u00f3n sacroil\u00edaca por lo que durante los \u00faltimos dos a\u00f1os ha recibido tratamiento con \u00e1cido zoledr\u00f3nico i.v. de forma mensual. Acude a la consulta por presentar exposici\u00f3n \u00f3sea de forma espont\u00e1nea a nivel del hueso mandibular de aproximadamente 2 cm. No existen signos de infecci\u00f3n aguda y la enc\u00eda alrededor de la exposici\u00f3n \u00f3sea es estrictamente normal. El TAC mandibular evidencia una zona de esclerosis \u00f3sea. \u00bfCu\u00e1l ser\u00eda su diagn\u00f3stico?", "full_answer": "\u00c9sta me ha resultado dif\u00edcil, vamos a ir descartando. La 1 no puede ser, porque habr\u00eda signos infecciosos. La 2 es posible porque los bifosfonatos crean necrosis mandibular. La 3 me parece poco probable porque la radioterapia que recibi\u00f3 la paciente estar\u00eda orientada sobre la mama y no sobre la mand\u00edbula. La 4 la descarto, no es un cuadro de ra\u00edz dental inclu\u00edda. La 5 es posible; al fin y al cabo, la paciente ya tiene met\u00e1stasis \u00f3seas. As\u00ed pues, dudo entre dos, la 2 y la 5, \u00bfcu\u00e1l escoger? Desde luego no s\u00e9 bien qu\u00e9 significado tiene una esclerosis \u00f3sea en un TAC, pero una met\u00e1stasis en un hueso tendr\u00eda un comportamiento m\u00e1s destructivo que esclerotizante. Por otro lado, usando psicolog\u00eda inversa, \u00bfqu\u00e9 quer\u00eda el que puso la pregunta que yo supiera? \u00bfQue los bifosfonatos pueden crear necrosis mandibular? \u00bfO que una mujer con varias met\u00e1stasis \u00f3seas puede tener una met\u00e1stasis tambi\u00e9n en la mand\u00edbula? Lo segundo parece demasiado sencillo, y har\u00eda que me inclinara m\u00e1s hacia la 2, pero no lo tengo claro. Sea como sea, entre dos respuestas posibles hay que contestar siempre, y entre la 2 y la 5, hay que marcar la 2, que es m\u00e1s probable.", "type": "OTORRINOLARINGOLOG\u00cdA Y CIRUG\u00cdA MAXILOFACIAL", "options": {"1": "Absceso odont\u00f3geno.", "2": "Osteonecrosis por bifosfonatos.", "3": "Osteorradionecrosis.", "4": "Ra\u00edz dental inclu\u00edda.", "5": "Met\u00e1stasis de carcinoma de mama."}, "correct_option": 2, "explanations": {"1": {"exist": true, "char_ranges": [[54, 106]], "word_ranges": [[9, 18]], "text": "La 1 no puede ser, porque habr\u00eda signos infecciosos."}, "2": {"exist": true, "char_ranges": [[570, 663]], "word_ranges": [[104, 117]], "text": "pero una met\u00e1stasis en un hueso tendr\u00eda un comportamiento m\u00e1s destructivo que esclerotizante."}, "3": {"exist": true, "char_ranges": [[174, 306]], "word_ranges": [[28, 51]], "text": "La 3 me parece poco probable porque la radioterapia que recibi\u00f3 la paciente estar\u00eda orientada sobre la mama y no sobre la mand\u00edbula."}, "4": {"exist": true, "char_ranges": [[307, 439]], "word_ranges": [[51, 78]], "text": "La 4 la descarto, no es un cuadro de ra\u00edz dental inclu\u00edda. La 5 es posible; al fin y al cabo, la paciente ya tiene met\u00e1stasis \u00f3seas."}, "5": {"exist": true, "char_ranges": [[570, 663]], "word_ranges": [[104, 117]], "text": "pero una met\u00e1stasis en un hueso tendr\u00eda un comportamiento m\u00e1s destructivo que esclerotizante."}}} {"id": 252, "year": 2014, "question_id_specific": 83, "full_question": "\u00bfQu\u00e9 enfermedad sospechar\u00edamos en un paciente joven que refiere historia frecuente de disfagia a s\u00f3lidos y a l\u00edquidos con impactaciones de alimentos repetidas, sin cl\u00ednica de pirosis y en el que en la endoscopia oral se observan m\u00faltiples anillos esof\u00e1gicos conc\u00e9ntricos con mucosa normal (aspecto traquealizado)?", "full_answer": "El aspecto endosc\u00f3pico de es\u00f3fago traquealizado es muy t\u00edpico de la esofagitis eosinof\u00edlica, que cursa con cl\u00ednica de disfagia e impactaciones repetidas de alimentos.", "type": "APARATO DIGESTIVO", "options": {"1": "Esofagitis herp\u00e9tica.", "2": "Esofagitis eosinof\u00edlica.", "3": "Esofagitis candidi\u00e1sica.", "4": "Esofagitis por citomegalovirus.", "5": "Adenocarcinoma de es\u00f3fago."}, "correct_option": 2, "explanations": {"1": {"exist": false, "char_ranges": [], "word_ranges": [], "text": ""}, "2": {"exist": true, "char_ranges": [[0, 166]], "word_ranges": [[0, 24]], "text": "El aspecto endosc\u00f3pico de es\u00f3fago traquealizado es muy t\u00edpico de la esofagitis eosinof\u00edlica, que cursa con cl\u00ednica de disfagia e impactaciones repetidas de alimentos."}, "3": {"exist": false, "char_ranges": [], "word_ranges": [], "text": ""}, "4": {"exist": false, "char_ranges": [], "word_ranges": [], "text": ""}, "5": {"exist": false, "char_ranges": [], "word_ranges": [], "text": ""}}} {"id": 57, "year": 2011, "question_id_specific": 77, "full_question": "En una mujer obesa de 42 a\u00f1os se le realiza determinaci\u00f3n del cortisol en sangre, orina y saliva y en todas las determinaciones est\u00e1 elevado. Tras la realizaci\u00f3n del test de supresi\u00f3n nocturna con dexametasona, los niveles de cortisol en sangre permanecen elevados, por lo que se sospecha s\u00edndrome de Cushing ACTH independiente, \u00bfqu\u00e9 prueba adicional solicitar\u00eda para confirmar el diagn\u00f3stico en este momento?", "full_answer": "De todas formas creo que est\u00e1 mal planteado el caso cl\u00ednico, ya que antes de hacer el TAC se deber\u00eda medir la ACTH y, una vez que la ACTH est\u00e9 suprimida hacer el TAC. Pero como la pregunta es \u00abqu\u00e9 har\u00eda para confirmar el diagn\u00f3stico de sospecha de un s\u00edndrome Cushing ACTH independiente\u00bb, la respuesta es el TAC.", "type": "ENDOCRINOLOG\u00cdA", "options": {"1": "Un cateterismo de senos petrosos.", "2": "Una gammagraf\u00eda con somatostatina marcada (Octreoscan).", "3": "Una gammagraf\u00eda con Sesta-MIBI.", "4": "Una ecograf\u00eda abdominal.", "5": "Una TC de suprarrenales."}, "correct_option": 5, "explanations": {"1": {"exist": false, "char_ranges": [], "word_ranges": [], "text": ""}, "2": {"exist": false, "char_ranges": [], "word_ranges": [], "text": ""}, "3": {"exist": false, "char_ranges": [], "word_ranges": [], "text": ""}, "4": {"exist": false, "char_ranges": [], "word_ranges": [], "text": ""}, "5": {"exist": false, "char_ranges": [], "word_ranges": [], "text": ""}}} {"id": 595, "year": 2022, "question_id_specific": 88, "full_question": "Ni\u00f1o de 23 meses que presenta fiebre, hipotensi\u00f3n, v\u00f3mitos repetidos, signos men\u00edngeos, petequias y equimosis. El hemograma muestra 25.000 leucocitos (75 % neutr\u00f3filos, 10 % cayados) y 12.000 plaquetas/microL y el estudio de coagulaci\u00f3n una actividad de protrombina de 35 %. \u00bfQu\u00e9 opci\u00f3n NO es adecuada en su manejo?:", "full_answer": "Paciente con sospecha de sepsis meningoc\u00f3cica. Es primordial su estabilizaci\u00f3n con fluidos (hipotensi\u00f3n) seg\u00fan el ABCDE. El hemocultivo puede obtenerse junto con la anal\u00edtica inicial. Es primordial iniciar antibioterapia de forma precoz (cefotaxima). Sin embargo, en un paciente inestable como el nuestro no est\u00e1 indicada la realizaci\u00f3n de punci\u00f3n lumbar.", "type": "PEDIATR\u00cdA", "options": {"1": "Administraci\u00f3n de cefotaxima intravenosa.", "2": "Realizaci\u00f3n de punci\u00f3n lumbar.", "3": "Recogida de hemocultivo.", "4": "Fluidos intravenosos.", "5": NaN}, "correct_option": 2, "explanations": {"1": {"exist": true, "char_ranges": [[184, 250]], "word_ranges": [[25, 33]], "text": "Es primordial iniciar antibioterapia de forma precoz (cefotaxima)."}, "2": {"exist": true, "char_ranges": [[251, 355]], "word_ranges": [[33, 50]], "text": "Sin embargo, en un paciente inestable como el nuestro no est\u00e1 indicada la realizaci\u00f3n de punci\u00f3n lumbar."}, "3": {"exist": true, "char_ranges": [[121, 183]], "word_ranges": [[16, 25]], "text": "El hemocultivo puede obtenerse junto con la anal\u00edtica inicial."}, "4": {"exist": true, "char_ranges": [[0, 120]], "word_ranges": [[0, 16]], "text": "Paciente con sospecha de sepsis meningoc\u00f3cica. Es primordial su estabilizaci\u00f3n con fluidos (hipotensi\u00f3n) seg\u00fan el ABCDE."}, "5": {"exist": false, "char_ranges": [], "word_ranges": [], "text": ""}}} {"id": 98, "year": 2012, "question_id_specific": 171, "full_question": "Paciente que presenta desde hace 2 semanas una erupci\u00f3n pruriginosa constituida por m\u00faltiples p\u00e1pulas eritemato-viol\u00e1ceas poligonales, de superficie aplanada, localizadas preferentemente en la cara anterior de mu\u00f1ecas, zona pretibial y \u00e1rea lumbar. Adem\u00e1s presenta lesiones reticuladas blanquecinas en mucosa yugal, \u00bfCu\u00e1l es el diagn\u00f3stico m\u00e1s probable?", "full_answer": "El liquen plano es una dermatosis muy frecuente de etiolog\u00eda desconocida que se caracteriza por la aparici\u00f3n de p\u00e1pulas poligonales pruriginosas, de color viol\u00e1ceo y superficie brillante, que se localizan de manera preferente en las \u00e1reas distales de las extremidades. Caracter\u00edsticamente estas p\u00e1pulas muestran una distribuci\u00f3n sim\u00e9trica y se localizan preferentemente en la cara anterior de mu\u00f1ecas (Figura 1) y tobillos. Respecto a la afectaci\u00f3n de la mucosa bucal es muy frecuente, y aproximadamente el 75% de los pacientes con liquen plano muestran un reticulado blanquecino asintom\u00e1tico afectando a la mucosa yugal o lingual.", "type": "DERMATOLOG\u00cdA, VENEREOLOG\u00cdA Y CIRUG\u00cdA PL\u00c1STICA", "options": {"1": "Pitiriasis versicolor.", "2": "Micosis fungoides.", "3": "Psoriasis en gotas.", "4": "Pitiriasis rosada de Gibert.", "5": "Liquen plano."}, "correct_option": 5, "explanations": {"1": {"exist": false, "char_ranges": [], "word_ranges": [], "text": ""}, "2": {"exist": false, "char_ranges": [], "word_ranges": [], "text": ""}, "3": {"exist": false, "char_ranges": [], "word_ranges": [], "text": ""}, "4": {"exist": false, "char_ranges": [], "word_ranges": [], "text": ""}, "5": {"exist": true, "char_ranges": [[0, 423]], "word_ranges": [[0, 61]], "text": "El liquen plano es una dermatosis muy frecuente de etiolog\u00eda desconocida que se caracteriza por la aparici\u00f3n de p\u00e1pulas poligonales pruriginosas, de color viol\u00e1ceo y superficie brillante, que se localizan de manera preferente en las \u00e1reas distales de las extremidades. Caracter\u00edsticamente estas p\u00e1pulas muestran una distribuci\u00f3n sim\u00e9trica y se localizan preferentemente en la cara anterior de mu\u00f1ecas (Figura 1) y tobillos."}}} {"id": 118, "year": 2012, "question_id_specific": 117, "full_question": "A un joven asintotom\u00e1tico de 14 a\u00f1os cuyo padre acaba de ser diagnostica de TBC pulmonar se le realiza un Mantoux, con resultado negativo. \u00bfQu\u00e9 actitud es la adecuada?", "full_answer": "En los j\u00f3venes que sean contactos, se iniciar\u00e1 quimioprofilaxis aunque el mantoux sea negativo y se repetir\u00e1 a los 2-3 meses para decidir si se sigue tratando hasta completar 6 meses (si vira a positivo) o puede dejarla.", "type": "INFECTOLOG\u00cdA", "options": {"1": "Tranquilizarle, ya que no tiene ning\u00fan riesgo.", "2": "Realizarle una radiograf\u00eda de t\u00f3rax.", "3": "Iniciar quimioprofilaxis y repetir la prueba cut\u00e1nea a los 3 meses.", "4": "Repetir la prueba cut\u00e1nea al mes.", "5": "Habr\u00eda que realizar el estudio de esputo antes de iniciar quimioprofilaxis con Isoniazida durante 1 a\u00f1o."}, "correct_option": 3, "explanations": {"1": {"exist": false, "char_ranges": [], "word_ranges": [], "text": ""}, "2": {"exist": false, "char_ranges": [], "word_ranges": [], "text": ""}, "3": {"exist": true, "char_ranges": [[0, 220]], "word_ranges": [[0, 38]], "text": "En los j\u00f3venes que sean contactos, se iniciar\u00e1 quimioprofilaxis aunque el mantoux sea negativo y se repetir\u00e1 a los 2-3 meses para decidir si se sigue tratando hasta completar 6 meses (si vira a positivo) o puede dejarla."}, "4": {"exist": false, "char_ranges": [], "word_ranges": [], "text": ""}, "5": {"exist": false, "char_ranges": [], "word_ranges": [], "text": ""}}} {"id": 388, "year": 2016, "question_id_specific": 233, "full_question": "Acude a consultas de pediatr\u00eda un ni\u00f1o de 13 a\u00f1os de edad por presentar sensaci\u00f3n de pirosis durante y tras las comidas y disfagia ocasional, que aumenta con la ingesta de alimentos s\u00f3lidos como a carne. En los antecedentes familiares destacar que su madre est\u00e1 diagnosticada de hernia hiatal. En los antecedentes personales nos refiere alergia a p\u00f3lenes, \u00e1caros y l\u00e1tex. En la exploraci\u00f3n f\u00edsica, no muestra hallazgos significativos salvo lesiones eccematosas en huecos popl\u00edteos y antecubitales. \u00bfCu\u00e1l de las siguientes pruebas complementarias NO realizar\u00eda de acuerdo a los diagn\u00f3sticos m\u00e1s probables?", "full_answer": "Nos est\u00e1n presentando el caso de una esofagitis eosinof\u00edlica (adolescente var\u00f3n, con pirosis y disfagia, y antecedentes de dermatitis at\u00f3pica). De todas las pruebas que nos proponen, la menos indicada en este caso ser\u00eda la opci\u00f3n 3.", "type": "PEDIATR\u00cdA", "options": {"1": "Endoscopia digestiva superior.", "2": "pHmetr\u00eda esof\u00e1gica de 24 horas.", "3": "Test de aliento con carbono 13 para Helicobacter pylori.", "4": "Estudio baritado es\u00f3fago-gastro-duodenal.", "5": NaN}, "correct_option": 3, "explanations": {"1": {"exist": false, "char_ranges": [], "word_ranges": [], "text": ""}, "2": {"exist": false, "char_ranges": [], "word_ranges": [], "text": ""}, "3": {"exist": true, "char_ranges": [[0, 232]], "word_ranges": [[0, 37]], "text": "Nos est\u00e1n presentando el caso de una esofagitis eosinof\u00edlica (adolescente var\u00f3n, con pirosis y disfagia, y antecedentes de dermatitis at\u00f3pica). De todas las pruebas que nos proponen, la menos indicada en este caso ser\u00eda la opci\u00f3n 3."}, "4": {"exist": false, "char_ranges": [], "word_ranges": [], "text": ""}, "5": {"exist": false, "char_ranges": [], "word_ranges": [], "text": ""}}} {"id": 421, "year": 2018, "question_id_specific": 78, "full_question": "Hombre de 52 a\u00f1os de edad derivado al servicio de digestivo por cuadro de hematoquezia, tenesmo y reducci\u00f3n del di\u00e1metro de las heces. Se realizan una serie de pruebas, diagnosticandose un adenocarcinoma de sigma sin metastasis a distancia. El paciente es intervenido quir\u00fargicamente y remitido a la consulta de oncologia m\u00e9dica para valorar tratamiento quimioterapico complementario. \u00bfCual de los siguientes factores es de mal pron\u00f3stico tras la resecci\u00f3n quir\u00fargica y habra que tener en cuenta a la hora de planificar el tratamiento de quimioterapia?", "full_answer": "En esta ocasi\u00f3n nos preguntan acerca de que factores tienen mayor influencia para llevar a cabo quimioterapia adyuvante en el c\u00e1ncer colorrectal. Para ello debemos conocer cuales son los factores de mal pron\u00f3stico en esta patolog\u00eda donde destacan la obstrucci\u00f3n o perforaci\u00f3n intestinal debida a la lesi\u00f3n, la invasi\u00f3n de \u00f3rganos adyancentes, elevaci\u00f3n de marcadores tumorales como CEA y la histolog\u00eda que muestra la neoplasia. Entre ellos no se otorga tanta importancia al tama\u00f1o de la lesi\u00f3n como a su grado de invasi\u00f3n de la pared intestinal u otras estructuras adyacentes, criterio que se debe tener en cuenta para realizar el estadiaje mediante la clasificaci\u00f3n TNM o la de Astler-Coller que guian la indicaci\u00f3n de tratamiento adyuvante tras el abordaje quir\u00fargico.", "type": "CIRUG\u00cdA GENERAL", "options": {"1": "La presencia de anemia al diagn\u00f3stico.", "2": "La existencia de antecedentes familiares de cancer colorrectal.", "3": "El tama\u00f1o de la lesi\u00f3n primaria y la diferenciaci\u00f3n histol\u00f3gica.", "4": "La perforaci\u00f3n o adhesi\u00f3n del tumor a \u00f3rganos adyacentes.", "5": NaN}, "correct_option": 4, "explanations": {"1": {"exist": false, "char_ranges": [], "word_ranges": [], "text": ""}, "2": {"exist": false, "char_ranges": [], "word_ranges": [], "text": ""}, "3": {"exist": true, "char_ranges": [[440, 493]], "word_ranges": [[67, 77]], "text": "no se otorga tanta importancia al tama\u00f1o de la lesi\u00f3n"}, "4": {"exist": true, "char_ranges": [[16, 305]], "word_ranges": [[3, 47]], "text": "nos preguntan acerca de que factores tienen mayor influencia para llevar a cabo quimioterapia adyuvante en el c\u00e1ncer colorrectal. Para ello debemos conocer cuales son los factores de mal pron\u00f3stico en esta patolog\u00eda donde destacan la obstrucci\u00f3n o perforaci\u00f3n intestinal debida a la lesi\u00f3n,"}, "5": {"exist": false, "char_ranges": [], "word_ranges": [], "text": ""}}} {"id": 500, "year": 2020, "question_id_specific": 45, "full_question": "Mujer de 15 a\u00f1os que presenta un retraso en la menarquia y una talla baja. No tiene discapacidad intelectual. \u00bfCu\u00e1l de las siguientes pruebas gen\u00e9ticas se utilizar\u00eda habitualmente para el diagn\u00f3stico de esta paciente?:", "full_answer": "En una mujer con pubertad retrasada y talla baja debemos pensar y descartar siempre el S\u00edndrome de Turner, cuyo diagn\u00f3stico se realiza mediante cariotipo (45X0).", "type": "PEDIATR\u00cdA", "options": {"1": "Secuenciaci\u00f3n masiva (NGS).", "2": "FISH.", "3": "Microarrays de ADN y/o de ARN.", "4": "Cariotipo.", "5": NaN}, "correct_option": 4, "explanations": {"1": {"exist": false, "char_ranges": [], "word_ranges": [], "text": ""}, "2": {"exist": false, "char_ranges": [], "word_ranges": [], "text": ""}, "3": {"exist": false, "char_ranges": [], "word_ranges": [], "text": ""}, "4": {"exist": true, "char_ranges": [[0, 161]], "word_ranges": [[0, 25]], "text": "En una mujer con pubertad retrasada y talla baja debemos pensar y descartar siempre el S\u00edndrome de Turner, cuyo diagn\u00f3stico se realiza mediante cariotipo (45X0)."}, "5": {"exist": false, "char_ranges": [], "word_ranges": [], "text": ""}}} {"id": 611, "year": 2022, "question_id_specific": 116, "full_question": "Mujer de 95 a\u00f1os que vive en una residencia, independiente para sus actividades b\u00e1sicas de la vida diaria, sale al jard\u00edn a pasear. Tiene antecedentes de HTA, dislipidemia, osteoporosis y deterioro cognitivo leve. Sufre una ca\u00edda al levantarse por la noche al ba\u00f1o. La radiograf\u00eda muestra una fractura subcapital desplazada de cadera derecha. \u00bfCu\u00e1l es el tratamiento recomendado?:", "full_answer": "Nos hablan de un caso de una paciente de 95 a\u00f1os con fractura de cadera intracapsular (subcapital) desplazada. En este caso, las opciones quir\u00fargicas pueden ser el uso de tornillos canulados o artroplastia de cadera. Los tornillos canulados se reservan para casos de fracturas no desplazadas (Respuesta 2 incorrecta) por o que la opci\u00f3n m\u00e1s indicada ser\u00eda la artoplastia, en este caso hemiartrioplastia (Respuesta 1 correcta). Las clavos trocant\u00e9ricos se reservan para las fracturas extracapsulares (Respuesta 2 falsa) y el tratamiento conservador \u00fanicamente se plantea en pacientes con muy baja demanda funcional o contraindicaci\u00f3n quir\u00fargica (Respuesta 4 incorrecta).", "type": "TRAUMATOLOG\u00cdA", "options": {"1": "Hemiartroplastia de cadera.", "2": "Fijaci\u00f3n con clavo trocant\u00e9rico.", "3": "Fijaci\u00f3n con tornillos canulados.", "4": "Conservador: vida cama-sill\u00f3n.", "5": NaN}, "correct_option": 1, "explanations": {"1": {"exist": true, "char_ranges": [[217, 426]], "word_ranges": [[35, 66]], "text": "Los tornillos canulados se reservan para casos de fracturas no desplazadas (Respuesta 2 incorrecta) por o que la opci\u00f3n m\u00e1s indicada ser\u00eda la artoplastia, en este caso hemiartrioplastia (Respuesta 1 correcta)."}, "2": {"exist": true, "char_ranges": [[217, 518]], "word_ranges": [[35, 78]], "text": "Los tornillos canulados se reservan para casos de fracturas no desplazadas (Respuesta 2 incorrecta) por o que la opci\u00f3n m\u00e1s indicada ser\u00eda la artoplastia, en este caso hemiartrioplastia (Respuesta 1 correcta). Las clavos trocant\u00e9ricos se reservan para las fracturas extracapsulares (Respuesta 2 falsa)"}, "3": {"exist": false, "char_ranges": [], "word_ranges": [], "text": ""}, "4": {"exist": true, "char_ranges": [[521, 669]], "word_ranges": [[79, 98]], "text": "el tratamiento conservador \u00fanicamente se plantea en pacientes con muy baja demanda funcional o contraindicaci\u00f3n quir\u00fargica (Respuesta 4 incorrecta)."}, "5": {"exist": false, "char_ranges": [], "word_ranges": [], "text": ""}}} {"id": 542, "year": 2022, "question_id_specific": 38, "full_question": "Mujer de 25 a\u00f1os con antecedentes de asma bronquial al\u00e9rgica por sensibilizaci\u00f3n a p\u00f3lenes de gram\u00edneas y dermatitis at\u00f3pica. Presenta de forma brusca hipotensi\u00f3n, taquicardia, urticaria y broncoespasmo tras ingerir cacahuetes y es trasladada a urgencias. \u00bfCu\u00e1l ser\u00eda el orden de los f\u00e1rmacos a administrar y qu\u00e9 prueba de laboratorio podr\u00eda solicitarse para ser evaluada posteriormente?:", "full_answer": "La paciente est\u00e1 sufriendo un shock anafil\u00e1ctico secundario a la ingesta de cacahuetes. El tratamiento de la anafilaxia y el shock anafil\u00e1ctico de primera elecci\u00f3n es la adrenalina intramuscular 1mg/ml (dosis de 0,01 mg/kg, en adultos 0,5 mg m\u00e1ximo). Seguido de tratamiento sintom\u00e1tico para estabilizar la v\u00eda respiratoria con O2 100% alto flujo, asegurar accesos venosos de grueso calibre, reposici\u00f3n de fluidos y monitorizaci\u00f3n continua (FC, TA, SatO2 , diuresis). Como tratamiento adyuvante se puede administrar un broncodilatador inhalado tipo SABA, dexclorfeniramina intramuscular o corticoides como Hidrocortisona IV o Metilprednisolona IV. La adrenalina se puede repetir a las mismas dosis cada 5 minutos un m\u00e1ximo de 3 veces y si a los 15 minutos no responde se deber\u00eda iniciar perfusi\u00f3n adrenalina IV, plantear Glucag\u00f3n (si tratamiento con bloqueantes beta), Atropina (si bradicardia prolongada) o Vasopresores (si hipotensi\u00f3n refractaria). El marcador de laboratorio que se debe solicitar si se sospecha una anafilaxia es la Triptasa s\u00e9rica en la primera hora para una determinaci\u00f3n basal y otra a las 4-6 horas para observar su curva de elevaci\u00f3n. No es un marcador de urgencias pero es muy \u00fatil de cara al estudio alergol\u00f3gico futuro.", "type": "ALERGOLOG\u00cdA", "options": {"1": "Adrenalina intramuscular, seguida de un broncodilatador inhalado tipo SABA y dexclorfeniramina intramuscular. A continuaci\u00f3n solicitar triptasa s\u00e9rica.", "2": "Un broncodilatador inhalado tipo SABA, seguido de dexclorfeniramina intramuscular y adrenalina intramuscular. A continuaci\u00f3n solicitar triptasa s\u00e9rica.", "3": "Dexclorfeniramina intramuscular, seguida de un broncodilatador inhalado tipo SABA y adrenalina intramuscular. A continuaci\u00f3n solicitar prote\u00edna cati\u00f3nica del eosin\u00f3filo.", "4": "Adrenalina intramuscular, seguida de un broncodilatador inhalado tipo SABA y dexclorfeniramina intramuscular. A continuaci\u00f3n solicitar prote\u00edna cati\u00f3nica del eosin\u00f3filo.", "5": NaN}, "correct_option": 1, "explanations": {"1": {"exist": true, "char_ranges": [[950, 1158]], "word_ranges": [[138, 175]], "text": "El marcador de laboratorio que se debe solicitar si se sospecha una anafilaxia es la Triptasa s\u00e9rica en la primera hora para una determinaci\u00f3n basal y otra a las 4-6 horas para observar su curva de elevaci\u00f3n."}, "2": {"exist": false, "char_ranges": [], "word_ranges": [], "text": ""}, "3": {"exist": false, "char_ranges": [], "word_ranges": [], "text": ""}, "4": {"exist": true, "char_ranges": [[950, 1158]], "word_ranges": [[138, 175]], "text": "El marcador de laboratorio que se debe solicitar si se sospecha una anafilaxia es la Triptasa s\u00e9rica en la primera hora para una determinaci\u00f3n basal y otra a las 4-6 horas para observar su curva de elevaci\u00f3n."}, "5": {"exist": false, "char_ranges": [], "word_ranges": [], "text": ""}}} {"id": 79, "year": 2012, "question_id_specific": 36, "full_question": "Cu\u00e1l de los siguientes datos cl\u00ednicos, registrados durante la realizaci\u00f3n de la historia cl\u00ednica de un paciente con diarrea de m\u00e1s de 4 semanas de evoluci\u00f3n, puede ser encontrado en personas sin patolog\u00eda org\u00e1nica:", "full_answer": "Aunque el moco suele asustar algo m\u00e1s en la consulta, entra dentro de los criterios de Roma II para el diagn\u00f3stico de S\u00edndrome de Intestino Irritable. Todos los dem\u00e1s son signos de organicidad t\u00edpicos, aunque creo que a pregunta est\u00e1 mal enunciada, ya que cualquiera de esos datos (sobre todo la edad) se pueden encontra en pacientes que al final no tienen nada, lo que pasa es que en esos casos hay que buscar organicidad siempre.", "type": "APARATO DIGESTIVO", "options": {"1": "P\u00e9rdida de peso con o sin rectorragia.", "2": "Persistencia de la diarrea durante la noche.", "3": "Inicio de los s\u00edntomas despu\u00e9s de los 50 a\u00f1os.", "4": "Presencia durante la exploraci\u00f3n de dermatitis y/o artritis.", "5": "Expulsi\u00f3n de moco en m\u00e1s del 25% de las defecaciones."}, "correct_option": 5, "explanations": {"1": {"exist": true, "char_ranges": [[151, 200]], "word_ranges": [[26, 34]], "text": "Todos los dem\u00e1s son signos de organicidad t\u00edpicos,"}, "2": {"exist": true, "char_ranges": [[151, 200]], "word_ranges": [[26, 34]], "text": "Todos los dem\u00e1s son signos de organicidad t\u00edpicos,"}, "3": {"exist": true, "char_ranges": [[151, 200]], "word_ranges": [[26, 34]], "text": "Todos los dem\u00e1s son signos de organicidad t\u00edpicos,"}, "4": {"exist": true, "char_ranges": [[151, 200]], "word_ranges": [[26, 34]], "text": "Todos los dem\u00e1s son signos de organicidad t\u00edpicos,"}, "5": {"exist": true, "char_ranges": [[0, 150]], "word_ranges": [[0, 26]], "text": "Aunque el moco suele asustar algo m\u00e1s en la consulta, entra dentro de los criterios de Roma II para el diagn\u00f3stico de S\u00edndrome de Intestino Irritable."}}} {"id": 524, "year": 2021, "question_id_specific": 54, "full_question": "Mujer de 86 a\u00f1os de edad en quien se ha detectado una fibrilaci\u00f3n auricular no valvular. Tiene una puntuaci\u00f3n CHADS2 de 3 puntos. En la bibliograf\u00eda, pacientes similares en tratamiento con warfarina tienen un riesgo de ictus del 2,2 % frente al 5,2 % en los pacientes sin warfarina. \u00bfCu\u00e1l ser\u00eda el n\u00famero necesario a tratar (NNT) para prevenir un ictus embol\u00edgeno con el tratamiento anticoagulante?:", "full_answer": "De media, si se tratan 100 pacientes como esta mujer con warfarina, se reduce en 3 el n\u00famero de ictus (pasa de 5.2 sin tratar a 3.2 tratando). Por lo tanto, si se tratan 33.3 pacientes como esta mujer con warfarina, se reduce en 1 el n\u00famero de ictus: el NNT es 33.3", "type": "ESTAD\u00cdSTICA", "options": {"1": "3.", "2": "19,2.", "3": "33,3.", "4": "49,5.", "5": NaN}, "correct_option": 3, "explanations": {"1": {"exist": false, "char_ranges": [], "word_ranges": [], "text": ""}, "2": {"exist": false, "char_ranges": [], "word_ranges": [], "text": ""}, "3": {"exist": true, "char_ranges": [[0, 265]], "word_ranges": [[0, 53]], "text": "De media, si se tratan 100 pacientes como esta mujer con warfarina, se reduce en 3 el n\u00famero de ictus (pasa de 5.2 sin tratar a 3.2 tratando). Por lo tanto, si se tratan 33.3 pacientes como esta mujer con warfarina, se reduce en 1 el n\u00famero de ictus: el NNT es 33.3"}, "4": {"exist": false, "char_ranges": [], "word_ranges": [], "text": ""}, "5": {"exist": false, "char_ranges": [], "word_ranges": [], "text": ""}}} {"id": 319, "year": 2016, "question_id_specific": 141, "full_question": "Chico de 20 a\u00f1os, que consulta por dolor lumbosacro de ritmo inflamatorio de 4 meses de evoluci\u00f3n. Tambi\u00e9n talalgia bilateral y rigidez matutina de 1 hora. En los \u00faltimos 2 meses aparici\u00f3n de cuadros diarreicos con p\u00e9rdida de 4 kg de peso. \u00bfCu\u00e1l es la aproximaci\u00f3n diagn\u00f3stica m\u00e1s correcta?", "full_answer": "El cuadro cl\u00ednico es el t\u00edpico de una espondiloartritis, y junto con la cl\u00ednica digestiva seguramente sea una asociada a enfermedad inflamatoria intestinal.", "type": "REUMATOLOG\u00cdA", "options": {"1": "Dada la edad del paciente, lo m\u00e1s probable es que padezca lumbalgia inespec\u00edfica y una tendinitis en los pies. Si persisten las diarreas realizar\u00eda estudio digestivo.", "2": "Realizar\u00eda estudio digestivo para descartar patolog\u00eda tumoral. El dolor lumbar puede ser debido a patolog\u00eda visceral.", "3": "El cuadro cl\u00ednico es muy sugestivo de espondiloartritis. Habr\u00eda que descartar enfermedad inflamatoria intestinal.", "4": "Solicitar\u00eda RNM lumbar para descartar hernia discal y si persiste diarrea, estudio digestivo.", "5": NaN}, "correct_option": 3, "explanations": {"1": {"exist": false, "char_ranges": [], "word_ranges": [], "text": ""}, "2": {"exist": false, "char_ranges": [], "word_ranges": [], "text": ""}, "3": {"exist": true, "char_ranges": [[0, 156]], "word_ranges": [[0, 23]], "text": "El cuadro cl\u00ednico es el t\u00edpico de una espondiloartritis, y junto con la cl\u00ednica digestiva seguramente sea una asociada a enfermedad inflamatoria intestinal."}, "4": {"exist": false, "char_ranges": [], "word_ranges": [], "text": ""}, "5": {"exist": false, "char_ranges": [], "word_ranges": [], "text": ""}}} {"id": 400, "year": 2016, "question_id_specific": 147, "full_question": "Un ni\u00f1o de 13 a\u00f1os presenta dolor dorsal de varios meses de evoluci\u00f3n, s\u00f3lo a la bipedestaci\u00f3n y a la marcha, \u00bfQu\u00e9 diagn\u00f3stico debemos de pensar?", "full_answer": "El dolor es de caracter\u00edsticas mec\u00e1nicas, y la causa m\u00e1s frecuente de dolor mec\u00e1nico de espalda en adolescentes es la sobrecarga mec\u00e1nica. No hay datos en el enunciado que orienten hacia ninguna de las otras etiolog\u00edas.", "type": "TRAUMATOLOG\u00cdA Y ORTOPED\u00cdA", "options": {"1": "Enfermedad de Scheuermann.", "2": "Escoliosis tor\u00e1cica.", "3": "Tumor maligno del cuerpo vertebral.", "4": "Se trata de una sobrecarga mec\u00e1nica.", "5": NaN}, "correct_option": 4, "explanations": {"1": {"exist": true, "char_ranges": [[139, 219]], "word_ranges": [[22, 36]], "text": "No hay datos en el enunciado que orienten hacia ninguna de las otras etiolog\u00edas."}, "2": {"exist": true, "char_ranges": [[139, 219]], "word_ranges": [[22, 36]], "text": "No hay datos en el enunciado que orienten hacia ninguna de las otras etiolog\u00edas."}, "3": {"exist": true, "char_ranges": [[139, 219]], "word_ranges": [[22, 36]], "text": "No hay datos en el enunciado que orienten hacia ninguna de las otras etiolog\u00edas."}, "4": {"exist": true, "char_ranges": [[0, 138]], "word_ranges": [[0, 22]], "text": "El dolor es de caracter\u00edsticas mec\u00e1nicas, y la causa m\u00e1s frecuente de dolor mec\u00e1nico de espalda en adolescentes es la sobrecarga mec\u00e1nica."}, "5": {"exist": false, "char_ranges": [], "word_ranges": [], "text": ""}}} {"id": 196, "year": 2013, "question_id_specific": 164, "full_question": "Ni\u00f1o de 4 a\u00f1os de edad que consulta por aparici\u00f3n a lo largo de los \u00faltimos tres d\u00edas, de lesiones cut\u00e1neas en piernas y gl\u00fateos. Sus padres refer\u00edan que su pediatra le hab\u00eda diagnosticado una infecci\u00f3n de v\u00edas respiratorias superiores 10 d\u00edas antes. En las \u00faltimas 12 horas presenta dolor abdominal intenso tipo c\u00f3lico, y ha realizado dos deposiciones diarreicas. Afebril. No p\u00e9rdida de peso. A la exploraci\u00f3n presenta numerosas petequias y lesiones purp\u00faricas palpables de predominio en gl\u00fateos y extremidades inferiores. Buen estado general aunque tiene dolor abdominal intenso, La palpaci\u00f3n abdominal es dif\u00edcil de valorar por dolor difuso. No visceromegalias. No otros hallazgos de inter\u00e9s a la exploraci\u00f3n. \u00bfCu\u00e1l de los siguientes datos NO apoya su sospecha diagn\u00f3stica?", "full_answer": "Est\u00e1 describiendo una p\u00farpura de Sch\u00f6nlein-Henoch que es un fen\u00f3meno vascul\u00edtico, de forma que las lesiones no son debidas a plaquetopenia. El edema escrotal (A veces incluso torsi\u00f3n de c\u00e1pside de Morgani del test\u00edculo) no es excepcional en varones con S Sch\u00f6nlein-Henoch.", "type": "PEDIATR\u00cdA", "options": {"1": "Artritis de rodillas y tobillos.", "2": "Hematuria.", "3": "Plaquetopenia.", "4": "Sangre oculta en heces.", "5": "Edema escrotal."}, "correct_option": 3, "explanations": {"1": {"exist": false, "char_ranges": [], "word_ranges": [], "text": ""}, "2": {"exist": false, "char_ranges": [], "word_ranges": [], "text": ""}, "3": {"exist": false, "char_ranges": [], "word_ranges": [], "text": ""}, "4": {"exist": false, "char_ranges": [], "word_ranges": [], "text": ""}, "5": {"exist": true, "char_ranges": [[0, 272]], "word_ranges": [[0, 42]], "text": "Est\u00e1 describiendo una p\u00farpura de Sch\u00f6nlein-Henoch que es un fen\u00f3meno vascul\u00edtico, de forma que las lesiones no son debidas a plaquetopenia. El edema escrotal (A veces incluso torsi\u00f3n de c\u00e1pside de Morgani del test\u00edculo) no es excepcional en varones con S Sch\u00f6nlein-Henoch."}}} {"id": 509, "year": 2021, "question_id_specific": 150, "full_question": "Var\u00f3n de 83 a\u00f1os que consulta por un cuadro recurrente de dolor abdominal en fosa iliaca izquierda, acompa\u00f1ado de diarrea de hasta 6 deposiciones al d\u00eda sin productos patol\u00f3gicos, de 10-15 d\u00edas de evoluci\u00f3n. No refiere fiebre ni afectaci\u00f3n del estado general. En una anal\u00edtica reciente no se observa leucocitosis y presenta una sangre oculta en heces negativa. En la exploraci\u00f3n se observa leve dolor a la palpaci\u00f3n de fosa iliaca izquierda sin irritaci\u00f3n peritoneal. Tiene realizada una colonoscopia hace tres a\u00f1os en la que se informa de divert\u00edculos a lo largo de todo el colon, m\u00e1s numerosos en colon izquierdo, sin otras lesiones asociadas. De las siguientes, \u00bfcu\u00e1l ser\u00eda la mejor actitud a seguir?:", "full_answer": "Hacen referencia a un concepto que ha surgido hace relativamente poco y que no est\u00e1 presente en todas las gu\u00edas cl\u00ednicas como es la enfermedad diverticular sintom\u00e1tica no complicada. En estos casos, se recomienda el ensayo terap\u00e9utico con rifaximina, siendo la verdadera la n\u00famero 1. La ausencia de reactantes de fase aguda en la anal\u00edtica nos orienta a que no presenta una diverticulitis aguda en el momento de la consulta que nos haga solicitar una prueba de imagen para descartarla.", "type": "DIGESTIVO", "options": {"1": "Realizar un tratamiento emp\u00edrico con antibi\u00f3ticos.", "2": "Solicitar una colonoscopia preferente.", "3": "Solicitar una ecograf\u00eda abdominal urgente.", "4": "Derivar a cirug\u00eda para valorar sigmoidectom\u00eda.", "5": NaN}, "correct_option": 1, "explanations": {"1": {"exist": true, "char_ranges": [[0, 283]], "word_ranges": [[0, 45]], "text": "Hacen referencia a un concepto que ha surgido hace relativamente poco y que no est\u00e1 presente en todas las gu\u00edas cl\u00ednicas como es la enfermedad diverticular sintom\u00e1tica no complicada. En estos casos, se recomienda el ensayo terap\u00e9utico con rifaximina, siendo la verdadera la n\u00famero 1."}, "2": {"exist": true, "char_ranges": [[284, 485]], "word_ranges": [[45, 80]], "text": "La ausencia de reactantes de fase aguda en la anal\u00edtica nos orienta a que no presenta una diverticulitis aguda en el momento de la consulta que nos haga solicitar una prueba de imagen para descartarla."}, "3": {"exist": true, "char_ranges": [[284, 485]], "word_ranges": [[45, 80]], "text": "La ausencia de reactantes de fase aguda en la anal\u00edtica nos orienta a que no presenta una diverticulitis aguda en el momento de la consulta que nos haga solicitar una prueba de imagen para descartarla."}, "4": {"exist": false, "char_ranges": [], "word_ranges": [], "text": ""}, "5": {"exist": false, "char_ranges": [], "word_ranges": [], "text": ""}}} {"id": 549, "year": 2022, "question_id_specific": 128, "full_question": "Mujer de 75 a\u00f1os con antecedentes de insuficiencia cardiaca con fracci\u00f3n de eyecci\u00f3n del 25% que acude a revisi\u00f3n, encontr\u00e1ndose estable en clase funcional III de la NYHA. Tiene un desfibrilador cardioversor implantado. La medicaci\u00f3n actual consiste en lisinopril, carvedilol y espironolactona en las dosis m\u00e1ximas toleradas. En la exploraci\u00f3n f\u00edsica tiene TA 118/74 mmHg, FC 78 lpm. En la auscultaci\u00f3n cardiaca se detecta un tercer tono, la auscultaci\u00f3n pulmonar es normal y no presenta edemas. \u00bfCu\u00e1l de los siguientes es el paso m\u00e1s apropiado en su manejo?:", "full_answer": "Suspender el lisinopril e iniciar sacubitrilo/valsartan.", "type": "CARDIOLOG\u00cdA", "options": {"1": "A\u00f1adir sacubitrilo/valsartan.", "2": "A\u00f1adir ivabradina.", "3": "Suspender el lisinopril e iniciar sacubitrilo/valsartan.", "4": "Suspender el carvedilol e iniciar ivabradina.", "5": NaN}, "correct_option": 3, "explanations": {"1": {"exist": false, "char_ranges": [], "word_ranges": [], "text": ""}, "2": {"exist": false, "char_ranges": [], "word_ranges": [], "text": ""}, "3": {"exist": true, "char_ranges": [[0, 56]], "word_ranges": [[0, 6]], "text": "Suspender el lisinopril e iniciar sacubitrilo/valsartan."}, "4": {"exist": false, "char_ranges": [], "word_ranges": [], "text": ""}, "5": {"exist": false, "char_ranges": [], "word_ranges": [], "text": ""}}} {"id": 221, "year": 2014, "question_id_specific": 48, "full_question": "Un joven de 20 a\u00f1os con cl\u00ednica de ataxia, cefaleas y masa s\u00f3lido-qu\u00edstica en hemisferio cerebeloso derecho es intervenido quir\u00fargicamente, resec\u00e1ndose una lesi\u00f3n que histol\u00f3gicamente muestra c\u00e9lulas con procesos citoplasm\u00e1ticos largos y finos, patr\u00f3n fascicular y microqu\u00edstico, numerosos vasos y fibras de Rosenthal. El diagn\u00f3stico anatomopatol\u00f3gico m\u00e1s probable es:", "full_answer": "Una pregunta que no parece revestir segunda intenci\u00f3n, al no haber elementos confusos. El enunciado resume de manera concisa la descripci\u00f3n del astrocitoma piloc\u00edtico. Las fibras de Rosenthal pueden confundirnos ya que son poco espec\u00edficas, y hacernos pensar,por ejemplo, en el xantoastrocitoma pleom\u00f3rfico en el que tambi\u00e9n pueden encontrarse, pero la morfolog\u00eda celular descrita (procesos citoplasm\u00e1ticos largos y finos, que son los que dan el nombre de \u00abpiloc\u00edtico\u00bb) nos mantiene la primera opci\u00f3n como la correcta. Podemos encontrar una buena descripci\u00f3n de la entidad, explic\u00e1ndonos lo que la distingue de otros tumores gliales en Practical Surgical Neuropathology. A Diagnostic Approach. A. Perry, D.J. Brat. p82-88. Churchill Livingstone Elsevier. 2010. (Philadephia).", "type": "ANATOM\u00cdA PATOL\u00d3GICA", "options": {"1": "Astrocitoma piloc\u00edtico.", "2": "Xantoastrocitoma pleom\u00f3rfico.", "3": "Neurocitoma central.", "4": "Liponeurocitoma.", "5": "Enfermedad por priones."}, "correct_option": 1, "explanations": {"1": {"exist": true, "char_ranges": [[168, 518]], "word_ranges": [[24, 76]], "text": "Las fibras de Rosenthal pueden confundirnos ya que son poco espec\u00edficas, y hacernos pensar,por ejemplo, en el xantoastrocitoma pleom\u00f3rfico en el que tambi\u00e9n pueden encontrarse, pero la morfolog\u00eda celular descrita (procesos citoplasm\u00e1ticos largos y finos, que son los que dan el nombre de \u00abpiloc\u00edtico\u00bb) nos mantiene la primera opci\u00f3n como la correcta."}, "2": {"exist": true, "char_ranges": [[168, 518]], "word_ranges": [[24, 76]], "text": "Las fibras de Rosenthal pueden confundirnos ya que son poco espec\u00edficas, y hacernos pensar,por ejemplo, en el xantoastrocitoma pleom\u00f3rfico en el que tambi\u00e9n pueden encontrarse, pero la morfolog\u00eda celular descrita (procesos citoplasm\u00e1ticos largos y finos, que son los que dan el nombre de \u00abpiloc\u00edtico\u00bb) nos mantiene la primera opci\u00f3n como la correcta."}, "3": {"exist": false, "char_ranges": [], "word_ranges": [], "text": ""}, "4": {"exist": false, "char_ranges": [], "word_ranges": [], "text": ""}, "5": {"exist": false, "char_ranges": [], "word_ranges": [], "text": ""}}} {"id": 606, "year": 2022, "question_id_specific": 113, "full_question": "Var\u00f3n de 35 a\u00f1os, cartero, sin antecedentes de inter\u00e9s, que acude a urgencias por un dolor agudo cervical de 24 horas de evoluci\u00f3n, sin traumatismo previo, que irradia a brazo izquierdo hasta la mano y se acompa\u00f1a de parestesias en el borde radial del antebrazo. No presenta p\u00e9rdida de fuerza objetivable, conserva la movilidad del cuello aunque es dolorosa y se aprecia contractura de la musculatura paravertebral. La primera actitud ser\u00e1:", "full_answer": "Respuesta 1 correcta. El 75% de los pacientes con radiculopat\u00eda mejoran con el tratamiento no quir\u00fargico, siendo este de elecci\u00f3n en fases iniciales. La resonancia magn\u00e9tica es de elecci\u00f3n ante determinadas \u00abred flags\u00bb: Fiebre, p\u00e9rdida de peso, dolor nocturno, persistencia de la sintomatolog\u00eda a pesar del tratamiento conservador y p\u00e9rdida de fuerza (respuesta 3 incorrecta).", "type": "TRAUMATOLOG\u00cdA", "options": {"1": "Tratamiento conservador con antiinflamatorios no esteroideos, calor local y reposo relativo.", "2": "Llamada urgente al neurocirujano para valoraci\u00f3n quir\u00fargica.", "3": "Solicitud preferente de resonancia magn\u00e9tica y electromiograma.", "4": "Derivaci\u00f3n preferente a consultas externas de traumatolog\u00eda.", "5": NaN}, "correct_option": 1, "explanations": {"1": {"exist": true, "char_ranges": [[0, 149]], "word_ranges": [[0, 23]], "text": "Respuesta 1 correcta. El 75% de los pacientes con radiculopat\u00eda mejoran con el tratamiento no quir\u00fargico, siendo este de elecci\u00f3n en fases iniciales."}, "2": {"exist": false, "char_ranges": [], "word_ranges": [], "text": ""}, "3": {"exist": true, "char_ranges": [[150, 376]], "word_ranges": [[23, 55]], "text": "La resonancia magn\u00e9tica es de elecci\u00f3n ante determinadas \u00abred flags\u00bb: Fiebre, p\u00e9rdida de peso, dolor nocturno, persistencia de la sintomatolog\u00eda a pesar del tratamiento conservador y p\u00e9rdida de fuerza (respuesta 3 incorrecta)."}, "4": {"exist": false, "char_ranges": [], "word_ranges": [], "text": ""}, "5": {"exist": false, "char_ranges": [], "word_ranges": [], "text": ""}}} {"id": 436, "year": 2018, "question_id_specific": 223, "full_question": "Mujer boliviana de 35 a\u00f1os de edad, embarazada de 3 meses, que lleva viviendo 10 a\u00f1os en Espa\u00f1a con su pareja, que es espa\u00f1ol y no ha viajado nunca a Latinoam\u00e9rica. La mujer tiene otros dos hijos con su pareja, nacidos en Espa\u00f1a y un hermano que vive en Bolivia y otro en Espa\u00f1a. Durante el embarazo le han diagnosticado infecci\u00f3n por Trypanosoma cruzi. \u00bfCual de las siguientes actuaciones a realizar sobre la enfermedad es INCORRECTA?", "full_answer": "La v\u00eda sexual no es medio de transmisi\u00f3n de Trypanosoma cruzi. En nuestro medio, al no encontrarse el vector transmisor, el medio m\u00e1s importante de transmisi\u00f3n es la v\u00eda de vertical de madre a hijo. La mujer ya al llegar a Espa\u00f1a se encontraba infectada por el par\u00e1sito, puede ser correcto sugerirles que sus hermanos se hagan la serolog\u00eda de T. cruzi por el riesgo que su madre haya transmitido verticalmente la infecci\u00f3n a todos sus hermanos. Igualmente, al haber tenido dos embarazos en estos a\u00f1os de estancia en Espa\u00f1a hay que sugerirle que se estudien tambi\u00e9n sus hijos por la probable transmisi\u00f3n vertical.", "type": "ENFERMEDADES INFECCIOSAS Y MICROBIOLOG\u00cdA", "options": {"1": "Recomendar que sus hijos se hagan la serolog\u00eda de T. cruzi por el riesgo de transmisi\u00f3n transplacentaria.", "2": "Informar a la paciente del riesgo de transmisi\u00f3n de T. cruzi y de la actitud que se va a tomar durante el embarazo, parto y postparto.", "3": "Recomendar que su marido se haga la serolog\u00eda de T. cruzi por el riesgo de haberle transmitido la enfermedad por v\u00eda sexual.", "4": "Recomendar que sus hermanos se hagan la serolog\u00eda de T. cruzi por el riesgo de transmisi\u00f3n vectorial.", "5": NaN}, "correct_option": 3, "explanations": {"1": {"exist": true, "char_ranges": [[457, 612]], "word_ranges": [[78, 104]], "text": "al haber tenido dos embarazos en estos a\u00f1os de estancia en Espa\u00f1a hay que sugerirle que se estudien tambi\u00e9n sus hijos por la probable transmisi\u00f3n vertical."}, "2": {"exist": false, "char_ranges": [], "word_ranges": [], "text": ""}, "3": {"exist": true, "char_ranges": [[0, 62]], "word_ranges": [[0, 11]], "text": "La v\u00eda sexual no es medio de transmisi\u00f3n de Trypanosoma cruzi."}, "4": {"exist": true, "char_ranges": [[199, 444]], "word_ranges": [[35, 77]], "text": "La mujer ya al llegar a Espa\u00f1a se encontraba infectada por el par\u00e1sito, puede ser correcto sugerirles que sus hermanos se hagan la serolog\u00eda de T. cruzi por el riesgo que su madre haya transmitido verticalmente la infecci\u00f3n a todos sus hermanos."}, "5": {"exist": false, "char_ranges": [], "word_ranges": [], "text": ""}}} {"id": 220, "year": 2014, "question_id_specific": 126, "full_question": "Un hombre de 42 a\u00f1os acudi\u00f3 a consulta por edemas. De las pruebas iniciales destacaba la presencia de proteinuria superior a 10 gramos/24 horas. \u00bfCu\u00e1l de las siguientes medidas NO est\u00e1 indicada en el tratamiento de este paciente?:", "full_answer": "El paciente del caso presenta un s\u00edndrome nefr\u00f3tico, hasta ah\u00ed vamos bien. En cuanto a la dieta, para disminuir el edema en estos pacientes, se recomienda que sea hipoproteica e hiposalina, por lo que las opciones 1 y 3 ser\u00edan verdaderas. Los IECAs tambi\u00e9n se administran como tratamiento primario del s\u00edndrome nefr\u00f3tico, ya que disminuyen la excreci\u00f3n de prote\u00ednas por la orina, al igual que los ARA II y los diur\u00e9ticos de asa, por lo que las opciones 2 y 5 tambi\u00e9n ser\u00edan correctas. Por tanto, simplemente por descarte, nos queda la opci\u00f3n 4 como falsa, y por tanto, respuesta correcta. Es m\u00e1s, nos tendr\u00eda que brillar desde el principio como la opci\u00f3n que hay que marcar, ya que precisamente una de las causas de s\u00edndrome nefr\u00f3tico es la toma de AINEs, por lo que \u00bfc\u00f3mo podr\u00edan estar indicados en el tratamiento?", "type": "NEFROLOG\u00cdA", "options": {"1": "Dieta hipoproteica.", "2": "Administraci\u00f3n de diur\u00e9ticos de asa.", "3": "Restricci\u00f3n de sal en la dieta.", "4": "Administraci\u00f3n de AINEs.", "5": "Administraci\u00f3n de inhibidores de la enzima conversora de angiotensina."}, "correct_option": 4, "explanations": {"1": {"exist": true, "char_ranges": [[97, 238]], "word_ranges": [[17, 41]], "text": "para disminuir el edema en estos pacientes, se recomienda que sea hipoproteica e hiposalina, por lo que las opciones 1 y 3 ser\u00edan verdaderas."}, "2": {"exist": true, "char_ranges": [[239, 484]], "word_ranges": [[41, 84]], "text": "Los IECAs tambi\u00e9n se administran como tratamiento primario del s\u00edndrome nefr\u00f3tico, ya que disminuyen la excreci\u00f3n de prote\u00ednas por la orina, al igual que los ARA II y los diur\u00e9ticos de asa, por lo que las opciones 2 y 5 tambi\u00e9n ser\u00edan correctas."}, "3": {"exist": true, "char_ranges": [[97, 238]], "word_ranges": [[17, 41]], "text": "para disminuir el edema en estos pacientes, se recomienda que sea hipoproteica e hiposalina, por lo que las opciones 1 y 3 ser\u00edan verdaderas."}, "4": {"exist": true, "char_ranges": [[675, 754]], "word_ranges": [[117, 132]], "text": "ya que precisamente una de las causas de s\u00edndrome nefr\u00f3tico es la toma de AINEs,"}, "5": {"exist": true, "char_ranges": [[239, 484]], "word_ranges": [[41, 84]], "text": "Los IECAs tambi\u00e9n se administran como tratamiento primario del s\u00edndrome nefr\u00f3tico, ya que disminuyen la excreci\u00f3n de prote\u00ednas por la orina, al igual que los ARA II y los diur\u00e9ticos de asa, por lo que las opciones 2 y 5 tambi\u00e9n ser\u00edan correctas."}}} {"id": 483, "year": 2020, "question_id_specific": 174, "full_question": "Una mujer de 22 a\u00f1os presenta una parada cardiaca s\u00fabita con un ritmo que se considera susceptible de desfibrilaci\u00f3n. Se est\u00e1n realizando compresiones tor\u00e1cicas y ventilaciones, se ha dado una descarga con el desfibrilador y se ha canalizado una v\u00eda venosa perif\u00e9rica. \u00bfCu\u00e1l de las siguientes actuaciones ser\u00eda correcta a continuaci\u00f3n?:", "full_answer": "Pregunta controvertida, puesto que la amiodarona (la dosis s\u00ed es la correcta, 300mg), se administra tras la tercera descarga, y no tras la primera desfibrilaci\u00f3n, que es lo que nos dicen en el enunciado. Aunque, ojo, eso es lo que dicen las gu\u00edas. Estoy segura de que m\u00e1s de uno y una hemos administrado amiodarona en cuanto hemos tenido un acceso venoso disponible en caso de ritmo desfibrilable. Las dem\u00e1s opciones son incorrectas.", "type": "CUIDADOS CR\u00cdTICOS", "options": {"1": "Administrar 150 mg de amiodarona intravenosa.", "2": "Administrar 300 mg de amiodarona intravenosa.", "3": "Administrar 1 mg de atropina intravenosa.", "4": "Administrar 2 mg de atropina intravenosa.", "5": NaN}, "correct_option": 2, "explanations": {"1": {"exist": false, "char_ranges": [], "word_ranges": [], "text": ""}, "2": {"exist": true, "char_ranges": [[35, 84]], "word_ranges": [[4, 13]], "text": "la amiodarona (la dosis s\u00ed es la correcta, 300mg),"}, "3": {"exist": false, "char_ranges": [], "word_ranges": [], "text": ""}, "4": {"exist": false, "char_ranges": [], "word_ranges": [], "text": ""}, "5": {"exist": false, "char_ranges": [], "word_ranges": [], "text": ""}}} {"id": 113, "year": 2012, "question_id_specific": 93, "full_question": "Mujer de 35 a\u00f1os sana que consulta por astenia. En el hemograma presenta anemia microc\u00edtica e hipocroma (hemoglobina de 7 gr/dl, volumen corpuscular medio de 68 fl, hemoglobina corpuscular media de 24 pg) con una amplitud de distribucion eritrocitaria elevada (ADE 20%), reticulocitos disminuidos (0.3%, 30000/l absolutos), contenido de hemoglobina disminuido en los reticulocitos (17 pg) y discreta trombocitosis (500000 plaquetas). Con los datos del hemograma, \u00bfcu\u00e1l ser\u00eda su sospecha diagn\u00f3stica a\u00fan sin tener la bioqu\u00edmica del paciente?", "full_answer": "Con los datos que nos dan, lo primero que nos viene a la cabeza es que se trata de una anemia arregenerativa, ya que la m\u00e9dula \u00f3sea no ha respondido con una exaltaci\u00f3n de la eritropoyesis; de ese modo eliminamos ya las opciones que nos hablan de anemia regenerativa. Lo siguiente que nos orienta hacia la causa es que es microc\u00edtica e hipocroma, lo cual es caracter\u00edstico de la anemia por d\u00e9ficit de hierro, ya que el d\u00e9ficit de f\u00f3lico da lugar a una anemia megalobl\u00e1stica y la hemorragia activa no da microcitosis ni hipocrom\u00eda, sino normocitosis y normocrom\u00eda. Ya casi-casi nos inclinamos por la respuesta 5, anemia central arregenerativa por d\u00e9ficit de hierro\u2026pero queda ah\u00ed la respuesta 1, \u00abrasgo talas\u00e9mico\u00bb, que nos incomoda y hace dudar si no estaremos yendo a lo aparentemente f\u00e1cil\u2026hasta que recordamos que en la talasemia la ADE es normal. Definitivamente, esta chica en edad f\u00e9rtil tiene una anemia por d\u00e9ficit de hierro, algo bastante com\u00fan. Quien tenga dudas puede consultar \u00abHematolog\u00eda. Manual b\u00e1sico razonado\u00bb de Jes\u00fas San Miguel.", "type": "HEMATOLOG\u00cdA", "options": {"1": "Rasgo talas\u00e9mico.", "2": "Anemia central, arregenerativa, por d\u00e9ficit de \u00e1cido f\u00f3lico.", "3": "Anemia perif\u00e9rica, regenerativa, hemol\u00edtica.", "4": "Anemia perif\u00e9rica, regenerativa, por hemorragia activa aguda.", "5": "Anemia central, arregenerativa, por d\u00e9ficit de hierro."}, "correct_option": 5, "explanations": {"1": {"exist": true, "char_ranges": [[816, 849]], "word_ranges": [[138, 145]], "text": "en la talasemia la ADE es normal."}, "2": {"exist": true, "char_ranges": [[117, 266]], "word_ranges": [[24, 49]], "text": "la m\u00e9dula \u00f3sea no ha respondido con una exaltaci\u00f3n de la eritropoyesis; de ese modo eliminamos ya las opciones que nos hablan de anemia regenerativa."}, "3": {"exist": true, "char_ranges": [[117, 266]], "word_ranges": [[24, 49]], "text": "la m\u00e9dula \u00f3sea no ha respondido con una exaltaci\u00f3n de la eritropoyesis; de ese modo eliminamos ya las opciones que nos hablan de anemia regenerativa."}, "4": {"exist": false, "char_ranges": [], "word_ranges": [], "text": ""}, "5": {"exist": true, "char_ranges": [[284, 562]], "word_ranges": [[52, 99]], "text": "nos orienta hacia la causa es que es microc\u00edtica e hipocroma, lo cual es caracter\u00edstico de la anemia por d\u00e9ficit de hierro, ya que el d\u00e9ficit de f\u00f3lico da lugar a una anemia megalobl\u00e1stica y la hemorragia activa no da microcitosis ni hipocrom\u00eda, sino normocitosis y normocrom\u00eda."}}} {"id": 59, "year": 2011, "question_id_specific": 51, "full_question": "Paciente de 47 a\u00f1os de edad que consulta en el servicio de Urgencias por p\u00e9rdida brusca de conciencia \u00bfCu\u00e1l de entre los siguientes tiene un mayor rendimiento diagn\u00f3stico?", "full_answer": "Evidentemente, es la \u00fanica respuesta que engloba todas las causas de p\u00e9rdida de conciencia brusca y que nos permite filiar de alguna forma el diagn\u00f3stico.", "type": "ANESTESIOLOG\u00cdA Y CUIDADOS CR\u00cdTICOS", "options": {"1": "Historia cl\u00ednica.", "2": "Electrocardiograma.", "3": "Registro Holter.", "4": "EEG.", "5": "TC de cr\u00e1neo."}, "correct_option": 1, "explanations": {"1": {"exist": true, "char_ranges": [[0, 154]], "word_ranges": [[0, 25]], "text": "Evidentemente, es la \u00fanica respuesta que engloba todas las causas de p\u00e9rdida de conciencia brusca y que nos permite filiar de alguna forma el diagn\u00f3stico."}, "2": {"exist": false, "char_ranges": [], "word_ranges": [], "text": ""}, "3": {"exist": false, "char_ranges": [], "word_ranges": [], "text": ""}, "4": {"exist": false, "char_ranges": [], "word_ranges": [], "text": ""}, "5": {"exist": false, "char_ranges": [], "word_ranges": [], "text": ""}}} {"id": 412, "year": 2018, "question_id_specific": 54, "full_question": "Isabel tiene tres hijos varones y una hija (Mar\u00eda), todos ellos sanos. Un hermano y un t\u00edo materno de Isabel fallecieron por estar afectos de la enfermedad de Duchenne: una enfermedad recesiva ligada al cromosoma X. Actualmente, Mar\u00eda desea quedarse embarazada y quiere conocer el riesgo de transmitir la enfermedad a su descendencia. Con los datos recogido, \u00bfcu\u00e1l es la probabilidad de que Mar\u00eda sea portadora de la enfermedad de Duchenne?", "full_answer": "La abuela materna era portadora de la mutaci\u00f3n en una copia del gen en un cromosoma X (no nos dicen que sufriera la enfermedad, y en cambio un hijo suyo, t\u00edo materno de Isabel, s\u00ed que la sufri\u00f3). La madre de Isabel sigue port\u00e1ndolo, ya que un hermano de Isabel ha sufrido la enfermedad pero no su padre. As\u00ed pues, Isabel tiene un 50% de probabilidades de haber heredado el gen mutado de la madre (como sus hijos est\u00e1n todos sanos, no estamos seguros de que sea portadora), y por lo tanto su hija un 25% de probabilidades de haber heredado el gen mutado de la abuela.", "type": "BIOESTAD\u00cdSTICA", "options": {"1": "2/3.", "2": "1/2.", "3": "1/4.", "4": "Menos de 1/4.", "5": NaN}, "correct_option": 3, "explanations": {"1": {"exist": false, "char_ranges": [], "word_ranges": [], "text": ""}, "2": {"exist": false, "char_ranges": [], "word_ranges": [], "text": ""}, "3": {"exist": true, "char_ranges": [[0, 566]], "word_ranges": [[0, 107]], "text": "La abuela materna era portadora de la mutaci\u00f3n en una copia del gen en un cromosoma X (no nos dicen que sufriera la enfermedad, y en cambio un hijo suyo, t\u00edo materno de Isabel, s\u00ed que la sufri\u00f3). La madre de Isabel sigue port\u00e1ndolo, ya que un hermano de Isabel ha sufrido la enfermedad pero no su padre. As\u00ed pues, Isabel tiene un 50% de probabilidades de haber heredado el gen mutado de la madre (como sus hijos est\u00e1n todos sanos, no estamos seguros de que sea portadora), y por lo tanto su hija un 25% de probabilidades de haber heredado el gen mutado de la abuela."}, "4": {"exist": false, "char_ranges": [], "word_ranges": [], "text": ""}, "5": {"exist": false, "char_ranges": [], "word_ranges": [], "text": ""}}} {"id": 347, "year": 2016, "question_id_specific": 159, "full_question": "Mujer de 69 a\u00f1os que acude a su consulta refiriendo sangrado genital de varios meses de evoluci\u00f3n. Niega tratamiento hormonal sustitutivo y anticoagulaci\u00f3n. Aporta citolog\u00eda cervicovaginal normal. Exploraci\u00f3n f\u00edsica general y genital sin hallazgos de inter\u00e9s. IMC de 38kg/m2. Indique la actitud m\u00e1s correcta:", "full_answer": "Biopsia endometrial. Peri o menopausia: \u2022 Por la alta prevalencia de patolog\u00eda org\u00e1nica durante este periodo es preciso realizar sistem\u00e1ticamente biopsia endometrial (Cornier o Histeroscopia).", "type": "GINECOLOG\u00cdA Y OBSTETRICIA", "options": {"1": "Prescribir progesterona c\u00edclica.", "2": "Biopsia endometrial.", "3": "Biopsias de c\u00e9rvix al azar.", "4": "Valoraci\u00f3n hormonal con FSH, LH y estradiol.", "5": NaN}, "correct_option": 2, "explanations": {"1": {"exist": false, "char_ranges": [], "word_ranges": [], "text": ""}, "2": {"exist": true, "char_ranges": [[21, 192]], "word_ranges": [[2, 25]], "text": "Peri o menopausia: \u2022 Por la alta prevalencia de patolog\u00eda org\u00e1nica durante este periodo es preciso realizar sistem\u00e1ticamente biopsia endometrial (Cornier o Histeroscopia)."}, "3": {"exist": false, "char_ranges": [], "word_ranges": [], "text": ""}, "4": {"exist": false, "char_ranges": [], "word_ranges": [], "text": ""}, "5": {"exist": false, "char_ranges": [], "word_ranges": [], "text": ""}}} {"id": 325, "year": 2016, "question_id_specific": 56, "full_question": "Paciente de 54 a\u00f1os que ingresa por fiebre termometrada de 38\u00b0C en los cinco d\u00edas previos y disnea de reposo( NYHA lV) que apareci\u00f3 6 horas antes de acudir al hospital. En urgencias la exploraci\u00f3n es compatible con insuficiencia cardiaca y el ECG muestra bloqueo auriculo-ventricular completo con una frecuencia ventricular de escape de 45 lpm. Los signos de insuficiencia cardiaca son refractarios al tratamiento m\u00e9dico y la ecocardiografia transesof\u00e1gica realizada muestra una v\u00e1lvula a\u00f3rtica con orificio regurgitante efectivo de 0.5 cm2. Los cultivos seriados son positivos para Streptococcus gallolyticus. Indique la actitud m\u00e1s acertada:", "full_answer": "La respuesta correcta es la 1, ya que entre las indicaciones de cirug\u00eda card\u00edaca se encuentran la aparici\u00f3n de insuficiencia card\u00edaca o de nuevos trastornos de conducci\u00f3n, y en este caso presenta ambas cosas.", "type": "CUIDADOS CR\u00cdTICOS, PALIATIVOS Y URGENCIAS", "options": {"1": "Cirug\u00eda cardiaca de reemplazo valvular a\u00f3rtico por pr\u00f3tesis mec\u00e1nica con terapia antibi\u00f3tica seg\u00fan antibiograma.", "2": "Terapia antibi\u00f3tica seg\u00fan antibiograma e implantaci\u00f3n de bal\u00f3n de contrapulsaci\u00f3n intra-a\u00f3rtico y marcapasos transitorio hasta 3 semanas, tras las cuales se implantar\u00e1 marcapasos definitivo.", "3": "Implantaci\u00f3n de marcapasos transitorio, terapia antibi\u00f3tica seg\u00fan antibiograma e implantaci\u00f3n percut\u00e1nea de pr\u00f3tesis valvular a\u00f3rtica.", "4": "Implantaci\u00f3n urgente de marcapasos definitivo con terapia antibi\u00f3tica seg\u00fan antibiograma durante 6 semanas.", "5": NaN}, "correct_option": 1, "explanations": {"1": {"exist": true, "char_ranges": [[38, 208]], "word_ranges": [[8, 34]], "text": "entre las indicaciones de cirug\u00eda card\u00edaca se encuentran la aparici\u00f3n de insuficiencia card\u00edaca o de nuevos trastornos de conducci\u00f3n, y en este caso presenta ambas cosas."}, "2": {"exist": false, "char_ranges": [], "word_ranges": [], "text": ""}, "3": {"exist": false, "char_ranges": [], "word_ranges": [], "text": ""}, "4": {"exist": false, "char_ranges": [], "word_ranges": [], "text": ""}, "5": {"exist": false, "char_ranges": [], "word_ranges": [], "text": ""}}} {"id": 335, "year": 2016, "question_id_specific": 175, "full_question": "Un hombre de 88 a\u00f1os viene a consulta porque se ha ca\u00eddo 3 veces en los \u00faltimos 6 meses. Ninguna de las ca\u00eddas se acompa\u00f1aba de mareo o s\u00edncope. Una ca\u00edda ocurri\u00f3 mientras caminaba por el jard\u00edn. Sus antecedentes m\u00e9dicos incluyen hipertensi\u00f3n arterial sin cambios posturales en la presi\u00f3n arterial, gota, artrosis y depresi\u00f3n. Toma 5 medicinas regularmente. \u00bfCu\u00e1l de las siguientes es la que m\u00e1s probablemente contribuya a las ca\u00eddas en este paciente?", "full_answer": "Paroxetina; pregunta complicada ya que tanto hidroclorotiazida como lisinopril pueden tambi\u00e9n producir ca\u00eddas en ancianos, pero generalmente se acompa\u00f1ar\u00edan de mareo.", "type": "FARMACOLOG\u00cdA", "options": {"1": "Alopurinol.", "2": "Hidroclorotiazida.", "3": "Lisinopril.", "4": "Paroxetina.", "5": NaN}, "correct_option": 4, "explanations": {"1": {"exist": false, "char_ranges": [], "word_ranges": [], "text": ""}, "2": {"exist": true, "char_ranges": [[39, 166]], "word_ranges": [[5, 21]], "text": "tanto hidroclorotiazida como lisinopril pueden tambi\u00e9n producir ca\u00eddas en ancianos, pero generalmente se acompa\u00f1ar\u00edan de mareo."}, "3": {"exist": true, "char_ranges": [[39, 166]], "word_ranges": [[5, 21]], "text": "tanto hidroclorotiazida como lisinopril pueden tambi\u00e9n producir ca\u00eddas en ancianos, pero generalmente se acompa\u00f1ar\u00edan de mareo."}, "4": {"exist": true, "char_ranges": [[39, 166]], "word_ranges": [[5, 21]], "text": "tanto hidroclorotiazida como lisinopril pueden tambi\u00e9n producir ca\u00eddas en ancianos, pero generalmente se acompa\u00f1ar\u00edan de mareo."}, "5": {"exist": false, "char_ranges": [], "word_ranges": [], "text": ""}}} {"id": 19, "year": 2011, "question_id_specific": 109, "full_question": "Una mujer de 32 a\u00f1os ha viajado a Cuba de donde ha llegado hace tres d\u00edas. Al d\u00eda siguiente de su regreso acude a su m\u00e9dico de cabecera por fiebre elevada, artromialgias intensas y cefalea que le hab\u00eda comenzado antes de iniciar el viaje de regreso y le prescribe paracetamol. Tres d\u00edas m\u00e1s tarde y sin haber experimentado mejor\u00eda presenta por la ma\u00f1ana un exantema maculo-papuloso pruriginoso generalizado que es m\u00e1s intenso en miembros inferiores en donde evoluciona a la formaci\u00f3n de petequias por lo que acude a Urgencias. En las EC no se aprecian datos relevantes excepto las plaquetas 75000/mm3 (htco 36%, leucocitos 4100 79% neutr\u00f3filos). \u00bfCu\u00e1l es el diagn\u00f3stico m\u00e1s probable?", "full_answer": "En este caso creo que la respuesta correcta es la 5. La 1 no debe ser ya que aunque la cl\u00ednica es sugestiva, no presenta linfocitosis. Tampoco parece un exantema al\u00e9rgico por la cl\u00ednica. Y desde luego no re\u00fane s\u00edntomas ni signos cl\u00ednicos compatibles con malaria o fiebre tifoidea. Lo que orienta al dengue, adem\u00e1s de la cl\u00ednica es la plaquetopenia.", "type": "INFECCIOSAS", "options": {"1": "Mononuclesis infecciosa.", "2": "Exantema al\u00e9rgico.", "3": "Malaria por P. falciparum.", "4": "Fiebre tifoidea.", "5": "Dengue."}, "correct_option": 5, "explanations": {"1": {"exist": false, "char_ranges": [], "word_ranges": [], "text": ""}, "2": {"exist": true, "char_ranges": [[135, 186]], "word_ranges": [[26, 34]], "text": "Tampoco parece un exantema al\u00e9rgico por la cl\u00ednica."}, "3": {"exist": true, "char_ranges": [[189, 280]], "word_ranges": [[35, 49]], "text": "desde luego no re\u00fane s\u00edntomas ni signos cl\u00ednicos compatibles con malaria o fiebre tifoidea."}, "4": {"exist": true, "char_ranges": [[189, 280]], "word_ranges": [[35, 49]], "text": "desde luego no re\u00fane s\u00edntomas ni signos cl\u00ednicos compatibles con malaria o fiebre tifoidea."}, "5": {"exist": true, "char_ranges": [[53, 134], [281, 348]], "word_ranges": [[11, 26], [49, 61]], "text": "La 1 no debe ser ya que aunque la cl\u00ednica es sugestiva, no presenta linfocitosis. Lo que orienta al dengue, adem\u00e1s de la cl\u00ednica es la plaquetopenia."}}} {"id": 401, "year": 2016, "question_id_specific": 147, "full_question": "Un ni\u00f1o de 13 a\u00f1os presenta dolor dorsal de varios meses de evoluci\u00f3n, s\u00f3lo a la bipedestaci\u00f3n y a la marcha, \u00bfQu\u00e9 diagn\u00f3stico debemos de pensar?", "full_answer": "Un tumor maligno tambi\u00e9n doler\u00eda en reposo. En cuanto a Scheuermann y escoliosis deber\u00edan darnos alg\u00fan dato f\u00edsico porque el paciente se presenta con una exploraci\u00f3n concreta que nos diera el diagn\u00f3stico.", "type": "TRAUMATOLOG\u00cdA Y ORTOPED\u00cdA", "options": {"1": "Enfermedad de Scheuermann.", "2": "Escoliosis tor\u00e1cica.", "3": "Tumor maligno del cuerpo vertebral.", "4": "Se trata de una sobrecarga mec\u00e1nica.", "5": NaN}, "correct_option": 4, "explanations": {"1": {"exist": true, "char_ranges": [[44, 204]], "word_ranges": [[7, 32]], "text": "En cuanto a Scheuermann y escoliosis deber\u00edan darnos alg\u00fan dato f\u00edsico porque el paciente se presenta con una exploraci\u00f3n concreta que nos diera el diagn\u00f3stico."}, "2": {"exist": true, "char_ranges": [[44, 204]], "word_ranges": [[7, 32]], "text": "En cuanto a Scheuermann y escoliosis deber\u00edan darnos alg\u00fan dato f\u00edsico porque el paciente se presenta con una exploraci\u00f3n concreta que nos diera el diagn\u00f3stico."}, "3": {"exist": true, "char_ranges": [[0, 43]], "word_ranges": [[0, 7]], "text": "Un tumor maligno tambi\u00e9n doler\u00eda en reposo."}, "4": {"exist": false, "char_ranges": [], "word_ranges": [], "text": ""}, "5": {"exist": false, "char_ranges": [], "word_ranges": [], "text": ""}}} {"id": 607, "year": 2022, "question_id_specific": 115, "full_question": "Var\u00f3n de 27 a\u00f1os, deportista habitual, que refiere dolor en la pierna derecha tras la pr\u00e1ctica de carrera continua. Ha acudido en varias ocasiones a un fisioterapeuta, siendo diagnosticado de una sobrecarga en gemelos. Han transcurrido varios meses, no ha mejorado y refiere dolor intenso tras la actividad f\u00edsica que cede con el reposo en las siguientes horas del ejercicio. \u00bfQu\u00e9 prueba puede ayudar en el diagn\u00f3stico?:", "full_answer": "Sospecha cl\u00ednica de s\u00edndrome compartimental cr\u00f3nico (SCC). El diagnostico se realiza mediante la obtenci\u00f3n de presiones compartimentales en reposo, durante el ejercicio y post ejercicio.", "type": "TRAUMATOLOG\u00cdA", "options": {"1": "Tomograf\u00eda de emisi\u00f3n de positrones con 18 FDG.", "2": "Determinaci\u00f3n de la presi\u00f3n del compartimento posterior inmediatamente tras actividad.", "3": "Ecograf\u00eda-Doppler para descartar un trastorno circulatorio de la extremidad inferior.", "4": "Espectrometr\u00eda por resonancia magn\u00e9tica.", "5": NaN}, "correct_option": 2, "explanations": {"1": {"exist": false, "char_ranges": [], "word_ranges": [], "text": ""}, "2": {"exist": true, "char_ranges": [[0, 186]], "word_ranges": [[0, 25]], "text": "Sospecha cl\u00ednica de s\u00edndrome compartimental cr\u00f3nico (SCC). El diagnostico se realiza mediante la obtenci\u00f3n de presiones compartimentales en reposo, durante el ejercicio y post ejercicio."}, "3": {"exist": false, "char_ranges": [], "word_ranges": [], "text": ""}, "4": {"exist": false, "char_ranges": [], "word_ranges": [], "text": ""}, "5": {"exist": false, "char_ranges": [], "word_ranges": [], "text": ""}}} {"id": 453, "year": 2018, "question_id_specific": 150, "full_question": "Un hombre de 25 a\u00f1os consulta por temblor. La exploraci\u00f3n objetiva disartria y distonia. Tiene una historia familiar de enfermedad psiqui\u00e1trica y trastornos del movimiento. \u00bfQu\u00e9 prueba diagn\u00f3stica considera mas acertada?", "full_answer": "Si hablan de un hombre joven de 20-30 a\u00f1os, con temblor y diston\u00eda, tenga o no antecedentes familiares, y sobre todo si una de las opciones me habla de cobre en orina, me est\u00e1n indicando que piense en una enfermedad de Wilson. No pienso en un Parkinson juvenil, ni en una cl\u00ednica epil\u00e9ptica. Por esto la respuesta correcta es la 4.", "type": "NEUROLOG\u00cdA", "options": {"1": "DATS CAN.", "2": "Estudio de conducci\u00f3n nerviosa.", "3": "Electroencefalograma.", "4": "Cobre en orina de 24.", "5": NaN}, "correct_option": 4, "explanations": {"1": {"exist": true, "char_ranges": [[0, 331]], "word_ranges": [[0, 61]], "text": "Si hablan de un hombre joven de 20-30 a\u00f1os, con temblor y diston\u00eda, tenga o no antecedentes familiares, y sobre todo si una de las opciones me habla de cobre en orina, me est\u00e1n indicando que piense en una enfermedad de Wilson. No pienso en un Parkinson juvenil, ni en una cl\u00ednica epil\u00e9ptica. Por esto la respuesta correcta es la 4."}, "2": {"exist": false, "char_ranges": [], "word_ranges": [], "text": ""}, "3": {"exist": true, "char_ranges": [[0, 331]], "word_ranges": [[0, 61]], "text": "Si hablan de un hombre joven de 20-30 a\u00f1os, con temblor y diston\u00eda, tenga o no antecedentes familiares, y sobre todo si una de las opciones me habla de cobre en orina, me est\u00e1n indicando que piense en una enfermedad de Wilson. No pienso en un Parkinson juvenil, ni en una cl\u00ednica epil\u00e9ptica. Por esto la respuesta correcta es la 4."}, "4": {"exist": true, "char_ranges": [[0, 331]], "word_ranges": [[0, 61]], "text": "Si hablan de un hombre joven de 20-30 a\u00f1os, con temblor y diston\u00eda, tenga o no antecedentes familiares, y sobre todo si una de las opciones me habla de cobre en orina, me est\u00e1n indicando que piense en una enfermedad de Wilson. No pienso en un Parkinson juvenil, ni en una cl\u00ednica epil\u00e9ptica. Por esto la respuesta correcta es la 4."}, "5": {"exist": false, "char_ranges": [], "word_ranges": [], "text": ""}}} {"id": 529, "year": 2021, "question_id_specific": 166, "full_question": "Paciente de 18 a\u00f1os que consulta por edemas. Se realiza anal\u00edtica completa que muestra proteinuria de 8 g/d\u00eda sin microhematuria, hipoalbuminemia e hipercolesterolemia con funci\u00f3n renal normal. Se le administran de forma emp\u00edrica corticoides. Al cabo de un mes, el cuadro cl\u00ednico ha desaparecido completamente. \u00bfCu\u00e1l es su hip\u00f3tesis diagn\u00f3stica?:", "full_answer": "Nos describen a una paciente joven con criterios de s\u00edndrome nefr\u00f3tico con excelente respuesta a corticoides. Sin biopsia, la presentaci\u00f3n y evoluci\u00f3n cl\u00ednica sugieren una nefropat\u00eda con cambios m\u00ednimos, que habitualmente cursa con funci\u00f3n renal normal o discretamente alterada junto con s\u00edndrome nefr\u00f3tico, y que en el 85-90 % de los casos se resuelve con el tratamiento esteroideo. La edad es el \u00fanico dato que chirr\u00eda un poco, ya que aunque es la causa m\u00e1s frecuente de s\u00edndrome nefr\u00f3tico idiop\u00e1tico en ni\u00f1os y adolescentes, habitualmente se recomienda realizar biopsia previa al tratamiento en los mayores de 16 a\u00f1os, en cualquier caso no admite mucha discusi\u00f3n.", "type": "NEFROLOG\u00cdA", "options": {"1": "Amiloidosis.", "2": "Nefropat\u00eda IgA o enfermedad de Berger.", "3": "S\u00edndrome de Alport.", "4": "Nefropat\u00eda con cambios m\u00ednimos.", "5": NaN}, "correct_option": 4, "explanations": {"1": {"exist": false, "char_ranges": [], "word_ranges": [], "text": ""}, "2": {"exist": false, "char_ranges": [], "word_ranges": [], "text": ""}, "3": {"exist": false, "char_ranges": [], "word_ranges": [], "text": ""}, "4": {"exist": true, "char_ranges": [[0, 306]], "word_ranges": [[0, 43]], "text": "Nos describen a una paciente joven con criterios de s\u00edndrome nefr\u00f3tico con excelente respuesta a corticoides. Sin biopsia, la presentaci\u00f3n y evoluci\u00f3n cl\u00ednica sugieren una nefropat\u00eda con cambios m\u00ednimos, que habitualmente cursa con funci\u00f3n renal normal o discretamente alterada junto con s\u00edndrome nefr\u00f3tico,"}, "5": {"exist": false, "char_ranges": [], "word_ranges": [], "text": ""}}} {"id": 258, "year": 2014, "question_id_specific": 96, "full_question": "Un hombre de 62 a\u00f1os con una diabetes mellitus tipo 2 de 10 a\u00f1os de evoluci\u00f3n realiza tratamiento con metformina y sitagliptina. Hace ejercicio f\u00edsico escaso y realiza una dieta adecuada. En los \u00faltimos 6 meses ha perdido peso y tiene m\u00e1s astenia. Sus controles gluc\u00e9micos se han deteriorado pasando de glucemias basales de 110-140 mg/dl a glucemias de 170-200 mg/dl, as\u00ed como su hemoglobina glicosilada que ha pasado de 7,1 a 8,5%. La medida terap\u00e9utica m\u00e1s adecuada a realizar es:", "full_answer": "Asociar al tratamiento una dosis de insulina basal. Pregunta esperable de un tema muy importante: la diabetes y su tratamiento. Paciente mal controlado con metformina y sitagliptina con discreta cl\u00ednica cardinal (p\u00e9rdida de peso, astenia). La mejor opci\u00f3n es la insulina para controlar los s\u00edntomas cardinales y disminuir la HbA1c a < 7 %.", "type": "ENDOCRINOLOG\u00cdA", "options": {"1": "Aumentar la ingesta de prote\u00ednas e hidratos de carbono de cadena larga en la dieta para mejorar la astenia y la p\u00e9rdida de peso.", "2": "Asociar al tratamiento una dosis de insulina basal.", "3": "Asociar al tratamiento ascarbosa.", "4": "Sustituir sitagliptina por pioglitazona.", "5": "Sustituir metformina por glimepirida."}, "correct_option": 2, "explanations": {"1": {"exist": false, "char_ranges": [], "word_ranges": [], "text": ""}, "2": {"exist": true, "char_ranges": [[128, 339]], "word_ranges": [[20, 54]], "text": "Paciente mal controlado con metformina y sitagliptina con discreta cl\u00ednica cardinal (p\u00e9rdida de peso, astenia). La mejor opci\u00f3n es la insulina para controlar los s\u00edntomas cardinales y disminuir la HbA1c a < 7 %."}, "3": {"exist": false, "char_ranges": [], "word_ranges": [], "text": ""}, "4": {"exist": false, "char_ranges": [], "word_ranges": [], "text": ""}, "5": {"exist": false, "char_ranges": [], "word_ranges": [], "text": ""}}} {"id": 375, "year": 2016, "question_id_specific": 128, "full_question": "Paciente de 45 a\u00f1os en estudio por posible meningitis, con fiebre, cefalea y v\u00f3mitos de 2 d\u00edas de evoluci\u00f3n. Le realizan una RM cerebral y una punci\u00f3n lumbar. Veinte horas despu\u00e9s, al levantarse para ir al ba\u00f1o, se queja de cefalea intensa, muy marcada al incorporarse pero que desaparece al tumbarse. Ya no presenta fiebre ni v\u00f3mitos. No puede caminar. \u00bfCu\u00e1l es con m\u00e1s probabilidad el origen de esta cefalea?", "full_answer": "El cuadro que describen de cefalea intensa con el ortostatismo que desaparece al tumbarse, aunque si bien es cierto que la presentaci\u00f3n m\u00e1s t\u00edpica es entre las primeras 24-48 horas y en este caso es algo anterior. A\u00fan as\u00ed el resto de respuestas son improbables: la primera no puede ser porque no nos han dicho como era la cefalea inicial, la tercera si bien pod\u00eda ser por el tiempo de aparici\u00f3n (como ya dije un poco antes de lo esperado) la realidad es que es tan t\u00edpica la cefalea con el ortostatismo que ser\u00eda la causa m\u00e1s probable; la \u00faltima tampoco podr\u00eda ser porque una HSA no sigue ese patr\u00f3n en relaci\u00f3n con el ortostatismo.", "type": "NEUROLOG\u00cdA", "options": {"1": "La meningitis sigue siendo la causa fundamental de su cefalea, pues es el mismo tipo de cefalea que al inicio de los s\u00edntomas.", "2": "Es una cefalea post-punci\u00f3n lumbar.", "3": "Hay que buscar una causa diferente a esta cefalea, pues no es tipica del sindrome post-punci\u00f3n lumbar ni de la meningitis.", "4": "Lo m\u00e1s probable es que no fuera una meningitis viral, sino una hemorragia subaracnoidea. Por este motivo, la cefalea inicial desaparece casi completamente al tumbarse.", "5": NaN}, "correct_option": 2, "explanations": {"1": {"exist": true, "char_ranges": [[262, 337]], "word_ranges": [[45, 60]], "text": "la primera no puede ser porque no nos han dicho como era la cefalea inicial,"}, "2": {"exist": true, "char_ranges": [[0, 89]], "word_ranges": [[0, 14]], "text": "El cuadro que describen de cefalea intensa con el ortostatismo que desaparece al tumbarse,"}, "3": {"exist": true, "char_ranges": [[340, 534]], "word_ranges": [[60, 98]], "text": "la tercera si bien pod\u00eda ser por el tiempo de aparici\u00f3n (como ya dije un poco antes de lo esperado) la realidad es que es tan t\u00edpica la cefalea con el ortostatismo que ser\u00eda la causa m\u00e1s probable;"}, "4": {"exist": true, "char_ranges": [[536, 632]], "word_ranges": [[98, 115]], "text": "la \u00faltima tampoco podr\u00eda ser porque una HSA no sigue ese patr\u00f3n en relaci\u00f3n con el ortostatismo."}, "5": {"exist": false, "char_ranges": [], "word_ranges": [], "text": ""}}} {"id": 352, "year": 2016, "question_id_specific": 161, "full_question": "Mujer de 27 a\u00f1os remitida a consulta de ginecolog\u00eda para su valoraci\u00f3n refiriendo dispareunia desde hace unos 8 meses, junto con disquecia y rectorragia ocasional coincidiendo con la menstruaci\u00f3n desde hace 3-4 meses. Tambi\u00e9n refiere dismenorrea desde hace a\u00f1os que controla bien con ibuprofeno. Lleva intentando quedarse embarazada unos 16 meses sin haberlo conseguido a\u00fan. En la exploraci\u00f3n ginecol\u00f3gica tan s\u00f3lo se aprecia dolor al presionar fondo de saco vaginal posterior. \u00bfQu\u00e9 prueba considera Ud que le permitir\u00eda llegar al diagn\u00f3stico de certeza de su patolog\u00eda?", "full_answer": "La respuesta correcta es la 2. Estamos ante una sospecha de endometriosis, por los s\u00edntomas y signos que nos da el enunciado. Nos piden un diagn\u00f3stico de certeza, y el Gold Standard para el diagn\u00f3stico de esta enfermedad es la laparoscopia diagn\u00f3stica.", "type": "GINECOLOG\u00cdA Y OBSTETRICIA", "options": {"1": "Ecograf\u00eda transvaginal.", "2": "Laparoscopia diagn\u00f3stica.", "3": "Resonancia magn\u00e9tica.", "4": "Colonoscopia.", "5": NaN}, "correct_option": 2, "explanations": {"1": {"exist": false, "char_ranges": [], "word_ranges": [], "text": ""}, "2": {"exist": true, "char_ranges": [[31, 252]], "word_ranges": [[6, 42]], "text": "Estamos ante una sospecha de endometriosis, por los s\u00edntomas y signos que nos da el enunciado. Nos piden un diagn\u00f3stico de certeza, y el Gold Standard para el diagn\u00f3stico de esta enfermedad es la laparoscopia diagn\u00f3stica."}, "3": {"exist": false, "char_ranges": [], "word_ranges": [], "text": ""}, "4": {"exist": false, "char_ranges": [], "word_ranges": [], "text": ""}, "5": {"exist": false, "char_ranges": [], "word_ranges": [], "text": ""}}} {"id": 575, "year": 2022, "question_id_specific": 102, "full_question": "Mujer de 29 a\u00f1os con antecedentes de crisis febriles y fumadora importante. Consulta por presentar desde hace unos 2 a\u00f1os episodios frecuentes, estereotipados, que comienzan con una sensaci\u00f3n epig\u00e1strica ascendente y un olor desagradable, seguidos de desconexi\u00f3n del medio. Los familiares que est\u00e1n con ella observan que hace movimientos de chupeteo y de apertura y cierre repetidos de la mano izquierda. Pasados dos minutos los movimientos ceden, pero le cuesta responder adecuadamente y solo recuerda parcialmente lo ocurrido. De los siguientes \u00bfcu\u00e1l es el diagn\u00f3stico m\u00e1s probable?:", "full_answer": "El caso cl\u00ednico hace una descripci\u00f3n de libro de una crisis temporal medial. El aura de la sensaci\u00f3n epig\u00e1strica ascendente y la alteraci\u00f3n olfativa y la posterior descripci\u00f3n de la crisis con la alteraci\u00f3n del nivel de conciencia (desconexi\u00f3n del medio) con automatismos bucales y manuales. La clasificaci\u00f3n vigente es la de la ILAE de 2017 que describir\u00eda esta crisis como crisis focal con alteraci\u00f3n del nivel de conciencia, pero en el MIR siguen usando la terminolog\u00eda antigua (compleja es como se llamaba a la alteraci\u00f3n del nivel de conciencia).", "type": "NEUROLOG\u00cdA", "options": {"1": "Crisis focales complejas del l\u00f3bulo temporal.", "2": "Accidentes isqu\u00e9micos transitorios en territorio carot\u00eddeo derecho.", "3": "Crisis focales simples motoras.", "4": "Episodios de amnesia global transitoria.", "5": NaN}, "correct_option": 1, "explanations": {"1": {"exist": true, "char_ranges": [[292, 551]], "word_ranges": [[46, 90]], "text": "La clasificaci\u00f3n vigente es la de la ILAE de 2017 que describir\u00eda esta crisis como crisis focal con alteraci\u00f3n del nivel de conciencia, pero en el MIR siguen usando la terminolog\u00eda antigua (compleja es como se llamaba a la alteraci\u00f3n del nivel de conciencia)."}, "2": {"exist": false, "char_ranges": [], "word_ranges": [], "text": ""}, "3": {"exist": false, "char_ranges": [], "word_ranges": [], "text": ""}, "4": {"exist": false, "char_ranges": [], "word_ranges": [], "text": ""}, "5": {"exist": false, "char_ranges": [], "word_ranges": [], "text": ""}}} {"id": 278, "year": 2016, "question_id_specific": 31, "full_question": "Mujer de 20 a\u00f1os con tumoraci\u00f3n ov\u00e1rica de 15 cm, s\u00f3lido-qu\u00edstica, detectada por ecograf\u00eda tras presentar s\u00edntomas abdominales inespec\u00edficos. En el estudio anatomopatol\u00f3gico de la pieza correspondiente se encuentran dientes, pelos, zonas de epitelio intestinal, \u00e1reas de epitelio escamoso (15%) y bronquial, as\u00ed como elementos neuroectod\u00e9rmicos y embrionarios en varias de las preparaciones histol\u00f3gicas. En referencia a este caso, se\u00f1ale el diagn\u00f3stico correcto:", "full_answer": "Tras leer la pregunta y las posibles respuestas, contamos con dos opciones tumorales y dos no tumorales. En ning\u00fan momento nos comentan rasgos at\u00edpicos celulares, con lo que podemos descartar sin miedo las lesiones tumorales, quedando dudas entre las respuestas 2 y 3. En este caso, tendr\u00edamos que decantarnos por el TERATOMA INMADURO, al mencionar la presencia de restos embrionarios.", "type": "ANATOM\u00cdA PATOL\u00d3GICA", "options": {"1": "Teratocarcinoma.", "2": "Teratoma inmaduro.", "3": "Teratoma qu\u00edstico maduro.", "4": "Disgerminoma.", "5": NaN}, "correct_option": 2, "explanations": {"1": {"exist": true, "char_ranges": [[105, 224]], "word_ranges": [[17, 35]], "text": "En ning\u00fan momento nos comentan rasgos at\u00edpicos celulares, con lo que podemos descartar sin miedo las lesiones tumorales,"}, "2": {"exist": true, "char_ranges": [[283, 385]], "word_ranges": [[46, 60]], "text": "tendr\u00edamos que decantarnos por el TERATOMA INMADURO, al mencionar la presencia de restos embrionarios."}, "3": {"exist": true, "char_ranges": [[283, 385]], "word_ranges": [[46, 60]], "text": "tendr\u00edamos que decantarnos por el TERATOMA INMADURO, al mencionar la presencia de restos embrionarios."}, "4": {"exist": true, "char_ranges": [[105, 224]], "word_ranges": [[17, 35]], "text": "En ning\u00fan momento nos comentan rasgos at\u00edpicos celulares, con lo que podemos descartar sin miedo las lesiones tumorales,"}, "5": {"exist": false, "char_ranges": [], "word_ranges": [], "text": ""}}} {"id": 225, "year": 2014, "question_id_specific": 64, "full_question": "Un paciente de 82 a\u00f1os acude a la consulta por presentar mareos repentinos y frecuentes, sin pr\u00f3dromos, de corta duraci\u00f3n y que ceden espont\u00e1neamente, sin presentar nunca s\u00edncope. La exploraci\u00f3n f\u00edsica y el electrocardiograma basal son normales. En un registro ambulatorio del electrocardiograma durante 24 horas se detecta en el periodo vigil fases de corta duraci\u00f3n de ausencia de ondas P previas al QRS con un ritmo de escape de la uni\u00f3n auriculoventricular con QRS estrecho a 40 lpm y una onda al inicio del segmento ST correspondiente a una P retr\u00f3grada. No se detectan periodos de asistolia superiores a 3 segundos. Ante esto usted dir\u00eda:", "full_answer": "\u00bfQu\u00e9 son esos hallazgos en el Holter? Si el nodo sinusal falla, el nodo AV toma las riendas y estimula el ventr\u00edculo. En un 10% de las personas existe conducci\u00f3n retr\u00f3grada, por lo que ese latido generado en el NAV estimula tanto ventr\u00edculos como aur\u00edculas, dando una P retr\u00f3grada como la que se informa en el estudio. En este sentido, sin problema. Asimismo, tampoco hay pausas superiores a 3 segundos que puedan provocar un s\u00edncope cardiog\u00e9nico (que, de hecho, nunca ha sufrido). La opci\u00f3n 1 queda descartada: no es un BAV completo. La 2 tampoco se puede contestar: excluir, lo que se dice excluir\u2026 Es como el nunca/siempre. La 3 no tiene mucho sentido: s\u00ed se podr\u00edan suspender aquellos que deterioran la conducci\u00f3n (betabloqueantes, antagonistas del calcio), pero f\u00e1rmacos que faciliten, como no le pongamos una perfusi\u00f3n de atropina para que se la lleve a casa\u2026 En la 4, afirmar que presenta un bloqueo sinoauricular es rapear un poco: sin un estudio electrofisiol\u00f3gico no puede afirmarse, no necesariamente tiene un bloqueo SA, y eso tampoco indicar\u00eda por s\u00ed solo el implante de un marcapasos. As\u00ed que, finalmente, yo marcar\u00eda la opci\u00f3n 5.", "type": "CARDIOLOG\u00cdA", "options": {"1": "El paciente tiene un BAV de tercer grado y precisa la implantaci\u00f3n de un marcapasos.", "2": "La ausencia de periodos de asistolia >3 segundos excluye una causa cardiaca de los mareos.", "3": "Estar\u00eda indicado un tratamiento farmacol\u00f3gico que incrementara la conducci\u00f3n en el nodo AV.", "4": "El paciente presenta una disfunci\u00f3n sinusal con bloqueo sinoauricular y requiere, por presentar s\u00edntomas, la implantaci\u00f3n deun marcapasos.", "5": "Las alteraciones detectadas en el electrocardiograma ambulatorio son propias de pacientes de esta edad y no hay indicaci\u00f3n de intervenci\u00f3n terap\u00e9utica."}, "correct_option": 5, "explanations": {"1": {"exist": true, "char_ranges": [[482, 534]], "word_ranges": [[82, 92]], "text": "La opci\u00f3n 1 queda descartada: no es un BAV completo."}, "2": {"exist": true, "char_ranges": [[535, 626]], "word_ranges": [[92, 108]], "text": "La 2 tampoco se puede contestar: excluir, lo que se dice excluir\u2026 Es como el nunca/siempre."}, "3": {"exist": true, "char_ranges": [[627, 865]], "word_ranges": [[108, 146]], "text": "La 3 no tiene mucho sentido: s\u00ed se podr\u00edan suspender aquellos que deterioran la conducci\u00f3n (betabloqueantes, antagonistas del calcio), pero f\u00e1rmacos que faciliten, como no le pongamos una perfusi\u00f3n de atropina para que se la lleve a casa\u2026"}, "4": {"exist": true, "char_ranges": [[866, 1098]], "word_ranges": [[146, 184]], "text": "En la 4, afirmar que presenta un bloqueo sinoauricular es rapear un poco: sin un estudio electrofisiol\u00f3gico no puede afirmarse, no necesariamente tiene un bloqueo SA, y eso tampoco indicar\u00eda por s\u00ed solo el implante de un marcapasos."}, "5": {"exist": false, "char_ranges": [], "word_ranges": [], "text": ""}}} {"id": 370, "year": 2016, "question_id_specific": 125, "full_question": "Un paciente ex fumador, diagnosticado previamente de EPOC acude a revisi\u00f3n. En la exploraci\u00f3n f\u00edsica se detecta la existencia de hipocratismo digital de aparici\u00f3n reciente. \u00bfCu\u00e1l es la explicaci\u00f3n m\u00e1s aceptable para este hallazgo en el contexto cl\u00ednico que se describe?", "full_answer": "En una hipocratismo digital de aparicion reciente en un paciente fumador, la primera opci\u00f3n a descartar es el carcinoma broncopulmonar. Las otras patolog\u00edas tambi\u00e9n pueden dar pero no de forma tan s\u00fabita.", "type": "NEUMOLOG\u00cdA Y CIRUG\u00cdA TOR\u00c1CICA", "options": {"1": "Carcinoma pulmonar.", "2": "Bronquiectasias.", "3": "Fibrosis pulmonar.", "4": "Cardiopat\u00eda cian\u00f3tica.", "5": NaN}, "correct_option": 1, "explanations": {"1": {"exist": true, "char_ranges": [[0, 135]], "word_ranges": [[0, 20]], "text": "En una hipocratismo digital de aparicion reciente en un paciente fumador, la primera opci\u00f3n a descartar es el carcinoma broncopulmonar."}, "2": {"exist": true, "char_ranges": [[136, 204]], "word_ranges": [[20, 32]], "text": "Las otras patolog\u00edas tambi\u00e9n pueden dar pero no de forma tan s\u00fabita."}, "3": {"exist": true, "char_ranges": [[136, 204]], "word_ranges": [[20, 32]], "text": "Las otras patolog\u00edas tambi\u00e9n pueden dar pero no de forma tan s\u00fabita."}, "4": {"exist": true, "char_ranges": [[136, 204]], "word_ranges": [[20, 32]], "text": "Las otras patolog\u00edas tambi\u00e9n pueden dar pero no de forma tan s\u00fabita."}, "5": {"exist": false, "char_ranges": [], "word_ranges": [], "text": ""}}} {"id": 332, "year": 2016, "question_id_specific": 88, "full_question": "En unos an\u00e1lisis de rutina de una mujer de 59 a\u00f1os, fumadora de 20 cigarrillos/d\u00eda desde hace 25 a\u00f1os, se detecta una hipercalcemia de 11,3 mg/dL con un f\u00f3sforo de 3,4 mg/dl. NO resultar\u00eda eficiente de entrada:", "full_answer": "Hidroxiprolina urinaria: La excreci\u00f3n urinaria de hidroxiprolina refleja la degradaci\u00f3n del col\u00e1geno \u00f3seo, pero tambi\u00e9n est\u00e1 influida por el metabolismo de otros tejidos (cart\u00edlago, piel) y por la absorci\u00f3n de productos ricos en col\u00e1geno, como carne o gelatinas. Debido a su origen tisular diverso y a su patr\u00f3n metab\u00f3lico, se correlaciona escasamente con la resorci\u00f3n \u00f3sea.", "type": "ENDOCRINOLOG\u00cdA", "options": {"1": "Determinar niveles s\u00e9ricos de PTH.", "2": "Determinar niveles s\u00e9ricos de vitamina D.", "3": "Determinaci\u00f3n de hidroxiprolinuria.", "4": "Una radiograf\u00eda simple de t\u00f3rax.", "5": NaN}, "correct_option": 3, "explanations": {"1": {"exist": false, "char_ranges": [], "word_ranges": [], "text": ""}, "2": {"exist": false, "char_ranges": [], "word_ranges": [], "text": ""}, "3": {"exist": true, "char_ranges": [[0, 262]], "word_ranges": [[0, 38]], "text": "Hidroxiprolina urinaria: La excreci\u00f3n urinaria de hidroxiprolina refleja la degradaci\u00f3n del col\u00e1geno \u00f3seo, pero tambi\u00e9n est\u00e1 influida por el metabolismo de otros tejidos (cart\u00edlago, piel) y por la absorci\u00f3n de productos ricos en col\u00e1geno, como carne o gelatinas."}, "4": {"exist": false, "char_ranges": [], "word_ranges": [], "text": ""}, "5": {"exist": false, "char_ranges": [], "word_ranges": [], "text": ""}}} {"id": 78, "year": 2012, "question_id_specific": 33, "full_question": "Una mujer de 55 a\u00f1os, postmenop\u00e1usica, consulta por astenia y disnea de esfuerzo. En la anamnesis refer\u00eda ligera epigastralgia y pirosis ocasional. No metrorragias. En la anal\u00edtica: Hb6 gr/dL, VCM 69 fl, sideremia 13 microgramos/dL, ferritina 4 ngr/mL. Endoscopia digestiva alata: peque\u00f1a hernia hiatal por deslizamiento sin signos de esofagitis. \u00bfCu\u00e1l es la actitud m\u00e1s correcta?", "full_answer": "Tenemos un caso de anemia ferrop\u00e9nica cr\u00f3nica, sin sangrado evidente. La primera causa de anemia ferrop\u00e9nica es el sangrado oculto de origen digestivo. La existencia de cl\u00ednica sugestiva de patolog\u00eda del tracto digestivo superior ha hecho que se comience el estudio con endoscopia digestiva alta, pero no presenta patolog\u00eda que justifique la anemia por lo que se debe seguir investigando, lo que descarta las respuestas 1 y 2. La ausencia de metrorragia descartar\u00eda la 5 hasta descartar origen digestivo del sangrado. Entre la 3 y la 4, las lesiones de intestino grueso son mucho m\u00e1s habituales y deben descartarse antes de las de intestino delgado. De hecho las recomendaciones de c\u00e1psula endosc\u00f3pica en estudio de anemia son restringidos.", "type": "APARATO DIGESTIVO", "options": {"1": "Administrar hierro oral y ver evoluci\u00f3n de la anemia.", "2": "Tratar con inhibidores de la bomba de protones y evaluar a los tres meses.", "3": "Recomendar una colonoscopia completa.", "4": "Realizar un estudio con c\u00e1psula endosc\u00f3pica.", "5": "Solicitar una evaluaci\u00f3n ginecol\u00f3gica."}, "correct_option": 3, "explanations": {"1": {"exist": true, "char_ranges": [[152, 426]], "word_ranges": [[23, 68]], "text": "La existencia de cl\u00ednica sugestiva de patolog\u00eda del tracto digestivo superior ha hecho que se comience el estudio con endoscopia digestiva alta, pero no presenta patolog\u00eda que justifique la anemia por lo que se debe seguir investigando, lo que descarta las respuestas 1 y 2."}, "2": {"exist": true, "char_ranges": [[152, 426]], "word_ranges": [[23, 68]], "text": "La existencia de cl\u00ednica sugestiva de patolog\u00eda del tracto digestivo superior ha hecho que se comience el estudio con endoscopia digestiva alta, pero no presenta patolog\u00eda que justifique la anemia por lo que se debe seguir investigando, lo que descarta las respuestas 1 y 2."}, "3": {"exist": true, "char_ranges": [[518, 649]], "word_ranges": [[81, 105]], "text": "Entre la 3 y la 4, las lesiones de intestino grueso son mucho m\u00e1s habituales y deben descartarse antes de las de intestino delgado."}, "4": {"exist": true, "char_ranges": [[518, 649]], "word_ranges": [[81, 105]], "text": "Entre la 3 y la 4, las lesiones de intestino grueso son mucho m\u00e1s habituales y deben descartarse antes de las de intestino delgado."}, "5": {"exist": true, "char_ranges": [[427, 517]], "word_ranges": [[68, 81]], "text": "La ausencia de metrorragia descartar\u00eda la 5 hasta descartar origen digestivo del sangrado."}}} {"id": 397, "year": 2016, "question_id_specific": 140, "full_question": "Usted valora a un paciente de 66 a\u00f1os con dolor inguinal acentuado con la bipedestaci\u00f3n prolongada algunos d\u00edas al mes. Una radiograf\u00eda simple de caderas muestra estrechamiento del espacio articular f\u00e9moro-acetabular, esclerosis y ostefitos. \u00bfCu\u00e1l es su actitud?", "full_answer": "Presenta una coxartrosis (no se necesitan m\u00e1s pruebas de imagen), y se agotan las opciones conservadoras antes de optar por el tratamiento quir\u00fargico. Los opioides d\u00e9biles no son el tratamiento inicial de elecci\u00f3n.", "type": "TRAUMATOLOG\u00cdA Y ORTOPED\u00cdA", "options": {"1": "Hago el diagn\u00f3stico de coxartrosis y env\u00edo al traumat\u00f3logo para colocar una pr\u00f3tesis de cadera.", "2": "Inicio tratamiento con opioides d\u00e9biles que han demostrado evidencia en detener la progresi\u00f3n de la enfermedad.", "3": "Instauro tratamiento con paracetamol, explico que la evoluci\u00f3n es muy variable y la indicaci\u00f3n quir\u00fargica depende de la funcionalidad y control del dolor.", "4": "Por las caracter\u00edsticas radiol\u00f3gicas descritas, necesito una RMN de cadera antes de tomar una decisi\u00f3n terap\u00e9utica.", "5": NaN}, "correct_option": 3, "explanations": {"1": {"exist": false, "char_ranges": [], "word_ranges": [], "text": ""}, "2": {"exist": true, "char_ranges": [[0, 214]], "word_ranges": [[0, 33]], "text": "Presenta una coxartrosis (no se necesitan m\u00e1s pruebas de imagen), y se agotan las opciones conservadoras antes de optar por el tratamiento quir\u00fargico. Los opioides d\u00e9biles no son el tratamiento inicial de elecci\u00f3n."}, "3": {"exist": true, "char_ranges": [[0, 214]], "word_ranges": [[0, 33]], "text": "Presenta una coxartrosis (no se necesitan m\u00e1s pruebas de imagen), y se agotan las opciones conservadoras antes de optar por el tratamiento quir\u00fargico. Los opioides d\u00e9biles no son el tratamiento inicial de elecci\u00f3n."}, "4": {"exist": false, "char_ranges": [], "word_ranges": [], "text": ""}, "5": {"exist": false, "char_ranges": [], "word_ranges": [], "text": ""}}} {"id": 275, "year": 2016, "question_id_specific": 74, "full_question": "Un hombre de 67 a\u00f1os, diagnosticado de cirrosis hep\u00e1tica y bebedor activo, ingresa por cuadro de distensi\u00f3n abdominal progresiva con malestar difuso, de dos semanas de evoluci\u00f3n. No refiere fiebre ni otros s\u00edntomas. A la exploraci\u00f3n destaca matidez cambiante a la percusi\u00f3n abdominal, con ausencia de edemas. Se realiza una paracentesis diagn\u00f3stica, encontrando un l\u00edquido ligeramente turbio, con 2.300 c\u00e9lulas/mL, de las cuales 30% son linfocitos, 60% polimorfonucleares y 10% hemat\u00edes. \u00bfCu\u00e1l es la primera medida terap\u00e9utica que pautar\u00eda en este paciente de forma inmediata?", "full_answer": "Se trata de una peritonitis bacteriana espont\u00e1nea (PBE), definida por presencia de > 250 polimorfonucleares/mL en el l\u00edquido asc\u00edtico (en este caso tiene 1.380, que son el 60% de 2.300). Es indicaci\u00f3n de iniciar cefalosporinas de 3\u00aa generaci\u00f3n (ceftriaxona o cefotaxima) y alb\u00famina. Lo dem\u00e1s son tratamientos que pueden esperar a una mejor\u00eda (no hablan de ascitis a tensi\u00f3n).", "type": "APARATO DIGESTIVO", "options": {"1": "Restricci\u00f3n de sal y l\u00edquidos.", "2": "Tratamiento diur\u00e9tico con espironolactona oral.", "3": "Tratamiento con una cefalosoporina de tercera generaci\u00f3n.", "4": "Paracentesis evacuadora.", "5": NaN}, "correct_option": 3, "explanations": {"1": {"exist": true, "char_ranges": [[283, 375]], "word_ranges": [[43, 59]], "text": "Lo dem\u00e1s son tratamientos que pueden esperar a una mejor\u00eda (no hablan de ascitis a tensi\u00f3n)."}, "2": {"exist": true, "char_ranges": [[283, 375]], "word_ranges": [[43, 59]], "text": "Lo dem\u00e1s son tratamientos que pueden esperar a una mejor\u00eda (no hablan de ascitis a tensi\u00f3n)."}, "3": {"exist": true, "char_ranges": [[0, 282]], "word_ranges": [[0, 43]], "text": "Se trata de una peritonitis bacteriana espont\u00e1nea (PBE), definida por presencia de > 250 polimorfonucleares/mL en el l\u00edquido asc\u00edtico (en este caso tiene 1.380, que son el 60% de 2.300). Es indicaci\u00f3n de iniciar cefalosporinas de 3\u00aa generaci\u00f3n (ceftriaxona o cefotaxima) y alb\u00famina."}, "4": {"exist": true, "char_ranges": [[283, 375]], "word_ranges": [[43, 59]], "text": "Lo dem\u00e1s son tratamientos que pueden esperar a una mejor\u00eda (no hablan de ascitis a tensi\u00f3n)."}, "5": {"exist": false, "char_ranges": [], "word_ranges": [], "text": ""}}} {"id": 418, "year": 2018, "question_id_specific": 77, "full_question": "Mujer de 80 a\u00f1os que acude urgencias con dolor abdominal de inicio en epigastrio e irradiado posteriormente a fosa iliaca izquierda. Se acompa\u00f1a de fiebre de 37,5\u00baC, Se le realiza una TC abdominal, objetivando inflamaci\u00f3n en las paredes del sigma y absceso mesenterico de 2 cm. El tratamiento de elecci\u00f3n es:", "full_answer": "Nos preguntan por el manejo de una patolog\u00eda muy habitual, la diverticulitis. Y nos plantean las distintas opciones terape\u00faticas seg\u00fan lo avanzada que se encuentre. La respuesta correcta es la 1, no cumple criterios para requerir drenaje mediante punci\u00f3n guiada (el absceso no es mayor de 5-6 cm). La colostom\u00eda de descarga en este caso no tiene sentido alguno, ya que no se trata de solventar un cuadro obstructivo. El drenaje mediante cirug\u00eda laparosc\u00f3pica no tiene indicaci\u00f3n en este caso. Podr\u00eda estar indicado en ausencia de posibilidad de drenaje percut\u00e1neo tras fracaso de tratamiento conservador, aunque es una opci\u00f3n muy discutida en este momento. La sigmoidectom\u00eda y anastomosis tampoco es requerida. En un alto n\u00famero de casos tras la resoluci\u00f3n del cuadro se reducen los brotes y pueden llegar a no volver a padecerlos con dieta y h\u00e1bitos higi\u00e9nico diet\u00e9ticos.", "type": "CIRUG\u00cdA GENERAL", "options": {"1": "lngreso en planta con dieta absoluta y tratamiento antibi\u00f3tico de amplio espectro.", "2": "Colostomia de descarga.", "3": "Drenaje mediante cirug\u00eda laparosc\u00f3pica.", "4": "Cirug\u00eda urgente con sigmoidectom\u00eda y anastomosis colorrectal.", "5": NaN}, "correct_option": 1, "explanations": {"1": {"exist": true, "char_ranges": [[711, 872]], "word_ranges": [[111, 140]], "text": "En un alto n\u00famero de casos tras la resoluci\u00f3n del cuadro se reducen los brotes y pueden llegar a no volver a padecerlos con dieta y h\u00e1bitos higi\u00e9nico diet\u00e9ticos."}, "2": {"exist": true, "char_ranges": [[298, 416]], "word_ranges": [[48, 69]], "text": "La colostom\u00eda de descarga en este caso no tiene sentido alguno, ya que no se trata de solventar un cuadro obstructivo."}, "3": {"exist": true, "char_ranges": [[196, 297]], "word_ranges": [[31, 48]], "text": "no cumple criterios para requerir drenaje mediante punci\u00f3n guiada (el absceso no es mayor de 5-6 cm)."}, "4": {"exist": true, "char_ranges": [[657, 710]], "word_ranges": [[104, 111]], "text": "La sigmoidectom\u00eda y anastomosis tampoco es requerida."}, "5": {"exist": false, "char_ranges": [], "word_ranges": [], "text": ""}}} {"id": 200, "year": 2013, "question_id_specific": 75, "full_question": "Mujer de 75 a\u00f1os que consulta por insomnio. Refiere que desde hace a\u00f1os se despierta por la noche con sensaci\u00f3n de hormigueo, quemaz\u00f3n en las piernas y a veces en los brazos; estos s\u00edntomas se presentan tambi\u00e9n por la tarde. Nota mejor\u00eda al mover las piemas, pero los s\u00edntomas recurren durante el reposo, por lo que no puede volver a conciliar el sue\u00f1o. Este cuadro nos podr\u00eda orientar al diagn\u00f3stico de un s\u00edndrome de piernas inquietas \u00bfCu\u00e1l de las siguientes es FALSA?", "full_answer": "El diagn\u00f3stico es basado en criterios cl\u00ednicos, por lo que la biopsia muscular s\u00f3lo se realiza para descartar otras enfermedades, no para confirmar \u00e9sta. Se debe realizar un perfil f\u00e9rrico, determinaci\u00f3n de hormonas tiroideas y de vitaminas para descartar otras enfermedades con sintomatolog\u00eda similar. El tratamiento es sintom\u00e1tico y se instaura cuando interfiere en el sue\u00f1o o en la calidad de vida, y el tratamiento de primera elecci\u00f3n son los agonistas dopamin\u00e9rgicos como el pramipexol y el ropirinol.", "type": "NEUROLOG\u00cdA", "options": {"1": "El diagn\u00f3stico de esta afectaci\u00f3n se basa en criterios cl\u00ednicos.", "2": "Habr\u00eda que realizar una anal\u00edtica b\u00e1sica que incluyera perfil f\u00e9rrico, hormonas tiroideas y B 12 y \u00e1cido f\u00f3lico.", "3": "El tratamiento estaria indicado en pacientes que presentan alteraci\u00f3n del sue\u00f1o o de la calidad de vida pero no altera el curso de la enfermedad.", "4": "El diagn\u00f3stico se confirma con biopsia muscular.", "5": "El pramipexol y el ropirinol se utilizan en el tratamiento."}, "correct_option": 4, "explanations": {"1": {"exist": false, "char_ranges": [], "word_ranges": [], "text": ""}, "2": {"exist": true, "char_ranges": [[154, 302]], "word_ranges": [[24, 44]], "text": "Se debe realizar un perfil f\u00e9rrico, determinaci\u00f3n de hormonas tiroideas y de vitaminas para descartar otras enfermedades con sintomatolog\u00eda similar."}, "3": {"exist": true, "char_ranges": [[303, 506]], "word_ranges": [[44, 78]], "text": "El tratamiento es sintom\u00e1tico y se instaura cuando interfiere en el sue\u00f1o o en la calidad de vida, y el tratamiento de primera elecci\u00f3n son los agonistas dopamin\u00e9rgicos como el pramipexol y el ropirinol."}, "4": {"exist": true, "char_ranges": [[0, 153]], "word_ranges": [[0, 24]], "text": "El diagn\u00f3stico es basado en criterios cl\u00ednicos, por lo que la biopsia muscular s\u00f3lo se realiza para descartar otras enfermedades, no para confirmar \u00e9sta."}, "5": {"exist": true, "char_ranges": [[303, 506]], "word_ranges": [[44, 78]], "text": "El tratamiento es sintom\u00e1tico y se instaura cuando interfiere en el sue\u00f1o o en la calidad de vida, y el tratamiento de primera elecci\u00f3n son los agonistas dopamin\u00e9rgicos como el pramipexol y el ropirinol."}}} {"id": 469, "year": 2020, "question_id_specific": 137, "full_question": "Mujer de 93 a\u00f1os con antecedentes de EPOC, HTA, dislipemia, fibrilaci\u00f3n auricular y cardiopat\u00eda isqu\u00e9mica. Vive sola. Presenta fracci\u00f3n de eyecci\u00f3n ventricular 53%, creatinina 1,5 mg/dL, coagulaci\u00f3n normal. Recibe tratamiento con broncodilatadores inhalados y AAS 100 mg al d\u00eda. Sufre una ca\u00edda en la calle, tras la cual le resulta imposible la bipedestaci\u00f3n y sedestaci\u00f3n, con dolor intenso en la cadera derecha a las movilizaciones, gran acortamiento y rotaci\u00f3n externa de la pierna. \u00bfCu\u00e1ndo y c\u00f3mo debe tratarse?", "full_answer": "\u201cActualmente se est\u00e1 desarrollando un nuevo enfoque para el tratamiento de estos pacientes, el cual trata de coordinar desde el principio la actividad de los diferentes sectores implicados evitando que el traslado del paciente de uno a otro se produzca al finalizar el tratamiento de cada parte. Con esta idea nacen las denominadas \u00abunidades de ortogeriatr\u00eda\u00bb, que combinan la actividad quir\u00fargica del cirujano con la de la rehabilitaci\u00f3n del paciente de forma inmediata y los controles y el seguimiento de medicina interna. Ya desde el inicio del proceso se pone en marcha la actividad de los asistentes sociales y las diferentes ayudas vitales y de soporte, para que la recuperaci\u00f3n sea lo m\u00e1s r\u00e1pida y completa posible. \u2026 En la mayor\u00eda de los casos, la intervenci\u00f3n quir\u00fargica debe realizarse lo antes posible, en cuanto el enfermo est\u00e9 estabilizado desde el punto de vista m\u00e9dico. Existe contraindicaci\u00f3n\u00a1 de intervenir a pacientes con comorbilidad m\u00e9dica inestable, pero en varios estudios se ha visto que la demora de la intervenci\u00f3n quir\u00fargica m\u00e1s all\u00e1 de 48 h tras el ingreso aumenta la mortalidad. Es por ello que el momento ideal para intervenir a estos pacientes es tras conseguir la estabilizaci\u00f3n de sus patolog\u00edas m\u00e9dicas, procurando que esto tenga lugar antes de que transcurran 48 h.\u201d No hay ning\u00fan dato que sugiera que tenga alguna comorbilidad inestable en el momento actual, por lo que interesa la cirug\u00eda en menos de 48 horas y el manejo ortogeri\u00e1trico.", "type": "CIRUG\u00cdA ORTOP\u00c9DICA Y TRAUMATOLOG\u00cdA", "options": {"1": "Ingreso para ecograf\u00eda transtor\u00e1cica, valoraci\u00f3n por Cardiolog\u00eda y despu\u00e9s reducci\u00f3n cerrada y fijaci\u00f3n con tornillo placa.", "2": "Ingreso para valoraci\u00f3n por Nefrolog\u00eda y Cardiolog\u00eda, y despu\u00e9s decidir\u00e1 Traumatolog\u00eda si usa una artroplastia o un clavo trocant\u00e9rico.", "3": "Ingreso en Medicina Interna / Geriatr\u00eda, estabilizaci\u00f3n progresiva de las comorbilidades, tratamiento ortop\u00e9dico conservador.", "4": "Reducci\u00f3n cerrada y fijaci\u00f3n de la fractura antes de 48 horas, sedestaci\u00f3n muy precoz, manejo ortogeri\u00e1trico.", "5": NaN}, "correct_option": 4, "explanations": {"1": {"exist": false, "char_ranges": [], "word_ranges": [], "text": ""}, "2": {"exist": false, "char_ranges": [], "word_ranges": [], "text": ""}, "3": {"exist": false, "char_ranges": [], "word_ranges": [], "text": ""}, "4": {"exist": true, "char_ranges": [[1107, 1473]], "word_ranges": [[179, 241]], "text": "Es por ello que el momento ideal para intervenir a estos pacientes es tras conseguir la estabilizaci\u00f3n de sus patolog\u00edas m\u00e9dicas, procurando que esto tenga lugar antes de que transcurran 48 h.\u201d No hay ning\u00fan dato que sugiera que tenga alguna comorbilidad inestable en el momento actual, por lo que interesa la cirug\u00eda en menos de 48 horas y el manejo ortogeri\u00e1trico."}, "5": {"exist": false, "char_ranges": [], "word_ranges": [], "text": ""}}} {"id": 378, "year": 2016, "question_id_specific": 132, "full_question": "Un paciente de 72 a\u00f1os acude a consulta con p\u00e9rdida de memoria progresiva y dificultad para encontrar las palabras de un a\u00f1o de evoluci\u00f3n. \u00bfCu\u00e1l es la exploraci\u00f3n m\u00e1s \u00fatil para caracterizar el tipo de d\u00e9ficit cognitivo que presenta?", "full_answer": "La caracterizaci\u00f3n de los precesos de deterioro cognitivo sigue siendo cl\u00ednica, siendo las pruebas de imagen (RM o PET) una herramienta de apoyo en algunas ocasiones. Ser\u00eda la opci\u00f3n correcta la primera porque permite medir de forma estandarizada cada uno de los dominios cognitivos lo que nos permitir\u00e1 encuadrar los d\u00e9ficis del paciente en un s\u00edndrome concreto y la monitorizaci\u00f3n del deterioro con exploraciones repetidas en el tiempo.", "type": "NEUROLOG\u00cdA", "options": {"1": "Evaluaci\u00f3n neuropsicol\u00f3gica.", "2": "Resonancia magn\u00e9tica cerebral.", "3": "Tomograf\u00eda de Emisi\u00f3n de Positrones.", "4": "Electroencefalograma.", "5": NaN}, "correct_option": 1, "explanations": {"1": {"exist": true, "char_ranges": [[0, 282]], "word_ranges": [[0, 44]], "text": "La caracterizaci\u00f3n de los precesos de deterioro cognitivo sigue siendo cl\u00ednica, siendo las pruebas de imagen (RM o PET) una herramienta de apoyo en algunas ocasiones. Ser\u00eda la opci\u00f3n correcta la primera porque permite medir de forma estandarizada cada uno de los dominios cognitivos"}, "2": {"exist": false, "char_ranges": [], "word_ranges": [], "text": ""}, "3": {"exist": true, "char_ranges": [[0, 282]], "word_ranges": [[0, 44]], "text": "La caracterizaci\u00f3n de los precesos de deterioro cognitivo sigue siendo cl\u00ednica, siendo las pruebas de imagen (RM o PET) una herramienta de apoyo en algunas ocasiones. Ser\u00eda la opci\u00f3n correcta la primera porque permite medir de forma estandarizada cada uno de los dominios cognitivos"}, "4": {"exist": false, "char_ranges": [], "word_ranges": [], "text": ""}, "5": {"exist": false, "char_ranges": [], "word_ranges": [], "text": ""}}} {"id": 582, "year": 2022, "question_id_specific": 208, "full_question": "Var\u00f3n de 53 a\u00f1os sin antecedentes de inter\u00e9s que tras un mal movimiento en el gimnasio nota un dolor lumbar que al pasar las horas se irradia hacia la pierna derecha por la parte posterior y llega al borde lateral del pie. Presenta en la exploraci\u00f3n maniobra de Las\u00e8gue positiva a 40o, reflejo aqu\u00edleo abolido y no se puede poner de puntillas. El diagn\u00f3stico etiol\u00f3gico m\u00e1s probable es:", "full_answer": "Describe afectaci\u00f3n de ra\u00edz S1 por la incapacidad para andar de puntillas y la ausencia de reflejo aqu\u00edleo, adem\u00e1s de la distribuci\u00f3n del dolor por la pierna. De nuestras opciones, la causa que puede producir un da\u00f1o en S1 es la hernia discal L5-S1.", "type": "NEUROLOG\u00cdA", "options": {"1": "Hernia discal L2-L3 derecha.", "2": "Hernia discal L3-L4 derecha.", "3": "Hernia discal L4-L5 derecha.", "4": "Hernia discal L5-S1 derecha.", "5": NaN}, "correct_option": 4, "explanations": {"1": {"exist": false, "char_ranges": [], "word_ranges": [], "text": ""}, "2": {"exist": false, "char_ranges": [], "word_ranges": [], "text": ""}, "3": {"exist": false, "char_ranges": [], "word_ranges": [], "text": ""}, "4": {"exist": true, "char_ranges": [[0, 249]], "word_ranges": [[0, 44]], "text": "Describe afectaci\u00f3n de ra\u00edz S1 por la incapacidad para andar de puntillas y la ausencia de reflejo aqu\u00edleo, adem\u00e1s de la distribuci\u00f3n del dolor por la pierna. De nuestras opciones, la causa que puede producir un da\u00f1o en S1 es la hernia discal L5-S1."}, "5": {"exist": false, "char_ranges": [], "word_ranges": [], "text": ""}}} {"id": 160, "year": 2013, "question_id_specific": 117, "full_question": "Hombre de 35 a\u00f1os que presenta hematuria tras infecciones respiratorias desde hace varios a\u00f1os, en la anal\u00edtica de sangre presenta creatinina 1 mg/dl sin otras alteraciones y en la orina aparecen hemat\u00edes 50/campo siendo el 80% dism\u00f3rficos, con proteinuria de 0,8 gramos en 24 horas. \u00bfCu\u00e1l es el diagn\u00f3stico m\u00e1s probable?", "full_answer": "La respuesta correcta es: 3. Nefropat\u00eda IgA. Dado que la presentaci\u00f3n cl\u00ednica de esta nefropat\u00eda aparece de forma muy inespec\u00edfica, para identificar la enfermedad responsable debemos basarnos en su epidemiolog\u00eda. Al ser un var\u00f3n joven, con historia de infecciones respiratorias previas, se establece una relaci\u00f3n entre esas infecciones y la patolog\u00eda renal. Bas\u00e1ndonos en ello pensamos en la Nefropat\u00eda IgA, que adem\u00e1s de ser la glomerulonefritis m\u00e1s frecuente se presenta en la mayor\u00eda de los casos de la forma que se describe.", "type": "NEFROLOG\u00cdA", "options": {"1": "Nefropat\u00eda de cambios m\u00ednimos.", "2": "Glomerulonefritis membranosa.", "3": "Nefropat\u00eda IgA.", "4": "Glomerulonefritis proliferativa y difusa.", "5": "Glomeruloesclerosis focal y segmentaria primaria."}, "correct_option": 3, "explanations": {"1": {"exist": false, "char_ranges": [], "word_ranges": [], "text": ""}, "2": {"exist": false, "char_ranges": [], "word_ranges": [], "text": ""}, "3": {"exist": true, "char_ranges": [[213, 528]], "word_ranges": [[30, 82]], "text": "Al ser un var\u00f3n joven, con historia de infecciones respiratorias previas, se establece una relaci\u00f3n entre esas infecciones y la patolog\u00eda renal. Bas\u00e1ndonos en ello pensamos en la Nefropat\u00eda IgA, que adem\u00e1s de ser la glomerulonefritis m\u00e1s frecuente se presenta en la mayor\u00eda de los casos de la forma que se describe."}, "4": {"exist": false, "char_ranges": [], "word_ranges": [], "text": ""}, "5": {"exist": false, "char_ranges": [], "word_ranges": [], "text": ""}}} {"id": 312, "year": 2016, "question_id_specific": 217, "full_question": "Mujer de 84 a\u00f1os que acude por p\u00e9rdida de visi\u00f3n en el ojo izquierdo de 4 d\u00edas de evoluci\u00f3n acompa\u00f1ada de metamorfopsias. En la m\u00e1cula se aprecian abundantes exudados duros, dos peque\u00f1as hemorragias profundas y un desprendimiento de la retina neurosensorial localizado. En el ojo contralateral presenta abundantes drusas blandas. Ante este cuadro, \u00bfCu\u00e1l de los siguientes diagn\u00f3sticos le parece m\u00e1s probable?", "full_answer": "Pregunta en principio f\u00e1cil, incide en una enfermedad que se repite con frecuencia en los ex\u00e1menes MIR. La descripci\u00f3n es la t\u00edpica de la opci\u00f3n 2. Se trata de una DMAE exudativa (aunque como dec\u00edamos en la pregunta 215, con la clasificaci\u00f3n nueva, hablar\u00edamos de una DMAE avanzada en su variante neovascular). De cualquier manera, tenemos una paciente mayor con p\u00e9rdida de visi\u00f3n y metamorfopsias en un ojo. En la exploraci\u00f3n hay lesiones en la m\u00e1cula. Ya con eso podemos descartar el desprendimiento de v\u00edtreo posterior agudo, que no produce p\u00e9rdida visual, ni metamorfopsias, ni lesiones en la m\u00e1cula. Y tambi\u00e9n la neuropat\u00eda \u00f3ptica isqu\u00e9mica anterior no arter\u00edtica, porque \u00e9sta, si bien produce p\u00e9rdida visual y es t\u00edpica de personas mayores, no produce metamorfopsias. Y no hay lesiones en la m\u00e1cula sino edema de papila. En la obstrucci\u00f3n de arteria central de la retina no es especialmente caracter\u00edstico las metamorfopsias (s\u00ed la perdida visual y la edad avanzada), pero var\u00edan las lesiones en el fondo del ojo. En el caso de oclusi\u00f3n arterial, hay un edema intracelular que se ve como ausencia de hemorragias y un fondo blanquecino, p\u00e1lido. Adem\u00e1s de que afecta en global a toda la retina, y puede aparecer una mancha rojo cereza en la m\u00e1cula. En este caso las lesiones son totalmente diferentes. Nos describen exudados duros, hemorragias profundas y desprendimiento de retina neurosensorial localizado. Todo ello en la m\u00e1cula. Son los hallazgos caracter\u00edsticos de la DMAE neovascular (o exudativa). Como pista, en el otro ojo hay abundantes drusas blandas. Las drusas los los hallazgos t\u00edpicos la DMAE. As\u00ed que tiene DMAE en ambos ojos y en el ojo derecho recientemente se ha complicado con la aparici\u00f3n de la neovascularizaci\u00f3n.", "type": "OFTALMOLOG\u00cdA", "options": {"1": "Desprendimiento posterior de v\u00edtreo agudo.", "2": "Degeneraci\u00f3n macular asociada a la edad (DMAE) exudativa.", "3": "Obstrucci\u00f3n de arteria central de la retina.", "4": "Neuropat\u00eda \u00f3ptica isqu\u00e9mica anterior no arter\u00edtica.", "5": NaN}, "correct_option": 2, "explanations": {"1": {"exist": true, "char_ranges": [[409, 604]], "word_ranges": [[68, 99]], "text": "En la exploraci\u00f3n hay lesiones en la m\u00e1cula. Ya con eso podemos descartar el desprendimiento de v\u00edtreo posterior agudo, que no produce p\u00e9rdida visual, ni metamorfopsias, ni lesiones en la m\u00e1cula."}, "2": {"exist": true, "char_ranges": [[1306, 1508]], "word_ranges": [[217, 244]], "text": "Nos describen exudados duros, hemorragias profundas y desprendimiento de retina neurosensorial localizado. Todo ello en la m\u00e1cula. Son los hallazgos caracter\u00edsticos de la DMAE neovascular (o exudativa)."}, "3": {"exist": true, "char_ranges": [[1020, 1305]], "word_ranges": [[167, 217]], "text": "En el caso de oclusi\u00f3n arterial, hay un edema intracelular que se ve como ausencia de hemorragias y un fondo blanquecino, p\u00e1lido. Adem\u00e1s de que afecta en global a toda la retina, y puede aparecer una mancha rojo cereza en la m\u00e1cula. En este caso las lesiones son totalmente diferentes."}, "4": {"exist": true, "char_ranges": [[605, 773]], "word_ranges": [[99, 124]], "text": "Y tambi\u00e9n la neuropat\u00eda \u00f3ptica isqu\u00e9mica anterior no arter\u00edtica, porque \u00e9sta, si bien produce p\u00e9rdida visual y es t\u00edpica de personas mayores, no produce metamorfopsias."}, "5": {"exist": false, "char_ranges": [], "word_ranges": [], "text": ""}}} {"id": 24, "year": 2011, "question_id_specific": 115, "full_question": "Un hombre de 68 a\u00f1os es evaluado por presentar Fiebr51e, p\u00e9rdida de peso y disnea de esfuerzo. La EF revela un nuevo soplo diast\u00f3lico a\u00f3rtico y estigmas de embolismos perif\u00e9ricos. La ecocardio muestra una vegetaci\u00f3n de 1 cm en la v\u00e1lvula a\u00f3rtica y en dos hemocultivos se a\u00edsla Streptococcus bovis sensible a penicilina. Tras iniciar tratamiento de endocarditis, \u00bfCu\u00e1l de las siguientes exploraciones nos podr\u00eda ayuda en el diagn\u00f3stico?", "full_answer": "Otro regalo de pregunta, la asociaci\u00f3n de endocarditis por S.bovis y el c\u00e1ncer de colon. Por cierto, es gracioso que den Rx T\u00f3rax como prueba a realizar... \u00bfme est\u00e1s diciendo que a un paciente con fiebre y disnea le haces antes una ecocardio que una rx de t\u00f3rax simple? Y por otro lado... \u00bfpor qu\u00e9 los que escriben el MIR no hacen las preguntas m\u00e1s cortas sin tanta vuelta? Si lo que quieren es preguntar la prueba a realizar a un paciente con endocarditis por S.bovis, pues podr\u00edan lanzar la pregunta directamente sin contarnos hasta los lunares del paciente...", "type": "INFECCIOSAS", "options": {"1": "Prueba de tolerancia a la glucosa.", "2": "Rx T\u00f3rax.", "3": "Endoscopia digestiva alta.", "4": "Colonoscopia.", "5": "Ecograf\u00eda abdominal."}, "correct_option": 4, "explanations": {"1": {"exist": false, "char_ranges": [], "word_ranges": [], "text": ""}, "2": {"exist": false, "char_ranges": [], "word_ranges": [], "text": ""}, "3": {"exist": false, "char_ranges": [], "word_ranges": [], "text": ""}, "4": {"exist": true, "char_ranges": [[25, 88]], "word_ranges": [[4, 15]], "text": "la asociaci\u00f3n de endocarditis por S.bovis y el c\u00e1ncer de colon."}, "5": {"exist": false, "char_ranges": [], "word_ranges": [], "text": ""}}} {"id": 260, "year": 2014, "question_id_specific": 99, "full_question": "Una mujer de 45 a\u00f1os acude a la consulta remitida desde Cirut\u00eda con el diagn\u00f3stico de un tumor neuroendocrino diagnosticado tras pancreatectom\u00eda parcial por un tumor de 2 cms en cola del p\u00e1ncreas. El tumor hab\u00eda sido detectado de forma casual en una TAC abdominal solicitada para completar en estudio de un quiste simple hep\u00e1tico. Interrogando a la paciente destacan como antecedentes reglas irregulares, estando en amenorrea desde hace 6 meses, y c\u00f3licos renoureterales de repetici\u00f3n desde los 20 a\u00f1os de edad por los que ha precisado litotricia en varias ocasiones. Adem\u00e1s presenta antecedentes familiares de c\u00f3licos renoureterales. \u00bfCu\u00e1l es su sospecha diagn\u00f3stica?", "full_answer": "Pregunta cl\u00ednica muy bonita de MEN. Tumor neuroendocrino pancre\u00e1tico + adenoma hipofisario (prolactinoma) + hiperparatiroidismo (c\u00f3licos renoureterales en gente joven): MEN 1. Dificultad media ya que hay que saber la triada de MEN y reconocer los diferentes tumores.", "type": "ENDOCRINOLOG\u00cdA", "options": {"1": "Una neoplasia endocrina m\u00faltiple tipo 1 o S\u00edndrome de Wermer.", "2": "Una neoplasia endocrina m\u00faltiple tipo 2 A o S\u00edndrome de Sipple.", "3": "Una neoplasia endocrina m\u00faltiple tipo 2B.", "4": "Somatostatinoma.", "5": "Un tumor neuroendocrino productor de PTH."}, "correct_option": 1, "explanations": {"1": {"exist": true, "char_ranges": [[36, 175]], "word_ranges": [[6, 22]], "text": "Tumor neuroendocrino pancre\u00e1tico + adenoma hipofisario (prolactinoma) + hiperparatiroidismo (c\u00f3licos renoureterales en gente joven): MEN 1."}, "2": {"exist": false, "char_ranges": [], "word_ranges": [], "text": ""}, "3": {"exist": false, "char_ranges": [], "word_ranges": [], "text": ""}, "4": {"exist": false, "char_ranges": [], "word_ranges": [], "text": ""}, "5": {"exist": false, "char_ranges": [], "word_ranges": [], "text": ""}}} {"id": 211, "year": 2014, "question_id_specific": 87, "full_question": "Una mujer de 78 a\u00f1os acude a Urgencias por dolor en fosa il\u00edaca izquierda de 24 horas de evoluci\u00f3n asociado a fiebre y alg\u00fan v\u00f3mito ocasional. A la exploraci\u00f3n destaca dolor a la palpaci\u00f3n de forma selectiva en la fosa il\u00edaca izquierda con sensaci\u00f3n de ocupaci\u00f3n, defensa y descompresi\u00f3n positiva. Ante la sospecha de diverticulitis aguda, \u00bfcu\u00e1l de las siguientes afirmaciones es correcta?", "full_answer": "La 1 es incorrecta porque las pruebas de imagen de elecci\u00f3n ser\u00edan ecograf\u00eda o TC; la 3 es incorrecta porque el abordaje laparosc\u00f3pico est\u00e1 indicado de entrada; la 4 es incorrecta porque la indicaci\u00f3n en el primer episodio es para menores de 50 a\u00f1os, en el resto se indica despu\u00e9s del segundo episodio o en casos de cl\u00ednica persistente o en sospecha de neoplasia; y la 5 es incorrecta porque la cirug\u00eda en casos de perforaci\u00f3n con peritonitis siempre debe resecar el segmento de colon afecto para controlar el foco de la infecci\u00f3n.", "type": "CIRUG\u00cdA GENERAL", "options": {"1": "La exploraci\u00f3n complementaria m\u00e1s segura y de mejor rendimiento es el enema con contraste baritado.", "2": "En caso de absceso p\u00e9lvico contenido, est\u00e1 indicada la colocaci\u00f3n de un drenaje percut\u00e1neo guiado con TAC o ecograf\u00eda.", "3": "En caso de precisar intervenci\u00f3n quir\u00fargica tras solucionarse el episodio agudo, el abordaje laparosc\u00f3pico est\u00e1 contraindicado.", "4": "En caso de diverticulitis aguda no complicada, est\u00e1 indicada la sigmoidectom\u00eda electiva tras la curaci\u00f3n del primer episodio agudo.", "5": "Si se produjera una peritonitis generalizada, la t\u00e9cnica quir\u00fargica m\u00e1s adecuada es la pr\u00e1ctica de una colostom\u00eda derivativa sin resecci\u00f3n del segmento sigmoideo afectado."}, "correct_option": 2, "explanations": {"1": {"exist": true, "char_ranges": [[0, 82]], "word_ranges": [[0, 15]], "text": "La 1 es incorrecta porque las pruebas de imagen de elecci\u00f3n ser\u00edan ecograf\u00eda o TC;"}, "2": {"exist": false, "char_ranges": [], "word_ranges": [], "text": ""}, "3": {"exist": true, "char_ranges": [[83, 160]], "word_ranges": [[15, 27]], "text": "la 3 es incorrecta porque el abordaje laparosc\u00f3pico est\u00e1 indicado de entrada;"}, "4": {"exist": true, "char_ranges": [[161, 363]], "word_ranges": [[27, 64]], "text": "la 4 es incorrecta porque la indicaci\u00f3n en el primer episodio es para menores de 50 a\u00f1os, en el resto se indica despu\u00e9s del segundo episodio o en casos de cl\u00ednica persistente o en sospecha de neoplasia;"}, "5": {"exist": true, "char_ranges": [[366, 531]], "word_ranges": [[65, 93]], "text": "la 5 es incorrecta porque la cirug\u00eda en casos de perforaci\u00f3n con peritonitis siempre debe resecar el segmento de colon afecto para controlar el foco de la infecci\u00f3n."}}} {"id": 162, "year": 2013, "question_id_specific": 121, "full_question": "Un paciente de 52 a\u00f1os con enfermedad renal cr\u00f3nica estadio V, secundaria a Poliquistosis autos\u00f3mica dominante, recibe un injerto renal de donante cad\u00e1ver. Un hermano suyo de 34 a\u00f1os acude a visitarlo y refiere desconocer si padece la enfermedad por no haber acudido nunca a un m\u00e9dico desde la adolescencia. \u00bfQu\u00e9 actitud o prueba le parece m\u00e1s adecuada para recomendarle en ese momento?", "full_answer": "La respuesta correcta es: 2. Ecograf\u00eda abdomino-p\u00e9lvica. La ecograf\u00eda abdomino-p\u00e9lvica es sin duda la prueba m\u00e1s espec\u00edfica y sensible para el diagn\u00f3stico de poliquistosis renal y, sobre todo, la m\u00e1s barata para demostrar si el hermano del paciente ha heredado o no la poliquistosis.", "type": "NEFROLOG\u00cdA", "options": {"1": "Estudio gen\u00e9tico mutacional y de ligamiento.", "2": "Ecograf\u00eda abdomino-p\u00e9lvica.", "3": "Tomograf\u00eda axial computerizada helicoidal con contraste iodado.", "4": "Resonancia nuclear magn\u00e9tica abdominal y cerebral.", "5": "Controles cl\u00ednicos peri\u00f3dicos."}, "correct_option": 2, "explanations": {"1": {"exist": false, "char_ranges": [], "word_ranges": [], "text": ""}, "2": {"exist": true, "char_ranges": [[57, 283]], "word_ranges": [[7, 44]], "text": "La ecograf\u00eda abdomino-p\u00e9lvica es sin duda la prueba m\u00e1s espec\u00edfica y sensible para el diagn\u00f3stico de poliquistosis renal y, sobre todo, la m\u00e1s barata para demostrar si el hermano del paciente ha heredado o no la poliquistosis."}, "3": {"exist": false, "char_ranges": [], "word_ranges": [], "text": ""}, "4": {"exist": false, "char_ranges": [], "word_ranges": [], "text": ""}, "5": {"exist": false, "char_ranges": [], "word_ranges": [], "text": ""}}} {"id": 207, "year": 2014, "question_id_specific": 181, "full_question": "Mujer de 43 a\u00f1os, remitida a la Consulta de Patolog\u00eda del Tracto Genital Inferior del Hospital por presentar una citolog\u00eda c\u00e9rvico-vaginal informada como \u00abL-SIL\u00bb (Lesi\u00f3n Intraepitelial de Bajo Grado). Refiere infecciones vaginales de repetici\u00f3n (Trichomoniasis,\u2026) y haber tenido m\u00e1s de 5 parejas sexuales en su vida. La mejor estrategia de atenci\u00f3n a esta mujer es:", "full_answer": "La mejor respuesta a esta pregunta es la n\u00famero 5. La mayor\u00eda de laslesiones identificadas como L-SIL se asocian a infecci\u00f3n por VPH: por ello, se aconseja dejar el tabaco, ya que \u00e9ste es un factor de progresi\u00f3n de la dicha infecci\u00f3n. Por otro lado y seg\u00fan los \u00faltimos estudios, el 70% de las citolog\u00edas informadas como LSIL remitir\u00e1n, un 15% persistir\u00e1 y otro 15% mostrar\u00e1 una citolog\u00eda m\u00e1s grave, sin poder diferenciarse si se trata de una progresi\u00f3n biol\u00f3gica o de una lesi\u00f3n oculta en la citolog\u00eda inicial. Debido a ello, lo m\u00e1s adecuado es volver a hacerle un control a los 6 meses. Dado los antecedentes personales de la mujer (m\u00e1s de 5 parejas sexuales e infecciones de repetici\u00f3n) y su edad, lo ideal ser\u00eda remitirla para realizar una colposcopia. No obstante, la respuesta 5 dice que control en 6 meses, pero no especifia qu\u00e9 control (podr\u00eda ser una colposcopia o una citolog\u00eda).", "type": "GINECOLOG\u00cdA Y OBSTETRICIA", "options": {"1": "Conizaci\u00f3n con asa de diatermia.", "2": "Legrado endocervical con legra de Kevorkian.", "3": "Biopsia de endometrio con c\u00e1nula de Cornier.", "4": "Histerectom\u00eda sin anexectom\u00eda.", "5": "Aconsejarle que deje el tabaco y los anticonceptivos y hacerle un control en 6 meses."}, "correct_option": 5, "explanations": {"1": {"exist": false, "char_ranges": [], "word_ranges": [], "text": ""}, "2": {"exist": false, "char_ranges": [], "word_ranges": [], "text": ""}, "3": {"exist": false, "char_ranges": [], "word_ranges": [], "text": ""}, "4": {"exist": false, "char_ranges": [], "word_ranges": [], "text": ""}, "5": {"exist": true, "char_ranges": [[51, 587]], "word_ranges": [[10, 105]], "text": "La mayor\u00eda de laslesiones identificadas como L-SIL se asocian a infecci\u00f3n por VPH: por ello, se aconseja dejar el tabaco, ya que \u00e9ste es un factor de progresi\u00f3n de la dicha infecci\u00f3n. Por otro lado y seg\u00fan los \u00faltimos estudios, el 70% de las citolog\u00edas informadas como LSIL remitir\u00e1n, un 15% persistir\u00e1 y otro 15% mostrar\u00e1 una citolog\u00eda m\u00e1s grave, sin poder diferenciarse si se trata de una progresi\u00f3n biol\u00f3gica o de una lesi\u00f3n oculta en la citolog\u00eda inicial. Debido a ello, lo m\u00e1s adecuado es volver a hacerle un control a los 6 meses."}}} {"id": 584, "year": 2022, "question_id_specific": 73, "full_question": "Mujer de 51 a\u00f1os, menopa\u00fasica desde hace un a\u00f1o y medio, que consulta por manchado vaginal de 2 semanas de evoluci\u00f3n. Refiere que el manchado es menor que una regla. En la ecograf\u00eda se objetiva un endometrio de 7 mm. Se\u00f1ale la opci\u00f3n correcta:", "full_answer": "El endometrio est\u00e1 engrosado para ser una mujer menop\u00e1usica (algunas gu\u00edas ponen el l\u00edmite en 5 y otras en 3mm). Y la primera prueba complementaria a realizar ser\u00eda una biopsia endometrial. Si ya \u00e9sta no es determinante, se valorar\u00e1 realizar histeroscopia como 2\u00aa opci\u00f3n.", "type": "OBSTETRICIA Y GINECOLOG\u00cdA", "options": {"1": "El endometrio est\u00e1 engrosado y ante la cl\u00ednica de la paciente, toma una biopsia endometrial con c\u00e1nula de Cornier.", "2": "El endometrio no est\u00e1 engrosado por lo que le pauta \u00e1cido tranex\u00e1mico y seguimiento ambulatorio.", "3": "Tiene indicaci\u00f3n de histeroscopia diagn\u00f3stica sin necesidad de toma de biopsia endometrial.", "4": "El endometrio no est\u00e1 engrosado, pero como tiene cl\u00ednica de metrorragia est\u00e1 indicada la realizaci\u00f3n de una biopsia endometrial.", "5": NaN}, "correct_option": 1, "explanations": {"1": {"exist": true, "char_ranges": [[0, 189]], "word_ranges": [[0, 31]], "text": "El endometrio est\u00e1 engrosado para ser una mujer menop\u00e1usica (algunas gu\u00edas ponen el l\u00edmite en 5 y otras en 3mm). Y la primera prueba complementaria a realizar ser\u00eda una biopsia endometrial."}, "2": {"exist": false, "char_ranges": [], "word_ranges": [], "text": ""}, "3": {"exist": false, "char_ranges": [], "word_ranges": [], "text": ""}, "4": {"exist": false, "char_ranges": [], "word_ranges": [], "text": ""}, "5": {"exist": false, "char_ranges": [], "word_ranges": [], "text": ""}}} {"id": 138, "year": 2012, "question_id_specific": 165, "full_question": "Ni\u00f1o que acude a consulta de ambulatorio porque se lleva la mano al o\u00eddo, sin fiebre ni signos inflamatorios agudos y que en la exploraci\u00f3n se observa un t\u00edmpano sonrosado/ambarino. Como antecedentes refiere que hace un mes present\u00f3 un episodio de otalgia, fiebre acompa\u00f1ada de irritabilidad y que fue tratado con amoxicilina 40 mg/kg/d\u00eda. \u00bfCu\u00e1l es el diagn\u00f3stico m\u00e1s probable?", "full_answer": "Leyendo el caso, vamos descartando opciones: ni\u00f1o con otalgia, pero sin fiebre ni signos inflamatorios agudos: ya no es ni la 1 (otitis media aguda recidivante) ni la 5 (mastoiditis). En la otoscopia, se observa un t\u00edmpano sonrosado y ambarino: no se observan escamas, luego no es la 4 (otitis media cr\u00f3nica colesteatomatosa); ni tampoco perforaciones ni adhesiones, luego tampoco es la 3 (otitis media cr\u00f3nica simple). La otoscopia es caracter\u00edstica de una otitis media serosa (respuesta 2 correcta). Es conocido por ORL, pediatras y m\u00e9dicos de familia que en el mes siguiente a una otitis media aguda, el o\u00eddo medio queda ocupado por un contenido mucoseroso cuya evoluci\u00f3n conviene vigilar. Tampoco creo que sea impugnable.", "type": "OTORRINOLARINGOLOG\u00cdA Y CIRUG\u00cdA MAXILOFACIAL", "options": {"1": "Otitis media aguda recidivante.", "2": "Otitis media serosa.", "3": "Otitis media cr\u00f3nica simple.", "4": "Otitis media cr\u00f3nica colesteatomatosa.", "5": "Mastoiditis."}, "correct_option": 2, "explanations": {"1": {"exist": true, "char_ranges": [[45, 183]], "word_ranges": [[6, 30]], "text": "ni\u00f1o con otalgia, pero sin fiebre ni signos inflamatorios agudos: ya no es ni la 1 (otitis media aguda recidivante) ni la 5 (mastoiditis)."}, "2": {"exist": true, "char_ranges": [[420, 501]], "word_ranges": [[67, 79]], "text": "La otoscopia es caracter\u00edstica de una otitis media serosa (respuesta 2 correcta)."}, "3": {"exist": true, "char_ranges": [[327, 419]], "word_ranges": [[53, 67]], "text": "ni tampoco perforaciones ni adhesiones, luego tampoco es la 3 (otitis media cr\u00f3nica simple)."}, "4": {"exist": true, "char_ranges": [[184, 326]], "word_ranges": [[30, 53]], "text": "En la otoscopia, se observa un t\u00edmpano sonrosado y ambarino: no se observan escamas, luego no es la 4 (otitis media cr\u00f3nica colesteatomatosa);"}, "5": {"exist": true, "char_ranges": [[45, 183]], "word_ranges": [[6, 30]], "text": "ni\u00f1o con otalgia, pero sin fiebre ni signos inflamatorios agudos: ya no es ni la 1 (otitis media aguda recidivante) ni la 5 (mastoiditis)."}}} {"id": 136, "year": 2012, "question_id_specific": 132, "full_question": "Hombre de 60 a\u00f1os acude a urgencias por crisis comicial. Ex-fumador desde hace 3 a\u00f1os, sin otros antecedentes de inter\u00e9s. Una TAC evidencia met\u00e1stasis m\u00faltiples. \u00bfCu\u00e1l es el origen m\u00e1s probable?", "full_answer": "Muy f\u00e1cil, \u00bfno? El c\u00e1ncer de pulm\u00f3n es la causa m\u00e1s frecuente de las met\u00e1stasis cerebrales y en un 20-30% de los casos es a ra\u00edz de ellas que se diagnostica el tumor primario (como aqu\u00ed).", "type": "ONCOLOG\u00cdA", "options": {"1": "C\u00e1ncer de cabeza y cuello.", "2": "C\u00e1ncer de pulm\u00f3n.", "3": "C\u00e1ncer de vejiga urinaria.", "4": "C\u00e1ncer de colon.", "5": "C\u00e1ncer de p\u00e1ncreas."}, "correct_option": 2, "explanations": {"1": {"exist": false, "char_ranges": [], "word_ranges": [], "text": ""}, "2": {"exist": true, "char_ranges": [[16, 187]], "word_ranges": [[3, 36]], "text": "El c\u00e1ncer de pulm\u00f3n es la causa m\u00e1s frecuente de las met\u00e1stasis cerebrales y en un 20-30% de los casos es a ra\u00edz de ellas que se diagnostica el tumor primario (como aqu\u00ed)."}, "3": {"exist": false, "char_ranges": [], "word_ranges": [], "text": ""}, "4": {"exist": false, "char_ranges": [], "word_ranges": [], "text": ""}, "5": {"exist": false, "char_ranges": [], "word_ranges": [], "text": ""}}} {"id": 518, "year": 2021, "question_id_specific": 105, "full_question": "Una mujer de 24 a\u00f1os con antecedentes de migra\u00f1a, en tratamiento con propranolol y anticonceptivos orales, que acude por disnea intensa, ronquera, erupci\u00f3n cut\u00e1nea, n\u00e1useas y v\u00f3mitos 30 minutos despu\u00e9s de la toma de metamizol. Presenta una presi\u00f3n arterial de 90/40 mmHg y una SatO2 de 90 %. El tratamiento inicial m\u00e1s correcto ser\u00eda administrar:", "full_answer": "Nos encontramos ante una reacci\u00f3n anafil\u00e1ctica por metamizol, en la que hay que actuar r\u00e1pidamente o la paciente podr\u00eda evolucionar a parada card\u00edaca. Es importante evaluar la gravedad de las reacciones al\u00e9rgicas, ya que, en este caso, el tratamiento precoz es fundamental para prevenir la elevada mortalidad si no se aplica. El tratamiento de elecci\u00f3n es la adrenalina IM (s\u00f3lo pasar a IV si anafilaxia refractaria), que ha demostrado el aumento de la supervivencia, pudi\u00e9ndose repetir la administraci\u00f3n cada 5-15 minutos si los s\u00edntomas no ceden. Como tratamiento adyuvante, se podr\u00eda administrar la dexclorfeniramina. En este caso concreto, al tomar la paciente tratamiento previo con propranolol, nos est\u00e1 indicando que la respuesta correcta es la 4, que es la que incluye la administraci\u00f3n de glucag\u00f3n, que se administra porque los pacientes que toman betabloqueantes pueden ser resistentes al tratamiento con adrenalina y desarrollar hipotensi\u00f3n refractaria y bradicardia prolongada. La dosis en adultos es de 1 a 2mg, por v\u00eda IV o IM, pudiendo ser repetida en 5 minutos o seguida de una infusi\u00f3n a 5-15mcg/min.", "type": "CUIDADOS CR\u00cdTICOS Y URGENCIAS", "options": {"1": "Adrenalina.", "2": "Adrenalina y dexclorfeniramina.", "3": "Adrenalina, dexclorfeniramina y metilprednisolona.", "4": "Adrenalina, dexclorfeniramina y glucag\u00f3n.", "5": NaN}, "correct_option": 4, "explanations": {"1": {"exist": false, "char_ranges": [], "word_ranges": [], "text": ""}, "2": {"exist": false, "char_ranges": [], "word_ranges": [], "text": ""}, "3": {"exist": false, "char_ranges": [], "word_ranges": [], "text": ""}, "4": {"exist": true, "char_ranges": [[621, 989]], "word_ranges": [[94, 148]], "text": "En este caso concreto, al tomar la paciente tratamiento previo con propranolol, nos est\u00e1 indicando que la respuesta correcta es la 4, que es la que incluye la administraci\u00f3n de glucag\u00f3n, que se administra porque los pacientes que toman betabloqueantes pueden ser resistentes al tratamiento con adrenalina y desarrollar hipotensi\u00f3n refractaria y bradicardia prolongada."}, "5": {"exist": false, "char_ranges": [], "word_ranges": [], "text": ""}}} {"id": 581, "year": 2022, "question_id_specific": 205, "full_question": "Var\u00f3n de 61 a\u00f1os, fumador e hipertenso, que acude a urgencias por p\u00e9rdida s\u00fabita de fuerza y sensaci\u00f3n de hormigueo en la mano derecha de unos 15 minutos de duraci\u00f3n, con recuperaci\u00f3n posterior casi completa. ECG: ritmo sinusal a 93 lpm. En relaci\u00f3n con el diagn\u00f3stico m\u00e1s probable, se\u00f1ale la respuesta FALSA:", "full_answer": "El caso es de un AIT (accidente isqu\u00e9mico transitorio). El estudio etiol\u00f3gico b\u00e1sico incluye ecodoppler de troncos supra\u00f3rticos +/-arterias cerebrales, ecocardiograma y anal\u00edtica de sangre dirigida.", "type": "NEUROLOG\u00cdA", "options": {"1": "Una causa probable es una embolia arterioarterial por desprendimiento de una placa carot\u00eddea.", "2": "El ecograf\u00eda-doppler de troncos supraa\u00f3rticos es poco \u00fatil para el diagn\u00f3stico.", "3": "Es necesario realizar una TC craneal para evaluar la repercusi\u00f3n sobre el par\u00e9nquima cerebral.", "4": "El tratamiento quir\u00fargico est\u00e1 indicado si las pruebas de imagen revelan una estenosis carot\u00eddea >70%.", "5": NaN}, "correct_option": 2, "explanations": {"1": {"exist": false, "char_ranges": [], "word_ranges": [], "text": ""}, "2": {"exist": false, "char_ranges": [], "word_ranges": [], "text": ""}, "3": {"exist": false, "char_ranges": [], "word_ranges": [], "text": ""}, "4": {"exist": false, "char_ranges": [], "word_ranges": [], "text": ""}, "5": {"exist": false, "char_ranges": [], "word_ranges": [], "text": ""}}} {"id": 543, "year": 2022, "question_id_specific": 39, "full_question": "Paciente de 35 a\u00f1os de edad que consulta para confirmar una sospecha de alergia a Anisakis simplex. Ha sufrido previamente un cuadro cl\u00ednico de anisakiasis gastrointestinal tras ingerir pescado. En las pruebas efectuadas se evidencian niveles de IgE espec\u00edfica de 10 KU/l frente a Anisakis simplex. De las siguientes \u00bfCu\u00e1l ser\u00eda la recomendaci\u00f3n diet\u00e9tica para este paciente?:", "full_answer": "El Anisakis simplex es un par\u00e1sito que se encuentra de forma frecuente en el pescado. Para evitar reacciones con dicho par\u00e1sito realizar\u00e1 una dieta exenta de pescado fresco y crudo (boquerones en vinagre, ahumados, salazones, sushi, etc.). Podr\u00e1 consumir pescado ultracongelado o pescado fresco previamente congelado a -20\u00baC al menos 5 d\u00edas antes de cocinarlo, siempre y cuando lo cocine a altas temperaturas >60\u00baC (asado, frito, etc.), evitando preparaciones crudas, a la plancha o cocciones en microondas. Se recomienda consumir preferentemente las colas porque las larvas del par\u00e1sito suelen estar cerca de las v\u00edsceras de la cabeza.", "type": "ALERGOLOG\u00cdA", "options": {"1": "No puede comer ning\u00fan pescado, crust\u00e1ceo, molusco ni cefal\u00f3podo.", "2": "Puede comer pescado fresco cocinado a la plancha.", "3": "Puede comer pescado ultracongelado comercialmente.", "4": "Puede comer pescado fresco marinado o ahumado en fr\u00edo.", "5": NaN}, "correct_option": 3, "explanations": {"1": {"exist": false, "char_ranges": [], "word_ranges": [], "text": ""}, "2": {"exist": false, "char_ranges": [], "word_ranges": [], "text": ""}, "3": {"exist": false, "char_ranges": [], "word_ranges": [], "text": ""}, "4": {"exist": false, "char_ranges": [], "word_ranges": [], "text": ""}, "5": {"exist": false, "char_ranges": [], "word_ranges": [], "text": ""}}} {"id": 2, "year": 2011, "question_id_specific": 36, "full_question": "Un paciente de 87 a\u00f1os con antecedentes de bronquitis cr\u00f3nica e insuficiencia cardiaca, ha sido diagnosticado de colecistitis aguda liti\u00e1sica. Tras 4 d\u00edas de hospitalizaci\u00f3n en tratamiento con dieta absoluta, sueroterapia piperacilina-tazobactam, el paciente contin\u00faa con fiebre, dolor abdominal persistente y leucocitosis.. La actitud m\u00e1s adecuada en este momento ser\u00eda:", "full_answer": "En esta pregunta, insisten en la edad y a comorbilidad del paciente, as\u00ed como en el tiempo de evoluci\u00f3n, por lo que la respuesta probablemente no ser\u00e1 la 1. De todas formas lo ideal es drenar esa ves\u00edcula que no acaba de remontar por lo que la respuesta que yo pondr\u00eda ser\u00eda la 2.", "type": "DIGESTIVO", "options": {"1": "Tratamiento quir\u00fargico urgente.", "2": "Drenaje biliar mediante colecistostom\u00eda percutanea.", "3": "Sustituir la piperacilina-tazobactam por metronidazol-cefotaxima.", "4": "Sustituir la piperacilina-tazobactam por amikacina-clindamicina.", "5": "A\u00f1adir al tratamiento un aminogluc\u00f3sido como gentamicina."}, "correct_option": 2, "explanations": {"1": {"exist": true, "char_ranges": [[0, 156]], "word_ranges": [[0, 29]], "text": "En esta pregunta, insisten en la edad y a comorbilidad del paciente, as\u00ed como en el tiempo de evoluci\u00f3n, por lo que la respuesta probablemente no ser\u00e1 la 1."}, "2": {"exist": true, "char_ranges": [[158, 281]], "word_ranges": [[29, 54]], "text": "De todas formas lo ideal es drenar esa ves\u00edcula que no acaba de remontar por lo que la respuesta que yo pondr\u00eda ser\u00eda la 2."}, "3": {"exist": false, "char_ranges": [], "word_ranges": [], "text": ""}, "4": {"exist": false, "char_ranges": [], "word_ranges": [], "text": ""}, "5": {"exist": false, "char_ranges": [], "word_ranges": [], "text": ""}}} {"id": 122, "year": 2012, "question_id_specific": 119, "full_question": "Mujer de 17 a\u00f1os de edad que acude a urgencias por un cuadro agudo de fiebre elevada, dolor far\u00edngeo y adenopat\u00edas cervicales. Previamente hab\u00eda sido diagn\u00f3sticada de faringitis aguda y recibi\u00f3 tratamiento con amoxicilina, presentando posteriormente un exantema cut\u00e1neo maculoso generalizado. Se realiza anal\u00edtica que presenta ligera leucocitosis y presencia de leucocitos activados, ligera trombopenia y transaminasas levemente aumentadas. \u00bfCu\u00e1l ser\u00eda el diagn\u00f3stico m\u00e1s probable de este cuadro cl\u00ednico?", "full_answer": "La respuesta correcta es la primera. Es un cuadro t\u00edpico de mononucleosis infecciosa. Aunque puede darse un cuadro similar en una toxoplasmosis aguda, menos del 1% de las infecciones agudas por toxoplasma se presentan como mononucleosis. Caracter\u00edstico de la mononucleosis infecciosa por Epstein-Barr es la aparici\u00f3n del exantema macular tras el tratamiento por amoxicilina.", "type": "MICROBIOLOG\u00cdA", "options": {"1": "Es un cuadro t\u00edpico de mononucleosis infecciosa.", "2": "Infecci\u00f3n por varicela zoster.", "3": "Toxoplasmosis aguda.", "4": "Enfermedad de Lyme.", "5": "Infecci\u00f3n por herpes virus 8."}, "correct_option": 1, "explanations": {"1": {"exist": true, "char_ranges": [[238, 374]], "word_ranges": [[36, 54]], "text": "Caracter\u00edstico de la mononucleosis infecciosa por Epstein-Barr es la aparici\u00f3n del exantema macular tras el tratamiento por amoxicilina."}, "2": {"exist": false, "char_ranges": [], "word_ranges": [], "text": ""}, "3": {"exist": true, "char_ranges": [[86, 237]], "word_ranges": [[13, 36]], "text": "Aunque puede darse un cuadro similar en una toxoplasmosis aguda, menos del 1% de las infecciones agudas por toxoplasma se presentan como mononucleosis."}, "4": {"exist": false, "char_ranges": [], "word_ranges": [], "text": ""}, "5": {"exist": false, "char_ranges": [], "word_ranges": [], "text": ""}}} {"id": 36, "year": 2011, "question_id_specific": 130, "full_question": "Una mujer de 85 a\u00f1os consulta por cansancio y debilidad especialmente por las ma\u00f1anas. A veces se encuentra inestable al caminar y tiene que sentarse para recuperar el equilibrio. En dos ocasiones ha tenido que sentarse para no caer pero niega s\u00edntomas de mareo. Tiene hipertensi\u00f3n arterial, incontinencia urinaria y artrosis. Su tratamiento es hidroclorotiazida (25 mg/d), oxibutinina (10 mg/d), lisinopril (10 mg/d), calcio (1500 mg/d) y paracetamol (3 gr/d). A la exploraci\u00f3n destaca una tensi\u00f3n arterial de 115/70 mm Hg, pulso 80 lpm. Sus movimientos son lentos. Tiene un temblor moderado las manos. Puede levantarse de la silla lentamente pero sin necesidad de apoyarse en los brazos. Camina levemente inclinada hacia adelante con poco balanceo de los brazos. Gira lentamente pero sin perder el equilibrio. No es capaz de mantenerse sobre un solo pie. \u00bfCu\u00e1l de las siguientes posibles actuaciones realizar\u00eda en primer lugar?", "full_answer": "La respuesta correcta es iniciar tratamiento con L-dopa, ya que la cl\u00ednica descrita es de un parkinsonismo cuyo diagn\u00f3stico es exclusivamente cl\u00ednico. El resto de respuestas pueden realizarse para diagn\u00f3stico diferencial, pero la realizaci\u00f3n de pruebas complementarias no justifica el retraso del inicio de tratamiento.", "type": "NEUROLOG\u00cdA Y NEUROCIRUG\u00cdA", "options": {"1": "Valorar agudeza visual.", "2": "Realizar una resonancia magn\u00e9tica.", "3": "Estudio con mesa basculante.", "4": "Medir la tensi\u00f3n arterial tumbada y levantada.", "5": "Intento terap\u00e9utico con L-dopa."}, "correct_option": 5, "explanations": {"1": {"exist": true, "char_ranges": [[151, 319]], "word_ranges": [[22, 45]], "text": "El resto de respuestas pueden realizarse para diagn\u00f3stico diferencial, pero la realizaci\u00f3n de pruebas complementarias no justifica el retraso del inicio de tratamiento."}, "2": {"exist": true, "char_ranges": [[151, 319]], "word_ranges": [[22, 45]], "text": "El resto de respuestas pueden realizarse para diagn\u00f3stico diferencial, pero la realizaci\u00f3n de pruebas complementarias no justifica el retraso del inicio de tratamiento."}, "3": {"exist": true, "char_ranges": [[151, 319]], "word_ranges": [[22, 45]], "text": "El resto de respuestas pueden realizarse para diagn\u00f3stico diferencial, pero la realizaci\u00f3n de pruebas complementarias no justifica el retraso del inicio de tratamiento."}, "4": {"exist": true, "char_ranges": [[151, 319]], "word_ranges": [[22, 45]], "text": "El resto de respuestas pueden realizarse para diagn\u00f3stico diferencial, pero la realizaci\u00f3n de pruebas complementarias no justifica el retraso del inicio de tratamiento."}, "5": {"exist": true, "char_ranges": [[0, 150]], "word_ranges": [[0, 22]], "text": "La respuesta correcta es iniciar tratamiento con L-dopa, ya que la cl\u00ednica descrita es de un parkinsonismo cuyo diagn\u00f3stico es exclusivamente cl\u00ednico."}}} {"id": 338, "year": 2016, "question_id_specific": 31, "full_question": "Mujer de 20 a\u00f1os con tumoraci\u00f3n ov\u00e1rica de 15 cm, s\u00f3lido-qu\u00edstica, detectada por ecograf\u00eda tras presentar s\u00edntomas abdominales inespect\u00edficos. En el estudio histopatol\u00f3gico de la pieza correspondiente se encuentran dientes, pelos, zonas de epitelio intestinal, \u00e1reas de epitelio escamoso (15%) y bronquial, as\u00ed como elementos neuroectod\u00e9rmicos y embrionarios en varias de las preparaciones histol\u00f3gicas. En referencia a este caso, se\u00f1ale el diagn\u00f3stico correcto:", "full_answer": "Teratoma qu\u00edstico maduro (frecuente en el ovario de la mujer adulta). Masa con un gran quiste que ocupa la mayor parte de ella; en su cavidad hay material seb\u00e1ceo y pelos. La pared, en general de pocos mil\u00edmetros de espesor, tiene la estructura de piel, con su superficie epid\u00e9rmica hacia la cavidad. Por este desarrollo preponderante de estructuras cut\u00e1neas se le suele denominar quiste dermoide. Los tejidos que componen el tumor son bien diferenciados (maduros o de tipo adulto); aparte estructuras cut\u00e1neas se pueden observar muchos otros tejidos,particularmente en un engrosamiento o espol\u00f3n que hace eminencia hacia la cavidad, en el que con frecuencia hay dientes, cart\u00edlago y hueso.", "type": "GINECOLOG\u00cdA Y OBSTETRICIA", "options": {"1": "Teratocarcinoma.", "2": "Teratoma inmaduro.", "3": "Teratoma qu\u00edstico maduro.", "4": "Disgerminoma.", "5": NaN}, "correct_option": 3, "explanations": {"1": {"exist": false, "char_ranges": [], "word_ranges": [], "text": ""}, "2": {"exist": false, "char_ranges": [], "word_ranges": [], "text": ""}, "3": {"exist": true, "char_ranges": [[398, 690]], "word_ranges": [[65, 109]], "text": "Los tejidos que componen el tumor son bien diferenciados (maduros o de tipo adulto); aparte estructuras cut\u00e1neas se pueden observar muchos otros tejidos,particularmente en un engrosamiento o espol\u00f3n que hace eminencia hacia la cavidad, en el que con frecuencia hay dientes, cart\u00edlago y hueso."}, "4": {"exist": false, "char_ranges": [], "word_ranges": [], "text": ""}, "5": {"exist": false, "char_ranges": [], "word_ranges": [], "text": ""}}} {"id": 432, "year": 2018, "question_id_specific": 117, "full_question": "Hombre de 85 a\u00f1os portador de cat\u00e9ter venosa perif\u00e9rico que, una semana despu\u00e9s de estar hospitalizado por un ictus, comienza con tiritona y fiebre. Se realizan hemocultivos y desde microbiolog\u00eda se informa que est\u00e1n creciendo cocos gram positivos en racimos. En espera del antibiograma, \u00bfcual es el tratamiento antibi\u00f3tico m\u00e1s adecuado?", "full_answer": "La causa m\u00e1s frecuente de infecci\u00f3n por cat\u00e9ter es una bacteria que coloniza la piel. La tinci\u00f3n de Gram nos dice que es un estafilococo, la mayor\u00eda de las especies de este g\u00e9nero adquirido de forma intrahospitalaria son meticilin resistente, por lo que se consideran resistente a Cloxacilina, Cefazolina y no hay datos cl\u00ednicos acerca de la eficacia de linezolid para el tratamiento de la infecci\u00f3n sist\u00e9mica relacionada con cat\u00e9ter. Esto, m\u00e1s la experiencia con el uso de vancomicina en este cuadro se considera de elecci\u00f3n emp\u00edricamente a vancomicina.", "type": "ENFERMEDADES INFECCIOSAS Y MICROBIOLOG\u00cdA", "options": {"1": "Cefazol\u00edna.", "2": "Cloxacilina.", "3": "Vancomic\u00edna.", "4": "Linezolid.", "5": NaN}, "correct_option": 3, "explanations": {"1": {"exist": true, "char_ranges": [[86, 305]], "word_ranges": [[15, 49]], "text": "La tinci\u00f3n de Gram nos dice que es un estafilococo, la mayor\u00eda de las especies de este g\u00e9nero adquirido de forma intrahospitalaria son meticilin resistente, por lo que se consideran resistente a Cloxacilina, Cefazolina"}, "2": {"exist": true, "char_ranges": [[86, 305]], "word_ranges": [[15, 49]], "text": "La tinci\u00f3n de Gram nos dice que es un estafilococo, la mayor\u00eda de las especies de este g\u00e9nero adquirido de forma intrahospitalaria son meticilin resistente, por lo que se consideran resistente a Cloxacilina, Cefazolina"}, "3": {"exist": true, "char_ranges": [[447, 555]], "word_ranges": [[72, 89]], "text": "la experiencia con el uso de vancomicina en este cuadro se considera de elecci\u00f3n emp\u00edricamente a vancomicina."}, "4": {"exist": true, "char_ranges": [[308, 435]], "word_ranges": [[50, 70]], "text": "no hay datos cl\u00ednicos acerca de la eficacia de linezolid para el tratamiento de la infecci\u00f3n sist\u00e9mica relacionada con cat\u00e9ter."}, "5": {"exist": false, "char_ranges": [], "word_ranges": [], "text": ""}}} {"id": 513, "year": 2021, "question_id_specific": 38, "full_question": "Paciente de 72 a\u00f1os de edad que acude a urgencias por presentar aparatoso e importante angioedema lingual. No presenta urticaria asociada ni refiere antecedentes de alergia a f\u00e1rmacos o alimentos. Entre sus antecedentes personales destacan diabetes mellitus tipo 2, dislipemia, hipertensi\u00f3n arterial, hipotiroidismo y enfermedad de Parkinson. En tratamiento habitual con metformina, simvastatina, enalapril, tiroxina y levodopa. \u00bfCu\u00e1l de los siguientes f\u00e1rmacos es con mayor probabilidad el f\u00e1rmaco causal del cuadro cl\u00ednico descrito?:", "full_answer": "De los f\u00e1rmacos propuestos, las opciones menos probables ser\u00edan la 3, ya que la metformina muy raramente causa angioedema lingual (en marzo de 2020 s\u00f3lo hab\u00eda un caso descrito en todo el mundo) y la 4, ya que, entre las reacciones adversas de la levodopa, el angioedema se clasifica entre las muy raras (frecuencia menor al 0,1%). De las dos opciones que quedan, los f\u00e1rmacos que con m\u00e1s frecuencia y m\u00e1s cl\u00e1sicamente se han asociado a angioedema lingual son los IECAs (frecuencia en torno al 0.2%), siendo su relaci\u00f3n con la simvastatina tambi\u00e9n menor al 0.1%.", "type": "CUIDADOS CR\u00cdTICOS Y URGENCIAS", "options": {"1": "Enalapril.", "2": "Simvastatina.", "3": "Metformina o levodopa por igual.", "4": "Levodopa.", "5": NaN}, "correct_option": 1, "explanations": {"1": {"exist": true, "char_ranges": [[363, 498]], "word_ranges": [[63, 86]], "text": "los f\u00e1rmacos que con m\u00e1s frecuencia y m\u00e1s cl\u00e1sicamente se han asociado a angioedema lingual son los IECAs (frecuencia en torno al 0.2%),"}, "2": {"exist": true, "char_ranges": [[500, 561]], "word_ranges": [[86, 96]], "text": "siendo su relaci\u00f3n con la simvastatina tambi\u00e9n menor al 0.1%."}, "3": {"exist": true, "char_ranges": [[77, 193]], "word_ranges": [[13, 33]], "text": "la metformina muy raramente causa angioedema lingual (en marzo de 2020 s\u00f3lo hab\u00eda un caso descrito en todo el mundo)"}, "4": {"exist": true, "char_ranges": [[210, 330]], "word_ranges": [[38, 57]], "text": "entre las reacciones adversas de la levodopa, el angioedema se clasifica entre las muy raras (frecuencia menor al 0,1%)."}, "5": {"exist": false, "char_ranges": [], "word_ranges": [], "text": ""}}} {"id": 580, "year": 2022, "question_id_specific": 198, "full_question": "Mujer de 35 a\u00f1os, remitida a urgencias por traumatismo craneoencef\u00e1lico por ca\u00edda en patinete, sin p\u00e9rdida de conciencia ni amnesia. En la exploraci\u00f3n se trata de un traumatismo craneoencef\u00e1lico cerrado con conmoci\u00f3n y cefalea leve sin presencia de n\u00e1useas y v\u00f3mitos. La exploraci\u00f3n neurol\u00f3gica es normal y presenta un \u00edndice de Glasgow de 15. Ante esta situaci\u00f3n \u00bfqu\u00e9 prueba de imagen est\u00e1 indicada en primer lugar?:", "full_answer": "Tras un TCE en un paciente no anticoagulado ni antiagregado, sin factores de riesgo conocidos para hemorragia, que no presenta datos de alarma ni en anamnesis ni en exploraci\u00f3n f\u00edsica no es necesario realizar prueba de imagen craneal.", "type": "NEUROLOG\u00cdA", "options": {"1": "TC craneal sin contraste.", "2": "TC craneal con contraste.", "3": "Ninguna prueba de imagen.", "4": "Radiograf\u00eda simple de cr\u00e1neo.", "5": NaN}, "correct_option": 3, "explanations": {"1": {"exist": true, "char_ranges": [[0, 234]], "word_ranges": [[0, 38]], "text": "Tras un TCE en un paciente no anticoagulado ni antiagregado, sin factores de riesgo conocidos para hemorragia, que no presenta datos de alarma ni en anamnesis ni en exploraci\u00f3n f\u00edsica no es necesario realizar prueba de imagen craneal."}, "2": {"exist": true, "char_ranges": [[0, 234]], "word_ranges": [[0, 38]], "text": "Tras un TCE en un paciente no anticoagulado ni antiagregado, sin factores de riesgo conocidos para hemorragia, que no presenta datos de alarma ni en anamnesis ni en exploraci\u00f3n f\u00edsica no es necesario realizar prueba de imagen craneal."}, "3": {"exist": true, "char_ranges": [[0, 234]], "word_ranges": [[0, 38]], "text": "Tras un TCE en un paciente no anticoagulado ni antiagregado, sin factores de riesgo conocidos para hemorragia, que no presenta datos de alarma ni en anamnesis ni en exploraci\u00f3n f\u00edsica no es necesario realizar prueba de imagen craneal."}, "4": {"exist": true, "char_ranges": [[0, 234]], "word_ranges": [[0, 38]], "text": "Tras un TCE en un paciente no anticoagulado ni antiagregado, sin factores de riesgo conocidos para hemorragia, que no presenta datos de alarma ni en anamnesis ni en exploraci\u00f3n f\u00edsica no es necesario realizar prueba de imagen craneal."}, "5": {"exist": false, "char_ranges": [], "word_ranges": [], "text": ""}}} {"id": 248, "year": 2014, "question_id_specific": 118, "full_question": "Hombre de 34 a\u00f1os que consulta por fiebre y malestar general. En la historia cl\u00ednica se recogen como antecedentes relaciones homosexuales desde hace 4 meses con una nueva pareja. 2 meses antes de la consulta actual tuvo una lesi\u00f3n ulcerosa en el glande, indolora, con adenopat\u00edas inguinales bilaterales, todo ello autolimitado. Se solicitan estudios serol\u00f3gicos con los siguientes resultados: HIV negativo, RPR 1/320, TPHA 1/128. \u00bfQu\u00e9 tratamiento indicar\u00eda en este paciente?", "full_answer": "Pregunta de respuesta directa. Se trata de una s\u00edfilis (tanto por el antecendente como por la serolog\u00eda) en fase secundaria por lo que el tratamiento se realiza con Penicilina Benzatina 2,4 MU en una \u00fanica dosis.", "type": "ENFERMEDADES INFECCIOSAS", "options": {"1": "Ninguno.", "2": "Penicilina G intravenosa, 24 MU cada d\u00eda durante 14 d\u00edas.", "3": "Penicilina Benzatina 2,4 MU intramuscular, 3 dosis en tres semanas consecutivas.", "4": "Ceftriaxona 2 gramos intramusculares en una \u00fanica dosis.", "5": "Penicilina Benzatina 2,4 MU intramuscular en una \u00fanica dosis."}, "correct_option": 5, "explanations": {"1": {"exist": false, "char_ranges": [], "word_ranges": [], "text": ""}, "2": {"exist": false, "char_ranges": [], "word_ranges": [], "text": ""}, "3": {"exist": false, "char_ranges": [], "word_ranges": [], "text": ""}, "4": {"exist": false, "char_ranges": [], "word_ranges": [], "text": ""}, "5": {"exist": true, "char_ranges": [[31, 212]], "word_ranges": [[4, 36]], "text": "Se trata de una s\u00edfilis (tanto por el antecendente como por la serolog\u00eda) en fase secundaria por lo que el tratamiento se realiza con Penicilina Benzatina 2,4 MU en una \u00fanica dosis."}}} {"id": 222, "year": 2014, "question_id_specific": 197, "full_question": "Var\u00f3n homosexual de 30 a\u00f1os de edad VIH (+) que participa como voluntario en un centro de ayuda a pacientes con SIDA. Seg\u00fan su historia cl\u00ednica recibi\u00f3 toxoide dift\u00e9rico (Td) hace 6 a\u00f1os, la vacuna triple v\u00edrica en la infancia y en la adolescencia, y la hepatitis B hace 3 a\u00f1os. Actualmente se encuentra asintom\u00e1tico con un recuento de CD4 superior a 200 cls/microlitro. \u00bfQu\u00e9 vacunas deber\u00edamos recomendarle?", "full_answer": "las recomendaciones de vacunaci\u00f3n para los adultos HIV+ incluyen vacuna de la hepatitis B, gripe, triple v\u00edrica, antineumoc\u00f3cica, Td y Tdap, y para algunos adultos, hepatits A, o A y B combinada, meningitis bacteriana, VPH y antimeningoc\u00f3cica (Fuente [1]). Teniendo en cuenta que el individuo ha recibido algunas vacunas, que tiene un recuento CD4 >100 (si fuera menor habr\u00eda alguna contraindicaci\u00f3n) y trabaja en un centro de ayuda a pacientes con SIDA (se le equipara a \u201cpersonal de salud\u201d) las respuestas 2, 4 y 5 son err\u00f3neas. Las cuatro vacunas de la opci\u00f3n 1 son administrables. Con algo m\u00e1s de detalle: Vacuna frente al t\u00e9tanos y la difteria: todas las personas deben estar vacunados frente a estas dos enfermedades. Vacuna frente a la hepatitis B: los virus de la hepatitis B y del VIH comparten las mismas v\u00edas de transmisi\u00f3n por lo que es frecuente que el paciente infectado por VIH est\u00e9 infectado tambi\u00e9n por el virus de la hepatitis B. Por esto es importante conocer si la persona infectada por VIH presenta marcadores de infecci\u00f3n por hepatitis B y proceder a la vacunaci\u00f3n en caso de que no los tenga. Vacuna frente a la hepatitis A: la hepatitis A en una persona portadora de hepatitis B, con hepatitis C o con otra enfermedad hep\u00e1tica puede ser muy grave. Dado que estas infecciones son m\u00e1s frecuentes en el paciente infectado por VIH se recomienda su vacunaci\u00f3n frente a la hepatitis A. Vacuna frente a la gripe: una gripe en un paciente con falta de respuesta inmune presenta mayor gravedad y riesgo de complicaciones, por lo que est\u00e1 indicada la vacunaci\u00f3n anual frente a esta enfermedad. Si la situaci\u00f3n inmunitaria del paciente est\u00e1 muy deteriorada, la respuesta a la vacunaci\u00f3n est\u00e1 disminuida. En este caso es fundamental la vacunaci\u00f3n de todos los convivientes de la persona infectada por VIH para evitar que le transmitan la enfermedad. Vacuna frente al neumococo: las infecciones neumoc\u00f3cicas (en especial las neumon\u00edas) son hasta 10 veces mas frecuentes en las personas infectadas por VIH que en los adultos no infectados por el VIH, por lo que se recomienda la vacunaci\u00f3n, si bien en pacientes con recuentos inferiores a 200/mm3 la respuesta protectora no suele ser suficiente. Deben recibir una segunda dosis a los 3-5 a\u00f1os. Los ni\u00f1os infectados por VIH deben vacunarse con la vacuna antineumoc\u00f3cica conjugada con el n\u00famero de dosis que le correspondan seg\u00fan la edad. Vacuna frente al Haemophilus Influenzae tipo b: esta bacteria es causa de neumon\u00edas y meningitis, en especial en ni\u00f1os. Aunque en los adultos infectados por VIH representa un n\u00famero peque\u00f1o de las causas de infecci\u00f3n, se recomienda la vacunaci\u00f3n. Vacuna frente al sarampi\u00f3n, la rub\u00e9ola y la parotiditis (triple v\u00edrica): las tres enfermedades pueden adquirir una mayor gravedad en los pacientes infectados por VIH, en especial el sarampi\u00f3n, por lo que todas ellas deben de estar vacunadas, siempre que la inmunodepresi\u00f3n no sea severa (recuento menor a 200/mm3).", "type": "EPIDEMIOLOG\u00cdA", "options": {"1": "Gripe estacional, neumoc\u00f3cica, meningitis tetravalente y hepatitis A.", "2": "Gripe estacional, Td, neumoc\u00f3cia y meningitis tetravalente.", "3": "Meningitis tetravalente, neumoc\u00f3cia y gripe estacional.", "4": "Td, meningitis tetravalente, neumoc\u00f3cica.", "5": "Triple v\u00edrica, gripe estacional, neumoc\u00f3cica."}, "correct_option": 1, "explanations": {"1": {"exist": true, "char_ranges": [[532, 2206]], "word_ranges": [[87, 370]], "text": "Las cuatro vacunas de la opci\u00f3n 1 son administrables. Con algo m\u00e1s de detalle: Vacuna frente al t\u00e9tanos y la difteria: todas las personas deben estar vacunados frente a estas dos enfermedades. Vacuna frente a la hepatitis B: los virus de la hepatitis B y del VIH comparten las mismas v\u00edas de transmisi\u00f3n por lo que es frecuente que el paciente infectado por VIH est\u00e9 infectado tambi\u00e9n por el virus de la hepatitis B. Por esto es importante conocer si la persona infectada por VIH presenta marcadores de infecci\u00f3n por hepatitis B y proceder a la vacunaci\u00f3n en caso de que no los tenga. Vacuna frente a la hepatitis A: la hepatitis A en una persona portadora de hepatitis B, con hepatitis C o con otra enfermedad hep\u00e1tica puede ser muy grave. Dado que estas infecciones son m\u00e1s frecuentes en el paciente infectado por VIH se recomienda su vacunaci\u00f3n frente a la hepatitis A. Vacuna frente a la gripe: una gripe en un paciente con falta de respuesta inmune presenta mayor gravedad y riesgo de complicaciones, por lo que est\u00e1 indicada la vacunaci\u00f3n anual frente a esta enfermedad. Si la situaci\u00f3n inmunitaria del paciente est\u00e1 muy deteriorada, la respuesta a la vacunaci\u00f3n est\u00e1 disminuida. En este caso es fundamental la vacunaci\u00f3n de todos los convivientes de la persona infectada por VIH para evitar que le transmitan la enfermedad. Vacuna frente al neumococo: las infecciones neumoc\u00f3cicas (en especial las neumon\u00edas) son hasta 10 veces mas frecuentes en las personas infectadas por VIH que en los adultos no infectados por el VIH, por lo que se recomienda la vacunaci\u00f3n, si bien en pacientes con recuentos inferiores a 200/mm3 la respuesta protectora no suele ser suficiente."}, "2": {"exist": true, "char_ranges": [[257, 531]], "word_ranges": [[39, 87]], "text": "Teniendo en cuenta que el individuo ha recibido algunas vacunas, que tiene un recuento CD4 >100 (si fuera menor habr\u00eda alguna contraindicaci\u00f3n) y trabaja en un centro de ayuda a pacientes con SIDA (se le equipara a \u201cpersonal de salud\u201d) las respuestas 2, 4 y 5 son err\u00f3neas."}, "3": {"exist": false, "char_ranges": [], "word_ranges": [], "text": ""}, "4": {"exist": true, "char_ranges": [[257, 531]], "word_ranges": [[39, 87]], "text": "Teniendo en cuenta que el individuo ha recibido algunas vacunas, que tiene un recuento CD4 >100 (si fuera menor habr\u00eda alguna contraindicaci\u00f3n) y trabaja en un centro de ayuda a pacientes con SIDA (se le equipara a \u201cpersonal de salud\u201d) las respuestas 2, 4 y 5 son err\u00f3neas."}, "5": {"exist": true, "char_ranges": [[257, 531]], "word_ranges": [[39, 87]], "text": "Teniendo en cuenta que el individuo ha recibido algunas vacunas, que tiene un recuento CD4 >100 (si fuera menor habr\u00eda alguna contraindicaci\u00f3n) y trabaja en un centro de ayuda a pacientes con SIDA (se le equipara a \u201cpersonal de salud\u201d) las respuestas 2, 4 y 5 son err\u00f3neas."}}} {"id": 244, "year": 2014, "question_id_specific": 114, "full_question": "Chico de 16 a\u00f1os que consulta por presentar amigdalitis pult\u00e1cea, fiebre de hasta 38,5\u00baC, adenopat\u00edas cervicales dolorosas, exantema macular no pruriginoso en t\u00f3rax y hepatoesplenomegalia leves, de 4-5 d\u00edas de evoluci\u00f3n. El test de Paul-Bunnell y la Ig M para el virus de Epstein-Barr son positivos. Durante su ingreso desarrolla fiebre continua de hasta 40\u00baC, pancitopenia, hepatitis ict\u00e9rica y coagulopat\u00eda de intensidad progresiva. A la semana del ingreso, se traslada a UCI por confusi\u00f3n e insuficiencia respiratoria. Los hemocultivos y un urocultivo son negativos, el LCR es normal y la placa de t\u00f3rax no muestra infiltrados. La procalcitonina es normal, pero PCR y ferritina est\u00e1n muy elevados. De los enunciados a continuaci\u00f3n, \u00bfcu\u00e1l ser\u00eda el planteamiento diagn\u00f3stico y terap\u00e9utico m\u00e1s correcto?", "full_answer": "Pregunta dif\u00edcil en la que la respuesta correcta podr\u00eda dar lugar a discusi\u00f3n. Se trata de un var\u00f3n de 16 a\u00f1os que se ingresa por un cuadro de mononucleosis por infecci\u00f3n por VEB bastante claro que se complica durante el ingreso. Los datos de los que disponemos son: - Pancitopenia. - Hepatopat\u00eda. - Coagulopatia. - Ausencia de foco infeccioso: hemocultivos negativos, urocultivos negativos, Rx normal, LCR normal. - PCR y Ferritina elevadas/ Procalcitonina normal. Las posibles complicaciones que ha podido producirse (y que se barajan en las respuestas) son una sepsis o bien una mala evoluci\u00f3n de la mononucleosis. Los datos cl\u00ednicos podr\u00edan cuadrar con un proceso s\u00e9ptico, sin embargo se han descartado de manera razonable los principales focos infecciosos y uno de los marcadores m\u00e1s sensibles y espec\u00edficos de bacteriemia (la procalcitonina) se encuentra en rango de normalidad, esto debe llamarnos la atenci\u00f3n. Por otra parte, cabe valorar una complicaci\u00f3n de la mononucleosis por VEB que sufre nuestro paciente, entre las complicaciones posibles est\u00e1n la anemia hemol\u00edtica, la meningoecefalitis (LCR normal) o el sd. De Guillain-Barr\u00e9, que no concuerdan mucho con el cuadro cl\u00ednico que nos presentan. Otra complicaci\u00f3n posible y de excepcional gravedad ser\u00eda el s\u00edndrome hemofagoc\u00edtico; se trata de una entidad rara que se produce en determinados sujetos predispuestos y se caracteriza por una activaci\u00f3n y proliferaci\u00f3n incontrolada de histiocitos y linfocitos T, que produce un estado de hipercitocinemia. Los criterios diagn\u00f3sticos revisados en 2004, incluyen fiebre elevada persistente, hepatoesplenomegalia, citopenias, hipertrigliceridemia, hiperferritinemia e hipofibrinogenemia. La hiperferritinemia >3000 en un contexto cl\u00ednico sugestivo hace necesario el tratamiento del paciente (principalmente con inmunosupresores y corticoides) puesto que la mortalidad es muy elevada y el tratamiento precoz es esencial. El diagn\u00f3stico definitivo se hace mediante el estudio microsc\u00f3pico de una biopsia de MO. Aunque en esta pregunta no nos cuantifican el valor de la ferritinemia, es un dato destacable y dado que la cl\u00ednica es sugestiva, llegando a cumplirse los criterios diagn\u00f3sticos, me parece que la respuesta 5 ser\u00eda la mas acertada.", "type": "ENFERMEDADES INFECCIOSAS", "options": {"1": "Tiene una sepsis bacteriana de origen indeterminado y se debe administrar ceftriaxona y tratamiento de soporte.", "2": "Tiene sepsis bacteriana de origen indeterminado y se debe administrar vancomicina, ceftacidima y tratamiento de soporte.", "3": "Se trata de una mononucleosis infecciosa de curso grave y se debe administrar glucocorticoides.", "4": "Se trata de una mononucleosis infecciosa de curso grave y se debe iniciar tratamiento con Aciclovir.", "5": "Realizar\u00eda una biopsia/aspirado de m\u00e9dula \u00f3sea y, si se confirma hemofagocitosis, iniciar\u00eda tratamiento con inmunosupresores."}, "correct_option": 5, "explanations": {"1": {"exist": true, "char_ranges": [[618, 917]], "word_ranges": [[99, 144]], "text": "Los datos cl\u00ednicos podr\u00edan cuadrar con un proceso s\u00e9ptico, sin embargo se han descartado de manera razonable los principales focos infecciosos y uno de los marcadores m\u00e1s sensibles y espec\u00edficos de bacteriemia (la procalcitonina) se encuentra en rango de normalidad, esto debe llamarnos la atenci\u00f3n."}, "2": {"exist": true, "char_ranges": [[618, 917]], "word_ranges": [[99, 144]], "text": "Los datos cl\u00ednicos podr\u00edan cuadrar con un proceso s\u00e9ptico, sin embargo se han descartado de manera razonable los principales focos infecciosos y uno de los marcadores m\u00e1s sensibles y espec\u00edficos de bacteriemia (la procalcitonina) se encuentra en rango de normalidad, esto debe llamarnos la atenci\u00f3n."}, "3": {"exist": true, "char_ranges": [[918, 1208]], "word_ranges": [[144, 188]], "text": "Por otra parte, cabe valorar una complicaci\u00f3n de la mononucleosis por VEB que sufre nuestro paciente, entre las complicaciones posibles est\u00e1n la anemia hemol\u00edtica, la meningoecefalitis (LCR normal) o el sd. De Guillain-Barr\u00e9, que no concuerdan mucho con el cuadro cl\u00ednico que nos presentan."}, "4": {"exist": true, "char_ranges": [[918, 1208]], "word_ranges": [[144, 188]], "text": "Por otra parte, cabe valorar una complicaci\u00f3n de la mononucleosis por VEB que sufre nuestro paciente, entre las complicaciones posibles est\u00e1n la anemia hemol\u00edtica, la meningoecefalitis (LCR normal) o el sd. De Guillain-Barr\u00e9, que no concuerdan mucho con el cuadro cl\u00ednico que nos presentan."}, "5": {"exist": true, "char_ranges": [[1696, 2017]], "word_ranges": [[248, 294]], "text": "La hiperferritinemia >3000 en un contexto cl\u00ednico sugestivo hace necesario el tratamiento del paciente (principalmente con inmunosupresores y corticoides) puesto que la mortalidad es muy elevada y el tratamiento precoz es esencial. El diagn\u00f3stico definitivo se hace mediante el estudio microsc\u00f3pico de una biopsia de MO."}}} {"id": 420, "year": 2018, "question_id_specific": 78, "full_question": "Hombre de 52 a\u00f1os de edad derivado al servicio de digestivo por cuadro de hematoquezia, tenesmo y reducci\u00f3n del di\u00e1metro de las heces. Se realizan una serie de pruebas, diagnosticandose un adenocarcinoma de sigma sin metastasis a distancia. El paciente es intervenido quir\u00fargicamente y remitido a la consulta de oncologia m\u00e9dica para valorar tratamiento quimioterapico complementario. \u00bfCual de los siguientes factores es de mal pron\u00f3stico tras la resecci\u00f3n quir\u00fargica y habra que tener en cuenta a la hora de planificar el tratamiento de quimioterapia?", "full_answer": "No est\u00e1n preguntando por factores que afectan al pron\u00f3stico oncol\u00f3gico y que por tanto implican un cambio en la estrategia QT. Esta pregunta es sencilla. La opci\u00f3n 4 nos est\u00e1 describiendo un estadio T4, bien por invasi\u00f3n de \u00f3rganos adyacentes o bien por perforaci\u00f3n. Los T4 conllevan una posibilidad de recidiva mucho mayor, de hecho hay estudios en marcha que plantean la HIPEC profil\u00e1ctica en T4 y la cirug\u00eda de second look + HIPEC en tumores de colon perforados.", "type": "CIRUG\u00cdA GENERAL", "options": {"1": "La presencia de anemia al diagn\u00f3stico.", "2": "La existencia de antecedentes familiares de cancer colorrectal.", "3": "El tama\u00f1o de la lesi\u00f3n primaria y la diferenciaci\u00f3n histol\u00f3gica.", "4": "La perforaci\u00f3n o adhesi\u00f3n del tumor a \u00f3rganos adyacentes.", "5": NaN}, "correct_option": 4, "explanations": {"1": {"exist": false, "char_ranges": [], "word_ranges": [], "text": ""}, "2": {"exist": false, "char_ranges": [], "word_ranges": [], "text": ""}, "3": {"exist": false, "char_ranges": [], "word_ranges": [], "text": ""}, "4": {"exist": true, "char_ranges": [[154, 465]], "word_ranges": [[25, 79]], "text": "La opci\u00f3n 4 nos est\u00e1 describiendo un estadio T4, bien por invasi\u00f3n de \u00f3rganos adyacentes o bien por perforaci\u00f3n. Los T4 conllevan una posibilidad de recidiva mucho mayor, de hecho hay estudios en marcha que plantean la HIPEC profil\u00e1ctica en T4 y la cirug\u00eda de second look + HIPEC en tumores de colon perforados."}, "5": {"exist": false, "char_ranges": [], "word_ranges": [], "text": ""}}} {"id": 321, "year": 2016, "question_id_specific": 144, "full_question": "Acude a una revisi\u00f3n programada encontr\u00e1ndose asintom\u00e1tico un hombre de 50 a\u00f1os diagnosticado de poliangeitis con granulomatosis 10 a\u00f1os antes y sin tratamiento desde hace 5. La radiograf\u00eda de t\u00f3rax, an\u00e1lisis de sangre y sedimento de orina son normales salvo unos anticuerpos anticitoplasma del neutr\u00f3filo (ANCA) positivos a titulo 1/320, con especificidad antiproteinasa 3, que previamente se hab\u00edan negativizado. \u00bfCu\u00e1l es la actitud terap\u00e9utica m\u00e1s aconsejable?", "full_answer": "En las vasculitis ANCA, sea cual sea, estos anticuerpos han demostrado relaci\u00f3n con la actividad de la enfermedad pero en ning\u00fan momento obligatoriedad de cambios en el tratamiento por sus niveles. En este caso, estando el paciente asintom\u00e1tico y con pruebas objetivas de falta de actividad, habr\u00eda que hacer un seguimiento estrecho sin reiniciar ni modificar tratamiento.", "type": "REUMATOLOG\u00cdA", "options": {"1": "Iniciar tratamiento con corticoides.", "2": "Iniciar tratamiento con ciclofosfamida.", "3": "Iniciar tratamiento con micofenolato de mofetilo.", "4": "Vigilancia expectante.", "5": NaN}, "correct_option": 4, "explanations": {"1": {"exist": true, "char_ranges": [[0, 197]], "word_ranges": [[0, 31]], "text": "En las vasculitis ANCA, sea cual sea, estos anticuerpos han demostrado relaci\u00f3n con la actividad de la enfermedad pero en ning\u00fan momento obligatoriedad de cambios en el tratamiento por sus niveles."}, "2": {"exist": true, "char_ranges": [[0, 197]], "word_ranges": [[0, 31]], "text": "En las vasculitis ANCA, sea cual sea, estos anticuerpos han demostrado relaci\u00f3n con la actividad de la enfermedad pero en ning\u00fan momento obligatoriedad de cambios en el tratamiento por sus niveles."}, "3": {"exist": true, "char_ranges": [[0, 197]], "word_ranges": [[0, 31]], "text": "En las vasculitis ANCA, sea cual sea, estos anticuerpos han demostrado relaci\u00f3n con la actividad de la enfermedad pero en ning\u00fan momento obligatoriedad de cambios en el tratamiento por sus niveles."}, "4": {"exist": true, "char_ranges": [[212, 372]], "word_ranges": [[34, 57]], "text": "estando el paciente asintom\u00e1tico y con pruebas objetivas de falta de actividad, habr\u00eda que hacer un seguimiento estrecho sin reiniciar ni modificar tratamiento."}, "5": {"exist": false, "char_ranges": [], "word_ranges": [], "text": ""}}} {"id": 516, "year": 2021, "question_id_specific": 103, "full_question": "Un paciente de 56 a\u00f1os es encontrado en coma en su domicilio. Tiene antecedentes de hipertensi\u00f3n arterial y diabetes mellitus. No tiene h\u00e1bitos t\u00f3xicos ni ninguna otra enfermedad cr\u00f3nica. Est\u00e1 en tratamiento con irbesartan y empaglifozina. No presenta signos de desnutrici\u00f3n. Presenta una presi\u00f3n arterial de 110/60 mmHg, una frecuencia cardiaca de 110 lpm, una SatO2 del 90 %, una glucosa capilar de 120 mg/dl y una frecuencia respiratoria de 7 rpm. \u00bfQu\u00e9 actitud terap\u00e9utica inicial le parece la m\u00e1s correcta?:", "full_answer": "Nos encontramos un paciente con hipoventilaci\u00f3n. La glucemia es correcta (descartamos coma hiperosmolar e hipoglucemia, opci\u00f3n 2 y 4), por lo que, aunque no tenga antecedentes de consumo de t\u00f3xicos, tenemos que ir a lo m\u00e1s frecuente y que podemos antagonizar con ant\u00eddotos: benzodiacepinas (flumazenilo) y opi\u00e1ceos (naloxona). Si la sospecha fuera de intoxaci\u00f3n por etilenglicol, administrar\u00edamos tiamina, pero no nos dan datos que nos haga sospechar.", "type": "CUIDADOS CR\u00cdTICOS Y URGENCIAS", "options": {"1": "Administrar naloxona, flumazenilo y tiamina.", "2": "Administrar naloxona, flumazenilo y glucosa hipert\u00f3nica.", "3": "Administrar naloxona y flumazenilo.", "4": "Administrar tiamina y glucosa hipert\u00f3nica.", "5": NaN}, "correct_option": 3, "explanations": {"1": {"exist": false, "char_ranges": [], "word_ranges": [], "text": ""}, "2": {"exist": false, "char_ranges": [], "word_ranges": [], "text": ""}, "3": {"exist": true, "char_ranges": [[147, 326]], "word_ranges": [[22, 48]], "text": "aunque no tenga antecedentes de consumo de t\u00f3xicos, tenemos que ir a lo m\u00e1s frecuente y que podemos antagonizar con ant\u00eddotos: benzodiacepinas (flumazenilo) y opi\u00e1ceos (naloxona)."}, "4": {"exist": true, "char_ranges": [[327, 451]], "word_ranges": [[48, 67]], "text": "Si la sospecha fuera de intoxaci\u00f3n por etilenglicol, administrar\u00edamos tiamina, pero no nos dan datos que nos haga sospechar."}, "5": {"exist": false, "char_ranges": [], "word_ranges": [], "text": ""}}} {"id": 44, "year": 2011, "question_id_specific": 153, "full_question": "Ni\u00f1o de 28 d\u00edas de vida, nacido a t\u00e9rmino. Antecedentes de embarazo y parto normales. Ha sido alimentado con lactancia materna desde el nacimiento. Consulta porque hace 8 d\u00edas comenz\u00f3 con v\u00f3mitos, inicialmente espor\u00e1dicos y desde hace 5 d\u00edas, despu\u00e9s de todas las tomas. Est\u00e1 permanentemente hambriento. Los v\u00f3mitos son de contenido alimentario \u201ca chorro\u201d. El abdomen, en la palpaci\u00f3n, es blando y depresible, sin visceromegalias. En los an\u00e1lisis destacaban en la gasometr\u00eda pH 7,49, bicarbonato 30 mEq/l, pCO2 53 mmHg, exceso de base +8 mEq/l. Iones: Na 137 mEq/l, K 3,1 mEq/l, Cl 94 mEq/l. Teniendo en cuenta el diagn\u00f3stico m\u00e1s probable en el cuadro cl\u00ednico descrito. \u00bfCu\u00e1l es la prueba complementaria de elecci\u00f3n para confirmar ese diagn\u00f3stico cl\u00ednico?", "full_answer": "La respuesta correcta es la 3. Pregunta f\u00e1cil si se sospecha la patolog\u00eda, que tambi\u00e9n creo que es sencillo porque la descripci\u00f3n cl\u00ednica es de libro de una estenosis hipertr\u00f3fica de p\u00edloro.", "type": "PEDIATR\u00cdA", "options": {"1": "Radiograf\u00eda simple de abdomen.", "2": "pHmetr\u00eda.", "3": "Ecograf\u00eda abdominal.", "4": "Determinaci\u00f3n de electrolitos en sudor.", "5": "Esofagogastroscopia."}, "correct_option": 3, "explanations": {"1": {"exist": false, "char_ranges": [], "word_ranges": [], "text": ""}, "2": {"exist": false, "char_ranges": [], "word_ranges": [], "text": ""}, "3": {"exist": true, "char_ranges": [[116, 190]], "word_ranges": [[20, 32]], "text": "la descripci\u00f3n cl\u00ednica es de libro de una estenosis hipertr\u00f3fica de p\u00edloro."}, "4": {"exist": false, "char_ranges": [], "word_ranges": [], "text": ""}, "5": {"exist": false, "char_ranges": [], "word_ranges": [], "text": ""}}} {"id": 54, "year": 2011, "question_id_specific": 72, "full_question": "Una estudiante de 22 a\u00f1os sin antecedentes patol\u00f3gicos ni uso de medicamentos salvo anovulatorios acude a Urgencias por deterioro del estado general y necesidad de respirar profundamente. Refiere haber perdido peso en los 2-3 \u00faltimos d\u00edas, polidipsia, poliuria y n\u00e1useas. No tos ni sensaci\u00f3n febril. Exploraci\u00f3n: aspecto de gravedad, presi\u00f3n arterial 100/60 mmHg, respiraci\u00f3n profunda y r\u00e1pida (28rpm), nivel de conciencia conservado, sequedad de mucosas. No fiebre. Anal\u00edtica: glucemia 420 mg/dL, Na+ 131 mEq/L, K+ normal, pH 7,08, bicarbonato 8 mEq/L y cetonuria (+++) \u00bfQu\u00e9 respuesta le parece m\u00e1s correcta?", "full_answer": "Debut de DM con cetoacidosis. La prioridad es la administraci\u00f3n de insulina iv y sueroterapia. Bicarbonato si pH < 7. Nunca debe retrasarse la administraci\u00f3n de la insulina.", "type": "ENDOCRINOLOG\u00cdA", "options": {"1": "Debut de una diabetes mellitus tipo 2, con disnea probablemente por neumon\u00eda o tromboembolismo, ya que toma anovulatorios.", "2": "Es una cetoacidosis diab\u00e9tica. Hay que tratar con insulina endovenosa, sueroterapia, medidas generales y buscar causa precipitante.", "3": "Debut de diabetes tipo 1 con cetoacidosis. Tratar con bicarbonato y tras corregir la acidosis, a\u00f1adir insulina endovenosa.", "4": "Parece una cetoacidosis diab\u00e9tica, pero podr\u00eda ser alcoh\u00f3lica. Se debe determinar la alcoholemia antes de iniciar tratamiento con insulina.", "5": "Tratar con insulina r\u00e1pida subcut\u00e1nea, sueroterapia y pedirle que beba l\u00edquidos en abundancia."}, "correct_option": 2, "explanations": {"1": {"exist": false, "char_ranges": [], "word_ranges": [], "text": ""}, "2": {"exist": true, "char_ranges": [[30, 173]], "word_ranges": [[5, 28]], "text": "La prioridad es la administraci\u00f3n de insulina iv y sueroterapia. Bicarbonato si pH < 7. Nunca debe retrasarse la administraci\u00f3n de la insulina."}, "3": {"exist": false, "char_ranges": [], "word_ranges": [], "text": ""}, "4": {"exist": false, "char_ranges": [], "word_ranges": [], "text": ""}, "5": {"exist": false, "char_ranges": [], "word_ranges": [], "text": ""}}} {"id": 553, "year": 2022, "question_id_specific": 177, "full_question": "Paciente de 33 a\u00f1os, residente en Valencia, que consulta por presentar en la mejilla derecha una placa eritemato-anaranjada tras picadura de insecto que ha ido creciendo lentamente hasta alcanzar 1,5 cm y que en las \u00faltimas semanas se ha ulcerado y cubierto de una escamocostra. La lesi\u00f3n solo le origina discretas molestias si se rasca. Entre sus antecedentes destaca psoriasis en placas en tratamiento con adalimumab subcut\u00e1neo. Tiene buen estado general y no toma otros f\u00e1rmacos ni refiere alergias a medicamentos. Indique el tratamiento que emplear\u00eda:", "full_answer": "El hecho que nos comenten el lugar de residencia ya nos da muchas pistas. Una \u201cpicadura\u201d que se transforma en una lesi\u00f3n en una zona descubierta con superficie costrosa y que molesta poco o nada\u2026 pues eso en la cuenca mediterr\u00e1nea es una leishmaniasis cut\u00e1nea o bot\u00f3n de Oriente. Y si bien es cierto que la lesi\u00f3n es de < 4 cm y \u00fanica, nos dicen que la paciente est\u00e1 en tratamiento con adalimumab, un anticuerpo monoclonal anti-TNF. A excepci\u00f3n del antimoniato de meglumina, ninguno de los otros tratamientos propuestos en las diferentes opciones est\u00e1 indicado en la leishmaniasis, as\u00ed que est\u00e1 claro. Lo \u00fanico que es m\u00e1s discutible es que, al tratarse de una paciente inmunodeprimida, probablemente el tratamiento sist\u00e9mico ser\u00eda m\u00e1s adecuado. No s\u00e9 si por ello impugnable, pero al menos, discutible.", "type": "DERMATOLOG\u00cdA", "options": {"1": "Isotretinoina oral.", "2": "Antimoniato de meglumina intralesional.", "3": "Corticoesteroides sist\u00e9micos.", "4": "Amoxicilina oral.", "5": NaN}, "correct_option": 2, "explanations": {"1": {"exist": true, "char_ranges": [[433, 601]], "word_ranges": [[78, 103]], "text": "A excepci\u00f3n del antimoniato de meglumina, ninguno de los otros tratamientos propuestos en las diferentes opciones est\u00e1 indicado en la leishmaniasis, as\u00ed que est\u00e1 claro."}, "2": {"exist": true, "char_ranges": [[433, 601]], "word_ranges": [[78, 103]], "text": "A excepci\u00f3n del antimoniato de meglumina, ninguno de los otros tratamientos propuestos en las diferentes opciones est\u00e1 indicado en la leishmaniasis, as\u00ed que est\u00e1 claro."}, "3": {"exist": true, "char_ranges": [[433, 601]], "word_ranges": [[78, 103]], "text": "A excepci\u00f3n del antimoniato de meglumina, ninguno de los otros tratamientos propuestos en las diferentes opciones est\u00e1 indicado en la leishmaniasis, as\u00ed que est\u00e1 claro."}, "4": {"exist": true, "char_ranges": [[433, 601]], "word_ranges": [[78, 103]], "text": "A excepci\u00f3n del antimoniato de meglumina, ninguno de los otros tratamientos propuestos en las diferentes opciones est\u00e1 indicado en la leishmaniasis, as\u00ed que est\u00e1 claro."}, "5": {"exist": false, "char_ranges": [], "word_ranges": [], "text": ""}}} {"id": 608, "year": 2022, "question_id_specific": 115, "full_question": "Var\u00f3n de 27 a\u00f1os, deportista habitual, que refiere dolor en la pierna derecha tras la pr\u00e1ctica de carrera continua. Ha acudido en varias ocasiones a un fisioterapeuta, siendo diagnosticado de una sobrecarga en gemelos. Han transcurrido varios meses, no ha mejorado y refiere dolor intenso tras la actividad f\u00edsica que cede con el reposo en las siguientes horas del ejercicio. \u00bfQu\u00e9 prueba puede ayudar en el diagn\u00f3stico?:", "full_answer": "Nos presentan un caso de s\u00edndrome compartimental cr\u00f3nico. Para su diagnostica es necesario medir la presi\u00f3n de los compartimentos afectos en reposo y tras la actividad (respuesta 2 correcta).", "type": "TRAUMATOLOG\u00cdA", "options": {"1": "Tomograf\u00eda de emisi\u00f3n de positrones con 18 FDG.", "2": "Determinaci\u00f3n de la presi\u00f3n del compartimento posterior inmediatamente tras actividad.", "3": "Ecograf\u00eda-Doppler para descartar un trastorno circulatorio de la extremidad inferior.", "4": "Espectrometr\u00eda por resonancia magn\u00e9tica.", "5": NaN}, "correct_option": 2, "explanations": {"1": {"exist": false, "char_ranges": [], "word_ranges": [], "text": ""}, "2": {"exist": true, "char_ranges": [[0, 191]], "word_ranges": [[0, 29]], "text": "Nos presentan un caso de s\u00edndrome compartimental cr\u00f3nico. Para su diagnostica es necesario medir la presi\u00f3n de los compartimentos afectos en reposo y tras la actividad (respuesta 2 correcta)."}, "3": {"exist": false, "char_ranges": [], "word_ranges": [], "text": ""}, "4": {"exist": false, "char_ranges": [], "word_ranges": [], "text": ""}, "5": {"exist": false, "char_ranges": [], "word_ranges": [], "text": ""}}} {"id": 294, "year": 2016, "question_id_specific": 44, "full_question": "Amalia es una ni\u00f1a de 3 a\u00f1os que ha sido intervenida de tetralog\u00eda de Fallot. Present\u00f3 adem\u00e1s una disfunci\u00f3n t\u00edmica en el periodo neonatal y una hipocalcemia as\u00ed como una voz nasal junto a retraso psicomotor. \u00bfQu\u00e9 prueba gen\u00e9tica considera m\u00e1s adecuada para llegar a su diagn\u00f3stico etiol\u00f3gico?", "full_answer": "Pregunta en la que hay que reconocer el cuadro cl\u00ednico: anomal\u00edas cardiovasculares \u2013 Tetralog\u00eda de Fallot -; aplasia/hipoplasia t\u00edmica -que conlleva inmunodeficiencia celular primaria por c\u00e9lulas T; aplasia/hipoplasia de paratiroides- de ah\u00ed la hipocalcemia-; anomal\u00edas del paladar \u2013 voz nasal ; retraso psicomotor. En mi opini\u00f3n, solo con las palabras \u201cdisfunci\u00f3n t\u00edmica\u201d, hipocalcemia y cardiopat\u00eda, tendr\u00edamos que irnos directos a pensar en S\u00edndrome de DiGeorge. Efectivamente el defecto gen\u00e9tico es una delecci\u00f3n 22q11.2 (que si nos lo sabemos, mejor, pero con la cl\u00ednica nos bastar\u00eda). La \u00fanica opci\u00f3n con la que podr\u00eda haber duda es con el Noonan, que tambi\u00e9n puede asociar cardiopat\u00eda, pero sobre todo se caracteriza por talla baja, cardiopat\u00eda y alteraciones \u00f3seas; adem\u00e1s, en la mayor\u00eda de los casos est\u00e1 producido por una alteraci\u00f3n en el gen PTPN11, no en los que menciona la pregunta.", "type": "GEN\u00c9TICA E INMUNOLOG\u00cdA", "options": {"1": "Estudio de delecci\u00f3n 22q11 (s\u00edndrome de Di George o Velo-cardio-facial).", "2": "Estudio de la expansi\u00f3n del gen FMR1, responsable del s\u00edndrome de X fr\u00e1gil.", "3": "Secuenciaci\u00f3n de los genes relacionados con el HRAS (s\u00edndrome de Noonan).", "4": "Estudio gen\u00e9tico de s\u00edndrome de Williams Beuren.", "5": NaN}, "correct_option": 1, "explanations": {"1": {"exist": true, "char_ranges": [[331, 465]], "word_ranges": [[46, 65]], "text": "solo con las palabras \u201cdisfunci\u00f3n t\u00edmica\u201d, hipocalcemia y cardiopat\u00eda, tendr\u00edamos que irnos directos a pensar en S\u00edndrome de DiGeorge."}, "2": {"exist": false, "char_ranges": [], "word_ranges": [], "text": ""}, "3": {"exist": true, "char_ranges": [[658, 773]], "word_ranges": [[99, 115]], "text": "tambi\u00e9n puede asociar cardiopat\u00eda, pero sobre todo se caracteriza por talla baja, cardiopat\u00eda y alteraciones \u00f3seas;"}, "4": {"exist": false, "char_ranges": [], "word_ranges": [], "text": ""}, "5": {"exist": false, "char_ranges": [], "word_ranges": [], "text": ""}}} {"id": 156, "year": 2012, "question_id_specific": 230, "full_question": "Paciente de 40 a\u00f1os, que acude a urgencias por estomatitits aftosa, conjuntivitis, uretritis, balanitis y dolor articular. \u00bfC\u00faal de los siguientes diagn\u00f3sticos es el m\u00e1s probable?", "full_answer": "Ante este cuadro cl\u00ednico y sin tener pruebas complementarias disponibles, el diagn\u00f3stico m\u00e1s probable es el de artritis reactiva. El \u00fanico problema es que en el enunciado se habla de dolor articular y no de artritis\u2026 El s\u00edndrome de Reiter (tr\u00edada de conjuntivitis, uretritis y artritis) es una de las formas en las que se puede presentar una artritis reactiva. En la enfermedad de Beh\u00e7et pueden aparecer aftas orales y genitales pero la uretritis no es una caracter\u00edstica. Y la afecci\u00f3n ocular propia es la uve\u00edtis.", "type": "REUMATOLOG\u00cdA", "options": {"1": "Beh\u00e7et.", "2": "S\u00edndrome de artritis reactiva.", "3": "S\u00edndrome de Sweet.", "4": "D\u00e9ficit de vitamina A.", "5": "Infecci\u00f3n por Gonococo."}, "correct_option": 2, "explanations": {"1": {"exist": true, "char_ranges": [[361, 472]], "word_ranges": [[60, 78]], "text": "En la enfermedad de Beh\u00e7et pueden aparecer aftas orales y genitales pero la uretritis no es una caracter\u00edstica."}, "2": {"exist": true, "char_ranges": [[217, 360]], "word_ranges": [[36, 60]], "text": "El s\u00edndrome de Reiter (tr\u00edada de conjuntivitis, uretritis y artritis) es una de las formas en las que se puede presentar una artritis reactiva."}, "3": {"exist": false, "char_ranges": [], "word_ranges": [], "text": ""}, "4": {"exist": false, "char_ranges": [], "word_ranges": [], "text": ""}, "5": {"exist": false, "char_ranges": [], "word_ranges": [], "text": ""}}} {"id": 82, "year": 2012, "question_id_specific": 45, "full_question": "Un hombre de 60 a\u00f1os de edad refiere dolor precordial tras los medianos esfuerzos. La coronariograf\u00eda muestra estenosis significativa en los segmentos proximales de los tres vasos principales con buen lecho distal. La funci\u00f3n ventricular est\u00e1 deprimida (<30%). \u00bfCu\u00e1l es la mejor opci\u00f3n terap\u00e9utica?", "full_answer": "Nos hablan de un paciente con enfermedad coronaria en 3 vasos y, muy importante, con disfunci\u00f3n ventricular severa. En estas condiciones, el tto. de elecci\u00f3n ser\u00eda quir\u00fargico (cirug\u00eda de revascularizaci\u00f3n mioc\u00e1rdica o by-pass). En el caso de que nos hubieran hablado de un paciente con funci\u00f3n del VI conservada, el elegir el tto. quir\u00fargico vs. percut\u00e1neo ser\u00eda m\u00e1s cuestionable, pues hoy en d\u00eda gracias a los stents farmacoactivos las diferencias en t\u00e9rminos de morbimortalidad entre la cirug\u00eda de by-pass y la angioplastia son m\u00ednimas.", "type": "CARDIOLOG\u00cdA Y CIRUG\u00cdA VASCULAR", "options": {"1": "Revascularizaci\u00f3n percut\u00e1nea.", "2": "Tratamiento m\u00e9dico.", "3": "Cirug\u00eda de revascularizaci\u00f3n mioc\u00e1rdica.", "4": "Implantar bal\u00f3n de contrapulsaci\u00f3n.", "5": "Trasplante cardiaco."}, "correct_option": 3, "explanations": {"1": {"exist": false, "char_ranges": [], "word_ranges": [], "text": ""}, "2": {"exist": false, "char_ranges": [], "word_ranges": [], "text": ""}, "3": {"exist": true, "char_ranges": [[0, 227]], "word_ranges": [[0, 33]], "text": "Nos hablan de un paciente con enfermedad coronaria en 3 vasos y, muy importante, con disfunci\u00f3n ventricular severa. En estas condiciones, el tto. de elecci\u00f3n ser\u00eda quir\u00fargico (cirug\u00eda de revascularizaci\u00f3n mioc\u00e1rdica o by-pass)."}, "4": {"exist": false, "char_ranges": [], "word_ranges": [], "text": ""}, "5": {"exist": false, "char_ranges": [], "word_ranges": [], "text": ""}}} {"id": 485, "year": 2020, "question_id_specific": 49, "full_question": "Hombre de 60 a\u00f1os en tratamiento con carbamazepina por epilepsia y que presenta fiebre (38.8\u00ba C), odinofagia, conjuntivitis, lesiones cut\u00e1neas parduzcas extendidas en una gran superficie corporal, de aspecto similar a una diana y que se acompa\u00f1an de desprendimiento epid\u00e9rmico (superior al 30%) al menor roce. Es muy probable que est\u00e9 presentando un cuadro cl\u00ednico de:", "full_answer": "Estamos ante un paciente grave, con lesiones en diana y signo de Nikolsky en > 30% de la superficie corporal. Junto con el antecedente del tratamiento con carbamazepina la sospecha ser\u00e1 de necrolisis epid\u00e9rmica t\u00f3xica (respuesta 4). En el caso de que el despegamiento epid\u00e9rmico fuera de menos del 10% entonces estar\u00edamos ante un Stevens-Johnson (y los casos intermedios se habla de overlap, ya que es todo un espectro). Por cierto, recomendaremos ingreso inmediato en una Unidad de Grandes Quemados.", "type": "DERMATOLOG\u00cdA", "options": {"1": "Urticaria aguda de causa farmacol\u00f3gica.", "2": "Eritema exudativo minor secundario a f\u00e1rmacos.", "3": "S\u00edndrome de Stevens-Johnson farmacol\u00f3gico.", "4": "Necrolisis t\u00f3xica epid\u00e9rmica.", "5": NaN}, "correct_option": 4, "explanations": {"1": {"exist": false, "char_ranges": [], "word_ranges": [], "text": ""}, "2": {"exist": false, "char_ranges": [], "word_ranges": [], "text": ""}, "3": {"exist": true, "char_ranges": [[233, 345]], "word_ranges": [[37, 55]], "text": "En el caso de que el despegamiento epid\u00e9rmico fuera de menos del 10% entonces estar\u00edamos ante un Stevens-Johnson"}, "4": {"exist": true, "char_ranges": [[0, 232]], "word_ranges": [[0, 37]], "text": "Estamos ante un paciente grave, con lesiones en diana y signo de Nikolsky en > 30% de la superficie corporal. Junto con el antecedente del tratamiento con carbamazepina la sospecha ser\u00e1 de necrolisis epid\u00e9rmica t\u00f3xica (respuesta 4)."}, "5": {"exist": false, "char_ranges": [], "word_ranges": [], "text": ""}}} {"id": 219, "year": 2014, "question_id_specific": 124, "full_question": "Mujer de 72 a\u00f1os con diabetes tipo 2 e insuficiencia renal cr\u00f3nica en estadio 5. Ante la sospecha de un tromboembolismo pulmonar indique qu\u00e9 prueba diagn\u00f3stica est\u00e1 contraindicada:", "full_answer": "Pregunta muy sencilla y muy \u00fatil para el manejo en Urgencias del TEP. No hay que comerse la cabeza: paciente con insuficiencia renal avanzada (estad\u00edo 5) tiene contraindicado cualquier estudio de imagen que precise la administraci\u00f3n de contraste IV, como es el angioTAC. Todas las dem\u00e1s exploraciones no lo precisan y por tanto estar\u00edan m\u00e1s que indicadas ante la presencia de un TEP.", "type": "NEFROLOG\u00cdA", "options": {"1": "Ecocardiograma trastor\u00e1cico.", "2": "Gammagraf\u00eda pulmonar.", "3": "Angio-TAC pulmonar.", "4": "Electrocardiograma.", "5": "Radiograf\u00eda de t\u00f3rax."}, "correct_option": 3, "explanations": {"1": {"exist": false, "char_ranges": [], "word_ranges": [], "text": ""}, "2": {"exist": false, "char_ranges": [], "word_ranges": [], "text": ""}, "3": {"exist": true, "char_ranges": [[100, 270]], "word_ranges": [[19, 43]], "text": "paciente con insuficiencia renal avanzada (estad\u00edo 5) tiene contraindicado cualquier estudio de imagen que precise la administraci\u00f3n de contraste IV, como es el angioTAC."}, "4": {"exist": false, "char_ranges": [], "word_ranges": [], "text": ""}, "5": {"exist": false, "char_ranges": [], "word_ranges": [], "text": ""}}} {"id": 578, "year": 2022, "question_id_specific": 107, "full_question": "Var\u00f3n de 55 a\u00f1os que sufre una hemorragia subaracnoidea por rotura de un aneurisma de arteria cerebral media. Es tratado de forma quir\u00fargica. Tras una buena evoluci\u00f3n en la que recupera la capacidad de caminar, un mes despu\u00e9s de sufrir la hemorragia presenta un deterioro progresivo, llegando a encontrarse obnubilado y con dificultad para obedecer \u00f3rdenes. Se\u00f1ale la afirmaci\u00f3n correcta:", "full_answer": "El vasoespasmo como complicaci\u00f3n de la hemorragia subaracnoidea suele producirse de una forma precoz, en la semana posterior a la hemorragia. El deterioro progresivo con alteraci\u00f3n del nivel de conciencia nos hace sospechar en una de las complicaciones que puede producirse en este cuadro: la hidrocefalia. La t\u00e9cnica diagn\u00f3stica de elecci\u00f3n en este caso es la neuroimagen con TC craneal.", "type": "NEUROLOG\u00cdA", "options": {"1": "Se encuentra en el momento de la evoluci\u00f3n en el que es m\u00e1s frecuente el vasoespasmo.", "2": "Probablemente este deterioro progresivo sea secundario al resangrado del aneurisma.", "3": "El deterioro subagudo que presenta con mayor probabilidad se debe a hidrocefalia.", "4": "La t\u00e9cnica diagn\u00f3stica de elecci\u00f3n es la angiograf\u00eda cerebral.", "5": NaN}, "correct_option": 3, "explanations": {"1": {"exist": false, "char_ranges": [], "word_ranges": [], "text": ""}, "2": {"exist": false, "char_ranges": [], "word_ranges": [], "text": ""}, "3": {"exist": true, "char_ranges": [[0, 306]], "word_ranges": [[0, 46]], "text": "El vasoespasmo como complicaci\u00f3n de la hemorragia subaracnoidea suele producirse de una forma precoz, en la semana posterior a la hemorragia. El deterioro progresivo con alteraci\u00f3n del nivel de conciencia nos hace sospechar en una de las complicaciones que puede producirse en este cuadro: la hidrocefalia."}, "4": {"exist": false, "char_ranges": [], "word_ranges": [], "text": ""}, "5": {"exist": false, "char_ranges": [], "word_ranges": [], "text": ""}}} {"id": 199, "year": 2013, "question_id_specific": 74, "full_question": "En un paciente que se presenta en la segunda d\u00e9cada de su vida con un cuadro progresivo de parkinsonismo, temblor, diston\u00eda y alteraci\u00f3n de conducta, usted debe siempre realizar un estudio para descartar la presencia de una enfermedad cuyo tratamiento adecuado puede mejorar los s\u00edntomas neurol\u00f3gicos y detener el curso cl\u00ednico. \u00bfDe qu\u00e9 enfermedad se trata?", "full_answer": "La clave en la pregunta es la de \u00abdetener el curso cl\u00ednico\u00bb, ya que la enfermedad de Hungtinton, el Tourette y la enfermedad de Parkinson no disponen de tratamientos que puedan detener el curso cl\u00ednico. La Corea de Sydenham es t\u00edpica de ni\u00f1os m\u00e1s peque\u00f1os con el antecedente de infecci\u00f3n estreptoc\u00f3cica y la resoluci\u00f3n suele ser espont\u00e1nea. Por otra parte, en los casos cl\u00ednicos se recomienda descartar la enfermedad de Wilson en pacientes j\u00f3venes con trastornos del movimiento y neuropsiqui\u00e1tricos.", "type": "NEUROLOG\u00cdA", "options": {"1": "Enfermedad de Huntington.", "2": "Enfermedad de Wilson.", "3": "S\u00edndrome de Tourette.", "4": "Corea de Sydenham.", "5": "Enfermedad de Parkinson de inicio juvenil."}, "correct_option": 2, "explanations": {"1": {"exist": true, "char_ranges": [[0, 202]], "word_ranges": [[0, 35]], "text": "La clave en la pregunta es la de \u00abdetener el curso cl\u00ednico\u00bb, ya que la enfermedad de Hungtinton, el Tourette y la enfermedad de Parkinson no disponen de tratamientos que puedan detener el curso cl\u00ednico."}, "2": {"exist": true, "char_ranges": [[357, 499]], "word_ranges": [[60, 80]], "text": "en los casos cl\u00ednicos se recomienda descartar la enfermedad de Wilson en pacientes j\u00f3venes con trastornos del movimiento y neuropsiqui\u00e1tricos."}, "3": {"exist": true, "char_ranges": [[0, 202]], "word_ranges": [[0, 35]], "text": "La clave en la pregunta es la de \u00abdetener el curso cl\u00ednico\u00bb, ya que la enfermedad de Hungtinton, el Tourette y la enfermedad de Parkinson no disponen de tratamientos que puedan detener el curso cl\u00ednico."}, "4": {"exist": true, "char_ranges": [[203, 340]], "word_ranges": [[35, 57]], "text": "La Corea de Sydenham es t\u00edpica de ni\u00f1os m\u00e1s peque\u00f1os con el antecedente de infecci\u00f3n estreptoc\u00f3cica y la resoluci\u00f3n suele ser espont\u00e1nea."}, "5": {"exist": true, "char_ranges": [[0, 202]], "word_ranges": [[0, 35]], "text": "La clave en la pregunta es la de \u00abdetener el curso cl\u00ednico\u00bb, ya que la enfermedad de Hungtinton, el Tourette y la enfermedad de Parkinson no disponen de tratamientos que puedan detener el curso cl\u00ednico."}}} {"id": 331, "year": 2016, "question_id_specific": 87, "full_question": "En un paciente hospitalizado que no es capaz de alimentarse por v\u00eda oral durante m\u00e1s de 6 d\u00edas, \u00bfen cu\u00e1l de las situaciones cl\u00ednicas siguientes hemos de usar necesariamente nutrici\u00f3n parenteral?", "full_answer": "El \u00edleo paral\u00edtico es un fallo en la propulsi\u00f3n intestinal que aparece de forma aguda en ausencia de obstrucci\u00f3n mec\u00e1nica; es secundario a diversas causas y afecta a la contractilidad intestinal. Por lo que es necesaria la v\u00eda parenteral.", "type": "ENDOCRINOLOG\u00cdA", "options": {"1": "Ictus cardioemb\u00f3lico con disfagia neurol\u00f3gica completa.", "2": "Caquexia por empiema cr\u00f3nico en paciente inmunodeprimido.", "3": "lleo paral\u00edtico prolongado.", "4": "Enfermedad de Alzheimer avanzada con grave riesgo de broncoaspiraci\u00f3n.", "5": NaN}, "correct_option": 3, "explanations": {"1": {"exist": false, "char_ranges": [], "word_ranges": [], "text": ""}, "2": {"exist": false, "char_ranges": [], "word_ranges": [], "text": ""}, "3": {"exist": true, "char_ranges": [[0, 238]], "word_ranges": [[0, 39]], "text": "El \u00edleo paral\u00edtico es un fallo en la propulsi\u00f3n intestinal que aparece de forma aguda en ausencia de obstrucci\u00f3n mec\u00e1nica; es secundario a diversas causas y afecta a la contractilidad intestinal. Por lo que es necesaria la v\u00eda parenteral."}, "4": {"exist": false, "char_ranges": [], "word_ranges": [], "text": ""}, "5": {"exist": false, "char_ranges": [], "word_ranges": [], "text": ""}}} {"id": 205, "year": 2014, "question_id_specific": 214, "full_question": "Joven de 24 a\u00f1os que a los 3 d\u00edas de un contacto sexual de riesgo presenta numerosas lesiones pustulosas, peque\u00f1as, muy pruriginosas y que evolucionan a diminutas erosiones afectando todo el glande y la cara interna del prepucio. Se\u00f1ale cu\u00e1l es, entre las siguientes, la orientaci\u00f3n diagn\u00f3stica m\u00e1s probable:", "full_answer": "El chancroide es una lesi\u00f3n solitaria. La s\u00edfilis secundaria suele darse como un exantema generalizado o como lesiones car\u00e1cteristicas en palmas y plantas (clavos sifil\u00edticos) con un periodo de latencia m\u00e1s largo. La balanitis por tricomonas asocia siempre uretrtitis, que no se refiere en la pregunta. Por tanto, la respuesta es la 1. Sin embargo, la cl\u00ednica sugiere m\u00e1s un herpes simple que una candidiasis y la 5 habla tambi\u00e9n de hongos (aunque los dermatofitos no dan balanitis)\u2026. desde nuestro punto de vista, a\u00fan dando por correcta la 1, la pregunta induce a confusi\u00f3n, podr\u00eda ser impugnable. Adem\u00e1s, la candidiasis genital no es necesariamente de transmisi\u00f3n sexual.", "type": "DERMATOLOG\u00cdA", "options": {"1": "Candidiasis genital.", "2": "Chancroide.", "3": "S\u00edfilis secundaria.", "4": "Balanitis por Tricomonas.", "5": "Infecci\u00f3n f\u00fangica por dermatofitos."}, "correct_option": 1, "explanations": {"1": {"exist": false, "char_ranges": [], "word_ranges": [], "text": ""}, "2": {"exist": true, "char_ranges": [[0, 38]], "word_ranges": [[0, 6]], "text": "El chancroide es una lesi\u00f3n solitaria."}, "3": {"exist": true, "char_ranges": [[39, 213]], "word_ranges": [[6, 32]], "text": "La s\u00edfilis secundaria suele darse como un exantema generalizado o como lesiones car\u00e1cteristicas en palmas y plantas (clavos sifil\u00edticos) con un periodo de latencia m\u00e1s largo."}, "4": {"exist": true, "char_ranges": [[214, 302]], "word_ranges": [[32, 46]], "text": "La balanitis por tricomonas asocia siempre uretrtitis, que no se refiere en la pregunta."}, "5": {"exist": false, "char_ranges": [], "word_ranges": [], "text": ""}}} {"id": 441, "year": 2018, "question_id_specific": 98, "full_question": "Mujer de 21 a\u00f1os de edad que acude a consulta aportando un estudio hormonal con FSH 0,29 mUl/mL (normal en fase folicular = 3,50-12,50 mUl/mL) y LH < 0,1 mUl/mL (normal en fase fol\u00edcular 2,40-12,60 mUl/mL). Deberemos realizar un diagn\u00f3stico diferencial entre las siguientes situaciones cl\u00ednicas EXCEPTO:", "full_answer": "Esto es as\u00ed porque en el fallo ov\u00e1rico precoz la FSH y la LH est\u00e1n elevados. As\u00ed lo confirma el protocolo llamado Menogu\u00eda Precoz de la SEGO. El resto de diagn\u00f3sticos s\u00ed concuerdan con dichos par\u00e1metros anal\u00edticos.", "type": "GINECOLOG\u00cdA Y OBSTETRICIA", "options": {"1": "Administraci\u00f3n de anticonceptivos orales combinados.", "2": "Fallo ovarico prematuro.", "3": "Trastorno del comportamiento alimentario.", "4": "Craneofaringioma.", "5": NaN}, "correct_option": 2, "explanations": {"1": {"exist": true, "char_ranges": [[142, 214]], "word_ranges": [[27, 37]], "text": "El resto de diagn\u00f3sticos s\u00ed concuerdan con dichos par\u00e1metros anal\u00edticos."}, "2": {"exist": true, "char_ranges": [[0, 141]], "word_ranges": [[0, 27]], "text": "Esto es as\u00ed porque en el fallo ov\u00e1rico precoz la FSH y la LH est\u00e1n elevados. As\u00ed lo confirma el protocolo llamado Menogu\u00eda Precoz de la SEGO."}, "3": {"exist": true, "char_ranges": [[142, 214]], "word_ranges": [[27, 37]], "text": "El resto de diagn\u00f3sticos s\u00ed concuerdan con dichos par\u00e1metros anal\u00edticos."}, "4": {"exist": true, "char_ranges": [[142, 214]], "word_ranges": [[27, 37]], "text": "El resto de diagn\u00f3sticos s\u00ed concuerdan con dichos par\u00e1metros anal\u00edticos."}, "5": {"exist": false, "char_ranges": [], "word_ranges": [], "text": ""}}} {"id": 302, "year": 2016, "question_id_specific": 111, "full_question": "En una paciente con artritis reumatoide en tratamiento con metotrexate, prednisona e indometacina que presenta de forma aguda edemas y aumento de la creatinina plasm\u00e1tica con un sedimento de orina poco expresivo y proteinuria inferior a 100 mg/24 h la causa m\u00e1s probable es:", "full_answer": "La insuficiencia renal aguda prerrenal por AINEs parece la opci\u00f3n m\u00e1s plausible en este caso. La nefritis t\u00fabulo-intersticial por metotrexate es menos probable por la ausencia de rash, fiebre o eosinofilia, que aunque son poco frecuentes en el MIR suelen aparecer. Aunque en la AR es relativamente frecuente la aparici\u00f3n de GMN y amiloidosis renal, el sedimento de orina anodino y la ausencia de proteinuria hacen poco probables estas opciones.", "type": "NEFROLOG\u00cdA", "options": {"1": "Amiloidosis renal.", "2": "Glomerulonefritis secundaria a la artritis reumatoide.", "3": "Insuficiencia renal por antiinflamatorios no esteroideos.", "4": "Nefritis intersticial por metotrexate.", "5": NaN}, "correct_option": 3, "explanations": {"1": {"exist": true, "char_ranges": [[265, 444]], "word_ranges": [[41, 70]], "text": "Aunque en la AR es relativamente frecuente la aparici\u00f3n de GMN y amiloidosis renal, el sedimento de orina anodino y la ausencia de proteinuria hacen poco probables estas opciones."}, "2": {"exist": false, "char_ranges": [], "word_ranges": [], "text": ""}, "3": {"exist": true, "char_ranges": [[0, 93]], "word_ranges": [[0, 15]], "text": "La insuficiencia renal aguda prerrenal por AINEs parece la opci\u00f3n m\u00e1s plausible en este caso."}, "4": {"exist": true, "char_ranges": [[94, 264]], "word_ranges": [[15, 41]], "text": "La nefritis t\u00fabulo-intersticial por metotrexate es menos probable por la ausencia de rash, fiebre o eosinofilia, que aunque son poco frecuentes en el MIR suelen aparecer."}, "5": {"exist": false, "char_ranges": [], "word_ranges": [], "text": ""}}} {"id": 517, "year": 2021, "question_id_specific": 104, "full_question": "Un var\u00f3n de 34 a\u00f1os sufre un accidente de tr\u00e1fico y es atendido en el lugar del accidente. El personal sanitario objetiva que est\u00e1 p\u00e1lido, sudoroso, tiene pulso filiforme con una presi\u00f3n arterial de 90/50 mmHg, una frecuencia cardiaca de 127 lpm, una frecuencia respiratoria de 28 rpm y una saturaci\u00f3n de ox\u00edgeno de 92 %. Se le infunden 20 ml/kg de cristaloides. Durante su traslado al hospital los signos vitales mejoran transitoriamente y despu\u00e9s se deterioran a su llegada al hospital. Se\u00f1ale la respuesta correcta:", "full_answer": "En un paciente politraumatizado, la primera causa del shock es la p\u00e9rdida hem\u00e1tica (shock hemorr\u00e1gico). Las pautas a seguir siempre deben contemplar la reposici\u00f3n de la volemia (a ser posible, con hemoderivados lo m\u00e1s precozmente posible) y el control del foco. En este caso, ante el compromiso vital, la necesidad de trasfusi\u00f3n es muy urgente, por lo que la indicaci\u00f3n es la trasfusi\u00f3n de sangre de emergencia (O Rh negativo).", "type": "CUIDADOS CR\u00cdTICOS Y URGENCIAS", "options": {"1": "Requiere transfusi\u00f3n de sangre tipo-espec\u00edfica.", "2": "Requiere transfusi\u00f3n de sangre de emergencia (O Rh negativo).", "3": "Debe prepararse una posible transfusi\u00f3n de sangre con pruebas cruzadas.", "4": "Debe probarse la infusi\u00f3n de nuevo de otros 20 ml/kg de cristaloides.", "5": NaN}, "correct_option": 2, "explanations": {"1": {"exist": false, "char_ranges": [], "word_ranges": [], "text": ""}, "2": {"exist": true, "char_ranges": [[262, 427]], "word_ranges": [[41, 70]], "text": "En este caso, ante el compromiso vital, la necesidad de trasfusi\u00f3n es muy urgente, por lo que la indicaci\u00f3n es la trasfusi\u00f3n de sangre de emergencia (O Rh negativo)."}, "3": {"exist": false, "char_ranges": [], "word_ranges": [], "text": ""}, "4": {"exist": false, "char_ranges": [], "word_ranges": [], "text": ""}, "5": {"exist": false, "char_ranges": [], "word_ranges": [], "text": ""}}} {"id": 520, "year": 2021, "question_id_specific": 38, "full_question": "Paciente de 72 a\u00f1os de edad que acude a urgencias por presentar aparatoso e importante angioedema lingual. No presenta urticaria asociada ni refiere antecedentes de alergia a f\u00e1rmacos o alimentos. Entre sus antecedentes personales destacan diabetes mellitus tipo 2, dislipemia, hipertensi\u00f3n arterial, hipotiroidismo y enfermedad de Parkinson. En tratamiento habitual con metformina, simvastatina, enalapril, tiroxina y levodopa. \u00bfCu\u00e1l de los siguientes f\u00e1rmacos es con mayor probabilidad el f\u00e1rmaco causal del cuadro cl\u00ednico descrito?:", "full_answer": "El angioedema es un tipo de urticaria profunda que puede ser de etiolog\u00eda medicamentosa (no mediado por IgE) y en este caso, los IECA (como el enalapril) se consideran la causa farmacol\u00f3gica m\u00e1s frecuente, as\u00ed que en este caso ser\u00eda la respuesta correcta (si hablamos en t\u00e9rminos probabil\u00edsticos).", "type": "DERMATOLOG\u00cdA", "options": {"1": "Enalapril.", "2": "Simvastatina.", "3": "Metformina o levodopa por igual.", "4": "Levodopa.", "5": NaN}, "correct_option": 1, "explanations": {"1": {"exist": true, "char_ranges": [[0, 297]], "word_ranges": [[0, 48]], "text": "El angioedema es un tipo de urticaria profunda que puede ser de etiolog\u00eda medicamentosa (no mediado por IgE) y en este caso, los IECA (como el enalapril) se consideran la causa farmacol\u00f3gica m\u00e1s frecuente, as\u00ed que en este caso ser\u00eda la respuesta correcta (si hablamos en t\u00e9rminos probabil\u00edsticos)."}, "2": {"exist": false, "char_ranges": [], "word_ranges": [], "text": ""}, "3": {"exist": false, "char_ranges": [], "word_ranges": [], "text": ""}, "4": {"exist": false, "char_ranges": [], "word_ranges": [], "text": ""}, "5": {"exist": false, "char_ranges": [], "word_ranges": [], "text": ""}}} {"id": 379, "year": 2016, "question_id_specific": 133, "full_question": "Una chica de 20 a\u00f1os acude a urgencias porque ha notado tras levantarse, al mirarse al espejo, debilidad en toda la mitad derecha de la cara (incluida la frente, cerrar el p\u00e1rpado y para sonreir) Se acompa\u00f1a de disgeusia, con sensaci\u00f3n de sabor met\u00e1lico de los alimentos asi como de hiperacusia y dolor mastoideo ipsilateral. En la exploraci\u00f3n no se evidencia d\u00e9ficit de la fuerza ni d\u00e9ficits sensitivos en extremidades ni alteraciones del habla ni lenguaje. En este caso,\u00bfcu\u00e1l de las siguientes aseveraciones es la correcta?", "full_answer": "Nos describen un cuadro de par\u00e1lisis facial perif\u00e9rica t\u00edpica, con disgeusia, hiperacusia y dolor mastoideo. En este caso el tratamiento es corticoideo. No ser\u00eda correcta la primera porque aunque dentro del rango de edad de debut de la esclerosis m\u00faltiple normalmente la presentaci\u00f3n es gradual a lo largo de d\u00edas y la afectaci\u00f3n nuclear del VII nervio craneal (que podr\u00eda simular un facial perif\u00e9rico) no provocar\u00eda disgeusia, hiperacusia ni dolor mastoideo. En la tercera respuesta nos proponen que se trate de un cuadro vascular y por tanto que la par\u00e1lisis facial sea central, por lo que queda descartada y la \u00faltima no es correcta porque la mayor\u00eda de las par\u00e1lisis faciales perif\u00e9ricas idiop\u00e1ticas (par\u00e1lisis de Bell) curan sin secuelas.", "type": "NEUROLOG\u00cdA", "options": {"1": "El diagn\u00f3stico m\u00e1s probable es una placa desmielinizante en hemiprotuberancia contralateral, la exploraci\u00f3n complementaria m\u00e1s necesaria Ser\u00eda una resonancia magn\u00e9tica craneal.", "2": "Los corticoides v\u00eda oral son de elecci\u00f3n en el tratamiento de la paciente.", "3": "Debe ser considerada la fibrinolisis endovenosa si el tiempo de evoluci\u00f3n es menor a 3h.", "4": "Lo m\u00e1s probable es que el cuadro sea irreversible.", "5": NaN}, "correct_option": 2, "explanations": {"1": {"exist": true, "char_ranges": [[153, 459]], "word_ranges": [[22, 71]], "text": "No ser\u00eda correcta la primera porque aunque dentro del rango de edad de debut de la esclerosis m\u00faltiple normalmente la presentaci\u00f3n es gradual a lo largo de d\u00edas y la afectaci\u00f3n nuclear del VII nervio craneal (que podr\u00eda simular un facial perif\u00e9rico) no provocar\u00eda disgeusia, hiperacusia ni dolor mastoideo."}, "2": {"exist": true, "char_ranges": [[0, 152]], "word_ranges": [[0, 22]], "text": "Nos describen un cuadro de par\u00e1lisis facial perif\u00e9rica t\u00edpica, con disgeusia, hiperacusia y dolor mastoideo. En este caso el tratamiento es corticoideo."}, "3": {"exist": true, "char_ranges": [[460, 608]], "word_ranges": [[71, 98]], "text": "En la tercera respuesta nos proponen que se trate de un cuadro vascular y por tanto que la par\u00e1lisis facial sea central, por lo que queda descartada"}, "4": {"exist": true, "char_ranges": [[611, 743]], "word_ranges": [[99, 119]], "text": "la \u00faltima no es correcta porque la mayor\u00eda de las par\u00e1lisis faciales perif\u00e9ricas idiop\u00e1ticas (par\u00e1lisis de Bell) curan sin secuelas."}, "5": {"exist": false, "char_ranges": [], "word_ranges": [], "text": ""}}} {"id": 600, "year": 2022, "question_id_specific": 184, "full_question": "Mujer de 70 a\u00f1os que refiere desde hace 1 mes dolor y rigidez en cinturas escapular y pelviana, que predomina al levantarse y mejora con la movilizaci\u00f3n, sin fiebre, cefalea ni afectaci\u00f3n del estado general. En la exploraci\u00f3n f\u00edsica se observa rigidez y dolor a la movilizaci\u00f3n de hombros y caderas. La anal\u00edtica muestra una elevaci\u00f3n de la VSG y la prote\u00edna C reactiva. \u00bfQu\u00e9 diagn\u00f3stico es m\u00e1s probable y qu\u00e9 tratamiento prescribir\u00eda?:", "full_answer": "En este caso, nos esta describiendo los s\u00edntomas cardinales de la polimialgia reum\u00e1tica. Paciente mayor, por encima de los 50 a\u00f1os, que presenta dolor y rigidez en ambas cinturas, asociado a una elevaci\u00f3n de los reactantes de fase aguda. Presenta los todos los criterios indispensables: edad igual o mayor a 50 a\u00f1os, omalgia bilateral y aumento de reactantes de fase aguda. De los criterios opcionales, presenta rigidez, dolor o limitaci\u00f3n en cadera. El tratamiento inicial son corticoides a dosis medias.", "type": "REUMATOLOG\u00cdA", "options": {"1": "Artritis reumatoide, iniciar\u00eda tratamiento con metotrexato.", "2": "Fibromialgia, iniciar\u00eda tratamiento con duloxetina.", "3": "Polimialgia reum\u00e1tica, iniciar\u00eda tratamiento con corticoides.", "4": "Espondilitis anquilosante, iniciar\u00eda tratamiento con antiinflamatorios no esteroideos.", "5": NaN}, "correct_option": 3, "explanations": {"1": {"exist": false, "char_ranges": [], "word_ranges": [], "text": ""}, "2": {"exist": false, "char_ranges": [], "word_ranges": [], "text": ""}, "3": {"exist": true, "char_ranges": [[0, 373]], "word_ranges": [[0, 61]], "text": "En este caso, nos esta describiendo los s\u00edntomas cardinales de la polimialgia reum\u00e1tica. Paciente mayor, por encima de los 50 a\u00f1os, que presenta dolor y rigidez en ambas cinturas, asociado a una elevaci\u00f3n de los reactantes de fase aguda. Presenta los todos los criterios indispensables: edad igual o mayor a 50 a\u00f1os, omalgia bilateral y aumento de reactantes de fase aguda."}, "4": {"exist": false, "char_ranges": [], "word_ranges": [], "text": ""}, "5": {"exist": false, "char_ranges": [], "word_ranges": [], "text": ""}}} {"id": 415, "year": 2018, "question_id_specific": 72, "full_question": "Un hombre de 45 a\u00f1os, con antecedentes de un cuadro febril y dolor toracico, acude al hospital con disnea y taquipnea. En la exploraci\u00f3n fisica las cifras de presi\u00f3n arterial estan bajas, la presi\u00f3n venosa yugular elevada con un seno x descendente prominente y tiene pulso parad\u00f3jico. \u00bfQu\u00e9 diagn\u00f3stico sospechar\u00eda?", "full_answer": "Es la exploraci\u00f3n cl\u00e1sica: tuvo cuadro febril y dolor, hizo una pericarditis con derrame, y ahora este est\u00e1 provocando un taponamiento. De ah\u00ed la hipotensi\u00f3n (su coraz\u00f3n no puede llenarse bien de sangre y bombear), la ingurgitaci\u00f3n yugular y la dependencia de la respiraci\u00f3n para conseguir precarga y gasto card\u00edaco (i.e. pulso parad\u00f3jico).", "type": "CARDIOLOG\u00cdA Y CIRUG\u00cdA CARDIOVASCULAR", "options": {"1": "Miocardiopat\u00eda dilatada.", "2": "Derrame pericardico con taponamiento cardiaco.", "3": "Pericarditis constrictiva.", "4": "Cardiopatia isqu\u00e9mica.", "5": NaN}, "correct_option": 2, "explanations": {"1": {"exist": false, "char_ranges": [], "word_ranges": [], "text": ""}, "2": {"exist": true, "char_ranges": [[27, 340]], "word_ranges": [[4, 53]], "text": "tuvo cuadro febril y dolor, hizo una pericarditis con derrame, y ahora este est\u00e1 provocando un taponamiento. De ah\u00ed la hipotensi\u00f3n (su coraz\u00f3n no puede llenarse bien de sangre y bombear), la ingurgitaci\u00f3n yugular y la dependencia de la respiraci\u00f3n para conseguir precarga y gasto card\u00edaco (i.e. pulso parad\u00f3jico)."}, "3": {"exist": false, "char_ranges": [], "word_ranges": [], "text": ""}, "4": {"exist": false, "char_ranges": [], "word_ranges": [], "text": ""}, "5": {"exist": false, "char_ranges": [], "word_ranges": [], "text": ""}}} {"id": 560, "year": 2022, "question_id_specific": 180, "full_question": "Mujer de 26 a\u00f1os diagnosticada de lupus eritematoso sist\u00e9mico, en tratamiento con hidroxicloroquina, que consulta por sensaci\u00f3n de debilidad generalizada que se ha instaurado progresivamente en los \u00faltimos 15 d\u00edas. En la exploraci\u00f3n f\u00edsica se aprecia palidez cut\u00e1nea y en la anal\u00edtica destaca Hb 7,4 gr/dL, Hcto 31 %,VCM 108. \u00bfCu\u00e1l de las siguientes determinaciones ser\u00e1 m\u00e1s \u00fatil para decidir la actuaci\u00f3n?:", "full_answer": "Dan a conocer de una paciente con LES en tratamiento con Hidroxicloroquina. Cuenta con hb en 7,4 y un VCM m\u00e1s de 100, a\u00f1adiendo la palidez y debilidad, estar\u00edamos frente a una anemia macrocitica. En mi opini\u00f3n, la m\u00e1s id\u00f3nea seria la alteranativa 2, ya que al estar recibiendo tratamiento farmacol\u00f3gico, se ha reportado que la hidroxiloroquina disminuye la absorci\u00f3n del \u00e1cido f\u00f3lico.Al tener una complicaci\u00f3n de LES, en este caso, podr\u00edamos estar frente a una Anemia hemol\u00edtica autoinmune con consumo de \u00e1cido f\u00f3lico. Y para ello ser\u00eda primero determinar con el coombs directo.", "type": "HEMATOLOGIA", "options": {"1": "Haptoglobina.", "2": "Test de Coombs.", "3": "Vitamina B12.", "4": "Anticuerpos antinucleares.", "5": NaN}, "correct_option": 2, "explanations": {"1": {"exist": false, "char_ranges": [], "word_ranges": [], "text": ""}, "2": {"exist": true, "char_ranges": [[196, 578]], "word_ranges": [[34, 94]], "text": "En mi opini\u00f3n, la m\u00e1s id\u00f3nea seria la alteranativa 2, ya que al estar recibiendo tratamiento farmacol\u00f3gico, se ha reportado que la hidroxiloroquina disminuye la absorci\u00f3n del \u00e1cido f\u00f3lico.Al tener una complicaci\u00f3n de LES, en este caso, podr\u00edamos estar frente a una Anemia hemol\u00edtica autoinmune con consumo de \u00e1cido f\u00f3lico. Y para ello ser\u00eda primero determinar con el coombs directo."}, "3": {"exist": false, "char_ranges": [], "word_ranges": [], "text": ""}, "4": {"exist": false, "char_ranges": [], "word_ranges": [], "text": ""}, "5": {"exist": false, "char_ranges": [], "word_ranges": [], "text": ""}}} {"id": 358, "year": 2016, "question_id_specific": 232, "full_question": "Una mujer de 24 a\u00f1os de edad consulta al haber apreciado adenopat\u00edas inguinales. En el interrogatorio no se recoge la presencia de ninguna molestia local ni datos sugerentes de infecci\u00f3n de transmisi\u00f3n sexual. En la exploraci\u00f3n se aprecian dos adenopat\u00edas, una en cada ingle, de 1 cm de di\u00e1metro mayor, blandas, m\u00f3viles, no dolorosas. No se aprecia ninguna lesi\u00f3n cut\u00e1nea en miembros inferiores, ano \u00f3 perin\u00e9. \u00bfQu\u00e9 prueba considera imprescindible?", "full_answer": "La respuesta correcta es la 3. En la regi\u00f3n inguinal, los ganglios linf\u00e1ticos pueden adquirir un tama\u00f1o de hasta 1,5 cm siendo estos normales. Adem\u00e1s, el resto de datos de la exploraci\u00f3n inguinal (no dolor, m\u00f3viles, blandos, etc) as\u00ed lo corrobora. Por otro lado, no hay signos ni s\u00edntomas que nos hagan pensar en una infecci\u00f3n de transmisi\u00f3n sexual.", "type": "GINECOLOG\u00cdA Y OBSTETRICIA", "options": {"1": "Una serolog\u00eda de l\u00faes puesto que lo m\u00e1s probable es que se trate de una infecci\u00f3n por Treponema pallidum.", "2": "Una exploraci\u00f3n ginecol\u00f3gica a fin de descartar un c\u00e1ncer de ovario.", "3": "Por las caracter\u00edsticas cl\u00ednicas parece tratarse de unos ganglios normales y no deben hacerse exploraciones complementarias.", "4": "Debe realizarse una prueba de Paul-Bunell, a fin de descartar una mononucleosis infecciosa.", "5": NaN}, "correct_option": 3, "explanations": {"1": {"exist": true, "char_ranges": [[248, 349]], "word_ranges": [[41, 59]], "text": "Por otro lado, no hay signos ni s\u00edntomas que nos hagan pensar en una infecci\u00f3n de transmisi\u00f3n sexual."}, "2": {"exist": false, "char_ranges": [], "word_ranges": [], "text": ""}, "3": {"exist": true, "char_ranges": [[31, 247]], "word_ranges": [[6, 41]], "text": "En la regi\u00f3n inguinal, los ganglios linf\u00e1ticos pueden adquirir un tama\u00f1o de hasta 1,5 cm siendo estos normales. Adem\u00e1s, el resto de datos de la exploraci\u00f3n inguinal (no dolor, m\u00f3viles, blandos, etc) as\u00ed lo corrobora."}, "4": {"exist": false, "char_ranges": [], "word_ranges": [], "text": ""}, "5": {"exist": false, "char_ranges": [], "word_ranges": [], "text": ""}}} {"id": 414, "year": 2018, "question_id_specific": 71, "full_question": "Hombre de 84 de edad, con insuficiencia cardiaca cr\u00f3nica en grado funcional II secundario a cardiopatia isqu\u00e9mica con dlsfunci\u00f3n sist\u00f3lica grave no revascularizable, enfermedad renal cr\u00f3nica estadio 3 (filtrado glomerular 45-50 mL/min) y fibrilaci\u00f3n auricular permanente con frecuencia cardiaca >80 latidos por minuto. \u00bfCu\u00e1l de los siguientes farmacos NO aporta beneficio al paciente seg\u00fan la evidencia disponible en la actualidad?", "full_answer": "La ivabradina, de hecho, est\u00e1 contraindicada en pacientes con fibrilaci\u00f3n auricular. Inicialmente (aunque recientemente han aparecido evidencias que matizan esa informaci\u00f3n) se dijo que la ivabradina era selectora sobre la corriente de sodio If en el nodo sinusal. Si el paciente est\u00e1 en FA, no tiene ning\u00fan sentido actuar sobre el nodo sinusal.", "type": "CARDIOLOG\u00cdA Y CIRUG\u00cdA CARDIOVASCULAR", "options": {"1": "Ivabradina.", "2": "Inhibidores de la enzima convertidora de angiotensina.", "3": "Antialdoster\u00f3nicos.", "4": "Betabloqueantes.", "5": NaN}, "correct_option": 1, "explanations": {"1": {"exist": true, "char_ranges": [[0, 84]], "word_ranges": [[0, 11]], "text": "La ivabradina, de hecho, est\u00e1 contraindicada en pacientes con fibrilaci\u00f3n auricular."}, "2": {"exist": false, "char_ranges": [], "word_ranges": [], "text": ""}, "3": {"exist": false, "char_ranges": [], "word_ranges": [], "text": ""}, "4": {"exist": false, "char_ranges": [], "word_ranges": [], "text": ""}, "5": {"exist": false, "char_ranges": [], "word_ranges": [], "text": ""}}} {"id": 131, "year": 2012, "question_id_specific": 221, "full_question": "Un individuo presenta \u201cdebilidad muscular\u201d (hemiparesia esp\u00e1stica) de ambas extremidades derechas, con hiperreflexia y signo de Babinski, junto a una \u201cpar\u00e1lisis fl\u00e1ccida facial\u201d de la hemicara izquierda, con incapacidad para cerrar el ojo izquierdo o de retraer el lado izquierdo de la boca, adem\u00e1s de otras alteraciones. Por los datos descritos se trata de una alteraci\u00f3n que afecta, entre otros elementos, a los fasc\u00edculos motores: c\u00f3rtico-espinal y c\u00f3rtico-nuclear, pero \u00bfa qu\u00e9 nivel del neuroeje localizar\u00eda la lesi\u00f3n?", "full_answer": "Pregunta siempre liante del tronco del enc\u00e9falo. Tenemos una hemiparesia derecha y afectaci\u00f3n del n\u00facleo del facial izquierdo, ya que tiene afectado el facial superior y el inferior; por lo tanto tenemos una cl\u00ednica cruzada, teniendo que pensar en afectaci\u00f3n del tronco del enc\u00e9falo. Si tenemos en un esquema r\u00e1pido de d\u00f3nde parten los n\u00facleos de los pares craneales: los 4 primeros por encima de la protuberancia, los 4 siguientes en la protuberancia, y los 4 \u00faltimos por debajo de la protuberancia, tenemos que el n\u00facleo facial est\u00e1 en los 4 del medio. O sea n\u00facleo facial izquierdo, protuberancia izquierda, as\u00ed que respuesta 4.", "type": "NEUROLOG\u00cdA Y NEUROCIRUG\u00cdA", "options": {"1": "A nivel del \u00e1rea 4 de Brodmann de la corteza cerebral del lado derecho.", "2": "En la c\u00e1psula interna, brazo posterior del lado derecho.", "3": "En el ped\u00fanculo cerebral izquierdo.", "4": "En la porci\u00f3n medial de la protuberancia caudal del lado izquierdo.", "5": "En el bulbo raqu\u00eddeo antes de la decusaci\u00f3n del fasc\u00edculo c\u00f3rtico-espinal derecho."}, "correct_option": 4, "explanations": {"1": {"exist": false, "char_ranges": [], "word_ranges": [], "text": ""}, "2": {"exist": false, "char_ranges": [], "word_ranges": [], "text": ""}, "3": {"exist": false, "char_ranges": [], "word_ranges": [], "text": ""}, "4": {"exist": true, "char_ranges": [[196, 631]], "word_ranges": [[31, 104]], "text": "tenemos una cl\u00ednica cruzada, teniendo que pensar en afectaci\u00f3n del tronco del enc\u00e9falo. Si tenemos en un esquema r\u00e1pido de d\u00f3nde parten los n\u00facleos de los pares craneales: los 4 primeros por encima de la protuberancia, los 4 siguientes en la protuberancia, y los 4 \u00faltimos por debajo de la protuberancia, tenemos que el n\u00facleo facial est\u00e1 en los 4 del medio. O sea n\u00facleo facial izquierdo, protuberancia izquierda, as\u00ed que respuesta 4."}, "5": {"exist": false, "char_ranges": [], "word_ranges": [], "text": ""}}} {"id": 70, "year": 2012, "question_id_specific": 127, "full_question": "Una mujer de 30 a\u00f1os con AP de drogadicci\u00f3n en periodo actual de desintoxicaci\u00f3n s tra\u00edda a Urgencias con cuadro de somnolencia profunda, sin otra sintomatolog\u00eda. Logramos averiguar que en su tratamiento se incluye olanzapina y benzodiacepinas. \u00bfQu\u00e9 debemos hacer en primer lugar?", "full_answer": "Se debe sospechar una sobreuso de los f\u00e1rmacos que toma. La olanzapina no tiene ant\u00eddoto, por lo que,en principio, deber\u00edamos antagonizar el efecto de las benzodiacepinas. Si la paciente contin\u00faa con somnolencia y no responde a est\u00edmulos, debemos proceder a intubar para asegurar la v\u00eda a\u00e9rea.", "type": "ANESTESIOLOG\u00cdA, CUIDADOS CR\u00cdTICOS Y URGENCIAS", "options": {"1": "Proceder a IOT.", "2": "Realizar una Rx de cr\u00e1neo.", "3": "Administrar flumazenilo.", "4": "Administrar naloxona.", "5": "Esperar a que despierte."}, "correct_option": 3, "explanations": {"1": {"exist": false, "char_ranges": [], "word_ranges": [], "text": ""}, "2": {"exist": false, "char_ranges": [], "word_ranges": [], "text": ""}, "3": {"exist": true, "char_ranges": [[0, 171]], "word_ranges": [[0, 26]], "text": "Se debe sospechar una sobreuso de los f\u00e1rmacos que toma. La olanzapina no tiene ant\u00eddoto, por lo que,en principio, deber\u00edamos antagonizar el efecto de las benzodiacepinas."}, "4": {"exist": false, "char_ranges": [], "word_ranges": [], "text": ""}, "5": {"exist": false, "char_ranges": [], "word_ranges": [], "text": ""}}} {"id": 463, "year": 2019, "question_id_specific": 71, "full_question": "Mujer con hipotensi\u00f3n e hipoperfusi\u00f3n tisular en la que se objetivan los siguientes par\u00e1metros tras la inserci\u00f3n de un cat\u00e9ter de Swan-Ganz: \u00edndice cardiaco 1,4 L/min/m2, presi\u00f3n de enclavamiento capilar pulmonar 25 mmHg y resistencias vasculares sist\u00e9micas 2000 dinas/m2. El tipo de shock que presenta la paciente es:", "full_answer": "No nos dan m\u00e1s datos de la paciente salvo que se encuentra en situaci\u00f3n de shock (hipoperfusi\u00f3n tisular) y al introducir un cat\u00e9ter de Swan-Ganz nos encontramos un \u00edndice card\u00edaco bajo (<2), una PCP elevada (25mmHg) y unas resistencias sist\u00e9micas elevadas (normal: 600-1200 dinas/m2). Estos datos son compatibles con origen cardiog\u00e9nico del shock.", "type": "CUIDADOS CR\u00cdTICOS Y URGENCIAS", "options": {"1": "Cardiog\u00e9nico.", "2": "S\u00e9ptico.", "3": "Hipovol\u00e9mico.", "4": "Anaf\u00edl\u00e1ctico.", "5": NaN}, "correct_option": 1, "explanations": {"1": {"exist": true, "char_ranges": [[0, 347]], "word_ranges": [[0, 53]], "text": "No nos dan m\u00e1s datos de la paciente salvo que se encuentra en situaci\u00f3n de shock (hipoperfusi\u00f3n tisular) y al introducir un cat\u00e9ter de Swan-Ganz nos encontramos un \u00edndice card\u00edaco bajo (<2), una PCP elevada (25mmHg) y unas resistencias sist\u00e9micas elevadas (normal: 600-1200 dinas/m2). Estos datos son compatibles con origen cardiog\u00e9nico del shock."}, "2": {"exist": false, "char_ranges": [], "word_ranges": [], "text": ""}, "3": {"exist": false, "char_ranges": [], "word_ranges": [], "text": ""}, "4": {"exist": false, "char_ranges": [], "word_ranges": [], "text": ""}, "5": {"exist": false, "char_ranges": [], "word_ranges": [], "text": ""}}} {"id": 476, "year": 2020, "question_id_specific": 93, "full_question": "Un hombre de 58 a\u00f1os, hipertenso, acude a urgencias por un s\u00edncope de 30 segundos mientras corre, con recuperaci\u00f3n espont\u00e1nea y sin secuelas. La tensi\u00f3n arterial es 135/65 mmHg. En la auscultaci\u00f3n cardiaca presenta un soplo sist\u00f3lico rudo, intenso, que se reduce con maniobras de Valsalva y un segundo tono atenuado. El ECG muestra un ritmo sinusal a 72 lpm, con criterios de hipertrofia ventricular izquierda y ondas T invertidas en cara anterior. Se\u00f1ale la afirmaci\u00f3n correcta:", "full_answer": "Los datos aportados sobre las caracter\u00edsticas del soplo (DISMINUYE con el Valsalva) y los datos de hipertrofia ventricular izquierda, nos orientan al diagn\u00f3stico de estenosis a\u00f3rtica grave.", "type": "CUIDADOS CR\u00cdTICOS", "options": {"1": "El cuadro cl\u00ednico sugiere un tromboembolismo pulmonar.", "2": "Los datos aportados indican miocardiopat\u00eda hipertr\u00f3fica con obstrucci\u00f3n grave del tracto de salida de ventr\u00edculo izquierdo.", "3": "Estos datos corresponden a estenosis a\u00f3rtica grave.", "4": "Debe excluirse una disecci\u00f3n de aorta mediante una tomograf\u00eda computarizada con contraste.", "5": NaN}, "correct_option": 3, "explanations": {"1": {"exist": false, "char_ranges": [], "word_ranges": [], "text": ""}, "2": {"exist": false, "char_ranges": [], "word_ranges": [], "text": ""}, "3": {"exist": true, "char_ranges": [[0, 189]], "word_ranges": [[0, 27]], "text": "Los datos aportados sobre las caracter\u00edsticas del soplo (DISMINUYE con el Valsalva) y los datos de hipertrofia ventricular izquierda, nos orientan al diagn\u00f3stico de estenosis a\u00f3rtica grave."}, "4": {"exist": false, "char_ranges": [], "word_ranges": [], "text": ""}, "5": {"exist": false, "char_ranges": [], "word_ranges": [], "text": ""}}} {"id": 546, "year": 2022, "question_id_specific": 124, "full_question": "Mujer de 82 a\u00f1os con fibrilaci\u00f3n auricular cr\u00f3nica de 10 a\u00f1os de evoluci\u00f3n y funci\u00f3n sist\u00f3lica biventricular conservada que presenta episodios de bloqueo aur\u00edculo-ventricular completo sintom\u00e1ticos, por lo que se decide implantar un sistema de estimulaci\u00f3n cardiaco definitivo. \u00bfCu\u00e1l de los siguientes est\u00e1 indicado?:", "full_answer": "Marcapasos VVI (unicameral ventricular).", "type": "CARDIOLOG\u00cdA", "options": {"1": "Marcapasos DDD (bicameral).", "2": "Marcapasos VVI (unicameral ventricular).", "3": "Marcapasos AAI (unicameral auricular).", "4": "Terapia de resincronizaci\u00f3n ventricular (TRC).", "5": NaN}, "correct_option": 2, "explanations": {"1": {"exist": false, "char_ranges": [], "word_ranges": [], "text": ""}, "2": {"exist": true, "char_ranges": [[0, 40]], "word_ranges": [[0, 4]], "text": "Marcapasos VVI (unicameral ventricular)."}, "3": {"exist": false, "char_ranges": [], "word_ranges": [], "text": ""}, "4": {"exist": false, "char_ranges": [], "word_ranges": [], "text": ""}, "5": {"exist": false, "char_ranges": [], "word_ranges": [], "text": ""}}} {"id": 313, "year": 2016, "question_id_specific": 177, "full_question": "Paciente de 57 a\u00f1os, diagnosticado de adenocarcinoma de colon estadio T3N1M0. Fue tratado con resecci\u00f3n seguida de quimioterapia postoperatoria FOLFOX (fluorouracilo, leucovorin y oxaliplatino) durante 6 meses. En un control rutinario, dos a\u00f1os despu\u00e9s, se detecta elevaci\u00f3n de ant\u00edgeno carcinoembrionario (CEA) a 30 ng/mL (previo menor de 2 ng/mL). No tiene s\u00edntomas, el examen f\u00edsico es anodino, la radiograf\u00eda de t\u00f3rax es normal. En la TC se aprecia una masa de 3 cm en el l\u00f3bulo hep\u00e1tico derecho que capta en PET. No se observan otras alteraciones en TC ni en PET. \u00bfQu\u00e9 actitud le parece m\u00e1s correcta?", "full_answer": "Nos encontramos ante una situaci\u00f3n de probable met\u00e1stasis hep\u00e1tica de un adenocarcinoma de colon localizado tratado hace 2 a\u00f1os. Probable porque no tenemos confirmaci\u00f3n histol\u00f3gica, aunque como suele suceder \u00bfqu\u00e9 otra cosa va a ser? Como se trata de una situaci\u00f3n de oligomet\u00e1stasis y los tratamientos para adenocarcinoma de colon avanzado permiten una superviviencia prolongada y con buena calidad de vida en estos pacientes, si no existe nada que contraindique la cirug\u00eda, esa debe ser la primera opci\u00f3n (respuesta 3 correcta). Adjunto el algoritmo que propone la NCCN en situaciones de recurrencia (http://www.nccn.org/professionals/physician_gls/pdf/colon.pdf). En caso de ser una situaci\u00f3n irresecable, se plantear\u00eda un tratamiento de quimioterapia con intenci\u00f3n de convertir la lesi\u00f3n metast\u00e1sica \u00fanica en resecable. Dado que han pasado m\u00e1s de 12 meses desde el tratamiento con FOLFOX, se podr\u00eda usar el mismo esquema, auqnue FOLFIRI+bevacizumab tambi\u00e9n ser\u00eda \u00fatil. Capecitabina en monoterapia, en la situaci\u00f3n actual del paciente tampoco ser\u00eda una buena opci\u00f3n de inicio, ya que las respuestas son menores en comparaci\u00f3n con las combinaciones FOLFOX o FOLFIRI m\u00e1s antiangiog\u00e9nicos. La radioterapia hep\u00e1tica podr\u00eda valorarse ya que existen t\u00e9cnicas de SBRT que consiguen muy buenos resultados pero en este caso lo mejor es que empecemos pidiendo valoraci\u00f3n por los cirujanos y si descartaran la cirug\u00eda ya plantear\u00edamos la RT.", "type": "ONCOLOG\u00cdA", "options": {"1": "Quimioterapia con FOLFIRI (fluorouracilo, leucovorin e irinotecan) m\u00e1s bevacizumab.", "2": "Monoquimioterapia con capecitabina.", "3": "Valoraci\u00f3n por cirug\u00eda de resecci\u00f3n de la lesi\u00f3n hep\u00e1tica.", "4": "Radioterapia hep\u00e1tica.", "5": NaN}, "correct_option": 3, "explanations": {"1": {"exist": true, "char_ranges": [[666, 971]], "word_ranges": [[93, 140]], "text": "En caso de ser una situaci\u00f3n irresecable, se plantear\u00eda un tratamiento de quimioterapia con intenci\u00f3n de convertir la lesi\u00f3n metast\u00e1sica \u00fanica en resecable. Dado que han pasado m\u00e1s de 12 meses desde el tratamiento con FOLFOX, se podr\u00eda usar el mismo esquema, auqnue FOLFIRI+bevacizumab tambi\u00e9n ser\u00eda \u00fatil."}, "2": {"exist": true, "char_ranges": [[972, 1188]], "word_ranges": [[140, 172]], "text": "Capecitabina en monoterapia, en la situaci\u00f3n actual del paciente tampoco ser\u00eda una buena opci\u00f3n de inicio, ya que las respuestas son menores en comparaci\u00f3n con las combinaciones FOLFOX o FOLFIRI m\u00e1s antiangiog\u00e9nicos."}, "3": {"exist": true, "char_ranges": [[233, 529]], "word_ranges": [[35, 81]], "text": "Como se trata de una situaci\u00f3n de oligomet\u00e1stasis y los tratamientos para adenocarcinoma de colon avanzado permiten una superviviencia prolongada y con buena calidad de vida en estos pacientes, si no existe nada que contraindique la cirug\u00eda, esa debe ser la primera opci\u00f3n (respuesta 3 correcta)."}, "4": {"exist": true, "char_ranges": [[1189, 1432]], "word_ranges": [[172, 211]], "text": "La radioterapia hep\u00e1tica podr\u00eda valorarse ya que existen t\u00e9cnicas de SBRT que consiguen muy buenos resultados pero en este caso lo mejor es que empecemos pidiendo valoraci\u00f3n por los cirujanos y si descartaran la cirug\u00eda ya plantear\u00edamos la RT."}, "5": {"exist": false, "char_ranges": [], "word_ranges": [], "text": ""}}} {"id": 482, "year": 2020, "question_id_specific": 173, "full_question": "Un paciente es tra\u00eddo a urgencias desde un incendio producido dentro de una nave cerrada donde hab\u00eda espumas de poliuretano. Est\u00e1 consciente, pero presenta creciente torpeza mental, cefalea e intensa disnea. La saturaci\u00f3n de ox\u00edgeno mediante pulsoximetr\u00eda es del 92% y el \u00e1cido l\u00e1ctico capilar es de 8 mEq/l. \u00bfQu\u00e9 tratamiento espec\u00edfico considera m\u00e1s adecuado?:", "full_answer": "Lo primero que tenemos que saber es que la combusti\u00f3n del poliuretano a alta temperatura produce la liberaci\u00f3n de cianuro (muy intuitivo, s\u00ed). Sabiendo esto, y que nos enfrentamos a una intoxicaci\u00f3n por cianuro, es m\u00e1s f\u00e1cil responder la pregunta. Entre las medidas b\u00e1sicas de tratamiento inicial estar\u00eda la administraci\u00f3n de oxigenoterapia con FiO2 1 (no 0.5 como se plantea en la opci\u00f3n 1), y un ant\u00eddoto, en este caso nos plantean la hidroxicobalamina (esto se pregunt\u00f3 de forma similar el a\u00f1o pasado). La ventilaci\u00f3n con c\u00e1mara hiperb\u00e1rica s\u00f3lo estar\u00eda indicada en caso de que coexistiera la intoxicaci\u00f3n por mon\u00f3xido de carbono (CO). Lo que se plantea en la opci\u00f3n 4 acerca de la fluidoterapia no est\u00e1 indicado; en caso de shock asociado se administrar\u00eda volumen, pero en cualquier caso no a esa velocidad, sino en bolus r\u00e1pidos.", "type": "CUIDADOS CR\u00cdTICOS", "options": {"1": "Administraci\u00f3n de ox\u00edgeno mediante mascarilla tipo Venturi al 50%.", "2": "Administraci\u00f3n de hidroxicobalamina intravenosa.", "3": "Ventilaci\u00f3n con c\u00e1mara hiperb\u00e1rica.", "4": "Fluidoterapia con infusi\u00f3n de soluci\u00f3n salina fisiol\u00f3gica a 21 ml/h.", "5": NaN}, "correct_option": 2, "explanations": {"1": {"exist": true, "char_ranges": [[248, 391]], "word_ranges": [[40, 64]], "text": "Entre las medidas b\u00e1sicas de tratamiento inicial estar\u00eda la administraci\u00f3n de oxigenoterapia con FiO2 1 (no 0.5 como se plantea en la opci\u00f3n 1),"}, "2": {"exist": true, "char_ranges": [[248, 454]], "word_ranges": [[40, 74]], "text": "Entre las medidas b\u00e1sicas de tratamiento inicial estar\u00eda la administraci\u00f3n de oxigenoterapia con FiO2 1 (no 0.5 como se plantea en la opci\u00f3n 1), y un ant\u00eddoto, en este caso nos plantean la hidroxicobalamina"}, "3": {"exist": true, "char_ranges": [[506, 638]], "word_ranges": [[83, 103]], "text": "La ventilaci\u00f3n con c\u00e1mara hiperb\u00e1rica s\u00f3lo estar\u00eda indicada en caso de que coexistiera la intoxicaci\u00f3n por mon\u00f3xido de carbono (CO)."}, "4": {"exist": true, "char_ranges": [[639, 834]], "word_ranges": [[103, 138]], "text": "Lo que se plantea en la opci\u00f3n 4 acerca de la fluidoterapia no est\u00e1 indicado; en caso de shock asociado se administrar\u00eda volumen, pero en cualquier caso no a esa velocidad, sino en bolus r\u00e1pidos."}, "5": {"exist": false, "char_ranges": [], "word_ranges": [], "text": ""}}} {"id": 105, "year": 2012, "question_id_specific": 233, "full_question": "En una paciente de 35 a\u00f1os con un s\u00edndrome depresivo en tratamiento con inhibidores de la recaptaci\u00f3n de la serotonina est\u00e1 contraindicado el empleo de los siguientes antimicrobianos:", "full_answer": "Se han comunicado notificaciones espont\u00e1neas de s\u00edndrome serotonin\u00e9rgico asociadas a la administraci\u00f3n concomitante de linezolid y agentes serotonin\u00e9rgicos, incluidos antidepresivos como los inhibidores selectivos de la recaptaci\u00f3n de serotonina (ISRSs).", "type": "FARMACOLOG\u00cdA", "options": {"1": "Doxiciclina.", "2": "Amoxicilina-Clavul\u00e1nico.", "3": "Daptomicina.", "4": "Linezolid.", "5": "Vancomicina."}, "correct_option": 4, "explanations": {"1": {"exist": false, "char_ranges": [], "word_ranges": [], "text": ""}, "2": {"exist": false, "char_ranges": [], "word_ranges": [], "text": ""}, "3": {"exist": false, "char_ranges": [], "word_ranges": [], "text": ""}, "4": {"exist": true, "char_ranges": [[0, 254]], "word_ranges": [[0, 30]], "text": "Se han comunicado notificaciones espont\u00e1neas de s\u00edndrome serotonin\u00e9rgico asociadas a la administraci\u00f3n concomitante de linezolid y agentes serotonin\u00e9rgicos, incluidos antidepresivos como los inhibidores selectivos de la recaptaci\u00f3n de serotonina (ISRSs)."}, "5": {"exist": false, "char_ranges": [], "word_ranges": [], "text": ""}}} {"id": 73, "year": 2012, "question_id_specific": 62, "full_question": "Un hombre de 37 a\u00f1os sin antecedentes de inter\u00e9s acude a urgencias por haber sufrido un s\u00edncope mientras caminaba. Hab\u00eda comenzado dos d\u00edas antes con disnea de esfuerzo. Exploraci\u00f3n: peso 75 Kg, TA 75/50 mm Hg. SatO2 89%. Auscultaci\u00f3n pulmonar normal. Auscultaci\u00f3n cardiaca: tonos r\u00edtmicos taquic\u00e1rdicos a 130 lpm. Abdomen normal. Extremidades: sin alteraciones. Hemograma normal. Troponina 1,2 ng/ml. ECG: Taquicardia sensual a 130 lpm. T invertida en V1-V4. Angio TC tor\u00e1cico: defecto de repleci\u00f3n en ambas arterias pulmonares principales. Una hora despu\u00e9s de su llegada a urgencias persiste TA 70/55 mm Hg. Cual ser\u00eda el tratamiento inicial mas adecuado?", "full_answer": "El caso cl\u00ednico es un Tromboembolismo Pulmonar t\u00edpico. En este caso asociado a hipotensi\u00f3n arterial persistente. La opci\u00f3n 2 ser\u00eda correcta si el paciente no estuviese hipotenso. La opci\u00f3n 3 no es correcta dado que no es necesaria confirmaci\u00f3n diagn\u00f3stica para iniciar el tratamiento anticoagulante en caso de TEP sin hipotensi\u00f3n. La opci\u00f3n 4 no est\u00e1 indicada. La opci\u00f3n 5 ser\u00eda correcta solo si el paciente tuviese contraindicaci\u00f3n de fibrinolisis. La hipotensi\u00f3n persistente en el TEP es la indicaci\u00f3n mas ampliamente aceptada de fibrinolisis, por lo que la opic\u00f3n correcta es la 1.", "type": "ANESTESIOLOG\u00cdA, CUIDADOS CR\u00cdTICOS Y URGENCIAS", "options": {"1": "Heparina no fraccionada, 10.000U iv ante la sospecha cl\u00ednica. Fibrinolisis con tPA 100 mg iv una vez confirmado el diagn\u00f3stico.", "2": "Enoxaparina: 80 mg sc cada 12 horas, comenzando ante la sospecha diagn\u00f3stica.", "3": "Enoxaparina; 80 mg sc cada 12 horas, comenzando ante la confirmaci\u00f3n diagn\u00f3stica.", "4": "Fondaparinux: 7,5 mg sc diarios.", "5": "Tromboendarterectom\u00eda pulmonar de urgencia."}, "correct_option": 1, "explanations": {"1": {"exist": true, "char_ranges": [[450, 584]], "word_ranges": [[70, 93]], "text": "La hipotensi\u00f3n persistente en el TEP es la indicaci\u00f3n mas ampliamente aceptada de fibrinolisis, por lo que la opic\u00f3n correcta es la 1."}, "2": {"exist": true, "char_ranges": [[113, 178]], "word_ranges": [[16, 27]], "text": "La opci\u00f3n 2 ser\u00eda correcta si el paciente no estuviese hipotenso."}, "3": {"exist": true, "char_ranges": [[179, 330]], "word_ranges": [[27, 51]], "text": "La opci\u00f3n 3 no es correcta dado que no es necesaria confirmaci\u00f3n diagn\u00f3stica para iniciar el tratamiento anticoagulante en caso de TEP sin hipotensi\u00f3n."}, "4": {"exist": true, "char_ranges": [[331, 360]], "word_ranges": [[51, 57]], "text": "La opci\u00f3n 4 no est\u00e1 indicada."}, "5": {"exist": true, "char_ranges": [[361, 449]], "word_ranges": [[57, 70]], "text": "La opci\u00f3n 5 ser\u00eda correcta solo si el paciente tuviese contraindicaci\u00f3n de fibrinolisis."}}} {"id": 538, "year": 2021, "question_id_specific": 2, "full_question": "Una ca\u00edda con fractura-luxaci\u00f3n anterior de hombro que fue reducida en urgencias. Le fue colocado un cabestrillo y se recomend\u00f3 su retirada a las 3 semanas. Al retirar la inmovilizaci\u00f3n se observa una zona de disestesia circunscrita a la regi\u00f3n lateral del hombro. La paciente puede realizar abducci\u00f3n, pero solo alcanza 15 grados. Dispone de un estudio de resonancia magn\u00e9tica en el que el manguito de los rotadores est\u00e1 indemne. \u00bfCu\u00e1l es el diagn\u00f3stico de sospecha m\u00e1s probable?:", "full_answer": "La disestesia de la cara lateral de la regi\u00f3n deltoidea del hombro (zona inervada por el nervio axilar) y la par\u00e1lisis del deltoides (m\u00fasculo primario para la abducci\u00f3n m\u00e1s all\u00e1 de 15\u00ba). El manguito de rotadores est\u00e1 indemne en la resonancia magn\u00e9tica, descartando la opci\u00f3n 1 (nos mostrar\u00eda la tendinopat\u00eda del redondo mayor). La capsulitis adhesiva cursa con dolor y rigidez del hombro, sobre todo a la rotaci\u00f3n externa tanto activa como pasiva. El nervio supraescapular inercia el m\u00fasculo supraespinoso, que contribuye a los primeros 15\u00ba de abducci\u00f3n del hombro (conservados en el enunciado) y no inerva la piel de la zona reflejada.", "type": "TRAUMATOLOG\u00cdA", "options": {"1": "Tendinopat\u00eda del redondo mayor.", "2": "Capsulitis adhesiva.", "3": "Neuropat\u00eda del nervio supraescapular.", "4": "Lesi\u00f3n del nervio axilar.", "5": NaN}, "correct_option": 4, "explanations": {"1": {"exist": true, "char_ranges": [[187, 327]], "word_ranges": [[32, 53]], "text": "El manguito de rotadores est\u00e1 indemne en la resonancia magn\u00e9tica, descartando la opci\u00f3n 1 (nos mostrar\u00eda la tendinopat\u00eda del redondo mayor)."}, "2": {"exist": true, "char_ranges": [[328, 447]], "word_ranges": [[53, 73]], "text": "La capsulitis adhesiva cursa con dolor y rigidez del hombro, sobre todo a la rotaci\u00f3n externa tanto activa como pasiva."}, "3": {"exist": true, "char_ranges": [[448, 636]], "word_ranges": [[73, 103]], "text": "El nervio supraescapular inercia el m\u00fasculo supraespinoso, que contribuye a los primeros 15\u00ba de abducci\u00f3n del hombro (conservados en el enunciado) y no inerva la piel de la zona reflejada."}, "4": {"exist": true, "char_ranges": [[0, 186]], "word_ranges": [[0, 32]], "text": "La disestesia de la cara lateral de la regi\u00f3n deltoidea del hombro (zona inervada por el nervio axilar) y la par\u00e1lisis del deltoides (m\u00fasculo primario para la abducci\u00f3n m\u00e1s all\u00e1 de 15\u00ba)."}, "5": {"exist": false, "char_ranges": [], "word_ranges": [], "text": ""}}} {"id": 139, "year": 2012, "question_id_specific": 166, "full_question": "Paciente de 64 a\u00f1os fumador y bebedor importante sin otros antecedentes de inter\u00e9s. El cuadro que explica es de odinodisfagia de 3 meses de evoluci\u00f3n y otalgia izquierda. Aporta informe del odont\u00f3logo que descarta causa de origen dental. Tambi\u00e9n refiere haber sido visitado por diferentes especialistas aportando un TAC de cabeza y cuello (sin contraste yodado) que es informado como normal. A la exploraci\u00f3n destaca una disminuci\u00f3n en la capacidad de propulsi\u00f3n de la lengua; la palpaci\u00f3n de la base de lengua izquierda objetiva una induraci\u00f3n de consistencia p\u00e9trea de aprox. 3cm y con laringoscopia indirecta no se oberva ninguna ulceraci\u00f3n de la mucosa. La palpaci\u00f3n cervical es negativa para adenopat\u00edas. \u00bfCu\u00e1l es su diagn\u00f3tico de presunci\u00f3n y conducta?", "full_answer": "Nos presentan un paciente con factores de riesgo para c\u00e1ncer de orofaringe, pero en el que la exploraci\u00f3n y las pruebas son normales. \u00bfQu\u00e9 pasa aqu\u00ed? Esta pregunta es de psicolog\u00eda inversa: \u00bfqu\u00e9 quer\u00eda que yo supiera el que puso la pregunta? Esta pregunta la ha hecho un otorrinolaring\u00f3logo. Ya te dice que el odont\u00f3logo \u00abno le ha visto nada\u00bb, as\u00ed que descarta la 1. Tampoco es un absceso de base de lengua, porque en tres meses le habr\u00eda matado: la 2 tampoco. As\u00ed que nos quedan anquiloglosia, c\u00e1ncer de base de lengua e inflamaci\u00f3n inespec\u00edfica. Cuando hay duda de que pueda ser un c\u00e1ncer, no hay que andarse con tonter\u00edas y pedir un nuevo TAC y una biopsia. Los c\u00e1nceres de base de lengua son muy traicioneros y es uno de los miedos de cualquier otorrinolaring\u00f3logo cuando sospecha una neoplasia. No es raro que debuten sin masas exof\u00edticas ni ulceraciones de la mucosa lingual y en la TAC son dif\u00edciles de detectar cuando no se usa contraste. La respuesta correcta es la 4 y no creo que pueda ser impugnada.", "type": "OTORRINOLARINGOLOG\u00cdA Y CIRUG\u00cdA MAXILOFACIAL", "options": {"1": "Causa dental / revaloraci\u00f3n por el odont\u00f3logo.", "2": "Absceso de la base de lengua / desbridamiento.", "3": "Anquiloglosia / liberaci\u00f3n quir\u00fargica de la misma.", "4": "Proceso maligno de orofaringe / repetir TC y biopsia.", "5": "Inflamaci\u00f3n inespec\u00edfica / tratamiento corticoideo."}, "correct_option": 4, "explanations": {"1": {"exist": true, "char_ranges": [[307, 366]], "word_ranges": [[53, 65]], "text": "el odont\u00f3logo \u00abno le ha visto nada\u00bb, as\u00ed que descarta la 1."}, "2": {"exist": true, "char_ranges": [[367, 460]], "word_ranges": [[65, 83]], "text": "Tampoco es un absceso de base de lengua, porque en tres meses le habr\u00eda matado: la 2 tampoco."}, "3": {"exist": false, "char_ranges": [], "word_ranges": [], "text": ""}, "4": {"exist": true, "char_ranges": [[548, 660]], "word_ranges": [[96, 119]], "text": "Cuando hay duda de que pueda ser un c\u00e1ncer, no hay que andarse con tonter\u00edas y pedir un nuevo TAC y una biopsia."}, "5": {"exist": false, "char_ranges": [], "word_ranges": [], "text": ""}}} {"id": 446, "year": 2018, "question_id_specific": 130, "full_question": "Hombre de 28 a\u00f1os de edad que acude a Urgencias del hospital por presentar hematuria macrosc\u00f3pica. \u00bfCu\u00e1l de las siguientes alteraciones en el an\u00e1lisis de orina apoyar\u00eda el diagn\u00f3stico de glomerulonefritis?", "full_answer": "La glomerulonefritis se asocia t\u00edpicamente con hemat\u00edes dism\u00f3rficos y cilindros hem\u00e1ticos en el sedimento (opci\u00f3n 1 correcta). Tambi\u00e9n puede aparecer proteinuria en rango variable, si bien no es habitual en forma de microalbuminuria aislada (opci\u00f3n 2 incorrecta), y hematuria con piuria aunque no es caracter\u00edstico (opci\u00f3n 3 incorrecta). Los co\u00e1gulos suelen aparecer en el contexto de alteraciones de la v\u00eda urinaria baja (opci\u00f3n 4 incorrecta).", "type": "NEFROLOG\u00cdA", "options": {"1": "Hemat\u00edes dism\u00f3rficos y/o cilindros hem\u00e1ticos.", "2": "Proteinuria de 1 g/d\u00eda, con resultado negativo en tira reactiva y con microalbuminuria mayor de 300 mg/24 horas.", "3": "Coexistencia de hematuria con piuria sin bacteriuria.", "4": "Co\u00e1gulos en la orina a simple vista.", "5": NaN}, "correct_option": 1, "explanations": {"1": {"exist": true, "char_ranges": [[0, 126]], "word_ranges": [[0, 17]], "text": "La glomerulonefritis se asocia t\u00edpicamente con hemat\u00edes dism\u00f3rficos y cilindros hem\u00e1ticos en el sedimento (opci\u00f3n 1 correcta)."}, "2": {"exist": true, "char_ranges": [[127, 262]], "word_ranges": [[17, 37]], "text": "Tambi\u00e9n puede aparecer proteinuria en rango variable, si bien no es habitual en forma de microalbuminuria aislada (opci\u00f3n 2 incorrecta),"}, "3": {"exist": true, "char_ranges": [[266, 337]], "word_ranges": [[38, 48]], "text": "hematuria con piuria aunque no es caracter\u00edstico (opci\u00f3n 3 incorrecta)."}, "4": {"exist": true, "char_ranges": [[338, 444]], "word_ranges": [[48, 65]], "text": "Los co\u00e1gulos suelen aparecer en el contexto de alteraciones de la v\u00eda urinaria baja (opci\u00f3n 4 incorrecta)."}, "5": {"exist": false, "char_ranges": [], "word_ranges": [], "text": ""}}} {"id": 551, "year": 2022, "question_id_specific": 130, "full_question": "Var\u00f3n de 85 a\u00f1os en fibrilaci\u00f3n auricular permanente que consulta por sufrir un dolor brusco y frialdad en el pie derecho desde hace 6 horas. En la exploraci\u00f3n presenta cianosis plantar con d\u00e9ficit parcial de sensibilidad y movilidad. El pulso femoral es palpable, estando ausentes los pulsos popl\u00edteo y distales de dicha extremidad. De las siguientes \u00bfcu\u00e1l es la actitud terap\u00e9utica id\u00f3nea?:", "full_answer": "Tratamiento quir\u00fargico urgente mediante tromboembolectom\u00eda.", "type": "CARDIOLOG\u00cdA", "options": {"1": "Desestimar la cirug\u00eda de revascularizaci\u00f3n, por el tiempo prolongado de isquemia.", "2": "Aplicar calor y avisar al cirujano vascular de guardia.", "3": "Tratamiento quir\u00fargico urgente mediante tromboembolectom\u00eda.", "4": "Tratamiento m\u00e9dico urgente con prostaglandinas intravenosas.", "5": NaN}, "correct_option": 3, "explanations": {"1": {"exist": false, "char_ranges": [], "word_ranges": [], "text": ""}, "2": {"exist": false, "char_ranges": [], "word_ranges": [], "text": ""}, "3": {"exist": true, "char_ranges": [[0, 59]], "word_ranges": [[0, 5]], "text": "Tratamiento quir\u00fargico urgente mediante tromboembolectom\u00eda."}, "4": {"exist": false, "char_ranges": [], "word_ranges": [], "text": ""}, "5": {"exist": false, "char_ranges": [], "word_ranges": [], "text": ""}}} {"id": 383, "year": 2016, "question_id_specific": 150, "full_question": "Ni\u00f1o de 12 meses de edad, que en los ex\u00e1menes de salud practicados desde el nacimiento presenta test\u00edculo derecho en conducto inguinal que no es posible descender hasta el escroto. Se\u00f1ale la respuesta correcta:", "full_answer": "El test\u00edculo retr\u00e1ctil se caracteriza porque el teste se encuentra permanentemente fuera de la bolsa escrotal, siendo posible descenderlo manualmente (aunque al soltarlo vuelve autom\u00e1ticamente a su ubicaci\u00f3n inicial). Se suele deber a un ped\u00edculo corto. En este caso nos est\u00e1n diciendo que el teste se encuentra siempre en canal inguinal (criptorquidia) por lo que podemos descartar la opci\u00f3n 1. El tratamiento hormonal con HCG ya ha ca\u00eddo en desuso, debido a la poca eficacia, posibilidad de reascenso y porque se han descrito fen\u00f3menos apopt\u00f3ticos e inflamatorios en el teste asociados a su uso (descartamos la opci\u00f3n 3). La orquidopexia es el tratamiento de elecci\u00f3n cuando os encontramos ante una criptorquidia, siendo deseable realizarla entre los 6 meses y el a\u00f1o, no debiendo diferirse m\u00e1s all\u00e1 de los dos a\u00f1os de edad.", "type": "PEDIATR\u00cdA", "options": {"1": "El diagn\u00f3stico m\u00e1s probable es el de test\u00edculo retr\u00e1ctil.", "2": "Se ha de esperar hasta los dos a\u00f1os de edad a que ocurra descenso espont\u00e1neo del mismo.", "3": "La gonadotropina cori\u00f3nica humana es el tratamiento de primera elecci\u00f3n.", "4": "La indicaci\u00f3n de orquidopexia no debe diferirse.", "5": NaN}, "correct_option": 4, "explanations": {"1": {"exist": true, "char_ranges": [[254, 395]], "word_ranges": [[36, 60]], "text": "En este caso nos est\u00e1n diciendo que el teste se encuentra siempre en canal inguinal (criptorquidia) por lo que podemos descartar la opci\u00f3n 1."}, "2": {"exist": false, "char_ranges": [], "word_ranges": [], "text": ""}, "3": {"exist": true, "char_ranges": [[396, 623]], "word_ranges": [[60, 98]], "text": "El tratamiento hormonal con HCG ya ha ca\u00eddo en desuso, debido a la poca eficacia, posibilidad de reascenso y porque se han descrito fen\u00f3menos apopt\u00f3ticos e inflamatorios en el teste asociados a su uso (descartamos la opci\u00f3n 3)."}, "4": {"exist": true, "char_ranges": [[624, 769]], "word_ranges": [[98, 121]], "text": "La orquidopexia es el tratamiento de elecci\u00f3n cuando os encontramos ante una criptorquidia, siendo deseable realizarla entre los 6 meses y el a\u00f1o,"}, "5": {"exist": false, "char_ranges": [], "word_ranges": [], "text": ""}}} {"id": 614, "year": 2022, "question_id_specific": 118, "full_question": "Var\u00f3n de 41 a\u00f1os que consulta por gonalgia de varios d\u00edas de evoluci\u00f3n. En la exploraci\u00f3n se realiza el test de Thessaly (dolor con los movimientos de rotaci\u00f3n interna y externa con la rodilla flexionada) que resulta positivo. \u00bfCu\u00e1l de las siguientes lesiones es m\u00e1s probable?:", "full_answer": "La maniobra de Thessaly consiste en que el paciente, mediante apoyo monopodal y con flexiones de rodilla de 5\u00ba y posteriormente de 20\u00ba realice rotaciones internas y externas de la pierna. En caso de que esto reproduzca el dolor que el paciente refiere, se considerara el test como positivo. Es un test utilizado para el diagnostico de patolog\u00eda meniscal (sensibilidad superior al 90% y una especificidad superior al 95%.) (Respuesta 1 correcta).", "type": "TRAUMATOLOG\u00cdA", "options": {"1": "Lesi\u00f3n meniscal.", "2": "Lesi\u00f3n por rotura del ligamento cruzado anterior.", "3": "Lesi\u00f3n por rotura del ligamento cruzado posterior.", "4": "Lesi\u00f3n por artropat\u00eda degenerativa.", "5": NaN}, "correct_option": 1, "explanations": {"1": {"exist": true, "char_ranges": [[0, 445]], "word_ranges": [[0, 72]], "text": "La maniobra de Thessaly consiste en que el paciente, mediante apoyo monopodal y con flexiones de rodilla de 5\u00ba y posteriormente de 20\u00ba realice rotaciones internas y externas de la pierna. En caso de que esto reproduzca el dolor que el paciente refiere, se considerara el test como positivo. Es un test utilizado para el diagnostico de patolog\u00eda meniscal (sensibilidad superior al 90% y una especificidad superior al 95%.) (Respuesta 1 correcta)."}, "2": {"exist": false, "char_ranges": [], "word_ranges": [], "text": ""}, "3": {"exist": false, "char_ranges": [], "word_ranges": [], "text": ""}, "4": {"exist": false, "char_ranges": [], "word_ranges": [], "text": ""}, "5": {"exist": false, "char_ranges": [], "word_ranges": [], "text": ""}}} {"id": 499, "year": 2020, "question_id_specific": 182, "full_question": "Hombre de 60 a\u00f1os con un cuadro de dolor abdominal y sangrado digestivo alto, en quien se encuentra una tumoraci\u00f3n abdominal relacionada con la pared g\u00e1strica. La histolog\u00eda muestra un tumor de c\u00e9lulas fusiformes con escasas mitosis, positivas para CD117. El diagn\u00f3stico m\u00e1s probable es:", "full_answer": "El tumor del estroma GI es m\u00e1s conocido como GIST, positivo CD117, ckit, PDGF, de manual.", "type": "ONCOLOG\u00cdA M\u00c9DICA", "options": {"1": "Neurofibroma de la pared g\u00e1strica.", "2": "Tumor del estroma gastrointestinal.", "3": "Leiomioma.", "4": "Sarcoma granuloc\u00edtico.", "5": NaN}, "correct_option": 2, "explanations": {"1": {"exist": false, "char_ranges": [], "word_ranges": [], "text": ""}, "2": {"exist": true, "char_ranges": [[0, 89]], "word_ranges": [[0, 16]], "text": "El tumor del estroma GI es m\u00e1s conocido como GIST, positivo CD117, ckit, PDGF, de manual."}, "3": {"exist": false, "char_ranges": [], "word_ranges": [], "text": ""}, "4": {"exist": false, "char_ranges": [], "word_ranges": [], "text": ""}, "5": {"exist": false, "char_ranges": [], "word_ranges": [], "text": ""}}} {"id": 613, "year": 2022, "question_id_specific": 118, "full_question": "Var\u00f3n de 41 a\u00f1os que consulta por gonalgia de varios d\u00edas de evoluci\u00f3n. En la exploraci\u00f3n se realiza el test de Thessaly (dolor con los movimientos de rotaci\u00f3n interna y externa con la rodilla flexionada) que resulta positivo. \u00bfCu\u00e1l de las siguientes lesiones es m\u00e1s probable?:", "full_answer": "Test de Thessaly. Test de provocaci\u00f3n realizado en sospecha de lesi\u00f3n meniscal.", "type": "TRAUMATOLOG\u00cdA", "options": {"1": "Lesi\u00f3n meniscal.", "2": "Lesi\u00f3n por rotura del ligamento cruzado anterior.", "3": "Lesi\u00f3n por rotura del ligamento cruzado posterior.", "4": "Lesi\u00f3n por artropat\u00eda degenerativa.", "5": NaN}, "correct_option": 1, "explanations": {"1": {"exist": true, "char_ranges": [[0, 79]], "word_ranges": [[0, 12]], "text": "Test de Thessaly. Test de provocaci\u00f3n realizado en sospecha de lesi\u00f3n meniscal."}, "2": {"exist": false, "char_ranges": [], "word_ranges": [], "text": ""}, "3": {"exist": false, "char_ranges": [], "word_ranges": [], "text": ""}, "4": {"exist": false, "char_ranges": [], "word_ranges": [], "text": ""}, "5": {"exist": false, "char_ranges": [], "word_ranges": [], "text": ""}}} {"id": 247, "year": 2014, "question_id_specific": 117, "full_question": "Acude a nuestra consulta un hombre de 54 a\u00f1os diagnosticado de diabetes mellitus tipo 2. Refiere fiebre de 40\u00baC con escalofr\u00edos y tiritona, artromialgias y cefalea por lo que ha iniciado tratamiento con paracetamol. El cuadro se inici\u00f3 24 h antes de su regreso a Espa\u00f1a. A los 3 d\u00edas presenta un exantema m\u00e1culo-papuloso generalizado que evoluciona a la formaci\u00f3n de petequias m\u00e1s intensas en miembros inferiores. Aporta anal\u00edtica donde destaca leucopenia con 3.200/mm3 y plaquetas 91.000/mm3 y elevaci\u00f3n grave de aminotransferaras. La gota gruesa, extensi\u00f3n de sangre perif\u00e9rica, PCR y ant\u00edgeno de malaria son negativas \u00bfCu\u00e1l es el diagnostico de sospecha m\u00e1s probable?", "full_answer": "Llama la atenci\u00f3n es esta pregunta que nos digan que nuestro paciente ha regresado a Espa\u00f1a, pero no de donde (lo cual nos podr\u00eda ayudar a hacer divagaciones sobre la posible causa). Sin embargo, la descripci\u00f3n del cuadro es muy exhaustiva asi como t\u00edpica, por lo que la pregunta es facil. Se trata de una enfermedad que produce fiebre, artralgias y un exantema generalizado con petequias en los miembros inferiores. En la anal\u00edtica destaca una leucopenia acompa\u00f1ada de plaquetopenia y elevaci\u00f3n de transaminasas. Se ha descartado la malaria como causa posible. Todos los datos sin muy sugestivos de Dengue, sobre todo la plaquetopenia y las petequias (sugestivas de fragilidad capilar), respuesta 3 correcta.", "type": "ENFERMEDADES INFECCIOSAS", "options": {"1": "Infecci\u00f3n por coronavirus.", "2": "Coriomeningitis linfocitaria.", "3": "Dengue.", "4": "Infecci\u00f3n por virus Chikungunya.", "5": "Encefalitis de Saint Louis."}, "correct_option": 3, "explanations": {"1": {"exist": false, "char_ranges": [], "word_ranges": [], "text": ""}, "2": {"exist": false, "char_ranges": [], "word_ranges": [], "text": ""}, "3": {"exist": true, "char_ranges": [[562, 686]], "word_ranges": [[91, 110]], "text": "Todos los datos sin muy sugestivos de Dengue, sobre todo la plaquetopenia y las petequias (sugestivas de fragilidad capilar),"}, "4": {"exist": false, "char_ranges": [], "word_ranges": [], "text": ""}, "5": {"exist": false, "char_ranges": [], "word_ranges": [], "text": ""}}} {"id": 88, "year": 2012, "question_id_specific": 56, "full_question": "Una mujer de 78 a\u00f1os con antecedentes de hipertensi\u00f3n arterial en tratamiento con amlodipino ingres\u00f3 hace 3 meses por debut de insuficiencia cardiaca y fibrilaci\u00f3n auricular. Acude a su consulta para revisi\u00f3n cardiol\u00f3gica. El informe de alta muestra el resultado del ecocardiograma que evidencia disfunci\u00f3n ventricular izquierda moderada y el tratamiento al alta: se suspendi\u00f3 el amlodipino y se inici\u00f3 furosemida y enalapril. La paciente refiere encontrarse mejor, sin cansancio y con menos edema de piernas, pero todav\u00eda no est\u00e1 bien del todo. A la exploraci\u00f3n f\u00edsica comprueba que no existen signos de insuficiencia cardiaca pero llama la atenci\u00f3n unas cifras de presi\u00f3n arterial y frecuencia cardiaca de 150/90 mmHg y 120 lpm, respectivamente. El ECG muestra una fibrilaci\u00f3n auricular a 110-120 lpm. \u00bfQu\u00e9 tratamiento tiene m\u00e1s probabilidades de mejorar los s\u00edntomas y reducir la mortalidad por eventos cardiovasculares?", "full_answer": "T\u00edpica pregunta muy larga y farragosa pero que en realidad no es demasiado complicada\u2026 La paciente est\u00e1 en insuficiencia cardiaca con disfunci\u00f3n ventricular moderada y adem\u00e1s presenta fibrilaci\u00f3n auricular; tiene criterios de anticoagulaci\u00f3n, y entre las dos opciones que incluyen el acenocumarol nos inclinamos por la 4, puesto que incluye el carvedilol, un f\u00e1rmaco que ha demostrado aumentar la supervivencia en la insuficiencia cardiaca. El verapamilo (y los calcioantagonistas en general) est\u00e1n contraindicados en la insuficiencia cardiaca sist\u00f3lica: no aumentan la supervivencia (incluso aumentan la morbimortalidad) y concretamente los no dihidropirid\u00ednicos (verapamilo y diltiazem) no se deben asociar a betabloqueantes por el riesgo aumentado de bloqueo del sistema de conducci\u00f3n.", "type": "CARDIOLOG\u00cdA Y CIRUG\u00cdA VASCULAR", "options": {"1": "Digoxina y \u00e1cido acetilsalic\u00edlico.", "2": "Atenolol y \u00e1cido acetilsalic\u00edlico.", "3": "Verapamilo y acenocumarol.", "4": "Carvedilol y acenocumarol.", "5": "Carvedilol y clopidogrel."}, "correct_option": 4, "explanations": {"1": {"exist": false, "char_ranges": [], "word_ranges": [], "text": ""}, "2": {"exist": false, "char_ranges": [], "word_ranges": [], "text": ""}, "3": {"exist": true, "char_ranges": [[441, 553]], "word_ranges": [[64, 78]], "text": "El verapamilo (y los calcioantagonistas en general) est\u00e1n contraindicados en la insuficiencia cardiaca sist\u00f3lica:"}, "4": {"exist": true, "char_ranges": [[207, 440]], "word_ranges": [[29, 64]], "text": "tiene criterios de anticoagulaci\u00f3n, y entre las dos opciones que incluyen el acenocumarol nos inclinamos por la 4, puesto que incluye el carvedilol, un f\u00e1rmaco que ha demostrado aumentar la supervivencia en la insuficiencia cardiaca."}, "5": {"exist": false, "char_ranges": [], "word_ranges": [], "text": ""}}} {"id": 536, "year": 2021, "question_id_specific": 122, "full_question": "Var\u00f3n de 75 a\u00f1os, hipertenso en tratamiento con enalapril (20 mg/d\u00eda), que consulta por deterioro del estado general, cefalea bitemporal y claudicaci\u00f3n mandibular de 3 semanas de evoluci\u00f3n. Adem\u00e1s, en las \u00faltimas horas, refiere dos episodios de amaurosis fugaz del ojo izquierdo. En la exploraci\u00f3n f\u00edsica destaca engrosamiento y ausencia de pulso de la arteria temporal izquierda. An\u00e1lisis: PCR 6 mg/dl (VN <1); hemoglobina 10,5 g/dl; VSG 92 mm. Radiograf\u00eda de t\u00f3rax normal. Teniendo en cuenta el diagn\u00f3stico m\u00e1s probable, marque la opci\u00f3n correcta en cuanto al tratamiento:", "full_answer": "Caso cl\u00ednico de arteritis de c\u00e9lulas gigantes. El tratamiento con corticoides sigue siendo la primera elecci\u00f3n (respuesta 1 incorrecta), que se administrar\u00e1 en forma de pulsos (difieren las dosis seg\u00fan la bibliograf\u00eda, pero al menos 125 mg/d) en caso de afectaci\u00f3n ocular (respuesta 3 incorrecta). El tratamiento de elecci\u00f3n actual para ahorrar corticoides es el Tocilizumab (respuesta 4 incorrecta), que adem\u00e1s ha demostrado reducir las recidivas (respuesta 2 correcta).", "type": "REUMATOLOG\u00cdA", "options": {"1": "Debe administrarse rituximab, puesto que los glucocorticoides han quedado relegados a un tratamiento de segunda l\u00ednea debido a sus efectos secundarios.", "2": "El tocilizumab se ha mostrado eficaz en reducir las recidivas y la dosis acumulada de prednisona en m\u00e1s del 50 % de los pacientes.", "3": "En presencia de cl\u00ednica isqu\u00e9mica, y para no empeorar el riesgo vascular del paciente, se debe evitar el uso de glucocorticoides a dosis superiores a 30 mg/d\u00eda.", "4": "Junto con bolos de glucocorticoides de 1 g/d\u00eda, el infliximab debe utilizarse como f\u00e1rmaco de primera l\u00ednea para la inducci\u00f3n a la remisi\u00f3n.", "5": NaN}, "correct_option": 2, "explanations": {"1": {"exist": true, "char_ranges": [[0, 135]], "word_ranges": [[0, 19]], "text": "Caso cl\u00ednico de arteritis de c\u00e9lulas gigantes. El tratamiento con corticoides sigue siendo la primera elecci\u00f3n (respuesta 1 incorrecta),"}, "2": {"exist": true, "char_ranges": [[412, 471]], "word_ranges": [[61, 69]], "text": "ha demostrado reducir las recidivas (respuesta 2 correcta)."}, "3": {"exist": true, "char_ranges": [[141, 297]], "word_ranges": [[20, 45]], "text": "se administrar\u00e1 en forma de pulsos (difieren las dosis seg\u00fan la bibliograf\u00eda, pero al menos 125 mg/d) en caso de afectaci\u00f3n ocular (respuesta 3 incorrecta)."}, "4": {"exist": true, "char_ranges": [[298, 399]], "word_ranges": [[45, 59]], "text": "El tratamiento de elecci\u00f3n actual para ahorrar corticoides es el Tocilizumab (respuesta 4 incorrecta),"}, "5": {"exist": false, "char_ranges": [], "word_ranges": [], "text": ""}}} {"id": 492, "year": 2020, "question_id_specific": 107, "full_question": "Paciente de 18 a\u00f1os que acude a urgencias con epistaxis de varios d\u00edas de evoluci\u00f3n, sin antecedentes personales ni familiares de inter\u00e9s. En la exploraci\u00f3n est\u00e1 afebril, se observan equimosis m\u00faltiples, no se palpa esplenomegalia. Anal\u00edtica: Leucocitos 7,2 x103/\u03bcL, Hb 12,3 g/dL, Plaquetas 6,0 x103/\u03bcL. La trombocitopenia se confirma en el frotis, donde se observan plaquetas de tama\u00f1o aumentado. Estudio de coagulaci\u00f3n y bioqu\u00edmica normales. \u00bfCu\u00e1l considera el diagn\u00f3stico m\u00e1s probable?:", "full_answer": "Se trata de un caso cl\u00ednico f\u00e1cil s\u00f3lo presenta trombopenia con cl\u00ednica, petequias y epistaxis. La 1 es falsa, hablar\u00edan de fiebre, datos de hem\u00f3lisis, cl\u00ednica neurol\u00f3gica, en el frotis habr\u00eda esquistocitos. En la 2 debe estar alterada la coagulaci\u00f3n y en este caso no se nombra.", "type": "HEMATOLOGIA", "options": {"1": "P\u00farpura tromb\u00f3tica trombocitop\u00e9nica.", "2": "Coagulaci\u00f3n intravascular diseminada.", "3": "Trombocitopenia inducida por infecci\u00f3n.", "4": "Trombocitopenia inmune primaria.", "5": NaN}, "correct_option": 4, "explanations": {"1": {"exist": true, "char_ranges": [[96, 207]], "word_ranges": [[15, 32]], "text": "La 1 es falsa, hablar\u00edan de fiebre, datos de hem\u00f3lisis, cl\u00ednica neurol\u00f3gica, en el frotis habr\u00eda esquistocitos."}, "2": {"exist": true, "char_ranges": [[208, 279]], "word_ranges": [[32, 47]], "text": "En la 2 debe estar alterada la coagulaci\u00f3n y en este caso no se nombra."}, "3": {"exist": false, "char_ranges": [], "word_ranges": [], "text": ""}, "4": {"exist": false, "char_ranges": [], "word_ranges": [], "text": ""}, "5": {"exist": false, "char_ranges": [], "word_ranges": [], "text": ""}}} {"id": 599, "year": 2022, "question_id_specific": 183, "full_question": "Mujer de 45 a\u00f1os sin antecedentes de inter\u00e9s que consulta por presentar sensaci\u00f3n disneica de unos 4 d\u00edas de evoluci\u00f3n. La exploraci\u00f3n muestra taquicardia r\u00edtmica, sin soplos y la auscultaci\u00f3n pulmonar es normal. La gasometr\u00eda arterial muestra una pO2 de 70 mmHg y una pCO2 32 mmHg. El hemograma, la funci\u00f3n renal y hep\u00e1tica son normales. Tiempo de protrombina 90 %, tiempo de tromboplastina parcial activada (TTPA) con ratio de 2 respecto al control (N <1,2). \u00bfDe los siguientes diagn\u00f3sticos cu\u00e1l es el m\u00e1s probable?:", "full_answer": "En este caso cl\u00ednico, nos plantean a una mujer que de manera r\u00e1pida presenta disnea con hipoxemia, cuadro compatible con un tromboembolismo pulmonar. El tiempo de protrombina del 90% y el TTPA con ratio de 2, nos est\u00e1 traduciendo un problema en la coagulaci\u00f3n; y los anticuerpos antifosfol\u00edpido pueden ser detectados a trav\u00e9s de la prolongaci\u00f3n de pruebas de coagulaci\u00f3n dependiente de fosfol\u00edpidos, respuesta correcta 1. Por otro lado, aunque el factor V de Leiden ha sido identificada como una causa com\u00fan de trombosis familiar, no alterar\u00eda el tiempo de coagulaci\u00f3n. En el caso de la hemofilia si que habr\u00eda un problema en la coagulaci\u00f3n de la sangre pero no es una causa frecuente de tromboembolismos. Por \u00faltimo, aunque la pericarditis aguda, podr\u00eda manifestarse en 4 d\u00edas y con taquicardia el resto de s\u00edntomas o de par\u00e1metros anal\u00edticos no son caracter\u00edsticos.", "type": "REUMATOLOG\u00cdA", "options": {"1": "S\u00edndrome antifosfol\u00edpido.", "2": "Factor V de Leiden.", "3": "Hemofilia.", "4": "Pericarditis aguda.", "5": NaN}, "correct_option": 1, "explanations": {"1": {"exist": true, "char_ranges": [[0, 421]], "word_ranges": [[0, 66]], "text": "En este caso cl\u00ednico, nos plantean a una mujer que de manera r\u00e1pida presenta disnea con hipoxemia, cuadro compatible con un tromboembolismo pulmonar. El tiempo de protrombina del 90% y el TTPA con ratio de 2, nos est\u00e1 traduciendo un problema en la coagulaci\u00f3n; y los anticuerpos antifosfol\u00edpido pueden ser detectados a trav\u00e9s de la prolongaci\u00f3n de pruebas de coagulaci\u00f3n dependiente de fosfol\u00edpidos, respuesta correcta 1."}, "2": {"exist": true, "char_ranges": [[422, 569]], "word_ranges": [[66, 91]], "text": "Por otro lado, aunque el factor V de Leiden ha sido identificada como una causa com\u00fan de trombosis familiar, no alterar\u00eda el tiempo de coagulaci\u00f3n."}, "3": {"exist": true, "char_ranges": [[570, 705]], "word_ranges": [[91, 116]], "text": "En el caso de la hemofilia si que habr\u00eda un problema en la coagulaci\u00f3n de la sangre pero no es una causa frecuente de tromboembolismos."}, "4": {"exist": true, "char_ranges": [[706, 867]], "word_ranges": [[116, 141]], "text": "Por \u00faltimo, aunque la pericarditis aguda, podr\u00eda manifestarse en 4 d\u00edas y con taquicardia el resto de s\u00edntomas o de par\u00e1metros anal\u00edticos no son caracter\u00edsticos."}, "5": {"exist": false, "char_ranges": [], "word_ranges": [], "text": ""}}} {"id": 134, "year": 2012, "question_id_specific": 169, "full_question": "Hombre de 35 a\u00f1os que acude al servicio de urgencias por dolor en su ojo derecho de 3 d\u00edas de evoluci\u00f3n. La exploraci\u00f3n biomicrosc\u00f3pica del segmento anterior muestra, tras tinci\u00f3n con fluoresce\u00edna, una \u00falcera corneal central en forma de dendrita. \u00bfCu\u00e1l es su diagn\u00f3stico?", "full_answer": "La \u00falcera corneal dendr\u00edtica es t\u00edpica de la queratitis por herpes simplex. Cursa con dolor, no demasiado llamativo, y ojo rojo escaso o ausente. Existen \u00falceras que recuerdan a dendritas que pueden confundirnos en el diagn\u00f3stico. Estas pseudodendritas nos las encontramos en el herpes Zoster y en las abrasiones corneales en resoluci\u00f3n. Si bien en la pr\u00e1ctica tendr\u00edamos que fijarnos bien en la forma de la dendrita (la dendrita aut\u00e9ntica es m\u00e1s profunda y tiene bulbos terminales al final de cada ramificaci\u00f3n), y fijarnos en la evoluci\u00f3n del dolor (en la pseudodendrita tras abrasi\u00f3n el dolor debe ir mejorando), a la hora de contestar una pregunta MIR es m\u00e1s sencillo: si es una dendrita, lo primero que tenemos que pensar es herpes. Las queratitis f\u00fangicas y bacterianas tienen infiltrados corneales. Y la crisis glaucomatocicl\u00edtica (s\u00edndrome de Posner Schlossman) es un tipo de uve\u00edtis anterior hipertensiva idiop\u00e1tica, que no cursa con \u00falcera.", "type": "OFTALMOLOG\u00cdA", "options": {"1": "Queratitis herp\u00e9tica.", "2": "Abrasi\u00f3n corneal.", "3": "Queratitis f\u00fangica.", "4": "Queratitis bacteriana.", "5": "Crisis glaucomatocicl\u00edtica."}, "correct_option": 1, "explanations": {"1": {"exist": true, "char_ranges": [[0, 145]], "word_ranges": [[0, 24]], "text": "La \u00falcera corneal dendr\u00edtica es t\u00edpica de la queratitis por herpes simplex. Cursa con dolor, no demasiado llamativo, y ojo rojo escaso o ausente."}, "2": {"exist": true, "char_ranges": [[673, 737]], "word_ranges": [[110, 122]], "text": "si es una dendrita, lo primero que tenemos que pensar es herpes."}, "3": {"exist": true, "char_ranges": [[738, 805]], "word_ranges": [[122, 130]], "text": "Las queratitis f\u00fangicas y bacterianas tienen infiltrados corneales."}, "4": {"exist": true, "char_ranges": [[738, 805]], "word_ranges": [[122, 130]], "text": "Las queratitis f\u00fangicas y bacterianas tienen infiltrados corneales."}, "5": {"exist": true, "char_ranges": [[808, 950]], "word_ranges": [[131, 151]], "text": "la crisis glaucomatocicl\u00edtica (s\u00edndrome de Posner Schlossman) es un tipo de uve\u00edtis anterior hipertensiva idiop\u00e1tica, que no cursa con \u00falcera."}}} {"id": 245, "year": 2014, "question_id_specific": 115, "full_question": "Un paciente en tratamiento quimioter\u00e1pico por leucemia ingresa por una neumon\u00eda para la que se ha prescrito tratamiento con cefepime. En una Rx/TAC t\u00f3rax se observa un infiltrado con signo del halo y menisco semilunar. La lesi\u00f3n es perif\u00e9rica y se indica una punci\u00f3n transtor\u00e1cica para toma de muestras. Hasta tener los resultados histol\u00f3gicos y microbiol\u00f3gicos definitivos, \u00bfQu\u00e9 antimicrobiano a\u00f1adir\u00eda al tratamiento?", "full_answer": "Pregunta de respuesta directa. Se trata de un paciente en tratamiento quimioterapico con un cuadro respiratorio y el \u201csigno del halo\u201d en la Rx, datos muy sugestivos de una aspergilosis pulmonar invasora. El tratamiento se realiza con voriconazol. Aspergillus es resistente a fluconazol, por lo que no ser\u00eda una opci\u00f3n v\u00e1lida.", "type": "ENFERMEDADES INFECCIOSAS", "options": {"1": "Ganciclovir.", "2": "Caspofungina.", "3": "Fluconazol.", "4": "Piperacilina-tazobactam.", "5": "Voriconazol."}, "correct_option": 5, "explanations": {"1": {"exist": false, "char_ranges": [], "word_ranges": [], "text": ""}, "2": {"exist": false, "char_ranges": [], "word_ranges": [], "text": ""}, "3": {"exist": true, "char_ranges": [[247, 325]], "word_ranges": [[38, 51]], "text": "Aspergillus es resistente a fluconazol, por lo que no ser\u00eda una opci\u00f3n v\u00e1lida."}, "4": {"exist": false, "char_ranges": [], "word_ranges": [], "text": ""}, "5": {"exist": true, "char_ranges": [[31, 246]], "word_ranges": [[4, 38]], "text": "Se trata de un paciente en tratamiento quimioterapico con un cuadro respiratorio y el \u201csigno del halo\u201d en la Rx, datos muy sugestivos de una aspergilosis pulmonar invasora. El tratamiento se realiza con voriconazol."}}} {"id": 387, "year": 2016, "question_id_specific": 156, "full_question": "Lactante de 5 meses de edad alimentado con lactancia materna exclusiva hasta la actualidad que por motivo de trabajo materno se le introduce biber\u00f3n con f\u00f3rmula de inicio y hace unos d\u00edas se le adiciona con cereales con y sin gluten. Como antecedentes familiares destaca madre de 28 a\u00f1os asm\u00e1tica, padre de 32 a\u00f1os sano y hermano de 5 a\u00f1os afecto de enfermedad cel\u00edaca y dermatitis at\u00f3pica. Como antecedentes personales, embarazo sin incidencias y parto mediante ces\u00e1rea habi\u00e9ndosele ofrecido biber\u00f3n con f\u00f3rmula de inicio el primer d\u00eda de vida en la maternidad. Desde hace unos d\u00edas comienza con distensi\u00f3n abdominal, deposiciones diarreicas, rechazo a las tomas, eritema peribucal con duraci\u00f3n m\u00e1s prolongada en el tiempo tras la ingesta del biber\u00f3n y \u00faltimamente v\u00f3mitos tras su ingesta, tolerando bien el pecho materno. \u00bfCu\u00e1l es su diagn\u00f3stico m\u00e1s probable?", "full_answer": "Ni\u00f1o con m\u00faltiples antecedentes familiares de alergia (asma, atopia), que ha recibido leche artificial en las primeras horas de vida (sensibilizaci\u00f3n a la prote\u00edna de la leche de vaca) y que coincidiendo con la toma de leche artificial inicia cl\u00ednica digestiva y exantema = APLV.", "type": "PEDIATR\u00cdA", "options": {"1": "Enfermedad cel\u00edaca.", "2": "Gastroenteritis aguda.", "3": "Alergia a las prote\u00ednas de la leche de vaca.", "4": "Alergia a los biberones.", "5": NaN}, "correct_option": 3, "explanations": {"1": {"exist": false, "char_ranges": [], "word_ranges": [], "text": ""}, "2": {"exist": false, "char_ranges": [], "word_ranges": [], "text": ""}, "3": {"exist": true, "char_ranges": [[0, 279]], "word_ranges": [[0, 45]], "text": "Ni\u00f1o con m\u00faltiples antecedentes familiares de alergia (asma, atopia), que ha recibido leche artificial en las primeras horas de vida (sensibilizaci\u00f3n a la prote\u00edna de la leche de vaca) y que coincidiendo con la toma de leche artificial inicia cl\u00ednica digestiva y exantema = APLV."}, "4": {"exist": false, "char_ranges": [], "word_ranges": [], "text": ""}, "5": {"exist": false, "char_ranges": [], "word_ranges": [], "text": ""}}} {"id": 41, "year": 2011, "question_id_specific": 139, "full_question": "Ante un paciente con mal estado general, fiebre (temperatura axilar 39,5\u00ba), tumefacci\u00f3n submandibular bilateral de seis d\u00edas de evoluci\u00f3n, dolor bucal y trismus, \u00bfcu\u00e1l de las siguientes afirmaciones es correcta?", "full_answer": "Este paciente est\u00e1 grave. El trismus y la fiebre obligan a pensar en una infecci\u00f3n del espacio profundo cervical y eso no es un juego porque puede obstruirse la v\u00eda a\u00e9rea. Sabiendo esto, las opciones 1, 2 y 5 est\u00e1n fuera de lugar y me quedan la 3 y la 4. Puede haber ya una mediastinitis; al fin y al cabo, la historia natural de las infecciones del espacio profundo cervical es \u00e9sa, pero la respuesta 4 es m\u00e1s correcta y adem\u00e1s, estamos en el bloque de ORL, la especialidad de la v\u00eda a\u00e9rea. Me gusta esta pregunta porque exige un conocimiento que es importante en la pr\u00e1ctica cl\u00ednica.", "type": "OTORRINOLARINGOLOG\u00cdA Y CIRUG\u00cdA MAXILOFACIAL", "options": {"1": "La primera probabilidad diagn\u00f3stica es de carcinoma de suelo de boca con met\u00e1stasis cervicales bilaterales.", "2": "La prioridad terap\u00e9utica es garantizar la alimentaci\u00f3n del paciente.", "3": "Nos encontramos casi con seguridad ante una mediastinitis.", "4": "Debemos considerar de forma prioritario el riesgo de obstrucci\u00f3n de la v\u00eda a\u00e9rea.", "5": "Debemos realizar como primera medida diagn\u00f3stica una citolog\u00eda mediante punci\u00f3n aspiraci\u00f3n con aguja fina."}, "correct_option": 4, "explanations": {"1": {"exist": true, "char_ranges": [[0, 229]], "word_ranges": [[0, 43]], "text": "Este paciente est\u00e1 grave. El trismus y la fiebre obligan a pensar en una infecci\u00f3n del espacio profundo cervical y eso no es un juego porque puede obstruirse la v\u00eda a\u00e9rea. Sabiendo esto, las opciones 1, 2 y 5 est\u00e1n fuera de lugar"}, "2": {"exist": true, "char_ranges": [[0, 229]], "word_ranges": [[0, 43]], "text": "Este paciente est\u00e1 grave. El trismus y la fiebre obligan a pensar en una infecci\u00f3n del espacio profundo cervical y eso no es un juego porque puede obstruirse la v\u00eda a\u00e9rea. Sabiendo esto, las opciones 1, 2 y 5 est\u00e1n fuera de lugar"}, "3": {"exist": false, "char_ranges": [], "word_ranges": [], "text": ""}, "4": {"exist": false, "char_ranges": [], "word_ranges": [], "text": ""}, "5": {"exist": true, "char_ranges": [[0, 229]], "word_ranges": [[0, 43]], "text": "Este paciente est\u00e1 grave. El trismus y la fiebre obligan a pensar en una infecci\u00f3n del espacio profundo cervical y eso no es un juego porque puede obstruirse la v\u00eda a\u00e9rea. Sabiendo esto, las opciones 1, 2 y 5 est\u00e1n fuera de lugar"}}} {"id": 385, "year": 2016, "question_id_specific": 153, "full_question": "\u00bfCu\u00e1l es el diagn\u00f3stico m\u00e1s probable de un reci\u00e9n nacido con microcefalia, retraso del crecimiento intrauterino, cardiopat\u00eda cong\u00e9nita, pie astr\u00e1galo vertical y una facies peculiar (microftalmia, hendiduras palpebrales peque\u00f1as, micrognatia y orejas displ\u00e1sicas), las manos con el dedo \u00edndice y me\u00f1ique sobre el medio y anular?", "full_answer": "Est\u00e1 describiendo fielmente un S. de Edwards. Aunque las cardiopat\u00edas y la alteraci\u00f3n del pie que describe son comunes a varias cromosomopat\u00edas, la alteraci\u00f3n de los dedos de las manos es muy caracter\u00edstica del Edwards.", "type": "PEDIATR\u00cdA", "options": {"1": "Trisom\u00eda 18. (S\u00edndrome de Edwards).", "2": "Trisom\u00eda 13. (Sindrome de Patau).", "3": "Trisom\u00eda 21. (S\u00edndrome de Down).", "4": "Trisom\u00eda 9.", "5": NaN}, "correct_option": 1, "explanations": {"1": {"exist": true, "char_ranges": [[46, 219]], "word_ranges": [[7, 35]], "text": "Aunque las cardiopat\u00edas y la alteraci\u00f3n del pie que describe son comunes a varias cromosomopat\u00edas, la alteraci\u00f3n de los dedos de las manos es muy caracter\u00edstica del Edwards."}, "2": {"exist": true, "char_ranges": [[46, 219]], "word_ranges": [[7, 35]], "text": "Aunque las cardiopat\u00edas y la alteraci\u00f3n del pie que describe son comunes a varias cromosomopat\u00edas, la alteraci\u00f3n de los dedos de las manos es muy caracter\u00edstica del Edwards."}, "3": {"exist": false, "char_ranges": [], "word_ranges": [], "text": ""}, "4": {"exist": true, "char_ranges": [[46, 219]], "word_ranges": [[7, 35]], "text": "Aunque las cardiopat\u00edas y la alteraci\u00f3n del pie que describe son comunes a varias cromosomopat\u00edas, la alteraci\u00f3n de los dedos de las manos es muy caracter\u00edstica del Edwards."}, "5": {"exist": false, "char_ranges": [], "word_ranges": [], "text": ""}}} {"id": 194, "year": 2013, "question_id_specific": 162, "full_question": "Ni\u00f1a de 3 a\u00f1os, procedente de Bangladesh, que consulta por episodios febriles intermitentes de 3 semanas de evoluci\u00f3n, asociados a debilidad y p\u00e9rdida de apetito. A la exploraci\u00f3n f\u00edsica llama la atenci\u00f3n esplenomegalia marcada y palidez mucocut\u00e1nea. En las pruebas complementarias destaca: hemoglobina 8,5 mg/dl, hematocrito 26%, VCM 86 fL, HCM 29 pg, leucocitos 2800/mL con 300 neutr\u00f3filos /ml, plaquetas 54000/ml, GOT 85 U/l, GPT92 U/l e hipergammaglobulinemia policlonal en proteinograma de suero. Se\u00f1ale el diagn\u00f3stico m\u00e1s probable con los datos disponibles hasta este momento:", "full_answer": "Los datos como la pancitopenia y la hipergammaglobulinemia son muy caracter\u00edsticos del kala-azar. Tambi\u00e9n orienta a este diagn\u00f3stico la procedencia de la ni\u00f1a, aunque es una enfermedad que se presenta tambi\u00e9n en el \u00e1rea mediterr\u00e1nea. Ante este cuadro es importante tener presente en el diagn\u00f3stico diferencial tambi\u00e9n la posibilidad de leucosis.", "type": "PEDIATR\u00cdA", "options": {"1": "Leucemia linfobl\u00e1stica aguda.", "2": "Linfoma de Burkitt.", "3": "Leishmaniasis visceral.", "4": "Tuberculosis miliar.", "5": "Malaria cr\u00f3nica."}, "correct_option": 3, "explanations": {"1": {"exist": false, "char_ranges": [], "word_ranges": [], "text": ""}, "2": {"exist": false, "char_ranges": [], "word_ranges": [], "text": ""}, "3": {"exist": true, "char_ranges": [[0, 158]], "word_ranges": [[0, 23]], "text": "Los datos como la pancitopenia y la hipergammaglobulinemia son muy caracter\u00edsticos del kala-azar. Tambi\u00e9n orienta a este diagn\u00f3stico la procedencia de la ni\u00f1a,"}, "4": {"exist": false, "char_ranges": [], "word_ranges": [], "text": ""}, "5": {"exist": false, "char_ranges": [], "word_ranges": [], "text": ""}}} {"id": 371, "year": 2016, "question_id_specific": 126, "full_question": "Una mujer de 59 a\u00f1os con antecedentes personales de c\u00e1ncer de mama intervenida hace 8 meses y osteoporosis, sufre una ca\u00edda y tiene una fractura de cuello de f\u00e9mur derecho. Es intervenida quir\u00fargicamente y permanece ingresada en reposo con escayola durante 10 d\u00edas. A la semana siguiente de su alta nota de forma s\u00fabita falta de aire que la lleva a consultar a su m\u00e9dico de cabecera que nota que su pantorrilla derecha tiene edema y dolor a la palpaci\u00f3n. \u00bfQu\u00e9 elementos cl\u00ednicos NO ser\u00edan importantes a la hora de determinar que tiene un riesgo cl\u00ednico alto de padecer una tromboembolia pulmonar?:", "full_answer": "El resto de antecedentes y hallazgos esta relacionados muy directamente con el riesgo aumentado de fen\u00f3menos tromb\u00f3ticos. La osteoporosis no es un factor de riesgo para trombosis.", "type": "NEUMOLOG\u00cdA Y CIRUG\u00cdA TOR\u00c1CICA", "options": {"1": "El antecedente de c\u00e1ncer de mama.", "2": "La historia de osteoporosis.", "3": "El antecedente de haber estado m\u00e1s de 3 d\u00edas en reposo.", "4": "La presencia de edema unilateral de la pantorrilla derecha.", "5": NaN}, "correct_option": 2, "explanations": {"1": {"exist": true, "char_ranges": [[0, 121]], "word_ranges": [[0, 17]], "text": "El resto de antecedentes y hallazgos esta relacionados muy directamente con el riesgo aumentado de fen\u00f3menos tromb\u00f3ticos."}, "2": {"exist": true, "char_ranges": [[122, 179]], "word_ranges": [[17, 27]], "text": "La osteoporosis no es un factor de riesgo para trombosis."}, "3": {"exist": true, "char_ranges": [[0, 121]], "word_ranges": [[0, 17]], "text": "El resto de antecedentes y hallazgos esta relacionados muy directamente con el riesgo aumentado de fen\u00f3menos tromb\u00f3ticos."}, "4": {"exist": true, "char_ranges": [[0, 121]], "word_ranges": [[0, 17]], "text": "El resto de antecedentes y hallazgos esta relacionados muy directamente con el riesgo aumentado de fen\u00f3menos tromb\u00f3ticos."}, "5": {"exist": false, "char_ranges": [], "word_ranges": [], "text": ""}}} {"id": 283, "year": 2016, "question_id_specific": 56, "full_question": "Paciente de 54 a\u00f1os que ingresa por fiebre termometrada de 38\u00b0C en los cinco d\u00edas previos y disnea de reposo( lV) que apareci\u00f3 6 horas antes de acudir al hospital. En urgencias la exploraci\u00f3n es compatible con insuficiencia cardiaca y el ECG muestra bloqueo auriculo-ventricular completo con una frecuencia ventricular de escape de 45 lpm. Los signos de insuficiencia cardiaca son refractarios al tratamiento m\u00e9dico y la ecocardiografia transesof\u00e1gica realizada muestra una v\u00e1lvula a\u00f3rtica con orificio regurgitante efectivo de 0.5 cm2_ Los cultivos seriados son positivos para Streptococcus gallolyticus. Indique la actitud m\u00e1s acertada:", "full_answer": "Cirug\u00eda cardiaca de reemplazo valvular a\u00f3rtico por pr\u00f3tesis mec\u00e1nica con terapia antibi\u00f3tica seg\u00fan antibiograma.", "type": "CARDIOLOG\u00cdA Y CIRUG\u00cdA VASCULAR", "options": {"1": "Cirug\u00eda cardiaca de reemplazo valvular a\u00f3rtico por pr\u00f3tesis mec\u00e1nica con terapia antibi\u00f3tica seg\u00fan antibiograma.", "2": "Terapia antibi\u00f3tica seg\u00fan antibiograma e implantaci\u00f3n de bal\u00f3n de contrapulsaci\u00f3n intra-a\u00f3rtico y marcapasos transitorio hasta 3semanas, tras las cuales se implantar\u00e1 marcapasos definitivo.", "3": "Implantaci\u00f3n de marcapasos transitorio, terapia antibi\u00f3tica seg\u00fan antibiograma e implantaci\u00f3n percut\u00e1nea de pr\u00f3tesis valvular a\u00f3rtica.", "4": "Implantaci\u00f3n urgente de marcapasos definitivo con terapia antibi\u00f3tica seg\u00fan antibiogramadurante 6 semanas.", "5": NaN}, "correct_option": 1, "explanations": {"1": {"exist": true, "char_ranges": [[0, 112]], "word_ranges": [[0, 14]], "text": "Cirug\u00eda cardiaca de reemplazo valvular a\u00f3rtico por pr\u00f3tesis mec\u00e1nica con terapia antibi\u00f3tica seg\u00fan antibiograma."}, "2": {"exist": false, "char_ranges": [], "word_ranges": [], "text": ""}, "3": {"exist": false, "char_ranges": [], "word_ranges": [], "text": ""}, "4": {"exist": false, "char_ranges": [], "word_ranges": [], "text": ""}, "5": {"exist": false, "char_ranges": [], "word_ranges": [], "text": ""}}} {"id": 109, "year": 2012, "question_id_specific": 149, "full_question": "Una mujer de 32 a\u00f1os asintom\u00e1tica consulta para una revisi\u00f3n en salud ginecol\u00f3gica porque desea quedarse embarazada. En dicha revisi\u00f3n se le detecta un mioma uterino de 4cm en parte intramural y en parte subseroso situado en la cara anterior del \u00fatero y que no deforma la cavidad endometrial. \u00bfQu\u00e9 actitud recomendar\u00eda?", "full_answer": "La respuesta correcta es la 5. Un mioma asintom\u00e1tico de este tama\u00f1o que no deforma la cavidad endometrial no molesta de cara a un embarazo.", "type": "GINECOLOG\u00cdA Y OBSTETRICIA", "options": {"1": "Miomectom\u00eda por v\u00eda laparosc\u00f3pica.", "2": "Miomectom\u00eda por v\u00eda laparot\u00f3mica.", "3": "Embolizaci\u00f3n del mioma por cateterismo arterial.", "4": "Tratamiento con an\u00e1logos de Gn-Rh durante tres meses antes de intentar el embarazo.", "5": "Intentar el embarazo sin ning\u00fan tratamiento previo."}, "correct_option": 5, "explanations": {"1": {"exist": false, "char_ranges": [], "word_ranges": [], "text": ""}, "2": {"exist": false, "char_ranges": [], "word_ranges": [], "text": ""}, "3": {"exist": false, "char_ranges": [], "word_ranges": [], "text": ""}, "4": {"exist": false, "char_ranges": [], "word_ranges": [], "text": ""}, "5": {"exist": true, "char_ranges": [[31, 139]], "word_ranges": [[6, 25]], "text": "Un mioma asintom\u00e1tico de este tama\u00f1o que no deforma la cavidad endometrial no molesta de cara a un embarazo."}}} {"id": 92, "year": 2012, "question_id_specific": 235, "full_question": "Paciente de 60 a\u00f1os diagnosticada de neoplasia de mama hace 10 a\u00f1os. Realiz\u00f3 tratamiento con radioquimioterapia y posteriormente hormonal durante 5 a\u00f1os. Un estudio con gammagraf\u00eda \u00f3sea realizado por dolores \u00f3seos polit\u00f3picos demostr\u00f3 la presencia de met\u00e1stasis \u00f3seas. Actualmente est\u00e1 en y tratamiento con opioides menores y AINEs con buen control del dolor. Acude a consulta por cefalea que no cede con la actual analgesia realizando una TC cerebral que muestra im\u00e1genes compatibles con met\u00e1stasis cerebrales. En relaci\u00f3n al tratamiento del dolor indique la CORRECTA:", "full_answer": "Las met\u00e1stasis cerebrales producen dolor por un cuadro de hipertensi\u00f3n craneal. El tratamiento de elecci\u00f3n son los corticoides. De elecci\u00f3n la dexametasona en dosis oral de 4 a 16 mg/d\u00eda.", "type": "CUIDADOS PALIATIVOS", "options": {"1": "Se debe cambiar a opioides mayores.", "2": "Se deben administrar las dosis extras de opioides que sean necesarias.", "3": "Se deben a\u00f1adir corticoides.", "4": "Se debe cambiar a un opioide mayor y mantener los AINEs.", "5": "Se debe ingresar al paciente para tratamiento endovenoso con opioide mayor."}, "correct_option": 3, "explanations": {"1": {"exist": false, "char_ranges": [], "word_ranges": [], "text": ""}, "2": {"exist": false, "char_ranges": [], "word_ranges": [], "text": ""}, "3": {"exist": true, "char_ranges": [[0, 127]], "word_ranges": [[0, 18]], "text": "Las met\u00e1stasis cerebrales producen dolor por un cuadro de hipertensi\u00f3n craneal. El tratamiento de elecci\u00f3n son los corticoides."}, "4": {"exist": false, "char_ranges": [], "word_ranges": [], "text": ""}, "5": {"exist": false, "char_ranges": [], "word_ranges": [], "text": ""}}} {"id": 342, "year": 2016, "question_id_specific": 158, "full_question": "Una mujer de 24 a\u00f1os, primigesta, sufre un aborto espont\u00e1neo a las 7 semanas de gestaci\u00f3n. El estudio anatomopatol\u00f3gico de los restos abortivos indica enfermedad molar. Le debemos informar que:", "full_answer": "La enfermedad trofobl\u00e1stica gestacional debe mantener un seguimiento posterior (independientemente de si la evacuaci\u00f3n ha sido completa o no) y se debe aconsejar a la paciente no quedar gestante de nuevo hasta conseguir al menos 6 meses con t\u00edtulos de BHCG negativos. El riesgo de recurrencia en otra gestaci\u00f3n es bajo (1/55) y el 90% de los casos evoluciona de forma satisfactorias sin desarrollar una neoplasia.", "type": "GINECOLOG\u00cdA Y OBSTETRICIA", "options": {"1": "El riesgo de una nueva gestaci\u00f3n molar en un futuro embarazo es del 50%.", "2": "No debe quedar embarazada hasta realizar controles peri\u00f3dicos y haber pasado un a\u00f1os con niveles de BHCG negativos.", "3": "No es necesario realizar controles posteriores si la evacuaci\u00f3n del tejido trofobl\u00e1stico fue completa.", "4": "Es necesario que se realice controles peri\u00f3dicos ya que en el 40% de los casos desarrollar\u00e1 una neoplasia trofobl\u00e1stica gestacional.", "5": NaN}, "correct_option": 2, "explanations": {"1": {"exist": false, "char_ranges": [], "word_ranges": [], "text": ""}, "2": {"exist": true, "char_ranges": [[0, 267]], "word_ranges": [[0, 42]], "text": "La enfermedad trofobl\u00e1stica gestacional debe mantener un seguimiento posterior (independientemente de si la evacuaci\u00f3n ha sido completa o no) y se debe aconsejar a la paciente no quedar gestante de nuevo hasta conseguir al menos 6 meses con t\u00edtulos de BHCG negativos."}, "3": {"exist": false, "char_ranges": [], "word_ranges": [], "text": ""}, "4": {"exist": false, "char_ranges": [], "word_ranges": [], "text": ""}, "5": {"exist": false, "char_ranges": [], "word_ranges": [], "text": ""}}} {"id": 357, "year": 2016, "question_id_specific": 232, "full_question": "Una mujer de 24 a\u00f1os de edad consulta al haber apreciado adenopat\u00edas inguinales. En el interrogatorio no se recoge la presencia de ninguna molestia local ni datos sugerentes de infecci\u00f3n de transmisi\u00f3n sexual. En la exploraci\u00f3n se aprecian dos adenopat\u00edas, una en cada ingle, de 1 cm de di\u00e1metro mayor, blandas, m\u00f3viles, no dolorosas. No se aprecia ninguna lesi\u00f3n cut\u00e1nea en miembros inferiores, ano \u00f3 perin\u00e9. \u00bfQu\u00e9 prueba considera imprescindible?", "full_answer": "La presencia de ganglios inguinales de hasta 1-1,5cm puede ser normal ante la ausencia de s\u00edntomas. En este caso, son ganglios muy inespec\u00edficos, sin signos de malignidad (m\u00f3viles, blandos, no dolorosos) que no nos hacen sospechar infiltraci\u00f3n neopl\u00e1sica y la falta de otra cl\u00ednica tambi\u00e9n nos lleva a descartar la presencia de ETS.", "type": "GINECOLOG\u00cdA Y OBSTETRICIA", "options": {"1": "Una serolog\u00eda de l\u00faes puesto que lo m\u00e1s probable es que se trate de una infecci\u00f3n por Treponema pallidum.", "2": "Una exploraci\u00f3n ginecol\u00f3gica a fin de descartar un c\u00e1ncer de ovario.", "3": "Por las caracter\u00edsticas cl\u00ednicas parece tratarse de unos ganglios normales y no deben hacerse exploraciones complementarias.", "4": "Debe realizarse una prueba de Paul-Bunell, a fin de descartar una mononucleosis infecciosa.", "5": NaN}, "correct_option": 3, "explanations": {"1": {"exist": false, "char_ranges": [], "word_ranges": [], "text": ""}, "2": {"exist": false, "char_ranges": [], "word_ranges": [], "text": ""}, "3": {"exist": true, "char_ranges": [[0, 332]], "word_ranges": [[0, 53]], "text": "La presencia de ganglios inguinales de hasta 1-1,5cm puede ser normal ante la ausencia de s\u00edntomas. En este caso, son ganglios muy inespec\u00edficos, sin signos de malignidad (m\u00f3viles, blandos, no dolorosos) que no nos hacen sospechar infiltraci\u00f3n neopl\u00e1sica y la falta de otra cl\u00ednica tambi\u00e9n nos lleva a descartar la presencia de ETS."}, "4": {"exist": false, "char_ranges": [], "word_ranges": [], "text": ""}, "5": {"exist": false, "char_ranges": [], "word_ranges": [], "text": ""}}} {"id": 596, "year": 2022, "question_id_specific": 121, "full_question": "Mujer de 79 a\u00f1os que ingresa por una fractura osteopor\u00f3tica de cadera. Respecto a la prevenci\u00f3n secundaria de las fracturas por fragilidad, se\u00f1ale la respuesta INCORRECTA:", "full_answer": "Esta pregunta corresponde al apartado de traumatolog\u00eda, no obstante se podr\u00eda contestar con los conocimientos de reumatolog\u00eda y con las recomendaciones de osteoporosis de la Sociedad Espa\u00f1ola de Reumatolog\u00eda. En la osteoporosis, uno de los principales riesgos asociados al incremento de riesgo de fractura es la baja adherencia al tratamiento, por lo que la respuesta 1 es correcta. La respuesta 2 se encuentra en la gu\u00eda de la SER, en la que se confirma que algunos estudios concluyen que los marcadores de remodelado \u00f3seo pueden ser \u00fatiles para monitorizar de forma precoz el cumplimiento y la respuesta al tratamiento. La respuesta 4 es correcta ya que nuevamente en la gu\u00eda de la SER 2019 citan textualmente: \u201cLa evidencia cient\u00edfica actual permite afirmar que ni aumentar el calcio diet\u00e9tico ni tomar suplementos de calcio de forma aislada protege frente a la aparici\u00f3n de fracturas\u201d. Por ello la respuesta correcta a esta pregunta es la opci\u00f3n 3. Los pacientes en tratamiento farmacol\u00f3gico para la OP se deben utilizar suplementos de calcio y vitamina D porque pr\u00e1cticamente todos los ensayos cl\u00ednicos que han demostrado eficacia de los f\u00e1rmacos antiosteopor\u00f3ticos incluyen de forma rutinaria suplementos de calcio y colecalciferol (vitamina D3), pero no en monoterapia.", "type": "REUMATOLOG\u00cdA", "options": {"1": "La baja adherencia al tratamiento se asocia a un incremento del riesgo de fractura.", "2": "Los marcadores de remodelado \u00f3seo pueden ser \u00fatiles para monitorizar de forma precoz la respuesta al tratamiento.", "3": "La vitamina D en monoterapia es eficaz en la reducci\u00f3n de dichas fracturas en personas mayores no institucionalizadas.", "4": "El aumento de calcio diet\u00e9tico o tomar suplementos de calcio de forma aislada no protegen frente a la aparici\u00f3n de fracturas.", "5": NaN}, "correct_option": 3, "explanations": {"1": {"exist": true, "char_ranges": [[209, 382]], "word_ranges": [[29, 58]], "text": "En la osteoporosis, uno de los principales riesgos asociados al incremento de riesgo de fractura es la baja adherencia al tratamiento, por lo que la respuesta 1 es correcta."}, "2": {"exist": true, "char_ranges": [[383, 621]], "word_ranges": [[58, 99]], "text": "La respuesta 2 se encuentra en la gu\u00eda de la SER, en la que se confirma que algunos estudios concluyen que los marcadores de remodelado \u00f3seo pueden ser \u00fatiles para monitorizar de forma precoz el cumplimiento y la respuesta al tratamiento."}, "3": {"exist": true, "char_ranges": [[953, 1276]], "word_ranges": [[155, 201]], "text": "Los pacientes en tratamiento farmacol\u00f3gico para la OP se deben utilizar suplementos de calcio y vitamina D porque pr\u00e1cticamente todos los ensayos cl\u00ednicos que han demostrado eficacia de los f\u00e1rmacos antiosteopor\u00f3ticos incluyen de forma rutinaria suplementos de calcio y colecalciferol (vitamina D3), pero no en monoterapia."}, "4": {"exist": true, "char_ranges": [[622, 889]], "word_ranges": [[99, 143]], "text": "La respuesta 4 es correcta ya que nuevamente en la gu\u00eda de la SER 2019 citan textualmente: \u201cLa evidencia cient\u00edfica actual permite afirmar que ni aumentar el calcio diet\u00e9tico ni tomar suplementos de calcio de forma aislada protege frente a la aparici\u00f3n de fracturas\u201d."}, "5": {"exist": false, "char_ranges": [], "word_ranges": [], "text": ""}}} {"id": 334, "year": 2016, "question_id_specific": 82, "full_question": "Mujer diagnosticada de diabetes mellitus tipo 1 desde hace 24 a\u00f1os. Acude a la consulta con cl\u00ednica de 3 meses de evoluci\u00f3n de hormigueos en ambos pies, con distribuci\u00f3n en calcet\u00edn, con dolor parest\u00e9sico y sensaci\u00f3n de pies calientes de predominio nocturno, que interfiere notablemente con el sue\u00f1o. \u00bfCu\u00e1l de los siguientes f\u00e1rmacos utilizar\u00eda en primera l\u00ednea para el tratamiento de su patolog\u00eda?", "full_answer": "Duloxetina; el cuadro cl\u00ednico descrito es de neuropat\u00eda diab\u00e9tica. De los f\u00e1rmacos propuestos el de primera elecci\u00f3n ser\u00eda un antidepresivo dual como la duloxetina.", "type": "FARMACOLOG\u00cdA", "options": {"1": "Ibuprofeno.", "2": "Oxicodona.", "3": "Duloxetina.", "4": "Paracetamol.", "5": NaN}, "correct_option": 3, "explanations": {"1": {"exist": false, "char_ranges": [], "word_ranges": [], "text": ""}, "2": {"exist": false, "char_ranges": [], "word_ranges": [], "text": ""}, "3": {"exist": true, "char_ranges": [[12, 164]], "word_ranges": [[1, 24]], "text": "el cuadro cl\u00ednico descrito es de neuropat\u00eda diab\u00e9tica. De los f\u00e1rmacos propuestos el de primera elecci\u00f3n ser\u00eda un antidepresivo dual como la duloxetina."}, "4": {"exist": false, "char_ranges": [], "word_ranges": [], "text": ""}, "5": {"exist": false, "char_ranges": [], "word_ranges": [], "text": ""}}} {"id": 81, "year": 2012, "question_id_specific": 44, "full_question": "Paciente de 78 a\u00f1os diagnosticado de miocardiopat\u00eda dilatada idiop\u00e1tica con disfunci\u00f3n ventricular izquierda ligera (fracci\u00f3n de eyecci\u00f3n 48%) y fibrilaci\u00f3n auricular cr\u00f3nica. \u00bfCu\u00e1l de los siguientes f\u00e1rmacos deber\u00eda ser evitado en su tratamiento?", "full_answer": "La digoxina es \u00fatil como tto. sintom\u00e1tico (es un inotr\u00f3pico positivo) en la insuficiencia cardiaca, aunque no mejora la supervivencia. El carvedilol es un a\u00df-bloqueante muy bien tolerado (por su efecto anti-a1) que s\u00ed se asocia a una mejora sintom\u00e1tica y de la supervivencia. El acenocumarol (Sintrom) estar\u00eda indicado en este paciente con FA cr\u00f3nica para evitar eventos tromboemb\u00f3licos. El enalapril y los IECAs en general, al inhibir el eje renina-angiotensina-aldosterona, suprimen parte de los efectos neurohormonales que se producen en la insuficiencia cardiaca; mejoran la supervivencia en los pacientes con disfunci\u00f3n sist\u00f3lica. El ibuprofeno y los AINEs en general est\u00e1n contraindicados en los pacientes con insuficiencia cardiaca porque inhiben la s\u00edntesis de prostaglandinas a nivel renal, provocando un aumento de las resistencias vasculares sist\u00e9micas, reducci\u00f3n de la perfusi\u00f3n renal, e inhibici\u00f3n de la excreci\u00f3n de sodio y agua, pudiendo precipitar una descompensaci\u00f3n de la insuficiencia cardiaca.", "type": "CARDIOLOG\u00cdA Y CIRUG\u00cdA VASCULAR", "options": {"1": "Digoxina.", "2": "Carvedilol.", "3": "Acenocumarol.", "4": "Enalapril.", "5": "Ibuprofeno."}, "correct_option": 5, "explanations": {"1": {"exist": true, "char_ranges": [[0, 134]], "word_ranges": [[0, 20]], "text": "La digoxina es \u00fatil como tto. sintom\u00e1tico (es un inotr\u00f3pico positivo) en la insuficiencia cardiaca, aunque no mejora la supervivencia."}, "2": {"exist": true, "char_ranges": [[135, 275]], "word_ranges": [[20, 44]], "text": "El carvedilol es un a\u00df-bloqueante muy bien tolerado (por su efecto anti-a1) que s\u00ed se asocia a una mejora sintom\u00e1tica y de la supervivencia."}, "3": {"exist": true, "char_ranges": [[276, 387]], "word_ranges": [[44, 59]], "text": "El acenocumarol (Sintrom) estar\u00eda indicado en este paciente con FA cr\u00f3nica para evitar eventos tromboemb\u00f3licos."}, "4": {"exist": true, "char_ranges": [[388, 635]], "word_ranges": [[59, 93]], "text": "El enalapril y los IECAs en general, al inhibir el eje renina-angiotensina-aldosterona, suprimen parte de los efectos neurohormonales que se producen en la insuficiencia cardiaca; mejoran la supervivencia en los pacientes con disfunci\u00f3n sist\u00f3lica."}, "5": {"exist": true, "char_ranges": [[636, 798]], "word_ranges": [[93, 117]], "text": "El ibuprofeno y los AINEs en general est\u00e1n contraindicados en los pacientes con insuficiencia cardiaca porque inhiben la s\u00edntesis de prostaglandinas a nivel renal,"}}} {"id": 386, "year": 2016, "question_id_specific": 154, "full_question": "La aparici\u00f3n de fiebre elevada de 39\u00b0C en un lactante de 10 meses que cede bruscamente despu\u00e9s de 3-5 d\u00edas, seguido de un exantema morbiliforme cefalocaudal con enantema constituido por p\u00e1pulas rojizas en paladar, y que se resuelve en una semana, asociado a buen estado general, suele estar producido por:", "full_answer": "La pregunta de enfermedades exantem\u00e1ticas que tanto gustan a los pediatras y a los que ponen el examen MIR. Por el cuadro de fiebre alta de varios d\u00edas que cede s\u00fabitamente con aparici\u00f3n de un rash estamos ante un exantema s\u00fabito, causado por el Herpes virus tipo 6 (acordaos de la regla mnemot\u00e9cnica de \u201cHexantema s\u00fabito\u201d). El parvovirus B19 es el causante del eritema infeccioso o \u201cquinta enfermedad\u201d, cuyo exantema caracter\u00edstico es en mejillas, dando aspecto de cara abofeteada. El coxackie A16 produce la herpangina y tambi\u00e9n la enfermedad boca-mano-pie, que presenta un exantema inicialmente macular, que no respeta palmas ni plantas y que evoluciona a ves\u00edculas que estallan. Por \u00faltimo, la infecci\u00f3n por el VEB produce un s\u00edndrome mononucle\u00f3sico que cursa con fiebre alta de varios d\u00edas de evoluci\u00f3n, con eritema far\u00edngeo y exudados gris\u00e1ceos en las am\u00edgdalas. Puede presentar si se dan penicilinas para tratarlo (al confundirlo con una faringoamigdalitis estreptoc\u00f3cica) un exantema generalizado. Adem\u00e1s, tambi\u00e9n cursa con fatiga, inflamaci\u00f3n hep\u00e1tica con aumento de transaminasas y esplenomegalia.", "type": "PEDIATR\u00cdA", "options": {"1": "Parvovirus B19.", "2": "Herpes virus tipo 6.", "3": "Virus Coxackie A16.", "4": "Primoinfecci\u00f3n por virus de Epstein-Barr.", "5": NaN}, "correct_option": 2, "explanations": {"1": {"exist": true, "char_ranges": [[325, 482]], "word_ranges": [[56, 79]], "text": "El parvovirus B19 es el causante del eritema infeccioso o \u201cquinta enfermedad\u201d, cuyo exantema caracter\u00edstico es en mejillas, dando aspecto de cara abofeteada."}, "2": {"exist": true, "char_ranges": [[108, 324]], "word_ranges": [[19, 56]], "text": "Por el cuadro de fiebre alta de varios d\u00edas que cede s\u00fabitamente con aparici\u00f3n de un rash estamos ante un exantema s\u00fabito, causado por el Herpes virus tipo 6 (acordaos de la regla mnemot\u00e9cnica de \u201cHexantema s\u00fabito\u201d)."}, "3": {"exist": true, "char_ranges": [[483, 682]], "word_ranges": [[79, 109]], "text": "El coxackie A16 produce la herpangina y tambi\u00e9n la enfermedad boca-mano-pie, que presenta un exantema inicialmente macular, que no respeta palmas ni plantas y que evoluciona a ves\u00edculas que estallan."}, "4": {"exist": true, "char_ranges": [[683, 868]], "word_ranges": [[109, 139]], "text": "Por \u00faltimo, la infecci\u00f3n por el VEB produce un s\u00edndrome mononucle\u00f3sico que cursa con fiebre alta de varios d\u00edas de evoluci\u00f3n, con eritema far\u00edngeo y exudados gris\u00e1ceos en las am\u00edgdalas."}, "5": {"exist": false, "char_ranges": [], "word_ranges": [], "text": ""}}} {"id": 239, "year": 2014, "question_id_specific": 146, "full_question": "Un hombre de 80 a\u00f1os es ingresado por un cuadro brusco de afasia y hemiparesia derecha. Como antecedentes destaca, hipertensi\u00f3n, bien controlada con dieta y deterioro cognitivo en el \u00faltimo a\u00f1o en estudio por su neur\u00f3logo. La TC craneal de urgencias demuestra un hematoma lobar frontal izquierdo sin captaci\u00f3n de contraste. \u00bfCu\u00e1l es la causa m\u00e1s probable del hematoma?", "full_answer": "La angiopat\u00eda amiloide es la causa m\u00e1s frecuente de hemorragia espont\u00e1nea no hipertensiva en pacientes ancianos, y suele ser de localizaci\u00f3n lobar (como en el caso). A menudo aparecen asociados a la enfermedad de Alzheimer (Manual CTO). La respuesta que m\u00e1s dudas puede general es la 2, la hipertensi\u00f3n, pero las localizaciones m\u00e1s frecuentes son: putamen, t\u00e1lamo, protuberancia y cerebelo. Por lo tanto la mayor\u00eda de las veces se trata de hemorragias profundas (Manual CTO). Adem\u00e1s, como pista nos dice que el paciente tiene hipertensi\u00f3n bien controlada sin f\u00e1rmacos, aunque este dato no servir\u00eda para descartar la respuesta. La respuesta 1, una malformaci\u00f3n es t\u00edpica de pacientes j\u00f3venes, raro en gente mayor. Adem\u00e1s disminuye la posibilidad de no hallarse una malformaci\u00f3n con la realizaci\u00f3n del TC con contraste. La respuesta 4, un tumor cerebral, no parece probable ya que no se ve la masa en el TC, y como en el caso anterior, en m\u00e1s dificil no encontrar dicha lesi\u00f3n tras la administraci\u00f3n de contraste. La respuesta 3, no parece correcta ya que la vasculitis es una entidad rara y m\u00e1s en gente mayor.", "type": "NEUROLOG\u00cdA", "options": {"1": "Malformaci\u00f3n arteriovenosa enmascarada por el hematoma agudo.", "2": "Hipertensi\u00f3n arterial cr\u00f3nica.", "3": "Vasculitis aislada del sistema nervioso.", "4": "Tumor cerebral.", "5": "Angiopat\u00eda cerebral amiloidea (angiopat\u00eda congof\u00edlica)."}, "correct_option": 5, "explanations": {"1": {"exist": true, "char_ranges": [[627, 712]], "word_ranges": [[98, 112]], "text": "La respuesta 1, una malformaci\u00f3n es t\u00edpica de pacientes j\u00f3venes, raro en gente mayor."}, "2": {"exist": true, "char_ranges": [[309, 475]], "word_ranges": [[50, 75]], "text": "las localizaciones m\u00e1s frecuentes son: putamen, t\u00e1lamo, protuberancia y cerebelo. Por lo tanto la mayor\u00eda de las veces se trata de hemorragias profundas (Manual CTO)."}, "3": {"exist": true, "char_ranges": [[1012, 1109]], "word_ranges": [[165, 184]], "text": "La respuesta 3, no parece correcta ya que la vasculitis es una entidad rara y m\u00e1s en gente mayor."}, "4": {"exist": true, "char_ranges": [[818, 1011]], "word_ranges": [[128, 165]], "text": "La respuesta 4, un tumor cerebral, no parece probable ya que no se ve la masa en el TC, y como en el caso anterior, en m\u00e1s dificil no encontrar dicha lesi\u00f3n tras la administraci\u00f3n de contraste."}, "5": {"exist": true, "char_ranges": [[0, 236]], "word_ranges": [[0, 37]], "text": "La angiopat\u00eda amiloide es la causa m\u00e1s frecuente de hemorragia espont\u00e1nea no hipertensiva en pacientes ancianos, y suele ser de localizaci\u00f3n lobar (como en el caso). A menudo aparecen asociados a la enfermedad de Alzheimer (Manual CTO)."}}} {"id": 525, "year": 2021, "question_id_specific": 53, "full_question": "Mujer de 42 a\u00f1os que acude a consulta de su m\u00e9dico de familia por gonalgia. De forma oportunista se aprovecha la consulta para valorar estilos de vida, especialmente el tabaquismo. Si se quiere seguir la estrategia de educaci\u00f3n para la salud basada en el modelo de las cinco \u201caes\u201d, \u00bfcu\u00e1l de las siguientes NO se incluye en esta estrategia?:", "full_answer": "Las 5A resume las actividades que deber\u00eda realizar el personal sanitario durante la intervenci\u00f3n breve con el paciente para dejar de fumar y son: averiguar, aconsejar, analizar, ayudar, acompa\u00f1ar o acordar. Por tanto la que no se incluye en la estrategia ser\u00eda la opci\u00f3n 2: aumentar.", "type": "MEDICINA PREVENTIVA", "options": {"1": "Averiguar: preguntar sobre los factores y las conductas de riesgo (preguntar a la paciente si fuma).", "2": "Aumentar: incrementar la percepci\u00f3n de riesgo para facilitar el cambio (explicar las consecuencias del tabaquismo).", "3": "Aconsejar: dar consejos claros, espec\u00edficos y personalizados (aconsejar el abandono del tabaco).", "4": "Acordar: pactar colaborativamente los objetivos de cambio (valorar la disponibilidad para hacer un intento de dejar de fumar).", "5": NaN}, "correct_option": 2, "explanations": {"1": {"exist": false, "char_ranges": [], "word_ranges": [], "text": ""}, "2": {"exist": true, "char_ranges": [[0, 283]], "word_ranges": [[0, 46]], "text": "Las 5A resume las actividades que deber\u00eda realizar el personal sanitario durante la intervenci\u00f3n breve con el paciente para dejar de fumar y son: averiguar, aconsejar, analizar, ayudar, acompa\u00f1ar o acordar. Por tanto la que no se incluye en la estrategia ser\u00eda la opci\u00f3n 2: aumentar."}, "3": {"exist": false, "char_ranges": [], "word_ranges": [], "text": ""}, "4": {"exist": false, "char_ranges": [], "word_ranges": [], "text": ""}, "5": {"exist": false, "char_ranges": [], "word_ranges": [], "text": ""}}} {"id": 287, "year": 2016, "question_id_specific": 64, "full_question": "Paciente de 65 a\u00f1os que presenta disnea progresiva de 5 d\u00edas de evoluci\u00f3n hasta hacerse de reposo, ortopnea de tres almohadas y episodios de disnea parox\u00edstica nocturna. A la auscultaci\u00f3n destaca crepitantes bilaterales, soplo holosist\u00f3lico irradiado a axila y ritmo de galope por tercer y cuarto ruido. Se\u00f1ale la afirmaci\u00f3n lNCORRECTA:", "full_answer": "El cuarto ruido suele aparecer cuando existe un cierto grado de estenosis valvular.", "type": "CARDIOLOG\u00cdA Y CIRUG\u00cdA VASCULAR", "options": {"1": "El tercer ruido coincide con la fase de llenado r\u00e1pido de la di\u00e1stole ventricular del ciclo cardiaco.", "2": "El diagn\u00f3stico m\u00e1s probable es el de insuficiencia cardiaca.", "3": "El soplo holosist\u00f3lico puede corresponder a una insuficiencia mitral.", "4": "El cuarto ruido suele aparecer cuando existe un cierto grado de estenosis valvular.", "5": NaN}, "correct_option": 4, "explanations": {"1": {"exist": false, "char_ranges": [], "word_ranges": [], "text": ""}, "2": {"exist": false, "char_ranges": [], "word_ranges": [], "text": ""}, "3": {"exist": false, "char_ranges": [], "word_ranges": [], "text": ""}, "4": {"exist": true, "char_ranges": [[0, 83]], "word_ranges": [[0, 13]], "text": "El cuarto ruido suele aparecer cuando existe un cierto grado de estenosis valvular."}, "5": {"exist": false, "char_ranges": [], "word_ranges": [], "text": ""}}} {"id": 472, "year": 2020, "question_id_specific": 140, "full_question": "Mujer de 65 a\u00f1os, obesa, que sufre una ca\u00edda sobre la mano con el codo en extensi\u00f3n. Presenta dolor en el brazo con tumefacci\u00f3n e impotencia funcional del mismo y con imposibilidad para realizar la extensi\u00f3n de la mu\u00f1eca y dedos. Lo m\u00e1s probable es que presente:", "full_answer": "Hay que tener en cuenta todos los datos cl\u00ednicos que nos aportan: despu\u00e9s de traumatismo tenemos dolor en el brazo con afectaci\u00f3n de partes blandas y la imposibilidad para la extensi\u00f3n de mu\u00f1eca y dedos. La \u00fanica opci\u00f3n que engloba todos los datos es la 4. La 1 no explica la afectaci\u00f3n de mu\u00f1eca y dedos. La luxaci\u00f3n de codo no explica la falta de movilidad de los dedos as\u00ed como la fractura diafisaria de h\u00famero y doble de antebrazo. \u201cEl mecanismo directo es posible, as\u00ed como tambi\u00e9n lo es la transmisi\u00f3n indirecta de fuerzas desde el codo y la mano\u2026 la afectaci\u00f3n del nervio radial es extremadamente frecuente, alcanzando una incidencia del 10-18% de los casos.\u201d La fractura-luxaci\u00f3n de h\u00famero proximal tendr\u00eda mayor afectaci\u00f3n en el hombro, la luxaci\u00f3n de codo en el codo, y no hay ning\u00fan dato que sugiera fractura doble del antebrazo.", "type": "CIRUG\u00cdA ORTOP\u00c9DICA Y TRAUMATOLOG\u00cdA", "options": {"1": "Fractura luxaci\u00f3n de h\u00famero proximal.", "2": "Luxaci\u00f3n de codo.", "3": "Fractura diafisaria de h\u00famero asociada a fractura doble de antebrazo.", "4": "Fractura diafisaria de h\u00famero con lesi\u00f3n del nervio radial.", "5": NaN}, "correct_option": 4, "explanations": {"1": {"exist": true, "char_ranges": [[257, 305]], "word_ranges": [[46, 56]], "text": "La 1 no explica la afectaci\u00f3n de mu\u00f1eca y dedos."}, "2": {"exist": true, "char_ranges": [[306, 435]], "word_ranges": [[56, 80]], "text": "La luxaci\u00f3n de codo no explica la falta de movilidad de los dedos as\u00ed como la fractura diafisaria de h\u00famero y doble de antebrazo."}, "3": {"exist": true, "char_ranges": [[667, 840]], "word_ranges": [[117, 146]], "text": "La fractura-luxaci\u00f3n de h\u00famero proximal tendr\u00eda mayor afectaci\u00f3n en el hombro, la luxaci\u00f3n de codo en el codo, y no hay ning\u00fan dato que sugiera fractura doble del antebrazo."}, "4": {"exist": true, "char_ranges": [[66, 256]], "word_ranges": [[12, 46]], "text": "despu\u00e9s de traumatismo tenemos dolor en el brazo con afectaci\u00f3n de partes blandas y la imposibilidad para la extensi\u00f3n de mu\u00f1eca y dedos. La \u00fanica opci\u00f3n que engloba todos los datos es la 4."}, "5": {"exist": false, "char_ranges": [], "word_ranges": [], "text": ""}}} {"id": 173, "year": 2013, "question_id_specific": 51, "full_question": "Acude a revisi\u00f3n a la consulta de alergia una mujer de 53 a\u00f1os de edad con asma bronquial. Refiere repetidas agudizaciones con cl\u00ednica nocturna, utilizaci\u00f3n frecuente de medicaci\u00f3n de rescate y disnea de esfuerzo al caminar en llano. Tiene una espirometr\u00eda que muestra un cociente prebroncodilatador del FEV1/FVC del 60% y un FEV1 del 55%. Las pruebas cut\u00e1neas son positivas para \u00e1caros y la IgE total de 150 UI/ml. La paciente est\u00e1 siendo tratada con una combinaci\u00f3n de salmeterol/budesonida a dosis altas (50/500 mg: 2 inhalaciones dos veces al d\u00eda), prednisona oral de mantenimiento (10 mgld\u00eda) y teofilina. De las siguientes opciones, \u00bfcu\u00e1l es la actitud terap\u00e9utica m\u00e1s aconsejable?", "full_answer": "La indicaci\u00f3n de omalizumab en una paciente como la referida, es una indicaci\u00f3n correcta, aunque dentro del asma grave de dificil control, hay otras medidas previas que no se contemplan y que hay que tener siempre presentes antes de recurrir a este tratamiento. Es preceptivo asegurar en primer lugar que es verdaderamente un asma y no otra enfermedad que simule asma, segundo que no tenga complicaciones que empeoren la evoluci\u00f3n como podr\u00eda ser la presencia de reflujo gastroesof\u00e1gico, bronquiectasias, presencia de g\u00e9rmenes oportunistas dado el tratamiento cr\u00f3nico con esteroides, etc; y en tercer lugar hay que confirmar que la paciente cumple los tratamientos de forma correcta, sobre todo el tratamiento inhalado, etc. Una vez chequeado todo lo anterior, ser\u00eda el momento de iniciar un tratamiento de prueba con omalizumab.", "type": "NEUMOLOG\u00cdA", "options": {"1": "A\u00f1adir sulfato de magnesio.", "2": "Aumentar la dosis de prednisona a 30 mg/d\u00eda.", "3": "A\u00f1adir omalizumab.", "4": "Pautar tratamiento nebulizado en el domicilio.", "5": "Cambiar a una combinaci\u00f3n con dosis altas de budesonida y formoterol."}, "correct_option": 3, "explanations": {"1": {"exist": false, "char_ranges": [], "word_ranges": [], "text": ""}, "2": {"exist": false, "char_ranges": [], "word_ranges": [], "text": ""}, "3": {"exist": true, "char_ranges": [[0, 88]], "word_ranges": [[0, 14]], "text": "La indicaci\u00f3n de omalizumab en una paciente como la referida, es una indicaci\u00f3n correcta,"}, "4": {"exist": false, "char_ranges": [], "word_ranges": [], "text": ""}, "5": {"exist": false, "char_ranges": [], "word_ranges": [], "text": ""}}} {"id": 480, "year": 2020, "question_id_specific": 171, "full_question": "Mujer de 67 a\u00f1os con antecedentes de dislipemia, que acude a urgencias por un cuadro de disuria y poliaquiuria seguido de fiebre, escalofr\u00edos y deterioro del estado general. A su llegada impresiona de gravedad y est\u00e1 taquic\u00e1rdica, taquipneica, con tensi\u00f3n arterial 60/40 mmHg y temperatura de 39\u00b0C. \u00bfQu\u00e9 medida de entre las siguientes NO estar\u00eda incluida en el manejo INICIAL?:", "full_answer": "Entre las opciones que se presentan, la \u00fanica medida no incluida en las gu\u00edas de la Surviving Sepsis Campaign sobre el manejo de la sepsis y el shock s\u00e9ptico (como la paciente ante la que nos encontramos) es la administraci\u00f3n de dobutamina IV. El procedimiento adecuado consistir\u00eda en administraci\u00f3n de fluidos, extracci\u00f3n de hemocultivos, medici\u00f3n del lactato s\u00e9rico y administraci\u00f3n precoz de antibioterapia de amplio espectro dirigida al foco s\u00e9ptico que sospechemos. En caso de no obtener presi\u00f3n arterial media para una adecuada perfusi\u00f3n de los \u00f3rganos, iniciar\u00edamos perfusi\u00f3n de noradrenalina, como se ha comentado en la pregunta 51.", "type": "CUIDADOS CR\u00cdTICOS", "options": {"1": "Perfusi\u00f3n intravenosa de dobutamina.", "2": "Medici\u00f3n de lactato s\u00e9rico.", "3": "Extracci\u00f3n de hemocultivos.", "4": "Administraci\u00f3n de fluidos.", "5": NaN}, "correct_option": 1, "explanations": {"1": {"exist": true, "char_ranges": [[0, 243]], "word_ranges": [[0, 43]], "text": "Entre las opciones que se presentan, la \u00fanica medida no incluida en las gu\u00edas de la Surviving Sepsis Campaign sobre el manejo de la sepsis y el shock s\u00e9ptico (como la paciente ante la que nos encontramos) es la administraci\u00f3n de dobutamina IV."}, "2": {"exist": false, "char_ranges": [], "word_ranges": [], "text": ""}, "3": {"exist": false, "char_ranges": [], "word_ranges": [], "text": ""}, "4": {"exist": false, "char_ranges": [], "word_ranges": [], "text": ""}, "5": {"exist": false, "char_ranges": [], "word_ranges": [], "text": ""}}} {"id": 58, "year": 2011, "question_id_specific": 199, "full_question": "Una mujer (Consultante) de 31 a\u00f1os de edad y asintom\u00e1tica, presenta gestaci\u00f3n de 10 semanas seg\u00fan estudio ecogr\u00e1fico (prim\u00edpara). Su hermano menor (caso \u00edndice) de 26 a\u00f1os est\u00e1 afecto de ataxia y diagnosticado gen\u00e9ticamente como portador heterocigoto de una mutaci\u00f3n expansiva de 70 repeticiones CAG en el gen SCA3 (cromosoma 14). La Consultante se deriva a la consulta de consejo gen\u00e9tico donde se valora una posible biopsia de corion para estudiar el genotipo fetal. \u00bfest\u00e1 indicado este procedimiento invasivo como diagn\u00f3stico pre-natal (DPN) en este caso?", "full_answer": "La SCA3 es la Ataxia espinocerebelar tipo 3 tambi\u00e9n llamada enfermedad de Machado-Joseph. Se produce por mutaciones en el gen ATXN3 por el mecanismo de expansi\u00f3n de repetici\u00f3n de tripletes CAG. Los afectos tienen entre 52 y 86 repeticiones del triplete CAG en heterocigosis con penetrancia completa. El que tiene la mutaci\u00f3n en heterocigosis padece la enfermedad pues su patr\u00f3n de herecia es autos\u00f3mico dominante. En este caso cl\u00ednico el que padece la enfermedad y tiene el gen mutado en heterocigosis es el hermano (de 26 a\u00f1os) de la gestante. Pienso que antes de hacer la biopsia de corion habr\u00eda que hacerle el estudio a la mujer gestante para conocer cual es su estatus en cuanto al gen ATXN3. Ella tiene un 50% por ciento de posibilidades de portar la mutaci\u00f3n y de padecer la enfermedad, pero al tener 31 a\u00f1os pienso que ya deb\u00eda haber tenido s\u00edntomas. No obstante, yo le har\u00eda el test gen\u00e9tico a la gestante para curarme en salud. Si saliera sin mutaci\u00f3n, no habr\u00eda m\u00e1s que hacer porque ya no se transmitir\u00eda la enfermedad. Si saliera portadora de la mutaci\u00f3n, entonces le har\u00eda la biopsia de corion. La respuestas 1 y 2 son ambas ciertas, pero creo que la respuesta 1 es m\u00e1s cierta que la 2.", "type": "GEN\u00c9TICA", "options": {"1": "Est\u00e1 indicado tras estudiar el genotipo de la Consultante y \u00fanicamente si \u00e9ste es heterocigoto.", "2": "No est\u00e1 indicado, puesto que la ataxia SCA3 es de penetrancia completa y la Consultante est\u00e1 asintom\u00e1tica y por tanto no ha heredado la mutaci\u00f3n.", "3": "Puede estar indicado en el siguiente embarazo, tras estudiar el genotipo del primer hijo y detectar la mutaci\u00f3n en el mismo.", "4": "La ataxia SCA3 es de herencia recesiva, por lo que no existe riesgo apreciable de transmisi\u00f3n de la enfermedad y el DPN no est\u00e1 indicado.", "5": "Est\u00e1 indicado sea cual fuere el genotipo de la Consultante, pues la ataxia SCA3 es de herencia materna (transmitida por las mujeres)."}, "correct_option": 1, "explanations": {"1": {"exist": true, "char_ranges": [[414, 858]], "word_ranges": [[65, 148]], "text": "En este caso cl\u00ednico el que padece la enfermedad y tiene el gen mutado en heterocigosis es el hermano (de 26 a\u00f1os) de la gestante. Pienso que antes de hacer la biopsia de corion habr\u00eda que hacerle el estudio a la mujer gestante para conocer cual es su estatus en cuanto al gen ATXN3. Ella tiene un 50% por ciento de posibilidades de portar la mutaci\u00f3n y de padecer la enfermedad, pero al tener 31 a\u00f1os pienso que ya deb\u00eda haber tenido s\u00edntomas."}, "2": {"exist": false, "char_ranges": [], "word_ranges": [], "text": ""}, "3": {"exist": false, "char_ranges": [], "word_ranges": [], "text": ""}, "4": {"exist": false, "char_ranges": [], "word_ranges": [], "text": ""}, "5": {"exist": false, "char_ranges": [], "word_ranges": [], "text": ""}}} {"id": 51, "year": 2011, "question_id_specific": 230, "full_question": "Embarazada de 37 semanas con ant\u00edgeno de superficie del virus de la hepatitis B positivo. Consulta sobre la pauta a seguir con el reci\u00e9n nacido y si puede lactar. \u00bfQu\u00e9 consejo le parece m\u00e1s adecuado?", "full_answer": "La respuesta correcta es la 4. A los ni\u00f1os hijos de madres portadoras del VHB se les debe administrar en las primeras 24 horas de vida la vacuna antihepatitisB y la inmunoglobulina en diferentes lugares de punci\u00f3n. Aunque el virus se excreta en la leche materna, el riesgo de contagio por esta v\u00eda es muy bajo por lo que actualmente no es una contraindicaci\u00f3n para la lactancia materna.", "type": "PEDIATR\u00cdA", "options": {"1": "Administrar vacuna frente a hepatitis B nada m\u00e1s nacer. Lactancia artificial.", "2": "Administrar vacuna e inmunoglobulinas al nacer. Lactancia materna a partir del mes.", "3": "Inmunoglobulinas al nacer y lactancia artificial.", "4": "Vacuna e inmunoglobulinas al nacer. Alimentaci\u00f3n materna.", "5": "Alimentaci\u00f3n artificial y aislamiento durante 4 semanas."}, "correct_option": 4, "explanations": {"1": {"exist": false, "char_ranges": [], "word_ranges": [], "text": ""}, "2": {"exist": false, "char_ranges": [], "word_ranges": [], "text": ""}, "3": {"exist": false, "char_ranges": [], "word_ranges": [], "text": ""}, "4": {"exist": true, "char_ranges": [[31, 214]], "word_ranges": [[6, 37]], "text": "A los ni\u00f1os hijos de madres portadoras del VHB se les debe administrar en las primeras 24 horas de vida la vacuna antihepatitisB y la inmunoglobulina en diferentes lugares de punci\u00f3n."}, "5": {"exist": false, "char_ranges": [], "word_ranges": [], "text": ""}}} {"id": 184, "year": 2013, "question_id_specific": 65, "full_question": "Remiten a su consulta a un paciente con obesidad (\u00edndice de masa corporal 38). En la historia cl\u00ednica el paciente refiere que presenta obesidad desde los 17 a\u00f1os (en la actualidad tienen 36 a\u00f1os), habiendo realizado m\u00faltiples dietas con p\u00e9rdidas ponderales que oscilan entre 5 y 10 kg pero que posteriormente ha recuperado el peso. En la actualidad el paciente realiza una dieta de 1500 Kcal (auto administrada y no restringida en grasas) con un buen seguimiento de dicha dieta, realizando una hora de ejercicio aer\u00f3bico durante 4 d\u00edas a la semana. Ha perdido 3 kg pero precisa una p\u00e9rdida a\u00f1adida de 7 kg m\u00e1s. Ante la posibilidad de a\u00f1adir un f\u00e1rmaco frente a la obesidad, \u00bfcu\u00e1l utilizar\u00eda para disminuir la absorci\u00f3n de grasas?:", "full_answer": "El Orlistat act\u00faa disminuyendo la absorci\u00f3n de grasas; actualmente es el \u00fanico f\u00e1rmaco aprobado para el uso de la obesidad.", "type": "ENDOCRINOLOG\u00cdA", "options": {"1": "Orlistat.", "2": "Topiramato.", "3": "Sibutramina.", "4": "Liraglutida.", "5": "Metformina."}, "correct_option": 1, "explanations": {"1": {"exist": true, "char_ranges": [[0, 123]], "word_ranges": [[0, 20]], "text": "El Orlistat act\u00faa disminuyendo la absorci\u00f3n de grasas; actualmente es el \u00fanico f\u00e1rmaco aprobado para el uso de la obesidad."}, "2": {"exist": false, "char_ranges": [], "word_ranges": [], "text": ""}, "3": {"exist": false, "char_ranges": [], "word_ranges": [], "text": ""}, "4": {"exist": false, "char_ranges": [], "word_ranges": [], "text": ""}, "5": {"exist": false, "char_ranges": [], "word_ranges": [], "text": ""}}} {"id": 295, "year": 2016, "question_id_specific": 75, "full_question": "Indique la situaci\u00f3n cl\u00ednica que, en relaci\u00f3n con la infecci\u00f3n por virus de la hepatitis B, presenta un paciente de 5 a\u00f1os procedente de Nigeria, con exploraci\u00f3n f\u00edsica normal y con la siguiente serolog\u00eda frente a hepatitis B: HBsAg + /ANTI- HBs \u2013 / HBeAg \u2013 / ANTI- Hbe + / ANTI- Hbc IgM \u2013 / ANTI- Hbc IgG +/ DNA VHB +:", "full_answer": "Pregunta en la que teniendo una tabla en la cabeza sale sola descartando opciones: Opci\u00f3n 1: La infecci\u00f3n aguda cursar\u00eda con HBeAg + y ANTI-HBe \u2013 . Adem\u00e1s el ANTIHBc seria tipo IgM (solo por este \u00faltimo dato, tendr\u00edamos que descartarla de manera directa). Opci\u00f3n 2: la correcta. Cumple con todas las caracter\u00edsticas. Opci\u00f3n 3: El paciente vacunado no puede tener DNA del virus!! (Lo \u00fanico que positiviza en un vacunado es el AntiBs. Tipo IgG). Opci\u00f3n 4: La opci\u00f3n que podr\u00eda ofrecer m\u00e1s dudas. En el portador asintom\u00e1tico (portador cr\u00f3nico /seroconvertido) la diferencia es que no hay DNA viral.", "type": "GEN\u00c9TICA E INMUNOLOG\u00cdA", "options": {"1": "Infecci\u00f3n aguda.", "2": "Infecci\u00f3n cr\u00f3nica.", "3": "Paciente vacunado.", "4": "Portador asintom\u00e1tico.", "5": NaN}, "correct_option": 2, "explanations": {"1": {"exist": true, "char_ranges": [[83, 255]], "word_ranges": [[14, 44]], "text": "Opci\u00f3n 1: La infecci\u00f3n aguda cursar\u00eda con HBeAg + y ANTI-HBe \u2013 . Adem\u00e1s el ANTIHBc seria tipo IgM (solo por este \u00faltimo dato, tendr\u00edamos que descartarla de manera directa)."}, "2": {"exist": true, "char_ranges": [[256, 316]], "word_ranges": [[44, 53]], "text": "Opci\u00f3n 2: la correcta. Cumple con todas las caracter\u00edsticas."}, "3": {"exist": true, "char_ranges": [[317, 378]], "word_ranges": [[53, 64]], "text": "Opci\u00f3n 3: El paciente vacunado no puede tener DNA del virus!!"}, "4": {"exist": true, "char_ranges": [[444, 595]], "word_ranges": [[76, 100]], "text": "Opci\u00f3n 4: La opci\u00f3n que podr\u00eda ofrecer m\u00e1s dudas. En el portador asintom\u00e1tico (portador cr\u00f3nico /seroconvertido) la diferencia es que no hay DNA viral."}, "5": {"exist": false, "char_ranges": [], "word_ranges": [], "text": ""}}} {"id": 559, "year": 2022, "question_id_specific": 160, "full_question": "Mujer de 35 a\u00f1os, que toma anticonceptivos, acude a urgencias con s\u00edndrome febril y parestesias en hemicuerpo izquierdo. En la anal\u00edtica de sangre se observa Hb 7,5 g/dl, plaquetas 7.000/microl, leucocitos normales con recuento diferencial normal, LDH 1.200 UI/l, reticulocitos 10 % (normal 0,5-2 %), haptoglobina en suero indetectable, test de Coombs directo negativo y frotis de sangre perif\u00e9rica con esquistocitos. Coagulaci\u00f3n (tiempo de protrombina y APTT) normal. \u00bfCu\u00e1l de los siguientes es el diagn\u00f3stico m\u00e1s probable?:", "full_answer": "El temor de todos los hemat\u00f3logos de guardia\u2026 que te llegue una PTT. Porque, aunque infrecuente, es la mayor urgencia hematol\u00f3gica. El caso cl\u00ednico es bastante t\u00edpico. Una persona joven con fiebre y s\u00edntomas neurol\u00f3gicos que le hacen acudir a urgencias (va a la carrera). Anal\u00edticamente destaca una anemia con datos de hem\u00f3lisis (LDH y retis aumentados, presencia de esquistocitos en sangre perif\u00e9rica). y una trombopenia grave. Adem\u00e1s, la coagulaci\u00f3n es normal. Para que lo distingas del resto de cosas (aunque no deber\u00edais dudar\u2026) os dice que el coombs es negativo. Como recordatorio: TRATAMIENTO URGENTE: RECAMBIOS PLASM\u00c1TICOS. La confirmaci\u00f3n diagn\u00f3stica es el d\u00e9ficit de ADAMTS 13, pero se inicia el tratamiento antes de tener ese valor, que no se suele realizar de manera urgente. Y, como todos sabemos, estas cosas suelen llegar por la noche y/o fines de semana, para poner las cosas m\u00e1s dif\u00edciles\u2026", "type": "HEMATOLOGIA", "options": {"1": "Anemia hemol\u00edtica autoinmune.", "2": "P\u00farpura tromb\u00f3tica trombocitop\u00e9nica.", "3": "S\u00edndrome de Evans (anemia hemol\u00edtica y trombopenia inmunes).", "4": "Trombocitopenia inmune idiop\u00e1tica.", "5": NaN}, "correct_option": 2, "explanations": {"1": {"exist": false, "char_ranges": [], "word_ranges": [], "text": ""}, "2": {"exist": true, "char_ranges": [[133, 568]], "word_ranges": [[21, 91]], "text": "El caso cl\u00ednico es bastante t\u00edpico. Una persona joven con fiebre y s\u00edntomas neurol\u00f3gicos que le hacen acudir a urgencias (va a la carrera). Anal\u00edticamente destaca una anemia con datos de hem\u00f3lisis (LDH y retis aumentados, presencia de esquistocitos en sangre perif\u00e9rica). y una trombopenia grave. Adem\u00e1s, la coagulaci\u00f3n es normal. Para que lo distingas del resto de cosas (aunque no deber\u00edais dudar\u2026) os dice que el coombs es negativo."}, "3": {"exist": false, "char_ranges": [], "word_ranges": [], "text": ""}, "4": {"exist": false, "char_ranges": [], "word_ranges": [], "text": ""}, "5": {"exist": false, "char_ranges": [], "word_ranges": [], "text": ""}}} {"id": 558, "year": 2022, "question_id_specific": 160, "full_question": "Mujer de 35 a\u00f1os, que toma anticonceptivos, acude a urgencias con s\u00edndrome febril y parestesias en hemicuerpo izquierdo. En la anal\u00edtica de sangre se observa Hb 7,5 g/dl, plaquetas 7.000/microl, leucocitos normales con recuento diferencial normal, LDH 1.200 UI/l, reticulocitos 10 % (normal 0,5-2 %), haptoglobina en suero indetectable, test de Coombs directo negativo y frotis de sangre perif\u00e9rica con esquistocitos. Coagulaci\u00f3n (tiempo de protrombina y APTT) normal. \u00bfCu\u00e1l de los siguientes es el diagn\u00f3stico m\u00e1s probable?:", "full_answer": "La paciente presenta criterios para P\u00farpura tromb\u00f3tica Trombocitop\u00e9nica (PTT): 1. Alteraci\u00f3n neurol\u00f3gica. 2. Sindrome febril (fiebre). 3. Anemia hemol\u00edtica coombs negativo migroangiop\u00e1tica (presencia de esquistocitos). 4. Trombopenia.", "type": "HEMATOLOGIA", "options": {"1": "Anemia hemol\u00edtica autoinmune.", "2": "P\u00farpura tromb\u00f3tica trombocitop\u00e9nica.", "3": "S\u00edndrome de Evans (anemia hemol\u00edtica y trombopenia inmunes).", "4": "Trombocitopenia inmune idiop\u00e1tica.", "5": NaN}, "correct_option": 2, "explanations": {"1": {"exist": false, "char_ranges": [], "word_ranges": [], "text": ""}, "2": {"exist": true, "char_ranges": [[0, 234]], "word_ranges": [[0, 27]], "text": "La paciente presenta criterios para P\u00farpura tromb\u00f3tica Trombocitop\u00e9nica (PTT): 1. Alteraci\u00f3n neurol\u00f3gica. 2. Sindrome febril (fiebre). 3. Anemia hemol\u00edtica coombs negativo migroangiop\u00e1tica (presencia de esquistocitos). 4. Trombopenia."}, "3": {"exist": false, "char_ranges": [], "word_ranges": [], "text": ""}, "4": {"exist": false, "char_ranges": [], "word_ranges": [], "text": ""}, "5": {"exist": false, "char_ranges": [], "word_ranges": [], "text": ""}}} {"id": 227, "year": 2014, "question_id_specific": 72, "full_question": "Mujer de 45 a\u00f1os con antecedentes de soplo cardiaco detectado en edad pedi\u00e1trica. Ingresa en el servicio de Urgencias por cuadro de palpitaciones, cansancio f\u00e1cil y edemas maleolares. La exploraci\u00f3n f\u00edsica pone de manifiesto ausencia de cianosis. Saturaci\u00f3n de ox\u00edgeno por pulsiox\u00edmetro 97% TA 120/80 mmHg. Ritmo cardiaco irregular a 100 lpm. Soplo sist\u00f3lico eyectivo (2/6) en foco pulmonar. 2\u00ba ruido desdoblado, amplio y fijo. No estertores. Ligera hepatomeglaia (2-3 cm). Ligeros edemas maleolares. ECG: arritmia completa por fibrilaci\u00f3n auricular a 100 lpm. QRS +120\u00ba. Trastorno de conducci\u00f3n de la rama derecha del haz de His. \u00bfCu\u00e1l es su orientaci\u00f3n diagn\u00f3stica?", "full_answer": "Si tiene un soplo pedi\u00e1trico, pienso en una cong\u00e9nita. Hepatomegalia, edemas maleolares y BRD, algo que sobrecargue el coraz\u00f3n derecho. Descartamos pues las estenosis a\u00f3rtica y mitral. Un ductus persistente dar\u00eda cianosis: descartado. \u00bfCIV o CIA? Si me dicen que tiene FA es porque las aur\u00edculas son de talla XL, y una CIV no produce dilataci\u00f3n auricular.", "type": "CARDIOLOG\u00cdA", "options": {"1": "Comunicaci\u00f3n interventricular.", "2": "Estenosis a\u00f3rtica.", "3": "Estenosis mitral.", "4": "Comunicaci\u00f3n interauricular.", "5": "Conducto arterioso persistente."}, "correct_option": 4, "explanations": {"1": {"exist": true, "char_ranges": [[247, 355]], "word_ranges": [[36, 57]], "text": "Si me dicen que tiene FA es porque las aur\u00edculas son de talla XL, y una CIV no produce dilataci\u00f3n auricular."}, "2": {"exist": true, "char_ranges": [[0, 184]], "word_ranges": [[0, 27]], "text": "Si tiene un soplo pedi\u00e1trico, pienso en una cong\u00e9nita. Hepatomegalia, edemas maleolares y BRD, algo que sobrecargue el coraz\u00f3n derecho. Descartamos pues las estenosis a\u00f3rtica y mitral."}, "3": {"exist": true, "char_ranges": [[0, 184]], "word_ranges": [[0, 27]], "text": "Si tiene un soplo pedi\u00e1trico, pienso en una cong\u00e9nita. Hepatomegalia, edemas maleolares y BRD, algo que sobrecargue el coraz\u00f3n derecho. Descartamos pues las estenosis a\u00f3rtica y mitral."}, "4": {"exist": true, "char_ranges": [[247, 355]], "word_ranges": [[36, 57]], "text": "Si me dicen que tiene FA es porque las aur\u00edculas son de talla XL, y una CIV no produce dilataci\u00f3n auricular."}, "5": {"exist": true, "char_ranges": [[185, 234]], "word_ranges": [[27, 33]], "text": "Un ductus persistente dar\u00eda cianosis: descartado."}}} {"id": 168, "year": 2013, "question_id_specific": 86, "full_question": "\u00bfQu\u00e9 exploraci\u00f3n aconsejar\u00eda en primer lugar a un paciente de 82 a\u00f1os que refiere angina de esfuerzo y que a la auscultaci\u00f3n presenta un soplo sist\u00f3lico eyectivo de intensidad 3/6?", "full_answer": "Soplo eyectivo, nuestra primera sospecha es una estenosis a\u00f3rtica. La angina puede ser debida a la propia estenosis o a coronariopat\u00eda subyacente (probable, dada la edad). Y con ese dato, no podemos saber si la estenosis es severa. Por lo tanto, lo primero que tendremos que hacer ser\u00e1 estudiar la valvulopat\u00eda: no queremos poner al paciente a correr o sobrecargarlo con dobuta y que tenga un s\u00edncope, ni hacerle un cate alegremente sin haber realizado antes otra prueba no invasiva. Y el cardioTC, en alguien de 80 a\u00f1os, nos va a decir lo que ya sabemos: que tiene calcio hasta en las cejas.", "type": "CARDIOLOG\u00cdA Y CIRUG\u00cdA CARDIOVASCULAR", "options": {"1": "Una prueba de esfuerzo.", "2": "Una coronariografia.", "3": "Un ecocardiograma de ejercicio.", "4": "Un TAC coronario.", "5": "Un ecocardiograma-Doppler."}, "correct_option": 5, "explanations": {"1": {"exist": true, "char_ranges": [[312, 350]], "word_ranges": [[51, 58]], "text": "no queremos poner al paciente a correr"}, "2": {"exist": true, "char_ranges": [[402, 483]], "word_ranges": [[67, 80]], "text": "ni hacerle un cate alegremente sin haber realizado antes otra prueba no invasiva."}, "3": {"exist": true, "char_ranges": [[353, 400]], "word_ranges": [[59, 67]], "text": "sobrecargarlo con dobuta y que tenga un s\u00edncope,"}, "4": {"exist": true, "char_ranges": [[486, 592]], "word_ranges": [[81, 103]], "text": "el cardioTC, en alguien de 80 a\u00f1os, nos va a decir lo que ya sabemos: que tiene calcio hasta en las cejas."}, "5": {"exist": false, "char_ranges": [], "word_ranges": [], "text": ""}}} {"id": 329, "year": 2016, "question_id_specific": 84, "full_question": "En un hombre diab\u00e9tico de 70 a\u00f1os con antecedentes de cardiopat\u00eda isqu\u00e9mica, \u00bfcu\u00e1l es la diana terap\u00e9utica respecto a cifras de colesterol LDL y Hemoglobina glicosilada (Hb A1c)?", "full_answer": "Diabetes con ecv ldlc<70.", "type": "ENDOCRINOLOG\u00cdA", "options": {"1": "LDLc<15 mg/dL y Hb A1c <6.5%.", "2": "LDLc<100 mg/dL y Hb A1c <7%.", "3": "LDLc<70 mg/dL y Hb A1c <7%.", "4": "LDLc< 115mg/dL y Hb A1c <7%.", "5": NaN}, "correct_option": 3, "explanations": {"1": {"exist": false, "char_ranges": [], "word_ranges": [], "text": ""}, "2": {"exist": false, "char_ranges": [], "word_ranges": [], "text": ""}, "3": {"exist": true, "char_ranges": [[0, 25]], "word_ranges": [[0, 4]], "text": "Diabetes con ecv ldlc<70."}, "4": {"exist": false, "char_ranges": [], "word_ranges": [], "text": ""}, "5": {"exist": false, "char_ranges": [], "word_ranges": [], "text": ""}}} {"id": 447, "year": 2018, "question_id_specific": 131, "full_question": "Hombre de 25 a\u00f1os sin antecedentes conocidos de inter\u00e9s. Lleva 2 meses con orinas espumosas y cargadas, sin otra sintomatolog\u00eda. Exploraci\u00f3n f\u00edsica sin hallazgos. Orina elemental: pH 5; glucosa negativo; hemoglobina ++; prote\u00ednas ++; esterasa leucocitaria negativo; proteinuria 1,5 g/24 horas; Na urinario 60 mEq/L; K urinario 30 mEq/L; Cl urinario 100 mEq/L. Bioqu\u00edmica sangu\u00ednea: creatinina 1,6 mg/dL; urea 80 mg/dL; Na 140 mEq/L; K 3,8 mEq/L. Estudio inmunol\u00f3gico: antiDNA negativo; ANCA negativo; antiMBG negativo, factor reumatoide negativo; C3 20 mg/dL (normal 60-120); C4 10 mg/dL (normal 20-40). Se realiza biopsia renal. \u00bfCu\u00e1l de estos diagn\u00f3sticos le parece m\u00e1s probable encontrar en la biopsia?", "full_answer": "Nos presentan muchos datos anal\u00edticos pero solo nos tenemos que fijar en dos: los niveles de las fracciones C3 y C4 del complemento son bajos. De las opciones que nos dan, \u00fanicamente la glomerulonefritis membranoproliferativa cursa con hipocomplementemia (opci\u00f3n 4 correcta).", "type": "NEFROLOG\u00cdA", "options": {"1": "Glomerulonefritis membranosa.", "2": "Glomerulonefritis de cambios m\u00ednimos.", "3": "Glomerulonefritis mesangial IgA.", "4": "Glomerulonefritis membranoproliferativa.", "5": NaN}, "correct_option": 4, "explanations": {"1": {"exist": false, "char_ranges": [], "word_ranges": [], "text": ""}, "2": {"exist": false, "char_ranges": [], "word_ranges": [], "text": ""}, "3": {"exist": false, "char_ranges": [], "word_ranges": [], "text": ""}, "4": {"exist": true, "char_ranges": [[78, 275]], "word_ranges": [[13, 41]], "text": "los niveles de las fracciones C3 y C4 del complemento son bajos. De las opciones que nos dan, \u00fanicamente la glomerulonefritis membranoproliferativa cursa con hipocomplementemia (opci\u00f3n 4 correcta)."}, "5": {"exist": false, "char_ranges": [], "word_ranges": [], "text": ""}}} {"id": 72, "year": 2012, "question_id_specific": 52, "full_question": "Hombre de 72 a\u00f1os, exfumador, hipertenso y diab\u00e9tico. Antecedentes de ictus con paresia residual en brazo izquierdo hace 4 messes. Acude al servicio de Urgencias de un hospital comarcal con dolor precordial intenso de 2 horas de evoluci\u00f3n; en la exploraci\u00f3n f\u00edsica presenta piel fr\u00eda y sudorosa, tensi\u00f3n arterial de 80 mm Hg; electrocardiograma con elevaci\u00f3n marcada del segmento ST en las derivaciones V1-V6, I y aVL. \u00bfCu\u00e1l de los siguientes tratamientos de repercusi\u00f3n coronaria es el mas adecuado?", "full_answer": "En esta pregunta se conjugan varios datos importantes. Para empezar, se trata de un paciente con un IAM anterior y lateral extenso, con shock cardiog\u00e9nico. Por otro lado dentro de los antecedentes, nos dicen que el paciente tuvo un ictus hace 4 meses. Es evidente por el cuadro cl\u00ednico del paciente que est\u00e1 indicada una estrategia de reperfusi\u00f3n urgente, esto invalida las opciones 4 y 5. La cuesti\u00f3n basica es si elegir fibrinolisis (la cual puede hacerse en un hospital comarcal) o ICP (para lo cual debe ser derivado a un centro terciario) . La opci\u00f3n 2 no tiene mucho sentido, puesto que en el caso de que estuviera indicada la fibrinolisis no ser\u00eda mas que una p\u00e9rdida de tiempo realizar el traslado. El antecedente de ictus (se supone que isqu\u00e9mico) hace 4 meses es una contraindicaci\u00f3n relativa de fibrinoisis (si el tiempo es menor de 3 meses es contraindicaci\u00f3n absoluta), pero lo definitivo que inclina la balanca hacia la ICP es la extensi\u00f3n del IAM y la presencia de shock, que son indicaciones de ICP urgente. Por tanto la respuesta correcta es la 3.", "type": "ANESTESIOLOG\u00cdA, CUIDADOS CR\u00cdTICOS Y URGENCIAS", "options": {"1": "Fibrinolisis sist\u00e9mica de forma inmediata en el hospital comarcal.", "2": "Traslado inmediato a un centro terciario para tratamiento con fibrinolisis.", "3": "Traslado inmediato a un centro terciario para procedimiento de revascularizaci\u00f3n coronaria percut\u00e1nea.", "4": "Estabilizaci\u00f3n hemodin\u00e1mica y procedimiento de revascularizaci\u00f3n diferido.", "5": "Tratamiento inmediato con fondaparinux y abciximab."}, "correct_option": 3, "explanations": {"1": {"exist": true, "char_ranges": [[707, 881]], "word_ranges": [[123, 151]], "text": "El antecedente de ictus (se supone que isqu\u00e9mico) hace 4 meses es una contraindicaci\u00f3n relativa de fibrinoisis (si el tiempo es menor de 3 meses es contraindicaci\u00f3n absoluta),"}, "2": {"exist": true, "char_ranges": [[546, 706]], "word_ranges": [[94, 123]], "text": "La opci\u00f3n 2 no tiene mucho sentido, puesto que en el caso de que estuviera indicada la fibrinolisis no ser\u00eda mas que una p\u00e9rdida de tiempo realizar el traslado."}, "3": {"exist": true, "char_ranges": [[888, 1023]], "word_ranges": [[152, 177]], "text": "lo definitivo que inclina la balanca hacia la ICP es la extensi\u00f3n del IAM y la presencia de shock, que son indicaciones de ICP urgente."}, "4": {"exist": true, "char_ranges": [[252, 389]], "word_ranges": [[43, 66]], "text": "Es evidente por el cuadro cl\u00ednico del paciente que est\u00e1 indicada una estrategia de reperfusi\u00f3n urgente, esto invalida las opciones 4 y 5."}, "5": {"exist": true, "char_ranges": [[252, 389]], "word_ranges": [[43, 66]], "text": "Es evidente por el cuadro cl\u00ednico del paciente que est\u00e1 indicada una estrategia de reperfusi\u00f3n urgente, esto invalida las opciones 4 y 5."}}} {"id": 505, "year": 2020, "question_id_specific": 85, "full_question": "Un paciente de 79 a\u00f1os es tra\u00eddo a urgencias a las 7 de la tarde desde una residencia, pero no disponemos del informe de derivaci\u00f3n y por tanto desconocemos sus antecedentes. Est\u00e1 desorientado en tiempo y espacio. Tiene problemas para recordar informaci\u00f3n sencilla. En ocasiones se pone muy nervioso y agitado, porque ve personas amenazantes, incluso cuando se encuentra a solas. En otras ocasiones aparece somnoliento y desatento. El principal dato que apoya el diagn\u00f3stico de delirium frente al de demencia es:", "full_answer": "Poco que comentar\u2026. es la definici\u00f3n de delirium. Alteraci\u00f3n del estado de conciencia.", "type": "PSIQUIATR\u00cdA", "options": {"1": "La presencia de alucinaciones visuales.", "2": "Los s\u00edntomas cognitivos.", "3": "La agitaci\u00f3n psicomotriz.", "4": "La fluctuaci\u00f3n del nivel de alerta.", "5": NaN}, "correct_option": 4, "explanations": {"1": {"exist": false, "char_ranges": [], "word_ranges": [], "text": ""}, "2": {"exist": false, "char_ranges": [], "word_ranges": [], "text": ""}, "3": {"exist": false, "char_ranges": [], "word_ranges": [], "text": ""}, "4": {"exist": true, "char_ranges": [[20, 86]], "word_ranges": [[3, 13]], "text": "es la definici\u00f3n de delirium. Alteraci\u00f3n del estado de conciencia."}, "5": {"exist": false, "char_ranges": [], "word_ranges": [], "text": ""}}} {"id": 360, "year": 2016, "question_id_specific": 93, "full_question": "Mujer de 46 a\u00f1os con antecedentes de valvulopat\u00eda reum\u00e1tica, que precisa sustituci\u00f3n valvular mitral por una pr\u00f3tesis mec\u00e1nica. La evoluci\u00f3n postoperatoria inicial es favorable. Sin embargo, tras comenzar tratamiento con acenocumarol desarrolla un cuadro de necrosis cut\u00e1nea afectando regi\u00f3n abdominal y extremidades. \u00bfCu\u00e1l de las siguientes alteraciones justificar\u00eda este cuadro?", "full_answer": "La necrosis cut\u00e1nea inducida por Warfarina es un efecto adverso grave y poco com\u00fan en la terapia con anticoagulantes orales, solo se da en un 0,01 a 0,1% de pacientes y est\u00e1 especialmente relacionado con la deficiencia de prote\u00edna C y S de la coagulaci\u00f3n.", "type": "HEMATOLOG\u00cdA", "options": {"1": "Deficiencia de antitrombina.", "2": "Factor V Leiden.", "3": "Hiperhomocisteinemia.", "4": "Deficiencia de proteina C.", "5": NaN}, "correct_option": 4, "explanations": {"1": {"exist": false, "char_ranges": [], "word_ranges": [], "text": ""}, "2": {"exist": false, "char_ranges": [], "word_ranges": [], "text": ""}, "3": {"exist": false, "char_ranges": [], "word_ranges": [], "text": ""}, "4": {"exist": true, "char_ranges": [[0, 255]], "word_ranges": [[0, 45]], "text": "La necrosis cut\u00e1nea inducida por Warfarina es un efecto adverso grave y poco com\u00fan en la terapia con anticoagulantes orales, solo se da en un 0,01 a 0,1% de pacientes y est\u00e1 especialmente relacionado con la deficiencia de prote\u00edna C y S de la coagulaci\u00f3n."}, "5": {"exist": false, "char_ranges": [], "word_ranges": [], "text": ""}}} {"id": 459, "year": 2018, "question_id_specific": 173, "full_question": "Hombre de 36 a\u00f1os de edad sin antecedentes de inter\u00e9s que tras un traumatismo de alta energ\u00eda al caer de una motocicleta presenta una fractura diafisaria media de tibia, abierta, grado II de Gustilo. \u00bfCu\u00e1l de las siguientes maniobras o terapias es m\u00e1s efectiva para la prevenci\u00f3n de la infecci\u00f3n?", "full_answer": "Creo que la m\u00e1s correcta es la 2, ya que en las fracturas Gustilo II se cubren sobre todo Gram (+) mediante una Cefa de 1\u00ba generaci\u00f3n, y tampoco se trata de prevenir sino de tratar (se da por contaminado).", "type": "TRAUMATOLOG\u00cdA Y ORTOPED\u00cdA", "options": {"1": "Antibioterapia inmediata intravenosa de amplio espectro.", "2": "Desbridamiento-lavado exhaustivo de la herida en quir\u00f3fano con al menos 10 litros de suero fisiol\u00f3gico.", "3": "Estabilizaci\u00f3n urgente de la fractura mediante fijaci\u00f3n externa y cierre precoz de la herida.", "4": "Cierre precoz de la herida con antis\u00e9pticos y oxigenoterapia a dos litros por minuto.", "5": NaN}, "correct_option": 2, "explanations": {"1": {"exist": false, "char_ranges": [], "word_ranges": [], "text": ""}, "2": {"exist": true, "char_ranges": [[0, 133]], "word_ranges": [[0, 27]], "text": "Creo que la m\u00e1s correcta es la 2, ya que en las fracturas Gustilo II se cubren sobre todo Gram (+) mediante una Cefa de 1\u00ba generaci\u00f3n,"}, "3": {"exist": false, "char_ranges": [], "word_ranges": [], "text": ""}, "4": {"exist": false, "char_ranges": [], "word_ranges": [], "text": ""}, "5": {"exist": false, "char_ranges": [], "word_ranges": [], "text": ""}}} {"id": 149, "year": 2012, "question_id_specific": 162, "full_question": "Mujer de 52 a\u00f1os que acude a urgencias con una par\u00e1lisis bilateral de las extremidades superiores que se hab\u00eda instaurado de manera brusca dos horas antes. En la entrevista se muestra risue\u00f1a y verbaliza no entender por qu\u00e9 sus familiares est\u00e1n tan preocupados. La familia refiere que el d\u00eda anterior su hijo hab\u00eda sido detenido por la polic\u00eda por tr\u00e1fico de drogas y todav\u00eda no hab\u00eda podido contactar con \u00e9l. \u00bfCu\u00e1l de los siguientes es el diagn\u00f3stico m\u00e1s probable?", "full_answer": "La palabra bonita para describir esto es: \u00abBelle indeference\u00bb, esta mujer se muestra con una elegante indiferencia hacia una par\u00e1lisis. El diagn\u00f3stico\u2026. se pueden excluir 2,3,4 porque la cl\u00ednica no coincide: no hay delirio, no hay preocupaci\u00f3n, no hay tristeza. La 1 ser\u00eda si sacase algo de provecho, si estuviese simulando a sabiendas y para sacar un beneficio finigiese su enfermedad. No es el caso. Conversivo ser\u00eda si el s\u00edntoma nos quisiese transmitir algo, es decir, fuese una met\u00e1fora de su mundo interno\u2026 Lo m\u00e1s exacto en este caso: El 5.", "type": "PSIQUIATR\u00cdA", "options": {"1": "Trastorno facticio.", "2": "Trastorno delirante.", "3": "Hipocondriasis.", "4": "Trastorno depresivo mayor.", "5": "Trastorno conversivo."}, "correct_option": 5, "explanations": {"1": {"exist": true, "char_ranges": [[262, 401]], "word_ranges": [[40, 65]], "text": "La 1 ser\u00eda si sacase algo de provecho, si estuviese simulando a sabiendas y para sacar un beneficio finigiese su enfermedad. No es el caso."}, "2": {"exist": true, "char_ranges": [[153, 261]], "word_ranges": [[22, 40]], "text": "se pueden excluir 2,3,4 porque la cl\u00ednica no coincide: no hay delirio, no hay preocupaci\u00f3n, no hay tristeza."}, "3": {"exist": true, "char_ranges": [[153, 261]], "word_ranges": [[22, 40]], "text": "se pueden excluir 2,3,4 porque la cl\u00ednica no coincide: no hay delirio, no hay preocupaci\u00f3n, no hay tristeza."}, "4": {"exist": true, "char_ranges": [[153, 261]], "word_ranges": [[22, 40]], "text": "se pueden excluir 2,3,4 porque la cl\u00ednica no coincide: no hay delirio, no hay preocupaci\u00f3n, no hay tristeza."}, "5": {"exist": true, "char_ranges": [[402, 511]], "word_ranges": [[65, 83]], "text": "Conversivo ser\u00eda si el s\u00edntoma nos quisiese transmitir algo, es decir, fuese una met\u00e1fora de su mundo interno\u2026"}}} {"id": 246, "year": 2014, "question_id_specific": 116, "full_question": "Un paciente con una historia de consumo excesivo de alcohol ha sido diagnosticado de tuberculosis pulmonar por un cuadro de tos, fiebre, expectoraci\u00f3n, aisl\u00e1ndose en el cultivo de esputo Mycobacterium tuberculosis. El paciente ha iniciado tratamiento con isoniacida, rifampicina, etambutol y pirazinamida, con adecuada tolerancia. A los 20 d\u00edas del inicio del tratamiento se recibe un informe de resistencia a rifampicina del M. tuberculosis aislado en el esputo. \u00bfQu\u00e9 r\u00e9gimen seleccionar\u00eda en funci\u00f3n de este informe?", "full_answer": "Otra pregunta que puede generar discusi\u00f3n. Cuando se realiza un diagn\u00f3stico de infecci\u00f3n tuberculosa activa, se inicia tratamiento con isoniazida, rifampicina y pirazinamida junto con etambutol en caso de sospecha de cepas resistentes a alguno de los f\u00e1rmacos. Una vez se descarta la existencia de resistencias, se puede prescindir del etambutol. En caso de resistencia a rifampicina, se retira la rifampicina complentando el tratamiento con los 3 f\u00e1rmacos restantes durante 12 meses. La opci\u00f3n de agregar una quinolona durante los dos primeros meses de tratamiento (moxifloxacino o levofloxacino) se contempla en los casos de afectaci\u00f3n pulmonar extensa. En este caso, no nos dicen nada de afectaci\u00f3n pulmonar extensa pero nos dan un dato \u201cconsumo excesivo de alcohol\u201d. Ya sab\u00e9is que en el MIR ning\u00fan dato es gratuito, y si la persona que puso esta pregunta lo quiso reflejar, probablemente su intenci\u00f3n era que reflexion\u00e1semos acerca de ello. El alcohol ejerce un efecto inmunosupresor directo, adem\u00e1s, el alcoholismo est\u00e1 relacionado con afectaci\u00f3n de la inmunidad en las v\u00edas respiratorias. El contexto social que normalmente acompa\u00f1a al paciente alcoh\u00f3lico tampoco ayuda. Todos los factores citados nos hacen decantarnos m\u00e1s hacia la respuesta n\u00famero 3 aunque es innegable que la numero 1 tambi\u00e9n podr\u00eda ser correcta.", "type": "ENFERMEDADES INFECCIOSAS", "options": {"1": "Isoniazida, etambutol, pirazinamida durante 12 meses.", "2": "Isoniazida, etambutol, pirazinamida durante 12 meses y estreptomicina por dos meses.", "3": "Isoniazida, etambutol, pirazinamida durante 12 meses y una quinolona por dos meses.", "4": "Isoniazida, etambutol, pirazinamida y una quinolona durante 18 meses.", "5": "Isoniazida, etambutol, pirazinamida durante 18 meses y estreptomicina y una quinolona durante 2 meses."}, "correct_option": 3, "explanations": {"1": {"exist": false, "char_ranges": [], "word_ranges": [], "text": ""}, "2": {"exist": false, "char_ranges": [], "word_ranges": [], "text": ""}, "3": {"exist": true, "char_ranges": [[945, 1094]], "word_ranges": [[147, 168]], "text": "El alcohol ejerce un efecto inmunosupresor directo, adem\u00e1s, el alcoholismo est\u00e1 relacionado con afectaci\u00f3n de la inmunidad en las v\u00edas respiratorias."}, "4": {"exist": false, "char_ranges": [], "word_ranges": [], "text": ""}, "5": {"exist": false, "char_ranges": [], "word_ranges": [], "text": ""}}} {"id": 457, "year": 2018, "question_id_specific": 126, "full_question": "Hombre de 47 a\u00f1os, con antecedente de un adenoma pleomorfo parot\u00eddeo derecho, tratado con cirug\u00eda (parotidectom\u00eda extrafacial) hace 6 meses, que acude a nuestra consulta por presentar durante la masticaci\u00f3n dolor con sudoraci\u00f3n y enrojecimiento de la piel de la regi\u00f3n preauricular. \u00bfQu\u00e9 tratamiento ser\u00eda de elecci\u00f3n?", "full_answer": "Este cuadro cl\u00ednico se llama s\u00edndrome de Frey y consiste en una reinervaci\u00f3n an\u00f3mala de estructuras regionales tras una parotidectom\u00eda por lesi\u00f3n del nervio aur\u00edculotemporal, de modo que durante la masticaci\u00f3n, se produce eritema y sudoraci\u00f3n preauricular. El tratamiento es la inyecci\u00f3n de toxina botul\u00ednica intrad\u00e9rmica.", "type": "OTORRINOLARINGOLOG\u00cdA Y CIRUG\u00cdA MAXILOFACIAL", "options": {"1": "Parotidectom\u00eda total ampliada ante la sospecha de recidiva tumoral.", "2": "Pregabalina.", "3": "Inyecci\u00f3n de toxina botul\u00ednica intrad\u00e9rmica.", "4": "Antibioterapia de amplio espectro.", "5": NaN}, "correct_option": 3, "explanations": {"1": {"exist": false, "char_ranges": [], "word_ranges": [], "text": ""}, "2": {"exist": false, "char_ranges": [], "word_ranges": [], "text": ""}, "3": {"exist": true, "char_ranges": [[0, 322]], "word_ranges": [[0, 46]], "text": "Este cuadro cl\u00ednico se llama s\u00edndrome de Frey y consiste en una reinervaci\u00f3n an\u00f3mala de estructuras regionales tras una parotidectom\u00eda por lesi\u00f3n del nervio aur\u00edculotemporal, de modo que durante la masticaci\u00f3n, se produce eritema y sudoraci\u00f3n preauricular. El tratamiento es la inyecci\u00f3n de toxina botul\u00ednica intrad\u00e9rmica."}, "4": {"exist": false, "char_ranges": [], "word_ranges": [], "text": ""}, "5": {"exist": false, "char_ranges": [], "word_ranges": [], "text": ""}}} {"id": 165, "year": 2013, "question_id_specific": 229, "full_question": "Mujer de 58 a\u00f1os, peso 130 Kg, talla 155 cm, \u00edndice de masa corporal > 30 con hipertensi\u00f3n arterial leve, glucemia 108 mg/dL y ausencia de edemas en miembros inferiores. En anal\u00edtica de sangre presenta Cr 2.0 mg/dl, Urea 86 mg/dl, Alb 3.8 g/1, Na 142 mEq/L, K 4 mEq/L. En anal\u00edtica de orina: sedimento sin alteraciones y en orina de 24 h proteinuria de 6.3 gr/ 24 h. \u00bfCual de las siguientes entidades presentar\u00e1 con mayor probabilidad?", "full_answer": "La respuesta correcta es: 2. Glomerulonefritis focal y segmentaria. Estamos ante una paciente con insuficiencia renal cuya manifestaci\u00f3n es una proteinuria. Eso nos coloca en el cuadro de GN no proliferativas (membranosa, focal y segmentaria y cambios m\u00ednimos, descartando las otras dos). Si adem\u00e1s tenemos en cuenta que nos est\u00e1 diciendo que la paciente era obesa, ligeramente hipertensa, lo m\u00e1s probable es que se trate de una Glomerulonefritis focal y segmentaria.", "type": "NEFROLOG\u00cdA", "options": {"1": "Glomerulonefritis membranosa secundaria.", "2": "Glomerulonefritis focal y segmentaria.", "3": "Nefropat\u00eda IgA.", "4": "Glomerulonefritis r\u00e1pidamente progresiva.", "5": "Nefropat\u00eda de cambios m\u00ednimos."}, "correct_option": 2, "explanations": {"1": {"exist": false, "char_ranges": [], "word_ranges": [], "text": ""}, "2": {"exist": true, "char_ranges": [[68, 288]], "word_ranges": [[9, 42]], "text": "Estamos ante una paciente con insuficiencia renal cuya manifestaci\u00f3n es una proteinuria. Eso nos coloca en el cuadro de GN no proliferativas (membranosa, focal y segmentaria y cambios m\u00ednimos, descartando las otras dos)."}, "3": {"exist": false, "char_ranges": [], "word_ranges": [], "text": ""}, "4": {"exist": false, "char_ranges": [], "word_ranges": [], "text": ""}, "5": {"exist": false, "char_ranges": [], "word_ranges": [], "text": ""}}} {"id": 157, "year": 2012, "question_id_specific": 73, "full_question": "Paciente de 75 a\u00f1os de edad, con importante gonartrosis con \u00abGenu Varo\u00bb que le condiciona marcada impotencia funcional, limitaci\u00f3n de la movilidad y dolor cont\u00ednuo que le exige estar con medicaci\u00f3n antirreum\u00e1tica, antiinflamatoria y analg\u00e9sicos de forma continuada. \u00bfQu\u00e9 tratamiento debe indicarse en este caso?", "full_answer": "Estamos ante un paciente que ha agotado la v\u00eda farmacol\u00f3gica dado que toma de todo y tiene dos problemas importantes: el genu varo y una importante gonartrosis. Esto significa que no podemos hacer nada con una artroscopia ni mucho menos con una sinovectom\u00eda (1 y 5 falsas). El genu varo podr\u00eda falsamente hacernos pensar en una osteotom\u00eda, no obstante aunque la hici\u00e9ramos y corrigi\u00e9ramos el varo ahora el da\u00f1o ya est\u00e1 hecho ya la artrosis es severa. Quiz\u00e1s hace a\u00f1os antes de que tuviera lugar la artrosis hubiera sido resolutivo pero no ahora (4 falsa). Adem\u00e1s, para plantearnos una osteotom\u00eda deber\u00edan darnos m\u00e1s datos para saber sobre qu\u00e9 hueso aplicar la osteotom\u00eda si sobre tibia o sobre f\u00e9mur. Por \u00faltimo quiz\u00e1s pensemos que lo mejor es intentar primero la rehabilitaci\u00f3n antes de pensar en la cirug\u00eda pero desecho esta opci\u00f3n porque la pregunta hace hincapi\u00e9 en que hemos agotado las medidas farmacol\u00f3gicas (aunque podr\u00eda considerarse porque no nos han especificado si se ha intentado o no). En cualquier caso con la pr\u00f3tesis de rodilla corregimos el problema del dolor de la artrosis y podemos corregir tambi\u00e9n el varo de rodilla.", "type": "TRAUMATOLOG\u00cdA Y ORTOPEDIA", "options": {"1": "Lavado artrosc\u00f3pico de la rodilla.", "2": "Rehabilitaci\u00f3n de la rodilla afectada.", "3": "Artroplastia de la rodilla afectada.", "4": "Osteotom\u00eda tibial supratuberositaria de abducci\u00f3n.", "5": "Sinovectom\u00eda de rodilla."}, "correct_option": 3, "explanations": {"1": {"exist": true, "char_ranges": [[0, 273]], "word_ranges": [[0, 47]], "text": "Estamos ante un paciente que ha agotado la v\u00eda farmacol\u00f3gica dado que toma de todo y tiene dos problemas importantes: el genu varo y una importante gonartrosis. Esto significa que no podemos hacer nada con una artroscopia ni mucho menos con una sinovectom\u00eda (1 y 5 falsas)."}, "2": {"exist": true, "char_ranges": [[712, 999]], "word_ranges": [[120, 166]], "text": "quiz\u00e1s pensemos que lo mejor es intentar primero la rehabilitaci\u00f3n antes de pensar en la cirug\u00eda pero desecho esta opci\u00f3n porque la pregunta hace hincapi\u00e9 en que hemos agotado las medidas farmacol\u00f3gicas (aunque podr\u00eda considerarse porque no nos han especificado si se ha intentado o no)."}, "3": {"exist": true, "char_ranges": [[1018, 1139]], "word_ranges": [[169, 190]], "text": "con la pr\u00f3tesis de rodilla corregimos el problema del dolor de la artrosis y podemos corregir tambi\u00e9n el varo de rodilla."}, "4": {"exist": true, "char_ranges": [[274, 555]], "word_ranges": [[47, 95]], "text": "El genu varo podr\u00eda falsamente hacernos pensar en una osteotom\u00eda, no obstante aunque la hici\u00e9ramos y corrigi\u00e9ramos el varo ahora el da\u00f1o ya est\u00e1 hecho ya la artrosis es severa. Quiz\u00e1s hace a\u00f1os antes de que tuviera lugar la artrosis hubiera sido resolutivo pero no ahora (4 falsa)."}, "5": {"exist": true, "char_ranges": [[0, 273]], "word_ranges": [[0, 47]], "text": "Estamos ante un paciente que ha agotado la v\u00eda farmacol\u00f3gica dado que toma de todo y tiene dos problemas importantes: el genu varo y una importante gonartrosis. Esto significa que no podemos hacer nada con una artroscopia ni mucho menos con una sinovectom\u00eda (1 y 5 falsas)."}}} {"id": 382, "year": 2016, "question_id_specific": 231, "full_question": "Hombre de 33 a\u00f1os aut\u00f3nomo para todas las actividades de la vida diaria y con antecedentes de HTA bien controlada, fibrilaci\u00f3n auricular antiagregada y un adenocarcinoma de pr\u00f3stata a los 73 a\u00f1os actualmente libre de enfermedad. Es tra\u00eddo a urgencias por cl\u00ednica de afasia y hemiparesia derecha de inicio brusco, 45 minutos antes. \u00bfCu\u00e1l es la actitud m\u00e1s correcta?", "full_answer": "El paciente sufre un d\u00e9ficit neurol\u00f3gico brusco en probable relaci\u00f3n con un evento vascular, por lo que en primer lugar tendremos que realizar un TC craneal para descartar que se trate de un ictus hemorr\u00e1gico. Si el TC no muestra sangrado, el paciente presenta un ictus isqu\u00e9mico agudo, probablemente cardioemb\u00f3lico por su patolog\u00eda cardiol\u00f3gica, no anticoagulada. Dado que su proceso tumoral se encuentra controlado, no existen contraindicaciones para la administraci\u00f3n de fibrinol\u00edsis en este caso, ya que el paciente presenta una buena situaci\u00f3n basal. Aunque el paciente presenta 88 a\u00f1os, la edad ya no se presenta como un criterio de exclusi\u00f3n absoluto, sino que debe tenida en cuenta la situaci\u00f3n general del paciente y en este caso especifican que presenta una buena situaci\u00f3n basal.", "type": "NEUROLOG\u00cdA", "options": {"1": "Realizar TC craneal y si no hay lesiones hemorr\u00e1gicas ni otras contraindicaciones en la anal\u00edtica, iniciar\u00eda trombolisis endovenosa de forma inmediata.", "2": "Realizar TC craneal urgente y si no hay sangrado iniciar anticoagulaci\u00f3n.", "3": "Realizar TC craneal e ingreso en centro de rehabilitaci\u00f3n, ya que su actitud no diferir\u00e1 independientemente de si la etiolog\u00eda es isqu\u00e9mica o hemorr\u00e1gica.", "4": "No hace falta TC craneal, aunque retirar\u00eda la antiagregaci\u00f3n.", "5": NaN}, "correct_option": 1, "explanations": {"1": {"exist": true, "char_ranges": [[97, 499]], "word_ranges": [[15, 75]], "text": "lo que en primer lugar tendremos que realizar un TC craneal para descartar que se trate de un ictus hemorr\u00e1gico. Si el TC no muestra sangrado, el paciente presenta un ictus isqu\u00e9mico agudo, probablemente cardioemb\u00f3lico por su patolog\u00eda cardiol\u00f3gica, no anticoagulada. Dado que su proceso tumoral se encuentra controlado, no existen contraindicaciones para la administraci\u00f3n de fibrinol\u00edsis en este caso,"}, "2": {"exist": false, "char_ranges": [], "word_ranges": [], "text": ""}, "3": {"exist": false, "char_ranges": [], "word_ranges": [], "text": ""}, "4": {"exist": false, "char_ranges": [], "word_ranges": [], "text": ""}, "5": {"exist": false, "char_ranges": [], "word_ranges": [], "text": ""}}} {"id": 496, "year": 2020, "question_id_specific": 71, "full_question": "Paciente de 40 a\u00f1os, con exploraci\u00f3n f\u00edsica normal, en el que se ha puesto en evidencia en la tomograf\u00eda computarizada una masa mediast\u00ednica y un n\u00f3dulo hep\u00e1tico, y en la anal\u00edtica una elevaci\u00f3n s\u00e9rica de l\u00e1cticodeshidrogenasa (LDH), betagonadotropina cori\u00f3nica humana (beta-GCH) y alfafetoprote\u00edna. \u00bfCu\u00e1l es su sospecha diagn\u00f3stica?:", "full_answer": "Muy t\u00edpica tambi\u00e9n. En urgencias se puede diagnosticar con un test de embarazo (en varones), porque detecta esa HCG beta en orina. El hepatocarcinoma puede elevar (solo) alfafeto, el seminoma LDH, el linfoma tambi\u00e9n LDH.", "type": "ONCOLOG\u00cdA M\u00c9DICA", "options": {"1": "Hepatocarcinoma metast\u00e1sico.", "2": "Seminoma extragonadal metast\u00e1sico.", "3": "Linfoma mediast\u00ednico.", "4": "C\u00e1ncer de c\u00e9lulas germinales no seminomatoso extragonadal metast\u00e1sico.", "5": NaN}, "correct_option": 4, "explanations": {"1": {"exist": true, "char_ranges": [[131, 220]], "word_ranges": [[22, 35]], "text": "El hepatocarcinoma puede elevar (solo) alfafeto, el seminoma LDH, el linfoma tambi\u00e9n LDH."}, "2": {"exist": true, "char_ranges": [[131, 220]], "word_ranges": [[22, 35]], "text": "El hepatocarcinoma puede elevar (solo) alfafeto, el seminoma LDH, el linfoma tambi\u00e9n LDH."}, "3": {"exist": true, "char_ranges": [[131, 220]], "word_ranges": [[22, 35]], "text": "El hepatocarcinoma puede elevar (solo) alfafeto, el seminoma LDH, el linfoma tambi\u00e9n LDH."}, "4": {"exist": true, "char_ranges": [[20, 130]], "word_ranges": [[3, 22]], "text": "En urgencias se puede diagnosticar con un test de embarazo (en varones), porque detecta esa HCG beta en orina."}, "5": {"exist": false, "char_ranges": [], "word_ranges": [], "text": ""}}} {"id": 14, "year": 2011, "question_id_specific": 213, "full_question": "Un hombre de 41 a\u00f1os acude a consulta por un n\u00f3dulo en el tiroides derecho se realiza una punci\u00f3n aspiraci\u00f3n con aguja fina. El an\u00e1lisis citol\u00f3gico nos permitir\u00e1:", "full_answer": "Creo que la respuesta correcta es la 5; la PAAF no permite hacer un estudio histol\u00f3gico si no citol\u00f3gico por lo tanto se eliminan las preguntas 1, 2. La 3 parece que se ha puesto de relleno porque no viene al caso. La 4 podr\u00eda hacer dudar si no eres pat\u00f3logo perola respuesta m\u00e1s l\u00f3gica es la 5 ya que el carcinoma papilar es el m\u00e1s frecuente.", "type": "CIRUG\u00cdA", "options": {"1": "Distingue entre adenoma folicular o carcinoma folicular m\u00ednimamente invasivo.", "2": "Identificar las zonas de invasi\u00f3n capsular o vascular del carcinoma folicular.", "3": "Reconocer la hiperplasia de c\u00e9lulas G asociadas a c\u00e1ncer medular familiar.", "4": "Distinguir entre un tumor de c\u00e9lulas de Hurthle y un adenoma folicular oxifilia.", "5": "Identificar las caracter\u00edsticas citol\u00f3gicas t\u00edpicas del carcinoma papilar."}, "correct_option": 5, "explanations": {"1": {"exist": true, "char_ranges": [[41, 149]], "word_ranges": [[8, 28]], "text": "la PAAF no permite hacer un estudio histol\u00f3gico si no citol\u00f3gico por lo tanto se eliminan las preguntas 1, 2."}, "2": {"exist": true, "char_ranges": [[40, 149]], "word_ranges": [[8, 28]], "text": "la PAAF no permite hacer un estudio histol\u00f3gico si no citol\u00f3gico por lo tanto se eliminan las preguntas 1, 2."}, "3": {"exist": true, "char_ranges": [[150, 214]], "word_ranges": [[28, 42]], "text": "La 3 parece que se ha puesto de relleno porque no viene al caso."}, "4": {"exist": true, "char_ranges": [[215, 258]], "word_ranges": [[42, 51]], "text": "La 4 podr\u00eda hacer dudar si no eres pat\u00f3logo"}, "5": {"exist": true, "char_ranges": [[266, 343]], "word_ranges": [[52, 67]], "text": "respuesta m\u00e1s l\u00f3gica es la 5 ya que el carcinoma papilar es el m\u00e1s frecuente."}}} {"id": 330, "year": 2016, "question_id_specific": 85, "full_question": "Un chico de 22 a\u00f1os de edad con hiposmia presenta falta de desarrollo de caracteres sexuales secundarios e infertilidad. Volumen testicular de 4 mL bilateral. Anal\u00edticamente, FSH 1,2 U/L (vn 5-15); LH 0,6 U/L (vn 3-15); testosterona 100 ng/diu (vn 300-1200), prolactina normal. Se\u00f1ale el tratamiento que le propondr\u00e1 para conseguir fertilidad:", "full_answer": "El s\u00edndrome de Kallman o s\u00edndrome de Morsier es una insuficiencia cong\u00e9nita que se asocia a un hipogonadismo hipogonadotr\u00f3pico por un d\u00e9ficit en la producci\u00f3n de hormona GnRH. Para restaurar la fertilidad se realizan inyecciones repetidas de GnRH.", "type": "ENDOCRINOLOG\u00cdA", "options": {"1": "Bomba de infusi\u00f3n de Gn_RH.", "2": "Administraci\u00f3n intramuscular mensual de triptorelina.", "3": "Administraci\u00f3n intramuscular de FSH y LH una vez por semana.", "4": "Tratamiento con bromocriptina.", "5": NaN}, "correct_option": 1, "explanations": {"1": {"exist": true, "char_ranges": [[0, 247]], "word_ranges": [[0, 38]], "text": "El s\u00edndrome de Kallman o s\u00edndrome de Morsier es una insuficiencia cong\u00e9nita que se asocia a un hipogonadismo hipogonadotr\u00f3pico por un d\u00e9ficit en la producci\u00f3n de hormona GnRH. Para restaurar la fertilidad se realizan inyecciones repetidas de GnRH."}, "2": {"exist": false, "char_ranges": [], "word_ranges": [], "text": ""}, "3": {"exist": false, "char_ranges": [], "word_ranges": [], "text": ""}, "4": {"exist": false, "char_ranges": [], "word_ranges": [], "text": ""}, "5": {"exist": false, "char_ranges": [], "word_ranges": [], "text": ""}}} {"id": 181, "year": 2013, "question_id_specific": 59, "full_question": "Diab\u00e9tico tipo 1 que acude a Urgencias por disnea y malestar general. En la anal\u00edtica presenta glucemia 450 mg/dl, Na 142 mEq/l, K 4 mEq/l, pH 7.15, bicarbonato 12 mmol/l. \u00bfQu\u00e9 tratamiento NO estar\u00eda indicado?", "full_answer": "Pregunta f\u00e1cil ya que el tratamiento de la cetoacidosis diab\u00e9tica es un tema muy repetido en el MIR. El bicarbonato est\u00e1 indicado con pH < 7.", "type": "ENDOCRINOLOG\u00cdA", "options": {"1": "Suero fisiol\u00f3gico iv r\u00e1pido.", "2": "Suero glucosado 5 % iv cuando la glucemia sea menor de 250 mg/dl.", "3": "Insulina r\u00e1pida humana iv.", "4": "Cloruro pot\u00e1sico 100 mEq / d\u00eda diluido en los sueros.", "5": "Bicarbonato s\u00f3dico 1M 100 cc iv en 30 minutos."}, "correct_option": 5, "explanations": {"1": {"exist": false, "char_ranges": [], "word_ranges": [], "text": ""}, "2": {"exist": false, "char_ranges": [], "word_ranges": [], "text": ""}, "3": {"exist": false, "char_ranges": [], "word_ranges": [], "text": ""}, "4": {"exist": false, "char_ranges": [], "word_ranges": [], "text": ""}, "5": {"exist": true, "char_ranges": [[101, 141]], "word_ranges": [[18, 26]], "text": "El bicarbonato est\u00e1 indicado con pH < 7."}}} {"id": 182, "year": 2013, "question_id_specific": 61, "full_question": "Una mujer de 76 a\u00f1os nos consulta porque est\u00e1 preocupada por su riesgo de sufrir un accidente cerebrovascular ya que su madre falleci\u00f3 por esta causa hace un a\u00f1o. Est\u00e1 diagnosticada de hipertensi\u00f3n arterial y diabetes mellitus tipo 2 por lo que sigue tratamiento con glipizida, aspirina, enalapril y atorvastatina. Fuma 20 cigarrillos al d\u00eda y lleva una vida sedentaria. A la exploraci\u00f3n se detecta una presi\u00f3n arterial de 150/80 mm de Hg. En la anal\u00edtica destaca una hemoglobina A1C de 8% y un LDL colesterol de 110 mg/dl. \u00bfCu\u00e1l de las siguientes acciones se asocia con una mayor reducci\u00f3n del riesgo de sufrir un accidente cerebrovascular?", "full_answer": "El factor cardiovascular m\u00e1s influyente para el ictus en las mujeres mayores de 70 a\u00f1os es la tensi\u00f3n arterial.", "type": "ENDOCRINOLOG\u00cdA", "options": {"1": "Conseguir un \u00f3ptimo control de la presi\u00f3n arterial.", "2": "Conseguir unos niveles \u00f3ptimos de hemoglobina A1C.", "3": "A\u00f1adir al tratamiento un antioxidante.", "4": "Abandonar el tabaco.", "5": "Conseguir unos niveles de LDL inferiores a 100 mg/dl."}, "correct_option": 1, "explanations": {"1": {"exist": true, "char_ranges": [[0, 111]], "word_ranges": [[0, 19]], "text": "El factor cardiovascular m\u00e1s influyente para el ictus en las mujeres mayores de 70 a\u00f1os es la tensi\u00f3n arterial."}, "2": {"exist": false, "char_ranges": [], "word_ranges": [], "text": ""}, "3": {"exist": false, "char_ranges": [], "word_ranges": [], "text": ""}, "4": {"exist": false, "char_ranges": [], "word_ranges": [], "text": ""}, "5": {"exist": false, "char_ranges": [], "word_ranges": [], "text": ""}}} {"id": 21, "year": 2011, "question_id_specific": 111, "full_question": "Un hombre de 32 a\u00f1os, ADVP y que ha salido recientemente de la c\u00e1rcel, tiene una prueba cut\u00e1nea con derivado proteico (PPD) con 10 mm de induraci\u00f3n. Hace un a\u00f1o, el PPD era negativo, actualmente est\u00e1 asintom\u00e1tico, la serolog\u00eda para VIH es negativa y su Rx T\u00f3rax es normal. \u00bfQu\u00e9 actitud entre las siguientes le parece m\u00e1s adecuada?", "full_answer": "Ante una conversi\u00f3n en el PPD en un periodo de 2 a\u00f1os, y descartando la enfermedad tuberculosa actual, est\u00e1 indicada la quimioprofilaxis con INH, aunque yo cre\u00eda que se hac\u00eda durante 6 meses y no 9. El resto no es cuestionable.", "type": "INFECCIOSAS", "options": {"1": "No realizar tto y control anual de Rx.", "2": "Cultivo de esputo para micobacterias y tratar s\u00f3lo si son positivos.", "3": "Isoniazida durante 9 meses.", "4": "Tratamiento con 4 drogas 4 meses.", "5": "Tratamiento con RIF y PRZ durante 2 meses."}, "correct_option": 3, "explanations": {"1": {"exist": false, "char_ranges": [], "word_ranges": [], "text": ""}, "2": {"exist": false, "char_ranges": [], "word_ranges": [], "text": ""}, "3": {"exist": true, "char_ranges": [[0, 144]], "word_ranges": [[0, 24]], "text": "Ante una conversi\u00f3n en el PPD en un periodo de 2 a\u00f1os, y descartando la enfermedad tuberculosa actual, est\u00e1 indicada la quimioprofilaxis con INH,"}, "4": {"exist": false, "char_ranges": [], "word_ranges": [], "text": ""}, "5": {"exist": false, "char_ranges": [], "word_ranges": [], "text": ""}}} {"id": 60, "year": 2011, "question_id_specific": 59, "full_question": "Un var\u00f3n de 40 a\u00f1os, turista extranjero, es remitido a un Servicio de Urgencias por bajo nivel de conciencia, temperatura de 37.7\u00baC y disnea. No tiene traumatismos. Se realizan varias pruebas. La m\u00e1s inmediata es una EAB sin ox\u00edgeno que muestra PH 7.33, PCO2 50, PO2 65, HCO3 27. Diagn\u00f3stico m\u00e1s probable:", "full_answer": "Realmente, pueden ser todas, pero la causa m\u00e1s frecuente de una acidosis aguda respiratoria no compensada en una persona joven es la bronconeumon\u00eda.", "type": "ANESTESIOLOG\u00cdA Y CUIDADOS CR\u00cdTICOS", "options": {"1": "TEP.", "2": "Neumon\u00eda lobar.", "3": "EAP.", "4": "Hipoventilaci\u00f3n alveolar.", "5": "Bronconeumon\u00eda."}, "correct_option": 5, "explanations": {"1": {"exist": false, "char_ranges": [], "word_ranges": [], "text": ""}, "2": {"exist": false, "char_ranges": [], "word_ranges": [], "text": ""}, "3": {"exist": false, "char_ranges": [], "word_ranges": [], "text": ""}, "4": {"exist": false, "char_ranges": [], "word_ranges": [], "text": ""}, "5": {"exist": true, "char_ranges": [[34, 148]], "word_ranges": [[5, 23]], "text": "la causa m\u00e1s frecuente de una acidosis aguda respiratoria no compensada en una persona joven es la bronconeumon\u00eda."}}} {"id": 455, "year": 2018, "question_id_specific": 155, "full_question": "En un hombre de 70 a\u00f1os, hipertenso, con una hemiplejia derecha de instauraci\u00f3n brusca con una leve disartria, sin alteraciones en la evocaci\u00f3n ni en la comprensi\u00f3n del lenguaje, \u00bfqu\u00e9 tipo de ictus cree que es mas probable que haya sufrido?", "full_answer": "La respuesta m\u00e1s cercana es la 1. Si somos estrictos estar\u00edamos hablando de un infarto lacunar motor puro (con una disartria leve). Ya que las otras 3 opciones no nos dan una cl\u00ednica como la del enunciado. Tanto la opci\u00f3n 2 como la 3 nos dan una cl\u00ednica mucho m\u00e1s florida y m\u00e1s extensa.", "type": "NEUROLOG\u00cdA", "options": {"1": "Un infarto lacunar capsular izquierdo.", "2": "Un infarto silviano cortico \u2013 subcortical fronto-temporal izquierdo.", "3": "Un infarto de tronco cerebral.", "4": "Una hemorragia lenticular izquierda.", "5": NaN}, "correct_option": 1, "explanations": {"1": {"exist": true, "char_ranges": [[0, 205]], "word_ranges": [[0, 37]], "text": "La respuesta m\u00e1s cercana es la 1. Si somos estrictos estar\u00edamos hablando de un infarto lacunar motor puro (con una disartria leve). Ya que las otras 3 opciones no nos dan una cl\u00ednica como la del enunciado."}, "2": {"exist": true, "char_ranges": [[206, 286]], "word_ranges": [[37, 54]], "text": "Tanto la opci\u00f3n 2 como la 3 nos dan una cl\u00ednica mucho m\u00e1s florida y m\u00e1s extensa."}, "3": {"exist": true, "char_ranges": [[206, 286]], "word_ranges": [[37, 54]], "text": "Tanto la opci\u00f3n 2 como la 3 nos dan una cl\u00ednica mucho m\u00e1s florida y m\u00e1s extensa."}, "4": {"exist": false, "char_ranges": [], "word_ranges": [], "text": ""}, "5": {"exist": false, "char_ranges": [], "word_ranges": [], "text": ""}}} {"id": 1, "year": 2011, "question_id_specific": 35, "full_question": "Hombre de 43 a\u00f1os sin enfermedades de inter\u00e9s que acude a Urgencias por 4 deposiciones mel\u00e9nicas en las \u00faltimas 12 horas. Refiere toma de AINEs en d\u00edas previos. En la exploraci\u00f3n f\u00edsica, estaca palidez de piel y mucosas. TA 95/65 y FC 110 lpm. El abdomen no es doloroso y en el tacto rectal se demuestran deposiciones mel\u00e9nicas. En la anal\u00edtica se encuentra una cifra de Hb de 8,1 gr/dl. Tras infusi\u00f3n de salino y comenzar la transfusi\u00f3n de concentrado de hemat\u00edes la TA es 120/85 y la FC 90 lpm. Se realiza endoscopia en la que se encuentran co\u00e1gulos en el est\u00f3mago y una \u00falcera de 2 cm en el antro g\u00e1strico con fondo de fibrina y una peque\u00f1a protuberancia blanco gris\u00e1cea de 3 mm en el centro de la \u00falcera (trombo plaquetario vaso visible). \u00bfqu\u00e9 actitud le parece la m\u00e1s correcta?", "full_answer": "Pregunta sobre HDA no varicosa. Paciente joven, sin antecedentes que tras toma de AINEs presenta sangrado con inestabilidad hemodin\u00e1mica que precisa transfusi\u00f3n. La 1 respuesta no est\u00e1 acorde con lo dicho, es un paciente inestable que ha sangrado bastante, que presenta una \u00falcera Forrest IIa que presenta un riesgo de resangrado alto. La 2 habla de somatostatina, que se utiliza para el sangrado por hipertensi\u00f3n portal. La 4 s\u00f3lo se plantear\u00e1 si no se controla el sangrado endosc\u00f3picamente . Entre la 3 y la 5, al tener un alto riesgo de sangrado se recomienda realizar terap\u00e9utica endosc\u00f3pica a poder ser con 2 t\u00e9cnicas por lo que la correcta ser\u00eda la 5.", "type": "DIGESTIVO", "options": {"1": "Dado que el paciente es joven, sin enfermedades asociadas y la \u00falcera ha dejado de sangrar, puede ser dado de alta precozmente con tratamiento con un IBP y erradicaci\u00f3n de H. Pylori.", "2": "Retirar el endoscopio, colocar sonda nasog\u00e1strica para aspiraci\u00f3n continua e iniciar tratamiento con un IBP y somatostatina.", "3": "Tomar biopsia para la detecci\u00f3n de H. Pylori, retirar el endoscopio procurando no desprender el trombo plaquetario y continuar tratamiento farmacol\u00f3gico con IBP i.v.", "4": "Consulta al servicio de Cirug\u00eda para considerar tratamiento quir\u00fargico urgente.", "5": "Tratamiento endosc\u00f3pico mediante inyecci\u00f3n de adrenalina en la \u00falcera y colocaci\u00f3n de clips (grapas met\u00e1licas) seguido de la administraci\u00f3n i.v. de IBP."}, "correct_option": 5, "explanations": {"1": {"exist": true, "char_ranges": [[32, 336]], "word_ranges": [[5, 52]], "text": "Paciente joven, sin antecedentes que tras toma de AINEs presenta sangrado con inestabilidad hemodin\u00e1mica que precisa transfusi\u00f3n. La 1 respuesta no est\u00e1 acorde con lo dicho, es un paciente inestable que ha sangrado bastante, que presenta una \u00falcera Forrest IIa que presenta un riesgo de resangrado alto."}, "2": {"exist": true, "char_ranges": [[337, 422]], "word_ranges": [[52, 66]], "text": "La 2 habla de somatostatina, que se utiliza para el sangrado por hipertensi\u00f3n portal."}, "3": {"exist": true, "char_ranges": [[495, 658]], "word_ranges": [[79, 111]], "text": "Entre la 3 y la 5, al tener un alto riesgo de sangrado se recomienda realizar terap\u00e9utica endosc\u00f3pica a poder ser con 2 t\u00e9cnicas por lo que la correcta ser\u00eda la 5."}, "4": {"exist": true, "char_ranges": [[423, 494]], "word_ranges": [[66, 79]], "text": "La 4 s\u00f3lo se plantear\u00e1 si no se controla el sangrado endosc\u00f3picamente ."}, "5": {"exist": true, "char_ranges": [[495, 658]], "word_ranges": [[79, 111]], "text": "Entre la 3 y la 5, al tener un alto riesgo de sangrado se recomienda realizar terap\u00e9utica endosc\u00f3pica a poder ser con 2 t\u00e9cnicas por lo que la correcta ser\u00eda la 5."}}} {"id": 66, "year": 2012, "question_id_specific": 106, "full_question": "Anciana que llevan sus vecinos a Urgencias porque la ven algo atontada y con manchas de deposici\u00f3n enla ropa. TA 100/60 mmHg, FC 100 lpm, sentada 70/30, FC 105 lpm. PVY normal. Respiraci\u00f3n de Kussmaul. No focalidad neurol\u00f3gica. Peso de 50 kgrs. Lab: PH 7.25, PCO2 14, Bicarbonato 5, Na 133, K 2.5, Cl 118, Creat 3.4, NUS 60, Prot 8. \u00bfCu\u00e1l de las siguientes respuestas es correcta?", "full_answer": "Estamos ante un caso de acidosis metab\u00f3lica hiperclor\u00e9mica con ani\u00f3n gap normal. La acidosis metab\u00f3lica grave causa depresi\u00f3n contr\u00e1ctil del miocardio, reducci\u00f3n de la efectividad de las catecolaminas, mayor incidencia de arritmias, vasodilataci\u00f3n y aumento de la permeabilidad capilar, con la consiguiente reducci\u00f3n de flujo tisular, a nivel cerebral (que lleva a estados de confusi\u00f3n e , incluso, coma), a nivel renal, a nivel hep\u00e1tico\u2026 Es frecuente que estos pacientes presenten una taquipnea intensa (respiraci\u00f3n de Kussmaul) en un intento de compensar la ACM mediante hiperventilaci\u00f3n. Tenemos que tener en cuenta que, en este caso, el potasio est\u00e1 disminuido, por lo que la causa m\u00e1s frecuente (apoyada por las manchas del vestido) ser\u00eda la diarrea, con lo que lo m\u00e1s probable es que su insuficiencia renal (a pesar de que no nos dan datos de iones en orina para poder confirmarlo)sea de tipo prerrenal. El tratamiento ser\u00eda la reposici\u00f3n hidroelectrol\u00edtica y, en principio, salvo en el caso de que la paciente empeorase y el PH fuera menor de 7.10, no deber\u00edamos administrar bicarbonato Con lo que la respuesta correcta es la 5.", "type": "ANESTESIOLOG\u00cdA, CUIDADOS CR\u00cdTICOS Y URGENCIAS", "options": {"1": "El trastorno \u00e1cido-base que presenta es una acidosis respiratoria.", "2": "La compensaci\u00f3n para corregir la acidosis no es la adecuada.", "3": "Con esta exploraci\u00f3n, descartamos que est\u00e9 deshidratada.", "4": "En ning\u00fan caso, debemos ponerle bicarbonato.", "5": "Tiene una insuficiencia renal aguda de causa prerrenal."}, "correct_option": 5, "explanations": {"1": {"exist": false, "char_ranges": [], "word_ranges": [], "text": ""}, "2": {"exist": false, "char_ranges": [], "word_ranges": [], "text": ""}, "3": {"exist": false, "char_ranges": [], "word_ranges": [], "text": ""}, "4": {"exist": false, "char_ranges": [], "word_ranges": [], "text": ""}, "5": {"exist": true, "char_ranges": [[591, 909]], "word_ranges": [[86, 144]], "text": "Tenemos que tener en cuenta que, en este caso, el potasio est\u00e1 disminuido, por lo que la causa m\u00e1s frecuente (apoyada por las manchas del vestido) ser\u00eda la diarrea, con lo que lo m\u00e1s probable es que su insuficiencia renal (a pesar de que no nos dan datos de iones en orina para poder confirmarlo)sea de tipo prerrenal."}}} {"id": 521, "year": 2021, "question_id_specific": 76, "full_question": "Ni\u00f1a de 7 a\u00f1os que acude a consulta porque los padres le notan la cara roja desde el d\u00eda anterior y en las \u00faltimas horas le han notado manchas rojas en el cuerpo. En la exploraci\u00f3n est\u00e1 afebril, con am\u00edgdalas y o\u00eddos sin alteraciones, auscultaci\u00f3n cardiopulmonar normal. Presenta un exantema maculo papuloso confluyente en ambas mejillas, respetando el labio superior, semejando cara de \u201cbofet\u00f3n\u201d. En el t\u00f3rax presenta un exantema reticular, en encaje, no pruriginoso. \u00bfCu\u00e1l es el diagn\u00f3stico cl\u00ednico m\u00e1s probable?:", "full_answer": "Aunque esta pregunta est\u00e1 a caballo entre derma y pediatr\u00eda, queremos aportar nuestro granito de arena. Nos est\u00e1n hablando del eritema s\u00fabito o quinta enfermedad (tambi\u00e9n llamado megaloeritema), producido por el parvovirus B19. Es t\u00edpico que despu\u00e9s de un episodio de fiebre alta aparezca un eritema en ambas mejillas en forma de \u201cbofet\u00f3n\u201d que suele tener una extensi\u00f3n cr\u00e1neo-caudal en los d\u00edas siguientes.", "type": "DERMATOLOG\u00cdA", "options": {"1": "Varicela.", "2": "Rubeola.", "3": "Eritema infeccioso o 5\u00aa enfermedad.", "4": "Exantema s\u00fabito o 6\u00aa enfermedad.", "5": NaN}, "correct_option": 3, "explanations": {"1": {"exist": false, "char_ranges": [], "word_ranges": [], "text": ""}, "2": {"exist": false, "char_ranges": [], "word_ranges": [], "text": ""}, "3": {"exist": true, "char_ranges": [[104, 407]], "word_ranges": [[16, 63]], "text": "Nos est\u00e1n hablando del eritema s\u00fabito o quinta enfermedad (tambi\u00e9n llamado megaloeritema), producido por el parvovirus B19. Es t\u00edpico que despu\u00e9s de un episodio de fiebre alta aparezca un eritema en ambas mejillas en forma de \u201cbofet\u00f3n\u201d que suele tener una extensi\u00f3n cr\u00e1neo-caudal en los d\u00edas siguientes."}, "4": {"exist": false, "char_ranges": [], "word_ranges": [], "text": ""}, "5": {"exist": false, "char_ranges": [], "word_ranges": [], "text": ""}}} {"id": 20, "year": 2011, "question_id_specific": 110, "full_question": "Paciente de 40 a\u00f1os de edad, fumador desde los 18 a\u00f1os y sin antecedentes de inter\u00e9s. Consulta por cuadro de 24 horas de evoluci\u00f3n de escalofr\u00edos, fiebre termometrada de 39\u00baC y tos con expectoraci\u00f3n. La Rx de t\u00f3rax pone de manifiesto un aumento de densidad con patr\u00f3n alveolar en el LSD y el paciente es diagnosticado de NAC. \u00bfCu\u00e1l es el agente etiol\u00f3gico m\u00e1s frecuente y por tanto de obligada cobertura al elegir el tratamiento atb emp\u00edrico?", "full_answer": "Esta es una pregunta de epidemiolog\u00eda bastante f\u00e1cil, en adultos sin otros datos, Neumococo es el 1\u00ba.", "type": "INFECCIOSAS", "options": {"1": "Mycoplasma pneumoniae.", "2": "Chlamydia pneumoniae.", "3": "Streptococcus pneumoniae.", "4": "Legionella pneumophila serogrupo 1.", "5": "Haemophilus influenzae."}, "correct_option": 3, "explanations": {"1": {"exist": false, "char_ranges": [], "word_ranges": [], "text": ""}, "2": {"exist": false, "char_ranges": [], "word_ranges": [], "text": ""}, "3": {"exist": true, "char_ranges": [[54, 101]], "word_ranges": [[8, 17]], "text": "en adultos sin otros datos, Neumococo es el 1\u00ba."}, "4": {"exist": false, "char_ranges": [], "word_ranges": [], "text": ""}, "5": {"exist": false, "char_ranges": [], "word_ranges": [], "text": ""}}} {"id": 422, "year": 2018, "question_id_specific": 88, "full_question": "\u00bfCual es el tratamiento de elecci\u00f3n de un hombre de 38 a\u00f1os diagnosticado de enfermedad de Cushing en relaci\u00f3n con un macroadenoma hipofisario de 22 mm de di\u00e1metro?", "full_answer": "A pesar de que la mayor\u00eda de los adenomas productores de ACTH recidivan, la primera opci\u00f3n terep\u00e9utica es siempre la cirug\u00eda. La medicaci\u00f3n disponible (ketoconazol,an\u00e1logos de la somatostatina) van dirigidos a disminuir los efectos del exceso de cortisol o disminuir la liberaci\u00f3n de ACTH, pero no son curativos.", "type": "ENDOCRINOLOG\u00cdA", "options": {"1": "Probar inicialmente tratamiento m\u00e9dico y, en caso de no ser efectivo, recurrir a cirug\u00eda.", "2": "Adrenalectomia.", "3": "Resecci\u00f3n quir\u00fargica selectiva del adenoma.", "4": "Radioterapia para intentar disminuir el tama\u00f1o tumoral.", "5": NaN}, "correct_option": 3, "explanations": {"1": {"exist": false, "char_ranges": [], "word_ranges": [], "text": ""}, "2": {"exist": false, "char_ranges": [], "word_ranges": [], "text": ""}, "3": {"exist": true, "char_ranges": [[0, 125]], "word_ranges": [[0, 21]], "text": "A pesar de que la mayor\u00eda de los adenomas productores de ACTH recidivan, la primera opci\u00f3n terep\u00e9utica es siempre la cirug\u00eda."}, "4": {"exist": false, "char_ranges": [], "word_ranges": [], "text": ""}, "5": {"exist": false, "char_ranges": [], "word_ranges": [], "text": ""}}} {"id": 348, "year": 2016, "question_id_specific": 160, "full_question": "Mujer de 45 a\u00f1os, madre de 3 hijos, acude a consulta de diagn\u00f3stico precoz. La citolog\u00eda cervicovaginal es compatible con lesi\u00f3n escamosa intraepitelial de alto grado. \u00bfCu\u00e1l de las siguientes opciones elegir\u00eda?", "full_answer": "Tienen indicaci\u00f3n de colposcopia todas las pacientes con citolog\u00eda alterada (lesi\u00f3n intraepitelial o sospecha de c\u00e1ncer invasor o infecci\u00f3n VPH), como segundo nivel de screening. En caso de observarse lesiones en esta prueba, se proceder\u00e1 a realizar una biopsia de la lesi\u00f3n. No tiene sentido repetir la citolog\u00eda (ya que esta es una prueba de screening, no diagn\u00f3stica), realizar una histerectom\u00eda por una lesi\u00f3n intraepitelial que no se ha confirmado por biopsia tampoco est\u00e1 indicado y la realizaci\u00f3n de legrado de la cavidad endometrial no aporta informaci\u00f3n ya que la lesi\u00f3n es del c\u00e9rvix.", "type": "GINECOLOG\u00cdA Y OBSTETRICIA", "options": {"1": "Repetir la citolog\u00eda en 1 mes.", "2": "Colposcopia con eventual biopsia.", "3": "Histerectom\u00eda con salpinguectom\u00eda bilateral y conservaci\u00f3n de ovarios.", "4": "Legrado uterino fraccionado.", "5": NaN}, "correct_option": 2, "explanations": {"1": {"exist": true, "char_ranges": [[276, 370]], "word_ranges": [[42, 58]], "text": "No tiene sentido repetir la citolog\u00eda (ya que esta es una prueba de screening, no diagn\u00f3stica),"}, "2": {"exist": true, "char_ranges": [[0, 275]], "word_ranges": [[0, 42]], "text": "Tienen indicaci\u00f3n de colposcopia todas las pacientes con citolog\u00eda alterada (lesi\u00f3n intraepitelial o sospecha de c\u00e1ncer invasor o infecci\u00f3n VPH), como segundo nivel de screening. En caso de observarse lesiones en esta prueba, se proceder\u00e1 a realizar una biopsia de la lesi\u00f3n."}, "3": {"exist": true, "char_ranges": [[372, 486]], "word_ranges": [[58, 75]], "text": "realizar una histerectom\u00eda por una lesi\u00f3n intraepitelial que no se ha confirmado por biopsia tampoco est\u00e1 indicado"}, "4": {"exist": true, "char_ranges": [[489, 594]], "word_ranges": [[76, 94]], "text": "la realizaci\u00f3n de legrado de la cavidad endometrial no aporta informaci\u00f3n ya que la lesi\u00f3n es del c\u00e9rvix."}, "5": {"exist": false, "char_ranges": [], "word_ranges": [], "text": ""}}} {"id": 411, "year": 2018, "question_id_specific": 52, "full_question": "Una pareja con un primer hijo var\u00f3n (caso \u00edndice) afecto de fibrosis qu\u00edstica (mucoviscidosis) acude a la consulta dc consejo gen\u00e9tico. El diagn\u00f3stico molecular indica que el caso \u00edndice es homocigoto para la mutaci\u00f3n F5O8del y sus padres portadores heterocigotos. El genetista cl\u00ednico les informar\u00e1 acerca de la forma de transmisi\u00f3n de la enfermedad. \u00bfQu\u00e9 afirmaci\u00f3n es la correcta?", "full_answer": "Los dos padres son portadores de una mutaci\u00f3n (porque no est\u00e1n enfermos y ya han tenido un hijo homocigoto), y el pr\u00f3ximo descendiente tiene un 50% de probabilidad de recibir la mutaci\u00f3n de cada uno de los padres, por lo que la probabilidad de recibirla de ambos es 50%x50%=25%.", "type": "BIOESTAD\u00cdSTICA", "options": {"1": "La probabilidad de que el siguiente descendiente est\u00e9 afecto es del 25%.", "2": "La probabilidad de que el siguiente descendiente est\u00e9 afecto es del 50%.", "3": "Al haber tenido un hijo afecto, el siguiente descendiente tambi\u00e9n estar\u00e1 afecto.", "4": "Al haber tenido un hijo homocigoto F5O8del, el siguiente descendiente ser\u00e1 portador heterocigoto (67%) o de genotipo homocigoto sin la mutaci\u00f3n (33%).", "5": NaN}, "correct_option": 1, "explanations": {"1": {"exist": true, "char_ranges": [[0, 279]], "word_ranges": [[0, 49]], "text": "Los dos padres son portadores de una mutaci\u00f3n (porque no est\u00e1n enfermos y ya han tenido un hijo homocigoto), y el pr\u00f3ximo descendiente tiene un 50% de probabilidad de recibir la mutaci\u00f3n de cada uno de los padres, por lo que la probabilidad de recibirla de ambos es 50%x50%=25%."}, "2": {"exist": false, "char_ranges": [], "word_ranges": [], "text": ""}, "3": {"exist": false, "char_ranges": [], "word_ranges": [], "text": ""}, "4": {"exist": false, "char_ranges": [], "word_ranges": [], "text": ""}, "5": {"exist": false, "char_ranges": [], "word_ranges": [], "text": ""}}} {"id": 590, "year": 2022, "question_id_specific": 64, "full_question": "Mujer de 68 a\u00f1os que utiliza para visi\u00f3n lejana gafas de +4,00 dioptr\u00edas en ambos ojos. Consulta porque desde hace unas semanas tiene dolor ocular con cefalea y visi\u00f3n borrosa por la noche cuando est\u00e1 viendo la televisi\u00f3n en penumbra. \u00bfCu\u00e1l de las siguientes patolog\u00edas es m\u00e1s probable que presente?", "full_answer": "Nos presentan una paciente de 68 a\u00f1os. A esa edad el cristalino, aunque normalmente no tiene una catarata que haya que operar, s\u00ed suele haber aumentado de grosor. Lo que implica que proyecta el iris hacia adelante y reduce la profundidad de la c\u00e1mara anterior. En un ojo con una c\u00e1mara anterior de profundidad normal no suele ser un problema. Pero las personas hiperm\u00e9tropes ya tienen de por s\u00ed una c\u00e1mara estrecha. Esta es la combinaci\u00f3n habitual que encontramos en el cierre angular primario: hipermetrop\u00eda y una edad mayor de 50-60 a\u00f1os. Los hiperm\u00e9tropes j\u00f3venes no suelen tener todav\u00eda este problema. Y las personas mayores em\u00e9tropes, miopes u operados de catarata tampoco. Con este \u00e1ngulo estrecho, lo que desencadena el cierre angulas es la midriasis. Es decir, cuando uno est\u00e1 en penumbra, la pupila se dilata y el iris se engruesa en la parte perif\u00e9rica, donde ya de por s\u00ed est\u00e1 muy cerca del endotelio corneal en caso de c\u00e1mara estrecha. Entonces la midriasis desencadena el cierre angular, que cursa con dolor ocular que se puede irradiar hacia atr\u00e1s (cefalea). Y el aumento de presi\u00f3n intraocular produce un edema corneal que causa visi\u00f3n borrosa. Por lo tanto, la opci\u00f3n 1 es la correcta. La migra\u00f1a no tiene que tener relaci\u00f3n con la penumbra, y el dolor ocular y la visi\u00f3n borrosa no es especialmente t\u00edpico. La queratitis act\u00ednica se produce por exposici\u00f3n a radiaci\u00f3n ultravioleta (como la solar), no tiene relaci\u00f3n ni con la hipermetrop\u00eda, ni con estar en penumbra. Adem\u00e1s el antecedente de exposici\u00f3n suele ser claro. Y por \u00faltimo el ojo seco tampoco producen esos s\u00edntomas: se trata m\u00e1s de quemaz\u00f3n, sensaci\u00f3n de cuerpo extra\u00f1o, pesadez, etc. Y tampoco tiene relaci\u00f3n con los dem\u00e1s elementos: la penumbra, la hipermetrop\u00eda, etc. Como detalle menor, yo hubiera redactado un poco mejor la pregunta. Se sobreentiende que nos preguntan qu\u00e9 patolog\u00eda es la m\u00e1s probable en relaci\u00f3n a los s\u00edntomas y la anamnesis. Pero tal como est\u00e1 redactado, nos preguntan por cu\u00e1l de las patolog\u00edas es m\u00e1s probable que presente, sin m\u00e1s. Realmente una mujer de 68 a\u00f1os es muy probable que tenga un ojo seco, al margen de que eso no le cause los s\u00edntomas por los que acude.", "type": "OFTALMOLOG\u00cdA", "options": {"1": "Glaucoma por cierre angular primario.", "2": "Migra\u00f1a.", "3": "Queratitis act\u00ednica.", "4": "S\u00edndrome de ojo seco.", "5": NaN}, "correct_option": 1, "explanations": {"1": {"exist": true, "char_ranges": [[759, 1201]], "word_ranges": [[125, 203]], "text": "Es decir, cuando uno est\u00e1 en penumbra, la pupila se dilata y el iris se engruesa en la parte perif\u00e9rica, donde ya de por s\u00ed est\u00e1 muy cerca del endotelio corneal en caso de c\u00e1mara estrecha. Entonces la midriasis desencadena el cierre angular, que cursa con dolor ocular que se puede irradiar hacia atr\u00e1s (cefalea). Y el aumento de presi\u00f3n intraocular produce un edema corneal que causa visi\u00f3n borrosa. Por lo tanto, la opci\u00f3n 1 es la correcta."}, "2": {"exist": true, "char_ranges": [[1202, 1323]], "word_ranges": [[203, 225]], "text": "La migra\u00f1a no tiene que tener relaci\u00f3n con la penumbra, y el dolor ocular y la visi\u00f3n borrosa no es especialmente t\u00edpico."}, "3": {"exist": true, "char_ranges": [[1324, 1483]], "word_ranges": [[225, 250]], "text": "La queratitis act\u00ednica se produce por exposici\u00f3n a radiaci\u00f3n ultravioleta (como la solar), no tiene relaci\u00f3n ni con la hipermetrop\u00eda, ni con estar en penumbra."}, "4": {"exist": true, "char_ranges": [[1537, 1748]], "word_ranges": [[258, 292]], "text": "Y por \u00faltimo el ojo seco tampoco producen esos s\u00edntomas: se trata m\u00e1s de quemaz\u00f3n, sensaci\u00f3n de cuerpo extra\u00f1o, pesadez, etc. Y tampoco tiene relaci\u00f3n con los dem\u00e1s elementos: la penumbra, la hipermetrop\u00eda, etc."}, "5": {"exist": false, "char_ranges": [], "word_ranges": [], "text": ""}}} {"id": 86, "year": 2012, "question_id_specific": 51, "full_question": "Un hombre de 26 a\u00f1os sin antecedentes de inter\u00e9s, asintom\u00e1tico, presenta en una revisi\u00f3n un soplo mesosist\u00f3lico en foco a\u00f3rtico y borde paraesternal izquierdo que aumenta con la maniobra de Valsalva. La TA y los pulsos perif\u00e9ricos son normales y el ECG muestra signos de marcado crecimiento ventricular izquierdo y ondas T negativas muy profundas en V3, V4, V5 y V6. \u00bfCu\u00e1l es el diagn\u00f3stico m\u00e1s probable?", "full_answer": "Un paciente var\u00f3n, joven, con un soplo sist\u00f3lico a\u00f3rtico que aumenta con la maniobra de Valsalva y signos de crecimiento ventricular izquierdo supone pr\u00e1cticamente un \u201cretrato\u201d de los hallazgos t\u00edpicos que podemos encontrar en la miocardiopat\u00eda hipertr\u00f3fica. Pregunta muy f\u00e1cil.", "type": "CARDIOLOG\u00cdA Y CIRUG\u00cdA VASCULAR", "options": {"1": "Soplo funcional.", "2": "Estenosis a\u00f3rtica.", "3": "Insuficiencia a\u00f3rtica.", "4": "Miocardiopat\u00eda hipertr\u00f3fica.", "5": "Comunicaci\u00f3n interventricular."}, "correct_option": 4, "explanations": {"1": {"exist": false, "char_ranges": [], "word_ranges": [], "text": ""}, "2": {"exist": false, "char_ranges": [], "word_ranges": [], "text": ""}, "3": {"exist": false, "char_ranges": [], "word_ranges": [], "text": ""}, "4": {"exist": true, "char_ranges": [[0, 258]], "word_ranges": [[0, 37]], "text": "Un paciente var\u00f3n, joven, con un soplo sist\u00f3lico a\u00f3rtico que aumenta con la maniobra de Valsalva y signos de crecimiento ventricular izquierdo supone pr\u00e1cticamente un \u201cretrato\u201d de los hallazgos t\u00edpicos que podemos encontrar en la miocardiopat\u00eda hipertr\u00f3fica."}, "5": {"exist": false, "char_ranges": [], "word_ranges": [], "text": ""}}} {"id": 498, "year": 2020, "question_id_specific": 129, "full_question": "Hombre de 65 a\u00f1os que consulta por disfagia a s\u00f3lidos desde hace dos meses. La esofagoscopia evidencia tumoraci\u00f3n a 30 cm de arcada dental, parcialmente estenosante, con anatom\u00eda patol\u00f3gica de carcinoma epidermoide. Se solicita ecograf\u00eda endosc\u00f3pica y PET-TC donde no se observan adenopat\u00edas patol\u00f3gicas. \u00bfCu\u00e1l de las siguientes opciones ser\u00eda la m\u00e1s correcta?:", "full_answer": "Quimiorradio preoperatoria en todas las gu\u00edas.", "type": "ONCOLOG\u00cdA M\u00c9DICA", "options": {"1": "Esofaguectom\u00eda transhiatal.", "2": "Esofaguectom\u00eda seg\u00fan la t\u00e9cnica de Ivor- Lewis.", "3": "Quimioterapia m\u00e1s radioterapia preoperatoria.", "4": "Quimioterapia neoadyuvante.", "5": NaN}, "correct_option": 3, "explanations": {"1": {"exist": false, "char_ranges": [], "word_ranges": [], "text": ""}, "2": {"exist": false, "char_ranges": [], "word_ranges": [], "text": ""}, "3": {"exist": true, "char_ranges": [[0, 46]], "word_ranges": [[0, 6]], "text": "Quimiorradio preoperatoria en todas las gu\u00edas."}, "4": {"exist": false, "char_ranges": [], "word_ranges": [], "text": ""}, "5": {"exist": false, "char_ranges": [], "word_ranges": [], "text": ""}}} {"id": 31, "year": 2011, "question_id_specific": 233, "full_question": "Un paciente diagnosticado de asma bronquial que est\u00e1 siendo tratado con esteroides inhalados a dosis bajas (200 microgramos de budesonida cada 12 horas) y medicaci\u00f3n de rescata (salbutamol a demanda) acude a control cl\u00ednico peri\u00f3dico. El paciente refiere que en los \u00faltimos 3 meses no ha tenido agudizaciones y que utiliza el salbutamol una vez al d\u00eda. Unicamente tiene disnea al subir cuestas y algunas noches dureme de forma irregular por la tos. \u00bfCu\u00e1l de las sigueinte opciones terap\u00e9uticas se consiedera de elecci\u00f3n?", "full_answer": "Algoritmo de tratamiento del asma. Descartamos seguir con el mismo tratamiento ya que el paciente est\u00e1 sintom\u00e1tico.", "type": "NEUROLOG\u00cdA Y CIRUG\u00cdA TOR\u00c1CICA", "options": {"1": "Aumentar la dosis de corticoides inhalados a 800 microgramos/d\u00eda.", "2": "A\u00f1adir tiotropio.", "3": "Mantener la misma pauta farmacol\u00f3gica.", "4": "Cambiar a una terapia ocmbinada con corticoides inhalados a dosis bajas y beta2 adren\u00e9rgicos de acci\u00f3n prolongada.", "5": "A\u00f1adir leucotrienos."}, "correct_option": 4, "explanations": {"1": {"exist": false, "char_ranges": [], "word_ranges": [], "text": ""}, "2": {"exist": false, "char_ranges": [], "word_ranges": [], "text": ""}, "3": {"exist": false, "char_ranges": [], "word_ranges": [], "text": ""}, "4": {"exist": true, "char_ranges": [[0, 115]], "word_ranges": [[0, 17]], "text": "Algoritmo de tratamiento del asma. Descartamos seguir con el mismo tratamiento ya que el paciente est\u00e1 sintom\u00e1tico."}, "5": {"exist": false, "char_ranges": [], "word_ranges": [], "text": ""}}} {"id": 618, "year": 2022, "question_id_specific": 119, "full_question": "Mujer de 13 a\u00f1os, sin antecedentes relevantes, con menarquia hace 3 meses, seguida desde los 10 a\u00f1os por escoliosis idiop\u00e1tica que ha empeorado. En la exploraci\u00f3n f\u00edsica presenta una giba de 7 grados en el test de Adams y en el escoliograma una curva toracolumbar T4-L1 de 35 grados de Cobb y un Risser 0. La actitud correcta a tomar ser\u00e1:", "full_answer": "Se puede plantear tratamiento con ortesis en Risser de 0 a 2 dado que a\u00fan queda crecimiento por darse y puede producirse la correcci\u00f3n. Su uso se plantea para curvas entre 30-40\u00ba.", "type": "TRAUMATOLOG\u00cdA", "options": {"1": "Recomendar nataci\u00f3n y revisi\u00f3n en tres meses.", "2": "Prescribir una ortesis tipo cors\u00e9.", "3": "Derivar a fisioterapia para elastificaci\u00f3n del raquis.", "4": "Revisar en 6 meses con una nueva radiograf\u00eda.", "5": NaN}, "correct_option": 2, "explanations": {"1": {"exist": false, "char_ranges": [], "word_ranges": [], "text": ""}, "2": {"exist": true, "char_ranges": [[0, 179]], "word_ranges": [[0, 32]], "text": "Se puede plantear tratamiento con ortesis en Risser de 0 a 2 dado que a\u00fan queda crecimiento por darse y puede producirse la correcci\u00f3n. Su uso se plantea para curvas entre 30-40\u00ba."}, "3": {"exist": false, "char_ranges": [], "word_ranges": [], "text": ""}, "4": {"exist": false, "char_ranges": [], "word_ranges": [], "text": ""}, "5": {"exist": false, "char_ranges": [], "word_ranges": [], "text": ""}}}